You are on page 1of 450

Textbook

Chapter No.

03
i
Motion in Plane

Hints

1 2
21. s = ut + at
Classical Thinking 2

ns
1 2
9. The straight line normal to time axis will s= at ( u = 0)
indicate multiple positions of a particle at one 2
2s 2  800
time, which is not possible.  a= 2 = = 6.25 m/s2
16 
2
t

io
10. Up to time t0, slope of the graph is constant and
after t0 slope is zero i.e., the particle travel with 22. v2 = u2 + 2as
constant speed up to time t0 and then stops. v2  u 2 (400) 2  (1000) 2
a= =

at
13. On reaching the greatest height the body has 2s 2  0.05
zero velocity and non-zero acceleration. a = 8.4  106 m/s2
(Refer Mindbender 3)
72  5
15. t1 =
x/2
v1
, t2 =
x/2
v2
lic 23. u = 72 km/hr =
v = u + at
18
= 20 m/s

x x 2 v1v 2 0 = 20 + a  10
Average speed = = = a = 2 m/s2
t1 + t 2 x/2 x/2 v1 + v2
ub
+ (negative sign indicates deceleration)
v1 v 2
1 2
 s = ut + at
v v 30  60 2
16. Average speed = 1 2 = = 45 km/hr
2 2 1
= 20  10 +  (2)  (10)2
P

2
s 80  100
17. v= = = 44.44 m s1 = 200  100
s/2 s / 2 180
+ s = 100 m
40 50
et

24. Distance travelled between 0 to 12 s


s s
18. Average speed = = = area (OAB) + area ( ABCD)
s2 s2 s s
  =
1
 2  6 + 10  6
40 60 80 120
rg

2
240s
= = 48 km h1 = 66 m
5s
26. Line BC indicates deceleration. At C, velocity is
x = a + bt2
Ta

19. zero. Hence there is a force which opposes the


dx motion.
= 2bt
dt
27. Since body returns to the starting point,
For t = 2 s h=0
dx
v= = 2bt = 2  2.5  2 = 10 m/s Also h = ut 
1 2
gt
dt 2
20. v = u + at 1
0 = u(6)   9.8  (6)2
2u = u + at 2
 u = at  6u = 9.8  18
u  u = 29.4 m/s
 t=
a 1 2 1
5  103 28. h = ut + gt = 0  4 +  (9.8)  (4)2
 t= 12
= 5  109 s 2 2
10 h = 78.4 m
1

MHT-CET Triumph Physics (Hints)

29. v2 = u2  2gh (upward motion) u 2 sin 2  100   sin 2 60


2

At maximum height, v = 0 60. H = =


2g 2  9.8
 u2 = 2gh
H  382.6 m
14  = 10 m
2
u2
 h= = u2
2g 2  9.8 61. Rmax =
g
32. Relative velocity of 1st car w.r.t 2nd car  u2 = g  Rmax = 9.8  980
v12 = v1  v2 = 58  40 = 18 km/hr  u = 98 m s1
33. The relative speed of policeman w.r.t. thief u2 1
Hmax = =  980 = 245 m
= 10  9 = 1 m/s. 4g 4
100
 Time taken by police to catch the thief = dy
1 62. vy = = 8  10t,

ns
= 100 s dt
dx
34. Relative velocity of 1st car w.r.t 2nd car vx =
dt
=6
v12 = v1  v2 = 58  40 = 18 km/hr
At t = 0, v = v 2x +v 2y = 36  64 = 10 m/s

io
36. An aeroplane in flight is propelled by
combustion of fuel and does not move under the  30  sin 2  30 = 11.48 m
2
u 2 sin 2 
effect of gravity alone. 63. H = =
2g 2  9.8

at
2usin  2  196  sin 30 u 2 sin 2  30   sin 2  30 
2
46. T= = = 20 s R = = = 79.53 m
g 9.8 g 9.8
u sin  49  sin 30
47. tA =
g
=
9.8
lic 64. When particle is projected vertically upward
 = 90
tA = 2.5 s u 2 sin 2  u 2
H= =
1 g 2g
tan  =
ub
48. tan 
2 u 2 sin 2
R= =0
(Refer Shortcut 6) g

u 2 sin 2 65. In U.C.M., direction of velocity and acceleration


51. R=
P

g change from point to point.


 200 
 sin 2  30 
2
68. At each point on circular path, the magnitude of
=
9.8 velocity remains the same for any value of .
et

(200)2  sin 60


= = 2041  3  2  103 3 m 71. n = 100 r.p.m. =
100
r.p.s.
9.8 60
2  100
u 2 sin 2  98   sin 2  45 
2
 = 2n = = 10.47 rad/s
rg

52. R = = 60
g 9.8
R = 980 m 72. n = 3.5 r.p.s.
Decrease in range = 980  900 = 80 m  = 2n = 2    3.5 = 7
Ta

= 7  3.14  22 rad/s
53. Out of given options, only option (D) is
dimensionally correct. 73. For earth, T = 24 hr = 24  3600 = 86400 s
2  2 2
u2 =  rad/hr = rad/s
54. Rmax = T 24 86400
g
 u2 = Rmax  g = 10  103  9.8 74. Using,  = 2n
 u = 98,000 m/s  125 = 2n
125
 n =
u 2 sin 2  u 2 sin 2  2
57. H =  = 6H
2g ' 2g  n  20 Hz
6
75. For a seconds hand of a watch, T = 60 s
58. Rmax = 100 m = 4 Hmax 2 2  
=   rad/s
 Hmax = 25 m T 60 30

22

Chapter 03: Motion in a Plane


76. For minute hand, TM = 60  60 s; for hour hand, 1200
86. n = 1200 r.p.m. = r.p.s. = 20 r.p.s.
TH = 12  3600 s 60
M TH 12  3600 1 a = 2r = (4 2 n2) r
 = = =12 : 1 ….[   ]
 H TM 60  60 T = 4  (3.142)2  (20)2  0.3
 4740 cm/s2
77. TE = 24 hr, TH = 12 hr
E 2 / TE TH 12 1 900
     87. n = 900 r.p.m. = r.p.s = 15 r.p.s,
H 2 / TH TE 24 2 60
1.2
78. n1 = 600 r.p.m., n2 = 1200 r.p.m., d = 1.2 m  r = = 0.6 m
Using, 2
Increment in angular velocity,  = 2(n2  n1) a = 2r = (2 n)2 
1.2
= 540 2 m/s2
 = 2(1200  600) rad/min 2

ns
= (2  600)/60 rad/s 88. r = 10 cm = 0.1 m, a = 1000  10 m/s2
 = 20 rad/s a = 2r
79. For an hour hand, T = 12 hr = 12  3600 s a
 2 =
2 2 

io
r
= = = rad/s
T 12  3600 21600 a 1000  10
  = =  316 rad/s
80. T1 = T2  1 = 2 r 10  102

at
v v n = 316/2 = 50.3 r.p.s.  50 r.p.s.
=  = constant
r r  n = 3000 r.p.m.
v1 v 2 v r R
 =  1= 1= mv2
r1 r2 v 2 r2 r
lic 89. Using, Fs =
r
210 Fr 10 10
5
81. n1 = 0, n2 = 210 r.p.m. = r.p.s.  v2 = s = = 104
60 m 102
 210  
ub
d = 2(n2  n1) = 2   0  = 7  rad/s v = 100 m/s
 60 
mv2
d 2  210 90. F=
=  = 4.4 rad/s2 r
dt 60  5
1
If m and v are constants, then F 
P

82. C = 2r r
C r 
 r 
F1
=  2
2 F2  r1 
et

C
 v = r(2n) =  2  f = fC
2 mv2
91. Using, F =
….[  = 2n] r
mv2 10   5
rg

2
Using, v = r = 0.2  10 m/s = 2 m/s 250
83.  r=   =2m
F 125 125
84. Using, v = r
= r  (2n) = 0.4  2  5 Using, v2 =
Tr
Ta

92.
= 0.4  2  3.14  5 = 12.56  12.6 m/s m
85. Angular velocity of particle P mv2
Breaking tension T =
about point A, r
v v (r = length of the string)
A   50 1
rAB 2r  v2 =
Angular velocity of particle P about point C, 1
v v  v = 5 2 m/s
C  
rBC r
93. Using, F = mr2 = m  42n2r
A v r  m  42n2r = 6  1014
= 
C 2r v 6  1014
A 1  n2 =
= 4  1.6 1027  3.142  0.12
C 2  n  5  106 cycles/s
3

MHT-CET Triumph Physics (Hints)


96. Centripetal acceleration, dx
 2av = 108v  18xv [Since v = ]
g l sin  dt
acp =  r =
2
= g tan
l cos   2a = 108  18x
= 10  tan 60 = 17.3 m/s2 108  18  3
 a=
2
97. Using,
a = 27 m/s2
mr2 = T and  = 2n
1 T 8.  v = 0 + na
n=  2Hz
2 mr  a = v/n
Now, distance travelled in n seconds,
Critical Thinking 1 2
Sn = an
2

ns
1. x
A B C O D E
distance travelled in (n  2) seconds,
+x
76 5432 1 0 1 2 3 4 5 6 7 8
(m) 9 8 1
(m) Sn 2 = a(n  2)2
2
i. The displacement of the man from A to E  Distance travelled in last two seconds,
is x = x2  x1 = 7 m (8m) = + 15 m

io
1 2 1
directed in the positive x-direction. = Sn  Sn  2 = an  a(n 2)
2
2 2
ii. The displacement of the man from E to C
a 2
is x = 3m – (7m) = 10m directed in = [n  (n  2)2]

at
2
the negative x-direction.
a
iii. The displacement of the man from B to D = [n + (n  2)][n  (n  2)]
is x = 3m  (7m) = +10m directed in 2
2v(n  1)

2.
the positive x-axis.
Equation of motion can be applied if the
lic v
= a (2n 2) = (2n 2) =
n n

acceleration is in opposite direction to that of 9. For 1st part,


velocity. In the uniform motion, acceleration of v = u + at = 0 + 2  10
v = 20 m s1
ub
the body is zero.
1 2
4. Particle is performing oscillatory motion s1 = ut + at
 Distance covered = 1 m + 1 m = 2 m 2
1

Total path length s1 =  2  100
P

Average speed = 2
Total time interval
s1 = 100 m
2
= = 20 m/s For 2nd part,
0.1 s2 = ut
et

| Average velocity | | displacement | s2 = 20  30


5.  1
| Average speed | | distance | s2 = 600 m
because displacement will either be equal or less For 3rd part,
rg

than distance. It can never be greater than v2 = u2 + 2as3


distance. 0 = (20)2 + 2  (4)  s3 = 400  8s3
400
x  s3 =
6. Instantaneous velocity v  8
Ta

t  s3 = 50 m
By using the data from the table  s = s1 + s2 + s3 = 100 + 600 + 50
0  (2) 60
v1 = = 2m/s, v2 = = 6 m/s  s = 750 m
1 1
16  6 10. Let initial velocity of particle = u
v3 = = 10 m/s For first 5 seconds of motion, s5 = 10 metre
1
1 2
So, motion is non-uniform but accelerated. s = ut + at
2
dv 1
7. Acceleration of the particle, a =
dt  10 = 5u + a(5)2
2
Now, v = 108x  9x
2 2
2u + 5a = 4 .…(i)
 Differentiating w.r.t t, For first 8 seconds of motion, s8 = 20 metre
dv dx dx 1
2v = 108  9  2x 20 = 8u + a(8)2
dt dt dt 2
44

Chapter 03: Motion in a Plane


 2u + 8a = 5 .…(ii) Given u = 0, a = constant acceleration and
Solving equations (i) and (ii), b = constant deceleration.
7 1 v = 0 + at
u= m/s and a = m/s2
6 3 Also 0 = v  b(t  t)
Now distance travelled by particle in total  v = at
10 seconds. From (ii), v = bt + bt
1 7 11  at = bt + bt
s10 = u  10 + a(10)2 =  10   100
2 6 2 3 b
 (a + b)t = bt  t = t
= 28.3 m (a  b)
so the distance travelled in last 2 seconds But v = at
= s10  s8 = 28.3  20 = 8.3 m  Maximum velocity attained = at
11. When two bodies meet, sA = sB ab

ns
v= t m/s
1 2 (a  b)
 at = vt
2
2v
17. At a particular instant, the particle will not have
 t= different positions.

io
a
19. Graph (D) indicates 2 values of velocity for a
dx
12. v= = v0 + gt + ft2 given instant. This is not possible.
dt

at
 dx = (v0 + gt + ft2) dt 21. The velocity of body is given by the slope of
x 1 displacement – time graph. So it is clear that
  dx =  (v0  gt  ft 2 )dt initially slope of the graph is positive and after
0 0 some time it becomes zero (corresponding to the
 x = v0 +
g f

lic peak of graph) and then it will become negative.
2 3
1 2 1
22. x1 = at and x2 = ut  x2 – x2 = at2 – ut
13. According to given relation, 2 2
acceleration, a = t + 
ub
1 2
dv dv y = at – ut. This equation is of parabola.
As a = t    2
dt dt dy d2 y
Since particle starts from rest, its initial velocity = at – u and 2 = a
dt dt
is zero i.e., At time t = 0, velocity = 0.
P

2
v t dy
As 2 > 0, graph shown possesses minima at
  dv   (t  )dt dt
0 0
u
t 2
t= .
et

 v  t a
2
23. The height is given by area under the v  t
1 graph.
14. 3s = 9t + 5t  s = (9t + 5t2)
2
rg

3 1
 Height = Area of trapezium = 3.6 (12 + 8)
ds 1 2
Velocity = = (9 + 10t)
dt 3 = 36.0 m
Ta

d  ds  d s 10
2
Acceleration =  = = m/s2 24. Speed of stone in a vertically upward direction
dt  dt  dt 2 3 is 20 m/s. So for vertically downward motion let
u = 20 m/s
dt 1
15.  2x    v  v2 = u2 + 2gh =(20)2 + 2  10  200 = 3600
dx 2x  
 v = 60 m/s
dv dv dx
 a  .
dt dx dt 25. Let the two balls meet after t at distance x from
dv v 2 the platform.
av   2 v v2  2v3 1 2
dx (2x  ) 2 Using h = ut + gt
2
 Retardation = 2v 3
For the first ball
16. Total time of motion = t u = 0, t = 18 s, g = 10 m/s2
Duration of acceleration = t 1
 x=  10  (18)2 ....(i)
Duration of deceleration = t – t 2
5

MHT-CET Triumph Physics (Hints)



For the second ball
34. r  3t ˆi  2t 2 ˆj  5kˆ
u = v, t = 12 s, g = 10 m/s2

 
1  dr d
 x = v  12 +  10  (12)2 ....(ii) v  3t ˆi  2t 2 ˆj  5kˆ  3iˆ  4t ˆj
2 dt dt
From equations (i) and (ii),
1 1 1 2
 10  (18)2 = 12v +  10 (12)2 35. s = ut + at
2 2 2
1 1
12v =  10  [(18)  (12)2] =  10  180
2 ˆ + 1  (2)2 (3iˆ  5j)
= 2 (2iˆ  4j) ˆ
2 2 2
1 10  30  6 = 10iˆ  2jˆ
v = = 75 m/s
2  12 
 | s | = 102  22 = 104 = 10.2 m
26. The initial velocity of stone = 29 m s1

ns
1
Using h = ut  gt2 36. The relative velocity of boat w.r.t. water
2 = vboat  vwater
1
h = 29  10   9.8  10 10 = (3 î + 4 ĵ ) – (– 3 î – 4 ĵ )
2

io
= 200 m (Neglecting negative sign) = 6 î + 8 ĵ
28 Assertion is based on visual experience. Reason is
formula of relative velocity. But it does not

at
explain assertion. The correct explanation of 37. Car N

assertion is due to visual perception of motion (due v ct 
angular velocity). The object appears to be faster
lic vc W E
when its angular velocity is greater w.r.t. observer.
29. If v1 is the speed of swimmer in still water and Train   S
  vt 
v2 is the speed of flow of river then relative  
velocity of swimmer in the direction of flow is
ub
v1 + v2 = 16 km/h ....(i)   
Relative speed in opposite direction v ct = vc + ( vt )
v1  v2 = 8 km/h ....(ii)  Velocity of car w.r.t. train (vct) is towards
On solving (i) and (ii) West-North.
P

v1 = 12 km/h, v2 = 4 km/h
38. Now,
30. Effective speed of the bullet
15 min = 0.25 hr
= speed of bullet + speed of police jeep
= 180 m/s + 45 km/h Let the velocity of the river be x km/h.
et

= (180 + 12.5) m/s = 192.5 m/s  (5  x)  0.25 = 1


Speed of thief ’s jeep = 153 km/h = 42.5 m/s  5  x = 4  x = 1 km/h.
Velocity of bullet w.r.t thief ’s car
39. Here,  = 90, t = 10 s,
rg

= 192.5 – 42.5 = 150 m/s  

31. Length of train A = 120 m, Resultant velocity = v E  v N


Velocity of train A, vA = 20 m/s  
Length of train B =130 m = 2v sin  
Ta

 2
Velocity of train B, vB = 30 m/s
The trains are moving in opposite direction, [Refer short cut 7]
1
 Relative speed of A w.r.t. B = vB + vA = 50 m/s =25
Total path length to be covered by 2
B = 130 + 120 = 250 m = 5 2 m/s
250
 Time taken by train B = =5s  Acceleration =
5 2
=
1
50 10 2
32. Relative velocity of bird w.r.t to train is v BT Acceleration acts in N  E direction
= vB  vT = (5 m/s) – (20 m/s) = 25 m/s
Length of the train, L = 150 m N
Time taken by the bird to cross the train is
L 150
t= = =6s
vBT 25

66

Chapter 03: Motion in a Plane


40.   N u = 500 m/s
 v1 v1


W E
v vx
 
v2 S vnet
vy
Magnitude of change in velocity,  Angle with which it strikes the ground,

 v   100  1  1 
| v | = 2v sin   ….[Refer Short cut 7]   tan 1  y   tan 1    tan  
2  vx   500  5

 90 
 | v | = 2  10  sin   = 10 2 = 14.14 m/s 48. x  36t
 2  dx
 vx   36 m/s

ns
Direction is south-west as shown in figure. dt
41. From the addition of two vectors, we know that y = 48 t – 4.9 t2  vy = 48 – 9.8 t
C2 = A2 + B2 + 2AB cos at t = 0 vx = 36 and vy = 48 m/s
From this expression it is clear that, So, angle of projection

io
C2  A2 + B2 when   90 v   4
  tan 1  y   tan 1  
i.e., when   90, the man can cross the river  vx  3
with shortest time.
   sin 1 (4/5)

at
North
B C
2u 2  96
49. Time of flight = = =6s
 
g 32
vB vR
lic 50. tA =
u sin 
 g

W  E 1
O A 1000 
ub
vA 2 = 500
=
From diagram it is clear that man should swim 9.8 9.8
in north-west direction.  tA = 51.02  51s

42. For shortest possible 51. For same range, angle of projection should be 
P

path man should vR and (90 – ).


swim with an angle W E 2usin 
So, time of flights t1 = and
(90 + ) with g

et

downstream. vm 2u sin(90  ) 2u cos 


From the figure, t2 = 
g g
vR 5 1 vR
sin      4u 2 sin  cos 
v m 10 2  t1t2 =
rg

g2
  = 30°
2 (u 2 sin 2) 2R
So angle with downstream = 90 + 30 = 120 t1t2 = 
g g g
45. At the highest point of the path, potential energy
Ta

is maximum, so the kinetic energy will be  t1t2  R


minimum. 2u sin  2u y 2  u vertical
52. Time of flight =  
46. At the highest point, velocity has only g g g
horizontal component.
v u 2 sin 2  2usin 
 v = v cos  = v cos 60 = . 53. H= and T =
2 2g g
4u 2 sin 2 
47. For horizontal projectile motion,  T2 =
g2
horizontal component of velocity
2
T 8
vx = ux + axt = u ( ax = 0) = 500 m/s  =
H g
vertical component of velocity
vy = uy + ayt = 0 + 10  10 ( ay = g)  T=
8H
=2
2H
g g
= 100 m/s
7

MHT-CET Triumph Physics (Hints)

v2 sin 2 u 2 sin 2 
54. R= 66. H=
g 2g
1  gR  Differentiating partially,
 = sin1  2 
2 v  u 2 (2sin .cos )
H =
2g
u 2 sin 2
55. R= H 2cos 
g  = = 0.1 (given)
H sin 
10  10  sin 60 3 T cos 
 R= = 10  = 8.66 m Similarly, = = 0.05
10 2 T sin 
u 2 sin 2 Therefore, T increases by 5%.
56. R =  R  u2. So if the speed of
g 67. Given, y = ax  bx2.

ns
projection is doubled, the range will becomes Comparing this equation with
four times, gx 2
y = (tan)x 
4  50 = 200 m 2u 2 cos2 
u2 g
 a = tan and b =

io
57. Hmax =
2g 2u 2 cos 2 
1 u 2 sin 2
Hmax  Horizontal range R =
g g

at
1 2u 2 sin  cos 
On planet B value of g is times to that of A. =
9 g
So value of Hmax will become 9 times 2u 2 cos 2  a
= (tan) =
2  9 = 18 metre
lic g b
2sin 
59. =2s u 2 sin 2
g 69. R=
g
 u sin = 10
ub
g
u 2 sin 2  100 On moon gm = . Hence Rm = 6R
 H= = =5m 6
2g 2  10
u 2 sin 2
60. The pilot will see the ball falling in straight line 70. R=
g
P

because the reference frame is moving with the


140  2
 sin 2
same horizontal velocity but the observer at rest  1000 =
will see the ball falling in parabolic path. 9.8
1000  9.8

et

x sin 2 =
61. x = 2t  t = . Substituting for t in y = 5t2 140 
2

2
5x
2  sin 2 = 0.5
gives y = , i.e., y  x2 which represents a   = 15
rg

4
parabola. Hence the correct choice is (D). u 2 sin 2
71. R=
63. Comparing with equation of projectile g
gx
2
For  = 15, R = 1.5 km
Ta

y = x tan  , Rg 1.5  9.8


2u cos 
2
u2 = = = 29.4
tan = 3   = 60 sin 2 sin 30
For  = 45
 g  29.4  sin 2  45 
64. y = (tan ) x   2 2  x2 29.4  sin 90
 2u cos   R= = = 3 km
9.8 9.8
For  = 45, u = 20 m/s
72.  = 90  30 = 60
  2
y = (tan 45) x  
g
x Horizontal velocity = u cos 60 = 19.6 m s1
 2  20  cos 45 
2 2
  19.6
 u= = 39.2 m s1
gx 2 0.5
y=x
u 2 sin 2 60  39.2 
2 2
400  3
H= =  
 gx  2g 2  9.8  2 
y = x 1  
 400   H = 58.8 m
88

Chapter 03: Motion in a Plane


73. For a given speed Total time 140s
86. No. of revolutions = =
since horizontal ranges are same, Time period 40s
 other angle is (90  60) = 30 = 3.5 Rev.
u 2 sin 2 60 3  u 2  So, distance = 3.5  2R = 3.5  2  10
H = =  
2g 4  2g   220 m
u 2 sin 2 30 1  u 2  87. In 15 seconds hand rotates through 90°
H = =  
2g 4  2g  
 
Change in velocity  v = 2v sin  
H 1 4 2  
=  
H 4 3  90  v2 
1 = 2(r) sin   v1
H = H  2 
3 90°
2 1

ns
=21 
74. v = u  2gy
2 2
T 2
2
1  4  2 cm
 u  = u  2gy
2
= =
 3  60 2 30 s

io
u2 (Note: Refer Shortcut 2)
= u2  2gy
9 88. Using,
 1  = 2n = 2  1 = 2 rad/s
u2  1   = 2gy

at
 9 a = r2 = 0.4  (2)2 = 0.4  4 2
8 u2 9  2y 9y a = 1.6 2 m/s2
u2  = 2gy  = =
9 g 8 4
89. Using,
Hmax =
u2 1  9y  9y
=  =
lic a = 2r = 42n2r = 4(3.14)2  12  20  103
4g 4  4  16  a  8  105 m/s2
75. Horizontal range of projectile is same for any T
Using, T = mr2  2 =
ub
90.
 
two angles,  and     , when projected with mr
2  6.4
same velocity.  =  3 rad/s
0.1  6
For  = 35,
P

  91. Using, T = m2r


  =  35 = 55
2 2  10 = 0.25  2  0.1
  = 20 rad/s
79. In uniform circular motion, acceleration is
et

caused due to change in direction and is directed 92. Using, F = mr2 = m 42n2r
radially towards centre.  m 42n2r = 4  1013
80. Work done by centripetal force in uniform 4 1013
 n=
rg

27
circular motion is always equal to zero. 1.6 10  4  3.142  0.1
82. The instantaneous velocity of a body in U.C.M.  n = 0.08  108 cycles/second
is always perpendicular to the radius or along mv2
93. The centripetal force, F =
Ta

the tangent to the circle at the point. r


83. For seconds hand, T = 60 s, mv2
 r=
r = 3 cm = 3  102 m F
2 2  r  v2 or v  r
= = = 0.1047 rad/s
T 60 v1 r 1
and v = r = 0.1047  3  102 = 0.00314 m/s (If m and F are constant),  1 
v2 r2 2
600
84. n = 600 r.p.m. = r.p.s. = 10 r.p.s. 300
60 94. Frequency of wheel, n = = 5 r.p.s.
60
v = r = r  2n = 10  2  3.142  10
Angle described by wheel in one rotation
= 628.4 cm/s.
= 2 rad.
85. Using, Therefore, angle described by wheel in 1 sec
v = r = 0.5  70 = 35 m/s  = 2  5 radians = 10  rad
9

MHT-CET Triumph Physics (Hints)


95. n = 2000, distance = 9500 m 20 1
104. T =  = 0.5 s
Distance covered in ‘n’ revolutions = n(2r) 40 2
= nD 2 2
=  = 4 rad/s
 2000D = 9500 T 0.5
9500 Let r = 50 cm = 0.5 m
 D= = 1.5 m
2000   v = r = 0.5  4 = 2 m/s
96. Period of second hand = Ts = 60 s and 105. T = 24 hr, r = 6400 km
Period of minute hand = Tm = 60  60 2 2 2  3.14  6400
= 3600 s v = r = r=  6400 =
T 24 24
2 2 v  1675 km/hr
Angular speed of second hand s = =
Ts 60
ˆi ˆj kˆ
2 2 

ns
 
Angular speed of minute hand m = = 106. v =   r = 3 4 1 = 18iˆ  13jˆ  2kˆ
Tm 3600
5 6 6
s 2 3600
 =  = 60 : 1
m 60 2 107.  = 2t3 + 0.5

io
d
97. For minute hand, T = 60 min = 60  60 s  = (2t3 + 0.5) = 6t2
dt
2 2
Angular speed,  = = rad/s At t = 2 s,  = 6  22 = 24 rad/s
60  60

at
T
 180 108. Radius of horizontal loop, r = 1 km = 1000 m
=  = 0.1
1800  900 103
o
v = 900 km/h = = 250 m/s
180 3600
lic
….[ 1 rad =

]
 a=
v2 250  250
 = 62.5 m/s2
angle described 2 r 1000
98. =  =  rad/s a 62.5
time taken 2  = = 6.25
ub
g 10
99. v = r.
where r is distance from axis of rotation. 109. Velocity, v = r
At the north-pole, r = 0  v = 0 r v
 v = r =  = 10 cm/s
2 2
P

n
100. Frequency = r.p.s., t = 1 min = 60 s  a = 2r
60 r a
n  a = 2r = 2  = = 10 cm/s2
Angular velocity,  = 2 2 2
60
et

2n   22n 110. In half a circle, the direction of acceleration is


 Linear velocity, v = r = = cm/s reversed.
60 60
v2 v2
It goes from to
rg

101. Using, r r
2 2 3.14
v = r = r  = 60  = 6.28 mm/s Hence, change in centripetal acceleration
T 60
v2   v 2  2v2
v = 6.28 2 mm/s  8.88 mm/s =  =
Ta

r  r  r
2
102. Speed of C1 = R1 = R1  2 
2
T 111. Using, Fcp = m2r = m   r
2  T 
Speed of C2 = R2 = R2 2
T  22 1 
= 500  10–3   2     0.49
Speed of C1 2R1 / T R  7 11 
 = = 1
Speed of C2 2R 2 / T R2 500 103 16  0.49
= = 0.08 N
15 49
103. r = 0.25 m, n = 15 r.p.m. = r.p.s.
60 112. m = 2 kg, r = 1 m, F = 32 N
2   15  Force, F = m2r
 = 2n =  rad/s
60 2 32
 
 2 = = 16
v = r = 0.25  = m/s 2 1
2 8   = 4 rad/s
10

Chapter 03: Motion in a Plane


 Frequency of revolution per minute 118. Using,
 47 r = l sin
n=  60 =  60  38 rev / min
2 2  22 r = 10 sin 30  r = 5 m, T = 3 s
2 2
113. r = 20 cm = 20  102 m = 0.2 m = =
T 3
mv2 Centripetal force = m2r
Using, F = = 10
r 42
1 2 r 0.20 = 5  102  5
 mv = 10  = 10  =1J 9
2 2 2 = 25  102  4
114. Breaking tension = 4  10 = 40 N = 100  102  1 N
 T = mr2 mg
T 40 119. T =
cos 

ns
  = 2
 = 200
mr 200  103  1
h L2  r 2 
   14 rad/s cos = = h
L
L L
115. Since car turns through 90 after travelling mg L

io
T=
471 m on the circular road, the distance 471 m is L2  r 2 r
quarter of the circumference of the circular path. If
R is the radius of the circular path, then 120. Distance s  v2

at
1 s1 u2 1
(2R) = 471  = 2
=
4 s2 (2u) 4
471  2 471  2 s2 = 4s1
 R= = = 300 m
If s1 = 20 m then s2 = 4  20 = 80 m

v = 12 m/s, m = 1000 kg
3.14
lic 122. Taking the motion from 0 to 2 s
 Centripetal force,
u = 0, a = 3 m s2, t = 2 s
1000  (12)
2 2

Fcp =
mv
= = 480 N  v = u + at = 0 + 3  2
ub
R 300  v = 6 m/s
116. T = ma = mr2 Taking the motion from 2 s to 4 s
T  2 v = 6 + (3) (2) = 0 m s1
Hence, graph (A) represents the correct option.
2 T 4T
P

  =4
2 T T 123. From acceleration time graph, acceleration is
 2 = 42   = 2  constant for first part of motion so, for this part
velocity of body increases uniformly with time
n = 2n = 2  5 = 10 r.p.m.
et

and as a = 0 then the velocity becomes constant.


117. Using,
Then again increases because of constant
T sin  = m2r = m2 l sin  ….(i) acceleration.
T cos  = mg ….(ii)
rg

124. For the given condition initial height h = d and


velocity of the ball is zero. When the ball moves
60
downward its velocity increases and it will be
T maximum when the ball hits the ground and just
Ta

after the collision it becomes half and in


T cos opposite direction. As the ball moves upward its
ar velocity again decreases and becomes zero at
Tsin
mg height d/2. This explanation matches with graph
(A).
g
From (i) and (ii), 2 = 125. v2 = u2 + 2as, If u = 0 then v2  s
l cos 
i.e., graph should be a parabola symmetric to
g
 = displacement axis.
l cos
2 l cos 
127. Athlete completes 1 round in 40 s
 Time period, T = = 2 2 min 20 s = 140 s
 g
He will complete 3 rounds in 120 s and half
1 cos60 round in 20 s only.
= 2  3.14  = 1.4 s
10  Displacement = Diameter of circular path = 2R
11

MHT-CET Triumph Physics (Hints)


128. The horizontal component of velocity is In this case, time taken to strike the ground is
dx d the time of flight of the body and is given by
vx = = (at) = a
dt dt 2h
t=T=
The vertical component of velocity is g
dy d
vy = = (bt2 + ct) = 2bt + c  vertical component of velocity
dt dt vy = uy + ayt
When t = 0, vx = a, vy = 2b + c = 0 + gT
 v = v 2x  v 2y = a 2  (2b  c) 2 2h
vy = g  = 2gh
Hence, the correct choice is (A). g
129. Horizontal velocity vx = 20 m/s  Resultant velocity, v = v 2x  v 2y = u 2  2gh
Vertical velocity,
Thus, for both the particles, their final
vy = u + gt = 0 + 10  5 = 50 m/s

ns
velocities, when they reach the earth's surface
Net velocity, are equal.
v = v 2x  v 2y = (20)2  (50) 2  54 m/s.  
134. At the highest point, angle between a and v is

io
131. Horizontal component of velocity zero. Hence, total acceleration is only normal or
u radial acceleration.
uH = u cos 60 =
2 v2
 a=

at
ut u
AC = (uH) t = R
2 but a = g
 ut   2  ut (u cos )
AB = AC sec 300 = 
2
 =  g=
 2   3 3 
lic
2usin  2 × 50 ×1
2
R
u cos  2

132. Total time of flight = = =5s  R=


g 2 ×10 g
g
Time to cross the wall = 3 s (given)
ub
Time in air after crossing the wall
= (5 – 3) = 2 s 135.
4 m/s
 Distance travelled beyond the wall
3
= (u cos) t = 50   2 = 86.6 m P
P

vx
2 
133. When particle is thrown in vertically downward v
vy
direction with velocity u, then final velocity at
et

the ground level is For a projectile given horizontal projection,


Motion along X-axis is given by,
u 1 2
x = x0 + uxt + axt
2
rg

h
2
v u  2gh Here x0 = 0, ux = u = 4 m/s, ax = 0, t = 0.4 s
Hence horizontal distance covered by the ball,
v2 = u2 + 2gh x = ut = 4  0.4 = 1.6 m
Ta

Thus, option (A) is incorrect.


 v  u 2  2gh The speed with which it hits the ground,
Another particle is thrown horizontally with v = v 2x  v 2y = u 2  g 2 t 2 ….(i)
same velocity then at the surface of earth.
= 16 102  (0.4) 2  5.66 m/s
h Thus, option (B) is incorrect.
Motion along Y-axis is given by
vx= u 1 2
y = y0 + uyt + a yt
vy = 2
2gh v
Here y0 = 0, uy = 0, ay = g, t = 0.4 s
For horizontal projectile motion, Hence height of the table
horizontal component of velocity 1 2 1
vx = ux + axt h=y= gt =  10  0.42 = 0.8 m
2 2
= u ( ax = 0) Thus, option (C) is correct.
12

Chapter 03: Motion in a Plane


The angle body makes with horizontal is Tension T in the string will provide centripetal
v  mv2
 = tan1  y  force  T ….(i)
 vx  l
Also, tension T is provided by the hanging ball
 16 
= tan1   ….Using equation (i) of mass m,
 16 
 T = mg ….(ii)
= 45.
mv2 v2
Thus, option (D) is incorrect. mg = g=
l l
136. In one complete revolution, total displacement
is zero. So average velocity is zero. 144. Centripetal force on electrons is provided by
electrostatic force of attraction.
p 1
137. r = , n =   r.p.s.  F and r  n2 where n is principal quantum
t 2

ns
r
p 22 p no.
v = r = r  2n =   2  = 4
t t F1 n 24  3  81
    
F2 n14  2  16
mv2

io
138. F =
r 145. As  is constant, acceleration is due to the
 F  v2. If v becomes double, then F (tendency to change in direction of velocity = 2r
overturn) will become four times. As rA > rB  aA > aB

at
139. r1 = 9 cm mv 2
In the given condition, friction provides the 146. p = mv; F 
r
required centripetal force and that is constant. i.e.
m2r = constant.
2
lic 
F
p
=
mv 2

r mv r
1
=
v

 
2
1 1
 r  r2 = r1  1  = 9   = 1 cm
2 
 2 3
Competitive Thinking
ub
140. Using,
smg  mr2 1. Average velocity =
Displacement
sg = r2 (For minimum angular speed) Timeinterval
s g 0.25  9.8 25 A particle moving in a given direction with non-
2 = =   9.8
P

r 5  102 5 zero velocity cannot have zero speed.


= 9.8  5 = 49.0 In general, average speed is not equal to
  = 7 rad/s magnitude of average velocity. However, it can
be so if the motion is along a straight line
et

mv2 without change in direction.


141. F =
r
2. As displacement is either smaller or equal to
F  v2 i.e. force will become 4 times.
distance but never be greater than distance.
rg

142. Let the bead starts slipping


after time t 3. Final velocity depends on acceleration and initial
For critical condition, velocity and is independent of mass of the bodies.
Ta

frictional force provides 4. When a person travelling in straight line covers


the centripetal force L equal distance in different time intervals then
m2L = R = m  a1 = Lm average velocity/speed is given by,
 m(t)2L = mL 2 v1 v 2
vavg =
 v1  v 2
t= ... [  = t]
 2
 = 1 + 1
v v1 v2
143.
5. Time covered for first half of the distance,
O T m s/2 s
l t1 = =
7 14
T Let t2 be the time for the second half of the journey,
m s
m = 14t2 + 21t2 = 35t2
2

13

MHT-CET Triumph Physics (Hints)


s A B
t2 =  x= (4  1) + (8 –1)
70 2 3
Total distance 3 7
average velocity =  x= A+ B
Total time 2 3
s s
= = = 10 m/s 9. x  t 1
t1  2t 2 s 2s
 Squaring both sides,
14 70
x = (t + 1)2 = t2 + 2t + 1
6. Let velocity of Preeti be v1, velocity of escalator Differentiating it w.r.t. time t,
be v2 and distance travelled be l. dx
 2t  2
distance dt
 Speed =
time dx
Velocity, v =  2t  2

ns
distance dt
time =
Speed
10. Given:
t=
l x = At3 + Bt2 + Ct + D
v1  v 2 dx
= 3At2 + 2Bt + C

io
v=
l tt dt
t= = 12
l

l t 2  t1 At t = 0, initial velocity vi = C
t1 t 2 dv
Also, a = = 6tA + 2B

at
dt
7. Displacement x = t(t  1)(t  2) ….(i) At t = 0, initial acceleration ai = 2B
 x = t3  2t2  t2 + 2t a i 2B
dx  
Velocity v =
dt
lic vi C

 v = 3t2  6t + 2
0 t
dv
For v = 0, we have
11. 
6.25 v
 2.5 dt
0
3t2  6t + 2 = 0 ….(ii)
ub
0
2 v = 2.5 t
Equation (ii) is quadratic in t 6.25

Solving we get, 2 6.25 = 2.5t


 1   1  t=2s
t = 1   or t = 1   ….(iii)
 3  3
P

 

Now;
 vi  v f
12. v avg 
x = t(t – 1) (t  2) ….from (i) 2
substituting equation (iii) in equation (i) u  u  at1
et

v1 =
 1   1   1  2
x = 1   1   1 1   2
 2v1 = 2u + at1 ….(i)
 3  3   3 
Solving we get, Similarly,
rg

2 2v2 = 2u + at2 ….(ii)


x=  m 2v3 = 2u + at3 ….(iii)
3 3
Similary, Subtracting equation (ii) from equation (i),
2(v1  v2) = a(t1  t2)
Ta

By substituting the other value of t from ….(iv)


equation (iii) in equation (i) and Subtracting equation (iii) from equation (ii),
Solving we get, 2(v2 – v3) = a(t2 – t3) ….(v)
2 Dividing equation (iv) by equation (v),
x= m
3 3 v1  v 2 t1  t 2

v 2  v3 t 2  t 3
8. Given : v = At + Bt2
dx dx
 = At + Bt2 13. Velocity v =
dt dt
2
dx P
 x =   At  Bt 2 dt  vP = = a + 2bt
dt
1
dx Q
 At 2 Bt 3 
2
vQ = = f  2t
 x=    dt
 2 3 1 as vP = vQ ....(given)
14

Chapter 03: Motion in a Plane


a + 2bt = f  2t 17. v = 6t – 3t2
 (2 + 2b)t = f  a 
dx
= 6t – 3t2
f a dt
 t=
2 1  b  2

 x =  6t  3t 2  dt
0
dv
14. = – 2.5 v 2
dt  6t 2 3t 3 
1  x =  
 dv = –2.5dt  2 3 0
v
0 t
 x=4
dv
Now, integrating,   2.5 dt Average speed =
total distance travelled
6.25 v 0 total time
0
 1
 4

ns
 2  v 2  = –(2.5)t Average speed = = 2 m/s
2
  6.25
1
18. Case A:
2   6.25  2
 t= In accelerated motion,
2.5 a = 5 m/s2, u = 0

io
 t = 2s 1 2
s1 = ut1 + at1
15. time taken (t) = 8s 2

at
Let t1 be time for acceleration and t2 for declaration 1
 s1 = 0 + 5 t12
 t1 + t2 = 8 2
 t2 = 8 – t1 …..(1) 5t12
For acceleration motion,  s1 = ….(1)
2
v0 = u + at
v1 = 0 + at1
lic Case B:
In uniform motion,
v1 = at1 …..(2) a = 5 m/s2, u = 0
For retardation,  v = u + at
ub
v = u + at
 v = 0 + 5t1
 v2 = v1 + at2 ( u = v1)  distance covered by the car, s2 = v.t
Substituting equation (1), (2) and a = – 3a, s2 = 5t1 t2 ….(2)
we get, Case (C):
P

0 = at1 – 3a (8 – t1) In decelerated motion,


 3a (8 – t1) = at1 a = 5t1, a = –5m/s2, v = 0
 24 – 3t1 = t1 1
 s3 = 5t1 t3 – 5t 32

et

4t1 = 24 2
 t1 = 6s 5t 2
 s3 = 5t1 t3 – 3 …..(3)
16. Let the car accelerate with acceleration  for 2
time t1 and decelerate for time (t – t1) with Using v = u + at
rg

deceleration .  0 = 5t1 – 5t3


  5t1 = 5t3
 v t .…(i)
  t1 = t3 ….(4)
Ta

1  2 Given, t1 + t2 + t3 = 25 ….(5)
 s t  Substituting (4) in (5)
2 
2t1 + t2 = 25
Given:  = 2 m/s2, t = 20 s, s = 100 m
 t2 = 25 – 2t1 ….(6)
2
   20 
2
100 = We have,
2  2  
Totaldistance
 2 +  = 4 Average velocity =
Total time
2
 = 5 s s s
3  72  = 1 2 3 ….(vii)
18 t1  t 2  t 3
Substituting this in equation (i),
2 Substituting (i), (ii), (iii) in equation (vii),
2  20  5t12
3 80 5t 32 
v= =
8
= 10 m/s  2  5t1t 2  5t1t 3  2 
 2
 2   3     20 =  
 3 3 25

15

MHT-CET Triumph Physics (Hints)

5t12 5t 2 B
 500 =  5t1t 2  5t1t 3  3
2 2 Speed (m/s)
From equation (iv), we have, A D
5t 2 5t 2
500 = 1  5t1t 2  5t12  1
2 2
 500 = 5t1t2 + 5t12 O t (s)
C
 500 = 5t1(25 – 2t1) + 5t 2
1 1
The area of the ABD represents the term ‘ at 2 ’
 100 = 25t1 – 2t  t 2
1
2
1 2
 t12 – 25 t1 + 100 = 0  The total area OABC is best described using
1 2
Solving, we get, s = ut + at
2

ns
t1 = 20 s, t1 = 5 s
t1 ≠ 20 s as it does not satisfy t1 + t2 + t3 = 25 25. Car at rest attains velocity of 6 m/s in t1= 1 s.
and t1 = t3 Now as direction of field is reversed, velocity of
car will reduce to 0 m/s in next 1 s. i.e., at
 t1 = 5 s
t2 = 2 s. But, it continues to move for next one

io
 time for uniform motion = 25 – 2t1 second. This will give velocity of –6 m/s to
= 25 – 2  5 car at t3 = 3 s.
= 15 s Using this data, plot of velocity versus time will

at
be
19.

Velocity (m/s)
6
B
Velocity

A
lic
3
50 time (s)
1 2
25
ub
Time
a = slope of v  t graph = tan  –6
a A tan 25
 
P

a B tan 50 Area under thegraph


Average considering sign

20. Instantaneous velocity of particle is the slope of velocity time
the distance v/s time graph. For given graph, the 3 33
et

slope is maximum around the point R. Hence, = = 1 m/s


3
velocity is maximum at point R. Area under thegraph
Average without considering sign
21. t (s) 
rg

speed time
O
3 3 3
= = 3 m/s
3
Ta

v (m/s) 26. u2 = 20 m/s


u1 = 40 m/s
a1 = –4 m/s2 10
a2 =  m/s2
4
23. The graphs (A), (B) and (C) represent the
uniformly retarded motion, i.e., velocity
decreases uniformly.
However, the slope of the curve in graph (D), Let s1, s2 be the distance travelled by train I and
indicates increasing velocity. Hence, graph (D) train II before halting.
is incorrect. u12 1600 u 22 400
 s1 = = ;  s2 = =
2a1 8 2a 2 5
24. The area under the speed time graph gives the s1 = 200 m ; s2 = 80 m
displacement ‘s’. separation between the trains when both have
The area of the  OADC represents the term stopped is,
‘ut’. s  s1  s2 = 300 – 200 – 80 = 20 m
16

Chapter 03: Motion in a Plane


27. At point A, u = 0 Now for,
t1 = 10 to t2
A u=0
h1
v2 = u2 + 2as
t = 5s
h2 B h2 = 2495  500 = 1995 m
h3 t = 5s v2 = v12 + 2ah2
C
= (100)2  (2  2.5  1995)
t = 5s
= 10000  (5  1995)
D
= 10000  9975
1 2 1 = 25
 h1 = gt =  10  25.
2 2 v = 5 m/s
 h1 = 125 m 1 2
Now, v = u + gt = 0 + 10(5) 30. h= gt
2

ns
 v = 50 m/s
At point B, final velocity from A to B = initial  t= 2h / g
velocity at B 2a 2b
ta = and tb =
1 1 g g
 h2 = ut + gt2 = 50  5 +  10  25

io
2 2
ta a
Now, h2 = 375 m  =
tb b
v = u + gt = 50 + 10(5)

at
 v = 100 m/s 31. Given
Similarly, At point C, h3 = 625 m t = 10 s
 h1 : h2 : h3 = 125 : 375 : 625 = 1 : 3 : 5 a = g = 10 m/s2
h2 h v = u + at
i.e., h1 =
3
= 3
5
lic  v = gt ….( u = 0)
1 2  v = 10  10 = 100 m/s
28. AB = s = gt1 ….(i) ( u = 0) i. If an object is moving with a uniform
2
ub
1 acceleration and velocity of the object changes
2s = g(t1  t 2 )2 ….(ii) from u to v in a time t, then average velocity is,
2
u  v 100
Substituting equation (i) in equation (ii), vavg = = = 50 m/s
2 2
1 2 1
2 gt1 = g(t1  t 2 )2
P

2 2 Alternate method:
2t1 = t1 + t2 1 2
s = ut + gt
2
t1  
2  1 = t2
et

1
 s=  10  102 …( u = 0)
t2 2
= 2 1  s = 500 m
t1
dis tan ce
vavg =
rg

29. t=0,u=0 time


500
 vavg = = 50 m/s
h1 10
Ta

t1 = 10 s h = 2495 m 32. For an object thrown upward,


Velocity half second before maximum height
h2
= velocity half second after maximum height
t2 = ? 1
 v = u + at = 0 + 9.8  = 4.9 m/s
2
For t = 0 to t = 10 s
1 2 33.
s = ut + at
2 u=0
1 h1
h1 = 0(10) +  10(10)2
2
h1 = 500 m
v1 = u + at
v1 = 0 + 10  10 u = 14 m/s
h2
v1 = 100 m/s
17

MHT-CET Triumph Physics (Hints)



For ball – 1,   
dv
1 2 a = =  2 cos t x  2 sin t y
h1 = gt [ u = 0] ….(i) dt
2  
For ball – 2, here, r  v = 0
  
1 2
h2 = 14 t – gt ….(ii)  v is perpendicular to r while a is directed
2
towards origin.
Substituting equation (i) in (ii),
   
h2 = 14 t – h1
39. v  u  a t  v  3iˆ  4jˆ  (0.4iˆ  0.3j)
ˆ 10
 h1 + h2 = 14t
 21 = 14t [ h1 + h2 = 21 m] v  3iˆ  4jˆ  4iˆ  3jˆ  7iˆ  7ˆj

 t=
3  | v | = 7 2 units
2

ns
  
Substituting in equation (i), 40. R = 4sin(2t) i + 4 cos 2t j
1 9 90
h1 =  10  = 

 
2 4 8 dR
v= = 8cos2t i – 8sin2t j
45 dt
 h1 =

io

4
P v = v2x  v2y
2
34. 2gH = nu (n – 2)
n=3 x = (8 cos 2 t) 2  ( 8 sin 2 t) 2

at
 2gH = 3u2 (3 – 2) = 8 m/s
2gh Q
 2
u =
3

(x 2  x1 )iˆ  (y 2  y1 )ˆj (13  2)iˆ  (14  3)ˆj
x 41. vav = =
t 2  t1
 u=
2gh
3 R
lic 11i  11j 11 ˆ ˆ
ˆ ˆ
50

= = (i  j)
5 5
35. Let t1 be the time taken from P to Q and t2 be
the time taken from Q to R. 
ub
42. v BA = v 2A  v B2  2v A v B cos 60
Motion of ball from P to Q,
1 2
x= gt1 ….(i) = 102  202  2  10  20  cos 60
2
Motion of ball from P to R, = 10 3 m/s
P

1
2x = g  t1  t 2 
2
….(ii)
2 43.

From (i) and (ii), we have, vA A
et

 2t12   t1  t 2 
2
W E
 2 t1 = t1 + t2 100 2 45

  C vB
 t1 2  1 = t2
rg

100 100 km
t1 1
 
t2  2 1  2
B
Ta

36. In both the cases, the coin is in free fall and the S
only force acting on it is gravity.
Velocity of ship A and ship B are:
2h 
 t1 = t2 = 
vA
g vA = 10 km/h
 

37. A geostationary satellite has same sense of vB = 10 km/h v AB  vB
rotation as that of the earth and has same period Velocity of A w.r.t B is
of rotation. Hence when observed from the   
surface of the earth, it appears stationary. v AB = vA  vB

  
38. r = cos  t x + sin  t y v AB  (10)2  (10)2 = 200

   
dr
v= =   sin t x +  cos t y  vAB  10 2 km/h directed along AC
dt

18

Chapter 03: Motion in a Plane


100 49. v v y
Displacement, AC = km
2 x
100
AC
 Time, t =   2
 
 1  1
v AB 10 2 r 1  vt ˆi  gt 2 ˆj , r 2  vt ˆi  gt 2 ˆj
2 2
Since both are perpendicular,
 t=5h  
r 1 r 2  0 ,
44. vR = 10 m/s
B 1
 v2 t 2  g 2 t 4  0
4
vMR 1 22
vM = 20 m/s
  v2 = gt
4

ns
gt
West A East  v=
2
2v
 t=

io
vM sin = vR g
vR 2v 4v2
 sin  =  x = 2vt = 2  v  =
vM g g

at
10 1
 sin  = = gx 2
20 2 50. Comparing with y = x tan –
2u cos 2 
2
  = 30 with normal (i.e., west)
 tan  = 1
46. H=
u 2 sin 2 
lic   = 45
2g g 2

According to given problem 2u cos  5
22

u 2 sin 2 30 u 2 sin 2 60 10 2


ub
 H1 = and H2 =  
2g 2g 1 5
2u 2 
sin 2 30 1 / 4  1
2
H1
= = = 50
H2 sin 2 60  3 / 4  3  u2 = = 25
2
P

47. Given : tA = 3 s ; tD = 6 s  u = 5 ms–1


 time of flight = tA + tD 51. We know, x = ux t
2u sin   x = u cos   t

et

=3+6
g
 2h = 2 gh  cos  60  t
2u    90 
 =9 ….   h 1
g  sin 90  1   t
rg

g 2
9  10
 u= h
2  t= 2
g
u = 45 m/s
Ta

Let ‘h’ be the vertical distance of point ‘X’ from 52. Total time of flight = 3 + 6
ground, 2u sin 
1 1 1  =3+6
 h = ut + at2 = ut  gt2 = 45(3)  (10)(3)2 g
2 2 2 2u    90 
 h = 90 m  =9 ….  
g  sin90  1
3 u2 0.866 u 2 9  10
48. RA = =  u=
2g g 2
u2 u = 45 m/s
RB = Let ‘h’ be the vertical distance of point ‘X’ from
g
ground,
3 u2 u2 1 2 1 1
RC = = 0.866  h = ut + at = ut  gt2 = 45(3)  (10)(3)2
2g g 2 2 2
 RA = RC < RB  h = 90 m
19

MHT-CET Triumph Physics (Hints)


53.  y2 = 5 m ….(ii)
3 ˆj
 after 1 s both bodies are at a distance of 5 m
from the horizontal.
 distance between the two bodies after 1s of
2iˆ projection is
x = (u2 cos)t = (20  cos 30) (1)
2iˆ 3
Horizontal (X) component remains the same = 20  = 10 3 m
2
while the vertical (Y) component changes.
Therefore, velocity at B = 2iˆ  3jˆ m/s.  56. Maximum height attained by projectile is,
u 2sin 2θ
H=
  2g
54. Given: u = i+ 2 j .…(i)

ns
 H  u2
Now
δH 2δu
 
=
u = u x i+ u y j H u
But 1 2δu δu 1
=  = ….(i)

io
ux = u cos …(ii) 10 u u 20
uy = u sin ….(iii) Time of flight is given by
comparing (i) and (ii) we get, 2usinθ
T=

at
u cos = 1 .…(iv) g
comparing (i) and (iii) we get,  Tu
u sin  = 2 ….(v) δT δu
dividing equation(v) by equation (iv), =
tan  = 2 ....(vi)
lic T
δT
u
1
Equation of projectile is given by, = ….from (i)
T 20
gx 2
y = x tan  2 2 % increase in time of flight is;
2u cos θ
ub
δT 1
….(Using equation (iv) and (vi) and 100 =  100
g = 10 m/s2) T 20
(10)x 2 δT
 y = (2)x   100 = 5%
2(1) 2 T
P

 y = 2x  5x2 v2 sin 2 v 2 sin 2 


57. R= and H =
55. y g 2g
et

Given : R = 2H
v 2 sin 2  v 2 sin 2  
u1  =2  
g  2g 
y1 = 5 m  2 sin  cos  = sin2 
rg

30  tan  = 2
x
but tan2 + 1 = sec2
x = (u2cos)t
1
Ta

Given: u1 = 10 m/s, u2 = 20 m/s, g = 10 m/s2,  22 + 1 =


cos 2 
 1 2
2 = = 30; 1 = 90  cos  =  sin  =
6 5 5
t1 = t2 = t = 1 s
v 2  sin 2 2sin  cos 
 distance travelled by the 1st body in 1 s is,  R= = v2 
g g
1 2 1
y1 = u1t + gt = (10)(1)  (10)(1)2 2 2 1
2 2 = v2   
 y1 = 5 m ….(i) g 5 5
distance travelled by the 2nd body in 1 s is, 4v2
R=
1 2 5g
y2 = (u sin  )t  gt
2
1 58. Refer Notes no. 16
= (20) (sin 30) (1)  (10)(1)2
2 59. Refer Notes no. 16
20

Chapter 03: Motion in a Plane

60. hour =
2π ∴ T A = TB
Thour 2 2  2 
 =  t   
=

×
180 c
...{ 1 =
180°
} A B  
12 × 60 × 60 π π A
1  =1:1
hour = degree / s B
120
69. The centripetal force acting on the particle is
61. Angular speed of second hand,
provided by the central force,
2
1 = (T = 60 seconds) mv 2 1
60  =K n
R R
Angular speed of hour hand,
2 R 1
2 = (T = 12 hr)  v =K
2
=K
12  60  60 mR n
mR n 1

ns
1 720  
= 12  60 =  v = K  1
 n 1
....  K   K 
2 1 2  m 
R
2 The time period of rotation is,
Angular speed of minute hand m =

io
62. n 1
60  60 2 R 2  R  R 2
2 n 1

2 T=   R 2
Angular speed of second hand s = v K K
60 n 1

at
2 2 59  TR 2
 s  m =  = rad / s
60 3600 1800
70.
63. v
ax 
a
P(R,)
lic T
l
m


R ay Here, tension provides required centripetal force.
ub
mv 2
 =T
l
 v2 v2 71.
a =  cos  ˆi  sin  ˆj
R R Tcos
P

64. They have same angular speed .  T



Centripetal acceleration = 2r h L Tsin m
2 r
et

a1 r
= 21 = 1
a2  r2 r2
mg
2 m
 2  2 r O
a = 2R =   (5  10 ) = 5 m/s
2
rg

65.
 0.2  

66. Since, n = 2,  = 2  2 = 4 rad/s2 The centripetal force required for circular


25 motion is given by
Ta

So acceleration = 2r = (4)2  m/s2 = 42


100 mv 2
= T sin ….(i)
67. Degree moved by hour hand, r
for 1 revolution = 360 Also we have,
360 mg = T cos  ….(ii)
for 1 hour = = 30 Dividing equation (i) by equation (ii) we get,
12
30 mv 2 1 Tsin 
for 1 min = = 0.5  =
60 r mg Tcos 
 for 20 mins = 20  0.5 = 10  v2 = rg tan
Hence, at 12.20 pm  v = rg tan  ….(iii)
Angular seperation = 120 – 10 = 110 From figure,
68. Time period of rotation of both the particles r
tan =
(A and B) is same h

21

MHT-CET Triumph Physics (Hints)

r Distance travelled by the wheel in half


 tan = ….(iv) (L2 = r2 + h2) C
L r
2 2
revolution = = AD
Substituting equation (iv) in equation (iii) we get, 2
 from figure;
r
v= rg Displacement of initial point of contact after
L2  r 2 half revolution = AB
 v= r
g  AB2 = AD2 +DB2
L  r2
2
C
2
AB2 =     2r 
2
2 2
mv 2 m  p  p2
72. Radial force =    = C
r r m mr But r =
2
….[ p = mv] 2 2
C C

ns
 AB2 =     
73. F = m2r  2 
2 C2 C2
Substituting for r = 2l,  =  AB =  = C
1

1
T 4 2  2 4

io
 2 
2

kl = m(2l)   ….(i) 77. For first particle


 T 
1 2
….( F = kx and x = l here) s = ut + at

at
2
Upon speeding, F1 = m12r1 240 = 10 t 
1
(10) t 2
2 2
Substituting for r1 = 3l, 1 =  5t2  10t  240 = 0

 2 
2
T1
lic  t2  2t  48 = 0
k(2l) = m(3l)   ….(ii)  t = 8 or t = – 6
 T1  The first particle strikes the ground at 8th
….( x = 2l here) second. Till 8 s, relative velocity
ub
240
Dividing equation (i) by equation (ii), = = 30 m/s
8
kl m(2l )(2 / T) 2 As relative velocity is constant, the relative
=
k(2l ) m(3l )(2 / T1 ) 2 acceleration is zero.
P

2
 T1  3 After 8 second the magnitude of relative
   = velocity increases upto 12 second when the
T 4
second particle strikes the ground.
3
 T1 = T
et

2 78. Case1: constant acceleration along straight line


u= 0, t1 = 2s
74. s=
1 2
at  v = u + at1
2  v = 0 + 2a
rg

For t = 4 s g sin  v = 2a ….(1)


1 1 2
s = g sin  (4)2 s = ut1 + at1
2 s 2
Ta

s 1 1
Now = g sin   (t)2   s=0+ a (2)2
4 2 2
 t = 2 s  s = 2a ….(2)
Case 2: After body reverses direction
75. B (Final velocity before direction reverse
= Initial velocity after direction reverse.)
r  v=u
1 2
r  s = ut2 – at 2
2
A C D
Substituting equation (1) and (2), we get
2
1 2
 – 2a = 2a t2 – at 2
Let A be initial position of point of contact and 2
B be its position after the wheel completes half 1 2
 – 2 = 2t2 – t
revolution. 2

22

Chapter 03: Motion in a Plane


4t 2  t 2 81. Given: v(x) = βx–2n
 –2 = dv dv dx
2  a=  
 –4 = 4t2 – t 2 dt dx dt
 t2 – 4t2 – 4 = 0 dv  dx 
 v
a=v 
  4   16  16 dx  dt 
 t2 =
d
2 a = x–2n (x2n)
 neglecting    ve  dx
….  
 part as time cannot be    ve  a = 2x–2n  x 2n 
d
 dx 
44 2
 t2 = =2+2 2 = 2x–2n(2n)x–2n–1
2 = 2n2x–2n–1–2n

Now t0 = t1 + t2 = 2 + 2  2 2 = 4 + 2 2  a = –2n2x–4n–1

ns
  82. Two balls are thrown from a height h.
79. Let the position vector be r1 and r2  Vertical velocities of the balls will be (gt)
 1 2 Horizontal velocities given are,
r1 = vt i – gt j
(v1)z = 4 m/s, (v2)z = 3 m/s

io
2
 1 2  total velocity can be,
r2 = vt (– i ) – gt j 
2 v 1 = 4 î  gt ĵ

at
Since, the position vectors should be perpendicular, 
  v 2 = –3 î  gt ĵ
 r1  r2 = 0
 
1 When the velocities v1 and v2 are
 – v t + g2t4 = 0
2 2
perpendicular,
 2
v = gt
4
1 22
lic  
v1  v 2 = 0
4
gt  (4 î  gt ĵ )  (–3 î  gt ĵ ) = 0
 v=
 12 + g2t2 = 0
ub
2
2v  g2t2 = 12
 t= 12
g  t2 =
Now, 100
2 3
P

2v 
x = 2vt = 2  v  t=
g 10
As, s = vt where,
4v 2
 x = v = relative velocity of balls = 7 m/s
et

g
7 2 3
 s= = 2.425  2.45 m
1 1 10
80. s1 = u1t1 + at12 ; s2 = u2t2 + at 22
2 2 83. The horizontal range is same for two angles of
rg

u1 = 0, a = g = 10 m/s2, t1 = t, t2 = t – 1 projection  and (90 – )


and s1 = s2
u 2 sin 2 
1 2 1 For , h1 = and
 gt = u2(t  1) + g(t  1)2 2g
2 2
Ta

 5t2 = u2(t  1) + 5 (t2  2t + 1) u 2 sin 2 (90  ) u 2 cos 2 


For (90  ), h2 = =
 u2(t  1)  10t + 5 = 0 ….(i) 2g 2g
Final velocity of stone dropped from rest is u4
h1h2 = sin 2  cos 2 
1 4g 2
mv2 = mgh
2 2
1  2u 2 sin  cos  
 v = 2gh = 2  10  45 = 30 m/s =  
16  g 
Now, v = u + at 2
 v = at 1  u 2 sin 2 
=  
30 16  g 
t= =3s
10 R2
Substituting t = 3 in equation (i), h1h2 =
16
 u2(3 – 1) – 10(3) + 5 = 0  R = 4 h1h 2
 u2 = 12.5 m/s
23

MHT-CET Triumph Physics (Hints)



84. Given: t1 = 2t2
p = m  v0 cos  ˆi  (v0 sin   gt)ˆj
 v
v1 v sinθ  a = t    
1
 =2 2 ….   L  r  p =  mgv0t2 cos k̂
g g t = v  2
 
 a 
gx 2
 v1 = 2v2sin ....(i) 88. y = x tan  
2u 2 cos 2 
v 2sin 2θ For equal trajectories for same angle of
h=
2g projection
v12sin 2 (90) g
 h1 = = constant
2g u2
9.8 g
h1 =
v12
....(ii)  2  2
7 2.5

ns
2g
9.8  6.25
v 2 2sin 2 g = = 1.25 m/s2
h2 = ….(iii) 49
2g
Dividing equation (ii) by equation (iii), 89. Time taken to reach highest

io
h1 v 2g 2 u
= 1  2 2 point is t1 =
v 2 sin  g
h2 2g u
Speed on reaching ground
v2

at
= 21 2 = u 2  2gH
v 2 sin 
As, v = u + at
(2 v 2sinθ) 2 H
=
v 2 2sin 2θ
….from (i)  u 2  2gH = u + gt
4 v 2sin 2θ
= 22 2
lic t=
u  u 2  2gH nu

v 2 sin θ g g u 2  2gH

h1 4  2gH = n(n – 2)u2


 =
ub
h2 1 1 2E
91. E= mv2  v2 =
85. Area in which bullet will spread = r 2 2 m
v2 v2 2E
For maximum area, r = Rmax = a= =
g r mr
P

[when  = 45] 92. P = Fv = (ma)  (at)


v 
2 2
v 4 P = ma2t
Maximum area  R 2max =    = 2  a = P / mt
 g 
et

g
1 2 1 2
s = ut + at = at ( u = 0)
86 For H = 5 m, maximum angle of projection a ball 2 2
can have without hitting the ceiling is given by Substituting for a
2Hg 2  5 10 1
rg

sin  = 2 =
2
= 1 P  2
202 s=   t
u 4 2  mt 
1
 sin  =  s  t3/2
2
Ta

1 93. r1 = 4 cm, 2 = 21


  = sin1   = 30
2 r2 = constant
The ball can cover horizontal distance,  r1 12 = r2 22
u sin 2
2
 r1 12 = r1 (21 )2 = r1 = 4 r2
R=
g r1 4
 r2 = = = 1 cm
(20) 2 sin(2  30) 400  3 4 4
= = = 20 3 m
10 10  2
  
87. As angular momentum L  r  p

 1 
where, r = v0 cos  t ˆi   v0 sin  t  gt 2  ˆj
 2 

24

Chapter 03: Motion in a Plane

Hints to Evaluation Test

1. The ant is moving on the paper to the west. The  u2 = 3u1 ….(v)
paper has to be moved in such a way that, in the Also, from (iii)
given frame the ant is moving to the north. First, T= ….(vi)
the motion of the ant to the west (X direction) Substituting (v) and (vi) in (iv),
should be nullified. If the paper is having a  L 
velocity component in the +X direction which is D = 3u1  .   .= 3L
 u1 
equal to the velocity of the ant in the X
From the cannon, second half will fall at
direction, it will appear stationary to the
L + D = 4L
observer.
2L L L 2LV

ns
v 4. T0 = and T =  =
V V  v V  v V2  v2
T V2 1
v1   = 2 2 =
T0 V v 1  (v 2 / V 2 )

io
If v1 is the velocity of the ant, v1 should be equal 5. Velocity is constant when x-t part is straight
and opposite to vcos. The vsin component line. Constant velocity means no acceleration
will be in the +Y-axis or the north. which implies zero force.

at
The resultant velocity of v1 and v will be
6. On earth,
vresultant = . (v1  vcos )2  (vsin )2 . 1 x2
y = x tan   g ….(i)
= v sin  ….( v1 = v cos ) 2 u cos 2 
2

Hence, the displacement is only along the Y-


lic On planet,
axis for the ant. The paper moves in N-E 1 x2
y = x tan   g ….(ii)
direction, in the 1st quadrant. 2 u cos 2 
2

Since trajectories are identical, comparing (i)


ub
2.
dv 
dt

 a .vˆ = 3iˆ  4jˆ   4 3  24
. ˆi  ˆj  
5 5  5
units and (ii)
g g g u2
   7. Interval between the
3. At the highest point of the trajectory, the u 2 u2 g u2
vertical velocity is zero. For the first half of the ball thrown = 3 s
P

shell to fall back into the cannon, vertical If we want minimum three (more than two) balls
velocity should not change during the explosion, to remain in air then time of flight of first ball
i.e., both halves initially should have zero must be greater than 6 s.
et

vertical velocity. Let the mass of the shell be M, 2u


T>6s > 6 s  u > 29.4 m/s.
the horizontal component of its velocity be v, g
initial velocity of the first half be u1, of the
second half be u2, time of falling (common to 7. Interval between the ball thrown = 3 s
rg

both halves) be T and the distance between the If we want minimum three (more than two) balls
explosion point and the point of landing of the to remain in air then time of flight of first ball
second half be D. must be greater than 6 s.
From the law of conservation of momentum, 2u
Ta

T>6s > 6 s  u > 29.4 m/s.


along horizontal, g
M M 8. The two bodies will collide at the highest point
Mv =   u1 +   u2 ….(i)
 2  2 if both cover the same vertical height in the
As first half falls into cannon, same time.
u1 = v ….(ii) v12 sin 2 60 v 22 v 3
  2 = sin 60 =
Hence, horizontal displacement of the first half, 2g 2g v1 2

u1T = L ….(iii) 9. If the Y-axis is divided by mass, we get


Displacement of second half, acceleration
D = u2T ….(iv) F(N)
From (i) and (ii), a (m/s2) =
8kg
u1 u 2 The area under acceleration-time graph is the
u1 = 
2 2 velocity at t, when acceleration changes up to t.
25

MHT-CET Triumph Physics (Hints)


The area of F-t graph (v)2 = 2a.2s ….(ii)
 = Velocity
mass  v = 2v
100 N s
 = 12.5 m/s 14. When the bottle is dropped from the bus, it has
8 kg
the horizontal component of velocity equal to
10. Relative velocity of train A w.r.t. train B is that of the bus and a vertical acceleration due to
vAB = vA  vB = 43  (29) gravity. This is similar to the second half of the
5
projectile problem where at the maximum
= 72 km/h = 72  = 20 m/s height, its vertical velocity is zero and it takes
18
the path of a parabola.
Total distance to be travelled by each train
for completely crossing the other train 15. Y
= 120 + 100 = 220 m
 Time taken by each train to cross the other train

ns
u u cos
220
= = 11s u sin H
20

1 2 X
11. Using s = ut + at O u cos

io
2
Here, u = 0 c
Here, angle of projection,  = = 45
1 25 4
 x1 = 0 + a  52 = a ….(i)

at
2 2 Let u be the velocity of projection of the particle.
1 Kinetic energy of a particle at a point of
x1 + x2 = 0 + a  102 = 50a 1
2 projection O, K = mu2
2
x2 = 50a  x1 = 50a 
25
2
a (Using (i))
lic where, m is the mass of particle.
75 Velocity of a particle at the highest point (i.e., at
 x2 = a ….(ii) maximum height) is ucos.
2
Kinetic energy of a particle at the highest point is
ub
1 225
x1 + x2 + x3 = 0 + a  152 = a 1
2 2 K = m(u cos )2
2
225
x3 = a  x1  x2 1
2 = mu2cos2 
2
P

225 25 75
= a a a 1
2 2 2 = mu2 cos2 45
2
….[Using (i) and (ii)] 2
125 1  1  K
et

 x3 = a = mu2   
2 2  2 2
Hence,
16. Since the initial position coincide with the final
25 75 125
rg

x1 : x2 : x3 = a: a: a=1:3:5 position, net displacement of the cyclist


2 2 2
= zero
2u sin 1 2u sin 2 Average speed of the cyclist
12. t1 = ;t 2 
g g totaldistance travelled
=
Ta

As horizontal range is same, 1 + 2 = 90 total time taken


t1 sin 1 OP  PQ  QO
Hence,  = km/min
t 2 sin 2 10
sin 1 sin 1 1
=  r  (2r)  r
sin(90  1 ) cos 1 = 4
10
= tan1

t1 t 2 2 2
Also, = 2 = 2
sin 1 sin  2 10
13. Initial velocity = 0 4
= km/min
Using, v2  u2 = 2as  v2 = 2as 10
v2 = 2as, for the distance s ….(i) =
7.142
 60 km/hr  42.9 km/hr
For distance 2s, the final velocity 10

26

Chapter 03: Motion in a Plane


u 2 sin 2   (a + b)t = bt  t =
b
t
17. Maximum height, H =
2g (a  b)
u 2 sin 2  But v = at
 180 =  u2 sin2 = 3600  Maximum velocity attained = at
2  10
u sin  = 60 m s1 v=
ab
t m/s
Horizontal velocity = u cos  (a  b)
As per question, u cos  = at 20. Maximum height,
u cos = 4  30 = 120 m s1 u 2 sin 2  u 2 sin 2 
u sin  60 1 H=  gH = ….(i)
   tan = 2g 2
u cos  120 2
Velocity at highest point, vH = u cos ….(ii)
  = tan1 (0.5) Let vx, vy be the horizontal and vertical velocity

ns
18. Let the time taken by two cars to complete the H
of projectile at height . Then,
journey be t1 and t2 and their velocities at the 2
finish be v1 and v2 respectively. vx = u cos
Given that, t1 = t2  t and v1 = v2 + v ….(i) H
and v 2y = u2 sin2   2g  = u2 sin2  gH

io
At start, u1 = u2 = 0 2

1 2 u 2 sin 2 
v 2y = u sin  
s1 = s = a1 t 1 2 2
2
….[Using (i)]
2

at
1 u 2 sin 2 
and s2 = s = a 2 t 22 ….(ii) =
2 2
Hence a1t12 = a 2t 22 = 2s H
 
1/ 2
 Net velocity at height = v2x  v2y
Also, v1 = a1t1 and v2 = a2t2
lic
 v1t1 = a1t12 = 2s and v2t2 = a 2t 22 = 2s ….(iii) As per question;
2

 vx  v2y  = vH  52  v2x  v2y   v2H


2 2 1/ 2
2s 2s
 t1 = and t2 =
v1 v2 5
ub
1 1 2 2 u2 2 
So, t2  t1 = 2s    ….(iv)   u cos 2
  sin  = u2 cos2
5 2 
 v2 v1 
From equations (i) and (iv) ….[Using (ii)]
 sin2  = 3 cos2 
P

1 1
2s    =t sin = 3 cos
 v2 v1 
 tan = 3 = tan 60
v  v 
 2s  1 2  = t   = 60
et

 v1v2 

 v  21. Here  = = 60
2s   =t 3
 v1v2 
rg

Time taken to cover horizontal distance,


v v  v12 v 22 sx
 v =  1 2t = t t =
 2s  (2s) 2 ux
From equation (iii) sx
Ta

=
 v1v 2  u cos
v=  t= (a1a 2 )  t 60
 t 1t 2  = = 4 s.
30  cos60
19. Total time of motion = t Vertical displacement of particle,
Duration of acceleration = t 1 2
sy = uy t  gt
Duration of deceleration = t  t 2
Given u = 0, a = constant acceleration and 1 2
= (u sin ) t 
gt
b = constant deceleration 2
v = 0 + at ….(i) 3 1
 v = at = 30   4  (10) (4) 2
2 2
Also 0 = v  b(t  t) ….(ii) = 23.92 m
v = bt + bt i.e., particle will hit the rod between O and A,
 at = bt + bt but not at its mid point.
27

MHT-CET Triumph Physics (Hints)


22. Using,
mr2 = T and  = 2n
1 T
n=  2Hz
2 mr


23. Distance covered, s =  2r
360o
90
660 =  2r
360
r = 420 m
mv 2 840 10  10
F= = = 200 N
r 420

ns
io
at
lic
P ub
et
rg
Ta

28
Textbook
Chapter No.

04 Laws of Motion

Hints

46. For ball ‘A’,


Classical Thinking
Initial momentum = 0.05  6 = 0.3 kg m s1
7. Force = Mass  Acceleration. If mass and Final momentum = (0.05)(6) = 0.3 kg m s1

ns
acceleration both are doubled, then force will  Change in momentum = 0.3  0.3
become four times. = 0.6 kg m s1
11. p = pi  pf = mv  ( mv) = 2 mv For ball ‘B’,
initial momentum = 0.05  (6)

io

12. |F| = (6)2  (8)2  (10)2 = 200 = 10 2 = 0.3 kg m s1
Also F = ma Final momentum = (0.05)  (6)
F 10 2 = +0.3 kg m s1

at
 m= = = 10 2 kg
a 1  Change in momentum = 0.3  (0.3)
= 0.3 + (0.3)
G m1 m 2
28. F= = 0.6 kg m s1
r2

=
6.67  1011  6  1024  4.8  1024
lic 52. As m2 < m1, v2 > v1
 2.5 10 
10 2
54. m1u1 + m2u2 = m1v1 + m2v2
= 3.1  10 N 18 mu + 2m  0 = m  0 + 2m  v2
ub
u
32. W = F s = Fs cos 180°  v2 =
2
=  Fs = 200  10 = 2000 J
u
0
37. Work done by the net force v 2  v1
e= = 2
= change in kinetic energy of the particle u1  u 2 u0
P

38. Work done is the increase in potential energy in u/2 1


e= = = 0.5
raising the weight 150 N of the ladder through a u 2
height 1 m and raising a weight 40 N through
et

2.5 m 55. m1u1 + m2u2 = (m1 + m2)v


 W = 150  1 + 40  2.5 = 250 J  0.1  5 + 0.2  1.2 = (0.1 + 0.2) v
0.5  0.24
MV 1000  30  v =
rg

42. v= = = 104 cm/s 0.3


m 3 0.74
=
mv 0.01  100 0.3
43. V= = = 0.4 m/s
M 2.5 = 2.467 m/s
Ta

44. MV = m1v1 + m2v2 56. m1u1 + m2u2 = (m1 + m2)v


MV m1u1  m 2 u 2
v2 = [ v1 = 0 m/s]  v =
m2 m1  m 2
30  48 3  m  2m  0
= = 120 m s1 =
12 m  2m
45. Let mass of bullet be m and mass of ice be M. = 1 km h1
According to the conservation of linear
vu
momentum, 57. F = m 
m  300 + M  0 = m  0 + Mv  t 
0.01  300 + 0 = 5v  15  20 
= 0.25   (ball rebounds  v = 0)
3  0.1 
 v = = 0.6 m/s = 60 cm/s
5 F = 87.5 N
29

MHT-CET Triumph Physics (Hints)


ˆi ˆj kˆ 1
   84. Loss of K.E. =  0.02  (250)2 = 625 J
64.  = r F = 3 2 3 2
2 3 4
Loss of K.E. = W = F  0.12
 625 = 0.12 F
= ˆi (8  9)  ˆj( 12  6)  kˆ (9  4) 625
 F=

= ˆi  18jˆ  13kˆ N m  
0.12
F = 5.2  103 N
d
65. r= = 20 cm = 0.2 m, Critical Thinking
2

As the mass of 10 kg has acceleration 12 m/s2,
 
 = r  F = rF sin  2.
therefore it applies 120 N force on mass 20 kg
In this case, motion of wheel is perpendicular to
in a backward direction.

ns
the axis of rotation. Hence,  = 90
 Net forward force on 20 kg mass = 200 – 120
  = rF = 0.2  10  9.8 = 19.6 N m = 80 N
71. 80
20 kg 30 kg  Acceleration = = 4 m/s2
A B 20

io
x 6x 3. Force, F = (M kg s1) (v m s1)
= Mv kg m s2 = Mv N

at
20  x = 30(6  x) dp d
4. F= = (a + bt2) = 2bt  F  t
20x = 180  30x dt dt
 50x = 180 
5. Given that p = p x ˆi  p yˆj = 2cos t ˆi  2sin t ˆj
 x = 3.6 m from 20 kg
lic 
3  dp
m x  F= = 2 sin t î  2 cos t ĵ
i i
0  50  50  5  0  50 dt
75. XC.M = i 1
=    
3
50  50  50
m Here, F p = 0 hence angle between F and p is
ub
i
i 1
90°.
250 5
= = cm 7. (A) is correct as 6th coin has
150 3
3 four coins on its top 10
9
m y
P

which exert a force 8


i i
0  50  0  50  5  50 7
YCM = i 1
3
= 4 mg on it. 6
50  50  50 5
m i (B) th
is correct as 7 coin has 4
3
i 1
three coins, placed over 2
et

th 1
250 5 it. Thus 7 coin exerts a
= = cm th
150 3 force 4 mg on 6 coin (downwards)
(C) is correct, as the reaction of 6th coin on
 2  0    3  0    5  1   7  1 = 12
the 7th coin is 4 mg (upwards)
rg

76. x= m
235 7 17 (D) is wrong as 10th coin, which is the topmost
y=
 2  0    3  1   5  1   7  0  = 8 m coin, experiences a reaction force of mg
235 7 17 (upwards) from all the coins below it.
Ta

77. m1r1 = m2r2 8. The frame of reference which are at rest or in


5 r1 = 35 (0.7  r1) uniform motion are called inertial frames while
 r1 = 0.6125 m frames which are accelerated with respect to each
other are non-inertial frames. Spinning or rotating
81. Because upper part of the body of passenger frames are accelerated frames.
is in motion but lower part is at rest when
B S2
passenger is getting down from a moving 11. W =  dw   F ds   F ds

bus. A S1

83. From the law of conservation of momentum x 4 x 4

3  16 = 6  v 12. W=  F dx =  (0.5x  12) dx


 v = 8 m/s x 0 x 0
x 4 x 4
1
 K.E. =
2
 6  (8)2 = 192 J = 
x 0
0.5x dx +  12
x 0
dx

30

Chapter 04: Laws of Motion


x 4
 x2  p2
= 0.5   + 12  x  xx ==04 E=
2m
 2  x 0
 Kinetic energy of the rifle is less than that of
 42  0 
= 0.5   + 12[4  0] bullet because E  1/m
 2  M
22. Momentum of one piece = 3
W = 4 + 48 = 52 J 4
M
x 4 x 4
Momentum of the other piece = 4
14. W= 
x 0
F dx =  (0.5x  12) dx
x 0
4
9M 2 5M
x 4 x 4  Resultant momentum =  M2 =
=  0.5x dx +  12 dx 16 4
x 0 x 0 The third piece should also have the same
 x2 
x 4 momentum. Let its velocity be v, then

ns
= 0.5   + 12  x  xx == 40 5M M 5
 2  x 0 =  v or v = = 2.5 m/s
4 2 2
 42  0 
= 0.5   + 12[4  0] 23. Let two pieces have equal mass m and third
 2  piece has a mass of 3 m.

io
 W = 4 + 48 = 52 J Y
15. From work - energy theorem,
K.E. = W 18 m/s m

at
But W = Fs 18 m/s
 K.E. = Fs m X
3m 135°
16. From work-energy theorem, kinetic energy of
block at x = 0 to x is;
lic v
x According to law of conservation of linear
K   (4  x 2 ).dx momentum, since the initial momentum of the
0
system was zero, therefore final momentum of
ub
x3 the system must be zero. i.e., the resultant of
 K = 4x 
3 momentum of two pieces must be equal to the
dK momentum of third piece.
For K to be maximum, =0
dx If two particle possess same momentum and
P

or 4 – x2 = 0 angle between them is 90°, then resultant will be


or x =  2 m given by
d 2K p 2 = mv 2 = 18 2 m.
At x = + 2 m, is negative.
et

dx 2 Let the velocity of mass 3m is v. So


i.e., kinetic energy (K) is maximum. 3mv = 18m 2
 Kmax = (4)(2) 
(2) 3
=
16
J = 5.33 J  v = 6 2 m/s and angle 135° from either.
3 3
rg

24. m1u1 + m2u2 = m1v1 + m2v2


17. Work done on the body = K.E. gained by the 0.25  400 + 4.75  0 = 400  0 + 4.75  v2
body 100 = 4.75  v2
Fs cos = 1 J  v2  21 m/s
Ta

1 1
 F cos = = = 2.5 N 25. Mass of each piece (m) = 1 kg.
s 0.4 Initial momentum = 0.
dx Final momentum = p1 + p2 + p3.
18. x = (t  3)2 or v = = 2(t – 3) From the principle of conservation of
dt
at t = 0; v1 = 6 m/s and at t = 6 s, v2 = 6 m/s momentum, we have
p1 + p2 + p3 = 0
 Work done = change in kinetic energy
p3 = (p1 + p2)
1 2 1 2
= mv 2  mv1 = 0 =  (mv1 + mv2) = m(v1 + v2)
2 2
ˆ m s1 =  (2iˆ  3j)
= 1 kg  (2iˆ  3j) ˆ kg m s1
21. Law of conservation of linear momentum is p3 (2i  3j)
correct when no external force acts. When bullet Force F = =
t 105
is fired from a rifle then both should possess
equal momentum but different kinetic energy. =  (2iˆ  3j)
ˆ  105 newton

31

MHT-CET Triumph Physics (Hints)


26. Y m1  m 2 2m 2
31. v1 = u1 + u2
v1 sin1 v1 m1 + m 2 m1  m 2
m1 u1  m1  m 2  u1  2m2u 2
 =
3 m1  m 2

v1 cos1 =
 0.1  m2  u1  2  m2  0
m1 m2 X 0.1  m 2
v2 cos2

u1  0.1  m2  u1
 =
3 0.1  m 2
m1
v2   =
1  0.1  m 2 
v2 sin2
3 0.1  m 2
Y  0.1 + m2 = 0.3 + 3m2

ns
According to law of conservation of momentum,  2m2 = 0.4
m1u1 + m2u2 = m1v1 cos 1 + m2v2 cos 2  m2 = 0.2 kg
In this case, m1 = m2 = m, u2 = 0 and
32. m1u1 + m2u2 = m1v1 + m2v2
1 = 2 = 45
 1  5  2  1.5 = 1  v1 + 2v2

io
 mu1 = mv1 cos + mv2 cos
m  10 = mv1 cos 45 + mv2 cos 45  v1 + 2v2 = 2 ....(i)
v 2  v1
m e=
10 m = (v1 + v2) u1  u 2

at
2
 0.8 (5 + 1.5) = v2  v1
v1 + v2 = 10 2
By conservation of momentum along the  v2  v1 = 5.2 ....(ii)
Solving equation (i) and (ii) simultaneously
m  0 + m  0 = mv1 sin 45 – mv2 sin 45
lic
direction perpendicular to the original line.
 v1 = 2.8 m/s , v2 = +2.4 m/s
m(v1  v 2 ) 33. During collision of ball with the wall, horizontal
0 =
2 momentum changes (vertical momentum
remains constant)
ub
 v1 = v2  2 v1 = 10 2
Change in horizontal momentum
 v1 = 5 2 m/s  v2 = 5 2 m/s  F=
Time of contact
27. This is a case of a perfectly inelastic collision in 2pcos 
which linear momentum is conserved but kinetic =
0.1
P

energy is not conserved. 2mv cos 


=
m1  m 2 2m 2 0.1 60°
29. v1 = u1 + u2 2  0.1  10  cos 60 p = mv
m1  m 2 m1  m 2 30°
et

=
 m1  m 2  u1 0.1
 0= ( u2 = 0) = 10 N
m1  m 2
 m1  m2 = 0 34. Impulse = change in momentum = 2 mv
rg

 m1 = m2 = 2  0.06  4 = 0.48 kg m/s


m1 35. Impulse = Ft = change in momentum
 =1
m2 = mv – (mv) = 2 mv = 2  0.01  5 = 0.1
Ta

30. m1u1 + m2u2 = m1v1 + m2v2 0.1


 F= = 10 N
2  4  1  2 = 2v1 + v2 0.01
 2v1 + v2 = 6 36. If a large force F acts for a short time dt the
 v2 = 6  2v1 impulse imparted J is
1 1 dp
Also [m1u12 + m2u22] = [m1v12 + m2v22] J = F dt = dt
2 2 dt
 2  (4)2 + 1  (2)2 = 2(v12) + (v22) J = dp = change in momentum
 32 + 4 = 2v12 + v22
37. Impulse = change in linear momentum
 36 = 2v12 + v22
= 0.5  20  0.5  (10)
 2v12 + (6  2v1)2 = 36
= 10 + 5 = 15 N s
 v1 = 0 or v1 = 4
When v1 = 0, v2 = 6 and v1 = 4, v2 = 2 38. =
dL
, if  = 0 then L = constant
 v1 = 0, v2 = 6 m/s dt

32

Chapter 04: Laws of Motion

39.

 = r F
 
50  10 + 100  40 + 100  60
   = (50 + 100 + 100) x
Vector  is perpendicular to both r and F . 500  4000  6000
   
x = = 42 cm
 r   = 0 and F    0 250

 45. Depends on the distribution of mass in the body.


40. F  Fkˆ Y
46. Centre of mass always lies towards heavier mass.
 

r  ˆi  ˆj
Fkˆ 47. m1r1 = m2r2
  
  
 = r  F = ˆi  ˆj Fkˆ 
r1 m 2

O X r2 m1
= F  i  k   F  j  k 
ˆ ˆ ˆ ˆ
1
 r
= F  ˆj  F  ˆi   F ˆj  Fiˆ  F  ˆi  ˆj m

ns
48. The position of centre of mass remains
   
  
41. r  r 1  r 2  ˆi  2ˆj  3kˆ  3iˆ  2ˆj  3kˆ unaffected because breaking of mass into two
parts is due to internal forces.
= 2iˆ  4ˆj  6kˆ

io
   49. Centre of mass lies always on the line that joins
Now   r  F the two particles.
  
= 2iˆ  4ˆj  6kˆ  4iˆ  5jˆ  3kˆ  For the combination cd and ab this line does not

at
pass through the origin.
ˆi ˆj kˆ

For combination bd, initially it passes through
  2 4 6 = ˆi(12  30)  ˆj(6  24)  k(10
ˆ  16) the origin but later on its moves towards
negative X-axis.
4 5 3
lic But for combination ac it will always pass
= (42iˆ  30ˆj  6k)
ˆ Nm
through origin. So we can say that centre of
mass of this combination will remain at origin.
42. Couple consists of two equal and opposite
ub
forces which causes pure rotational motion.  
 m r  m2 r 2
50. r cm = 1 1
43. RA m1  m 2
RB
1(i  2ˆj  k)
ˆ  3( 3iˆ  2ˆj  k)
ˆ
0.5 m G =
A B 1 3
P

C 1m 
W1 W  r cm = 2iˆ  ˆj  kˆ
50 kg wt 50 kg wt
m1x1  m 2 x 2  m 3 x 3  m 4 x 4
et

For equilibrium, 51. X=


m1  m 2  m3  m 4
Considering moments about point A,
0  40 x 4 40 x 4
RA  0 – W1  AC – W  AG + RB  AB X=  3
100 100
RA  0  50  0.5  50  1 + RB  2 = 0
rg

300
 2RB = 75 x4 = = 7.5
40
 RB = 37.5 kg wt
Similarly y4 = 7.5 and z4 = 7.5.
RA = 100  37.5 = 62.5 kg wt
Ta

44. 53. No external force is acting on the system so


R1 R2
x cm C.M. will not shift.
R 80 cm 54. Assuming point A as origin, let AE be along Y-
A G B S
axis and AF be along X-axis. Due to uniform
10 cm 20 cm
density, let mass of AF be m and mass of AE be
50 g wt 2 m.
The centre of mass of AE is at a distance of l
100 g wt 100 g wt
from A and the centre of mass of AF is at a
Let the knife – edge be balanced at x cm from distance of l/2 from A.
point R. For equilibrium, considering moments Hence distance of centre of mass of the metal
about point R, strip from A is
W1  RA + W  RG + W2  RB m  (l / 2)  2m(0)
Xc.m. = = l /6
= (W1 + W + W2)  x m  2m

33

MHT-CET Triumph Physics (Hints)


m  (0)  2m(l ) 2l 60. Centre of mass is closer to massive part of the
Yc.m. = =
m  2m 3 body therefore the bottom piece of bat has larger
Thus, the coordinates of centre of mass of strip mass.
= (l /6, 2l /3) 62. If the man starts walking on the trolley in the
In the given figure, point ‘c’ is the only point forward direction then whole system will move
having approximately same coordinates. in backward direction with same momentum.
m1x1  m 2 x 2  ....
55. xCM = 1m/s
m1  m 2  ....
ml  2m.2l  3m.3l  .... v
=
m  2m  3m  ....
l n(n  1)(2n  1)
ml (1  4  9  ....) 6
= = Momentum of man in forward direction

ns
m(1  2  3  ....) n(n  1)
= Momentum of system (man + trolley) in
2
backward direction
l (2n  1)
=  80  1 = (80 + 320)  v
3
 v = 0.2 m/s

io
m1v1  m 2 v 2 10  14  4  0 So the velocity of man w.r.t. ground
56. vCM = = = 10 m s1
m1  m 2 4  10 1.0  0.2 = 0.8 m/s
 Displacement of man w.r.t. ground,

at
 
 m r  m1 r 2 y = 0.8  4 = 3.2 m
57. r1 = 1 1
m1  m 2 63. Gravitational field is a conservative field.
 35.5  1.27 H Cl Therefore work done in moving a particle from
r =
1  35.5

m1
lic m2
x A to B is independent of path chosen.
 35.5
r =  1.27 ˆi 64.
36.5 1.27 Å Y
= 1.24 î
ub
 
 m1 v 1  m 2 v 2 2 3  3 2
v m/s v cos
58. v cm = = v sin Hmax
m1  m 2 23
12
P

= = 2.4 m/s 
5 X
v cos
59. According to problem
Let the mass of shell be m. At the highest point it
m1 = 6 m, m2 = m3 = m4 = m5 = m
et

  
has only horizontal component of velocity.
r 1  0 ˆi  0ˆj , r 2  a ˆi  a ˆj , r 3  a ˆi  a ˆj ; Hence its momentum at that point = mv cos
  It breaks into two equal mass. One piece traces
r 4  0 ˆi  0 ˆj , r 5  0 ˆi  a ˆj
its path with speed v cos.
rg

y Let speed of other piece just after explosion be v


m1 then,
m2 m3 m m
Final momentum = vcos   v
Ta

x 2 2
m4 By the principle of conservation of momentum,
m5 m m
mv cos = v cos + v
Position vector of centre of mass 2 2
      1  m
 m r + m 2 r 2 + m3 r 3 + m 4 r 4  m5 r 5 1   mv cos  = v
r cm  1 1  2 2
m1  m 2  m3  m 4  m5
v = 3 v cos

0  m(aiˆ  aj)
ˆ  m(aiˆ  aj)
ˆ  0  m(aj)
ˆ
r cm = 65. From the principle of momentum conservation,
10m
mgvg = mbvb (considering magnitudes)
a ˆ
= 0iˆ  j 0.05  400
10  vg = = 4 m/s ( mb = 50 g = 0.05 kg)
5
 a  The gun fires 30 bullets in 1 minute i.e., in
So, the coordinate of centre of mass =  0, .
 10  60 s. This means 1 bullet is fired every 2 s.
34

Chapter 04: Laws of Motion


From Newton’s second law, 1 2 6
For 200 g weight, K.E. = mv = 10 J
p p 2
F= 2 1
t  2  106 
1/ 2

Where p2, p1 is final and intial momentum of  v=   = 105/2


gun respectively.  0.2 
mg vg  0 5 4 Hence, its momentum = 0.2  105/2 kg m/s
 F= = = 10 N ….(iii)
2 2
3
For a weight falling from h = 1 km = 10 m
66. Let ‘m0’ be the initial mass of rocket. Its
(P.E)max = (K.E)max
 dm 
ejection speed of gases   = 16 kg/s 1 2
 dt  mgh = mv
2
Hence after t = 1 min = 60 s, its mass will be
 dm 
 v = 2gh = 2  9.8  103 = 140 m/s
m = m0    t = 6000  (16)  60

ns
 dt  Hence, its momentum = 0.2  140
 m = 5040 kg = 28 kg m/s ….(iv)
At this time instant thrust on the rocket is, Comparing the values, momentum of a 200 g
dm weight after falling through 1 km has maximum

io
F=u value.
dt
where u is constant relative speed. 71. If man slides down with some acceleration, then
dm its apparent weight decreases. For critical

at
ma = u
dt condition, rope can bear only 2/3 of his weight.
(u dm / dt) If a is the minimum acceleration then,
a=
m Tension in the rope = m (g  a) = Breaking strength
=
11103 16
= 34.92 m/s2
lic 
2
m (g  a) =
3
mg
5040
 35 m/s2  a=g
2g g
=
67. To hold the gun stable, rate of change of 3 3
ub
momentum of the gun should not exceed 72. Gas will come out with sufficient speed in
maximum exerted force. forward direction, so reaction of this forward force
p will change the reading of the spring balance.
Hence F =
t
P

For t = 1 s, F = p  144 N s 73. According to law of conservation of momentum


From the principle of conservation of momentum, the third piece has momentum
Momentum of gun = momentum of bullet = 1   (3 î + 4 ĵ ) kg m s1
p = 40  103  12  102
et

y
= 48 kg m/s.
So, number of bullets that can be fired per second,
p 144 1  4 ĵ
= =3
rg

p 48 F

68. As v = 5t ˆi  2t ˆj 1  (3 î + 4 ĵ ) 1  3 î

Ta

 a = a x ˆi  a yˆj = 5iˆ  2jˆ



F = max î + m(g + ay) ĵ W

 | F | = m a 2x  (g  a y ) 2 = 26 N
Impulse = Average force  time

  m v Impulse
69. F t = m v  F =  Average force =
t time
By doing so time of change in momentum increases Changein momentum
=
and impulsive force on knees decreases. time
70. Momentum of vehicle = 100  0.02 = 2 kg m/s
=
 ˆ ˆ
 3i  4 j 
….(i) 10 4

Momentum of weight = 4  103  103  102


= 4  102 kg m/s ….(ii)  
=  3iˆ  4ˆj  104 N.

35

MHT-CET Triumph Physics (Hints)


74. 7. Given: m = 5 g, v = 4 cm/s, t = 2.5 s
we know,
RA RB v
0.5 m
a=
t
A B and F = ma
0.3 m C G D
4
 a= cm/s2
0.8 m 2.5
4
W1 = 4 kg wt W2 = 6 kg wt F=5 = 8 dyne
2.5
8.
W = 10 kg wt F = 14 N
A B C

ns
1m
For translational equilibrium Acceleration of system of blocks is
 RA + RB  W  W1  W2 = 0 F 14
a=  = 2 m/s2
 RA + RB = W + W1 + W2 mA  mB  mC 4  2  1

io
= (10 + 4 + 6) kg wt Let contact force between A and B be f then,
 RA + RB = 20 kg wt
For rotational equilibrium, considering moments a = 2 m/s2

at
about A, F = 14 N A f
RA  0  W1  AC  W  AG  W2  AD
+ RB  AB = 0
14 – f = mA  a
4  0.3  10  0.5  6  0.8 + RB  1 = 0
 RB = 11 kg wt
lic 

14  f = 4  8
f = 14 – 8 = 6 N
 RA = 9 kg wt
= 9  9.8 N = 88.2 N 9. anet =
F
m1 + m 2 + m3
ub
Competitive Thinking anet =
24
= 4 m/s2
6
1. Since all three blocks are moving up with a Fnet = m anet
constant speed v, acceleration a is zero.
Fnet = 2  4
P

F=0 Fnet = 8 N
 Net force is zero.
10. From the figure, tension
2. Given three forces are acting along the three between masses T3
et

sides of triangle in same order, so 2m and 3m is T2.


   
 Fnet = F1 + F2 + F3  
We know that, T1
 F3 F2
 Fnet = 0  
rg

2 m1m3 T1
T2 =  g m1 m

 m1  m 2  m3  m2
 a = 0,  2m
Velocity will remain constant F1
 T2 =
 2m  (3m) g = mg
Ta

T2
3. Initial thrust must be m[g + a] 6m
= 3.5  104 (10 + 10) = 7  105 N 11. First case: 3m m3
4. For a freely falling lift, (a = g) T1
Apparent weight = m(g  a)
= m (g – g) a = 4.9 m/s2
=0
5. F = m  g = 0.05  9.8 = 0.49 N. As the weight mg
of ball acts downwards, the net force will act T1 – mg = ma
vertically downward. mg  g
T1 – mg = ….  a  4.9 ms 2  
6. F = m(g – a) 2  2 
= 60(9.8 – 1.8) 3mg
= 480 N T1 = ....(i)
2

36

Chapter 04: Laws of Motion


Second case: 14.
T2
T2 P

a = 4.9 m/s2 T1
T1
a m1
mg
A m2 a
mg – T2 = ma B
mg  g  m  m1 
mg – T2 = ….  a  4.9 ms 2   a=  2 g
2  2  m1  m2 
mg m1 = 2 kg, m2 = 3 kg
T2 = ....(ii)

ns
2
 asystem =
 m2  m1  g =  3  2 10 = 2 m/s
Dividing equation (i) by (ii), m1  m 2 3 2
T1 3mg 2 3
   m1 a 1 + m 2 a 2 2   a   3 a 
T2 2 mg 1  acm = =
m1 + m 2 3 2

io
12. Tension in spring before cutting the strip 2  2   3 2 
=
4 mg 5
T  acm = 0.4 m/s2

at
 Velocity of the centre of mass of the system
A B
after 1.5s,
mg
v = at
3 mg T
lic
 v = 0.4  1.5
 v = 0.6 m/s in downward direction
 T = mg 15. If on a wedge of rough surface inclined at angle
, time of descent of a block is t, then the time
ub
After cutting the strip
of descent of the block on the same wedge of
4 mg t
smooth surface becomes , and the coefficient
n
B
A of friction is given as,
P

 = tan  1 
1 
3 mg mg 
 n2 
Here,  = 45
et

Acceleration in brick A 1
 =1–
4mg  3mg g p2
aA = =
3m 3 N
16.
rg

Acceleration in block B
mg F W F1 = 2N
aB = =g
m
F2 = 20 N F
Ta

F
13. For mass m1, a1 = 6 =
m1 Fmax = mg = 0.5  2  10
F  Fmax = 10 N
 m1 = ....(i)
6 20   2 
acceleration of the system a = = 3 m/s
F 42
For mass m2, a2 = 3 =
m2 Force of friction opposing motion of 2 kg block
F = ma + force acting in direction of friction.
 m2 = ....(ii) = (2  3) + 2 N = 8N
3
 a=
F 17. We know that, v2 = u2 + 2as ….(i)
m1  m 2 Now, initial velocity at midpoint
From equations (i) and (ii), L
u= 2g sinθ
F 2
a= = 2 m/s2
F/6F/3 and final velocity for the lower half = v = 0
37

MHT-CET Triumph Physics (Hints)


At lower half, acceleration = g sin   g cos  

L
22. (
F = 3iˆ + ˆj )
and s = 
2
s =  r2  r1  = éê 2iˆ + 3 ˆj- 2kˆ ùú
 
L/2
  ë û

We know,
L/2  
 W = F  s = (3iˆ + ˆj)  éê 2iˆ + 3jˆ - 2kˆ ùú = 6 + 3 + 0
ë û
 From equation (i),  W=9J
2 L L
0  2g sin  = 2[g sin   g cos ] 
2 2 23. The net force acting on particle,
L   
  2g sin  = gL sin   gL cos  F  F1  F2 = 5iˆ  3jˆ  kˆ

ns
2
 2gL sin  = gL cos  Displacement,
  = 2 tan    
s  r2  r1 = 20iˆ 15jˆ  7kˆ cm
Alternate Method

io
 
According to work energy theorem,  W = F s = (–100 + 45 + 7)  10–2 = –0.48 J
W = K = 0
 Initial and final speeds are zero 24. Work done = Area under F-x graph

at
 work done by friction + work done by gravity = 0 1
L  W= × (3 + 2) × (3 – 2) + 2 × 2
 –mg cos  + mg sin  L = 0 2
2


cos  = sin 
lic  W = 2.5 + 4
2  W = 6.5 J
2 sin  From work energy theorem,
= = 2 tan 
cos 
ub
W = Δ K.E.
18. F = ma = 0 N  Δ K. E. = 6.5 J
μmg
 ma =
2 25. From work-energy theorem
P

μg 0.6 10 K.E. = work = area under F-x graph


 a= =
2 2
From x = 0 to x = 8 m
 a = 3m/s2
1
et

19. a1 a2 mv2 = (5  20) + (3  10)


2
m1 F F
m2 1
r  mv2 = 100 + 30
rg

2
Gm1m 2 F Gm
F=  a1 = = 22  v2 = 520
r2 m1 r
 a1  m2  v= 520 = 22.8  23 m/s
Ta

 Similarly, from x = 0 to x = 12 m
20. Given: F  4iˆ  3jˆ
1
    mv2 = 100 + 30  47.5 + 20
  
s  r2  r1 = 4jˆ  3kˆ  2iˆ  5jˆ = 2iˆ  ˆj  3kˆ 2

= F s =  4iˆ  3jˆ    2iˆ  ˆj  3kˆ  = 8  3 + 0


 
 W 1
 mv2 = 102.5
2
=5J
2  102.5

 v2 =
21. Given: F  ˆi  5kˆ , 0.5
   
  
s  r 2  r 1  3iˆ  4ˆj  5kˆ  ˆi  3kˆ  v= 410  20.6 m/s

= 4iˆ  4jˆ  2kˆ


 
26. W = K.Ef  K.Ei
W = F  s = 1  (– 4) + 5  2 = – 4 + 10 = 6 J  K.Ef = W + K.Ei

38

Chapter 04: Laws of Motion


  1 2 t2
 K.Ef =  F.d s + 2
mu 31. s=
4
30
1 2t t
 ds = dt = dt
= 0.1 20 xdx + 2 10 10
2
4 2
d 2s d  ds  d t 1
= –25 + 500 = 475 J a= =  =  =
dt 2 dt  dt  dt  2  2
27. From graph, work done is area under the curve 1
 F = ma = 6  = 3 N
1  1  2
 W =   3  20   (3  20)    3  20 
2  2  2 2
t
Now, W =  Fds   3 dt
= 30 + 60 + 30 0 0 2
= 120 J

ns
2
3  t2  3
=   = (2)  (0)  = 3 J
2 2
From work energy theorem,
2  2 0 4 
1 2
mv = W = 120
2 

io
32. dW = F  dx
1
  2.4  v2 = 120  
2 x y
=K ˆi  ˆj  dx ˆi  dy ˆj
  x 2  y2   x 2  y2    
3/ 2 3/ 2
v2 = 100

at
 
 v = 10 m/s  
xdx  ydy 
= K ….(i)
28. F = 6t = ma   x 2  y 2 3/ 2 
m = 1 kg
lic  
Let x2 + y2 = r2
 a = 6t
 2xdx + 2ydy = 2rdr
dv
 = 6t 
ub
xdx + ydy = rdr
dt
Substituting in equation (i),
Integrating we get,
 rdr  K
v 1
dw = K  3  = 2 dr
 dv   6t dt r  r
P

0 0
r2 r
 K 
2
K
v =  3t 
2 1
0
= 3 m/s Integrating, W = r r 2 dr   r  r
1 1

From work energy theorem,


et

r1 = x12  y12 = a, r2 = x 22  y 22 = a
1 1
W = m (v2 – u2) = (1) (9 – 0) = 4.5 J  W=0
2 2
rg

K
33. Work done by gravitation force is given by (Wg)
29. F= Wg = mgh = 103  10  103 = 10 J
v
 W = Fs cos According to work energy theorem
Ta

K Wg + Wres = KE
 W= s (  = 0)
v 1
10 + Wres =  103  50  50
s 2
 v=
t 5
10 + Wres =
t 4
 W=K s
s Wres = 8.75 J
 W = Kt 34. From law of conservation of momentum,
  
30. Displacement is in x direction and force is in p1  p 2  p 3  0
y –direction,  

 Force is perpendicular to displacement, hence Let p 1 and p 2 go off at right angles to each
work done will be zero. other.
39

MHT-CET Triumph Physics (Hints)



h = 0.1 m
 | p3 | = p12  p 22

 m3  4 = (1 12)2  (2  8)2 = 122  162 = 20


 0.016 vb = 4.016   2  9.8  0.1 
20
 vb = 351.4 m/s
 m3 = = 5 kg
4 39. By law of conservation of momentum,
  m1u1 + m2u2 = m1v1 + m2v2
35. Let, P A   3Piˆ and P B  2Pjˆ here, m1 = mass of bullet = 10 g = 0.01 kg
According to law of conservation of m2 = mass of block = 2 kg
momentum, u1 = initial velocity of bullet = 400 ms1
  
PA  PB  PC  0 u2 = initial velocity of block = 0
 v1 = final velocity of bullet
 3Piˆ  2Pjˆ  P C = 0
v2 = final velocity of block = 2gh

ns

 P C  3Piˆ  2Pjˆ  (0.01)  400 + 0 = 0.01 v1 + 2  2  9.8  0.1
 |PC| = 9P  4P  13 P
2 2
4 = 0.01v1 + 2  1.96
4  2  1.4
36. As the bullet explodes at highest point of  v1 = = 120 m/s

io
0.01
trajectory, it only has horizontal velocity.
1 p2 36
vH = vcos 60° = 30  =15 m/s 40. K.E. = = = 4.5 J
2 2m 2 4

at
According to law of conservation of
momentum, momentum before and after 1
41. (K.E.)1 = m1v12
explosion must be same. 2
(m1 + m2 ) vH = m1v1 + m2v2
But, m1 = m and m2 = 3m (given)
lic =
1 m12 v12
2 m1
 4m  15 = m  0 + 3m v2
1 p12
15  4 =
 v2 = = 20 m/s 2 m1
ub
3
1 p 22
37. By law of conservation of momentum, (K.E.)2 =
  
2 m2
| p1  p 2 |  | p 3 |  (K.E.)1 = (K.E.)2 ….(given)

P

 p  p  2p1p 2 cos  = | p 3 |
2 2 p12 p2
1 2   2
2m1 2m 2
as p1 = p2 = p and p1  p2, θ = 90°.

p1 m1
 2p2 = | p 3 |  
et


 p2 m2
p1
 2 p = | p3 |   
 p1  p 2  42. From principle of conservation of momentum,
as v1 = v2 = v = 30 m/s  
Final momentum = Initial momentum

rg

30 2 m = m3v3 
m2v2
also, m + m + 3m = M p2  m1v1  m2v2 = 0  v1 =
m1
M
 m= 1
5 2
 K.E.1 E1 2 m1v1 m1v12
Ta

3M 
  
m3 = 3m =
5
p3  K.E.2 E 2 1 m 2 v 22 m2 v22
2
M 3M
 30 2 = v3 Substituting for v1
5 5
m 22 v 22
 v3 = 10 2 m/s m1
E1 m12 m
 = 2
38. By conservation of linear momentum, E2 m 2 v 22 m1
initial momentum final momentum of
= 43. From conservation of linear momentum,
of bullet system
MV = m1v1 + m2v2
 mvb = (M + m) vsys
As bomb is at rest initially, its initial momentum
here, m = mass of bullet = 0.016 kg
will be zero.
M = mass of block = 4 kg
 m1v1 + m2v2 = 0
vsys = velocity of system = 2gh
 20 + 8v2 = 0
40

Chapter 04: Laws of Motion

 v2 = 
20 5  By principle of linear momentum,
=  m/s
8 2 Pinitial = Pfinal
 Kinetic energy of the 8 kg piece is, 

m 
m v i   v1 
5m  
v2 
1 1  25   6 6 
K.E. = m 2 v 22 =  8    = 25 J
2 2  4   
 v 5 
 vi   1  v 2 
44. Let the point B represents the position of bat.  6 6 
 
The ball strikes the bat with velocity v along the
path AB and gets deflected with same velocity  20iˆ  25jˆ  12kˆ 
100iˆ  35jˆ  8kˆ   v 

5
v2
i
along BC, such that  ABC =  6 6

A  120iˆ  150ˆj  72kˆ  100iˆ  35ˆj  8kˆ  5 v 2

 20iˆ  115ˆj  80kˆ  5 v 2

ns
B
N 
 20iˆ  115jˆ  80kˆ
 v2 
5

C  v 2  4iˆ  23jˆ  16kˆ

io

Initial momentum of the ball = mv cos   49. When two bodies with same mass collide
2   elastically, their velocities get interchanged.
(along NB)

at
50. For collision, the relative velocity of one

Final momentum of the ball = mv cos   particle should be directed towards the relative
2   position of other particle.
(along BN) 

Hence, Impulse = change in momentum


lic Let v R be direction of relative velocity of B

     w.r.t. A and r R be direction of relative position
= mv cos      mvcos   
2
   2   of A w.r.t. B.
   

ub
= 2mv cos  
 v 2  v1  r1  r2
2
 vR = and r R =
     
v 2  v1 r1  r2
45. Change in linear momentum = 18 kg ms–1    
 m(v2 – v1) = 18   r1  r2 v 2  v1
 vR = r R  
P

18    
 m= = 1 kg r1  r2 v 2  v1
20  2

46. Impulse = change in momentum 51. Before collision:


et

 I = pf  pi 1 1
(K.E.)1 = m1u12  m 2 u 22
Resultant of two vectors having same magnitude 2 2
and separated by angle , After collision:
rg

 1 1
R = 2A cos (K.E.)2 = m1v12  m 2 v 22
2 2 2
here,  = 60 + 60 = 120 Total energy being conserved in collision,
1 1 1 1
Ta

I = 2p cos 
120  m1u12  m 2 u 22    m1v12  m 2 v 22
  = 2mV cos (60) = mV
 2  2 2 2 2

47. Let ‘m’ be the mass of the object 52. Coefficient of restitution is a ratio of same
After breaking, physical quantity viz., velocity. Hence, it has no
Ratio of masses = 1:5 dimensions.
m1 = k, m2 = 5k ….(i) 53. Coefficient of restitution:
 m1 + m2 = m v 2  v1
e=
 6k = m u1  u 2
m Given: u1 = v, u2 = 0
 k= ….(ii)
6 v 2  v1 v v
Substituting (ii) in (i)  e= = 2 1 .…(i)
v v v
m 5m By law of conservation of momentum,
 m1 = and m2 =
6 6 mu1 + mu2 = mv1 + mv2
41

MHT-CET Triumph Physics (Hints)


mv = mv1 + mv2 2h
v = v 1 + v2 =
g
1  2e  2e2  ....
v1 v 2
1=  2h 1  e 
v v =
v1 v g 1  e 
 =1– 2 .…(ii)
v v 2  0.4  1  e 
From equation (i) and (ii),  10 =  
10  1  e 
v2  v2 
e=  1   25 2 1 17
v  v  e= 
25 2 1 18
v2 v
 e= –1+ 2
v v 57. Let v1 and v2 be their respective velocities after
2v 2 collision.
e= 1

ns
v Applying the law of conservation of linear
2v 2 momentum,
 =e+1  m1u1 + m2u2 = m1v1 + m2v2
v
v2 e 1  m  2 + 2m  0 = m  v1 + 2m  v2
 =

io
v 2 2m = mv1 + 2mv2
2 = (v1 + 2v2) …(i)
54. Given: m1 = m, m2 = 4m, u1 = v, u2 = 0, v1 = 0 By definition of coefficient of restitution,
According to law of conservation of v 2  v1

at
momentum, e=
u1  u 2
m1u1 + m2u2 = m1v1 + m2v2
e(u1  u2) = v2 v1
mv + 4m  0 = m  0 + 4mv2
0.5(2  0) = v2 v1 …(ii)
 v2 =
v
4
lic 1 = v2  v1
Solving equations (i) and (ii),
v
0 v1 = 0 m/s, v2 = 1 m/s
v 2  v1 1
e= = 4 = = 0.25
u1  u 2 v0 58. Total mechanical energy of ball,
ub
4
1
55. T= mv2 + mgh
2
h Total energy after the collision,
e2h e4h 2 1 2 
P

 mv  mgh 
3 2 
Total distance = h + 2e2h + 2e4h …. The ball rebounds back to the same height after
= h + 2e2h (1 + e2 +….) collision,
et

Using binomial expansion, 2 1 2 


2 4 1   mv  mgh   mgh
(1 + e +e + …) = 3 2 
1 e2  2 1 2 
rg

  v  gh   gh ,
 1 
2 3 2 
 Total distance = h + 2e h  2 
1 e  1 2 2
v  gh  gh ,
2e 2 h 3 3
Ta

= h+
1  e2 v2 2gh
  gh 
h  e 2 h  2e 2 h 3 3
=
1 e2  v 2 gh

3 3
h(1  e2 )
= v= gh = 40  10 = 20 m/s
(1  e2 )
59. Velocity v after rebound can be given as,
h1
56. As e = v = 2gh = 2  10  20 = 20 m/s
h0
 kinetic energy just after collision is,
 h1 = e2h0
1 1 400m
For n number of bouncing, hn = e2nh K= mv2 = m  (20)2 = = 200m
2 2 2
2h 2he 2 2he 4 As the ball loses 50% of energy in collision,
 t = 2 2  ......
g g g its initial energy would be 400m

42

Chapter 04: Laws of Motion


By conservation of energy, 63. m
2v
1 2
mv 0 + mgh = 400m v
2 v
1 2 =
 mv 0 + m  10  20 = 400m 45
2 45
 v 02 + 400 = 800
2m 3m
 v0 = 20 m/s
Before collision After collision
60. In elastic collision
(K.E)before collision = (K.E)After collision Collision being perfectly inelastic,
speed of second body after collision v2 can be m(2v) cos 45 + 2m(v) cos 45 = (m + 2m)v
found as 1 1
2  2mv + 2mv = 3mv
1 2 1 v 1 2 2

ns
mv  0  m    m(v 2 ) 2
2 2 3 2 2 2 mv 2 2
 = v  v = v
v2 8v  3m 3
 v2 =  v 22  = v 22
9 9 Loss in K.E. = total initial K.E.  total final K.E.

io
2
2 2 1 1 1  2v 2 
 v2 = v = m(2v) 2  (2m)v 2   (3m)  
3 2
2 2  3 
61. According to law of conservation of momentum, 3 8  5

at
mv0 = mv1 + mv2 = 2mv2 + mv2   m  v 2   = mv 2
 2  9  3
 v0 = v1 + v2 ....(i)
1 2 5 2
mv
Initial KE, (K.E.)i = mv 0 3
1
2
1
lic Percentage loss in K.E. =
2mv 2  mv 2
100

Final KE, (K.E.)f = mv12  mv 22 5


2 2 = 100 = 55.56%
Given that, 9
 56%
ub
3
(K.E.)f = 0.5 (K.E.)i + (K.E.)i = (K.E.)i
2 64. Say mass of 2 kg is at rest initially, then
1 3 1 
 m  v12  v 22    mv02  3  15 + 2  0 = 3v1 + 2v2
2 2 2   45 = 3v1 + 2v2 ….(i)
P

3 5 1
 2
v + v = v 02
1
2
2 ....(ii)  e= 
2 15 3
On squaring equation (i) and subtracting v 2  v1 1
equation (ii) from it, we get,  
et

15  0 3
3
v  v 22  2v1 v 2    v12  v 22   v02  v 02
2
1
2
 15 = 3v2  3v1 ….(ii)
Solving equations (i) and (ii),
1 2
 – 2v1v2 = v0 v1 = 7 , v2 = 12
rg

2
 Loss of kinetic energy
 – 4v1v2 = v02 ....(iii) 1 1 1
=  3  152   3  72   2  122
Now, (v1  v2)2 = (v1 + v2)2  4v1v2 2 2 2
Ta

 (v1  v2)2 = v02  v02  2v02 = 337.5  73.5  144 = 120 J


....[using equations (i) and (iii)]
1
 vrel = |v1  v2| = v0 2 65. Here m1 = 4m, m2 = 2m  m1 = m2
2
62. In case of inelastic collision The fractional decrease in the kinetic energy
m1m 2 after elastic collision:
K.E. 
2  m1  m 2 
1  e2   u1  u 2 2 For m2 = nm1,
1 0.5   1  Ki  Kf 4n

1   6   9  
2
=
2 1  0.5   9   1  n 
2
Ki

18 1
= 4 
  225 Ki  Kf 2 = 2 = 8
69 =
8  225 Ki  1
2
9 9
= = 33.33 J  1    
54  2  
4

43

MHT-CET Triumph Physics (Hints)


66. Change in momentum = Area below the F  For 1st sphere = x1 = 0
versus t graph in that interval 2nd sphere = x2 = AB
1  3rd sphere = x3 = AC
=   2  6   (2  3)  (4  3)
2  m1x1  m 2 x 2  m3 x 3
 x=
= 6 – 6 + 12 = 12 N s m1  m 2  m3
0  m(AB)  m(AC) AB  AC
67. At t = 4s, v = 0, u = 3/4 m/s = =
3m 3
 Impulse = m(v – u)
= 2(0 – 3/4) = – 1.5 kg m/s 73.
d C d
68. R1 R2
m1 m2
x1 x2

ns
d m1x1 = m2x2 ....(i)
A CM B m1(x1 – d) = m2(x2 – d) ....(ii)
x d-x  m1x1 – m1d = m2x2 – m2d
m1d = m2d ....[From (i)]

io
W m
For equilibrium,  d = 1 d
m2
N1d = W(d  x)

at
W(d  x) 74. The (x, y, z) co-ordinates of masses 1 g, 2 g,
 N1 =
d 3 g and 4 g are
(x1 = 0, y1 = 0, z1= 0), (x2 = 0, y2 = 0, z2 = 0)
69. The rule hanging from a peg is at equilibrium,
(x3 = 0, y3 = 0, z3 = 0),
hence, the principle of moments applies here.
lic (x4 = , y4 = 2, z4 = 3)
L/2 m1x1  m 2 x 2  m3 x 3  m 4 x 4
 XCM =
(90– )  L m1  m 2  m 3  m 4
x1
ub
1 0  2  0  3  0  4  
x2 XCM =
1 2  3  4
W1 = (L)g 4 5
1= =
W2 = (2L)g 10 2
P

For equilibrium, Similarly,


W1x1 = W2x2 1  0  2  0  3  0  4  2
YCM =
L 1 2  3  4
Where, x1 = sin  90°  θ  and x2 = Lsin θ
et

2 8 20 5
2= = =
L 10 8 2
 (L)g  sin(90 – ) = 2(L)g  Lsin 1  0  2  0  3  0  4  3
2 ZCM =
 cos() = 4sin ....[ sin(90 – ) = cos] 1 2  3  4
rg

12 30 5
1 3= = =
 tan = 10 12 2
4
75.
Ta

1
  = tan1   Y
4  
m3
(3, 3)
71. xCM m2
(2, 2)
(0,0) (R,0) m1
(1, 1)
m1 R m2
X
(0, 0)
m  0  m2R m2R
xCM = 1 
m1  m 2 m1  m 2 The co-ordinates of the centre of mass are
72. Considering A as origin m1x1  m 2 x 2  m3 x 3
XC.M =
m1  m 2  m3
m 1  m  2  m  3
B
= =2
A C mmm

44

Chapter 04: Laws of Motion


m1y1  m 2 y 2  m 3 y3 79. Mass = density  volume
YC.M =
m1  m 2  m3 dm = r2dz
m 1  m  2  m  3 From the figure, 
= =2 z
mmm r R
tan  = = r
Hence, the co-ordinates of centre of mass are (2, 2). z h h
R
76. Velocity of centre of mass in X-direction is zero  r= z
h
since there is no external force in X-direction.
This means centre of mass can’t change its Now,
h
position in X-direction. In other words, gun and
 z dm = 
r zdz 2
R
bullet move in opposite direction along X-axis zCM = 0

to maintain same position of C.M. in horizontal  dM 1 R h 2

direction. v
3

ns
where dM = mass element of entire cone
2
3  R2  3
h h
3 R 
2  2 
 zCM = z  z dz = 2 
z dz
R h 0 h  hR  h  0
h
In Y-direction, external force is exerted by 3  z4 

io
3h
horizontal surface on gun and hence gun is at =   =
h3  4 0 4
rest and only bullet moves with velocity
 distance of centre of mass from base is
mv sin in Y-direction.

at
3h h
velocity of C.M. is h =
4 4
m1v1  m 2 v 2
v=  h
m1  m 2  centre of mass has co-ordinates  0,0, 
4  
 vy =
mvsin   m  0
mM
lic    
 m a 1  m 2 a 2  m3 a 3  m 4 a 4
mvsin  80. a CM = 1
 vy = m1  m 2  m3  m 4
Mm
ub

m  a( i)  2m  a(j)  3m  a(i)
  4m  a( j)
77. As particles are placed around  a CM =
A B m  2m  3m  4m
origin they form arc. R 
2a i  2 a j
If arc length  0, centre of mass =
is at a distance R from the 10
P

origin. a
i  j

O  a cm =
But as the arc length AB increases, 5
centre of mass starts moving down.
81. (a) Centre of mass of a body not always
et

78. Mass = density  volume coincides with the centre of gravity of the
dm = r2dz body.
z 
From the figure, (c) A couple on a body produces purely
r
r R h rotational motion.
rg

tan  = =
z h Hence, (b) and (d) are correct.
R 82. Total initial momentum of balls = mnu
 r= z
h Total final momentum of balls = mnu
Ta

Now, R Force experienced by the surface = Rate of


h
change of momentum
 z dm = 
r zdz 2

zCM = 0 = mnu  ( mnu) ….(Assuming unit time)


 dM 1 R h 2 = 2mnu
3
83. F = ma = kt
where, dM = mass element of entire cone. Since m = 1 kg,
h 2
3 R  a = kt
R 2 h 0  h 
 zCM =  z  z dz
dv
 = kt
3 R  3 2 h dt
hR 2  h 2  0
=   z dz dv = kt dt
Integrating both sides,
h
3  z4  3h kt 2
=   = v
h3  4 0 4 2

45

MHT-CET Triumph Physics (Hints)


kt 2  dx  Let m0 be the mass that should be removed then
 dx = dt ....  v  
2  dt  B
Integrating both sides,
kt 3 6 6 6
x  1  36m a B – (m – m0)g = (m – m0)a ….(ii)
6 6
 
 m1 v1  m 2 v 2
84. v (m – m0)g
m1  m 2


  
3 2iˆ  3jˆ  3kˆ  4 3iˆ  2ˆj  3kˆ  Adding equations (i) and (ii),
34  mg  mg + m0g = ma + ma  m0a
2ma
18iˆ  17ˆj  3kˆ  m0 =
 ga

ns
7
dv
  
88. F = ma = m
85. p = Acos kt i  Asin kt j dt

mdv = Fdt
 dp  

io
 F= =  Ak sin kt i  Ak cos kt j integrating on both side,
dt v2 t

Now, to find angle between F and p


 
m  dv =  (3t 2  30)dt
v1 0

at
 
F p = (Ak sin kt) (A cos kt) + (Ak cos kt)  3t 3 
5

m(v2 – v1) =   30t 


(A sin kt)  3 0
 Fpcos = A2 k sin kt (cos kt + cos kt)  10(v2 – 10) = 125 – 30  5
= A2 k sin kt(0)
lic  v2 = 7.5 m/s
 cos = 0
  = 90 89. Given:
1
86. v
ub
u0cos x
dv 1
 =
dx 2x 3/ 2
u0 45 Dividing throughout by dt, we get,
u0cos
P

dv / dt 1
 3/ 2
dx / dt 2x
dv 1 dx
H  
dt 2x 3/ 2 dt
et

 u0 dx 1
But v =  k 1/ 2
m m dt x
dv 1 1
st
Speed of 1 particle at highest point = u0cos   32  12
rg

dt 2x x
Speed of 2nd particle at highest point =
....[Considering constant of
u 02  2gH ….(i) proportionality as (–1)]
By formula, maximum height dv 1
Ta

  2
u 2 sin 2  dt x
H= 0 substituting in (i) and solving,
2g 1  dv 
nd
 F 2 ….  F  ma  m 
Speed of 2 particle = u0cos x  dt 
Collision being inelastic, final momentum of
90. Let v1 and v2 be initial and final velocity of body
composite system = mu0cos î + mu0cos ĵ 1
 Final K.E. = m v12 ….(given)
Hence angle made w.r.t. horizontal = 8
4 1 1
87. B m v 22 = m v12
2 8
v1 10
v2 = = = 5 m/s
a 2 2
mg B = ma ….(i) frictional force is given as, F = kv2
(B is buoyant force) ma = kv2

46

Chapter 04: Laws of Motion


dv But W = f  s where f is resistive force and
m = kv2
dt s = 1 cm = 102 m.
dv 1
102 = kv2  (M + m)V2 = f  10–2
dt 2
5
dv
10 Substituting for V using equation (i),
10 v2   100k 0 dt 10 2  M  20 
2

(M + m)    =f
1 1 2  Mm 
 = 100k (10)
5 10 400M 2  102
=f
k = 104 kgm–1 2(M  m)
91. According to conservation of linear momentum, 2M 2
 f=  104
pf = pi Mm
here, uranium at rest decays,

ns
95. Initially both the particles are at rest, so velocity
 pf = pi = 0 of centre of mass is equal to zero and no
i.e., pHe  pTh = 0 external force acts on the system, therefore its
 pHe = pTh velocity of centre of mass remains constant i.e.,
p2 zero.

io
As, K =
2m
96. Initial velocity of C.M in X-direction
1
K m1 u X1  m 2 u X2 m(2  0)
m ux = = =1

at
m1  m 2 2m
 KHe > KTh ( mHe < mTh)
acceleration of C.M in X-direction
92. As the spherical bodies have their own size m1 a X1  m 2a X2 m(3  0) 3
ax = = 
so the distance covered by both the body
12R – 3R = 9R, but individual distance covered
lic m1  m 2 2m
From v = u + at, final velocity of C.M in
2

by each body depends upon their masses. X-direction is


5M 3
M vx = ux + axt  vx = 1 + t
ub
9R 2
R 2R
12R Initial velocity of C.M in Y-direction
m1 u Y1  m 2 u Y2 m(0  2)
x uy = = =1
9R-x m1  m 2 2m
These bodies are moving under the effect of
P

acceleration of C.M in Y-direction


mutual attraction only, so their position of m1 a Y1  m 2a Y2 m(3  0) 3
centre of mass remains unaffected. ay = = 
m1  m 2 2m 2
Let smaller body cover distance x just before
et

collision Now, vy = uy + ayt


From m1r1 = m2r2, 3
 vy = 1 + t
 M x = 5 M (9R – x)  x = 7.5R 2
As C.M travels with same velocity in X and Y
rg

a
93. For free fall, sn = u +  2n 1 direction, it must be travelling in straight line.
2
Where, sn = distance covered during nth second. 97. According to law of inertia (Newton’s first law),
 hn  (2n  1) when cloth is pulled from a table, the cloth comes
Ta

When the ball is released from the top of tower, in state of motion but dishes remain stationary due
then ratio of distances covered by the ball in to inertia. Thus we can pull the cloth from table
first, second and third second is without dislodging the dishes.
 h I : h II : h III = 1: 3: 5 98. ma cos 
 Ratio of work done, ma 
mghI : mghII : mghIII = 1:3:5 mg sin 
94. According to law of conservation of momentum, mg a

M  20 = (M + m)V
M  20 Let the mass of block be m. It will remain
V= ….(i)
Mm stationary if forces acting on it are in
Work done in penetration, equilibrium i.e., ma cos  = mg sin 
1 Here, ma = Pseudo force on block.
W=  (M + m) V2
2  a = g tan 
47

MHT-CET Triumph Physics (Hints)


99. Change in momentum of one molecule,
P = 2mv cos 45 = 2 mv
90 P
Force F = = n  P
t
Where, n = no. of molecules incident per unit
45 45 time
Force
Pressure P =
2 mgsin Area
Acceleration of the centre of mass of the system n  2mv
 P=
is given by, A
Fext 1023  2  3.32  1027  103
a = …(M  mass of the system) P=
M 2  104

ns
2mgsin  2 gsin  3.32
= = P=  103 = 2.35  103 N/m2
mm 2 1.41
10  sin 45 5 2  
= = 103. P = F  v ….(i)

io
2 2
2 F = mg = 250  9.8 = 2450 N
a = 5 m/s vertically downward
v = 0.2 m/s
100. Initial momentum = pi = 0 From equation (i),

at
Final momentum pf = 0 = mviˆ  mvjˆ  p 3

P = 2450  0.2 = 490 W
As, 1 hp = 746 W
 p3 = mv 2 and m3 = 4m  2(m) = 2m 490
 P= hp = 0.65 hp
rd p2
K.E. of 3 piece = 3 =
2m3
p32
2  2m
lic 746
104. Mass of deuterium is twice that of a neutron.
p32 1 1 Now, according to law of conservation of
Total KE =  mv 2  mv 2
2  2m 2 2 momentum,
ub
2m 2 v 2 3mv 2 mu = mv1 + 2m v2
=  mv 2 =  u = v1 + 2v2 ….(i)
4m 2
Coefficient of restitution for perfectly elastic
W F  s ma  s dv s collision,
101. P = = = =m 
P

t t t dt t v 2  v1
e= =1
Here P = k u
dv  v2  v1 = u ….(ii)
 k = mv ....(i)
On solving equations (i) and (ii),
et

dt
v2 k 2u u
 t v2 = and v1 = 
2 m 3 3
1
2kt Initial K.E. of neutron is, (K.E.)i = mu2
rg

v= 2
m
Final K.E. of neutron,
dv
F=m 2
1 2   u  1  1 2
dt (K.E.)f = mv1  m     mu 
Ta

Using (i) 2 2  3  9 2 
k k mk 21  Loss in K.E. = (K.E.)f  (K.E.)i
F= =  t 8 1 2
v 2kt 2 = E =  mu 
m 9 2 
E 8
102. Fractional loss = pd = = 0.89
 K.E.i 9
 Mass of carbon nucleus
v = 12  (mass of a neutron)
45  In case of collision of neutron with carbon
v cos
 nucleus,
v mv1 + 12 mv2 = mu
m v1 + 12v2 = u ….(iii)
v sin On Solving equations (ii) and (iii)
48

Chapter 04: Laws of Motion


2u 11u 107. pf m
v2 = and v1 = 
13 13
For neutron, 45
2
1  11u  121  1 2
Final K.E. = m    mu 
2  13  169  2 
45
48  1 2 pi
 Loss in K.E. =  mu 
169  2  m
48
Fractional loss = pc = = 0.28  
p  pf  pi

169

105. Mass of rope = 4  0.5 = 2 kg p  pf2  pi2 + 2pf picos 
 Total mass = 48 + 2 = 50 kg 

ns
F p = p2  p2
Acceleration of system =
m
25 = p f2 + pi2 (  = 90)
=
50 =p 2

io
1 =5 2
=
2 
1
= 0.5 m/s2 p = 7.07 kg ms

at
Force used for pulling rope = 2  0.5 = 1N
 Force applied on block = 25 – 1 = 24 N 1  m1m 2  2 2
108. KEloss =   (1  e ) (u1 – u2)
106. 20% of fat burned is converted into mechanical 2  m1  m 2 
energy
Here, mechanical energy is potential energy
lic =
1  m1m 2 

2
 (1  0 ) (v – 0)
2
2  m1  m 2 
 P.E. = mgh
When person lifts the mass 1000 times, 1  4.2  102  9  4.2  102  2
=   (300)
42  102
ub
Total P.E. = U = 10  9.8  1  1000 = 9.8  104 J 2 
Let total fat burned be x kg, = 1701 J
Hence the energy supplied by x kg fat is 1701
=
E = x  3.8  107 4.2
20 % of which is converted to U = 405 cal
P

20
 x  3.8  107  = 9.8  104
100
76 x = 9.8  101
et

 x = 12.89  103 kg

Hints to Evaluation Test


rg

1. The momentum of hammer = m12 2gh 1 m 2  2gh


 F  m1  m2  1   m 2  m1  g
 m2  m1 
2
Also, momentum of (hammer + pile) 2d
Ta

= (m1 + m2) v ….[Using (i)]


According to law of conservation of momentum, m12gh
 F   m 2  m1  g
(m1 + m2) v = m1 2gh  m 2  m1  d
momentum
m1 2gh 2. Initial velocity u 
 v ….(i) mass
m 2  m1
20
Let opposition to penetration be F, then from = = 2.5 m/s
8
work energy theorem, Force 12
Work done = Change in K.E. Acceleration    1.5m / s 2
mass 8
1 From equation of motion
(m2 + m1)gd  Fd = 0   m1  m 2  v 2
2 1 1
s = ut  at 2 = 2.5  4   1.5  4  4
1 2 2
 F  m1  m 2  v 2   m 2  m1  g
2d = 10 + 12 = 22 m

49

MHT-CET Triumph Physics (Hints)


According to work energy theorem, m1d = m2d
Increase in kinetic energy = work done. m1
 d = d
 Work done = 12  22 = 264 J. m2
3. 7. Let m1 = 2 kg, m2 = 12 kg and m3 = 4 kg. If ‘a’
30 is acceleration of the system to the right, then
the equations of motion of the three bodies are
m1a = T1  m1g,
30 m2a = T2  T1 and
m3a = m3g  T2
Components of momentum parallel to the wall Adding the three equations,
add each other and components of momentum (m1 + m2 + m3)a = (m3  m1)g
perpendicular to the wall are opposite to each  m3  m1  g
other. a

ns
m1  m 2  m3
 Change in momentum
= Final momentum  initial momentum =
 4  2 10 = 1.11 m/s2
= mvsin  ( mvsin) = 2 mvsin ….(i) 2  12  4
1 m  mv    mv 
2 2
(After (Before

io
1
8. K  mv 2  
collision) collision) 2 2 m 2m
Also, change in momentum = F  t ….(ii) p2
From (i) and (ii)  K

at
2m
2mvsin  2  1  20  sin 30
 F= = = 40 N K1 p 2 2m 5  p  10
2

t 0.5   1  22    1  
K 2 2m1 p 2 2  p2  4
4. Originally, centre of mass is at the centre O.
lic
After square 1 is removed, C.M. lies in quadrant  p1 : p2 = 1 : 1
  
3. After squares 1 and 2 are removed, C.M. lies 9. Torque is given by,  = r  F . Hence option
on Y-axis below below O. When squares 1 and (A) is incorrect.
3 are removed, C.M. will remain at O. When Though torque and work have same dimensions
ub
squares 1, 2, 3 are removed, C.M. will shift to and unit, they are different physical quantities.
fourth quadrant. When all the four squares are Hence option (B) is incorrect.
removed, C.M. will shift back to O. The direction of moment of force is
 perpendicular to the plane of figure. Hence
5. p  t   A ˆi cos kt  ˆjsin kt 
P

  option (C) is incorrect.


Mass of rope, m = 0.2 kg,  = 30
d 
  10.
 
F  p  t    Ak ˆi sin kt  ˆjcos kt
dt  
et

 T sin 
Fx px T T sin
T
 
rg


 Fy p T cos T cos
F py
 
mg
F p  A 2 k (cos kt sin kt + sin kt cost) = 0 From figure, 2Tsin = mg
Ta

 The momentum and force are perpendicular to mg 0.2  9.8


 T   1.96 N
each other at 90. 2sin  2sin 30
6. Suppose x1 is distance of m1 and x2 is distance 11. From the F.B.D.,
of m2 from centre of mass C, as shown in figure
below. Let m1 be pushed towards C through a
distance d. If m2 is pushed through a distance d
to keep the centre of mass at C, then taking C as T
the origin, we have a

d C d m m1g
m1 2 T
x1 x2 a
N N
m1 x1 = m2x2 .…(i)
and m1 (x1  d) = m2 (x2  d) .…(ii) m2g
50

Chapter 04: Laws of Motion


For m1 : N + T  m1g = m1a .…(i) u1 m P  m Q
  …(iv)
For m2 : T  N  m2g = m2a .…(ii) v1 m P  mQ
From (i) and (ii), From (ii) and (iv),
(m  m 2 ) m P  mQ mP  mQ
N 1 (g  a).
2  
mP mP  mQ
12. From F.B.D., at the moment of breaking off the mP 1
inclined plane, normal reaction will be zero. On solving,  .
mQ 3
N F sin 
F 16. Earth revolves around the sun in almost circular
 F cos 
orbit and has spinning motion about its axis.
Due to this, the velocity of earth is changing
with time. Hence Newton’s first law of motion

ns
mg does not hold good for the earth. Thus, Reason
is correct.
F sin  = mg  kt sin = mg But for the object moving on the earth, the earth
mg can be taken at rest and the frame of reference
 t

io
k sin  attached to motion on the earth is taken as
dv inertial.
Since F cos  = ma  kt cos  = m
dt 17. When an explosion breaks a rock, its initial

at
v t
k cos   k cos   2 momentum is zero. Hence, according to the law
 0 dv  m 0 t dt  v =  2m  t of conservation of momentum, final momentum
mg will be zero.
At the time of breaking off t 
k sin 
2
lic mv
k cos   mg  mg cos  2
 v    1.5 kg  12 m/s = m1v1
2m  k sin   2k sin 2 
ub
13. Initial momentum = 0
 
Final momentum = 2mv  2mv  0 m2v2 = 2 kg  8 m/s
Relative velocity of one with respect to the other Total momentum of the two pieces of 1.5 kg and
= 2v 2 kg
P

1 E
Final K.E. = 2   2mv 2  E  v  = 182  162  24 kg m s1.
2 2m
The third piece will have the same momentum
E but in direction opposite to the resultant of these
 Relative velocity = 2v  2
et

2m two momenta.
4E 2E  Momentum of the third piece = 24 kg m s1
  velocity = 6 m s1.
2m m
rg

mv 24
14. Assertion is true, but the Reason is not true.  Mass of the 3rd piece =   4 kg
v 6
Infact, the centre of mass is related to the
distribution of mass of the body. 18.
Y
Ta

15. According to law of conservation of


momentum,
mPu1 + mQ  0 = mPv1 + mQ(v1)
mPu1 = (mP  mQ)v1 …(i) C
u1 m P  m Q
  …(ii)
v1 mP
60
According to law of conservation of kinetic O
X
energy, A N B
1 1
m P u12   m P  m Q  v12 …(iii)
2 2 Taking A as the origin, the co-ordinates of the
Dividing (iii) by (i), three vertices of the triangle are:

u1 
m P  m Q  v1 A(x1, y1) = (0, 0); B(x2, y2) = (2 r, 0) and
m P  mQ 
C(x3, y3) = r, r 3 
51

MHT-CET Triumph Physics (Hints)


 Co-ordinates of centre of mass O are
m  x1  x 2  x 3  0  2r  r
x  r
3m 3
m  y1  y 2  y3  0  0  r 3 r
y  
3m 3 3
19. The velocity of ball hitting the bat = v m/s
The velocity of recoil in the opposite direction =
(v/2) m/s
 mv 
 Change of momentum = mv    
 2 
3mv
 p  .

ns
2
3mv
 Force on the ball = .
2t
F

io
20. Common acceleration, a 
m1  m 2  m3
5
a  2.5m / s 2
10  8  2

at
Equation of motion of m3 is T3  T2 = m3a
 50  T2 = 2  2.5  T2 = 45 N
21. Amongst the given balls, glass balls have
maximum coefficient of restitution i.e.,
lic
e = 0.94.
22. For the completely filled bob, C.G. coincides
ub
with its centre. As the liquid flows out, C.G.
shifts downward. When more than half of liquid
flows out, it starts shifting upwards and when
the bob gets emptied completely, C.G. is at
centre again.
P
et
rg
Ta

52
Textbook
Chapter No.

05 Gravitation

Hints

21. Gravitational constant ‘G’ is independent of the


Classical Thinking
medium intervening the two masses interacting
5. From Kepler’s second law of planetary motion, gravitationally.

ns
the velocity of a planet is maximum when its
m1  m 2
distance from sun is the least. 22. F=G
r2
6. Kinetic and potential energies vary with position m2
of earth w.r.t sun. Angular momentum remains = 6.67  1011 

io
r2
constant everywhere. 2
1
7. Kepler’s third law is a consequence of law of = 6.67  1011   
1

at
conservation of angular momentum. 11
= 6.67  10 N
T2
8. = constant GM
r3 26. g= . If the earth shrinks, its mass remains
R2
 T2r3 = constant
lic unchanged and its radius decreases. So, the
9. r2 = 2r1 value of acceleration due to gravity increases.
3/ 2
T2 r  GM GM 0 4GM 0
 =  2 = (2)3/2 = 2 2 27. g= = =
ub
T1  r1   D0 / 2 
2
R2 D0 2
 T2 = 2 2 years ….(As, T1= 1 year)
28. MP = 2M, RP = 2R
10.
1
r2 = r1 Using Shortcut 1,
4 M MP R2
P

g = g   g  
 R M  R P 2
3 3 2
T2  r2  2
 1 2
     T2 = T1   2
T1  r1  4  2M  R  g 2
 g  g      = 4.9 m/s
et

1  M  2R  2
 T2 = 24  = 3 hr
8
Re M
29. Rm = , Mm = e
 Gm1m 2 rˆ Gm1m 2 r Gm1m 2  4 80
13. F = = r̂ =
rg

2 3
r
r r r3 Using Shortcut 1,
2
14. From Newton’s law of gravitation, gm Mm  R e  1 2
      (4)
Gm1m 2 g e M e  R m  80
F=
Ta

r2 g
If m1 = m2 = 1 unit of mass  gm =
5
r = 1 unit of distance
F = G = universal gravitational constant 4
32. g= GR  for R constant; g  
3
m1m 2 Fr 2
18. F =G G=
r2 m1m 2 1 d
33. = 1 : 2, 1 = 4 : 1
Nm 2 2 d2
 Units of G is
kg 2 4
Since, g = GR
3
20. The value of universal gravitational constant  g  R  g1  1R1 and g2  2R2
is always same. As r varies, the force between
g1 R 1 4 2
the two bodies changes, but G remains  = 1 1 =  =
g2 2 R 2 2 1 1
constant.
53

MHT-CET Triumph Physics (Hints)

34. p = 2e, gp = ge 2GM


53. ve =
4 r
g = GR
3 Thus, escape velocity is independent of mass of
R p  g p   e  1 satellite and depends on the radius of orbit.
       1    Hence, they have equal escape velocities.
R e  g e   p  2
R R 2GM
 Rp  e  54. ve =
2 2 R
 ve  M if R = constant
35. Using Shortcut 3,
1
 If the mass of the planet becomes four times
Here, n = then escape velocity will become 2 times.
3
2
gh  n  1 2GM M
 ve 

ns
    55. ve =
g  n + 1  16 R R
If mass and radius of the planet are three times
36. Using Shortcut 3, than that of earth then escape velocity will
Here, n = 1 remain same.

io
2
gh  n  1
    56. Mp = 2Me, Rp = 3Re
g  n + 1 4
vp Mp Re 1 2
g    2 

at
 gh  ve Me Rp 3 3
4
2
37. Using Shortcut 5,  vp  ve
3
Here, n = 16,
 h=  
n 1 R =  
lic
16  1 R = (4 – 1) R
57. MA = 2MB, DA= 2DB RA = 2RB
2GM
ve =
 h = 3R R
ub
 d R d gr (v e ) A 2M B / 2R B
 = =1
38. gd = g  1   = g    gd = (ve ) B MB / R B
 R  R  R
 (ve)A = (ve)B
 R / 2 g 10 ms 2
39. gd = g 1   or gd =  = 5 ms2 58. ve    v1  1 and v2  2
P

 R  2 2
v1 1
40. At the centre of earth g = 0;  
v2 2
Weight = mg = 100  0 = 0
et

ve 1 2GM 2GM
42. When the earth stops rotating, the centripetal 59. =  
force of mR2 vanishes. As a result of this, the R R R R3
acceleration due to gravity increases. 2GM 2G  4 3 
rg

2 2
60. ve = =  R  
43. g = g  R cos θ. Hence value of g changes R R 3 
with θ. 8GR 2 2G
ve = = 2R
3 3
Ta

46. Change in potential energy in displacing a body


from r1 to r2 is given by
2GM 2GM
1 1  61. ve = , ve =
U = GMm    R Rh
 r1 r2  As R + h > R  ve > ve
 1 1 
= GMm    v1 2g1R1
 2R 3R  62. = = k1k 2
v2 2g 2 R 2
GMm
=
6R RE
63. Rp = , gp = 2gE
4
GM GM
50. V=– =– 2 R ve = 2gR
R R
GM v eP gp Rp 1 1
= – gR ….(Since, g = )     2 
R2 v eE ge RE 4 2

54

Chapter 05: Gravitation

64. Escape velocity, 2r 2r 42 r 3


89. T= = = ….(i)
2GM vc GM GM
ve =
R r
1/ 2
 2  6.67  1011  6  1024  Since r  Rp
=  
 6.4  106  where Rp = Radius of the planet,
= 1.12  104 m/s 4
Substituting, M =  R 3p  in equation (i),
= 11.2 km/s 3
42 R 3p 3
2GM  T= =
65. On earth, ve = = 11.2 km/s 4
G  R 3p G
R
3
2GM  4 2 2GM 1
On moon, v m    T
81  R

ns
9 R 
2
=  11.2 3 2
9 1 1
90. T  r 2 i.e., r  T 3 ; K.E.   2
= 2.5 km/s r
T3

io
67. Geostationary satellite remains stationary with 2

respect to the earth.  K.E.  T 3


Since the time period of earth is 24 hours, 3

at
3
therefore time period of a geostationary satellite T2  r2  2  6400  2
91.     T2 = 24   ≈ 2 hour.
is also 24 hours. T1  r1   36000 
74. If it is not so, then the centrifugal force would
92. T = 83 min, R = 4R
gravitational force won’t be able to keep the
lic
exceed the force of attraction or we can say that
T'  R 
3/ 2
 4R 
3/ 2

 =   =  
satellite in circular motion. T R R 
1 K mv 2 T is increased by a factor of [4]3/2 i.e., 8 times.
ub
76. F F=   v = constant T  = 8  83 minutes = 664 minutes
r r r
77. As, v is independent of mass of the satellite. GmM GM mR 2 mgR 2
93. B.E. =  2  
v1 r 2r R 2r 2r
 = 2  r1 > r2  v2 > v1
P

v2 r1 GmM
94. B.E. = = mgR
Orbital speed of satellite does not depend upon R
the mass of the satellite. = 100  10  6.4  106
et

78. r1 = 4r, r2 = r = 6.4  109 J


1
Orbital speed vc  GM 2 Mv 2 GM
r 95. 2
= or = v2
(2R) R 4R
rg

v1 r2 r 1 1
 = = = = 1 GM
v2 r1 4r 4 2  v=
2 R
79. RA = 9R, RB = R
Ta

96. r = 1.5  108  103 m


GM When orbiting, gravitational force
v
R
F = m2r ….(Since, v = r)
vA RB R 1 = 6  1024  (2  10–7)2  1.5  108  103
   
vB RA 9R 3 = 36  1021 N
v A 4v 1
    vB = 12v GM m
vB vB 3 97. W = mg = and
R2
80. In a free fall, even near the earth, a body is in a GM m
W = mg =
state of weightlessness. (R  h) 2
85. Longer period and slower velocity as GM m 4
=  X
1 (R  R / 2) 2 9
T r 3 and v 
r Hence, assertion is false but reason is true.
55

MHT-CET Triumph Physics (Hints)


1 1 F
 F  = 2  F =
Critical Thinking (2r) 2 4r 4
1. According to Kepler’s law T2  R3  Force is reduced to one-fourth.
If n is the frequency of revolution then 8. r = 50 cm = 50  102 m
n2  (R)3 m1m 2 6.67  1011  625  625
3/ 2 2/3 F=G =
n2  R 2  R1  n 2  r 2
50  50  104
     
n1  R 1  R 2  n1  = 1.042  104 N = 10.42 dyne

2. Angular momentum, 9. G = 6.67  10–11 Nm2 / kg2


A A 105 dyne  104 cm 2
L = 2M  =
L = 6.67  10–11 
t t 2m 106 g
= 6.67  10–11+3

ns
3. TA = 8 T B = 6.67  10–8 dyne cm2/g2
TA2 rA3
Using Kepler’s third law, = GMm
TB2 rB3 10. Fe = = 50 N ….(i)
3 R2
r 

io
(8TB ) 2 GMm '
 2
=  A ….[ TA = 8TB] Fs  F ….(ii)
TB  rB  4R 2
 rA 
3
rA  Dividing equation (ii) by (i) we get

at
3
  = (4)  = 4 or rA = 4rB F

m ' 200

 rB  rB 50 4m 4  5
4. rM = 1.525 rE 
lic F = 10  50 = 500 N
rM Gm1m 2
 = 1.525 11. F=
rE r2
Gm1m 2 F
 TM 
2
 rM 
3
 F = =
 3r 
2
 3 9
  =   = (1.525)
ub
T
 E r
 E  F  F 
 % decrease in F =   100
 TM = TE  (1.525)3 = (1)2 (1.525)3
2 2
 F 
 TM = (1.525)3/2 = 1.883 years 8
 100  89%
=
9
P

(2) 2 3
5. T2 = R
12. F = mg = 81 =
GM m
GM
R2
3  4  2
GM m
 log10T = log10R + log10   
et

F = mg =
2  GM   R
2

R 
2 1  42   2
 log10 R = log10T  log10  
3 3  GM  
4 GM m 4
=  81 = 36 N
rg

F=
4 2 9 R2 9
 = 1018  M = 6  1029 kg
GM Gm1m 2
13. F=
6. Let  be the density of the material of each r2
Ta

sphere. Gm1m 2 6.6  1011  1  1


 r2 = =
4 4 F 109  9.8
Then, M1 = r 3 and M2 = (2r 3 )
3 3 2
=  10–2
Distance between their centres = r + 2 r = 3 r 3
 4  4  = 0.673  10–2
G  r 3  8r 3 
GM1M 2  3  3   r  0.08 m  8 cm
Now, F = 
(3r) 2 9r 2
14. r = 20  102 m, total mass = 5 kg
4
This gives F  r Let m and (5 – m) be the two masses
 Assertion is false. G m1 m 2
F=
r2
7. r = 2r ….[Given]
6.67  10 11  m  (5  m)
1
Now, F  2  1  10–8 =
r (2  101 ) 2

56

Chapter 05: Gravitation

m(5  m) 21. M = M/7, R = R/2


 1  108 = 6.67   10–9
4 Using Shortcut 1,
2 2
40 m(5  m) g M  R   M  2R  4
 10 =         
6 4 g M  R    7M  R  7
 m2 – 5m + 6 = 0  (m  2) (m 3) = 0 4g
 weight on the planet = 700  = 400 g-wt
 m = 3 or m = 2 7

15. At point P, GM
22. g=
G(81M) G M R2
= gR 2 9.8  (6  106 ) 2
 60R  x 
2 2
x M  M= =
2
G 6.67  1011
x x 60 R
 (60R  x)2 = 9.8  36
81 81M  M=  1023 = 52.89  1023 kg

ns
6.67
x
 60R  x =  M  5.3  1024 kg
9
 x = 54R and (60R  x) = 6 R 23. d
 b

io
Kr 2
16. G= ag
mML
As r has highest power in the formula, error in a

at
measurement of r, when all quantities measured
have same percentage error, will cause
maximum error in measurement of G. lic c
17. Gravitational pull depends upon the acceleration Ri
due to gravity on that planet. As the star collapses,
1 1 its mass remains the same and radius decreases.
Mm = M e, g m = g e GM 1
81 6 ag =  2
R2
ub
Ri
Using Shortcut 1,
1/ 2 1/ 2 ag increases as radius decreases. This is depicted
R e  Me gm   1 correctly by plot b.
    =  81  
R m  Mm ge   6 
24. Weight is least at the equator.
9
P

 Re  Rm 25. The weight will go on decreasing as the value of


6 g decreases with increasing height.
18. Using Shortcut 2(iv), for constant mass, 26. As the body moves away from surface of the
et

Percentage change in g earth, its weight decreases due to decrease in


= 2  (Percentage change in R) gravitational acceleration of the earth. But as it
= 2  1% = 2% enters gravitational field of moon, its weight
starts increasing due to increase in gravitational
rg

0.99 M GM acceleration of moon.


19. g = G = 1.01 2 = 1.01 g
(0.99 R) 2 R 16g
27. gh = 16 % g =
g 100
  1 = 0.01
Ta

g g h 16
 =
g  g g 100
 × 100 = 1%
g g
 gh = 100
 
20. Mp = 2Me  
 16 
4 3 4 Using Shortcut 5,
 R P  = 2  R 3e 
3 3 100
 R 3P = 2R 3e  Rp = 2 Re
1/3 Here, n =
16
GM p G  2M e  1
2
GM e  100  5 
 gp = = 2
2 3
 h =   1 R =   1 R
R 2p  21/3 R e  R e2  16  4 
 gp = 21/3ge R 6400
=  = 1600 km
 mgP = 21/3 mge = 21/3W 4 4

57

MHT-CET Triumph Physics (Hints)

 2h  gd
28. g = g 1    = R2 cos2 ϕ
 R  R
w2  w1 = error in weighing gd
 cos2ϕ = 2 2
h h  GM h R
= 2mg  1  2  = 2m 2 gd
R R R R  cosϕ =
2mG 4 3 h 8 R
 w2  w1 =  R   = Gmh  gd 
R2 3 R 3  ϕ = cos 1  
 R 
 d
29. mgd = mg 1 
 R  34. g = g  R2 cos2 θ;
 d When θ = 45,
 31.5 = 63 1  
 R 1
g = g  R2  

ns
d 31.5 1 2  
 1 = = When earth stops rotating, g = 0,
R 63 2
d 1 1 R2
 =1 = so, g =

io
R 2 2 2
R Hence the weight of the body increases by
 2d = R or d = = 0.5R R2
2 .

at
2
 d  80 
30. g = g 1   = 10 1   35. g = gp  R2 cos2 θ = gp  2R cos2 60
 R  6400 
1
 1  10  79 = gp  R2
= 10 1   =
 80  80
lic 4

= 9.87 m/s2  990 cm/s2 36. θ = 0, g = g  R 2 cos2 θ = 0


  = g / R = 10 / (6400  103 ) = 1/800
31. For scientist A who goes down in mine,
ub
 d 37. U = Loss in gravitational energy
g = g 1  
 R  = gain in K.E.
For scientist B, who goes up in air, 1 2U
 U= mv2  m = 2
 2h  2 v
g = g 1 
P


 R  Re 2
38. Rm = , m = e
So, it is clear that value of g measured by each 4 3
will decrease at different rates. Energy spent = mgehe = mgm hm
et

 hm= gehe/ gm
 2h 
32. gh = g  1   4 
 R   R eeG   h e
hm = 
3 
  R  
rg

4
 2  20    1 R mm G
 9 = g 1     = g 1   3
 R   10  R  3 4
   hm = e  e  h e    0.5 = 3 m
R m m 2 1
Ta

9g
 9 =  g = 10 ms2 39. Using Shortcut 16 (i) and equating work done
10
to change in potential energy,
  R 
   mgh
 d  20   = 10  19  U =
 gd = g 1   = 10 1    1  h/R
 R  R   20 
  Substituting R = 5h
mgh 5
 gd = 9.5 ms2 we get, U = = mgh
1  1/5 6
33. Given: gd = g Alternate Method:
 d 2 2 Increase in the P.E. is given by,
 g 1   = g – R cos ϕ
 R  U = UB  UA
=  
gd GMm GMm  5GMm
 g– = g – R2 cos2 ϕ UB =   =–
R Rh  R R /5 6R

58

Chapter 05: Gravitation

GMm dv e 1 dR 1
UA =   =– = –  4 % = 2%
R ve 2 R 2

 U = 
5GMm
+
GMm GMm  5 
=  As radius decreases, escape velocity increases.
1  
6R R R  6
GMS
GMm 47. vc =
U = r
6R
Orbital speed of all planets depends upon the
mgR 2
 U = …..( GM = gR2) mass of Sun and the separation. So,
6R
1
mgR vc 
 U = r
6
5 Since Jupiter is having more orbital radius in
 U = mgh ….(R = 5h) comparison to earth, so orbital speed of Jupiter
6

ns
is less than that of earth.
40. Gravitational potential energy from centre of the
48. Critical velocity of a satellite is independent of
G  81M m  m
earth U1 = GM e m = mass of a satellite.
R R

io
Gravitational potential energy from centre of the GM gR 2
49. vc =  and vc  r
moon U2 =  GM m m r r
r
R 2g
This gives r3 =

at
Total gravitational potential energy,
2
U = U1 + U2
G  81M m  m GM m m  81 1  GM GM
=  =  GMmm    50. v1 = , v2 =
R r  R r Rh R
lic v1 R R 1
2GM  = = =
41. ve = v2 Rh R  7R 2 2
R
v
8 4  v1 =
ub
 ve = R G ….( M = R3) 2 2
3 3
Now, ve  R and vp  2R 51. In the problem, orbital radius is increased by 1%.
vp vp Time period of satellite T  r3/2
 = 2 or ve = Percentage change in time period
P

ve 2
3
= (% change in orbital radius)
2GM 2
42. ve = 3
Rh = (1%) = 1.5%.
et

2
2GM
 (ve)1 = = v and
2R 52. Time period of satellite which is very near to
2GM planet
rg

(ve)2 =
8R R3 R3
T = 2 = 2
GM 4
(ve ) 2 2GM 2R 1 1 G R 3
 =  = = (ve)1 = v/2 3
(ve )1 8R 2GM 4 2
Ta

1
43. Using Shortcut 8,  T

1 1
ve = – R% = –  4 % = – 2% i.e., Time period of nearest satellite does not
2 2 depend upon the radius of planet, it only
As radius decreases, escape velocity increases. depends upon the density of the planet.
Alternate Method: In the problem, density is same so time period
2GM will remain the same.
ve =
R
3 3  3.14
ve 
1 53. T= = s  4200 s
R G 6.67  10 11  8  103
 ve  R–1/2 1

1  42 r 3  2
 dve  – dR R–3/2 54. Ts =   = 6831 s and Te = 86400 s
2  GM earth 

59

MHT-CET Triumph Physics (Hints)

Relative angular velocity = satellite  earth 60. If G starts to decrease, the force between sun
2 2 and earth will also start to decrease. Earth will
T= = try to follow a path of larger radius. Hence, its
s  e  2 2 
   period of revolution round the sun will increase.
 Ts Te  But rotation of earth around its own axis will
TT remain unchanged. The radius of the circular
T = s e = 7417 s
Te  Ts path of the earth will increase or the earth will
follow a path of increasing radius. Thus, P.E.
55. Orbital radius of satellites: r1 = R + R = 2R will increase so K.E. decreases.
r2 = R + 7R = 8R
GMm GMm 61. Apparent weight = actual weight – upthrust force
P.E1 = and P.E2 = Vdg = Vdg  Vg
r1 r2
 d 
GMm GMm  g =  g

ns
K.E.1 = and K.E2 =  d 
2r1 2r2
GMm GMm 62. In pendulum clock, the time period depends on
T.E1 = and T.E2 = the value of g while in spring watch, the time
2r1 2r2
period is independent of the value of g.

io
P.E1 K.E1 T.E1
 = = =4
P.E 2 K.E 2 T.E 2 2GM E
63. (ve)Earth = = 11.2 km /s
RE

at
GMm
56. U= and As radius and mass of the planet are one third to
r
those of earth, escape velocity at planet (ve)p is
GMm
Kinetic energy = same as the escape velocity at the earth.
2r

 U = (2)
GMm
lic (ve)p = 11.2 km/s
Now, average thermal velocity of oxygen (v)
2r molecules at temperature T is,
= 2 × Kinetic energy 1 3
mv2 = kT
1
ub
= 2 × mv2 2 2
2 3kT 3  1.38  1023  1000
=  mv 2  v= =
m 5.3  1026
GMm 1 = 883.82 m/s  0.88 km/s
57. B.E1 = = mgR and
P

2R 2 Using Mindbender 7,
GMm
Thus, thermal velocity of oxygen molecules is
B.E2 = = mgR much less than the escape velocity. Hence, it is
R
possible to find oxygen molecules in the
et

1 1 atmosphere around the planet.


 B.E2 – B.E1 = mgR – mgR = mgR
2 2
1
GMm 64. v
58. E=  r
rg

2r 1
dE GMm 1 dr  % increase in speed = (% decrease in radius)
  = 2
dt 2 r 2 dt 1
= (1%)
Ta

t R
GMm dr  dE  2
 dt 
0
2C r r 2
…. 
 dt
 C J/s 
 = 0.5%
i.e., speed will increase by 0.5%
GMm  1 1 
 t=   
2C  R r  65.
N
GMm r  R
59. P.E. = 2
r
1  GM  
Fg  m  3  r
 P.E.   R 
r
Similarly,
1 1 Pressing force = N
T.E.  and K.E.   GMm 
2r 2r =  3  r cos
This is depicted correctly by option (C).  R 

60

Chapter 05: Gravitation
2
GMm R /2
= gE  2   
1
= 3
r  
R  r  2
GMm gE gE
= = constant. =  2
2R 2 2 gP
1
66. Let vapp = velocity of approach Now, for a pendulum, T 
vsep = velocity of separation O g
vsep 2 gE
e=  R  TP = TE   TE 2  2 2 s
v app 3 gP

GMm 1 2 R 71. If radius of earth decreases by 2% then


 mv app
2R 2 using Shortcut 2(iv), for constant mass,
GM 2GM g increases by 4%.

ns
 vapp =  vsep = L2 1
R 3R Now, K.E. =  K.E.  2 and hence K.E.
GMm 1 GMm 2I R
Also,  mvsep
2
 increases by 4%.
Rh 2 R

io
1 2 1 1  Alternate Method:
 vsep = GM   
2 R Rh GM 1
g=  g
R2 R2
GM GM  R 
 = 1 

at
 2dR dg 2dR
3R R  Rh  dg   
R3 g R
1 R
 =1 dg dR
3 Rh   100 = 2 (  100) = 2  2% = 4%
g R

R
Rh
=
2
3
lic i.e., g increases by 4%
L2 1
R Now, K.E. =  K.E. 
 h= 2I I
2
ub
2
I= MR2  I  R2
ve 5
67. Given: v =
2 1
 K.E.  2
Using Shortcut 14, R
R R R 2dR
P

 h=    dK  – 3
 ve 
2
4 1 3 R
  1
 100 = – 2    100 
dK.E. dR
 v e /2  
K.E.  R 
et

Fg = – 2  2% = 4%
68. a1 = Fg Fg
m1 m1 m2 K.E. increased by 4%
Fg 72. Applying law of conservation of energy for
rg

a2 =
m2 asteroid at a distance 10 Re and at earth’s
Since there is no external force, centre of mass surface,
remains at rest and energy remains same. Ki + Ui = Kf + Uf ….(i)
Ta

1 2 vi
Now, Ki = mvi 10 Re
69. ve = 2 vc = 1.414 vc 2 vf
= vc + 0.414 vc GM e m
and Ui =
ve  vc 10R e
 = 0.414 Re
vc 1
Kf = mvf2 and
 % increase in speed = 0.414  100 = 41.4% 2 Me
GM e m
70. Mp = 2ME, DP = 2DE  RP = 2RE Uf = Earth
Re
TE = 2 s
Substituting these values in eq.(i), we get
GM E GM P
gE = , gP = 1 2 GM e m 1 2 GM e m
RE R 2P mvi  = mvf 
2 10R e 2 Re
2
M R  2GM e  1
 gP = gE  P   E   vf2  vi2  1  
ME  R P  R e  10 

61

MHT-CET Triumph Physics (Hints)


Gm(M  m) 1 1  1 2 1
73. F= GMem    = mv B  mv A
2
r2  R (R  h)  2 2
dF
For maximum force =0 1  1 1 
dm = GM e m  2  2 
2 R (R  h) 
d  GmM Gm 2 
   2 = 0 2Rr
dm  r 2 r   = ….( r = R + h)
Rr
m 1
 M  2m = 0   GM e R
M 2  v 2A =  2GM e
(R  h) 2 r(R  r)
1  42  1
74. m2R  n
 m  2  R  n  T2  Rn+1 79. Since ve = 2 vc = 1.414 vc
R  T  R
 n 1 
Additional velocity = ve  vc = vc  
2 1

ns
 
 T R  2 
= vc (1.414  1)
75. Both the stars rotate with same angular velocity = 1  0.414
 around the centre of mass (CM) in their = 0.414 km/s

io
respective orbits as shown in figure.
2GM
80. ve =
 R
1

at
r1 r2  K.E1 = mve2
2
m1 m2
C.M. 1 2GM
= 2 m
R R

lic 1
= 2 m (2gR) = mgR
R
The magnitude of gravitational force m1 exerts GM
Gm1m 2  vc = at h ≈R
R
ub
on m2 is F 
 r1  r2 
2
1 1
 K.E2 = mv c2 = mgR
2 2
Gm A m B m A rA 4 2
m r 4 2
76.   B B2 K.E1 2mgR 2
 rA  rB 
2
TA2 TB  = =
P

K.E 2 mgR 1
 mArA = mBrB Alternate Method:
 TA = T B 1
K.E1 = mve2
et

GMm 2
77. If r < R then F = .r 1
R3 = m × 2gR = mgR ….(ve = 2gR )
mv 2
GMm 2
  r vr
When orbit is close to Earth, vc = gR
rg

r R3
mA rA rB
GMm mB 1 1
If r > R then F = C K.E2 = mv2 = mgR
r2 2 2
K.E1  mgR 
2
mv GMm 1
Ta

  2 v   =2
r r r K.E 2 1 mgR
2
78. Suppose the velocity of vA
projection at A is vA and 81. Weight of the body at equator
at B is vB. h 3
mv 2A GM e m = of initial weight
 = and 5
A (R  h) 2 R
3
Earth  g = g (because mass remains constant)
mv 2B GM e m 5
= vB
B R2 g = g  2R cos2θ
A = B =  are the radii of curvatures at A, B. 3
g = g  2R cos2 (0)
Energy conservation gives, 5
GM e m 1 GM e m 1 2 2g
 mv 2A =   mv B  2 =
Rh 2 R 2 5R

62

Chapter 05: Gravitation

2g 2  10 3. Point A indicates perihelion position while point


  = = C represents aphelion position.
5R 5  6400  103
4 This means point A is closest to the sun
= 62.5108 = 7.9  10 rad/s followed by point B and C.
Hence, vA > vB > vC
82. Since, F = Mr2,
 KA > KB > KC
R R
 T  T2  4. Using Kepler’s law,
F F
2/3
R 5
r T 
 T2   T2  R 2 T  r , E  E 
2 3
  32  rP  TP 
R 
  2 1
As  = ,T
T 
85. T2  r3

ns
2/3
Ts r 
3/ 2
r/2
3/ 2
1
3/ 2 rE  P 
 = s = =    
 rP  E 
Tm  rm   r  2
Ts R
Let Ts = n Tm  =n  = (2)2/3

io
Tm rP
1  rP = (2)2/3 R
 n= = 2–3/2
23/ 2 5. Using Kepler’s law,

at
86. Gravitational potential at a point on the surface T2  r3
GM T2 = kr3
of Earth =  Take ln on both side
R
lnT2 = lnkr3
If Earth is assumed to be a solid sphere, then the
gravitational potential at the centre of Earth
lic 2lnT2 = lnk + 3lnr
Differentiating both side w.r.t. x
 3 GM 
=  1 dT 1 dk 1 dr
2 R  2  3
T dx k dx r dx
ub
 Decrease in gravitation potential 2T k r
1 GM Rg  3
=   T k r
2 R 2 2 T r
Rg 3
 Loss in potential energy = ×m T r
P

2 3 r
Now, gain in kinetic energy = loss in potential T = T
2 r
energy
1 1 6. According to Kepler’s law,
et

 mv2 = mgR  v = gR T2  r3
2 2
r1  r2
Where, r =
2
Competitive Thinking
rg

R
As r1 = R and r2 =
dA 1 d 1 3
1. = r2 = r 2 …(i) R
dt 2 dt 2 R
Angular momentum L = mr ω …(ii) 2 3 = 4R = 2R
Ta

Here, r =
dA L 2 6 3
Using (i) and (ii), = .
dt 2m r3
Now, T = 2
Gm
2. According to Kepler’s second law,
Angular momentum is conserved during the 4 2 3
 T2 = r
motion of the planet around the Sun. Gm
3
 
42  2R  8  42 3
 LA = LB =   = R
Gm  3  27Gm
 m vA rA sin A = mvB rB sin B
vA r sin B 60  106   sin 60o   T2 =
322
R3
 = B = 27Gm
vB rA sin A 90  106   sin 30o 
Comparing with T2 =  R3,
2 3/2 2 322
=  = =
3 1/ 2 3 27Gm

63

MHT-CET Triumph Physics (Hints)


7. Under mutual gravitational force, astronauts move AC = a 2
towards each other with very small acceleration.
AC a 2 a
a1 a2 r= = =
8. 2 2 2
F F
m1 m2 Resultant force on particle,
r GM 2 ˆ GM 2 ˆ GM 2
|F| = i  2 j
 
2 2
a a a 2
Gm1m 2 F Gm 2
F= 2 and a1 = = 2
r m1 r (cos 45 î + sin 45 ĵ )
 a1  m2
GM 2
 2  + GM
2
 |F| =
9. Gravitational attraction force on particle B a2 2a 2

GM P m GM 2  1
Fg = =  2 

ns
2
 DP  a2  2
 
 2  Resultant force acting towards the centre
Acceleration of particle due to gravity Mv 2 Mv 2
Fg 4GM P = 
a =  r  a 

io
m D 2P  
 2
L
10. For a mass m placed at distance, x = from the Mv 2 GM 2  1
2  = 2  2 

at
 a  a  2
rod, force of attraction between the two is,  
GmM GmM GMm  2
F=  
x  x  L L  L  3L2  v2 =
GM 
1
1 
  L   
2 2


GMm
4

lic a  2 2

Comparing with F =   2  GM  1 
 v=  1 
 L  a  2 2
4
=
ub
GM
3 = 1.16
a
11. We know that, F  m1m2
 F   xm   (1  x) m  xm 2 (1  x) 14. 2 = 21, R1 = R2
dF g  R  g1  1R1 and g2  2R2
P

For maximum force, =0


dx g1 1 R1 1 1
     1
dF g 2 2 R 2 2 2
 = m2  2xm2 = 0
dx  g2 = 2  9.8 = 19.6 m/s2
et

1
x=
2 15. Using Shortcut 1,
F F Mv 2 1 1
12.   F1  Here, M = , R =
7 2
rg

2 2 R M F M
2  GM 2 GM 2 Mv 2 F M 4
   v R 1  =
 R
2
7
  F
2
2 R 2 4R 2 R
O 4
Ta

GM 2  1 1   gplanet = g
  = Mv2 M 7
R  4 2 
M
4
 Wp = 700   g = 400g
GM  2  4  1 GM 7
 v=  
R  4 2  2 R
1 2 2  
16. At the surface,
13. GM
A B g=
R2
At the height, h = nR
r
GM GM
O a gh = 
R  h R 2 1  n 
2 2

45 g GM / R 2
45   = (n + 1)2
D C g h GM / R 2 1  n 2
a

64

Chapter 05: Gravitation

17. Using Shortcut 5, Given : gh = gd


For n = 4,  d = 2h
 h=  
n 1 R =  
4  1 R = (2 – 1) R
24. Given: gd = gh ….(i)
 h=R  d
But, gd = g 1   and
18. Using Shortcut 3,  R
For n = 2,  2h 
gh = g  1  
   
2 2  R 
n 2 4
gh = g= g= g
n+1 2 1 9  d  2h 
 g 1  = g 1   ….[From (i)]
4 4  R  R 
Wh = mgh =  mg =  72 = 32 N
9 9  d = 2h

ns
19. Using Shortcut 3,  d=21 ….( h = 1 km)
2
gh  n  1 n 1  d = 2 km
    
g  n + 1  16 n 1 4
25. Acceleration due to gravity at h = 5 km above
1

io
 n=  2h   25  2
3 gh = g  1   = 9.8 1    9.78 m/s
R  R   6400 
h = = 3R OR
n

at
GM GM GM R2
20. Refer Shortcut 6. gh =   2 
(R  h) 2
(R  5) 2
R (R  5) 2
Alternate Method:
 d gR 2 9.8  (6400) 2
= 9.78 m/s2
gd = g  1  
 R
lic =
(R  5) 2
=
(6400  5) 2
g  d g Acceleration due to gravity at depth = 5 km,
= g 1   ….(Given: gd = )
n  R n  d
gd = g  1  
ub
d 1  R
 = 1
R n  5  2
= 9.8 1   = 9.79 m/s
 n 1  6400 
 d = R 
 n 
26. Acceleration due to gravity at a depth x below
P


d  1600  surface of earth is
21. gd = g  1  = 9.8 1  

R  6400  GM  x  x
g = 1   = g 1  
3 R2  R   R 
et

gd = 9.8  = 7.35 ms–2


4 at the depth x, distance of point from centre of
22. Acceleration due to gravity at depth d, the earth is (R  x) i.e., d = R  x
In this case,
gd = g  1  
d
rg

 R  gx  R  x
 gx  d
= g 1   ….  d 
1 R
 At height h distance from centre of the earth is
 2  2
Ta

(R + h) i.e., d = R + h Y
g
 gd =  R 
2 gx  d
2 In this case, gh = g  
Weight of the body at depth d = R/2, Rh 1
g

2
gh  d
2
1 gR
Wd = mgd = m  g/2 = × 200 =
2 d2
 Wd = 100 N 1 d
X
 gh  d=R
d2
23. Gravity at height h,

gh = g  1 
2h  28. g = g  2R cos2θ

 R  Rotation of the earth results in the decreased
Gravity at depth d, weight apparently. This decrease in weight is
 d not felt at the poles as the angle of latitude is
gd = g  1   90.
 R

65

MHT-CET Triumph Physics (Hints)


29. i. Going down from surface towards centre – n 1
 =
g n 1 3
gdepth =
 d  3n = n + 1
1  
 R 1
As d increases, g decreases.  n=
2
ii. Going up from surface – R
g  h=
gheight = 2 2
 h
1   33. Using Shortcut 7,
 R
As h increases, g decreases. n = 10
iii. Going from equator to pole –  10  10 mgR
 U = mgR   =
g is less at equator and more at poles  10  1  11

ns
owing to bulge at equator and flattening at
GM
poles. Thus, g increases in moving As, gR =
R
towards poles.
iv. Changing rotational velocity – 10 GMm
 U =
11R

io
g = g – R2 cos2 
As  increases, g decreases. 34. Using Shortcut 7,
30. g = g  2 R cos2 θ n=2

at
1  2  2
 0 = 1  2  6400  103  ….(As, θ = 60)  U = mgR   = mgR
4  2  1  3
104
 2 =
16
102
lic 35. Work done = change in P.E.
Using Shortcut 7,
= here, n = 1
4
  = 2.5  103 rad/s  1  1
 W = U = mgR   = mgR
ub
11 2
31. Refer Shortcut 7.
Alternate Method: 36. Potential energy of object of mass m on the
GMm  GMm
U=  surface of earth, U =
 R  nR  R
P

Change in potential energy Potential energy of object of mass m at a height


GMm  GMm  h from the surface of the earth,
U2  U1 =    GMm
R  nR  R  U =
et

Rh
GMm GMm
= 
R R  n  1  Change in potential energy
= U  U
GMm  1 
rg

= 1    GMm GMm
R  n 1 = +
Rh R
GMm  n  1  1 
=   GMmh
R  n 1  =
R R  h
Ta

GMm n
= 
R  n  1 37. Gravitational potential is given as,
GMm  R  n  GM
= V=
  R
R2  n 1
1 1 1 1 
= mgR 
 n   V =  GM      ....   
 1 2 4 8 
 n 1
 1 1 1 
32. Using Shortcut 7, =  G  2 1  2
 3  ....   
 2 2 2 
 n 
U = mgR   1
 n  1 =  2G
 1
1  n   1 
 mgR = mgR   2

3  n  1  V =  4G
66

Chapter 05: Gravitation

38. Given: MP = 3Me, RP = 3Re 46. For a satellite,


2GM e T2  r3
(ve) =
Re  T2  (RE + h)3
3
2GM P T2  R E  h 2  2
vP =   
RP T1  R E  h1 
3
vP MP × R e 3Me × R e
 = = =1 T2  R E  2.5 R E  2
ve R P× Me 3R e × M e   
T1  R E  6R E 
 vp = ve
T2 1
 
8 24 2 2
39. Escape velocity, ve = R G
3  T2 = 6 2 h

ns
v R 
 e  47. T = 6 hr = 6  60  60s
v p R p p
Time period of satellite is,
Given: Rp = 2R and p = 2 r3
ve 1 T = 2

io
 = GM
vp 2 2
4 2 r 3
 T2 =
2GM GM

at
40. ve = =c ….(Refer Mindbender 6) 3 GM
R  r =  T2
2GM 2  6.67  10 11  5.98  1024 4 2
R= 2  = 8  1012  (6  60  60)2
c (3  108 ) 2

=
2  6.67  5.98
 103 m
lic = 8  1012  (63)2  104
Taking cube roots on both the sides,
9
r = 2  104  62  (10)  101/3
= 8.86  103 m  102 m
= 72  105  2.1
ub
41. K.E. = P.E.  R + h = 151.2  105
1 2 GMm
mvs =  h = (15.12  106) – (6.4  106)
2 2R
= 8.72  106 m
GM
vs2   h = 8720 km
P

R
vs = gR ….(GM = gR2) 48. ve = 2v
1 2 1
 
2
But ve = 2gR  K.E. = mve  m 2v  mv 2
2 2
et

ve = 2vs
ve 1 2
vs = 49. When satellite is in orbit, K.E. = K = 2 m vc
2
rg

For satellite to escape vc = ve


43.
vB r
 A 
4R
2  K.E. to escape,
vA rB R 1 1
 
2
2
Ke = 2 mve = 2 m 2 vc
 vB = 2  vA = 2  3v = 6v
Ta

44. Force on satellite is only gravitational force, which = 2K 


.…  ve  2 vc 
will always be towards the centre of earth.  Extra K.E. required = 2K  K = K
GMm mv 2 50. By energy conservation,
45.  where, r = R + h
r2 r  K.E. =  P.E.
GM GMR 2 g Using Shortcut 7,
 v=   R
r R 2r r 1  n 
mv2 = mgR  
 9.8  6 2  n 1
 v =    6.38  10
 .25  10 6
 6.38  106  2gRn
 v2 =
1.47 n 1
=  6.38  106
106 2gRn
3
 v=
= 7.76 × 10 m/s = 7.76 km/s n 1

67

MHT-CET Triumph Physics (Hints)


GMm GMm 56. An object of mass m1 placed at the equator of
51. K.E. (K) = and P.E. (V) =
2r r the star, will experience two forces: (i) an
GMm attractive force due to gravity towards the centre
 E=K+V= of the star and (ii) an outward centrifugal force
2r
V
due to rotation of the star. The centrifugal force
K= arises because the object is in a rotating (non-
2
inertial) frame; this force is equal to the inward
GMm centripetal force but opposite in direction. Force
52. Orbital Energy E0 =
2 R  h  on object due to gravity
 GMm  GMm GmM
 E0 = = ….[  h = 2R] Fg =
2  R  2R  6R R2
Force on object is
 GMm
Energy at surface E = Fc = mR2

ns
R The object with remain stuck to the star and not
 Min. energy required = E0  E fly off if
 GMm   GMm  Fg > Fc
=  
6R  R  GmM R 32

io
2
i.e., > mR  M >
5 GMm R2 G
=
6R
57. Orbital velocity of satellite vc = gR

at
53. Total energy of a satellite is,
Escape velocity of satellite ve = 2gR
GMm
T.E. =  ....(i) Minimum increase required,
2(R  h)
 Multiplying and dividing the equation (i) by R2. v = ve  vc = 2gR  gR = gR 2  1  
T.E. = 
GMmR 2
lic
GM
2(R  h)R 2 58. vc =
R h
g 0 mR 2 GM
 T.E. =  .... ( g0 = )
ub
2(R  h) R2
2GM
ve =
R
54. Binding energy of a satellite on the surface of But, 4vc = ve ….(given)
the earth is,
GM 2GM
GMm 4 
B.E. =
P

R h R
R
Binding energy of satellite revolving around the 16GM 2GM

earth at height h is, R h R
et

GMm  8R = R + h
(B.E.)h =
R  h = 7R
B.E. 2(R  h)  
2
 =  R  2
 2R 
rg

(B.E.) h R
59. gh = g   = g  
GM  RR   3R 
55. Gravitational acceleration of earth, g =  2 
R2
4g
Where, M is mass of the earth. gh = ….(i)
Ta

9
As g is independent of mass of the Sun, increase
in G will increase value of g. Hence, statement (vc)h = ghR h
(D) is incorrect. 4g R
Also, terminal velocity of raindrop depends on g = 
9 2
therefore increase in g will cause raindrops to 1
fall faster. = 2gR ….( ve = 2gR )
3
Hence, statement (A) is correct.
1
Increased value of g will make walking on (vc)h = ve
ground more difficult. Hence, statement (B) is 3
correct. 60. dm = dV
Time period of simple pendulum will decrease K
1 dm =   (4r2dr)
as T  . Hence, statement (C) is correct.  r 
g
dm = 4Krdr
68

Chapter 05: Gravitation
R R
GM
M =  dm   4Krdr  vx = 9  ….( vx = orbital velocity)
0 0
2R
R Similarly,
 r2 
M = 4K   m
2  0  vy = m  v
10
M = 2K(R2 – 0) GM
M = 2KR2  vy = 10  u 2 
R
For circular motion, gravitational force will
provide required the necessary centripetal force.  The kinetic energy of the rocket is,
1 m 
   vy    vx  
2 2
GMm mv2 K.E. =
  2 10  
R2 R
m   GM  
2
GM  
2

G  2KR 2  m mv2 =  10  u  2


  9  
v= 20  R   2R  

ns
 2GKR
R2 R  
2R m  GM   GM  
Time period, T = =  100u 2  100    81 
v 20  R   2R  
2R

io
T=  R m  GM  81  2
2GKR =   100    100u 
20  R  2 
 T2  R
m  119 GM 
 100u 2 

at
61. At equator, = 
20  2 R 
g = g – R2
 119 GM 
v g = 5m  u 2  
but  = and g = …(Given)  200 R 
r 3
g v 
2
lic63. FCP = FG
 = g – R 2 
3 R  mv 2 GMm
 2
2g v2 r r
 =
ub
3 R GM
2
v=
 3v = 2gR …(i) r
We know, ve = 2gR 2r
T=
From equation (i), v
P

4 2 r 3
ve = 3 v T2 = ….(i)
GM
62. Let the satellite attains final velocity v. T2 = Kr3 ….(ii)
Therefore, by conservation of energy, 4 2
et

GMm 1 2 GMm 1 2 K=
  mu =   mv GM
R 2 2R 2 GMK = 42
2 2 GM
 v =u – v1
rg

R 64.
GM
 v= u2  ….(i) r2 m
R
r1
vy
Ta

v
m vx 9m v2
10 vx 10 From law of conservation of angular momentum,
vy mv1r1 = mv2r2
2R
v1r1
 v2 = .…(i)
r2
From law of conservation of energy,
M GMm 1 GMm 1
 mv12 =  mv 2 2 … (ii)
r1 2 r2 2
When the rocket is ejected, consider
From equations (i) and (ii),
conservation of momentum along X-axis,
m 9m 2GMr2
  vx =  vx v1 =
10 10  r1  r2  r1
69

MHT-CET Triumph Physics (Hints)


Angular momentum, 68. For the planet to orbit around the star, the
2GMr1r2 centripetal force must be provided by
L = mv1r1 = m gravitational force. Hence, FG = Fa
r1  r2
Fa  r5/2 ....(Given)
g1 5
65.  (ve sign indicates force is towards centre of
g2 2
orbit)
4
G1 R13 Hence, a   r5/2
3  2 r   r5/2  2  r7/2
R12 5
  4 2 7/2
4 3 2   r or T2  r7/2
G2 R 2 T 2
3
R 22 GM GM
69. V=  and g =
R1 5 2 5 1 5 (R  h) (R  h) 2

ns
  =  
R 2 2 1 2 2 4 Taking ratio of both,
v1 g1R1 5 5 |V|
= =  =R+h
v2 g 2R 2 2 4 g
5.4  107

io
v1 5  =R+h
 = 6.0
v2 2 2
 9  106 = R + h
66. (T.E.) on surface = (T.E.) at height ‘h’  h = (9 – 6.4)  106 = 2.6  106 = 2600 km

at
 (K.E.)1 + (P.E.)1 = (K.E.)2 + (P.E.)2
70. Refer Mindbender 5.
1  GMm   GMm 
 mu 2      0  P
2  R   R h 71.
1  GMm   GMm 
mu 2   
lic 2r S
   
2  Rh   R  r
GMm GMm
=  d
ub
R Rh
Here, work has to be done to displace a body
1 1 
= GMm    d
 R R h from distance   to . Let mass of the body be
2
1 1  m and mass of planet and satellite be MP and MS
u2 = 2GM  
P

 
R R h respectively.
 R hR   Total work W = WP + WS
u2 = 2gR2   ….( GM = gR2) 1 1  1 1 
 R  R  h  
et

= GMPm     GMSm   
 h   (d / 2)    (d / 2) 
u2 = 2gR   GM P m GM Sm 2Gm
Rh =  = (MP + MS)
d/2 d/2 d
rg

u2  h  Escape velocity should be such that it can


  
2gR  R  h  perform work W.
R  h 2gR 1 2Gm
  2 i.e., mv e2 = (MP + MS)
Ta

h u 2 d
R 2gR 4 4
1  2 But, MP = (2r)3 and MS = r3(2)
h u 3 3
R 2gR 4G  4 4 
  1  ve2 = (2r)3   r 3 (2) 
h u2 d  3 3 
R 2gR  u 2 4G 4
 =  10  r3
h u2 d 3
u 2R 10Gr 3
 h=  ve = 4
2gR  u 2 3d
M
67. VP = Vsphere + Vpartical a
GM GM 3GM P
=  = M a/2
a a/2 a

70

Chapter 05: Gravitation

Hints to Evaluation Test

ve
1. Me = 20 Mm 6. Given: v = 0.33 ve =
3
Using Shortcut 1,
2 2
Using Shortcut 14,
R 
 
gm M Mm 6400  R R R
 = m  e =   h=  
ge Me R
 m 20M  3200  2
9 1 8
m  ve 
mg m 4   1
 =  v e /3 
mg e 20
4 7. Here, the escape velocity of parcel is to be
 Weight on Mars = 500  = 100 N calculated but, with the launching done from
20
beneath the surface

ns
r3 GM(m) 1 2
2. T = 2   mve = 0
GM R 2
  R
2
  P
4 2
 T2 = (R + h)3 2

io
GM 4GM
 ve = = 2 (11.2 km/s) R
C
1/3 R M
 GMT 2 
 R+h=  2 
= 15.84 km/s.
 4 

at
1 8. Weight of the body at equator
 GMT 2  3 2
 h=  2 
R = of initial weight
 4  5

Gm1m 2
lic 
2
g = g (because mass remains constant)
5
3. F=
r2 g = g  2R cos2θ
6.67  1011  1.9  1027  1.99  1030 2
= g = g  2R cos2 (0)
ub
(7.8  1011 ) 2 5
= 4.14  1023 N 3g
 2 =
5R
Gm1m 2
4. Fg = 3g 3  10
r2   = =
P

and M = m1 + m2 5R 5  6400  103


( m1 and m2 are made from M) = 9.7  104 rad/s
G(m1 )(M  m1 )  2 R 
et

 d
 Fg = 10. g = g 1   g = g 1  ,
r2  R  g 
….[Using product rule of derivation] (gravity at a depth d) (gravity at the equator)
dF G gd g2 R 2 R 2
rg

 = 2 [m1(1) + (M  m1)(1)]    d=
dm1 r R g g
dF
For F to be maximum, =0 1 2 R1 1
dm 11.  , 
Ta

 m1 + (M  m1) = 0 2 3 R 2 2
M g  R  g1  1R1 and g2  2R2
 M = 2m1  m1 =
2 g1 1 R1 2 1 1
     
M g 2 2 R 2 3 2 3
 m2 = M  m1 =  m1 = m2
2
12. According to Kepler’s third law, T2  r3
2 3/ 2
GMm1 m1v T1 r 
5. Fg =   =  1
r n 1 r T2  r2 
GM
 v= 2
r n 1 gh  R 
13.  
2r 2r 2 r g Rh
T=   r n 1  r (n 1) / 2
v GM GM g 1
Given: h 
r n 1 g 100

71

MHT-CET Triumph Physics (Hints)

R 1 18. ve = 2 vc. Hence, if vc becomes 36%, ve will


 
R  h 10 also become 36%
 h = 9R = 9  6400 = 57600 km 36 9
 v e = × 11.2 km s1 = × 11.2 km s1
100 25
14. For a satellite circling around the Earth, the time
(R  h) 2
gd = g  1   ,
d
period is given by T = 2 . 19.
GM  R 
The above equation implies, the time period is R
For d = ,
independent of the mass of the satellite. 2
Hence ratio of time periods is 1 : 1
gd = g 1 
R / 2 g
= = 0.5 g
k k  R  2
15. F=  E= 
r2 r 20. MM = M/10, DM = D/2  RM = R/2

ns
Energy conservation implies, Using Shortcut 1,
K1 + U1 = K2 + U2
M M R2
1 k 1 k k g = g   g M 
 R M  R M 2
2
mv12  = mv22  where v1 =
2 a 2 b 2ma

io
2
and, mv1a = mv2b  M  R  4g
 g  g     
a a k  10M  R / 2  10
 v2 = v1 =  g= 0.4 × 9.8 m s2 = 3.92 ms2

at
b b 2ma
2
1  k  k 1 a  k  k 1 2 GMm 1 2 GMm
 m   = m    22. mv1  mv 2 
2  2ma  a 2  b   2ma  b 2 (R  h1 ) 2 R  h2
1 5 GM GMm 1 3 GM GMm

a
b
a
= 3 or = 1
b
lic 2
m.
7 R2
R
R  h1 2
m.
5 R2
R
R  h2
2 2
16. During total eclipse, total attraction due to sun  h1 = R h2 = R
and Moon, 5 3
ub
GM s M e GM m M e  h1 : h2 = 3 : 5
F1 = 
r12 r2 2 GMm Gm  2 
2

23.   m  R
When moon goes on opposite side, effective R2 4R 2  T 
force of attraction is V
2
Gm  m  2 
P

G MsMe G M m Me 2 
M    m  R
F2 =  R  4  T  M
r12 r22 m M
m 4 R 2 3

 F = F1  F2 =
2GM m M e  M+ = 2
v
et

4 T G
r2 2
m 4  10 8  1033
2GM m  M+ = 
 a = 4 30  10 14 20
r2 2  1011
3
rg

Average force on earth, m 200


F1  F2 GM s M e  10  1030 + =  1030
Fav =  4 15
2 r12 m 10
 =  1030
Ta

GM s 4 3
aav =
r12 40
 m=  1030 kg
 Percentage change in acceleration is 3
a 2GM m r2 2
 100   1  100 4 
a avg r2 2
GM s G   R 3 
Gmm  3 
24. F= =
2
r  M (2R) 2 4R 2
= 2  1  m  100
 r2  M s 4
  2 2 R 4
9
17. Change in energy =
GMm 1
= mv2  F  R4
2R 2
25. T1 = T, T2 = 8T
 Escape velocity is independent of the angle of 2/3 2/3
projection as gravitational field is a conservative  T2  8 T
 R2 = R1   R  = 4R
one.  T1   T 

72
Textbook
Chapter No.

07 Thermal Properties of Matter

Hints

67
T1 =  273 = 385.07 K
Classical Thinking

ns
47.5
T2 = 385.07  273 = 112.07 C
14. 27 C = 27 + 273 = 300 K
29. At constant pressure,
15. tk = 6400 + 273 = 6673 K
V1 V

io
= 2
t c - 0 t f -32 T1 T2
16. =
100 180 Here T1 = 27 C = 300 K

20 - 0 t - 32 T2 = 297 C = 570 K

at
= f
100 180 1 V
 = 2
 tf = 36 + 32 = 68 F 300 570
V2 = 1.9 litre
P1 T1
24. =
P2 T2
lic 30. PV = nRT
T2 (273 + 198) 50  100 = 1RT and 100 V = 3RT
 P2 = P1 = ´1 100V 3RT
T1 (273 + 41)  =
50 ´100 1RT
ub
 P2 = 1.5 atmosphere  V = 150 ml
PV
25. Since = constant 39. L2 = L1(1 + t)
T  50 = L1 (1 + 16  106  65)
P1V1 PV 50 = L1 (1 + 1040  106) = L1 (1.001)
P

 = 2 2 
T1 T2  L1 = 49.95 cm
Here T1 = 27 C = 300 K
P1V1 3P ´ 3V1
40. L2  L1 = L1  (t2  t1)
et

 = 1  0.5  102 = 12  11  106  (t2  10)


300 T2
 t2 = 47.8 C
 T2 = 2700 K
A 2 - A1
43. =
rg

26. At constant volume of a gas


A1 (t 2 - t1 )
P1 P
= 2 A 2 -110
T1 T2 0.000036 =
110(200 - 20)
20 14
Ta

 = 0.7128 = A2  110
273.15 T2
A2 = 110.71 cm2
 T2 = 191.21 K
change in volume
P1 T1 46. =
27. = original volume × change in temperature
P2 T2
0.84
P2 90 ´ 300 =
T2 = .T1 = = 375 K = 102 C 100 ´ 200
P1 72  = 42  106/ C
28. At constant pressure, Q
V1 V 53. c=
= 2 mDT
T1 T2 1200
=
V2 500 ´ (90 -10)
 T2 = T1
V1 = 0.03 cal/g C
73

MHT-CET Triumph Physics (Hints)

84. R1  (1  0) and R2  (2  0) 10tf – 320 = 18tf – 4916.7
R1 k (θ1 - θ 0 ) 8tf = 4596.7
 =
R2 k (θ 2 - θ 0 )  tf = 574.58 F
1.5 30 1
 = 9. PV = constant,  V 
1 (θ 2 - θ 0 ) P
 (2  0) = 20 C. V2 P1
=
V1 P2
64 - 55 9
85. R1 = = P1
10 10 V2 = V1
P2
55 - 42 13
R2 = = 1
10 10 V2 = 60  = 15 cm3
R1 9 / 10 9 4
 = =

ns
R 2 13 / 10 13 10. Comparing with
PV = nRT
86. L1 = L1(1 + 1t1), L2 = L2(1 + 2t2) Here, n = 3
DL1 L (1 + α1t1 ) Hence V represents volume of 3 moles of gas.
= 1

io
DL 2 L 2 (1 + α 2 t 2 )
11. On mixing, n1 + n2 = n
t α P1V1 PV P(V1 + V2 )
For 1 = 2 , 1t1 = 2t2 + 2 2 =
t 2 α1 RT1 RT2 RT

at
DL1 L1 P(V1 + V2 )(T1T2 )
 = i.e., independent of temperature of T=
DL 2 L 2 P1V1T2 + P2 V2T1
rods. P1V1 + P2V2 = P(V1+ V2) (From Boyle’s law)
æ100 + 70
çç -15÷÷÷
ö
lic T=
(P1V1 + P2 V2 )T1T2
R1 è ç 2 ø (85 -15) 7 (P1V1T2 + P2 V2T1 )
87. = = =
R2 æ 70 + 40 ö÷ ( 55 -15) 4
çç - 15 ÷ 13. In summer alcohol expands, density decreases,
çè ø÷
ub
2 so 1 litre of alcohol will weigh less in summer
R1t1 = R2t2 than in winter.
R1 7
 t2 = t1 =  4 = 7 minute 14. Boiling occurs when the vapour pressure of
R2 4
liquid becomes equal to the atmospheric
P

pressure. At the surface of moon, atmospheric


Critical Thinking pressure is zero, hence boiling point decreases
and water begins to boil at 30 °C.
4. t = required temperature then
et

t t - 32 17. A bimetallic strip upon heating bends in the


= form of an arc with more expandable metal (A)
100 180
t t - 32 outside as shown.
=
rg

10 18
18t = 10t  320
 t =  40 C 
B
A B
A
Ta

5. Let reading of celsius scale be x C B


 reading of fahrenheit scale will be 2x F A B
A
t f - 32 t c - 0
 =
180 100 L 2 - L1
18. T2 = + T1
2x - 32 x L1α
 =
180 100 1´10-3
 10x  160 = 9x T2 = + 20 = 0.5  102 + 20
1´2´10-5
 x = 160 C and 2x = 320 F
 T2 = 70 C
t f - 32 t k - 273.15
6. = 19. Using Shortcut 2
180 100
Since tf = tk d2 = d1 [1 + (t2  t1)]
t f - 32 t f - 273.15 = 10[1 + 12  106 (90  10)]
=
18 10  d2 = 10.0096 cm
74

Chapter 07: Thermal Properties of Matter


20. Diameter of sphere = Diameter of ring 33. Initial diameter of tyre = 1000 – 6 = 994 mm,
10.01 [1 + 12  106(t2  10)] so initial radius of tyre R =
994
= 497 mm
= 10[1 + 18  106(t2  10)] 2
1.001 + 1.001  12  106 (t2  10) and change in diameter D = 6 mm so
6
= 1 + 18  106 (t2  10) DR = = 3mm
2
1.001  1 = 18  106 (t2  10)  1.001  12
After increasing temperature by  tyre will fit
 106 (t2  10) onto wheel
103 = (t2 10)  106 [18  12.012] Increase in the length (circumference) of the
103 iron tyre
 t2  10 =  167 γ γ
5.988  106 L = L     = L ´ ´Dθ [As α = ]
 t2 = 177 C 3 3

ns
æγö
2π DR = 2π R çç ÷÷÷Dθ
21. (OR)2  (PR) 2  (PO) 2 çè 3 ø

æl ö
2
él ù
2
3 DR 3´ 3
l 2 - çç ÷÷÷ = [l (1 + α 2 t)]2 - ê (1 + α1t)ú  Dθ = = = 503 C
çè 2 ø êë 2 úû γ R 3.6 ´10-5 ´ 497

io
l2 l2 Nearest answer is (C).
l2 - = l 2 (1 + α 22 t 2 + 2α 2 t) - (1 + α12 t 2 + 2α1t)
4 4 34. Change in volume of flask

at
Neglecting α t and α t
2 2
2
2 2
1
= Change in volume of mercury.
V(3) r = Vt
l2 2α
0 = l 2 (2α 2 t) - (2α1t)  2α 2 = 1  α1 = 4α 2 V (3α)
4 4 V =
23.  = 2
lic γ
2000 ´ 3´ 9 ´10-6
  = 4  106 / C = = 300 c.c
1.8´10-4
A2 = A1 (1+  t) = 0.32 (1 + 4  106  80)
Let the original temperature be 0 C;
ub
A2 = 0.3201 m2 35.
Volume of A = V1 = l  (2r)2 ;
27. Since the expansion of isotropic solids is in all After heating volume of A will become,
directions, on heating the system, x, r, d all V1 = V1(1 + T)
increase.
(V1¢- V1 )
P

28. When a copper ball is heated, its size increases. = T  V1  V1  V1
V1
As volume  (radius)3 and area  (radius)2, so Similarly for rod B,
percentage increase will be largest in its volume.
(V2¢ - V2 )
et

Density will decrease with rise in temperature. = T  V2  V2  V2


V2
29. Due to volume expansion of both liquid and
DV1 l (2r) 2 2
vessel, the change in volume of liquid relative to  = =
rg

container is given by V = V0 [  L   g ] DV2 2lr 2 1

Also, 36. Force developed = AYT


4 4
 g  3 g  3  0.1  10 / C  0.3  10 / C  force is independent of length of the bar.
Ta

 V = 1000 [1.82 × 10–4 – 0.3 × 10–4] × 100 37. The value of specific heat will depend upon
= 15.2 cc nature of substance and will vary for different
substances. Also, it depends on the state of the
30. Water will overflow, both when heated or substance. For example, specific heat of ice,
cooled because water has maximum density or water and steam is 0.5 cal/g C, 1 cal/g C and
minimum volume at 4 C. 0.47 cal/g C respectively.
31. Since, the coefficient of linear expansion of
40. Using Q = mc D T = mc(T1  T2)
brass is greater than that of steel. On cooling,
Q
the brass contracts more, so it get loosened. c=
m(T1 - T2 )
32. Given l1 = l2  l1α a t = l2αs t In this case, T1 = T2 = 100 C
l1 αs l1 αs Q
 =  = . c= =
l2 α a l1 + l2 α a + α s m(0)

75

MHT-CET Triumph Physics (Hints)


Q Total heat required
42. c=
mDT Q = Q1 + Q2 + Q3 + Q4
Q = cmΔT = mci1 + mLf + mcW2 + mLV
For copper, = 1×0.5(10)+1×80+1×1×(100 – 0)+1×540
Q = 420  50  103  10 = 210 calories = 725 cal
For water, Work done W = JQ = 4.2  725 = 3045 J
Q 54. Suppose m g ice is melted, then heat required
ΔT =
cm for its melting = mL = m  80 cal
210 Heat available with steam for being condensed
ΔT = = 5 ºC
4200 ´10 ´10-3 and then brought to 0°C
= 1  540 + 1  1  (100 – 0) = 640 cal
43. Let the final temperature be T °C.
 Heat lost = Heat taken
Total heat supplied by the three liquids in

ns
 640 = m  80  m = 8 g
coming down to 0 C
Short trick: You can remember that amount of
= m1c1T1 + m2c2T2 + m3c3T3 ....(i)
steam (m) at 100°C required to melt m g ice at
Total heat used by three liquids in raising m
0°C is m  .

io
temperature from 0 C to T C 8
= m1c1T + m2c2T + m3c3T ....(ii) Here, m = 8  m = 8  1 = 8 g
By equating (i) and (ii), æ DQ ö÷

at
58. çç  
(m1c1 + m2c2 + m3c3) T çè t ÷ø÷
= m1c1T1 + m2c2T2 + m3c3T3 æ DQ ö÷
çç
m c T + m 2c2T2 + m3c3T3 çè t ÷ø÷ Δθ1
T= 1 1 1 .
m1cl + m 2c2 + m3c3
lic æ DQ ö÷
çç
1
=
Δθ 2
çè t ÷÷ø
2
49. For same mass and material, latent heat is
60 80 - 20
independent of configuration. =
æ DQ ÷ö 40 - 20
ub
çç
50. The latent heat of vaporization is always greater çè t ÷÷ø
2
than latent heat of fusion because in liquid to æ DQ ö÷
çç = 20 cal/s
vapour phase change there is a large increase in çè t ÷÷ø
volume. Hence more heat is required as compared 2
P

to solid to liquid phase change. Q Δθ


59. = K
51. Initially ice will absorb heat to raise its At Δx
temperature to 0 C then its melting takes place. Δθ Δθ 1
 K = constant  
et

Δx Δx K
If mi = Initial mass of ice, mi = Mass of ice that
Hence If Xc = Xm = Xg, then
melts and mW = Initial mass of water æ Δθ ö÷ æ Δθ ÷ö æ Δθ ö÷
Heat gained by ice = Heat lost by water çç ÷ < çç ÷ < çç ÷
çè Δx ø÷ çè Δx ÷ø çè Δx ø÷
rg

 mi  c  (20) + mi  L = mWcW (20)


c m g

(Tg)c < (Tg)m < (Tg)g


 2  0.5(20) + mi  80 = 5  1  20 because higher K implies lower value of the
 mi = 1 kg temperature gradient.
Ta

So final mass of water = Initial mass of water Q KA Δθ


60. =
+ Mass of ice that melts t Δx
= 5 + 1 = 6 kg. Q A d2
  (d = Diameter of rod)
t Dx Dx
52. Ice (–10 °C) converts into steam as follows 2
(ci = Specific heat of ice, cW = Specific heat of (Q / t)1 æd ö Dx 2 æç 1 ÷ö
2
æ1ö 1
= ççç 1 ÷÷÷  = çç ÷÷  ççç ÷÷÷ =
water, Lf = Latent heat of fusion and (Q / t) 2 èç d 2 ÷ø Dx1 è 2 ø è 2ø 8
LV = Latent heat of vaporization)
æ Q ö÷ 2 2
61. çç ÷ = Kπr (θ1 - θ 2 )  r
Q1 Q2
çè t ø÷ Δx Dx
ice ice Q3 Q4
2
–10 °C 0 °C Q1 ær ö æ Dx 2 ÷ö æ 1 ö2 æ 2 ÷ö
 = ççç 1 ÷÷÷ çç ÷ = ç ÷÷  çç ÷ = 1
Water Water Steam Q2 çè r2 ÷ø ççè Dx1 ÷÷ø ççè 2 ÷ø çè 1 ÷ø 2
(0 °C) (100 °C) (100 °C)
Q2 = 2Q1
76

Chapter 07: Thermal Properties of Matter


63. Heat passes quickly from the body into the 80. R1 = 0.5 C/s, R2 = x,
metal which leads to a cold feeling R = k (temp. difference)
Q r2
 R1 = k (50) , R2 = k (30)
64.  ; from the given options, option (B) has  0.5 = k  50
t x
 x = k (30)
r2
higher value of . x k (30)
x  =
0.5 k (50)
dQ KA
66. = ( 1  2)  x = 0.3 C/s
dt x
1 2 81. According to Newton’s law of cooling
dQ KADθ θ1 - θ 2 éθ + θ ù
67. = , For both rods K, A and  are = k ê 1 2 - θs ú
dt l t êë 2 úû

ns
dQ 1 80 - 70 é 80 + 70 ù
same  µ = kê - 40ú
dt l 5 ê
ë 2 úû
(dQ / dt)semi circular l 2r 2 2 = 35 k …(i)
So = straight = = .
(dQ / dt)straight lsemicircular π r π 80 - 60 é 80 + 60 ù

io
= kê - 40ú
t êë 2 úû
68. Let the heat transferred be Q.
20
= 30 k …(ii)
t

at
l l Dividing equation (i) by (ii),
l
t 35
=
When rods are joined end to end. Heat 10 30
transferred by each rod
KAΔθ
lic t=
35
10 = 12 minute
=Q=  12 ....(i) 30
2l
When rods are joined lengthwise, 82. According to Newton’s law of cooling
Rate of cooling  Mean temperature difference
ub
K2ADθ
Q= t ....(ii) Fall in temperature æç θ1 + θ 2 ö
l  µç - θ 0 ÷÷÷
From equation (i) and (ii), t = 3 s Time çè 2 ø
æθ + θ ö æθ + θ ö æθ + θ ö
69.
DQ
=
KAΔθ  ççç 1 2 ÷÷÷ > ççç 1 2 ÷÷÷ > ççç 1 2 ÷÷÷
P

t Δx è 2 ø1 è 2 ø2 è 2 ø3
Thermal gradient  T1 < T2 < T3
Δθ (DQ / At)
= 83. According to Newton’s law of cooling
et

Δx K
Rate of cooling  mean temperature difference.
20
= = 25 C/cm Initially, mean temperature difference
0.8 æ 70 + 60 ö
= ççç - θ 0 ÷÷÷ = (65  0)
rg

74. For same mass, volume and material, rate of è 2 ø


cooling will depend upon area of the body. Finally, mean temperature difference
Smaller the area, lesser will be rate of cooling. æ 60 + 50 ö
= ççç - θ 0 ÷÷÷ = (55  0)
Ta

75. For both spheres, surface area, material and è 2 ø


temperature difference are same hence rate of In second case mean temperature difference
dθ 1 decreases, so rate of fall of temperature
cooling  and msolid > m hollow .
dt m decreases, so it takes more time to cool
Hence hollow sphere will cool fast. through the same range.
78. For -t plot, dQ
84. Using  T,
dθ dt
rate of cooling = = slope of the curve.
dt 60 - 40 2
= k(60  10)  k =
dθ 7 35
At P, = tan 2 = k(θ 2 - θ 0 ) ,
dt 40 - 28 2
where k = constant. = k (40  10) = (30)
t 35
dθ tan 2 θ 2 - θ 0 12 ´ 35
At Q = tan 1 = k(θ1 - θ 0 )  = t= = 7 minutes
dt tan 1 θ1 - θ 0 60

77

MHT-CET Triumph Physics (Hints)

50 - 49.9 é 50 + 49.9 ù æ m ö÷ æ ö
85. = kê - 30ú ….(i) çç ÷ = çç m ÷÷  2
5 ê
ë 2 úû çè t ø÷ èç t ø÷1
2

40 - 39.9 é 40 + 39.9 ù æ m ö÷
= kê - 30ú ….(ii) çç ÷ = 0.1  2 = 0.2 g/s
t ëê 2 ûú çè t ø÷
2

from equations (i) and (ii) t  10 s 91. The plate is made up of anisotropic material with
dθ different coefficients of thermal expansion. Hence,
86. = k(  0) upon heating, plate will not remain circular. Also,
dt
0.2 as coefficients of thermal expansion are in
k= = 0.01/min mutually perpendicular direction, it will become
20
elliptical in shape.
87. According to Newton’s law of cooling,
θ1 - θ 2 éθ + θ ù

ns
= k ê 1 2 - θ0 ú XA
heating
t ëê 2 ûú XB
Case 1:
60 - 50 é 60 + 50 ù
= kê - θ0 ú 92. Let L be the length of each rod.

io
10 êë 2 úû
Temperature of A = 120 C,
1 = k(55  ) ….(i) Temperature of E = 20 C
Case 2:

at
50 - 42 é 50 + 42 ù C (2)
= kê - θ0 ú Q6
10 êë 2 úû x
Q2 x x
0.8 = k(46  0) ….(ii) A Q1
E (20 C)
Dividing equation (i) by equation (ii),
1 55 - θ
lic (120 C) y B
(1)
y
Q4
y
Q5
= Q3
0.8 46 - θ
46  0 = 44 – 0.8 0 D (3)
Let 1, 2, 3 be respective temperatures of B, C,
ub
0 = 10 C
D. If Q1, Q2, Q3, Q4, Q5, Q6 are the amounts of
89. Since the relation between tc and tf is given by heat flowing per second respectively from A to
9 B; B to C; B to D; C to D; D to E and C to E
tf = tc + 32
5 then
P

At tc = 0, tf = 32 F and 0.45 A(120  1 )


Q1 = ,
At tf = 0, L
32  5 0.92 A(1  2 )
tc =  C = 17.7 C Q2 =
et

9 L
1st graph satisfies the above condition. 0.46 A  1  3 
Q3 = ,
Q KADθ L
90. = 0.92A  2  3 
rg

t Dx Q4 =
mL K(πr 2 )Dθ L
= 0.46 A  3  20 
t Dx Q5 = ,
For 1st rod L
Ta

æ m ÷ö 2 0.92 A  2  20 
çç ÷ = K1r1 ….(i) Q6 =
÷
çè t ø x1 L
1

For 2nd rod As Q1 = Q2 + Q3


æ m ö÷ 2 0.46A 120  1  0.92A  1  2  0.46A  1  3 
çç ÷ = K r  
çè t ø÷
2 2
….(ii) L L L
2 x2
120 – 1 = 2(1 – 2) + 1 – 3
K1 x
But K2 = , r2 = 2r1, x2 = 1  41 – 22 – 3 = 120 C ….(i)
4 2 Q2 = Q4 + Q6 gives
 Dividing (ii) by (i) 1  32 + 3 =  20C ….(ii)
æ m ö÷ K1 æ K1 ÷ö 2 Again, Q5 = Q3 + Q4 gives
çç ÷ (2r1 )
2 çç ÷ 4r1
çè t ø÷ x çè 4 ÷ø x
 2
= 4  12 = ´ 12 1 + 22 – 43 = –20C ….(iii)
æ m ö÷ x1 K 1r1 x1 K1r1 Solving (i), (ii) and (iii),
çç ÷
çè t ø÷ 2
1 1 = 60 C, 2 = 40 C, 3 = 40 C
78

Chapter 07: Thermal Properties of Matter


93. Heat delivered by burner in first 10 mins, Differentiating both sides,
H1 = Pt1 3n RT2 dT = K dV ….(ii)
where, P is power delivered by burner. Dividing equation (ii) by equation (i),
Let mass of water in the beaker be m then, 3 dV
dT =
Pt1 = mcT T V
Since settings of burner are unchanged, same dV 3
Coefficient of volume expansion = =
power will be used for evaporation process. V dT T
If t2 is time taken to evaporate the water,
dQ KA 0.01´1
Pt2 = mL 104. = dθ =  30 = 6 J/s
mL mL t1 Lt dt l 0.05
t2 = = = 1 Heat transferred in one day (86400 s)
P mcT cT
 = 6  86400 = 518400 J
2.3  106  10
= = 68.45 min.  68 min Q 518400

ns
4.2  103  (100  20) Now Q = mL  m = =
L 334 ´103
= 1 hr 8 min. = 1.552 kg = 1552 g
96. For cooking utensils, low specific heat is 105. As the coefficient of cubical expansion of
preferred for its material as it should need less

io
metal is less as compared to the coefficient of
heat to raise its temperature and it should have cubical expansion of liquid, we may neglect
high conductivity, because, it should transfer the expansion of metal ball. So when the ball is
heat quickly. immersed in alcohol at 0 ºC, it displaces some

at
volume V of alcohol at 0 C and has weight
97. According to Newton’s law of cooling, the body
W1.
whose rate of cooling is more, its specific heat
will be less. W1 = W0 – V0g

98.
lic
Substances are classified into two categories
where W0 = weight of ball in air
Similarly, W2 = W0 – V50g
i. water like substances which expand on where 0 = density of alcohol at 0 ºC
solidification. and 50 = density of alcohol at 50 ºC
ub
ii. CO2 like (Wax, Ghee etc.) which contract As 50 < 0,  W2 > W1 or W1 < W2
on solidification.
Their behaviour regarding solidification is
opposite. Competitive Thinking
Melting point of ice decreases with rise of
P

temperature but that of wax etc increases with 1. Relation between any two scales can be found
increase in temperature. Similarly ice starts as follows –
forming from top downwards whereas wax ¢ - (Freezing point)¢ Tscale
Tscale ¢¢ - (Freezing point)¢¢
et

starts its formation from bottom. =


(Parts between boiling (Parts between boiling
99. Heat lost in t seconds = mL and freezing)¢ and freezing)¢¢
mL Tx - 40 Ty - (-30)
Heat lost per second = . 
rg

t =
80 160
This must be the heat supplied for keeping the 50- 40 T + 30
substance in molten state per second.  = y
1 2
mL Pt
Ta

 =P  L=  Ty =  10
t m
2. Assuming the graph for a gas of given mass, we
100. Freezing point of water decreases when pressure have,
increases, because water expands on solidification PV = nRT
while “except water” for other liquid freezing point V 1
increases with increase in pressure.  
T P
Since the liquid in question is water. Hence, it V
expands on freezing. From the graph, = tan 
T
nRT 1
103. From ideal gas equation PV = nRT  P =   tan 
V P
nRT 2  as angle  increases, tan  increases and
Given PT2 = K  3
⋅ T = K = nRT = KV pressure decreases.
V
….(i)  P1 > P2

79

MHT-CET Triumph Physics (Hints)


RT ρRT ρ1
3. Using ideal gas equation: P = = 9. 2 =
V M (1 + γDT)
P1 ρ M Fractional changes
 = 1 ´ 2
P2 M 2 ρ 2 ρ1 - ρ 2 ρ 1
= = 1 - 2 = 1  (1 + T)
4 ρ1 3 ρ1 ρ1
 = ´
3 ρ2 2 = 1 (1  T) [  (1 + x )n  1 + nx]
ρ1 8
Thus = = T = 5  104  40
ρ2 9 = 0.020
4. 10. Thermal expansion of water:
15.2 14.6

74.8 75.4
1.04343

Volume of water (103 m3)

ns
30 C 10 C
Case I Case II
In first case:

io
When atmospheric pressure is Pa and barometric
pressure is Pb, pressure difference P1 = Pa  Pb = 1.00013
76 – 74.8 = 1.2 cm 1.00000

at
In second case, let atmospheric pressure be Pa 0 5 10 100 T
and corresponding barometric pressure Pb. Temperature (C)
 Pressure difference P2 = Pa  Pb = Pa  75.4 Graph of Volume v/s Temperature
length of air column in the barometer.
lic
Volumes in both cases will be equivalent to the
11. l2 – l1 = l1 α (T2 – T1)
 V1 = 90 – 74.8 = 15.2 units 6  10 5
and V2 = 90 – 75.4 = 14.6 units. T2 – T1 = = 5.45 C  6 C
1  11  10 6
As number of moles of gas in the barometer
ub
tube is constant, Hence, range of temperature is (25 – 6)C to
P1V1 P2 V2 (25 + 6)C
= i.e., 19 C to 31 C
T1 T2
1.2 ´15.2 P ´14.6 12. When two rods of different materials (1 and
P

= 2 P2 = 1.166 cm
303 283 2) have same difference between their lengths
 Pa = 75.4 + 1.166 = 76.566 cm L1 T1 2
at all temperatures then,   .
7. L = L (1 + t) L2 T2 1
et

i.e. l2 = l2(1 + 2t)  LCu Cu = LAl Al


and l1 = l1(1 + 1t)  88 × (1.7 × 10–5) = LAl (2.2 × 10–5)
It is given,
88  1.7
rg

l2 – l1 = l2 – l1  LAl =


2.2
 l2 – l1= l2(1 + 2t) – l1(1 + 1t)
 LAl = 68 cm
= l2 + l22 t – l1  l11t
 l2 + l2 + l22 t = l1 + l1 + l11 t
Ta

 A  1  l b  1
13. =   =  
 l2 2t = l1 1t A
 0 t  l b  t
i.e., l22 = l11 l b
= + = 1 + 2
8. 1 = 13600 kg/m3, t1 = 0 C, t2 = 50 C l t b t
 = 1.82  104/C
m m 14. Q = mcT
=  V= Q
V ρ  T =
mc
V2 = V1 (1 + T)
1
1 1 i.e., Rise in temperature 
 = (1 + γDT) c
ρ 2 ρ1
….(assuming Q and m to be constant)
ρ1 13600 Amongst the given curves, rise in temperature is
 2 = =
(1 + γDt) 1 + (1.82 ´ 10-4 ´ 50) least for curve C.
= 13477.35 kg/m3 Hence, substance C has highest specific heat.
80

Chapter 07: Thermal Properties of Matter


15. As the heat capacity increases with temperature, Heat gained by ice of Heat gained by ice
the body at 100 C will have more heat capacity mass m to change its m
than that of the body at 0 C. Hence the common + of mass to
temperature from –10 C 2
temperature will be closer to 100 C. to 0 C convert into water
16. Heat gained by water = Heat lost by steam
20 × 1 × (80 – 10) = m × 540 + m × 1 Heat lost by water to
× (100 – 80) = change its temperature
 1400 = 560 m from 15 C to 0 C
 m = 2.5 g 1 m
Total mass of water = 20 + 2.5 = 22.5 g m  10 +  80 = 60  1  15
2 2
17. Let temperature of bodies kept in thermal 60 15
contact be T. m= = 20 g

ns
45
Heat lost by bodies = Heat gained by bodies
m1c1T1  m 2c 2T2  m3c3T3 Heat lost by Heat gained by calorimeter
=T 21. 
m1c1  m 2c 2  m3c3 copper ball and water
 mb cc 1 = mc cc 2 + mw cw 2

io
As bodies are made of same material,
c1 = c2 = c3 = c(say)  (100)(0.1)(T – 75) = (100)(0.1)(75 – 30)
(m  40  m  50  3m  60)  c + (170)(1)(75 – 30)
 T= 10(T – 75) = 450 + 7650 = 8100
mc  mc  3mc

at
T – 75 = 810
270
= =54 C T = 885 C
5
22. mw = 150 g = 0.15 kg
18.
m c t m c t
t = 1 1 1 2 2 2 and c1 = c2
m1c1  m 2c 2
lic The heat required to evaporate ‘m’ grams of water,
Qrequired = mLv ....(i)
250  90  20  5 (0.15 – m) is the amount of mass that converts
t= = 83.7 C
250  20 into ice
ub
19. Mixing A and B:  Qreleased = (0.15 – m) Lf ....(ii)
Heat gained by A = Heat lost by B Amount of heat Amount of heat
 mA cA TA = mB cB TB required = released
 m cA(16 – 12) = m cB (19 – 16)
P

From (i) and (ii),


 4cA = 3cB ….(i) mLv = (0.15 – m) Lf
Mixing B and C :  m (Lf + Lv) = 0.15 Lf
 mB cB TB = mC cC TC 0.15 L f
et

 m cB(23 – 19) = m cC (28 – 23)  m=


Lf  Lv
 4cB = 5cC ….(ii)
Multiplying equation (i) by 4 and equation (ii) 0.15  3.36  105
=
by (3), 2.10  106  3.36  105
rg

16cA = 12 cB and 12 cB = 15 cC  m = 0.0206 kg  20 g


15
 16cA = 15cC  cA = cC 23. Total mass of water = M kg
16
mass converted to ice = m
Ta

Mixing A and C:
 mass converted to steam = (M  m)
 mAcATA = mC cC TC
Heat lost during Heat gained during
m cA(x – 12) = m cC (28 – x) conversion of water to ice = vapourisation
where, x is final temperature of mixture
i.e., mcwi + mLi
15
 cC (x  12) = cC (28 – x) = (M  m) cw s + (M  m) Ls
16
m  1  (t  0) + m  (80)
 x = 20.26 C
= (M  m)  1  (100  t) + (M  m) 540
20. 15 C ice  m(t + 80) = (M  m) (100 – t) + 540M  540m
0 C
m mt + 80m = 100M  100m  Mt + mt +
water water ice 540M  540m
60 g 10 C 720m = 640M  Mt
m m m 640  t
60 g + (water) + (ice)  
2 2 M 720
81

MHT-CET Triumph Physics (Hints)

24. Heat required to melt the ice = 1  80 = 80 cal Where, Rs = Thermal resistivity
Heat required to change the temperature of Q  l1 l 
i.e. ts =   2 
water to 100 C = 11 (100  0) = 100 cal   K1A1 K 2 A 2 
Total heat required Q1 = 180 cal
Q  2l 
Now, heat to be given out for 1 g of steam to  8= ….(i)
condense into liquid Q2 = 540 cal   KA 
As Q2 > Q1, the whole system is not condensed. Case II :
 Temperature remains 100 C. When rods are connected in parallel,
QR p Q 1
25. Heat required to melt ice = mi Li tp =  ….(ii)
   2KA 
= 60  80 = 4800 cal  l 
Heat required to change the temperature of
Dividing equation (i) by equation (ii),
water at 100 C (steam)


ns
8 QR s
= ms cw    
tp  QR p
= 60  1  (100  0) = 6000 cal
 Total heat Q1= 6000 + 4800 = 10800 cal  l   KA 
Now, heat required to condense 60 g of steam
= 2  2  l =4
 KA   

io
Q2 = 60  540 = 32400 cal  tp = 2 s
As Q2 > Q1, whole 60 g of steam does not get
condensed. 28. The amount of heat flow in time t through a

at
Hence, temperature of mixture remains cylindrical metallic rod of length x and uniform
100 C. area of cross-section A with its ends maintained
But Q1 amount of heat will condense M g of at temperatures 1 and 2 is given by
steam, KA(θ1  θ 2 )t

 M=
Q1 10800
= = 20 g
lic Q=
x
Ls 540 where K is the thermal conductivity of the
material of the rod.
Hence, out of 60 g , 20 g of steam is converted
Area of cross-section of new rod
into water.
ub
2
 R  R
2
 mixture contains 40 g of steam and A =    =
120  40 = 80 g of water. 2 4
A
Q KA   A =
26.  4
P

t l
As the volume of the rod remains unchanged
All the four rods are kept at same temperature
Ax = Ax
difference.
where x is the length of the new rod
et

Q A
  A
t l x = x = 4x
A'
Q r2
  Now, the amount of heat flows in same time t in
t l
rg

the new rod with its ends maintained at the same


Hence, the rod to conduct maximum heat, temperatures 1 and 2 is given by
should have largest r and smallest l
K  A / 4  θ1  θ 2  t 1 KA (θ1  θ 2 )t 1
r2 Q = = = Q
i.e., largest ratio 4x 16 x 16
Ta

l
r 2
29. Rate of flow of heat  temperature difference
Ratio is maximum in option (D).
l ....( K, A and x being unchanged)
27. The rods are identical and are of same material, 4 10
 
ie. l1 = l2 = l Q 10
And K1=K2=K ….K = thermal conductivity  Q = 4 J/s
also, A1 = A2 = A
Case I : Q T
30. Using formula, =
When rods are connected end to end (series), t  x / KA 
Q  For first configuration, blocks are arranged in
 
ts R s series combination.
Q Rs x l l
 ts =  = +
 KA KA 2KA

82

Chapter 07: Thermal Properties of Matter


Q T1  T2 Q 0.0005  150  80  30
Thus = ….(i)  
t  l l  t 6
 KA  2KA  Q
 = 30  Q = heat loss per min = 1800 cal
For second configuration, arrangement of blocks 60
resemble parallel combination.
1 36. 1 cm 1 cm
 x  KA 2KA
   = 
 KA  l l
Q  KA 2KA 
Thus = (T1  T2)    ….(ii)
t  l l 
18 C Air –2 C
Dividing equation (i) by equation (ii),
t 2
 =
t 9

ns
2 2 5 cm
 t =  t   9 = 2 s
9 9
The window can be considered to be a series
 dT  combination of three layers, such that,
31.  KA  t = mL,

io
 dx  K1 = K3 = 0.8 Wm–1 K–1, K2 = 0.08 Wm–1 K–1
1 K t A1 = A2 = A3 = 2.6 m2,
K So, 1 = 2
t K2 t1 l1 = l3 = 1 cm and l2 = 5 cm

at
 Equivalent thermal resistance,
32. 100 C Req = R1 + R2 + R3
l1 l2 l3
Cu = + +
K1 A1 K 2 A2 K 3 A3
T
lic 1  102 5  102 1  102
= + +
B Brass 0.8  2.6 0.08  2.6 0.8  2.6
Steel
2  102  5  101
0 C =
ub
0 C 0.8  2.6
    
Q = Q1 + Q2 …  Q  KA  t 0.52 1
  x   = =
0.8  2.6 4
0.92  4(100  T) 0.26  4  (T  0) 0.12  4  T
  Q  18   2  
= 
P

46 13 12  = = 20  4 = 80 W.
t R 1
 200 – 2T = 2T + T  
 T = 40 °C 4
0.92  4  60
et

Q= = 4.8 cal/s 37. The rods are connected in parallel.


46 1 1 1
 In parallel, = +
33. RP R1 R 2
l
rg

But, R = and l is same for both rods i.e.,


KA
l1 = l2 = d
0 C K P  2A  KA K A
30 C 150  = 1  2
Ta

d d d
K1  K 2
 KP =
2
80
6 38. The temperature of the metal will decrease
K = 0.0005 cal/cm s C exponentially with time to 0.
Q 
 KA 39. According to Newton’s law of cooling
t x d
Here, A = area of cross-section of refrigerator  K(  0 )
dt
= 150  80 cm2
where K is constant of proportionality.
x = length of (conductor) refrigerator Integrating
door through which heat is lost 0 t
= 6 cm d
   0 0
 Kdt
 = 30 C
83

MHT-CET Triumph Physics (Hints)



d
t
 285  50 = 195  30
   Kdt
0   0 0
 20 = 90
 t
0 = 45 C
d
   0   0 Kdt  d 
0 44.    k  1  0 
loge (  0) = Kt + c  dt 1
2.303 log10 (  0) = Kt + c  62  50 
   k (62  26)
As 2.303 is constant above equation can be  10 
equated to, log (  0) = Kt + c. 12 1
 k=  / min
10  36 30
40. Newton’s law of cooling can

 loge(0)
 d 
be expressed as,    k   2  0 
 dt  2
loge (  0) = kt + c

ns
50  42 1
which represents a straight  (50  26)
line graph. 0 t dt 30
8  30
 dt = = 10 min
41. According to Newton’s law of cooling, 24

io
70  60  70  60  45. From Newton’s law of cooling,
In first case, =K  30 
5  2  dQ
= K ( – 0)

at
2 dt
K= C/min
35 When the liquid is maintained at  = 57 C by
60  50  60  50  heater of power 30 W,
In 2nd case,  K  30 
30 = K (57 – 27)
t  2 


10 2
 [55 – 30]
lic  K=1 ….(i)
t 35 Also, Q = mc 
10  35 dQ mc d
 t= = 7 min  = = K ( – 0)
2  25 dt dt
ub
As temperature difference is too small,  can be
94  86  94  86  considered as 47 C.
42. = K  20 
2  2  250 103  c   47  46.9 
 = K (47 – 27)
 4 = K [90 – 20] 10
P

4  0.0025  c = 20 K
K=
70  0.0025  c = 20 ….[From (i)]
74  66 4  74  66  20
Also, =   20   c= = 8000 J kg–1 K–1
et

t 70  2  0.0025
8 4
= (70 – 20) 46. By Newton’s law of cooling,
t 70 1   2    2 
 K 1  0 
rg

8 4  50 t
=  2 
t 70
70  8 72
 80  70  = K(75  25)
t= = 12
4  50 5
Ta

10 = K  600
t = 2.8 minutes 1
 K= ….(i)
43. According to Newton’s law of cooling, 60
1  2    2
 k 1  0 
  70  60  = K(65  25)
t  2  t
70  60 1
First   k  65  0   K= ….(ii)
5 t  4 
 2 = k[65  0] From (i) and (ii)
60  54 1 1
Next   k 57  0  
5 4t 60
Dividing (i) and (ii) 60
 t =
5 65  0 4

3 57  0  t = 15 minutes
84

Chapter 07: Thermal Properties of Matter


47. Let mass of each substance be m.  v2 = 8 (0.01  4200  300) + 8 (2  4200)
Total mass mm 212
mix =  
Total volume m  m 1  2 = 8  4200 (3 + 2)
1 2 = 168000
v = 168000  410 m/s
49. Period of pendulum,
L 51. When the piece of ice falls from the height h, it
T = 2 possesses potential energy, mgh.
g
This P.E. is converted to heat energy.
 T L  Q = mgh
But, L = L0(1 +  t) th
1
 T  L0 (1   t) But only of it is absorbed by ice which is
4
As L0 is constant, used to change the state.

ns
 T  (1 +  t)1/2 mgh
Calculating fractional change in time period of   mL
pendulum, 4
T 1 10  h
 = 3.4  105

io
= (t) 4
T 2
For the given pendulum,  h = 13.6  104 m = 136 km
T = 24  60  60 = 86400 s 52. F =  AYT

at
When t1 = 40 C, T = 12 s, = 1.2  105  2.5  106  2  1011  40
T 1 = 240 N
  (40  t0)
T 2
53. When external pressure is applied on the cube,
show correct time.
lic
Where, t0 is temperature at which the clock will
the compression produced in volume is
12 1 V P
 =  (40  t0) ….(i)  ....(i)
86400 2 V K
Similarly , when t2 = 20 C, T = 4 s When heated, the cube will expand through,
ub
4 1 V = V ( T)
 =  (t0  20) ….(ii) V
86400 2  = 3T ....(ii) (  = 3)
Dividing equation (i) by (ii), V
12 (40  t 0 ) Hence, equating equations (i) and (ii),
P

=
4 (t 0  20) P
3T =
 3t0  60 = 40  t0 K
 t0 = 25 C P
et

 T =
Substituting it in equation (i), 3K
12 1
=  (40 – 25) 
86400 2 54. Q = KA   dt
 x 
rg

12  2
 = Now, Q = mL
15  86400
 
= 18.5  106 = 1.85  105 /C  mL = KA   dt
 x 
Ta

50. When a bullet is fired it has KA  0   26   dt


1  mL =
K.E. = mv2 x
2
 0   26  
This K.E. is converted into heat energy, out of  (A dx)L = KA  dt
1 x
which th of heat is absorbed hence remaining dx 26K
4  
energy is used to melt the bullet. dt xL
3 1 2 55. For two identical ice blocks,
  mv  = mc + mL
4 2  m
3 2
m1 = m2   =
 v = c + L 2
8 Relative velocity (vrel) = u1 – (–u2)
3 2 = u1 + u2
 v = 0.03  4200  (600  300) + 6  4200
8 = 2u
85

MHT-CET Triumph Physics (Hints)

1 dQ
 K.E. =  v 2rel 56. = 50% of input P
2 dt
1 m dQ 15  103
=   (2u)2  Pout = = W
2 2 dt 2
2
= mu dQ d
Also this K.E. is used to completely melt both Also, = mc
dt dt
the blocks.
15  10 3
d
 mu2 = Q = (mL + mc )  2  = 10  0.91  103 
2 2  60
 u2 = 2 (L + c)
= (2  3.36  105) + {(2  2100  [0 – (–8)]} 15  103  2  60
 d = = 98.9 C
= 2  (336000 + 16800) 2  10  0.91  103
= 705600

ns
 u = 840 m/s

Hints to Evaluation Test

io
1. In steady state the quantity of heat absorbed and 4. Coefficient of linear expansion for brass
quantity of heat radiated is same. (1.8  105 C) > coefficient of linear expansion

at
for steel (1.1  105 C). On cooling the disc
2. According to Newton’s law of cooling, shrinks to a greater extent than the hole and
1  2     hence it will get loose.
= K  1 2  0 
t  2 
where, 0 = tempeature of surrounding
lic 5. Let the temperature of junction be 
 Q   Q 
  =  
60  50  60  50   t copper  t steel
 = K  30 
8  2 
100    = K 2 A    2.5 
ub
10 K1A
= K × 25 ….(i) 20 5
8
9 K2
    = K ( – 2.5)
100
( K1 = 9K2)
After another 20 min, let the temperature be . 4
2

50    50    900 – 9 = 4 – 10
P

 = K  30  ….(ii)
20  2   13 = 910
50   10  50      = 70  C.
=  30  using (i)
20 8  25  2 
et

6. Density of water is maximum at 4 C. In both


50   1  50    60  heating and cooling of water from this
=
20 20  2 
 temperature, level of water rises due to decrease
in density, i.e., water will overflow in both A
rg

2(50 – ) = 50 +  – 60 and B.
100 – 2 = –10 + 
3 = 110 7. If l is the original length of wire, then change in
length of first wire, lA = (lA – l)
Ta

110
= = 36.67 C. Change in length of second wire, lB = (lB – l)
3
Now Young’s Modulus,
3. Let ‘m’ grams be the mass of the steam. TA l T l
Y=  = B
Heat lost by the steam = m  L A lA A lB
+ m  1  (100 – 0) TA T TA T
  B   B
= m × 540 + 100m lA lB lB  l lB  l
= 640m  TAlB – TAl = TBlA – TBl
Heat gained by ice = mi  c  T + miL TBlA  TAlB
 l=
= 1600  0.5  [0 – (–8)] + 1600  80 TB  TA
= 134400 cal. 8. Increase in volume of flask
According to principle of calorimetry, = 40 ×10–6 × 4000 × 80
640m = 134400  m = 210 g. = 12.8 cc
86

Chapter 07: Thermal Properties of Matter


Increase in volume of mercury dQ KAθ
16. Using = ,
= 180 × 10–6 × 4000 × 80 = 57.6 cc dt Dx
 Volume of mercury overflow dQ Dx
= 57.6 – 12.8 = 44.8 cc  = 
dt KA
9. Using standard gas equation, 6000×1
=
P1V1 PV 200× 0.75
= 2 2   = 40 C
T1 T2
P1V1T2 Cp 3 C +R 3
V2 = 18. =  v =
P2T1 Cv 2 Cv 2
1  600   273  13 This gives Cv = 2R, and hence Cp = 3R
=  629 m3
0.8   273  37  20. Heat lost by hot ball = Heat gained by water

ns
10. Colour is an indication of temperature of the m1  c1 (t2  t0) = m2  c2 (t0  t1)
body. If two pieces of same substance appear of 200  0.08  (t  22.8) = 500  1  (22.8  10)
different colours, then their temperatures must  t = 422. 8 C
be different. In this case, TA < TB

io
11. Number of moles of gas in two flasks are
P1V1 PV
n1 = and n2 = 2 2

at
RT RT
 n = n1 + n2
 P =
 n1  n 2  RT = P1V1  P2 V2
V1  V2 V1  V2

13.
lic
Fahrenheit scale and Absolute scale are related
as
TF  32 TK  273  15
= ….(i)
ub
180 100
For another set of temperature TF and TK,
TF   32 TK   273 15
= ….(ii)
180 100
P

Subtracting (i) from (ii)


TF   TF T   TK
= K
180 100
et

180
TF-TF =  TK    
100
180 9
If TK – TK = 1 K then, TF  TF = 1 =
100 5
rg

For a temperature of triple point i.e., 273.16 K,


the temperature on the new scale is
9
= 273  16   491.69
Ta

5
14. At absolute zero temperature, pressure P of gas
would reduce to zero. The volume V of the gas
would also become zero. If we were to imagine
going below this temperature, volume of gas
would be negative, which is impossible. That
suggests that the lowest attainable temperature
is absoulate zero.
At absolute zero, the translatory motion of
molecules ceases but other forms of molecular
energy (like inter molecular potential energy) do
not become zero. Therefore absolute zero
temperature is not the temperature of zero-
energy.
87
Textbook
Chapter No.

08 Sound

Hints

E
Classical Thinking 55. v=

 E = v2 = (4  103)2  2200 = 3.52  1010 N/m2

ns
Distance travelled by wave (λ)
14. Velocity of wave =
Time period (T)
P 0.76  13600 9.8
Wavelength is also defined as the distance 56. v= = = 279.9 m/s
 1.293
between two nearest points in phase.

io
v 3  108 P 1.36  0.76  13000  9.8
15. n= = = 1.5  106 Hz 57. v= =
 200  1.29
v 340 v = 319.5 m/s

at
16. 1 = = = 17 m,
n1 20
58. At constant temperature, change in pressure has
340 no effect on velocity of sound.
2 = = 17  103 m
20  103
va 340
lic 60. T = 27 °C = 27 + 273 = 300 K
17. a = = = 3.40  103 m v T 300
n 100  103 = = v0
v0 T0 273
v 1486
w = w = = 1.486102 m v  1.05v0
ub
n 100  10 3

 2 2t  62. v  v0 = (0.61) t


36.  y = a cos  t   = 0.5 cos (4t + 2x)
    = (0.61)  10 …(Using Shortcut)
= 6.1 m/s
37. Compare with y = a sin (t  kx)
P

2 2 64. For constant temperature and pressure


we have k = = 62.4   = = 0.1
 62.4 conditions,
1
et

38. Compare the given equation with v


y = a sin (t + kx)   = 2n = 100 M
50 M is least for hydrogen, hence hydrogen has
n= Hz greatest velocity.

rg

2 77. If the observer is receding from a stationary


k= = 1   = 2 and  = /k = 100 m/s
 source, then
Since ‘+’ is given between t terms and x term,  v  v0 
Apparent frequency =  n
Ta

so wave is travelling in negative x-direction.


 v 
39. A wave travelling in positive x-direction may be
2 83. n = actual frequency of sound produced by
represented as y = A sin (t  x). On putting source

2 v = speed of sound in given medium
values y = 0.2 sin (360 t  x) vs = speed of source of sound
60
vl = speed of observer
 x 
 y = 0.2 sin 2  6t   As source of sound and the listener are both
 60 
moving in the same direction,
40. Positive sign in the argueent of sin indicating  v  vl 
that wave is travelling in negative x-direction.  n = n  
 v  vs 
T 25 n v  vs
54. v= = = 5 m/s  =
m 1 n v  vl

88

Chapter 08: Sound


v 1.7  103  320 1 = 340 (1  0.06)
85. = = = 4  104 m
n 4.2  106 320 1 = 340 1  20.4
201 = 20.4
86. v = n = 500  0.1 = 50 m/s
1 = 1.02 m
Distance s
velocity = =  v = 320  1.02 = 326.4 m/s
time t
s 300 15. Since maximum audible frequency is 20,000 Hz,
 t= = =6s v 340
v 50 hence  min    20 mm
n max 20,000
Distance between crests 9
87. = = = 0.9 m 16. The minimum distance between compression
No.of crest 10

v and rarefaction of the wire l 
v = n n= 2

 Wavelength  = 2l

ns
450 360
n= = 500 Hz v = n  n  = 180 /s.
0.9 2 1
v T 17. Time lost in covering the distance of 2 km by
89. =

io
v0 T0 d 2000
the sound waves t =   6.06 s  6 s
v 330
v T
 = 1.5 = v 352
v0 273 18.   ; during 1 vibration of fork sound

at
 T = 273  (1.5) 2 n 384
= 614.2 K 352
will travel m
= 614.2  273 = 341.2 °C 384
 during 36 vibration of fork sound will travel
Critical Thinking
lic 352
 36  33 m
384
5. If the temperature changes then velocity of
wave and its wavelength changes. Frequency, 19. Surface waves are transverse.
ub
amplitude and time period remains constant. 21. Distance between two successive crests
==2m
6. Velocity of sound is independent of frequency.
Therefore it is same (v) for frequency n and 4n.  Distance between a crest and trough

= =1m
P

7. Period, T = 0.14  4 = 0.56 s 2


1 1 Y
Frequency = = 23. By comparing given equation of progressive
T 0.56
wave with standard equation y = a cos (kx  t),
100
et

= = 1.79 Hz we get,
56 Time 2 2
k= == = 25
(0,0)  0.08
0.14 s
2 2
rg

and  = == =
48 8 T 2
8. n= = waves/s.
60 10 24. A comparing with y = (x, t) =  sin (t  kx)
8 2
v = n =  10 = 8 m/s = 0.01    = 200 m.
Ta

10 K=

9. From the figure, wavelength  = 0.4 cm. 25. y = A sin(t + bx + c) represents equation of
n = 250 Hz (given) simple harmonic progressive wave as it
 v = n = 250  0.4 = 100 cm s–1 = 1 m s–1. describes displacement of any particle (x) at any
10. Distance between crest and nearest trough time (t) or it represents a wave because it
 2y 2  y
2

= = 2.5   = 5 cm satisfies wave equation  


2 t 2 x 2
n = 4 /s 28. According to Laplace, the changes in pressure and
 v = n = 4  5 = 20 cm/s volume of a gas, when sound waves propagate
through it, are not isothermal, but adiabatic. A gas
11. 1  2 = 0.06 is a bad conductor of heat. It does not allow the free
2 = 1  0.06 exchange of heat between compressed layer,
v = n11 = n22 rarefied layer and surrounding.
89

MHT-CET Triumph Physics (Hints)


29. The speed of sound in gaseous medium is given 1 1
v 
by, m A
P vA AB 1
v ….(i) = =
 vB AA 2
At constant temperature vA  1
PV = constant ….(ii) = A =
vB B 2
If V is the volume of one mole of a gas, then
M M 35. If T is the tension corresponding to extension x,
density of gas  = V= then T = Kx.
V 
Where M is the molecular weight of the gas. If T is the tension corresponding to extension
 equation (ii) becomes, 1.5x, then T = K(1.5)x
PM T
= constant v=

ns
 m
P v T K(1.5)x
 = constant as M is a constant = = = 1.5 = 1.22
 v T Kx
From equation (i), v = 1.22 v

io
v = constant   vH  H M He  7 / 5 . 4 42
Thus, change in air pressure does not effect the 36. = . = =
v He M H  He 2  5 / 3 5

at
speed of sound.
N
37. Since v  T
v He 214
30.  2
= = 7 :1  v1  T1 and v2  T2
vN He 4

5
2

1
lic Also
v2
v1
=2
31. m= kg/m = kg/m
20 4 v2 T2 T2 T2
Tension in the middle of wire  = = =
v1 T1 273  27 300
ub
T = weight of half the wire
5 5 T2
= kg =  10 N = 25 N  2 =
2 2 300
T2
v=
T
=
25
= 10 m/s  4=
P

m 1/ 4 300
 T2 = 4  300 = 1200 K
T  T2 = 1200  273
32. v=
m  T2 = 927 °C
et

M γP
where, m = mass per unit length = 38. Speed of sound, v =
L 
TL 77 × 5.5
rg

v= = = 110 m/s v1 ρ2
M 0.035 = ( P constant)
v2 ρ1
T 1
33. v= 39. vsound 
Ta

m 
M V AL
m= = = = A v1 2 4
L L L    2
v2 1 1
T
v= v1 vs
A  v2 = 
2 2
103
= = 106 40. At given temperature and pressure
10  106  100
1 v  4
 v = 1000 m/s v  1 2   2 :1
 v2 1 1
T
34. v= T2 v 2 v 
2

m 41. v T   T2  T1  2 
AL T1 v1  v1 
Let m = = A
L  T2 = 273  4 = 1092 K

90

Chapter 08: Sound

RT  IC 
42. v=  n = 10 log10   = 10 log10 4 = 10 log10 4
M  IA 
v  T = 10  0.6 = 6
v2 T2
= I1 4 a2
v1 T1 52. = = 2
I2 1 b
T  600
=
T
= 3

I max
=
 a  b  =  2  1 = 9
2 2

T = 300 K = 27 C
I min  a  b 2  2  12
43. Speed of sound in gases is given by Imax I
Now, L1 – L2 = 10 log  10 log min
I0 I0
RT
v= Im ax
M = 10 log = 10 log 9

ns
1 Imin
v , L1 – L2 = 10 log 32 = 20 log 3
M
v1 M2 53. v = 340 m/s,
=

io
v2 M1 km 72  103
vs = 72 = m/s = 20 m/s
hr 3600
γRT
44. v= vn 340  640
M Using, n = =

at
v  vs 340  20
TN M
= N 340  640
T0 M0 = = 680 Hz
320
TN 14 7
273 + 55
=
16
=
8
lic 54. v0 = 720 km/hr = 200 m/s
TN = 287 K = 14 C  v  vo 
Using, n =   n
1 v M O2 v 4  v  vo 
32
45. v  H2    H2   340  200 
ub
v O2 1 140
M v O2 M H2 2  n =  n=  1080 = 280 Hz
 340  200  540
RT
46. Velocity of sound in gas v  v  vo 340  60
M 55. n = n=  133
v  vs 340  60
P

T  n = 190 Hz
v
M
56. There is no relative motion between source and
7
4 listener.
et

v N2  N2 M He 5 3
    
v He  He M N2 5 5 57. Let n = actual frequency of sound produced by
 28
3 source.
 v  vl 
rg

47. Intensity  (amplitude)2  n = n  


Let the constant amplitude of each wave be ‘a’.  v  vs 
At the minimum loudness, amplitude is zero n v  vs
(ideally)  =
n v  vl
Ta

 Ratio = 0 : a = 0
48. The loudness is higher if area of the vibrating  v 
58. n =   n
body is more. The hollow boxes are set into  v  vs 
forced vibrations along with the strings. Thus  v  vn  vn  vs n
providing higher area of vibrating body and  n – n =  n–n=
increase in the loudness of sound.  v  vs  v  vs
n  n vs 25 1
49. Loudness depends upon intensity while pitch  = = =
n v  vs 100 4
depends upon frequency.
 4vs = v – vs  5vs = 332  vs = 66.4 m/s
51. If a is amplitude of sound from A and B each,
then 5
59. v = 108 km/hr = 108  = 30 m/s
IA = IB = ka2, where k is constant. 18
Loudness due to C (i.e., A + B) If observer moves towards stationary source,
= IC = k (2a)2 = 4IA then the apparent frequency
91

MHT-CET Triumph Physics (Hints)


 v  vo  n 'v The apparent frequency, when observer is
n =  n  n = moving away from the source is
 v  v  vo
504  330 504  330  300  v 
 n= = = 462 Hz n2 =  n
330  30 360  300 
According to given question,
 v  vo 
60. n =  n 2
n1 – n2 = n
 v  vs  100
1  v  vo  1  1 300  v 300  v 2
 =   ....  n    – =
T  v  vs  T  T  300 300 100
 2v = 2  3  v = 3 m/s
1  340  20  1 360
 =  =
T  340  20  10 3200  v0   40 
68. n = n  1   = 1000 1   = 1121 Hz
3200

ns
 v   330 
 T = = 8.9 s
360
 vs  v 0  n
61. Since there is no relative motion between the 69. n =   n=
source and listener, the apparent frequency  v s  2

io
equals original frequency.  2vs  2v0 = vs or 2v0 = vs
62. Frequency of the note reflected by the wall is n1  vsound = 2v (If v = velocity of observer)
 v  70. 20 m/s
=n 

at

 v  vo 
S L
 Frequency of the note heard by the engine driver 15 m/s
will be
n = 500 Hz, vS = 20 m/s
n =
(v  v o )
v
n1 =
v  vo
v

nv
v  vo
lic vL = 15 m/s, v = 335 m/s
 v  vL   335  15   350 
 v  vo   n = n  = n  = n 
= n  v  v S   335  20   315 
 v  vo 
ub
315 315  500
 340  60   n= n = = 450 Hz.
350 350
=   1400 ….[ n = 1400 Hz]
 340  60 
71. If the motion of a source is along a direction
400
=  1400 = 2000 Hz other than when they are in a straight line, the
P

280
component of velocity of source vscos is along
63. When source is moving towards listener, the line joining the source and listener. In this
v n 300  600 situation, apparent frequency na is not constant
n1 = = = 1800 Hz
and it depends on . It is given by
et

v  vs 300  200
When source is moving away from listener,  v 
na =  n
vn 300  600  v  v cos  
n2 = = = 360 Hz S

v  vs 300  200 Here, v A  72 km / hr  20 m / s and


rg

 Change in frequency = n1 – n2 = 1800 – 360 v B  36 km / hr  10 m / s


= 1440 Hz
 v  v B cos 45o 
 n'  n o 
 v  ul   vv/ 2
Ta

64. n =  n= 
3v / 2  v  v A cos 45 
n= n
 v  us   vv/2 v/2
n
 n = 3n  = 3
n vA
A
n  n 45°
 =2
n vA cos 45° vB
n  n
 Percentage change =  100
n B
= 2  100 = 200 %
45°
65. The apparent frequency, when observer is
vB cos 45°
approaching source is
 300  v   340  10 / 2 
n1 =  n  n = 280   = 298 Hz
 300   340  20 / 2 

92

Chapter 08: Sound


72. For an observer approaching a stationary source, v
v
 v  v0  v  v0 5 .f  6 f  1.2f
n   n 77. n = n
 v  v v 5
 an   Source is stationary, wavelength remains
v0 = at  n   tn unchanged for observer.
 v 
This is the equation of straight line with positive 78. Wave number is the reciprocal of wavelength
n and is written as  or n .
intercept n and positive slope   .
 v
54 waves 9
120  v  79. n= = /s
73. In first case, n=  n 60 second 10
100  v  vS  1 1
v = = = 10 metre
 1.2 = or vs = v Wave number 0.1

ns
v  vS 6 9
 v   Velocity = n =  10 = 9 m/s
 v  10
In second case, n =  n =  n
 v  vS   v v/6
82. The time taken by the stone to reach the lake

io
6n
 n =  2h   2  500 
7  
t1 =    10 s
 6n   g   10 
n  

at
n  7  100 1
  100 =  100 =  14.3% (Using h  ut  gt 2 )
n n 7 2
 v  n' v v Now time taken by sound from lake to the man
74. n  n     h 500
 v  v S 

2v
n v  v S v  vS
lic t2 = 
v 340
 1.5 s

 3  vS   Total time = t1 + t2 = 10 + 1.5 = 11.5 s


3
75. Frequency of sound heard by the man from 83. Amongst three, only wave created by an
ub
approaching train oscillating electric field is a transverse wave.
 v   320  Hence can be polarised.
na = n   = 240   = 243 Hz
 v  v s   320  4  84. Let epicentre be located at distance d then,
Frequency of sound heard by the man from d = vPtP = vStS
P

 v  where vS and vP be speed of S-wave and


receding train nr = n  
 v  vs  P-wave respectively.
Also, tS – tP = 4 min = 240 s
 320 
et

= 240   = 237 Hz v
 320  4   tS  S t S = 240
Hence, number of beats heard by man per second vP
= na  nr = 243  237 = 6  4.5 
rg

t S 1   = 240
Alternate method :  8 
2nvvs
 Number of beats heard per second = 240  8
v 2  vs2  tS = = 548.57 s
3.5
Ta

2nvvs 2  240  320  4


= = =6  d = vS tS = 4.5  548.57
(v  vs )(v  vs ) (320  4)(320  4)
= 2468.6  2500 km
76. n1 = Frequency of the police car’s horn heard
by motorcyclist 2h 2  10 1
85. Time of fall =  
n2 = Frequency of the siren heard by g 1000 50
motorcyclist. In this time number of oscillations are eight.
v = Speed of motor cyclist 1
330  v 330  v So time for 1 oscillation =
n1 =  176 and n2 =  165 8 50
330  22 330
Frequency = 8 50 Hz  56 Hz
 n1  n2 = 0
330  v 330  v 86. Interference, diffraction and reflection occurs in
  176 =  165
308 330 both transverse and longitudinal waves.
 v = 22 m/s Polarisation occurs only in transverse waves.

93

MHT-CET Triumph Physics (Hints)


88. Suppose the distance between two fixed points 96. Let the distance of the cliffs be x1 and x2.
d If x2 > x1, then
is d then t = also v  T
v 2x 2
= 3.6  x2 = 612 m
t1 v 2 T2 340
   2x
t 2 v1 T1 And 1 = 1.2  x1 = 204 m
340
2 303
   t2 = 1.9 s Total distance = x1 + x2 = 816 m
t2 283
97. Given,  = 62.5 cm. Phase difference of 2
VO2 O2  VH2 H2 occurs after a path difference equal to . Hence
89. Density of mixture = mix 
VO2  VH2 a phase difference of  will occur after a path

=

V O2   H 2  O2  H2
[since VO2  VH2  V ]
difference of
62.5
2
cm = 31.25 cm

ns
2V 2
H2  16H2 1 98. Waves on the surface of water are transverse.
=  8.5H2  v  These waves spread out till the ocean waves
2 
reach the beach shore. The radius of curvature
H2 H2

io
Vmix 2 of these waves becomes so large that they can
   
be assumed plane waves.
VH2 mix 8.5H2 17
99. The smallest change in sound intensity that can
90. Mass per unit length

at
be detected by human ear is 1 dB.
M 2.5
m= = kg / m 100. The figure (A) does not represent periodic
l 20
motion, as the motion neither repeats itself nor
T 00
v=
m
=
2.5 / 20
lic comes to its mean position.
The figure (B) represents the periodic motion
200  200 200 with period equal to 2 s.
v= = = 40 m/s
25 5 The figure (C) does not represent periodic
motion, because it is not identically repeated.
ub
l 20
Time taken, t = = = 0.5 s The figure (D) does not represent periodic
v 40
motion, because it does not have constant
91. A motor boat sailing in water produces ripples. period.
Distance
P

92. Velocity = 101. The frequency of note produced by the whistle


time is not equal to 1/20 or 0.05 Hz. 0.05 Hz is only
Assuming, the man is standing at a distance ‘x’ the frequency of action of blowing whistle.
from one cliff. From given data, frequency of note cannnot be
et

 The sound must be reflected from first cliff. calculated.


 Distance travelled is 2x
B
2x 102. For water, v =
 v= = 2x = 340 
rg

1
340 2100  106
 x= = 170 m =
2 103
= 1449.14 m/s
Ta

93. The sounds from different sources are said to


differ in quality. The number of overtones and 103. From Laplace’s formula,
their relative intensities determine the quality of
v P.
any musical sound.
If pressure is doubled then, v  2P .
94. Distance covered by two complete waves v 2P
= 2   = 2  5 = 10 cm. Taking ratio,   2
v P
This is the distance covered by the wave in one
second.  v = 2v
 v = 10 cm s1 104. At /2 of distance, phase of a particle reverses.
95. The frequency of sound of the source is If a particle at a point has displacement 2 cm,
independent of the medium. Hence, frequency at the same time, another particle 15 cm (/2)
of sound recorded by the observer will remain away from the first particle will have
same, that is, 512 Hz. displacement 2 cm, due to phase reversal.

94

Chapter 08: Sound


105. Time taken to cover distance of 332 m is,  v   320 
distance 332 na = n   = 240   = 243 Hz
t= = =1s  v  v s   320  4 
speed 332
Frequency of sound heard by the man from
The tuning force has frequency 512 Hz. Hence receding train,
in time t (i.e., 1 s) it will vibrate 512 times.
 v 
nr = n  
106. After two seconds each wave travels a distance  v  vs 
of 2.5  2 = 5 cm i.e., the two pulses will meet
= 240 
320 
in mutually opposite phase and hence the  = 237 Hz
 320  4 
amplitude of resultant will be zero.
Hence, number of beats heard by man per second
 n a  n r  243  237  6

Alternate method:

ns
Number of beats heard per second
107. A tuning fork is made of a material for which 2nvv 2nvvS
elasticity does not change. Since the alloy of = 2 S2 =
v  vS (v  vS )(v  vS )
nickel, steel and chromium (elinvar) has

io
constant elasticity, therefore it is used for the 2  240  320  4
=
preparation of tuning fork. (320  4)(320  4)
=6

at
1 2
108. s = gt
2
Competitive Thinking
1
 78.4 =  9.8  t2
2 2. As frequency is same in both the medium,
 t=4s
lic speed  
Sound of splash is heard (4.23  4) s = 0.23 s
3. Distance between a compression and the nearest
later at the ground level after travelling 78.4 m.

The velocity of sound v is rarefaction is 1m.
ub
Distance 78.4 2
v= = = 340.9 m/s v 360
time 0.23 Hence n    180Hz .
 2
nv
109. no = ....(n = 1, 2, 3,……) 4. Given equation of the wave can also be written as,
2L
P

nv   t x  
nc = ....(n = 1, 3, 5,……) Y = 3 sin  2    
4L   6 10  4 
For the first overtone, n = 2, for open pipe and  
Comparing with y = A sin  2     
t x
et

n = 3, for closed pipe   T   4


3v c 2v o (where, x and y are in metre)
 =
4L c 2L o we get,
rg

Lo 4 v 1
 =  o A = 3 m, n= = 0.17 Hz,  = 10 m and
Lc 3 vc T
vo v = n = 1.7 m/s
c
Also, = Hence, option (D) is correct.
Ta

vc o
5. y = A sin (kx t) ….(i)
4L 1
 Lo = 2 2
3 2 Now, k =  =1
 2
110. 1
Stationary observer  = 2n= 2   2

S A=1m
S Substituting these in equation (i),
vS
 y = sin (x  2t)
vS
Receding train Approaching train 6. In case of a sound wave, the reflection at a rigid
boundary will take place with a phase reversal
Frequency of sound heard by the man from of  but the reflection at an open boundary takes
approaching train, place without any phase change.

95

MHT-CET Triumph Physics (Hints)


 The reflected wave is  R T1
64 v1 M
y= A0 sin (t + kx + ) 13. 
100 v2  R T2
64 M
= A0 sin (t + kx)
100 v1 293

7. Comparing the given equation v2 313

ω 600 313
8. v=  = 300 m/sec v2 = 344.2 × = 355.75  356 m/s
k 2 293

9. Equation of the harmonic progressive wave T


given 14. Velocity of transverse string v = , where,
m
y = a sin 2(bt  cx) m is linear density.

ns
Here 2v =  = 2b  v = b Tension T = Mg = mxg
2π 1
k= = 2c  = c  v=
mxg
λ λ
m
1

io
(Here c is the symbol given for and not the dx
λ  xg
dt
velocity)
For string of length L, integrating over,
1 b

at
 Velocity of the wave = v = b = L
dx
t
c c
 xg 0
 dt
dy 0
= a2b cos 2(bt  cx) = acos(t  kx) t L
dt 1
  dt = x
1/ 2
dx

b
lic
Maximum particle velocity = a = a2b = 2ab 0 g 0
20
given this is 2  1  x1/2 
c  t= 1 / 2  ( L = 20 m)
g  0
2 1
i.e., 2a = or c =
ub
c πa 2
=  20 = 2 2 s
10
P
10. v= 16. Apparent frequency for source moving towards
He
the stationary observer is given by,
P

hg  v 
= n = n 
 He 
 v  vs 
0.76  13600  9.8 As the source moves towards the observer,
et

=
0.179 frequency increases, hence wavelength decreases.
 752 m/s
17. Apparent frequency is given by,
RT  V ± V0 
rg

11. Speed of sound in gases is v = F =  F


M  V  VS 
Hydrogen and oxygen being diatomic have  source is stationary,
same value of .  VS = 0 ; V0 = V1
Ta

Also v is constant,  V + V1 
TM  F1 =  F
 V 
TH2 M H2
 =  V  V1 
TO2 M O2 F2 =  F
 V 
TH2 2
= F1 V + V1
(273  100) 32  =
F2 V  V1
TH2 = 23.3 K = 249.7 C V + V1
 2=
1 V  V1
12. Velocity of sound wave v  where, M is  2V – 2V1 = V + V1
M
weight of gas.  V = 3V1
Hence, smaller the molecular weight of the gas, V
 =3
greater is velocity of sound in that gas. V1

96

Chapter 08: Sound


18. Apparent frequency heard by observer while  v  330
moving towards the source of sound is, n   n =  800  838 Hz
 v  v s  330  15
 v  v0 
n = n   As listener is stationary, he will hear sound of
 v  same frequency after reflection.
Apparent frequency heard by observe while
26. the expression for apparent frequency is
moving away from the source is,
 v  vo 
 v  v0  n = n  
n = n    v  vs 
 v 
n 2nv0 the frequency received by the wall from moving
 n  n = (v + v0 – v + v0) = car is
v v
 330  0 
nwall = 620   = 660 Hz
n approaching v  vs  330  20 
19. Using 

ns
n receding v  vs this frequency is reflected as an echo towards car.
1000 350  50 Hence, frequency of echo heard by the driver is
   n r  750 Hz
nr 350  50  330  20 
ndriver = 660   = 700 Hz
 330  0 

io
 v   320  320
20. n1 = n 
v  v   n  320  20   n  300 Hz 27. fincident = freflected =
v
n=
320
 8 kHz
 s
v  vs 320  10
 v  320

at
n2 = n    n  340 Hz 320  10 330
 v  v s 
 fobserved = freflected = 8 
320 310
Percentage change in frequency = 8.51 kHz  8.5 kHz.
n 2  n1 n
n1
 100 = 2 1  100
n1
lic 28. As source crosses stationary listener then, ratio
of apparent frequencies before crossing (n1) and
after crossing (n2) is,
= 100 
300 
 1  12 %
 340  n1 v  v s

ub
n 2 v  vs
 v 
21. n = n   n1  v  vs  500  350  50 
 v  vs   n2 = =
v vs 350  50
n  v   n2 = 375 Hz
 
P

n  v  vs 
29. As observer is at rest, frequency heard by observer
5 350  v 
 Case I: n = n 
6 350  vs 
 v  vs 
et

vs = 70 m/s
Case II: n = n 
v 
22. The frequency of reflected sound heard by the  v  vs 
driver, As speed vs is constant, n = constant  n.
rg

 v  ( v0 )   v  v0  Thus, as engine approaches observer, apparent


n  n    n 
 v  v s   v  vs  frequency heard is higher and as source moves
 330  (72  5 / 18)  124  35 away, apparent frequency heard is lesser. Hence,
 124  
Ta

the graph (D) represents the situation best.


 330  (72  5 / 18)  31
30. Vs sin 60
= 140 vibration/s Vs
 v  v0  333  33 
23. n = n   = 1000   = 1220 Hz.
 v  vs   333  33  60
S O
Vs cos 60 Doppler
24. Both source and observer are moving towards line
each other,
 V 
 v  v0   340  16.5  n = n 
 n = n0   
 = 400    V  Vs cos 60 
 v  v s   340  22 
n = 448 Hz  
 330 
= 100  
25. As siren moves towards cliff, frequency incident  330  19.4  1 
on cliff is,  2

97

MHT-CET Triumph Physics (Hints)


 330   340  20 
= 100   nseparation = 850  
 330  9.7   340 
 330  850
= 100   napproach  nseparation =  40 = 100 Hz
 320.3  340
= 103.02 Hz 36. The velocity of sound is greatest in solids.
31. T mg
drone 37. v= 
 

m g2  a 2
v =

Observer Building
v g2  a 2
 

ns
In this case observer and building are stationary v g
with respect to each other hence no doppler shift 2
g2  a 2
 
of reflected sound. 60.5 
 
 v   60  g
Hence na =   n

io
g
 v  vs  a
5
 340 
=   780 = 816 Hz 38. As string is clamped resulting wave is a
 340  15 

at
standing wave of equation y = 2A sin kx cost
32. Case I: Comparing with given equation,
When passenger is sitting in train ‘A’ 2
 = 60 and k =
3
 n1  n  v = v ….(i)
n1 = 
 v  vA 
 v
n 
 n
A
lic 
Now velocity v = =
60 
= 90 m/s
k 2
Case II: 3
When passenger is sitting in train B, Also, velocity of transverse wave,
ub
 v  vB   n 2  n  v = v ….(ii) T T
n2 =  n B v= =
 v  n m M/L
Dividing (ii) by (i), M 902  3  102
vB n n  T = v2  = = 162 N
= 2 L 1.5
P

vA n1  n
39. Let the sound reach the other boy through metal
vB 6000  5000 in t seconds and let distance between them be x.
= =2
vA 5500  5000  Distance = speed  time
et

x = 3630  t
 v  v0  Now, the sound through air reaches in
33. n =  n
 v  vs  (t + 1) seconds.
rg

As siren is at rest, vs = 0  x = 330  (t + 1)


 v  vA   3630 t = 330 (t + 1)
 nA =  n  3630 t  330 t = 330
 v 
 3300 t = 330
Ta

340  v A
 4.5 = 4 1
340  T= second
10
 v  vB 
 vA = 42.5 m/s and nB =  n 
 1 
x = 330    1  = 363 m
 v   10 
340  v B 40.
 5= 4
340
 vB = 85 m/s m1
34. Since there is no relative motion between the
source and observer, no change in real
frequency is observed. m2
 v  v0   340  20  Let velocity of pulse at lower end be v1 and at
35. napproach = n    850  ,
 v   340  top be v2

98

Chapter 08: Sound


2 v 2 v 42. Given radius = 0.5 Å
  (  = and n = constant)
1 v 1 n  diameter d = 1 Å = 1  1010 m
velocity of transverse wave on string RT
=
T 2 d 2 N A P
v=
m K B NAT
=
where, m is linear density. 2 d 2 N A P
In this case, v  T K BT
=
2 v 2 T  m2  m1  2 d 2 P
   2 
1 v1 T1 m2
=
1.38  10 23
JK 1   314 K 
2  3.14  1010 m  105 Pa 
2
Where, T2 is tension at upper end of rope and T1
is tension at lower end of rope.
1.38

ns
41. =  106 m
A A 2
N As Mean free path = wavelength of sound wave
1.38 1380
  106 =

io
2
1 1380  2
L=  60 cm = = 1000 2 MHz
2 106  1.38

at
v
Fundamental frequency, n0 = 43. Speed of wave v  T
1
But, tension (T)  extension (x)
here, 1 = 2L
 v x
Also, v =
Y

lic When extension is increased, let speed becomes
v
v 1 Y v x
 n0 =   =
1 2L  v 1.5 x
ub
1 9.27  1010 v = 1.5 v = 1.22v
 n0 = 2

2  60  10 2.7  103
= 4.88  103 Hz  5 kHz
P

Hints to Evaluation Test


et

1. Given: calculated velocity at 30 C,  BC = 300  6 = 1800 m


v = 35078 cm/s,  OB = BC2  OC2
then velocity at 0 C is, OB = 994.99  995 m
273 The plane is 995 m above the ground.
rg

v0  v  33296 cm/s
273  30
3. Given: Molecular mass of neon (MNe) = 20MH.
v = 33296 – 33200 = 96 cm/s (where MH is the molecular mass of hydrogen)
96 Volume of neon (VNe) = volume of hydrogen
 % error =  100 = 0.29%
Ta

33200 (VH)
2. O 1500 m
Molecular mass of mixture,
A C Mmix = MNe + MH = 20MH + MH = 21MH
Volume of the mixture,
(Vmix) = VNe + VH = VH + VH = 2VH.
1800 m Density of a gas,
M

V
mix M mix VH 21M H VH 21
B Therefore,     
H M H Vmix MH 2VH 2
Time for the echo = 12 s (i.e., for sound to travel
ABC). Velocity of the plane = 250 m/s. Velocity of sound in a gas is given by,
OC = 250  6 = 1500 m P 1
v , i.e., v 
BC = velocity of sound  6  

99

MHT-CET Triumph Physics (Hints)


v mix H 2 RT
Therefore,   13. v
vH mix 21 M
For similar pressure and temperature conditions,
4. Longitudinal waves travel in an elastic medium 
in the form of compressions and rarefactions v
M
which change its volume, elasticity and
pressure. Since air is completely an elastic Gases H2 N2 He CO2
medium, i.e., it does not have a modulus of  1.41 1.40 1.66 1.30
rigidity, therefore sound waves in air are always M 1 14 4 44
longitudinal.
1.40
5. Sound wave is an example of mechanical wave  vH 2  1.41 , vN2 
14
which requires a medium for its propagation. In

ns
1.66 1.30
outer space, we have a vacuum and hence sound and v He  , vCO2 
wave propagation is not possible. 4 1.44
Comparing values, velocity of sound will be
RT largest in hydrogen.
6. v ;  is the same as both hydrogen and

io
M
chlorine are diatomic.
vH2 M Cl 70 35.5

at
  2
  6
vCl2 M H2 2 1

7. The concept of rarer and denser media for a


wave is related to the velocity of propagation of
wave in the medium. Lesser the velocity, denser
lic
is said to be the medium and vice-versa. For
sound waves, air is denser than water as velocity
ub
of sound in air is less than velocity of sound in
water. However, for light waves, air is rarer than
water.

8. Mechanical transverse waves can propagate


P

through solids only as solids have elasticity of


shape.

RT
et

9. Velocity of sound in gas, v 


M
T
 v
rg

M
7
v N2  N2 R4
M 5 3
   He  
v He  He M N2 5 5
Ta

R  28
3

10. After two seconds the pulses will overlap


completely. The string becomes straight and
therefore does not have any potential energy and
its entire energy must be kinetic.

11. Frequency of sound does not change with


medium, because it is a characteristic of source.

12. Number of waves per minute = 42


 Number of waves per second = 42/60
42
v = n  n =  12 = 8.4 m/s
60

100
Textbook
Chapter No.

09 Optics

Hints

31. When m = – 3 (real image)


Classical Thinking v

ns
m=–3=–
11. According to laws of reflection, i = r u
 When i = 2i,  v = 3u
r = 2i = 2(15) = 30 1 1 1
Also + =
v u f

io
c 100 100
13. n= = = = 1.43 1 1 1
v 100  30 70  + =
u 3u 15
sin i sin 45  u = – 20 cm

at
14. n= = = 2
sin r sin30 When m = + 3 (virtual image)
c 3  108 v
v= = = 2.12  108 m/s m = +3 = –
n 2 u

3  108
lic  v = – 3u
15. vg = = 2  108 m/s 1 1 1
3/ 2  + =
u 3u 15
 Distance = vg  t
 u = – 10 cm
ub
= 2  108  10–12 = 2  10–4 m
= 200 m 1 1 1
39. + =
v u f
 360 
17. n=   –1=5 1 1 1 1 1 1 1
 60   =  =  = +
P

v f u 15 12 15 12
18. As, the object is placed symmetrically,  v = + 6.7 cm
 360   360 
n=   1  3 =   1   = 90 1 1 1 1 1 1 1 4
et

      40. = + = + =  =
f v u 5 25 5 25 25
24. Focal length of convex/concave mirror depends  f = + 6.25 cm
only on radius of curvature (R) of the mirror. It f is positive therefore the mirror is convex.
rg

does not depend upon u and v.


90
29. Using mirror equation with Cartesian sign 41. f= = 45 cm
2
conventions,
From mirror formula,
Ta

1 1 1 1 1 1
= + = + = 1 1 1 1 1
f v u 24 40 15   =  = 11.25 cm
v f u 45 15
 f = 15 cm
v 11.25
1 1 1  m=– =– = 0.75
30. + = u 15
v u f
Using mirror equation with Cartesian sign 42. Using Shortcut 2,
conventions, v =  (m  1) f
1 1 1 1 1 1 
= + =  =    1  60
f 30 10 30 10 2 
15  v = + 30 cm behind mirror.
 f= cm
2
a
15 w ng 1.5
and R = 2f =   2 = 15 cm 48. ng = a

2 nw 1.3

101

MHT-CET Triumph Physics (Hints)

49. i = 90  30 = 60 1 1 1


Also  =
w sin i v u f
ng =
sin r 1 1 1
  =
sin i sin 60 3 1 1 36 18 f
 sin r = w = =  =
ng 3 2 3 2 1 1 1
 + =
36 18 f
 r = 30
 f = 12 cm
v
50. w
ng  w 1  1 1 
vg 84.
f
=  a
n g 1 
R

R
,
 1 2 
9
 vw = w n g  vg   2  108 = 2.25  108 m/s R1 = R, R2 = R
8
1 1 1  2 1
 = (1.5  1)    = (0.5)   =

ns
h h f  R R  R R
53. n  h 
h n  f = R = 30 cm
Realdepth
54. n= 1  1 1 
Apparent depth 85. =  a
n g 1   

io
f  R1 R 2 
Realdepth 46
 Apparent depth = = = 34.5 cm For a plano-convex lenx,
n 4/3
R2 =  and R1 = R

at
1 1 1
62. sin iC = therefore iC will be maximum when  = (1.5  1)  
n 20 R
n is minimum which is for red light. 1 0.5
 =
63. n=
1
=
1
=
2
= 1.15
lic 
20 R
R = 10 cm
sin i C sin 60 3
v 1
1 1 1 88. m= =
64. n= = = = 2.41 u 4
ub
sin i C sin (24.5) 0.414
u
 v=
n2 n n n 4
75.  1 = 2 1
v u R 1 1 1
Also  =
1.5 1 1.5  1 v u f
P

  =
v 20 5 4 1 1
  =
 v = 30 cm u u f
3 1
et

1 1 1  =
81.  = u f
v u f
 u = 3f
1 1 1 1 1 5
 = + = + =
v f u 0.15 0.2 3 1  1 1 
rg

3
89.
f
=  a
n g 1 
R

R

 v = = 0.6 m  1 2 
5
 1 1 
= (1.6  1)   
 20 30 
Ta

v
82. m= =3
u 1 1 1
 =  (0.6)   = 
 v = 3u f 12
  20
1 1 1 1 1 4  f = 20 cm
=  =  =
f v u 3u u 3u
4f 1  1 1 
 u= 90. = (ng  1)   
3 f  R1 R 2 

v
 | R1 | = | R2 | = R
83. (u) + v = 54 cm and m = =2 1  1 1 
u  = (1.5  1)   
f   R  R
 v =  2u
 (u) + (2u) = 54 1 2
= (0.5)  
 u = 18 cm f R
 v = 2 (18) = 36 cm  f = R = 30 cm
102
102

Chapter 09: Optics


1 1 1 1 1 1 121.  = A(n 1)
94. = + = + =
f f1 f2 15 30 10  2.4 = 4 (n  1)
 f = 10 cm  n  1 = 0.6
 n = 1.6
1 1 1
95. = + 126.  = v  r = A (nv  nr) = 5(1.665  1.645)
f f1 f2
v  r = 0.1
1 1 1 1 1 1
= + =  =
f 30 20 30 20 60 127.  = v  r = (nv  nr) A
 f =  60 cm = (1.66 – 1.64)  10 = 0.2°
1 1 1 v  r
96. = + 128. = 
f f1 f2  mean
1 1 1 Angular dispersion = v  r = mean

ns
 = +
13 10 f 2 129.  depends only on nature of material.
1 1 1 3
 =  = nv  nr 1.7  1.65
f2 13 10 130 131.  = =
 nv  nr  1.675  1

io
130   1
 f2 =  =  43.33 cm  2 
3
0.05
= = 0.074
97. Power of the combination P = P1 + P2 0.675

at
= 12 – 2 = 10 D
144. Chromatic aberration is caused due to inability
 Focal length of the combination
of lens to bring all wavelengths of colour to
100 100
f=  = 10 cm same focal plane.
P 10
lic 146. An achromatic combination of lenses provide
112. In minimum deviation position, deviation without dispersion. As a result, images
i = e are unaffected by variation of refractive index
with wavelength.
ub
113. In minimum deviation condition
A 60o D 
r= = = 30 159. M =   1
2 2 f 
A  m  25 
P

115. i = = 50° =  1 = (10 + 1) = 11


2  2.5 

3 3 163. Intermediate image means the image formed by


116. Given i = e = A   60 = 45
objective, which is real, inverted and magnified.
et

4 4
In the position of minimum deviation M 35 35
2i = A + m or m = 2i – A = 90 – 60 = 30 167. mo = = =
Me D 25
1 1
fe 8
rg

117. By the hypothesis, we know that


i + e = A +   55 + 46 = 60 +   mo  8.48
  = 41 168. In telescope fo >> fe as compared to microscope.
But m < , so m < 41
Ta

170. Three lenses are: objective, eye piece and


 A  m  erecting lens.
sin  
118. n =  2 
A fo  f 
sin   177. M = = 1  e 
2 fe  D
  30 
60 fo 2
sin   179. M = = = 40
=  2  fe 0.05
 60 
sin  
 2  fo
180. M =
sin 45 fe
=
sin 30 If fe = 2 fe ,
fo 1 fo M
= 2 then M = = =
= 1.414 2f e 2 fe 2

103

MHT-CET Triumph Physics (Hints)


fo
181. M = =5
fe Critical Thinking
 fo = 5fe 2. Light exhibits wave as well as particle nature.
L = fo + fe = 36 Hence, it is said to exhibit dual nature.
 6fe = 36
 fe = 6 cm, fo = 30 cm 5. Snell’s law in vector form is ˆi  nˆ  (rˆ  n)
ˆ

fo 1  2 3 1
182. M = = 10 6. 2 1   3 2  43    
fe  2 3  4  4
 fo = 10 fe  41 
1
Also, L = fo + fe = 44 cm 
1 4

 10fe + fe = 44
a ca x t

ns
 11 fe = 44 7. ng = = a
cg xgt
 fe = 4 cm and fo = 44  4 = 40 cm
 xa = xg × ang
183. Referring to Mindbender 4, a completely 3
transparent material will be invisible in vacuum =6 = 9 cm

io
2
when its refractive index will equal refractive
index of vacuum. x x nx
8. t= = =
v c / n c

at
1 1
184. n = = =2 1.33  500
sin i C sin 30 =
3  108
c 3  108
 v= = = 2.22  10–6 s
n 2
= 1.5  108 m/s
lic = 2.22 s
9.
185. Object should be placed at focus of a concave
mirror.
ub
i r
x
90 – x
Point image 90 – r1
r1
P
et

D  sin i sin x
186. M =   1 n= =
f  sin r1 sin (90  x)
25 [ (90  x) + (90 – r1) = 90]
 6= +1
rg

f
sin x
 f = 5 cm = 5  102 m = = tan x
cos x
1
 P=  x =  i = tan1 (n)
Ta

5  102
100 sini
= = 20 D 10. Refraction at air-oil interface, n oil 
5 sin r1
sin 40
1  sin r1 = = 0.443
187. n  1.45

Refraction at oil-water interface,
R > Y > G > V
sin r1
nR < nY < nG < nV oil
nwater 
sin r
1 1.33 0.443
188. n   
 1.45 sin r
As R > B, nR < nB 0.443  1.45
 sin r =
nR 1.33
 <1
nB  r = 28.9

104
104

Chapter 09: Optics


11. Incident
18. Concave mirror forms inverted and enlarged
Emergent image when object is placed between focus and
30 30 ray 30 30 ray
centre of curvature, while convex mirror always
 forms erect and diminished image. As the
r r distance of person is not changed from the
mirror, mirror B cannot be concave.
19. Given u = (f + x1) and v = (f + x2)
Referring the figure, the angle between the The focal length f =
uv
=
 f  x1  f  x 2 
incident ray and the emergent ray is 60 u  v  f  x1    f  x 2 
On solving, f 2 = x1x2
360
12.  = 0 and n =  f = x1x 2

ns
360 1 1 1 1 1 3
 n= = 20. = – = – =–
0 v f u 10 25 50
13. If plane mirror is rotated through ‘’, reflected 50
 v=– = – 16.67 cm
ray would rotate through double the angle i.e., 3

io
2. h v
m= 2 =–
A h1 u
14.
50

at
h2 2
  = 3 =–
3 25 3
 h2 = – 2 cm

B (90 – i1) (90 – i2)


C
lic 
Negative sign indicates real inverted image.
Area = 2  2 = 4 cm2
i1 i2
i1 i2 21. At u = f, v = 
At u = 0, v = 0 (i.e., object and image both lies
ub
at pole). Satisfying these two conditions, only
Let the angle between the two mirrors be ‘’. option (A) is correct.
Since, the angle of deviation i.e., angle between 1 1 1
reflected and incident ray without disturbing its 22. Since = +
f v u
P

direction is,  = 180  2i.


1 1 1
Total deviation d = d1 + d2 = +
v u f
= (180 – 2i1) + (180 – 2i2)
1 1 1
= 360 – 2 (i1 + i2) Using the sign conventions, = +
et

But the resultant ray is parallel (  v) (  u) (  f)


 d = 180 1 1 1
= +
 180 = 360 – 2(i1 + i2) v u f
rg

 i1 + i2 = 90 Comparing this equation with y = mx + c


In ABC,  + (90 – i1) + (90 – i2) = 180 Slope = m = tan = –1
 i1 + i2 =   = 135° or –45° and intercept
  = 90 1
Ta

c =+
f
17.
23. No parallax between two images.

40 m 20 m
C F
20 m
O O
When the boy moves by 40 m towards the
mirror, he reaches at centre of curvature (2F) of
mirror. Hence his image formed is inverted and 30 cm 20 cm
of same size. The lamp lies between infinity and
centre of curvature hence image formed is
inverted and diminished. 50 cm 10 cm
105

MHT-CET Triumph Physics (Hints)


1 1 1 2 31. In BCE
= + = N
f 50 10 25 A
25 25 
 f= , R = 2f = 2  = 25 cm
2 2 P B Q
(  r)
m 4 r
24. ng =
3
t
vm 4 4
=  vm = vg 90 E
vg 3 3
d
4  M
 vm  vg =   1 vg = 6.25  107 m/s S C R
3  D
F
vg = 18.75  107 m/s

ns
 vm = 6.25  107 + 18.75  107 CE
sin(  r) =  CE = BC sin(  r)
= 25  107 = 2.5  108 m/s BC
 d = BC sin(  r) ….(i)
26. In BMC

io
R.I. = 1 BM BM t
y cos r =  BC = = ….(ii)
BC cos r cos r
R.I. = n From equations (i) and (ii),

at
x t
d= sin (  r)
cos r
t
d= (sin  cos r  cos  sin r)
The distance of the surface of water for the fish cos r
=x
lic = t (sin   cos  tan r)
For reference frame of fish, as light rays will If n is the refractive index of material of slab
travel from denser to lighter (air) medium, they (glass) w.r.t.air, then
will bend away from normal and bird will sin 
ub
appear farther. n=
sin r
Thus, apparent height = n  real height = ny.
 Distance estimated = x + ny For small angle,
 
27. As both the diver as well as the fish are in water, n  r= and d = t(  1.r)
r n
P

refraction effects such as bending of light are


…( sin    and cos   1 if  is small)
not present.
   
d real d = t     = t   
et

28. n=  n n  
d apparent
t n 
12.5 d=
nw = = 1.33 n
9.4
rg

When water is replaced by liquid, 32. To see the container half-filled from the top, the
d real 12.5 apparent depth of the bottom must be equal to
dapparent = =  7.7 cm the height of the container above the surface of
nl 1.63
the water.
Ta

The distance by which microscope should be Let water be filled up to height x so that bottom
moved, of the container appears to be raised upto height
d = dreal  dapparent = 9.4 – 7.7 = 1.7 cm (21 – x).
d d 1 1  As shown in figure apparent depth
29. h   = d   h = (21 – x)
n1 n 2  n1 n 2  Real depth h = x
d1 d
30. Apparent depth = + 2 (21 – x)
n1 n2
36 5 3 h = (21 – x)
 = +
7 5 n2 21 cm x=h
3
7
 n2 = = 1.4 Bottom
5 Raised bottom
106
106

Chapter 09: Optics


a h sin 
 nw = At interface 1: n =
h' sin r
4 x  sin  = n sin r ….(i)
 
3 21  x At interface 2: (90  r) = iC
 x = 12 cm  sin(90  r) = sin iC
34. In total internal reflection, 100% of incident 1  1
 cos r =  sin iC  n 
light is reflected back into the same medium and n  
there is no loss of intensity. While in reflection 1
 cos r = = 3 / 2  r = 30
from mirrors and refraction from lenses, there is 2/ 3
some loss of intensity. Therefore images formed 2 1
by total internal reflection are much brighter From equation (i), sin  = sin 30 =
3 3
than those formed by mirrors or lenses.
 1 

ns
36. When incident angle is greater than critical   = sin1  
 3
angle, then total internal reflection takes place
and will come back in same medium. To signal 43. I = I0ex is an equation of decreasing
light out he has to direct the beam at an angle exponential curve with I0 as intercept on I-axis.

io
lesser than the critical angle.
44.
37. For glass, n = 2

at
1 1 1  1 
 iC = sin   = sin   = 45

   2
This means the ray is incident at critical angle
hence will come out just grazing the surface,
i.e., angle of refraction equal to 90.
lic The principle of the periscope is that the image
of an object (a ship for example) is formed at a
38. For total internal reflection i  iC lower level (in a submarine). Light is incident
normal on a right angled prism which makes
sin i  sin iC
total internal reflection of the ray coming from
ub
1
sin i  the right at the hypotenuse of the prism. This is
n
again reflected by another prism to give an
1 image to a person in the lower level (say, in a
 n
sin i submarine). This can be combined with
P

1 1 telescopes.
39. iC = sin1   and n 
n  n 2 n1 n 2  n1
45. – =
Yellow, orange and red have higher wavelength v u R
et

than green, so n will be less for these rays,



1.5

1
=
1.5  1  v =  30 cm.
consequently critical angle for these rays will be v  15  30
high, hence if green is just totally internally
Negative sign shows that, image is obtained on
rg

reflected then yellow, orange and red rays will


the same side of object i.e., towards left.
emerge out.
46. Eye lens being convergent forms a real image of
1 1  a virtual object (i.e., the virtual image being
40. Critical angle = sin  
Ta

n seen on the retina of the eye).


 1  1  1 
 = sin1   and  = sin    1 1 
 n   n  47. Since, P = (n – 1)   
 1   2   R1 R 2 
Since n 2  n 1 , hence  <  Thick lens has R less than thin lens, hence more
power.
41. 48. 2f 2f
2 90 f along
YOY
90r
r along f
 1 XOX
f

107

MHT-CET Triumph Physics (Hints)


49. From lens-maker’s formula | u | = 75  15 = 60 cm
1  1 1  v v 60
= (n – 1)    Magnification, m = = = =4
f  R1 R 2  u u 15
1  1 1 
 = (n1 – 1)    and 55. Here,R1 = + R, R2 =  R
f1 R
 1 R 2 
According to lens maker’s formula,
1  1 1  1  1 1 
= (n2 – 1)    = (n  1)   
f2 R
 1 R 2  f  R1 R 2 
f1 and f2 are focal lengths corresponding to n1
1  ng  1 1 
and n2 respectively.  =   1   
Hence, there are two focal lengths giving two fa  n a   R  R  
images. 1  1.5   2 
=   1  

ns
1 1 1 fa  1  R
50. – =
v f / 2 f 1 1
=
 v=–f fa R
v f n  1 1 

io
m= = =2 1
u f / 2
=  g  1   
fw  nw   R (  R) 
The image is virtual, double the size.
3 
 

at
51. v 2
A C =  2  1  
 4  R
3 
P 2
2f
lic =
8R
45 B  fw = 4R
O 2f u
1  1 1 
The experimental plot of v vs u is represented = (n  1)   
ub
56.
by curve AB. The line OC meets the curve at f R
 1 R 2 

point P(2f, 2f). R1 = +20 cm, R2 =  20 cm, n = 1.5


1 1 1 1  1 1 
52. Referring lens formula – = ; = (1.5  1)   
f  20 20 
P

v u f
u is always negative,  1 1 
v is positive. = 0.5   
 20 20 
et

1 1 1 1  1  2 2 0.5
53. =  =   = 0.5  =
f v u 10  10  10 20 10
 f = 5 cm  f = 20 cm.
uv Parallel rays converge at focus. Hence,
rg

f=
uv L = f.
f u v u  v
=   1 n  1 1 
f u v uv 57 =  g  1   
Ta

fw  nw   R1 R 2 
Least count of u and v being 1 mm,
f 0.1 0.1  0.1  0.1 1  1 1 
=   
3 = (ng 1)   
f 10 10 10  10  100 fa R
 1 R 2 

f = 0.15 cm ….(Since, f = 5 cm) 1  1 1 


= (1.5  1)    ….(i)
 Focal length of lens is 5.00  0.15 cm fa R
 1 R 2 

54. u =  (75  v) 1  1.5   1 1 


1 =  1    ….(ii)
1 1 1 fw  4 / 3   R1 R 2 
 = v
v u f solving equations (i) and (ii),
1 1 1 c = +1 fa (0.5) = fw (0.125)
  = f
135°
v (75  v) 12 10  0.5
–45°  fw =
 v = 60 cm or 15 cm 1 0.125
 | u | = 75  60 = 15 cm or u  fw = 40 cm
108
108

Chapter 09: Optics


1 65. In each case two plane-convex lens are placed
58. For convex lens, P =
f 1 1 1
close to each other and = + . Hence
Using lens maker’s equation, f f1 f2
1 2 focal length is same for all given combinations.
  a n g  1  
f R 1 1 1 1 1
2 66. = + = +
5 = (1.5 – 1) ….(i) f f1 f 2 f f
R
f=
1
 = 5 /m
R 67. Let the resultant focal length of combination be
When the lens is placed in liquid, it acts like f then,
plano-concave lens. 1 1 1
 
For concave lens, f = 100 cm = 1 m. f f1 f 2

ns
Using lens maker’s equation, 1 1 1
  f=
1 1 f 20 ( 20)
  l n g  1  
f R Hence, it behaves as a plane slab of glass.
a
ng 1.5

io
Here l n g   70. Effectively there is no deviation or dispersion.
a a
nl nl
 1.5 
 1 =  a  1 5 ….[From (i)]

at
 nl 
1.5 1.5  5
 a
nl   = 1.875
 1 4
 1  
 5
lic 71. A glass slab is equivalent to two prisms
59. Virtual Real mounted such that the second prism is placed
object Image inverted upon the first. As a result, while light
dispersed by first prism enters the second prism,
ub
P recombines and emerges out as white light
O again.
u = 12 cm 73. Net deviation caused by prisms Q and R is zero
hence the ray suffers same deviation.
P

v
74. The angle of deviation depends on the refractive
By using lens formula index of prism. As n decreases,  decreases.
1 1 1 1 1 1 43 Refractive index of prism relative to water is
= –  = – =
et

16 v  12  v 12 16 48 less than that relative to air. Hence, when a glass
 v = 48 cm prism is immersed in water, the deviation
caused by prism decreases.
1  1 1  2  2
= (gna – 1)    =   1  
rg

61. 75. In minimum deviation position refracted ray


f R
 1 R 2   3   10 
inside the prism is parallel to the base of the
f = 15 cm, so behaves as concave lens. prism.
Ta

62. If n1 > ng then the lens will be in denser 76.


medium. Hence referring to Shortcut 3, the
convex lens will behave like a concave lens. A
63. Referring to Shortcut 2, using relation between
u and f we get, r1 r2
f f 1
m=  m
u  f  x x

64. As seen from a rarer medium (L2 or L3), the Angle of prism, A = r1 + r2
interface L1L2 is concave and L2L3 is convex. For minimum deviation
The divergence produced by concave surface is r1 = r2 = r
much smaller than the convergence produced by A = 60
convex surface. Hence the arrangement A 60o
 r= = = 30
corresponds to concavo-convex. 2 2

109

MHT-CET Triumph Physics (Hints)


77. r2 = 0 A = r1 + r2 82. i=0
 A = r1 = 30  r1 = 0
sin i sini  e = A +  and A = r2
n= = sin  A   
sin r1 sin A sin e
O n= =
sin i sin r2 sin  A 
 2 =
sin 30   A  (180  2A) 
 A  m  sin 
sin   
 sin i = 2  sin30  2  =  2 
83. n=
1 1 A A
= 2 = sin   sin  
2 2 2 2
 1    A  A
i = sin1   = 45 sin 90     cos  
 2   2   = 2
 n=

ns
 
A A
sin   sin  
78. Since, i + e = A +  2 2
e = (A + )  i  
A
= (30 + 30)  60  n = cot  
2

io
=0
This means if angle of emergence (measured  A  m 
sin  
with respect to normal to the second face) is 84. n = 1.5 =  2 
A

at
zero, therefore angle made by emergent ray with sin  
the second face of prism is 90. 2
Since A = m
79. i  e = 10 ....(i)
 2A  A A


i+e=A+
i + e = 60 + 30 = 90
lic 1.5 =
sin 


2 
=
2sin cos
2 2
A A
sin sin
i + e = 90 ….(ii) 2 2
Solving equation (i) and (ii) A
cos = 0.75
ub
i = 50, r = 50  10 = 40 2
A
80. A = cos1 (0.75) = 41
2
30°  A = 2  41 = 82
P

90° 90°  A  m 
sin  
85. n=  2 
30° A
e sin  
et

 2
Here A = m
B C AA
sin  
rg

 n=  2 
1 sin 30 A
For surface AC,  sin
n sin e 2
1 sin A
 sin e = n sin 30 = 1.5 
Ta

= 0.75 3 =
2 A
sin
 e = sin–1(0.75) = 48 36 2
A A
From figure,  = e – 30 2sin cos
3 = 2 2 = 2 cos A
= 4836 – 30 = 1836 A 2
sin
81. e=0 2
 r2 = 0 A 3
cos =
sin i 2 2
Also, r1 = 30 and n =  3
sin r A
 = cos1  
sin i 2  2 
 1.5 =
sin 30 A
 = 30
 sin i = 1.5  sin 30 = 1.5  0.5 2
 i = sin1 (0.75)  A = 60
110
110

Chapter 09: Optics


86. m = A = 60 Also,  = (n  1)A = An  A
 A  m  Comparing with y = mx + c
sin  
 2  Slop of line PQ = m = A
n=
A Thus, option (C) is correct.
sin  
2
92. The dispersive power for crown glass
AA nv  nr
sin   =
= 
2 
ny 1
A
sin   1.5318  1.5140 0.0178
2 =  = 0.034
sin  A  (1.5170  1) 0.5170
= Dispersive power for flint glass,
A
sin   1.6852  1.6434
2  = = 0.064

ns
(1.6499  1)
A A
2sin   cos  
= 2 2 Anb  nr  6 1.531  1.520 
A 93. A = – =–
sin    b r
n   n  1.684  1.662 
2

io
A = – 3.
A
= 2cos   Negative sign for opposite manner of flint glass
2 prism.

at
 60o  Hence refracting angle = 3
n = 2cos   ….(As A = 60°)
 2  Net deviation = A (n – 1) + A (n – 1)
n = 2cos(30°) = 3 1.531  1.520 
= 6   1
87. By formula  = (n – 1)A  34 = (n –1)A
lic  2 
A 1.684  1.662 
and in the second position  = (n – 1) + (– 3)   1
2  2 
34 (n  1)A = 1.134
ub
34
  or  = = 17
 (n  1) A 2
2 94.
nv  nr  = 
(n  1)
9   (nv – nr) = (n – 1)
  1
P

w ( w n g  1) 1
= 
8 
88. = a =  A (nv – nr) = A  (n – 1)
a ( n g  1) 3  4
  1  Achromatic combination
 2 
(n – 1)  A = (n – 1)  A
et

89.  = A (n – 1), b = A (nb – 1), r = A (nr – 1)



A
=
 n  1  = 0.517  0.03 = 0.50 …(i)
 D2 = A (1.525 – 1) A  n  1  0.621 0.05
 D1 = A (1.520 – 1) Net deviation =  –  = (n – 1) A – (n – 1) A
rg

 D2 > D1  1 = 0.517 A – 0.621 A …(ii)


w
a
ng 3/ 2 9 On solving equations (i) and (ii),
90. ng = a
= = A = 4.8 and A = 2.4.
nw 4/3 8
Ta

3  9  95. Alexandar’s dark band between the primary and


 a = A   1 ,w = A   1
2  8  secondary rainbows is because light scattered
A A into this region interferes destructively. Further,
a = and w = primary rainbow subtends an angle of 41  43
2 8
at the eye of the observer w.r.t. the incident
w A/8 1
 = = light, and secondary rainbow subtends an angle
a A/2 4
of about 51 to 54 at the eye of the observer
91. At P,  = 0 w.r.t. the incident light. Therefore, the region
For thin prism  = m between the angles of 41 to 51 is dark.
  = A(n  1) 96. As refractive index of lens is different for
 0 = A (n  1) different colours/wavelengths, therefore,
n=1 different colours are focussed at different points.
Thus, option (A) is correct. As a result, the image is coloured.
111

MHT-CET Triumph Physics (Hints)


97. A lens can be considered as made up of several 102. Given are the refractive indices for extreme
prisms mounted one above the other. In a thick colours. As, nR < nV
lens, angle of prisms are larger than those in a nR = 1.60 and nV = 1.66
thin lense. For convex lens,
Since, angular dispersion produced by a prism is R1 = 10 cm and R2 = – 12 cm
directly proportional to angle of prism, a thick  1 1  1 1 
lens suffers from greater chromatic aberration.    =    =
11
R
 1 R 2   10  12  60
Hence, assertion is false but reason is true.
For red colour,
98. One way to reduce chromatic aberration, is by 1  1 1 
increasing focal length of the lense.   n R  1   
fR  R1 R 2 
This can be achieved by using combination of
multiple lenses. 11
= (1.60 – 1)
60

ns
99. Unit of longitudinal chromatic aberration of a = 0.11
thin lens is metre as it involves measurement  fR = 9.09 cm
of length or distance. Similarly, for violet colour,
100. According to condition for achromatism of a 1  1 1 

io
  n V  1   
combination of lenses, fV  R1 R 2 
f1 2 11
 = (1.66 – 1) = 0.121
f2 1

at
60
12 3  fV = 8.26 cm
 
f2 2  Longitudinal chromatic aberration
 f2 =  8 cm = fV – fR = 9.09 – 8.26 = 0.83 cm
lic
101. For a concavo-convex lens, with convex shape 103. A single point focus in case of lenses is possible
facing the object, both the radii of curvature are only for small aperture spherical lenses and for
positive as shown in the figure. paraxial rays.
ub
The rays coming from a distant object farther
from principal axis no longer remain parallel to
the axis.
Thus, the focus gradually shifts towards pole
giving rise to spherical aberration.
P

C2
P C1 104. The image distance from the eye lens remains
constant because for healthy eye, image is
always formed on retina.
et

105. For small value of fo and fe


+R1 vo  L and uo = fo
+R2
L  D
rg

M= 1  
 R1 = 8 cm and R2 = 40 cm fo  f e 
 1 1   1 1  –1 15  25 
 K =   =    = 0.1 cm  375 =
R
 1 R 2   8 40  1  
0.5  fe 
Ta

1  1 1   fe = 2.17 cm  2.2 cm.


   n R  1   
fR  R1 R 2 
106. Let the closest distance be u and farthest
= (1.51 – 1)  0.1 = 0.051 distance be u.
 fR = 19.61 cm 1 1 1 1 1 6
1  1 1  =  =  = ….( v = 25 cm)
and   n V  1    u v f 25 5 25
fV  R1 R 2  25
 u= cm
= (1.53 – 1)  0.1 6
= 0.053 1 1 1 1 1
 fV = 18.87 cm Also =  =  ….( v = )
u v f  5
 Longitudinal (lateral) chromatic aberration  u = 5 cm
= fV – fR u 25 / 6 5
= 19.61 – 18.87 = 0.74 cm Ratio, = =
u 5 6
112
112

Chapter 09: Optics


107. Telescope is used to observe distant object nearer. 117. For total internal reflection
at AC B A

 D A  = (3)2 = 9
2
Light gathered by A  > iC 
109. =
 DB 
2
Light gathered by B  sin   sin iC
1
 fo  100  sin   C
110.      = 25 w
ng
 fe 0.5 o
2
nw 8
111. The objective of a telescope must have large  sin    sin  
ng 9
aperture to gather more light. It should also have
 fo  118. A
large focal length  m =  . Therefore, lens A is
 fe  r
selected as objective lens. 30°

ns
The eyepiece should have small aperture and B 60° 120°  
small focal length. Therefore, lens D is selected D
as eye lens. r
C
112. L = fo + fe = 1.53 m ….(i)

io
fo
M= = 50 sin 30 1
fe At point A, 
sin r 1.44
50 fe + fe = 1.53

at
51 fe = 1.53  r = sin1(0.72) also BAD = 180  r
In quadrilateral ABCD,
 fe = 0.03 m
From equation (i) fo = 1.5 m  A +  B +  C +  D = 360°
 (180  r) + 60 + (180  r) +  = 360
 fo = 1.5 m and fe = 0.03 m
lic   = 2[sin1(0.72)  30]
1
113. Refractive index  sin r 1
(Temperature) 119. From graph, tan 30 = 1
sin i n2
ub
114. All colours are reflected. n 2 v1
 1n2  3    1.73  v1  1.73v 2
115. Yellow  Blue = Green n1 v 2
(Primary) (Primary) (Secondary)
Thus, option (B) is correct.
116. From the following figure 1
Also, from n =
P

sin i C
1
i  sin iC = Rarer
n Denser
r
et

 1 1
n  sin iC = 1
 .
n2 3
r + i = 900  i = 900 – r Hence, options (C) and (D) are incorrect.
rg

For ray not to emerge from curved surfac i > iC


120. Refractive index of liquid C is same as that of
 sin i > sin iC  sin (90o – r) > sin iC glass piece. So, it will not be visible in liquid C.
 cos r > sin iC
 1 r
Ta

1 121. tan iC =
 1  sin 2 r  ….(i) sin iC  n  h
r
n  
From Snell’s law,  r = h tan iC
sin  1 n2 1 iC
n= sin iC =  cos iC 
sin r n n
1 h
sin 2   tan iC =
 sin2 r = iC
n2 n 12

Substituting in equation (i), h


 r=
sin 2  1 1 n2 1
 1  2  2  1  2 (1  sin 2 )
n n n 7
=
 n 2  1  sin 2  16
1
 n  2 (as sin   1) 9
 Least value = 2 = 3 cm
113

MHT-CET Triumph Physics (Hints)


122. At point Q of ray PQ A A
but, r = where, A is angle of prism.
sin i 1 1 2
= =
sin r n 2 90 A
30  >C
sin 30 1 2
= P
sin r 2 30  A>2C
30 
 sin r = 2 sin 30 Q 128. Reflection takes place in the same medium.
1 1
= 2 = B C 129. Since light transmitting area is same, there is no
2 2
effect on intensity.
 r = 45,  = r  i = 45 – 30  = 15
130. If a mirror is placed in a medium other than air
123. Whenever any surface of convex or plano- R
convex or concavo-convex lens is silvered, it its focal length does not change as f = . But
2

ns
behaves like a concave mirror. Similarly
for the lens
whenever any surface of a concave or plano-
1  1 1 
concave or convexo-concave lens is silvered, it
fa
=  a
n g 1   
behaves like a convex mirror.  R1 R 2 

io
1  1 1 
fw
=  w
n g 1  
R R

n1  1 2 
x
As w n g  a n g , hence focal length of lens in water

at
n2
increases.
4
The refractive index of water is and that of
When ray travels from n1 to n2, 3
n 2 n1

n n
= 2 1
lic air is 1.
Hence, nw > na.
v u R
The ray refracts from R1 and falls normally of 131. e = 90, r2 = iC = 45
R2. Let the pin be placed at distance x from lens. A = r1 + r2
ub
i.e., u = x. A
r1 = A  r2
1.5 1 1.5  1 r1 = A  iC
  = i r1 r2 e
10  x 30 = 75  45 = 30
1 0.5 1.5 sin e sin e
 =  n= =
P

x 30 10 sin r2 sin i C
 x = 7.5 cm 1
Image of object coincides with the object itself  2 =
sin i C
as the ray after refraction from first surface falls
et

normally on second surface.  1 


 iC = sin 1  
 2
124. For prism, n = 1.5
 iC  42  iC = 45
rg

sin i
For ray B, angle of incidence in the prism is 45. n=
Hence, for ray B angle of incidence is greater sin r1
than critical angle. sin i
 2 =
Ta

sin 30
125. From the figure,
1 1
 + 2 = 180  sin i = 2  sin 30 = 2  =
and  = 2 2 2
  = 36  1 
 i = sin1   = 45
 2
126.
132. i = 2r
60 60
o o sin i
90o n=
60 o sin r
sin 2r 2sin r cos r
2= = = 2 cos r
sin r sin r
127. When angle of refraction exceeds value of
critical angle, no emergent ray is observed. 2 1
 cos r = =
Thus, r > C 2 2

114
114

Chapter 09: Optics


 1  137. This is case of total internal reflection.
 r = cos1  
 2  1
   i C   sin 1 
 r = 45  n
But, A = 2r for minimum deviation 1 R
 sin  vacuum
 A = 2  45 = 90 n
1 glass Q
v1 s /t s 2  sin 45 
133. 1
n2 = = 1 = 1= n
v2 s2 / t s2 3
n> 2
1 3 P
But 2 n1  1  c
n2 2  v
n
134. R r P Q c 3  108
 v 
2 2

ns
v < 2.1  108
 only (B) is not possible.
h
Competitive Thinking

io
i
2. All the rays will be incident normally on the
surface of the sphere. Hence, the rays will not

at
O be refracted but will pass through the sphere
Let the bulb be placed at point O. The light rays undeviated.
originating from it will spread at the surface of 3. The emergent ray will be parallel to incident ray
water as shown in the figure, forming a circle.
Angle of semi vertex ( i) here equals critical
lic only if the mediums have same refractive indices.
4. i = 2r
angle of water i.e.,  i =  iC
sin i
From the figure, PQ = PR = r, say, =
sin r
then, r = h tan iC
ub
 1  sin 2r
 1  =
h  h  sin r
h sin i C  nw   nw 
r= = = 2sin r cos r
cos i C 1  sin 2 i C  1 
2
=
1   sin r
P

 nw   
 cos r =  r = cos1  
 1  2 2
 sin i C  
 nw 
et

5. Using Snell’s law,


For h = 80 cm = 0.8 m and nw = 1.33, a sin i Incident Reflected
 1  ng= ray ray
0.8   sin r
r=  1.33  = 0.912 m
ng
rg

 1 
2
but ang = Air
1   na i r (na=1)
 1.33 
 na sin i = ng sin r
Area of circle = r2 = 3.142  (0.912)2 = 2.61 m2.  Glass
1 sin i = 2 sin r
Ta

r (ng= 2 )
135. When glass surface is made rough then the light 1
sin r = sin i
falling on it is scattered in different direction 2
due to which its transparency decreases. 1 1
= sin 45 = Refracted
2 2 ray
1
r = sin1   = 30
2
Smooth surface Rough surface
From figure, r +  + r = 180
i +  + 30 = 180 ….( i = r)
136. By focussing a lens, energy can be concentrated
into a small beam. This does not violate 45 +  + 30 = 180
principle of conservation of energy, as lens does   = 180  75 = 105
not generate energy but merely concentrates the Hence, the angle between reflected and
available energy. refracted rays is 105.

115

MHT-CET Triumph Physics (Hints)


R 1 2 1
7. f=  =
2 v 3f f
But for a plane mirror R =  1 1 2
 
If f is expressed in metres, then power of mirror v f 3f
is given as 1 3  2
1 1 
P= = =0 v 3f
f  v = –3f
8. When a plane mirror is rotated through angle , 15.
1 1 1
 
the reflected ray turns through angle 2. v u f f =15 cm
light spot  1 1 1
   O
v f u
1 1 40 cm

ns
y = 
15 40
2 1
Source x =
24
(L)
 v = –24 cm

io
From figure, Negative sign indicates image is formed in front
y of the mirror.
tan 2  2 = ….(Since angle θ is small)
x Given: u = –20 cm

at
y Now, according to mirror formula,
 =
2x 1 1 1 1 1 1
  =  = 
v f u 15 20 60
10.
lic  v = – 60 cm
Negative sign indicates that image is formed in
Object C f=4 front of the mirror.
Displacement of image
= v  v = 36 cm away from mirror
ub
20 16 12 8 4
u = 10 cm 16. Given: R = –15 cm, h1 = 10 cm, u = –10 cm
From mirror formula,
1 1 1
 
P

f v u
For a concave mirror formed, object is placed
1 1 1 2 1 2 1 1
beyond centre of curvature.    =  =  = 
Hence, the nature of image is inverted, real and v f u R u 15 10 30
 v = –30
et

unmagnified.
 v   30 
12. In case of mirrors, convex mirror always Magnification, m =  =–3
u 10
produces diminished (i.e., m < 1) and virtual
 The image formed is magnified and inverted.
images (i.e., m is positive).
rg

Hence, (a) can map to (iv) only. 17. For concave mirror,
Also, in mirrors, virtual image always forms on R 30
m = –3, f = =  = –15 cm
right hand side. Hence magnification produced 2 2
Ta

is always positive. (i.e., m for virtual image, m = Also, for spherical mirrors,
1
+ or m = +2). This means (d) can map to only  m 1  3  1 
u=  f=   ( 15) = (+ 4)(–5)
2  m   3 
(iii) and (iv).  u =  20 cm
These conditions are satisfied by option (C).
1 1 1
18. By mirror formula:  
13. As the medium has no role in focal length of u v f
mirror, it doesn’t change. As u > f, image formed is real,
1 1 1 1 1 1
14. By mirror formula, + =   
v u f u  v f
Considering sign convention, 1 1 1
 
1 1 1 3 v f u
 = ….(Given: u = – 1.5 f = – f) uf
v 3f f 2  v=
2 u f

116
116

Chapter 09: Optics


The image of the nearer end will be formed at Similarly, for oil filled beaker,
distance v, while the other end of rod is at infinite h
no = ….(ii)
distance, hence its image will be formed at focus. ho
2
uf f Dividing equation (i) by (ii)
 L = |v|  | f | = f =
u f u f nw h ho
 
19. Initial distance of object u0 = 50 cm n o hw h
As f =  25 cm, using mirror formula, 4 h
  o
1 1 1 1 1 1 3  1.6 hw
= – =  =
v f u 25 50 50  hw = 1.2 ho
 v =  50 cm. i.e., apparent depth of water is 1.2 times greater
Velocity of the object is given to be 1 m/s. than that of oil.
 The distance covered by the object in 0.25 s will

ns
be 25 cm. 27. Due to large refractive index of diamond
i.e., object distance at t = 0.25 s is, (n = 2.42), critical angle of diamond is very
u0.25 = 25 cm, small. This causes total internal reflection in
i.e., object is at focus of the mirror. diamond which makes it sparkle.

io
 v0.25 =  1 1
As image is formed at infinity, average velocity 28. sin iC = and n 
n 
of image formed between t = 0 s to t = 0.25 s For greater wavelength (i.e., lesser frequency) n

at
will be infinity. is less. Hence, iC would be more. Thus, these
1 1 1 wavelengths will not suffer internal reflection
20. From mirror formula,  
v u f lic and come out at angles less than 90.
R 1
We know, f =  29. As the beam just suffers total internal reflection
2 2
at interface of region III and IV, it almost grazes
1 1 region IV
  =2
v 1.5  i  90
ub
 v = s = 0.375 m Hence,
As the image distance is positive, image is virtual. n0 n n
Magnification of a mirror, n0 sin  = sin 1 = 0 sin 2 = 0 sin 90
2 6 8
 v 0.375 3 / 8 1
m=    = 0.25 1 1  1 
u 1.5 3/ 2 4  sin  =   = sin  
P

As magnification is positive the image is erect 8 8


(upright).
 1  1  1 
21. a
nw = x, wng = y, gna = z 30. iC = sin1  rarer  = sin  r 
et

 n denser   nd 
nw n n
 = x, g = y, a = z  
na nw ng  1 
1 n 
 iC = sin   = sin1  r 
n w ng na  nd
rg

 xyz =   =1   nd 
na nw ng n 
 r 
22. Of all the colours in spectrum, red shows least  1.5  1  15 
iC = sin1   = sin  
deviation.  1.6   16 
Ta

Real depth n n n
24. n= n2
Apparent depth 31. + 1 = 1 2
u v R
Let t be the real thickness of the slab, Assuming medium on the right hand side of the
Given apparent thickness = 3 + 5 = 8 cm convex surface to be air,
t
 n= 1.6 1 1  1.6
8 – =
u 5 3
i.e., t = 8  1.5 = 12 cm 1.6 1
Real depth  = 0.2 +
25. n= u 5
Apparent depth 1.6
 u=  =  4 mm
 In case of water filled beaker, 0.4
h Negative sign indicates that the object is placed
nw = ….(i)
hw on the left side of the convex surface.
117

MHT-CET Triumph Physics (Hints)


32. The convex lens can form enlarged and erect 1 1 1
 =  =2
image only when the object is kept between pole v2 1 1
and focus.
 v2 = 0.5 m
As f = 20 cm, u < 20 cm
 Distance from object = 4 + 3 + 0.5 = 7.5 m
33. When an equiconvex lens is cut symmetrically
along the principal axis, the two parts thus 39. The person to be able to see object at infinity,
formed have equal focal length. Hence, the the image should be formed at 400 cm.
power of the two parts also remains same.  u=
v =  400 cm =  4 m
35. Lens formula gives,
By lens formula,
1 1 1
  1 1 1 1 1
f v u   = 
f v u 4 
1 1 1

ns
   f=4m
f 75 25
As focal length is negative, the lens used is
1 100
  concave.
f 75  25
1
=  0.25 D

io
75 P=
 f= = 18.75 cm f
4
As the focal length is positive, the lens is 40. From lens formula,

at
convex. 1 1 1
 
v u f
1 1 1
36. Using lens equation,   fu
v u f  v= ….(i)
uf
Here, u =  25 cm
lic
Also, since image formed is virtual, v =  75 cm Now, magnification of lens is,
1  1  1 v
    = m=
75  25  f
u
ub
fu
 f = 37.5 cm  m= ….[from (i)]
 f u
u 
37.
f
 u–f=
m
P

 u=
 1 f
m 
S S m
O
u is kept same for both lenses,
5cm
et

 u=
 m1 1 f1   m 2 1 f 2
m1 m2
15cm f1 m1 1  m 2 

rg

10cm f 2 m 2 1  m1 
1 1 1
Using lens equation, – = 41. For bifocal convex lens:
v u f
1 1 
Ta

Substituting u = 10 cm, v = 15 cm, 1


= (n  1)   
1 1 1 f  R1 R 2 
– =  f = –30 cm
15 10 f (n  1)  2
= ….(R1 = R2 = R)
38. Using lens formula, R
1 1 1 For plane surface: R2 = 
  For half plane-convex lens:
v u f
For first lens: u1 = –4 m, f1 = 2 m 1 1
 (n  1)
1 1 1 1 f R
   =
v1 2 (4) 4 1/ f (n  1) R
= 2 =2
 v1 = 4 m 1/ f  R n 1
For 2nd lens: f
=2
 image formed by first lens will act like source. f
u2 = 1 m and f2 = 1 m  f  = 2f
118
118

Chapter 09: Optics


42. The focal length of a plano-convex lens is, 47. By lens maker’s formula,
R 1 1 1 
f=   a g  1   
n 1 fg  R1 R 2 
60 60
 f=  = 120 cm 1 1 1 
1.5  1 0.5 Also,   l g  1   
f liquid R
 1 R 2
43. According to lens maker’s formula, Dividing above equations,
 1 1 
1
= (n  1)    f liquid


a g  1

 g  1
=
(1.45  1)
= 3.9
f  R1 R 2  fg 
l g  1  g


 1
 1.45 
  1
1 1 1  1   1.3 
 = (1.5 – 1)   = f=R  l 
f  R  R R
48. By Lens maker’s formula,
44. Focal length of a convex lens is given as,

ns
1 3/ 2   1 1  1 1 1 
1  1 1  =  1   =   
= (n  1)    f1  4 / 3   R1 R 2  8  R1 R 2 
f R
 1 R 2 
1  3 / 2  1 1  1  1 1 
Given that, R1 = 2R2, n = 1.5 and f = 25 cm =  1   =   

io
f 2  5 / 3   R 1 R 2  10  R 1 R 2 
1  1 1 
 = (1.5  1)   
25 2R
 2  R 2 
1  3  1 1  1 1 1 
= 1    =   
 3 
f  2   R 1 R 2  2  R 1 R 2 
1

at
 =   f1 = 4f and f2 =  5f
25  0.5  2R 2 
25  3 49. When two convex lenses kept in contact, the
 R2 = = 18.75 cm
4 focal length of combination,
 R1 = 2R2 = 37.5 cm
lic 1
=
2
=
2
 1 F1 f convex f
1 1 
45. = (n – 1)    When the space between the lenses is filled with
f R
 1 R 2 
glycerin,
ub
1
As P = = 10 dioptre 1 2 1
f  in m  = +
F2 f convex f concave
 1 1  Now, in this case,
10 = (n –1)   
 0.1 0.1  R
P

fconvex = =R
1 2    1
(n – 1) =
2 R
3  F1 =
n= 2
et

2 Similarly,
46. According to lens maker’s formula, R
fconcave =  =R
1 1 1  2    1
= (  1)
 
rg

 2 1 1
f R
 1 R 2
1
 = – =
1  1.5   1 1  F2 R R R
=   1    F1 R 1 1
fa  1   R1 R 2   =  =
Ta

F2 2 R 2
1 1 1 
= (0.5)    ….(i)
fa R
 1 R 2 50. Focal length of combination,
  1 1  1 
 1.5  =  2 
1 1 1  f f concave  f convex 
=  1   
fw 4  R
 1 R 2 2(n oil  1)  n 1
3  =  2  lens 
   R  R 
1 1 1  2(1.7  1)  1.5  1 
= (0.125)    ….(ii) =  2 
fw  R1 R 2  R  R 
Dividing equation (i) by equation (ii) 1.4 1 0.4
=  
fw 0.5 R R R
 =
fa 0.125 R 20
 f=   =  50 cm
fw = 4 fa = 4  8 = 32 cm 0.4 0.4
119

MHT-CET Triumph Physics (Hints)


51. Focal length of first lens, 54. Focal length of combination,
1 1 1  1  1 1 1 1 1
= (1 – 1)   =   
f1   R  R f com f convex f concave f convex
Focal length of second lens, By lens maker’s formula,
1  1
= (2 – 1) 
1    1
  =– 1
1 1
 (n g  1)  
1 

f2  R   R f convex  R1 R 2 
So focal length of the combination,  3  2 
=   1  
1
=
1
+
1  1
= 1
   1
– 2  2 R 
f f1 f2 R R 1 1
 = ...(i)
1   f R
= 1 2
f R 1 1 1 
R  (n w  1)   

ns
 f= f concave R
 1 R 2
1   2
1  1   2 
    
52. According to lens maker’s formula, f concave  3   R 
the focal length of plano-convex lens is 2

io
= ...(ii)
1 1 1  3R
= (n  1)   
f   R  1 2 2
 =  ...[from (i) and (ii)]
1  1 1  0.6 f com R 3R

at
 = (1.6  1)   =
f1   R  R But R = f ...[from (i)]
Similarly focal length of concavo plane lens is 3R 3f
 fcom = 
1  1 1 4 4
= (n  1)   
f  R  
lic 55.
1  1 1 0.5 R
 = (1.5 1)   = R
f2  R   R
For the combination of lenses, nw
ub
1 1 1 0.6  0.5  0.1 R 1  1 1 
= + =    = f=   n w 1   
f f1 f 2 R  R  R 0.1 fw  R R
1  2
53.    n w 1   
P

fw  R
1 1 1
But,   n l 1   
f1 f2
fl R R
et

P 4 2
Given: 1  = (nl – 1)  
P2 3 R
1 2 1
As P =  
rg

f R  n l  1 f l
1 4
  f2  1 n 1  1 
f1 3   w  
fw n l 1  f l 
Ta

f2 4
  1 1 2 1  n w 1 
f1 3     
f eq F fl f l  n l 1 
4
 f2 =  12 ….(Given: f1 = 12 cm)
3 1 n w 1 
= 2 
f2 = 16 cm fl  n l 1 
As the lens is concave, Given: nl > nw
f2 = –16 cm nl – 1 > nw – 1
then, focal length of combination is given by n w 1
 1
1 1 1 n l 1
 
f eff f1 f 2 1 1 2
  
1 1 1 43 fl F fl
  
 f eff 12 16 48 f
  F f
feff = 48 cm 2

120
120

Chapter 09: Optics


56. n = 1.5 n = 1.2 59. After refraction at two parallel faces of a glass
slab, a ray of light emerges in a direction
parallel to the direction of incidence of white
light on the slab. As rays of all colours emerge
1 2 in the same direction (of incidence of white
light), hence there is no dispersion, but only
lateral displacement.
R1 R2
60. Angle of deviation decreases initially with
1 1 1  increase in angle of incidence, attains minimal
(n  1)    value. On further increase in angle of incidence,
f1 R
 1 R 2
angle of deviation increases.
For n = 1.5 and R1 = 14 cm
1 1 1 0.5 61. Angle of prism, A = 60°

ns
= (1.5  1)    =
f1 14   14 For minimum deviation, angle of refraction,
For n = 1.2 and R2 = 14 cm A 60o
r = =
1 1 1  0.2 2 2
= (1.2  1)    = = 30° for both the colours

io
f2   14  14
1 1 1 0.5 0.2 0.7 62. Given: e = 0
=  =  =
f f1 f 2 14 14 14  r2 = 0, A = r1

at
1 1 1 Since ‘i’ is small, Snell’s law of refraction can
Using lens equation,   be modified to,
v u f
i

1
=
7

1 =
r1
v 140
1 1
40
lic  i = r1 = A
= 
20 40 63. Given: i = 60, A = 60
1 2 1 At minimum deviation position,
 =
ub
v 40 A  m
 v = 40 cm i=
2
57. For lens separated by distance d,  m = 2 i – A = 60
1 1 1 d Using prism formula,
  
P

f f1 f 2 f1f 2  A  m 
sin  
1 f1  f 2 d n  2 
 =  A
f f1f 2 f1f 2 sin  
et

2
1 f1  f 2  d
 = sin  60 
f f1 f 2 =
sin  30 
1
But, P = (if focal length is measured in
rg

f = 3 = 1.732
metres)
64. From the given data,
f1  f 2  d
 P= i+e=A+
f1f 2
Ta

A = i + e   = 35 + 79  40 = 74.


Thus, for P = 0, d = f1 + f2
 A  m   A 
sin   sin  
58. The image formed by diverging lens will be Now, n =  2 
  2 
virtual and at a distance v1 = – 25 cm. A A
sin   sin  
This image acts as an object for the converging 2 2
lens.  74   40  
 u2 = –25 + (–15) = –40 cm sin  
 n <  2 
 By lens formula,  74 
1 1 1 sin  
   2 
v2 u 2 f 2 sin 57
1 1 1  n<
   sin 37
v 2   40  20 n < 1.39
 v2 = +40 cm from the converging lens. The nearest value amongst given options is 1.5
121

MHT-CET Triumph Physics (Hints)


65. 1  60o   m 
2 = sin  
60 2  2 

45 1  60o   m 
45 = sin  
2  2 
 60o   m 
sin (45°) = sin  
 2 
90° = 60° + m or m = 30°
As ray suffers minimum deviation,
A  m 60o  30o
i=e i= =
2 2
 m = (i + e)  A = (45 + 45)  60 = 30
 i = 45°
 A  m 
sin  

ns
n =  2  68. At the minimum deviation m the refracted ray
A inside the prism becomes parallel to its base.
sin  
2  A  m 
sin  
  30 
60  2 
sin   n=

io
 2 = 1 2= A
= 2 sin  
 60  2 2
sin  
 2   60o   m 

at
sin  
66. m = A, n = 1.5  2 
3 =
 A  m   60o 
sin  sin  
  2 
n=  2 
A
sin  
lic o  60o   m 
3 sin 30 = sin  
2  2 
 2A 
sin   3  60o   m 
 2  = sin  
=
ub
2  2 
A
sin  
2 60o   m
60 =
A A 2
2sin   cos  
sin A 2 2 m = 60°
= =
P

A A As m = 2i  A,
sin   sin  
2 2 where i is the angle of incidence
A  i=
 n = 2 cos  
et

m  A
2  =
2
A
 1.5 = 2 cos   60  60o
o

2 = = 60°
rg

2
3 A
 = cos
4 2  A  m 
sin  
A  2 
  cos 1  0.75 69. n=
Ta

2 A
sin  
= 90° – sin1 (0.75) 2
= 90  4836 = 4124  A  m  A
sin   = n sin  
 A = 8248  2  2
 A  m   60o   m   60o 
sin   sin   = 1.6 sin  
 2   2   2 
67. n=
A  60o   m 
sin   sin 
2  = 0.8
 2 
Substituting the values,
60o   m
 60o   60o   m  45° < < 60°
2 sin   = sin   2
 2   2  90° < 60° + m < 120°
….( Prism is equilateral) 30° < m < 60°

122
122

Chapter 09: Optics


70. Using prism formula, 72. For thin prism,
 A  m 
sin 
 = (n – 1)A

n=  2   3.6 = (1.6 – 1)A
A  A = 6
sin  
2
 A  m 
74. Magnifying power for simple microscope when
sin   image is formed at infinity,
A
.…  n  cot
 2  A
cot =  D 25
2 A
sin    2 M=  =2
2 f 12.5
 A  m  A A  D
 sin   = cot sin L
 2  2 2 76. M= 1  
fo  fe 
A
cos 150  250 
 A  m 

ns
 2 sin A 
sin  = 375 = 1  
 2  sin A 2 5  fe 
2
....( D = 25 cm = 250 mm)
 A  m   A
 sin   = sin     fe = 21.74 ≈ 22 mm
 2  2 2

io
A  m  A
 =  77.
2 2 2 fe= 5 cm
c
 A + m =   A

at
 m = 180 – 2A f0 = 1 cm

71. A
x vo ue
lic
Image
Object
12.2 cm
 r1
r2
ub

Given: f0 = 1 cm, fe = 5 cm,


B C L = v0 + ue = 12.2 cm,
Using Snell’s law, ve = –25 cm
P

sin  =  sin r1 For eyepiece,


sin  1 1 1
 sin r1 =   ….(i)
 ve u e fe
et

 sin   1 1 1 1 1 6
 r1 = sin1    =  =  =
   ue ve f e 25 5 25
 sin   25
 r2 = A  sin1  
rg

 ....(i) ue = cm
   6
1 As ue is on left side of eyepiece, from sign
 r2 < sin1   conventions, ue is negative. Hence, neglecting

Ta

negative sign,
Substituting for r2 in equation (i), 25
 sin   1  1 
ue = cm
 A  sin1   < sin   6
    As, L = v0 + ue = 12.2 cm
1  sin   25
 A  sin1   < sin1    v0 = 12.2  = 8.03 cm
    6
  1   sin  For objective,
 sin  A  sin 1     1 1 1
      
v0 f 0 u 0
   1   
  sin  A  sin 1      sin  1 1 1 1 1 7.03
          =  =
u 0 v 0 f 0 8.03 1 8.03
   1    8.03
 sin 1  sin  A  sin 1     <   u0 =  = –1.14 cm
       7.03

123

MHT-CET Triumph Physics (Hints)


80. Given: fo = 40 cm, fe = 4 cm sin i sin 2A
 n= =
For objective, sin r sin A
1 1 1 2sin A cos A
  = = 2 cos A
vo u o fo sin A
1 1 1
   84. The ray will retrace its path from mirror if it
vo 200 40
falls normal to the surface of the mirror.
1 1 1 5 1 1
    A
v 0 40 200 200 50
vo = 50 cm
30
For normal adjustment, L = vo + fe = 54 cm r P
O
81. As, m = fo/fe, fo = m  fe = 10  3 = 30 cm 30 = r
For an object at 180 cm from objective, the

ns
image formed by objective is at a distance vo.
1 1 1
  B C
fo vo uo

io
1 1 1  AOP = 60 and angle of refraction = 30
  
v o 30 180 Using Snell’s law of refraction,
1 1 1 sin i
   n=

at
v o 30 180 sin r
vo = 36 cm  sin i = n sin r = 2  sin 30
Now if this image is made to fall at focus of 1
eyepiece so that final image is at infinity, the =
lic
total length of telescope would now be
 1 
2

L = vo + fe = 36 + 3 = 39 cm  i = sin–1   = 45
 2
82.
85. By Snell’s law of refraction,
ub
L sin i
n=
sin r
I  sin 40 = 1.31 sin θ2
f0 + fe 0.64
 sin θ2 = = 0.49 ≈ 0.5
P

1.31
Magnification of telescope:  θ2 = 30
f
M= o
et

fe 2m
For a convex lens: x
fe I
|M|= 
fe  u O 20 m n = 1.31
rg

The object being line on objective, u = fo + fe θ2


and O = L 40
fe I
 
Ta

f e  (f 0  f e ) L
fe I Let reflection take place at every x m length of
  fibre.
fo L
x
L Now, cot θ2 =
 M= 20 106
I
 x = cot θ2× 20 × 10–6
83. Normal incidence at silvered surface  Number of reflections
2 2
= 
A x 20  10 6  cot  2
(90  A)
=
2
20  106  3

….  cot 30  3 
2A A
= 57735
Nearest option is option (D).

124
124

Chapter 09: Optics


86. From given data, for convergent lens, 1
u = – 40 cm, f = + 20 cm  v= m
15
 Using lens equation, As the object oscillates along the axis with
1 1 1 amplitude A,
 
f v u du = A cm = A  102 m
1 1 1 1 1 1 Differentiating the equation (i), as f remains
   = + = constant,
v f u 20 40 40
dv du
 v = + 40 cm  2
=  2
As, v = – u, v u
v v2
m= =–1  Amplitude of image, Aimage = dv = A 
u u2
1 / 15
2
This implies, image formed is of same size as
 Aimage = A  102 

ns
that of object but inverted. 1 / 5
2

2
 5 
=    A  102
20 cm  15 

io
A
=  102 m
F 9
2F F1 2F1 F
89. Initially image is formed at distance

at
20 cm
vi = 2f = 10 cm  f = 5 cm
40 cm
Due to glass plate,
60 cm
 1  2
Shift = t 1   = 1.5 1  
mirror.
lic
Now, this image acts as an object for convergent n  3  
= 0.5 cm
From figure,  u = OP = 9.5 cm
u = – 20 cm, f = – 10 cm 1 1 1
 
ub
 Using mirror formula, v u f
1 1 1 1 1 1
     ….( Using sign convention)
f  v u v 9.5 5

1 1 1
  =
1

1
=–
1  v = 10.55 cm
P

v f  u 10 20 20  Shift = v – vi = 0.55 cm away


 v = – 20 cm
90.
This implies, the final image formed is at 20 cm
in front of mirror.
et

nl A = 60
As, v = u,
v i e
m=– =–1 r
u
rg

Since, object is inverted, the final image thus ng


formed is erect and of same size as that of
object.
Ta

At minimum deviation,
1 1
87. n= = = 2  A  m 
sinic sin 45 sin  
l
ng =  2 
Velocity of light in medium, A
c sin  
v= 2
n
 60  30 
3108 sin  
 v= m/s ng  2 
2 =
nl  60 
sin  
88. Given that, u =  0.2 m and f = 0.05 m  2 
1 1 1 sin 45 1
   ....(i) =  2
v f u sin 30 2
1 1 n 1
=  = 15 i.e., l  ….(i)
0.05 0.2 ng 2

125

MHT-CET Triumph Physics (Hints)


Now critical angle for prism – medium Consider the ray AB is incident on plane P1.
interface, After reflection the ray takes the path BD and
nl 1 passes through point D (3, 3). If the reflected
sin (iC) = = ….[from (i)]
ng 2 ray is extended below X-axis, it intersects the
Y-axis at point C (0, –1).
 1 
 iC = sin1   Hence, the path length of the ray can be
 2 calculated from C to D using Pythagoras
 iC = 45 theorem for CED,
91. In case of critical angle, CD2 = CE2 + DE2
1  CD = (4) 2  (3) 2 = 5 units
n=
sin i C
94.
For symmetry iC = 45 

ns
1
 n= = 2 = 1.414 d 
sin 45 D
nr = 1.39
f
ng = 1.44
x

io
nv = 1.47
nr < n = 1.414 From geometry of given figure,
while, nv > n and ng > n D d
= 

at
Hence, only red colour part will not undergo x f
total internal reflection and emerge out D
separately, while blue and green parts will suffer  d= f
x
total internal reflection. D R
92. A
lic = 
x 2
where, R is radius of curvature.

 D 
 d = 0.009  0.2 ….  Given:  0.009 
 x 
= 1.8  103 m
ub
i
i
95. The rays incident from object on the lens travel
parallel after refraction. These parallel rays are
B C incident on plane mirror and trace back their
P

path after reflection.


For total internal reflection. Hence, the final image will be formed on object
i > iC ….(i) itself.
Also, from the symmetry of diagram,
et

96. A
 i = 45
 sin i > sin iC ….from(i)
1 1 1 R
  but , ang=
rg

sin i sin i C sin i C R1


1 0.3cm
 < ang C
sin  45  6 cm
O R
Ta

a
 2 < ng
 Minimum value of ang= 2
93. Y

(0, 3) D (3,3) B
E
c 3108
(0, 1) A R. I. of lens, n = = = 1.5
v 2108
As D1 = 6 cm, R1 = 3 cm.
(0, 0) X  From ΔACO, radius of curvature of lens is,
B
P1 R2 = R12 + (R  0.3)2
(0, –1) C
R2 = 32 + (R  0.3)2
R2 = 9 + R2 + 0.09  0.6 R
0.6 R = 9.09
126
126

Chapter 09: Optics


R = 15.15 cm. 100. According to lens maker’s formula,
R the focal length of plano-convex lens is
F=
n 1 1 1 1 
= (n  1)   
=
15.15
= 30.3 cm f    R
1.5  1 1  3  1  1 f
=   1  = R= ….(i)
c f 3 f  2  R  2R 2
97. n = = air =
v f med 2 The focal length of liquid lens is
Given, v = +8 m, 1  1 1
 (n l 1)   
1 v fl  R  
m= =
3 u 1  n 1 
Glass
= l  n plate
 u = 24 m. fl  R 

ns
1 1 1 1  1  2(n l  1)
Using formula, =  =   1
= [using (i)]
f v u 8  24  fl f
1 4
= Effective focal length of the combination is
f 24

io
f = 6 m. 1 1 1
= +
Using lens maker’s equation, 2f f fl
1  1 1  1 1 2(n  1)
=  1

at
  n  1   
f R R 2f f f
 1 2 
1 1
For plano-convex lens, R1 = , R2 = R.  2(n1  1) = 1 =
2 2
1 1
  n  1

f R
lic 1 5
 n1  1 =  n1 = = 1.25
4 4
 R = f (n  1)
= 6 (1.5  1) 101. By lens maker’s formula,
=3m 1 1 
ub
1
= (ang  1)   
98. When a convex lens is introduced, object forms fa  R 1   R 2  
two images.  2 
2 = (1.5 – 1)  
One is diminished, I1 = cm  20 
P

3 0.5
and another is magnified, I2 = 6 cm  Pair =
10
Magnification for magnified image (m2) and
Similarly, when the lens is immersed in a liquid,
that for diminished image (m1) are related as
et

1 1  ng   1 1 
m2 =   1   
m1 f liquid  n l   R1   R 2  

 m1m2 = 1 
1  1.5

 1 
1   
rg

I1 I2 f liquid  1.25  10 


  1
O O 1
 O2 = I1I2  Pliquid =
50
Ta

i.e., O = I1I 2 Pair 50 5


   0.5 =
2 Pliquid 10 2
hence, size of object O =  6 = 2 cm
3 102. For combination of lenses,
99. (lens + cornea) forms an image of distance 1 1 1 1
  
object at retina. F f1 f 2 f 3
 converging power (40+20) D = 60 D 1 3
 =1
From Lens equation, F 3
1 1 60 Total magnification,
 
v  100 D
M=1+
5 F
 v = cm
3 25
=1+ = 26
 v = 1.67 cm. 1

127

MHT-CET Triumph Physics (Hints)

Hints to Evaluation Test

1. The field of view is maximum for convex mirror When lens is in water
because the image of an object formed by a 1 n  1 1 
convex mirror is always diminished. Each =  g  1  
fw n
 w R
 1 R 2 
image is thus confined to small area and many
objects can be viewed in the mirror. 1.5  1.33   1 1 
= 
1
    ….(ii)
fw  1.33 R
 1 R 2 
2.
Dividing equation (i) by (ii),
2 cm fw  1.33 
 = (1.51)  

ns
C 30  1.5  1.33 
2m u 1.33
fw = 30  0.5 
wall 0.17
= 117.35 cm

io
Let the candle C be placed u metre away from The change in focal length
pole of the mirror. = 117.35 – 30
According to question, image distance = 87.35

at
v=u+2  87.4 cm
Also, magnification of a concave mirror
v 7. Magnifying power of a telescope in normal
m= fa
u
  u  2 image height
lic adjustment =
fe
= =
u object height Tube length = Distance between objective and
Here, negative sign indicates, image is inverted. eyepiece
ub
u2 6 = fo + fe
 |m|= = u=1m fo
u 2 = 9  fo = 9fe
Distance of the wall from the mirror is fe
u + 2 = (1 + 2) m = 3 m = 300 cm. Tube length = fo + fe
P

1 1 1 60 = 9fe + fe = 10fe
3. For near end the bar, =   fe = 6 cm and
f u v
Here, u and f are negative fo = 9fe = 9  6 = 54 cm
et

uf
 |v| = 8. As shown in the figure, the system is equivalent
u f to combination of three thin lenses in contact
Far end of the bar is at infinity. Therefore,
1 1 1 1
image will be formed at focus.    
rg

f f1 f 2 f 3
 Length of the image = | v | f
By lens maker’s formula
uf f2
= f = 1  3  1 1  1
u f uf    1   
Ta

f1  2   20  40
5. We cannot interchange the objective and eye A B
1  4  1 1 1
lens of a microscope to make a telescope. The    1   
focal lengths of lenses in microscope are very f 2  3  20 10  20
small, of the order of mm or a few cm and the 1  3  1 1 1
   1  
difference (fo  fe) is also very small. While in f 3  2  10   20
the telescope, objective has a very large focal
1 1 1 1 1
length.    
f 40 20 20 40
6. When the lens is in air,  f =  40 cm
1  1 1 
=  n g  1    Hence system behaves as concave lens of focal
fa R
 1 R 2  length 40 cm.
1  1 1  10. A dentist uses concave mirror to converge light
= 1.5  1    ….(i)
30  R1 R 2  and obtain enlarged image.
128
128

Chapter 09: Optics


11. For a given compound microscope, 16. If the distance travelled by a ray of light in two
v D media are s1 and s2 in the same time ‘t0’ then the
M= o ratio of refractive index of the 2nd medium to 1st
uo ue
medium is given by
and L = vo + ue
v1 s
When L is increased, ue increases as vo is fixed. 1
n2 = = 1
Hence, its magnifying power decreases. v2 s2
n 2 s1
12.  
n1 s 2
B i = 60 4
Refracted ray  n2 = 1.5   1.25
4.8
r r
17.

ns
iC refractive index of the rod
Reflected 1

ray
r O
60 3 Here incident angle is .

io
n2 =
The light ray will graze along the rod, if it gets
n1 = 1
incident on rod at ciritical angle and will get
reflected internally as shown in the figure

at
At B:
above.
n2sin r = n1sin i
1
 3 sin r = 1 sin 60 If iC is the critical angle, iC  sin 1
n
 r = 30
lic But iC = 90  1.
 Angle between reflected ray and refracted ray at
B is 90. From Snell’s law,
sin  sin 
=n= 3 = n.
ub
13. As, m = fo/fe, fo = m  fe = 20  2 = 40 cm sin 1 cosi C
For an object at 200 cm from objective, the But
image formed by objective is at a distance vo.
1 1 1 n2 1  1
  cosiC   sin iC  
n  n 
P

fo vo uo
1 1 1 n2 1
    sin  = n  n2 1
vo 40 200 n
et

 vo = 50 cm  2
  = sin 1 3  1  sin 1
Now if this image is made to fall at focus of
eyepiece so that final image is at infinity, the Thus, for  = sin1  2  , light ray grazes along
rg

total length of telescope would now be the wall of the rod.


L = vo + fe = 50+ 2 = 52 cm
18. As ABC is an isosceles right angled prism,
14. Focus alone depends on whether the rays are
angle of incidence of each ray is 45. If critical
Ta

paraxial or not. The rest of the three factors do


angle for a colour, iC is less than 45, the ray of
not depend on whether the rays are paraxial or
that colour will be totally internally reflected at
not.
AC. When  iC > 45, the ray will be
15. When light is incident from core (higher transmitted through the face AC.
refractive index medium) to cladding (lower For red ray, n = 1.39
refractive index medium), the condition for total 1 1
n core 1 sin iC = = = 0.719  iC = 46.0
internal reflection of light is,  R n 1.39 R

n cladding sin i C
Hence, red ray will be transmitted.
If the angle of incidence of ray(y) in the core to For blue ray, n = 1.47
cladding interface is greater than the critical 1 1
angle iC, the ray is totally internally reflected sin iC = = = 0.68  iC = 42.8
B n 1.47 B
i.e., y > iC. Also, for this condition, x < critical
angle. Hence, blue ray will be reflected at face AC.

129

MHT-CET Triumph Physics (Hints)


19.
A
90 – 



B C
1 1 2
sin iC = = =
n 1.5 3
2
 > iC  sin  > sin iC  sin  >
3
But  +  = 90

ns
  = 90 – 
2
 sin (90 – ) >
3
2
cos  >

io
3
2
  < cos–1  
3

at
2
 Largest value of  is cos–1  
3

20. For light waves, medium in which waves travel


with lesser velocity is said to be denser medium.
lic
The velocity of light is more in water than in
diamond. Hence water is rarer than diamond.
ub
1 1
21. Here, sin 1    0.6666
ng 3 / 2
1 1 3
And sin 2     0.75
nw 4 / 3 4
P

As ng > nw
 1 < 2
If  is the critical angle between glass and water
et

then,
nw 4 / 3 8
sin      0.8888
ng 3 / 2 9
  > 2.
rg

22. nv > nb > ng > ny


1
But iC  sin 1  
Ta

n  
  iC  y   i C g   i C  b   i C  V

130
130

Chapter 10: Electrostatics

10 Electrostatics

Hints

31. F = qE  ma = qE
Classical Thinking qE 0.5  200
 a= = = 200 m/s2
m 5  101

ns
1
10. F ; so when distance is halved, the force 1 q
r2 32. E=
becomes four times. 4πε 0 r 2

11. The same force will be acting on both bodies 9  109  1.6  1019
= = 1.44  1011 N/C

io
1010 
2
although their directions will be different.
q 1q 2 q1q 2 12  109
17. Fa = , Fb = 33. E1 = 9  109  = 12 N/C
4 0 r 2 4 0 kr 2  3 2

at
 Fa : Fb = k : 1 6  109
E2 = 9  109  = 13.5 N/C
Fa (2)2
18. k=
Resultant intensity E = E2  E1 = 1.5 N/C
Fm
Fa
lic 34. Using Shortcut 2
Fm = =2N
k x x x
x1 = = =
Q2 4e 3
1 q q 1 1
ub
19. F=  12 2 Q1 e
4 0 r
9  109  1.6  1.6  1038 43. S.I. unit of electric flux is
=
 5.3  10 
11 2 N  m2 J  m
 = volt  m
C C
P

= 8.2  108 N
51. Two equal and opposite charges form an electric
20. q = ne dipole.
1 = n  1.6  1019
52. Dipole moment,
et

1
n= = 6.25  1018 p = q (2l)
1.6  1019 = 3.2  1019  (2.4  1010)
21. q = ne = 7.68  1029 C m
rg

= 4  1016  1.6  1019 54. In non-uniform electric field, the dipole experiences
= 6.4  103 = 6.4 mC two unequal and opposite forces. Hence a net force
since electrons are removed, the conductor acts on it. Also, the two forces act on different
acquires positive charge.
Ta

points of the body, giving rise to torque.


22. 1 C  6.25  1018 electrons 57. Maximum torque = pE = q  2l  E
96500 C  x electrons = 2  106  3  102  2  105
x = 96500  6.25  1018 = 12  103 N m
= 6.03  1023 electrons q 10  106
i.e., 1 mole of electrons 62. = = = 2  10 6 C/m
l 5
28. Three equal forces inclined equally at a given q 20×106
point give zero resultant. 65. = =
4πr 2 4× 3.14×102
F mg = 1.59  104 C/m2
29. E= =
q e
q 2  106
F mg 1.7 ×1027 × 9.8 67. = =  0.48 C/m3
30. E= = = 19
 107 V/m V 4
 3.14  1  10 6
q q 1.6×10 3

131

MHT-CET Triumph Physics (MCQs)


0.075  9
q1 q2 = = 0.075  10–9 = 75  1012
Critical Thinking 9  109
….(i)
1. Since both are metals so equal amount of charge Given, q1 + q2 = 20  10–6
will be induced in them.
 q1 = 20  10–6 – q2
2. There is loss in electrical energy on sharing of substituting in equation (i),
charges because a part of electrical energy is (20  10–6 – q2) q2 = 75  10–12
converted into heat or light energy during the  20  10–6 q2 – q22 = 75  10–12
process of charge transfer from one body to  q22 – 20  10–6 q2 + 75  10–12 = 0
another.
 q22 – 15  10–6 q2 – 5  10–6 q2 + 75  10–12 = 0
3. The attraction between pair (2, 4) indicates that  q2 (q2 – 15  10–6 ) –5 10–6 (q2 – 15  10–6) = 0
ball 2 is oppositely charged as that of ball 4.  q2 = 15  10–6 or q2 = 5  10–6

ns
Repulsion between ball 1 and 2 indicates ball 1  Two charges are 15 C, 5 C.
will have charge similar to ball 2 and thus
opposite to ball 4. Hence ball 1 and 4 will show 11. New charges will be –2 C and +3 C.
attraction. As ball 1 repels ball 3, ball 3 also has 1 3×8
In first case, 40 =  2

io
charge similar to ball 1. Hence ball 3 and 4 will 4πε 0 r
show attraction. 1 (2×3)
In second case, F = 
5. When a positively charged body is connected to 4πε 0 r2

at
the earth, electrons flows from earth to body and F 2× 3
body becomes neutral.  =
40 3×8
7. F = –10 N
q
r
Qq
lic 12. Initially,
Q Q
q(Q  q) A B
F = 9  109  r
ub
r2
F will be maximum if product q(Q  q) is q2
F=K ....(i)
Maximum. r2
q(Q  q) will be maximum if q = Q  q Finally,
 Q = 2q
P

Q/2 Q/2 Q
1 q1q 2 FB FA
8. Fnet = (110/100) (90/100) times
4 0 r 2 A C B
r/2 r/2
et

99
i.e., times
100
Force on C due to A,
99
 Net force =  100 = 99 N K(Q / 2) 2 KQ 2
100 FA = = 2
rg

(r / 2) 2 r
9. x
Force on C due to B,
+4q q0 +q KQ(Q / 2) 2KQ 2
FB = = 2
Ta

a (xa) (r / 2) 2 r

4q×q 0 q×q 0  Net force on C,


F= =
a2 (x  a) 2 KQ 2
Fnet = FB  FA = =F
4(x  a)2 = a2 r2
2(x – a) = a
13. Force between the two charges is independent
2x – 2a = a
of the charges in its surroundings.
2x
3a = 2x  a =
3 14. The force of repulsion between +2Q and +Q is
1 q1q 2 directed along ve Y-axis, while the force of
10. F= attraction between +Q and Q is directed along
4 0 r 2
+ve X-axis. Considering the magnitudes of the
9  109 (q1q 2 ) two forces, net force will act downward as
0.075 =
(3) 2 shown in figure (C).
132
132

Chapter 10: Electrostatics


15. F  q1q2 FA  (5) (+10) = 50 Fnet = 202  152
FR  (5) (20) = +100 Fnet = 25 dyne
As FR > FA In I  Left or Right depends
on distance. 20. FA = Force on C due to charge placed at A
In II  Always Left 106  2  106
= 9  109  = 1.8 N
In III  Always Right 10  10  2 2

q1 q 2
16. F FB = Force on C due to charge placed at B
r2
106  2  106
= 9  109  = 1.8 N
FAB  82 …(i)  0.1
2

x
FAD  82 …(ii) FA
y +2 C

ns
FBD  216 2 …(iii) C 120°
x y
FB
FAC  24 2 …(iv)
x y

io
+1 C 1 C
From all equations given above we can say that A 10 cm B
the electric force will be least between charges Net force on C is given by
A and C. Fnet = (FA ) 2  (FB ) 2  2FA FB cos120

at
17. The force between 4q and q = 1.8 N
1 4qq
F1 = . 21.
4 0 l 2 QA = Q q QB = Q
The force between Q and q
lic A FC B
FA
1 Qq x C x
F2 =
l / 2
2
4 0 2x
For, F1 + F2 = 0
ub
For equilibrium, net force on Q = 0
 F1 =  F2
1 Q2 1 Qq
4q 2 4Qq + =0
 = 4 0 4x 2
4 0 x 2
l2 l2
 Q =q Q

P

q=
4
18. The position of the sphere will become as 22.
shown below, FC
180° FD
et

L L +q +q FAC
+Q +Q A FA
B
1 Q2
rg

Thus angle is 180° and force =


 2L 
2
4 0

19. A +15 e.s.u D +q +q C


Ta

Fnet = FAC + FD = FA2  FC2 + FD


3 cm
FC C kq 2
20 e.s.u Since FA = FC = and
+12 e.s.u B 4 cm a2
kq 2
Fnet = FA  FC
2 2
FD =
a 2 
2

FA
2kq 2 kq 2
In e.s.u. system,
1 Fnet = +
1 a2 2a 2
4 0
15  12  1  kq 2
 FA = = 20 dyne and =  2  2
 2 a
 3
2

12  20 q2 1 2 2 
FC = = 15 dyne =  
 4 40a 2
2
 2 

133

MHT-CET Triumph Physics (MCQs)


  Q2 28. Using Shortcut 2
23. | F B | = | F C |  K.
a2 x 11
x1 = = = 5 cm
FC
Q2 36
FC sin 60o 1+ 1+
o
FC cos 60 60o Q1 25
FB cos 60o A
60o q  1 
FB 60
o
FB sin 60o 29. | EA | = | EB | = K  where,K  
a2  4 0 
EB Enet EA
60o 60o +Q
–Q C
a C
B 60°
a a
Hence force experienced by the charge at A in

ns
the direction normal to BC is zero.
q q
24. The schematic diagram of distribution of A a B
charges on x-axis is shown in figure below: 3 Kq
So, Enet = E 2A + E B2 + 2E A E Bcos60o =
a2

io
1C 1C 1C 1C 1C 3q
Enet =
4πε 0a 2
O x =1 x =2 x =4 x=8

at
30. In curve (A), field lines are not drawn normal to
the conductor. Hence, curve (A) cannot
Total force acting on 1 C charge is given by,
represent field lines.
1 1  1  106 1  1  106
F = 
4 0  (1) 2

(2) 2
lic In curve (B), electrostatic field lines are shown
forming closed loops. Hence, curve (B) cannot
1  1  106 1  1  106  represent field lines.
 2
 2
 ....  In curve (D), field lines are shown intersecting
(4) (8)  each other. Hence, curve (D) cannot represent
ub
106  1 1 1 1  field lines.
      ... 
40  1 4 16 64  Amongst the given curves, only (C) can
represent field lines.
Using Binomial theorem, 31. Electric lines of force never intersect the
P

  conductor. They are perpendicular and slightly


 1 
 6
F = 9  10  10 
9
 curved near the surface of conductor.
1 1 
32. Electric field inside a conductor is always zero.
et

 4
4 4 33. C
 F = 9  109  106   9  103  = 12000 N
3 3
EB EA
rg

25. At A and C electric lines are equally spaced and


dense.
M EC
 E A = EC > EB
Ta

26. At mid point, E = 0 A B


Before mid point, E is positive. This is Let EA, EB and EC be the electric field at M due
maximum near the charge and decreases to charge placed at A, B and C respectively.
towards mid point.  
After mid point, E is negative, The curve As seen from figure | E B |  | E C | so net electric
crosses x-axis at x = d/2. From centre to end, E field at M, in the direction of vector 2.
decreases. C
34. D
The variation is shown by curve (D).
EB
27. Electric field due to a point charge, EA
q EC O
E ED
4 0 r 2
2
1  30  –11 A B
 q  E  40 r 2  2    = 2  10 C
9  109  100  E A  E,E B  2E,E C  3E,E D  4E

134
134

Chapter 10: Electrostatics


Adding them vectorially, 45. The electric field is always perpendicular to the
D C surface of a conductor. On the surface of a
metallic solid sphere, the electrical field is
oriented normally (i.e. directed towards the
centre of the sphere).
2E 2E
Enet 46. T.N.E.I. = q enclosed

A B  T.N.E.I. for A = zero


T.N.E.I. for B = (2q  q) = q  (0, q)
Thus, direction of electric field at the centre of
the square is along CB. 47. T.N.E.I. over the closed surface
= q = 5 + 7  4 = 8 C
35. E= 1  q = 9  109  q

ns
40 r
2
r2 48. Electric dipole moment of a water molecule
p = 6.4  1030 C m
q = E  r 9 = 3  10  (2.5)
2 6 2
 p = qd
9  10 9
9  10
where, d is the distance between the centre of
= 2.0833  103 C
positive and negative charge of the molecule

io
q should be less than 2.0833  103 C. In the
p 6.4  1030
given set of options 2  103 C is the maximum d= = = 4  1011 m
q 1.6  1019
charge which is smaller than 2.0833  103 C.

at
1 ne 49. max = pE = 2  10–6  3  10–2  2  105
36. E = 12  103 N m
40 r 2
Er 2 50. max = pE = q(2l)E = 2  106  0.01  5  105
 n=
e
40
lic = 10  103 N m
 0.1 0.1
n = 0.036 
360
 105 51.
9  109 1.6 1019 144
ub
 n = 2.5  105
P
2p kp 0(xy)
37. Eaxial = Eequatorial  k 3  3
x y (0, 0) O
P

1/3
x 2
   3 2 :1
y 1

38. Electric field lines do not form closed loop. This Since the centre of the square lies at origin,
et

follows from the conservative nature of electric hence each quadrant will have the charge that
field. cancels the charge of diagonally opposite
quadrant. This results in the zero net charge on
Q square.
rg

41. Flux through one face of cube =


60 52. In case of spherical metal conductor the charges
Q 4 quickly spread uniformly over the entire surface
 
6ε 0 ε 0 because of which charges stay for longer time
Ta

 Q = 24 C on the spherical surface. While in case of non-


spherical surface, the charge concentration is
42. q = (2 – 1)0 = (4 – 5)105  8.85  10–12 different at different points due to which the
= 8.85  10–7 C charges do not stay on the surface for longer
time.
43. The total flux
Net charge enclosed by the closed surface 53. Surface charge density of small drop of radius r
= and carrying charge q is given by,
0
q
Here, q3 and q4 are the charges enclosed by  …. (i)
4r 2
surface S. So, flux is only due to charge q3
and q4. Since the volume remains the same, when 64
small drops combine to form a big drop, then we
44. Electric flux coming out through a closed have,
surface is q/0. Volume of big drop = 64  (volume of small drop)
135

MHT-CET Triumph Physics (MCQs)


62. Suppose electric field is zero at N.
That is, 4 R3 = 64  r 3 
4
3 3  Hence |EA| = |EB|
 R = 4r …. (ii)
Surface charge density of big drop, Q1 = 10 C E Q2 = 20 C
B N EA
64q 64q A B
'  [Using (ii)] x1 x2
4R 2 4(4r) 2
x = 80 cm
Or  '  4 q  4( ) [Using (i)] x
4 R 2 x1 = …(Using Shortcut 2)
Q2
' 1 1
  Q1
 4
80
54. After redistribution, the new charges on spheres =  33
20

ns
 10  10 1
are Q1     10  C 10
 10  20  3
 20  20 63. e–
and Q2     10  C
 10  20  3

io
+ –
Ratio of charge densities,
1 Q1 r2 2  Q  A B
  ….   
2 Q2 r12 4r 2  10 cm

at

2
10 / 3  20  2 Number of atoms in given mass
   
20 / 3  10  1 10
= × 6.02×1023
55.
A
lic 
p
63.5
= 9.48  1022
+q 
p net Transfer of electron between balls
l 9.48×1022
l = = 9.48  1016
ub
106
Hence magnitude of charge gained by each ball,
B C 60° q = 9.48  1016  1.6  10–19 = 0.015 C
+q l 2q 
p Force of attraction between the balls
P

(0.015) 2
pnet = p 2  p 2  2p 2 cos60 F = 9×109 × = 2×108 N.
(0.1) 2
pnet = 3 ql
 
64. They will not experience any force if | FG |=| Fe |
et

q 1 1 1 
56. E + + + ......
4πε 0 12 22 42   G
m2
=
1
.
q2
(16×102 )2 4πε 0 (16×102 ) 2
Using Binomial theorem,
rg

  q
 = 4πε 0G
q  1  m
E=  
4πε 0 1  1 
 4
Ta

Competitive Thinking
= 12  109 q N/C
57. If the charge is moved against the electric field, 1. Let us consider ball 1 has any type of charge. 1
work done by the external force is positive. and 2 must have different charges, 2 and 4 must
have different charges i.e., 1 and 4 must have
58. After contact, charge on each sphere = same charges but electrostatic attraction is also
(q1 + q2)/2. Force on given charges q1 and q2 is present in (1, 4) which is impossible. Hence,
maximum when they have same charge equal to ball 1 must be neutral.
(q1 + q2)/2.
2. F  Q1Q2 ( r is same in both cases)
59. Only in the case (D), the electrostatic force is F QQ 69
directed opposite to the weight.  = 1 2 = =3
F  
Q1Q 2 (6  3)  (9  3)
60. Negative charge means excess of electron F
 F =
which increases the mass of sphere B. 3
136
136

Chapter 10: Electrostatics


3. Fnet 8. Q q
F F
 FA
q0
FA

y
q Q
q q FA FR
O
a a
(FA)2 = (FR)2  2  FA  = FR2
2

2  Kq 2
Fnet = 2 F cos  =  cos 
2  a 2  y2   2 (FA) = FR
1 2Qq 1 Q2
Kq 2 y   = ….(opposite direction)
= 2 2  4πε 0 a2 4πε 0 2a 2
a  y  a  y2 

ns
2

 Q =  2 2q
For y << a
Kq 2 y 9. Here, q1 = q2 = q3 = 2 C,
F
AB = 3 cm, BC = 4 cm

io
a3
Fy 2 2 4K
F1 = K  =
(3) 2 9
4. The force between two charges is maximum if

at
their magnitude is equal. 2 2 4K
F2 = K  2
=
(4) 16
5. Electric force acting between the two charges is,
1 q q F A
F=  1 2 2  F = q1
4πε 0 K r K
lic 3 cm
16
 Net force on each charge = =2N 
8
q2
q3 F2
D
ub
6. Force acting between given charges + Q and C B 
Q 2
4 cm
 Q is, F = 
40r 2 R
When 25% of charges are transferred, charge on 
P

point A becomes, F1
3
q1 = + Q  0.25Q = + 0.75Q = + Q
4
F E
Charge on point B becomes,
et

3 DE F
q2 =  Q + 0.25Q = – 0.75Q =  Q In BDE, tan  = = 1
4 BD F2
 The new force between points A and B will be, 4K 16 16  16 
 tan  =  =   = tan–1
rg

 3  3  9  2  
9 4K 9 9
  Q    Q  Q
F =  4   4 =   16 
10.
40r 2 4  0 r 2 Y  axis
F sin 
Ta

9
= F +10 C
16
F
7. Force between charges will be, a
1 q1q2 Fnet 
F = O X  axis
20 C

40 r  t  t K 2
 a
x

 r r 
2 2 F
r      4  3r  9 2 F sin 
r +10 C
F   2   2    
  =  22  = 42
F r2 r r
F 9 Resultant force acting on 20 C charge is,
 
F 4 Fnet = 2Fcos
4 1 Qq
 F = F =2  2 cos
9 4 0 r

137

MHT-CET Triumph Physics (MCQs)

 20  106 10 106   x  17. From dimension we can check the answer, only
= 2  9 109    0 (1 + 2) having the same dimension qy to the
  a  x    a 2  x 2 
2 2
charge
3.6 x q net
Fnet =  =
a  x2 
3/ 2
2
0
Given x << a qnet =  0
3.6 x all other options don’t having the dimension
 Fnet  N equal to charge
a3
So answer is
11. q2 a q1 0 (1 + 2)

q Inc
2a 18. Electric flux () =

ns
a 0
10 C
q3 =
0
If more 10 C charge is placed.

io
1 q2 1 q2
F12 =  2 and F13 =  20 C
4πε0 a 4πε 0 (a 2) 2 Electric flux = = 2
0
F13 1

at
=
F12 2 20. On equatorial line, dipole moment and electric
field are opposite to each other.
1 q
13. E=
4 0 r2
lic 21. The relation between the electric fields at same
distances on axis as well as equator from a
9  109   4  1010  1.6  1019  dipole is, Eaxial = 2 Eequator
=
 20 10  2 2 
i.e., E axial = –2 Eequator

ub
= 1440 N/C
22. For small dipole,
kq At 2r:
14. E= 2
r 1 p
Eequator =
Er 2 2   0.3
P

2
2  9  102  109 40  2r 3
 q=  =
k 9  109 9
1 p
=  E2
 q = 2  10–11 C 40 8r 3
et

qenc At r:
15. Electric flux () = 1 2p
0 Eaxis =  E1
10 40 r3
rg

 =
0 E 2 1 / 40    p / 8r  1
3

  
If more 10 C charge is placed, E1 1 / 40    2p / r 3  16
20
Ta

 = = 2 23. A 
p
0 +2q
pnet
q x x
16. Total flux,  =
0
60°
B C
1 q x 
Flux passing through the cube,  = +2q – 4q p
8 0
 the electric flux passing through the 3 adjacent p net  p 2  p 2  2pp cos 60 
faces to the charge is zero,
= 3p  2 3 qx ….( p =2qx)
 flux passing through each of the remaining 3
faces of the cube is,
24. By using Q = ne
1 11 q  1 q Q = 1019  1.6  1019
0 =  =
3 3  8 0  24 0 Q = +1.6 C
138
138

Chapter 10: Electrostatics


25. Q = ne 2m h
 t2 =
Q eE
 n=
e 2h m
 t=
Given : Q = 2  C = 2  106 C eE
and total charge
e = number of atoms  charge on each atom 29. On equatorial line, dipole moment and electric
field are opposite to each other.
= 2  1022  29 e = 2  1022  29  1.6  1019
2 10 6 30. From figure,
 n=
2 10 22  29 1.6 10 19 F = Tsin l
T
T cos  
 n = 2.16  1011 mg = Tcos
F 
26. Initially, charges on the two spheres will be,  = tan q T sin 

ns
mg q
F
q1 =  (4R2) and x
q2 =  [4 (2R)2] =  (16R2) x 2  
mg  
 q1 + q2 = 20 R2 
Kq 2
 2 
….  tan  
x/2 

io
After redistribution of charges, final charges on x 2 mg x2  x2 
l2  l 2

the spheres will be, 4  4 
Q1 R x2
=  x << l, is negligible.

at
Q2 2R 4
 Q2 = 2Q1 Kq 2 x
As charges are conserved, total number of  
x 2mg 2l
charges on both the spheres will remain constant.
i.e., q1 + q2 = Q1 + Q2
lic  q2  x3
 Q1 + 2Q1 = 20 R2  q  x 3/ 2
20 5 dq d  x 3/ 2 
 Q1 = R2 = (4R2)  
ub
3 3 dt dt
27. Force acting on electron in electric field, dq d(x 3/ 2 ) dx

F=eE dt dx dt
 ma = e E ….(i) dq  dx 
  x1/ 2 v ….   v 
P

Using kinematical equation of motion, dt  dt 


1 2 dq
s = ut + at Given : = constant
2 dt
et

1  eE  2 1
 h=0+  t ….( s = h; u = 0)  v
2 m  x1/ 2

rg

Hints to Evaluation Test


Ta

1.  
Net electric field at O = | E B |  | E A |
Q EB O EA 8Q
4Q  8 
A x/2 3x/2
=   1
B 40 x 2  9 
Q
=
From figure, 0 x 2
Electric field at O due to charge Q, Here, negative sign indicates net electric field is

1 Q directed along E A , i.e., towards 8Q.
EA =
40 (x / 2) 2
2. If two charges are of the same sign, they repel
Electric field at O due to charge 8Q, each other. Thus if two protons are brought
1 8Q closer, some work is done against the force of
EB = repulsion. Therefore the electric potential energy
40 (3x / 2) 2
of the system increases.
139

MHT-CET Triumph Physics (MCQs)

3. The electric potential V (x, y, z) = 3y2  2z 8. Metal plate acts as an equipotential surface,
dV therefore the field lines should enter normally to
E = 
dr the surface of the metal plate.
  V ˆ V ˆ V 
E =   ˆi j k  9. Point T lies at equatorial positions of dipole 1
 x y z 
and 2 and axial position of dipole 3.

=  6y ˆj  2kˆ  +Q +Q
E2
=  6y ˆj  2kˆ
r E1 r
At point (1, 2, 4) 1 2
 T
E =  6(2)ˆj  2kˆ E3

= 12ˆj  2kˆ Q Q

ns
 Q
 |E|= 122  22  148  2 37 units
3
4. A 
+3/2 q p +Q

io
pnet
l l Hence potential at T
due to dipole 1, V1 = 0

at
 60°
B C p due to dipole 2 near Q,
l
+3/2 q –3q 1 p Kp
V2   2 = 2
40 r r
p net = p 2  p 2  2ppcos60
lic due to dipole 3, V3 = 0
= 3p Kp
 net potential at T =
3  3 r2
= 3  ql  ( p = ql )
ub
2  2 V 400
1 1 10. E=  2
= 104 V/m
5. For a charge V  and for a dipole V  2 d 4  10
r r
Y 10 cm
6. Charge density is inversely proportional to
P

radius of curvature of surface. Hence     


2 cm u
()at X > ()at Y. E
X
The surface of the conductor is an equipotential O
2 cm + + + + +
surface since there is free flow of electrons
et

within the conductor. Thus VX = VY = V.


According to problem, the particle will come
Potential at Y is the same as that at X.
The electric field E at a point on the out of the space between the plates, if
rg

equipotential surface of the conductor is y  2 cm at x = 10 cm.


inversely proportional to the square of radius of From the equation of trajectory,
curvature r at that point i.e., E  r2.  x  1  x
2

Since point Y has larger radius of curvature than y =  u sin      a  
Ta

 cos   2  u cos  
2 2

at point X, the electric field at Y is less than that


at X i.e., (E)at X > (E)at Y. F qE
Here,  = 0 and a  
m m
1 2
7. s = ut + at qE  x 
2

2  y=  
F qE 2m  u 
Here u = 0 and a = = (As sign indicates direction, neglecting it)
m m
qEx 2
 s=
1  qE  2   2cm  2  102 m
 t 2mu 2
2 m 
1.6  1019  104  101 
2
The time required to fall through distance s is, 2 qEx 2
 u = =
2dm 2m(2  102 ) 2  18  1024  2  102
t
qE = 22.22  106 m2/s2
Since t2  m, a proton will take longer time.  u = 22.22  103  4.7  103 m/s = 4.7 km/s.

140
140

Chapter 10: Electrostatics


11. Distance between two points
= 52  28 = 24 cm = 0.24 m
The potential gradient = 12 V/m
 Potential difference = potential gradient
 length
= 12  0.24 = 2.88 V
12. Work done by electrostatic field is product of
potential difference and charge. Potential
difference is independent of path. Hence
assertion is true.
When a charge moves along a closed loop, its
initial and final potential is same. Hence net

ns
work done is zero.

13. q +Q q
x 2x

io
Total potential energy of the system is
1  (q)( Q) ( Q)(q) (q)(q) 
U=  
40  x 2x 3x 

at
U = 0 (given)
1  qQ qQ q 2 
 0=    
40  x 2x 3x 
3 qQ q 2
lic
0= 
2 x 3x
Q 1 2 2
   
ub
q 3 3 9

14. Induced charge can be lesser or equal to


inducing charge but never greater than inducing
charge.
P

15. Copper is a metal with k = 


Hence force experienced will be zero.
max = pE = q(2l) E
et

17.
= 2  10–6  0.01  5  105
= 10  10–3 Nm
rg

19. F  Q1Q2 ....( r is same in both cases)


F QQ 7   5
 = 1 2 = =1
F Q1Q2  7  2   5  2
Ta

 F = F
2p kp
20. Eaxial = Eequatorial  k 
x3 y3
x 21/3 3
   2 :1
y 1

141
Textbook
Chapter No.

14 Semiconductors

Hints

28. When a free electron is produced, a hole is also


Classical Thinking
produced at the same instant.
52. At absolute zero, semiconductor behaves as an
31. In reverse bias, no current flows.

ns
insulator.
34. Diodes D1 and D3 are forward biased and D2 is
reverse biased. So the circuit can be redrawn as
Critical Thinking follows. R

io
4. With temperature rise, the conductivity of
semiconductors increases. R
6. Resistivity of a semiconductor decreases with

at
temperature. The atoms of a semiconductor vibrate
with larger amplitudes of higher temperatures I R
thereby increasing its conductivity. E
 I= E
7. The conduction and valence bands in the R
conductors merge into each other.
lic 35. This is because n-side is more positive as
10. The energy gap between valence band and compared to p-side.
conduction band in germanium is 0.76 eV and
the energy gap between valence band and 37. In forward bias, the diffusion current increases
ub
conduction band in silicon is 1.1 eV. Also, it is and drift current remains constant. Hence no
true that thermal energy produces fewer current flows due to diffusion.
minority carriers in silicon than in germanium. In reverse bias, diffusion becomes more difficult.
11. In semiconductor, the forbidden energy gap Hence net current (very small) is due to drift.
P

between valence band and conduction band is 42. When a light (wavelength sufficient to break the
very small (almost equal to kT). Further, the covalent bond) falls on the junction, new hole-
valence band is completely filled and the electron pairs are created. Number of electron-
et

conduction band is empty. hole pairs produced depends upon number of


photons. Hence photo e.m.f. or current is
12. Atomic number of silicon is 14.
proportional to intensity of light.
Hence, electronic configuration is 1s2, 2s2, 2p6,
3s2, 3p2 43. When a junction diode is forward biased as
rg

shown in figure, energy is released at the


13. At 0 K, semiconductor behaves as an insulator. junction due to recombination of electrons and
17. By using mass action law, holes. In the junction diode made of gallium
Ta

n i2 = nenh arsenide or indium phosphide, the energy is


released in visible region. Such a junction diode
n i2 (1016 ) 2 is called light emitting diode or LED. The
 nh = = = 1011 per m3
ne 1021 radiated energy emitted by LED is equal or less
than the band gap of semiconductor.
18. Gallium is trivalent impurity.
19. Antimony and phosphorous are both pentavalent. Light
p
20. Extrinsic semiconductors (n-type or p-type) are RL
neutral.
n
25. In p-type semiconductors, holes are the majority
charge carriers.
– +
27. Phosphorus is a pentavalent impurity. 45. Resistance of NTC thermistors decreases
Hence, ne >> nh. non-linearly with temperature.
142
142

Chapter 14: Semiconductors


46. Since diode D1 is reverse biased, therefore it
will act like an open circuit. 6. n  n h n e  ne =
2  3  10 
16 2


9  1032
i
4.5  1022 4.5  1022
Effective resistance of the circuit,
R = 5 + 3 = 8 . = 2  1010 m–3
Current in the circuit, I = E/R = 10/8 = 1.25 A. 7. nenh = ni2
47. Applying Kirchhoff’s second law, we have  4  1010  nh = 4  1016
I  R + 0.7 = 4  nh = 106 m–3
4  0.7 3.3 8. In n-type semiconductors, minority carriers are
 R   1650 
I 2  103 holes, majority carriers are electrons and
Power dissipated across R = I2R pentavalent atoms are dopants.
= (2  103)2  1650 = 6.6  103 W 12.
R

ns
48. 0.4 V
p-side n-side

For forward bias, p-side must be at higher


I potential than n-side.

io
13. For diode to be in forward bias, p-side of
diode needs to be connected at potential
3V
higher than potential to which n-side of diode

at
The value of R should be such that the current in is connected.
the circuit does not exceed 5 mA. By Ohm’s This condition is satisfied in option (A) only.
law, we have
I  R + 0.4 V = 3 V 14. As in both the figures the p-type material of
 5  103  R = 2.6
lic diode is connected to positive terminal of
battery and n-type to negative terminal, both are
 R = 520 
forward biased.
49. The energy of emission, 15. In case of p-n junction diode, width of the
ub
hc 6.62  1034  3  108 depletion region decreases as the forward bias
E = h = = voltage decreases.
 5780  1010
= 3.43  1019 J 16. When p-side of junction diode is connected to
3.43  1019 positive of battery and n-side to the negative,
= = 2.14 eV
P

1.6  1019 then junction diode is in forward biased mode.


For,  = 5780 Å, E = 2.14 eV Hole Junction Electron
The condition for emission of electrons is,
et

h > Eg.
But here, h < Eg [Eg = 2.8 eV]
 For emission of electrons,  < 5780 Å is a must.
rg

Competitive Thinking
+ 
4. Band gap of insulator is highest, while that of
Ta

conductor is least. So,


Eg  Eg  Eg Battery
1 3 2

i.e., Eg  Eg ,
In this mode, more number of electrons enter in
1 2 n-side from battery thereby increasing the
Eg  Eg number of donors on n-side.
3 2

 Eg  Eg  Eg V 0.6
1 2 3 17. Rd =  = 500 
I 1.2  103
5. The electronic configuration of C and Si are:
6
C = 1S2, 2S2 2P2 and 14Si = 1S2, 2S2 2P6, 3S2 3P2 18. Potential difference, V = 4 (6) = 10 V
Thus, the electrons in the outer most shell of V 10
 I= = 3 = 102 A
carbon atoms are more tightly bound to the R 10
nucleus unlike for silicon and are not available
19. VAB = 0.2  10–3 (5  10–3 + 5  10–3) + 0.2
for conduction. Hence it acts as an insulator.
= 2.2 V

143

MHT-CET Triumph Physics (Hints)


20. Since the diode in reverse bias offers infinite 24. p-n junction diode works only in forward bias
resistance, the equivalent circuit becomes. and not in reverse bias.
50  50  50  50  150  25. Diode will be in forward bias only in 0-5 volt
hence, it will conduct.

10 V 150   26. Heating will have effect on number of minority


10 V as well as majority charge carriers. This change
in charge carriers will affect overall V-I
10 10 characteristics of p-n junction.
I = = = 0.04 A
50  50  150 250 29. For a Solar cell, Open circuit  I = 0 and
21. For VA > VB : potential V = e.m.f.
Both diodes are forward biased so equivalent Also,  Short circuit  I = I and potential

ns
50  50 V=0
resistance R1 = = 25 
50  50 30. With decrease in temperature, resistance of metal
For VB > VA : decreases and semiconductor increases.
Both diodes are reverse biased so equivalent
31. Majority charge carriers in n-type semiconductor

io
resistance is infinity.
are electrons.
22. In given circuit, the diode D1 is connected in
reverse biased. Hence, no current flows through 32. The voltage-current curve for GaAs material is

at
resistance R2. As diode D2 is ideal, the as shown in figure below.
equivalent circuit can be given as, Thus, there exists a region where increase in
2 voltage leads to decrease in current which is a
non-ohmic behaviour and is attributed to
R1
lic negative resistance.

10 V hc
R3 2 33. Eg =

ub
hc
 =
Eg
V 10 6.63  1034  3  108
 I=   2.5 A =
R1  R 3 2  2 1.9  1.6  1019
P

23. Voltage drop across Si diode will be = 6.54  107 m


approximately 0.7 V.
V  Vdiode 3  0.7
 I =  = 0.0115 A = 11.5 mA
et

R 200

Hints to Evaluation Test


rg

3. At very low temperature, electrons cannot jump


13. When A is V(0) or B is V(0) or both are 0,
from the valence band to conduction band.
accordingly D1 or D2 or both are forward biased.
6. The intrinsic concentration of electron-hole Current flows via R, the potential at Y is 0. But
Ta

pairs is given by, ni2 = nenh when both A and B are at V(1), then D1 and D2
n 2 (1019 )2 do not conduct current. So potential at Y is
 ne = i = = 1017/m3
nh 1021 V(1). Y is 1 only when A and B are both 1.
Thus, this represents an AND gate.
7. When a p-n junction diode is formed, n-side  Option (B) is correct.
attains positive potential and p-side attains
negative potential. When ends of p and n of a hc
14. E
p-n junction are joined by a wire, there will be a 
steady conventional current from n-side to hc 6.63  1034  3  108
p-side through the wire and p-side to n-side   =
E 0.74  1.6  1019
through the junction. = 16.798  107
n i2  2  10  2  10  = 1679.8  109 m
16 16

12. ne    1.1  1010 m3.  1680 nm


np  3.5 10 
22

144
144

Chapter 01: Rotational Dynamics

01
a

Rotational Dynamics

Hints

14. In U.C.M., direction of velocity and acceleration


Classical Thinking change from point to point.

ns
4. For a seconds hand of a watch, T = 60 s 16. While moving along a circle, the body has a
2π 2π π constant tendency to regain its natural straight
= = = rad/s line path.
T 60 30
This tendency gives rise to a force called

io
5. For earth, T = 24 hr = 24  3600 = 86400 s centrifugal force. The centrifugal force does not
2 2 2 act on the body in motion, the only force acting
=  rad/hr = rad/s on the body in motion is centripetal force. The
T 24 86400

at
centrifugal force acts on the source of
6. For minute hand, TM = 60  60 s; for hour hand, centripetal force to displace it radially outward
TH = 12  3600 s lic from centre of the path.
M TH 12  3600 1 17. In circular motion,
 = = =12 : 1 ….[   ]
 H TM 60  60 T Centripetal force  Displacement
 work done is zero.
100
7. n = 100 r.p.m. = r.p.s.
60
ub
18. L = I. In U.C.M.,
2100
 = 2n = = 10.47 rad/s  = constant
60  L = constant
8. n = 3.5 r.p.s. 20. In uniform circular motion, acceleration is
P

 = 2n = 2    3.5 = 7 caused due to change in direction and is directed


= 7  3.14  22 rad/s radially towards centre.
9. Using,  = 2n 24. At each point on circular path, the magnitude of
et

 125 = 2n velocity remains the same for any value of .


125 25. The particle performing circular motion
 n =
2π flies-off tangentially.
rg

 n  20 Hz 2
34. p = mv; F  mv
d r
10.  = 0 ….(  = constant)
dt F mv 2 1 v

Ta

=  =
p r mv r
11. Using,
ω-ω0 2(n  n 0 ) mv 2
= = 35. F=
t t r
2  3.14  (350  0) 1
=  10 rad/s2 If m and v are constants, then F 
220 r
12. C = 2r F1 r 
 =  2
C F2  r1 
 r
2 2

C 36. Using, F = mv
 v = r(2n) =  2  f = fC ….[  = 2n] r
2 2 2
250
 r = mv = 10×(5) = =2m
13. Using, v = r = 0.2  10 m/s = 2 m/s F 125 125

145

MHT-CET Triumph Physics (Hints)


2
68. M.I. depends on the distribution of mass about
37. Breaking tension T = mv
r the axis of rotation. Also, M.I. is proportional to
Where, r = length of the string the mass.
Tr 77. Through bending, weight of opponent is made
 v2 =
m to act through the hip of the judo fighter to make
 v2 =
50×1 its torque zero.
1 2
1 2 1  2  2I2
 v = 5 2 m/s 78. K.Erot = I = I    2
2 2  T  T
3000  K.E.rot  T 2
38. f = 300 r.p.m. = r.p.s;
60
3000 1 2
 = t = 2   1 = 100  rad 79. E= I

ns
60 2
2E 29
47. Centripetal acceleration,  =  = 3 rad/s
I 2
acp = 2r = g l sin  = g tan
l cos 

io
1
80. E= I2
= 10  tan 60 = 17.3 m/s2 2
2E 2  360
48. Using,  I= 2 = = 9= 0.8 kg m2

at
 (30) 2
mr2 = T and  = 2n
n= 1 T 81. MK2 = I
 2Hz
2 mr  MK2 = MR2  K2 = R2
2
lic i.e. K is independent of M.
49. Using, Fs = mv
r 5MR 2 2 5R2 5R
86. Idisc =  MK2  K = K
4 4 2
= 10 210 = 104
5
Fs r
 v2 =
ub
m 10 90. Ix = Iy = I
 v = 100 m/s According to principle of perpendicular axes,
Ix + Iy = Iz
50. v = 36 km/h = 10 m/s
 Iz = 2I z
Using,
P

y
mv 2 500  100
 F= = = 1000 N
r 50
o x
et

55. For looping the loop, minimum velocity at the


highest point should be gl .

57. Thrust at the lowest point of concave bridge


rg

mv 2 MR 2
= mg + 91. IC =
r 2
2
MR 2 R
mv 2  I0 = + M 
Ta

58. N = mg cos   ,  = angle with vertical. 2 2


R
As vehicle descends, angle increases, its cosine MR 2 MR 2
= +
decreases, hence N decreases. 2 4
2
3MR
61. v1 = rg =
4
v2 = 5rg = 5  rg = 5  v1
MR 2
2 92. For a solid cylinder, M. I. about axis =
65. Moment of Inertia of a given body is I = MR 2
Thus, M.I. of a body depends on position of the  According to the theorem of parallel axes,
axis of rotation and hence is not constant. MR 2
M. I. about line of contact = + MR2
66. As axis of rotation changes, distribution of mass 2
about the axis of rotation is changed. = 3 MR2
I = MR2  ‘I’ will change. 2

146

Chapter 01: Rotational Dynamics


94. M.I. of a rod about an axis passing through its 120. P = .
ML2   = P = 50 W  0.42 Nm
edge and perpendicular to the rod =
3  120 rad / s
 
2
ML2 ML2 2 1 3
 Ix = + = = 2 kg m2 121. P =  = 60  2  25 = 3000  W
3 3 3

95. L = I Critical Thinking


[L] = [I] [] = [M1L2T0] [M0L0T1]
= [M1L2T1] 1. In non-uniform circular motion, particle
possesses both centripetal as well as tangential
100. Unit of angular momentum, L = kg m2 / s accelerations.
2 s
= kg m 300
2. Frequency of wheel, n = = 5 r.p.s.

ns
s s 60
2
= kg m s = J-s Angle described by wheel in one rotation
s2 = 2 rad.
2 Therefore, angle described by wheel in 1 sec
101. Angular momentum L = I = Ml .

io
3  = 2  5 radians = 10  rad

105.  = I  = kg m s = [M L T ]2 2 1 2 –2 3. n = 2000, distance = 9500 m

at
Distance covered in ‘n’ revolutions = n(2r)
108.  = I = MK2 = nD
109.  = I = 2.5  18 = 45 Nm  2000D = 9500
9500

110.  = I
lic D=
2000  
= 1.5 m

  =  = 500 = 5 4. Period of second hand = Ts = 60 s and


I 100
 Period of minute hand = Tm = 60  60 = 3600 s
 =   = .t = 5  2 = 10 rad/s
ub
2π 2 
t Angular speed of second hand s = =
Ts 60
111. I =   2000 = 100 kg m2 Angular speed of minute hand m = 2 =
2
 20 Tm 3600
P

113.  = dL = 4L  0 = L  ωs = 2π  3600 = 60 : 1
dt 4 ωm 60 2

 K2 
et

1 5. For minute hand, T = 60 min = 60  60 s


116. Etotal = mv2 1  2 
2  R  2π 2π
Angular speed,  = = rad/s
1
=  10  25  104  1  
2 T 60×60
2  5  180
rg

=  = 0.1
= 0.0175 J = 175  104 J 1800 
o
2 180
117. For solid sphere, K = 2 ….[ 1 rad =

]
Ta

R2 5
2gh 2gh 6. TE = 24 hr, TH = 12 hr
 v= = E 2 / TE TH 12 1
K2 2
1 2 1     
R 5 H 2 / TH TE 24 2

 10gh = 10  9.8  0.6 =


v= 8.4  2.9 m/s angle described 2
7 7 7. =  =  rad/s
time taken 2
118. For a ring,
8. n1 = 600 r.p.m., n2 = 1200 r.p.m.,
a= gsin   gsin  Using,
 K2  11
Increment in angular velocity,  = 2(n2  n1)
1  2 
 R   = 2(1200  600) rad/min
= (2  600)/60 rad/s
 a = gsin  = gsin 30 = g
2 2 4  = 20 rad/s
147

MHT-CET Triumph Physics (Hints)

9. n=
540
= 9 r.p.s.,  = 2n = 18 rad/s 18. Speed of C1 = R1 = 2 R1
60 T
Angular acceleration Speed of C2 = R2 = 2 R2
Gain in angular velocity 18  T
= = = 3 rad s–2
time 6 Speed of C1 = 2R1 / T = R1

dω Speed of C2 2R 2 / T R2
10. Using,  =
dt
15π-10π 5π 19. r = 0.25 m, n = 15 r.p.m. = 15 r.p.s.
 = = = 2.5  rad/s2 60
4-2 2 π
 = 2n = 2×π×15 = rad/s
210 60 2
11. n1 = 0, n2 = 210 r.p.m. = r.p.s. π π
60 v = r = 0.25  = m/s

ns
 210  2 8
d = 2(n2  n1) = 2   0  = 7  rad/s
 60 
20. T = 20 = 1 = 0.5 s
dω 2π×210 40 2
= = = 4.4 rad/s2
dt 60×5 2π = 2π = 4 rad/s

io
=
T 0.5
12. Using,
Let r = 50 cm = 0.5 m
 = 2t + 3t2
v = r = 0.5  4 = 2 m/s

at
  = dθ = 2 + 6t
dt 21. v = r
 = dω = 6 rad/s2  =
v
= constant [As v and r are constant]
dt r
13. v = r.
lic
22. T1 = T2  1 = 2
where r is distance from axis of rotation. v v
At the north-pole, r = 0  v = 0 =  = constant
r r
ub
14. A particle will describe a circular path if the angle v1 v2 v r R
 =  1= 1=
  r1 r2 v2 r2 r
between velocity, v and acceleration a is 90.
23. For seconds hand, T = 60 s,
n
r = 3 cm = 3  102 m
P

15. Frequency = r.p.s., t = 1 min = 60 s


60
2 2
Angular velocity,  = 2 n = = = 0.1047 rad/s
T 60
60
and v = r = 0.1047  3  102 = 0.00314 m/s
et

Linear velocity, v = r = 2n   = 2 n cm/s


2

60 60 600
24. n = 600 r.p.m. = r.p.s. = 10 r.p.s.
60
16. Using,
rg

2 23.14 v = r = r  2n
v = r = r  = 60  = 6.28 mm/s = 10  2  3.142  10
T 60
= 628.4 cm/s.
v = 6.28 2 mm/s  8.88 mm/s
Ta

25. Using,
17. Angular velocity of particle P
v = r = 0.5  70 = 35 m/s
about point A,
v v 26. T = 24 hr, r = 6400 km
A  
rAB 2r 2π 2π 2  3.14  6400
v = r = r= 6400 =
Angular velocity of particle P T 24 24
about point C, v  1675 km/hr
v v
C   ˆi ˆj kˆ
rBC r   
27. v =   r = 3 4 1 = 18iˆ  13jˆ  2kˆ
A v r
=  5 6 6
C 2r v
A 1 d 2
= 28. Angular acceleration = = 22
C 2 dt 2

148

Chapter 01: Rotational Dynamics


Total time 140s
29. No. of revolutions = = 1200
Time period 40s 36. n2 = 1200 r.p.m. = = 20 r.p.s.
60
= 3.5 Rev.
600
So, distance = 3.5  2R = 3.5  2  10 n1 = 600 r.p.m. = = 10 r.p.s., t = 5 s
60
 220 m
ω -ω 2  n 2  n1  2  20 10 
30. Refer Shortcut 1 = 2 1 = =
t t 5
In 15 seconds hand rotates through 90°
20π 2

 = = 4 rad/s
Change in velocity  v = 2v sin   5
2
1 1
 90   = 1t +  t2 = 20  5 +  4  25
= 2(r) sin    2 2
 2  v2 
v1 = 100  + 50  = 150 

ns
2 1 θ 150π
=21  90° Number of revolutions = = = 75
T 2 2π 2π
4  2 cm
= =  
60 2 30 s 37. = and  =

io
t t
31. Let velocity at A = v1 
Velocity at B = v2  = 2
t
 Velocity is constant,

at
But  = constant    t2
 v1 = v2 = v (say) 1 (2)2
AOB = 60 So, =
1 2 (2  3)2
 Change in velocity,
|v1  v2 |= v12  v 2 2  2v1v 2 cos 
lic or
1
1 2
=
4
25
= v 2  v 2  2v 2  cos  1 2 25
or =
 1 4
= 2v 2 1  cos   = v 2  2sin 2
ub
2 2 25
or 1 + =
 1 4
= 2 v sin = 2 v sin 30
2 2 21
 =
32.  = 2t3 + 0.5 1 4
P

d
 = (2t3 + 0.5) = 6t2
dt 38. By using equation 2 = 02  2
At t = 2 s,  = 6  22 = 24 rad/s  0 
2
et

  = 0  2(2n)
2

33. Using,  2 
  0 36  0 3 02
= = = 6 rad/s2  = ....(i)
t 6 4 4  36
rg

1 1 Now let fan complete total n revolutions from


 = 0t + t2 =  6  6  6 = 108 rad
2 2 the starting to come to rest
1 1 0 = 02  2(2n)
 3  (2)2 = 10 rad
Ta

34.  = 0t + t 2 = 2  2 +
2 2 02
 n =
5 4
35. v = 72 km/hr = 72× = 20 m/s, Substituting the value of  from equation (i),
18
0.5 ω02 4×4π×36
d = 0.5 m  r = m n = = 48 revolutions
2 4π 3ω02

 0 = =
v 20
= 80 rad/s Number of rotations = 48 – 36 = 12
r 0.5/2
2
40. Tangential force acting on the car increases with
2 =  0 + 2 the magnitude of its speed.
0 = (80)2 + 2(2  20)  at = time rate of change of its speed
 6400 = 80  = change in the speed of the car per unit time
80 which is 3 m/s
= =  25.48 rad/s2
  Tangential acceleration = 3 m /s2
149

MHT-CET Triumph Physics (Hints)


41. There is no relation between centripetal and 54. Velocity, v = r
tangential acceleration. Centripetal acceleration
 v = r = ωr = v = 10 cm/s
is a must for circular motion but tangential 2 2
acceleration may be zero.  a=r 2
r a
42. When a body is moving with constant speed, the  a = 2r = 2  2 = 2 = 10 cm/s2
tangential acceleration developed in a body is
zero.
55. Using,
43. Work done by centripetal force in uniform  = 2n = 2  1 = 2 rad/s
circular motion is always equal to zero. a = r2 = 0.4  (2)2 = 0.4  4 2
mv 2 a = 1.6 2 m/s2
46. F=
r 56. Using,

ns
 F  v2. If v becomes double, then F (tendency to a = 2r = 42n2r = 4(3.14)2  12  20  103
overturn) will become four times.  a  8  105 m/s2
47. Angular momentum is an axial vector. It is 57. Net acceleration in non-uniform circular motion
directed always in a fixed direction (perpendicular

io
to the plane of rotation either outward or inward), a = a 2t  a c2
if the sense of rotation remains same.  900 
2

= (2) 2   
48. The instantaneous velocity of a body in U.C.M.  500 

at
is always perpendicular to the radius or along  2.7 m/s2
the tangent to the circle at the point.
58. Using,
p
49. r = , n =   r.p.s.
t
lic 2
ar = v = 20×20 = 40 m/s2, at = 30 m/s2
r 10
v = r = r  2n 2
a= a +a 2
r
2
t = 40 2 +30 2 = 50 m/s
p
=   2 
ub
t 59. As  is constant, acceleration is due to the
22p change in direction of velocity = 2r
=
t As rA > rB  aA > aB
50. The radius vector points outwards while the 60. In half a circle, the direction of acceleration is
P

centripetal acceleration points inwards along the reversed.


radius.
It goes from v to  v
2 2

1200 r r
et

51. n = 1200 r.p.m. = r.p.s. = 20 r.p.s.


60 Hence, change in centripetal acceleration
a = 2r = (4 2 n2) r = 4  (3.142)2  (20)2  0.3
= v    v 
2 2

 4740 cm/s2 r  r 
rg

2
52. r = 10 cm = 0.1 m, a = 1000  10 m/s = 2v
2

a = 2r r
2 a
  = 61. If ar = 0, there is no radial acceleration and
Ta

r
circular motion is not possible
a 1000  10  316 rad/s So ar  0
  = =
r 10  10 2 If at  0 the motion is not uniform as angular
n = 316/2 = 50.3 r.p.s.  50 r.p.s. velocity will change
 n = 3000 r.p.m. So ar  0 and at = 0 for uniform circular motion
53. Radius of horizontal loop, r = 1 km = 1000 m mv2
3 62. Centripetal force = and is directed always
v = 900 km/h = 900×10 = 250 m/s r
3600 towards the centre of circle. Sense of rotation
v2 250×250 does not affect magnitude and direction of this
 a= = = 62.5 m/s2 centripetal force.
r 1000
a 62.5 63. The surface will rise from the sides, due to
 = = 6.25
g 10 centrifugal force.

150

Chapter 01: Rotational Dynamics


64. Distance covered, 73. T = ma = mr2
s =   2r T  2
360o ω'2 = T' = 4T = 4
660 = 90  2r ω2 T T
360  2 = 42
r = 420 m   = 2 
F = mv = 840  10  10 = 200 N
2
n = 2n = 2  5 = 10 r.p.m.
r 420
74. Breaking tension = 4  10 = 40 N
65. L = r p sin  = r p for U.C.M. [  = 90]  T = mr2
L2 r 2m2 v2 mv 2 T 40
 3
= 3
=  2 = = = 200
mr mr r mr 200×10-3×1

ns
66. Using, T = m2r    14 rad/s
 10 = 0.25  2  0.1 75. Using,
  = 20 rad/s 2πr
v=
2 T

io
67. F = m R
2πr 2π 20 1
 R  12 (m and F are constant)  T= = × = s
ω v 80 π 2
If  is doubled, then radius will become 1/4

at
 T = Time taken for one revolution
times i.e., R/4
There are 2 revolutions  total time taken = 1 s
68. Using, F = mr2 = m  42n2r 2π
= =4 ....( T = 1)
 m  42n2r = 6  1014
 n2 = 6  1014
lic T
dω 4
4  1.6  1027  3.142  0.12 = =  = 2
dt 2
 n  5  106 cycles/s 20
at =  . r i.e. = 2  = 40 m/s2
ub
mv 2 π
69. The centripetal force, F =
r 2
mv 2 76. n= r.p.s.
 r= 
F T sin = M2R .…(i)
P

2
 r  v or v  r T sin = M2L sin .…(ii)
v1 r 1 From (i) and (ii),
(If m and F are constant),  1 
v2 r2 2 T = M2L = M 42n2L
et

2
æ 2ö
2
= M 42 ççç ÷÷÷ L = 16 ML
70. Using, Fcp = m r = m  2  r
2
èπø
 T 
2
77. Linear velocity, v = r = 2nr
rg

= 500  10–3   2  22  1   0.49


 7 11  = 2  3.14  3  0.1
3 = 1.88 m/s
= 500  10  16  0.49 = 0.08 N Acceleration, a =  r = (6)2  0.1 = 35.5 m/s2
2
49
Ta

Tension in string, T = m 2r = 1  (6)2


71. m = 2 kg, r = 1 m, F = 32 N = 1  (6)2  0.1
Force, F = m2r = 35.5 N
32
 2 = = 16   = 4 rad/s 78. This horizontal inward component provides
2×1
 Frequency of revolution per minute required centripetal force to negotiate the curve
ω 4×7
safely.
n=  60 =  60  38 rev / min
2π 2×22 79. Because the reaction on inner wheel decreases
2 and becomes zero. So it leaves the ground first.
72. r = 20 cm = 20  10 m = 0.2 m
mv 2 82. m = 100 kg, v = 9 m/s, r = 30 m
Using, F = = 10
r Maximum force of friction = centripetal force
1 r 0.20 mv 2 100  (9) 2
 mv 2 = 10  = 10  =1J = = 270 N
2 2 2 r 30

151

MHT-CET Triumph Physics (Hints)

83. Since car turns through 90 after travelling  R  (10)2  1 = 12.5 m
471 m on the circular road, the distance 471 m is 0.8  10
quarter of the circumference of the circular path. If R  12.5 m
R is the radius of the circular path, then  R = 12 m
1
(2R) = 471
4 92. v  12m / s, v = 4 2 m / s
471  2 471  2
 R= = = 300 m v= rg
π 3.14
v = 12 m/s, m = 1000 kg  12 = μrg , 4 2 = rg
 Centripetal force,
12 =   3 = 
1000  12 
2 2

Fcp = mv  = 480 N 4 2  2 
R 300
2

ns
  = 
84.
2
tan  = v  tan   v2 9
rg
93. For the crate not to slide, the centripetal force
 tan 1 = v12 = v2 = 1 2
should be mv = mg

io
tan 2 v22 4v 2 4 r
 tan 2 = 4 tan 1  v2 = rg = 0.6  35  9.8 = 205.8
 v = 14.3 m/s

at
2
h
85. sin  = and tan  = v
l rg 94. Using,
2

 tan sin 1  h   = v
2
mg = mv
  l  rg
lic 2
r

 0.5 mg = mv
1  v2h  r
86. Reaction on inner wheel, R1 = M g   2
2  ra  v = 0.5  r  g = 0.5  10  9.8 = 49
ub
1  v h
2  v = 7 m/s
Reaction on outer wheel, R2 = M g  
2  ra  95. Using,
where, r = radius of circular path, h
tan    =
2a = distance between two wheels and l
P

h = height of centre of gravity of car. h = l  = 1.5  0.01 = 0.015 m


87. Using, 96. l = 1 m, g = 110 m/s2
mg = m2r
et

5
r = 400 m, v = 72 km/hr = 72  = 20 m/s,
μg 0.4×10 18
 = = = 4 = 2 rad/s
r 1 v2 = h
rg l
rg

88.  mr2  mg;   g


2 20  20  1
r  h= vl = = 0.1 m = 10 cm
rg 400  10
89. Using,
Ta

v2 = rg = 0.8  100  9.8 = 784 97. Using,


 v = 28 m/s vmax = rg = 0.2  100  9.8 = 14 m/s

90. v= gr 34.3


98. C = 34.3 m  r = ,
2 
When  becomes  , v becomes v i.e.
2 2  2
2 T= 22 s   = 
T 22
10 10 2
= = 5 2 ms–1  r2  1  34.3 2  2 1 
2 2   = tan1   = tan    
 g   2  22 9.8 
36103
91. v = 36 km/hr = = 10 m/s  22 1  1  4.9  2 
3600 = tan1  34.3  2    = tan  
 7  22 9.8   9.8 
The speed with which the car turns is
v2  Rg = tan1 (1) = 45

152

Chapter 01: Rotational Dynamics

v2 106. The maximum velocity for a banked road with


99. Using, tan  = friction,
rg
   tan  
 tan12 =
(150) 2 v2 = gr  
r  10  1   tan  
 r = 10.6  103 m = 10.6 km  v2 = 9.8  1000  
 0.5  1 
 …. [ tan 45 = 1]
 1  0.5  1 
100. Using Shortcut 11  v  172 m/s
h h
tan  = 2 2 1/ 2  107. Using,
(l  h ) l l 2
(l2 >> h2)  h tan  = v
rg
v2 r
tan  =  v= tan θ rg
rg

ns
h v2 = tan 30 o  17.32  10
=
l rg 1
=  17.32  10 = 10 m/s
v2l 3
 h=

io
rg
108. Using,
2
101. The inclination of person from vertical is given 20×20 20
tan  = v = = = 2.04
by, rg 20×9.8 9.8

at
v2 (10) 2 1 –1
 = tan (2.04) = 63.90
tan  =  
rg 50  10 5
5 50
  = tan1(1/5) 109. v = 60 km/h = 60  = m/s,
102.  = sin1(0.2), N = 2000 N,
lic r = 0.1 km = 0.1  1000= 100 m
18 3

2
1 v2  50  1
sin  = 0.2 = tan  = =  
5 rg  3  0.1  103  9.8
ub
5   = tan–1  (50 / 3) 
2
1  
100  9.8 

24 5
110. v = 180 km/hr =  180 = 50 m/s
P

mg = N cos  18
24 Using,
 Weight = N cos  =  2000 = 1959.6 N 2
50×50 5
5 tan  = v = = = 1
rg 500×10 10
et

 2
24 
2

….  cos  1    
1

 = tan1   = tan1 (0.5)
 5  1
 5  
 2
103. Using,
rg

111. m = 80 kg, v = 20 m/s,  = tan1(0.5)


v = rg tan  = 10 10  tan  In order for the cyclist to turn,
10 = 10 tan  frictional force = centripetal force
tan  = 1 mg = m  v  = mg v
Ta

2 2

  = 45  r  rg
2
104. Using, h = l sin  But v = tan 
h 1.2 rg
 sin   tan  =  = 0.15
l 8  mg = mg tan  = 80  10  0.5 = 400 N
 tan  = 0.15
112. Let initial velocity = v1
Now, v = rg tan   40  9.8  0.15  8 m/s
New velocity v2 = v 1 
20  6v
 =
105. r = 50 m, l = 10 m, h = 1.5 m  100  5
v2 h 2 2
= r1 = 30 m, tan 1 = v1 , tan 2 = v2
rg l r1g r2g
rgh 50  9.8  1.5 As there is no change in angle of banking,
 v= = = 8.6 m/s
l 10 1 = 2

153

MHT-CET Triumph Physics (Hints)

 tan 1 = tan 2 Centripetal force = m2r


2 2 2
 v = v
1 2 = 5  102  4π  5
r1g r2g 9
2 = 25  102  4
 
= 100  102
2 2

 r1 =  v1  =  v1  =  5  = 25
  6    1N
r2  v2   v1  6 36
5  mg
117. T =
36 36 216 cos 
 r2 = r1 =  30 = = 43.2 m
25 25 5 
h L2  r 2
cos = = L
113. Using, L L h
2 v2 mg L
Fs = mv But, tan  =  T=

ns
r rg L2  r 2 r

v2
= g tan  118. The particle is moving in circular path.
r
From the figure, mg = R sin  …(i)

io
Fs = mg tan  = 90  10  tan 30  520 N mv 2
= R cos  …(ii)
r Rsin
114. For banking of road,  = tan–1  v 
2

From equation (i) and (ii) R


 rg  

at
–1
we get Rcos
 = tan (0.24) rg r
 tan  = 0.24 tan  = but tan  =
v2 h mg
v2
Also, tan  =
rg
=    = 0.24
lic  h

115. Using,
T sin  = m2r = m2 l sin  ….(i) v2 3gr
119. v = 3gr and a = = = 3g
ub
T cos  = mg ….(ii) r r

121. At the highest point,


mv2
60 mg 
T r
P

 v= rg = 4000  10 = 200 m/s


T cos
ar 122. Minimum velocity at the bottom,
Tsin
et

mg v = 5gr = 5×9.8×6.4
g
From (i) and (ii), 2 = = 313.6 = 17.7 m/s
l cos 
rg

g 123. Using,
 =
l cos  mv2
F= = m2r = mg
2π l cos  r
 Time period, T = = 2
Ta

  =
ω g g 9.8
 =
r T 4
= 2  3.14  1 cos60 2 2
10  T= 4s
9.8
= 1.4 s
 h=
v2
=
(0.5) 2
= 0.025 m 124. TL  TH = 6 mg
g 10
125. Using,
= 2.5 cm
mv 2 2  (4) 2
= = 32 N
116. Using, r 1
r = l sin It is clear that tension will be 52 N at the bottom
r = 10 sin 30  r = 5 m, T = 3 s of the circle because we know that,
2π 2π mv 2
= = TBottom = mg +
T 3 r

154

Chapter 01: Rotational Dynamics


126. TL = 350 N 132. According to law of conservation of energy,
Using, 1 1
mgh = mv2 = m  5  Rg
mv 2 = T – mg = (2  350 – 40  10) = 300 2 2
L
r 2 2
 R = h =  5 = 2 cm
 v2 = 300  3 = 22.5 m/s 5 5
40
133. When body is released from the position
v  4.7 m/s
(inclined at angle  from vertical), then velocity
127. At the highest point of the circle, at mean position,
mv 2  4104  v = 2gl (1- cosθ)
F=  mg = 70    10 = 6300 N
r  400  mv 2
 Tension at the lowest point = mg +
128. Using, l

ns
m
1 1
(K.E.)L(K.E.)H = m  v2L  v2H  = m[5 rg  rg] = mg + [2gl(1  cos60)]
l
2 2
= 2 mrg = 2  1  1  10 = 20 J = mg+ mg = 2mg
mv 2

io
129. Even though particle is moving in a vertical 134. Tension, T = + mg cos 
loop, its speed remain constant. r
mv 2 mv 2
Tension at lowest point, Tmax = + mg For,  = 30, T1 = + mg cos 30
r

at
r
mv 2 mv 2
Tension at highest point, Tmin =  mg  = 60, T2 = + mg cos 60
r r
mv 2  T 1 > T2
Tmax
= r2
+ mg
=
5
lic 135. T = mg + m2r = m {g  42 n 2 r} ….[ = 2n]
Tmin mv 3
- mg
r   2
   2n 2r  
= m g   42 
n 
 r    m g   
By solving we get, v = 4gr = 4´9.8´ 2.5   60  
     900  
ub

= 98 m/s
136. Minimum angular velocity,
130. Using, g
2 min =
mg – N1 = mv1 R
P

r
2 R 2
 mv12 = 667 – 556 = 111  Tmax = = 2 = 2 =2 2  3 s
min g 10
r
et

Let v2 = 2v1 137. Using, mr2 = mg


 mv 2 2 = 4mv12 = 4  111 = 444 2
4 2 r
 2  2
r r  r  = g  T =
 T  g
2
rg

mg – N2 = mv 2
r 4
r  T = 2 = 2  3.14  4s
g 9.8
 N2 = 667 – 444 = 223 N
Ta

131. By conservation of energy, 138. Critical velocity at highest point = gR


1
mv2 = mgh = 10´1.6
2 = 4 m/s
v = 2gh ….(i)
139. Tmax = 30 N
For looping the loop, the lower velocity must be
Using,
greater than 5gr
Tmax = mω 2max r + mg
5gD
vmin = 5gr = ….(ii) Tmax
2  = 2r + g
m
From (i) and (ii),
30
5gD  10 = 2max r
2gh = 0.5
2
5D 50 50
h= max = = = 5 rad/s
4 r 2

155

MHT-CET Triumph Physics (Hints)


140. Max. tension that string can bear = 3.7 kg-wt For rod C, all points are always at distance L
= 37 N from the axis of rotation, so
Tension at lowest point of vertical loop IC = mL2 = ML2
= mg + m 2r = 0.5  10 + 0.5  2  4 2 2
 I = 0 + ML + ML2 = 4ML
= 5 + 22 3 3
2
 37 = 5 + 2
  = 4 rad/s 146. Hard boiled egg acts just like a rigid body while
rotating. It is not in the case of a raw egg
141. Using, because of liquid matter present in it. In case of
mv 2L a raw egg, the liquid matter tries to go away
TL =  mg = 6 mg = 6  5  10 = 130 N
r from the centre, thereby increasing its moment
 The mass is at the bottom position. ( I) raw egg
of inertia i.e., >1
142. Using, ( I) boiled egg

ns
mv12 As moment of inertia is more, raw egg will take
Maximum tension, Tmax = +mg
r more time to stop as compared to boiled egg
mv22 (Law of Inertia).
Minimum tension, Tmin =  mg

io
r I 0.25
Using the law of conservation of energy, 147. R2 = =
M 1
1 2 1  R = 0.5 m  d = 1 m
mv1 = mv 22 + 2mgr

at
2 2
v12  v22  4rg
150. As the mass of disc is negligible, only the

moment of inertia of five particles will be
v12 considered.
 g v 2  rg
Hence Tmax  r
Tmin v 2 2
 12
v 2  rg
lic I =  mr2 = 5 mr2 = 5  2  (0.1)2 = 0.1 kg-m2
g 151. Let the mass of loop P having radius r be m
r
v 22  5rg 4
So the mass of Q having radius = nr will be nm
 ….[ v12 = v22 + 4rg]
ub
v 2  rg 1
2

This gives, 4v22  4rg  v22  5rg nr


r
 3v = 9 rg = 9  10  10
2
2
3 P Q
P

 v22 = 9  10  10
3 3
 Moment of inertia of loop P, Ip = mr2
v22  100
et

Moment of inertia of loop Q, IQ = nm(nr)2 = n3mr2


 v2 = 10 m/s IQ
2
  n3  8  n  2
143. I  R IP
rg

dI 2RdR 2dR
 = = 152. Moment of inertia of system about YY,
I R2 R
= 2  1% = 2% I = I1 + I2 + I3 Y
Ta

2 1 3 3
144. Earth is solid sphere, so M.I. = MR2 = MR 2  MR 2  MR 2
5 2 2 2 1
4
where M = R3 7
3 = MR 2 2 3
2
2 4 8
 M.I. = ( R3)R2 = R5
5 3 15
145. Moment of inertia of the system about the given
axis I = IA + IB + IC
MR 2
As rod is thin, 153. M.I. of ring about diameter I = ….(i)
2
IA = m  02 = 0
Rod B is rotating about one end  L = R  R = L / 
2
 IB = ML  From equation (i), I =
ML2
3 2 2

156

Chapter 01: Rotational Dynamics


2 2
154. MR s2 = MR 2h  I = 0 + m(r2)2 + m(r3)2 + m(r4)2
5 3
2 2
 l 
   l 
2
Rs 5 I=m 
 =  m l 2  m 
Rh 3  2  2
 I = 3ml2
2
ML
155. M.I. of thin rod, I1 = ....(i) 164. From triangle BCD,
12
2
M.I. of ring, I2 = MR ....(ii) C 2
The rod is bend to form a ring  L = 2R
 Dividing equation (i) by (ii),
2
I1 = ML  1 a
x
I2 12 MR 2

ns
= M(2R) 
2
1
1 3
12 MR 2
A D a/2 B
4M 2R 2 2
= =

io
2 2
12MR 3 a
CD2 = BC2  BD2 = a2   
2
156. Let mass of the ring = mass of the disc = M
MR12 3a 2
 x2 =

at
M.I. of the ring about the diameter = ….(i)
2 4
2 Moment of inertia of system along the side AB,
M.I. of disc about the diameter = MR 2 Isystem = I1 + I2 + I3
4
Since M.I.s are equal,
lic = m  (0)2 + m  (x)2 + m  (0)2
MR 12 MR 22 3ma 2
  = mx2 = ….[From (i)]
2 4 4
R1 2  R1  1
2
ub
  1 2
R2 4 R2 2 165. E = I = 1500
2 2
1
157. I = 2 MR2 = 2  4 R 3  R 2 I(t)2 = 1500
2
5 5 3

P

 (1.2) (25)2 t2 = 3000


= 8  22  R  = 176 R5
5
 t2 = 4  t = 2 s
15 7 105
1 2
et

159. M.I. of the circular disc will be 166. E = I


2 2
2I = (2M)R
2 1 2
E1 I11
1  = 2
rg

 M.I. of the semicircular disc, I = MR2 E2 1


2 I 222
2
1 I1 = I2 ….[Given]
163. r2 = r4 = OA = and r3 = l 2
Ta

2 E1 
2
 1
2

Moment of inertia of the system about given  =  1  =  1  =


E2 
 2 2 
 1 4
axis, I = I1 + I2 + I3 + I4
 E2 = 4E1
167. For a uniform thin rod suspended from one end,
r2 ml 2
1 2 I= ,  = 2f
A 3
B
1 2 1 ml 2
 E= I    (2f ) 2
O 2 2 3
1 ml 2
r4 r3 =   4 2f 2
2 3
D C 2
4 l 3 = 2f 2 ml 2
3
157

MHT-CET Triumph Physics (Hints)


240 173. As kinetic energy is same,
168. n = 240 r.p.m. = = 4 r.p.s.
60 1 I  2= 1 I 2
R R d d
2 2 2
 I = MR = (10)  (0.1) = 0.1 kg m 2 2

 E=
1 2 1
I = I (2n)2 = 22 In2 I R R = d
2 2
 ….(i)
2 2 Id d R
= 2 (0.1)  16 = 3.2  J
As same torque is applied,
169. K.E.rot
1
= I2 = 
1 ML2
 2
IRR = Idd
2 2 12 I R R = Idd
1 L2 tR td
=  A  L  D   2
2 12 I R R = t R
 ….(ii)
1 Id d td
 K.E.rot = DAL32
24

ns
From equations (i) and (ii),
170. Additional rotational K.E. = 800 J d = t R
1 2 1 R td
 I  I 02 = 800
2 2  dtd = RtR  d = R = n

io
1
As 0 = 0  I2 = 800 174. Initial moment of Inertia I1 = 1 kg-m2
2
Moment of Inertia of lump of wax = MR2
1600 1600 = 50  103  (20  102)2
 =   21 rads1

at
I 3.6 = 2  103 kg m2
From  = 0 +  t Final moment of inertia,
 21 = 0 + 15 t, t =
21
= 1.4 s I2 = 1 + 2  103 = 1.002 kg m2
15
lic  % Increase in M.I. =  1.002  1   100 %
1
20 1  
171. n1 = 20 r.p.m. =  r.p.s.,
60 3 = 0.002  100 % = 0.2 %
60
 1r.p.s., 175. M.I. of disc of central zone,
ub
n2 = 60 r.p.m. =
60 4  (0.2) 2
Work done by torque is the change in its I1 = = 0.08 kgm2
2
rotational K.E.
M.I. of wooden annular disc,
W = (K.E.)f  (K.E.)i
I2 = 3 [(0.2)2 + (0.5)2] = 3 [0.04 + 0.25]
P

=
1
2
1
2
1
I f2  I i2 = I f2  i2
2
  2 2
= 1.5  0.29 = 0.435 kg m2
1
= MK2  2n f    2n i  
2 2
 M.I. of whole disc = I1 + I2 = 0.08 + 0.435
et

2
= 0.515 kgm2
1 9  1 
2

=  1  2  42 (1) 2     178. M.I. of disc about tangent in plane


2    3   5
= mR2 = I
rg

1 9 2  4
=  1  2  4 
2  9 4
 mR2 = I
= 16 J 5
Ta

300 3
172. n1 = 300 r.p.m. = = 5 r.p.s., M.I. of disc about tangent  to plane I = mR2
60 2
600 Substituting the value of MR2 from equation (i),
n2 = 600 r.p.m. = = 10 r.p.s. we get
60
 Work done = Change in K.E.rot 3 4  6
I =  I = I
1 2 5  5
= I   22  21 
2 179. 2 = 1.1 1, E  2
1 MR 2  E1  K12 , E 2  K22
= × × 4π 2  n 22  n12 
2 2
 E2  E1 = K  22  12   K12 (1.12  12)
= MR2 2  n 22  n12 
= K12 (0.21)
= 2  (1)2  (3.14)2  (102  52)
= 2  (3.14)2  75 E 2  E1 K12  0.21
  100   100  21%
 1479 J E1 K12

158

Chapter 01: Rotational Dynamics


180. I11 = I22  MK12 1 = MK22 2 188. According to the theorem of parallel axes, M.I.
K1 2 of disc about an axis passing through P and
  perpendicular to the disc,
K2 1

181. Moment of inertia of solid sphere about its R P


2 R
diameter, I = MR2 O
5
I 2 MR 2
 K=  = 0.4 R I = 1 MR2 + MR2 = 3 MR2
M 5 M 2 2
Total M.I. of the system,
182. M.I. of thin rod about axis passing through
3
centre perpendicular to length is = MR2 + m (2R)2 + m( 2 R)2 + m( 2 R)2
2

ns
ML2
Using, I = MK2 = = (3 M + 16 m) R
2
12
2
L L 1
 K= = = m
12 2 3 2 3 189. By the principle of parallel axes, IP = IG + Mh2

io
IP = MK 2P , IG = MK G2
183. I = m ri
i i
2
= 4 Mb2 ….(i)
 MK 2P = MK G2 + Mh2

at
If K = radius of gyration of the system then,  K 2P = K G2 + h2
 
I =   mi  K2 = 4 MK2 ….(ii)  100 = KG2 + 36
 
K G2 = 64  K G = 8 cm
i

 Comparing equations (i) and (ii),
K=b
lic 190. I0 = 1 ML2
12
R
184. Radius of gyration of circular disc k disc = By applying theorem of parallel axes,
2
ub
2
L
Radius of gyration of circular ring k ring = R I = I0 + M  
2
 
k disc 1
 Ratio =  .
k ring 2 = 1 ML2 + 1 ML2
12 4
P

R 1 
185. For disc, K = = 4   ML2 
2  12 
…. [ axis passes through centre of disc and  I = 4 I0
et

perpendicular to its plane] 191. I = 2 MR2


5 5
=  3.54 cm
2  According to the theorem of parallel axes,
rg

186. I = MK2 = 2  (50  102)2 I = 2 MR2 + MR2 = 7 MR2


5 5
= 2  2500  104
7  2 
=  MR 2  = 3.5 I
= 50  102 kg m2
Ta

2 5 
= 0.5 kgm2
2
187. According to theorem of parallel axes, moment 192. M.I. at end of rod = ML = 0.33ML2
of inertia of a rod about one of its ends, 3
2
2 2
M.I. at its centre = ML = 0.083ML2
I = ML + M L
12 4 12
2 M.I. at a point midway between end and centre
= ML = Ix = Iy 2
3 = 7ML = 0.145ML2
48
 Moment of inertia of two rods about Z-axis
= Iz = Ix + Iy 1
M.I. at a point length from centre
= Moment of inertia of 2 rods placed along 8
X and Y-axis = 2ML
2
67ML2
= = 0.087ML2
3 768

159

MHT-CET Triumph Physics (Hints)


2
193. IA = MR = 0.5 MR2 198. M.I. of ring (A)  to plane = MR2
2 MR 2
M.I. of ring (B) passing through plane =
2IB = IA 2
MR 2 3
 IB = I A = 0.25 MR2  M.I. of system = + MR2 = MR2
2 2 2
2
 IC = IA + MR2 = MR + MR2 199. M.I. of sphere about the diameter = 2 MR2
2 5
= 3 MR2 = 1.5 MR2 2
MR2 = 20 or MR2 = 50
2 5
 ID = IB + MR2 = 0.25 MR2 + MR2 = 1.25 MR2 According to theorem of parallel axes,
 IB < IA < ID < IC M.I. about the tangent
= 2 MR2 + MR2 = 7 MR2 = 7  50 = 70 kg m2

ns
2
ML 5 5 5
194. IA = , IB = 0
12
2 200. I1 = 1 MR2 + 1 ML2
ML2 L L ML2 ML2 2 12
 IC = +M    = +

io
12 2 4 12 16
2
 I1 = MR + 1 M(4R2)
1 2

ML2 L L 2 12
 ID = +M   
12 2 3 = MR + 1 MR2 = 5 MR2
1 2

at
2 2 3 6
ML2 L
= +M    1 2 1
I2 = MR + M (4R ) 2
12 6
2 3
=
ML2
12
+
ML2
36
lic = MR + 4 MR2 = 11 MR2
1
2
2

3 6
 I1 = 5 and I > I
195. Consider two perpendicular diameters, one 2 1
along the X-axis and the other along the I 2 11
ub
Y-axis. Then, Ix = Iy =
1
MR2  I2 – I1 = 11 MR2 – 5 MR2 = MR2
4 6 6
According to the perpendicular axes theorem, 201. M.I. of the solid sphere about a diameter
the moment of inertia of the disc about an axis 2
P

passing through the centre is, I= MR2


5
1 1 1 M.I. of the disc about an axis through its edge
Ic = Ix + Iy = MR2 + MR2 = MR2
4 4 2 and perpendicular to its plane is
et

Mr 2
196. M = V = R2t I= + Mr2
2
 MX = RX2tX and MY = RY2tY
2 Mr 2 3
2
 MR2 = + Mr2 = Mr 2
Let I = MR
rg

5 2 2
2
2
R y 4 t y  r= R
IX = R x t x and IY =
4
 15
2 2
Ta

Ry ty 4 4 ML2
IY (4R) (t / 4) (4) 4 202. I =
 = = = = 64 12
IX R x 4t x R 4t 4
Applying the theorem of parallel axes,
 IY = 64 IX L
2
ML2 ML2 7ML2
 I1 = I + M    =  =
197. The moment of inertia of ring about a tangent in 4 l2 l6 48
2 2
its plane = MR + MR2 = 3MR MR 2
2 2 203. Ic =  M.I. of disc about any diameter,
2
The moment of inertia of disc about its diameter 1 MR 2 MR 2
2 Id = 
= MR 2 2 4
4
 Applying theorem of parallel axes,
Ratio = 3MR / 2 = 6
2
 MR 2 5
MR / 4 2 1 It = Id + MR2 =  MR 2  MR 2
4 4

160

Chapter 01: Rotational Dynamics


204. Ic = 4 kg m2 = MR2 1
209. E = L
Using theorem of perpendicular axes, 2
M.I. of ring about any diameter, 1
Ic 4  225   L  25
Id =   2kg m 2 2
2 2  L = 9  2 = 18 J s
Applying theorem of parallel axes,
M.I. about tangent in its plane. 210. R = 6400  103m = 6.4  106 m, T = 24  3600 s
It = Id + MR2 = 2 + 4 = 6 kg m2 2 2
L = I = MR 2 
205. M.I. of the plate about an axis perpendicular to 5 T
 2 
its plane and passing through its centre = 2  6  1024   6.4  106 2   
2 5  24  3600 
ma
I0 = L = 7.145  1033 kg m2 s1
6

ns
211. I = MR2 = 1  (0.5)2 = 0.25 kg m2
O a
 = 2n = 2    100 = 200 rad/s
a/ 2 
L = I = 0.25  200 = 50 kg m2/s

io
A
212. L = I = I  2 (n2  n1)
Applying parallel axes theorem, = 0.06  2  (5  0) = 0.6 

at
2
 a  ma 2 ma 2 2
IA = I0 + m   =  = ma 2 1 L2
 2 6 2 3 213. K.E. =  L2 = 2  K.E.  I
2 I
206. Moment of inertia of a uniform circular disc  L = 2  4  2 = 4 kg m2/s
about an axis through its centre and
lic 2
1
perpendicular to its plane is IC = MR 2 214. E = 1 I2 = L  E  L2
2 2 2I
2 2
 Applying theorem of parallel axes,  E f =  Lf  =  150  = 9
ub
moment of inertia of a uniform circular disc    
Ei  Li   100  4
about an axis touching the disc at its diameter
 E f  E i  100 =  E f  1   100
and normal to the disc,
Ei  Ei 
1 3
I = IC + Mh2 = 2
MR 2 + MR = MR 2
P

2 2 =  9  1 100 = 500 = 125%


 4  4
207. Moment of inertia of rod AB about point P and
215. L = I  L = I
2
Ml
perpendicular to the plane =
et

12 L I M  R / 2 
2
1
 = = =  L = L
A l B L I MR 2 4 4

216. E = 1 I2
rg

P
2
L = I  L2 = I22
O 2
 E = 1 L
Ta

2 I
But I = MR2
By applying parallel axes theorem, 2
L2
 E= 1 L =
M.I. of rod AB about point ‘O’ 2 MR 2
2MR 2
2
Ml 2  l  Ml
2
=  M   L2 1
12 2 3 217. E =  E  when L is constant
But the system consists of four rods of similar 2I I
type. Hence by the symmetry,  As I1 > I2  E1 < E2
 Ml 
2
Isystem = 4   218.  = I = I d
 3  dt
where  = constant
1 2 (I) 2 L2 d = 0   = 0
208. E = I = = 
2 2I 2I dt
161

MHT-CET Triumph Physics (Hints)


219. n1 = 300 r.p.m. 1  1
  M  M b   = M
300 2  2
= = 5 r.p.s;
60  (100 + 50) = 100
 = 2(5) = 10  rad/s
  = 3 = 15 r.p.m.
 = I  =  2 MR 2      0  2
5   t 
229. Refer Shortcut 8
= 2  2000  25   2  10 
5  2  230. Li = 1 ma2
4 5 2
=  2  10  4   =  2.5  10 dyne cm
Negative sign shows that it is a retarding torque  Lf = 1 ma2 + ma2 = 3 ma2 
2 2
220.  = I
As Li = Lf ,

ns
  = çæçI + 50I ÷ö÷  = 1.5I = 1.5 1 ma2 = 3 ma2
çè ÷ 100 ø
2 2

221.  = f  i   = 3 or  = 
3

io
t
i = 2n = 2  20 = 40  rad/s .2 1
233. L = I = L
  = 0  40 = 4 rad/s2 (retardation) T T

at
10 1
  = I = 5  103  (4) = 2  102 Nm Hence, by doubling T, L becomes times.
2
Negative sign shows that it is a retarding torque.
 || = 2  102 Nm 234.  =
dL 4J  1J 3J

222. 0 = 2  240 = 8 = 25.12 rad/s,


lic dt
=
4
=
4

60 1 2
235. E1 = I
Using,  = I, 2
ub
 =  = – 0.81 = –5.06 In second case, I = 3I
I 0.16  According to conservation of angular momentum,
  = 0 + t = 25.12 – (5.06  2) = 15 rad/s I = I
223. n = 1800 rev/min = 30 rev/s I I 
 = = =
P

 = 2n = 60 rad/s I 3I 3


P 1
 = = 100000  531 Nm Now, E2 = I2
ω 2
60π
et

1 2 1 1  1
 2000 =  3I  =  I2  = E
224.  = I   I =  = 1000 kg-m2 2 9 3 2  3
 2
1
E E
225. Torque producing acceleration 1, E1  E 2 3 = 2
rg

 =
 = I11 = 2mD21 E1 E 3
Same torque produces 2
236. L1 = I11, L2 = I2 2
  = I22 = 2m(2D)22
Let I1 = MR2
Ta

 4(2mD2)2 = 2mD21
1 = 500 r.p.m.
 2 = 1 1  I2 = MR2 + MR2 = 2MR2
4
From conservation of angular momentum,
227. According to conservation of angular L1 = L2  I11 = I22
momentum, L = L  MR2 (500) = 2MR2 (2)
 I  = I  2 = 500 = 250 r.p.m.
 I  = I   = n 2
n
237. By principle of conservation of angular
1 2 1 2 momentum, I = I11 …..(i)
228.  MR  M b R   = MR 
2

2  2 Assuming earth to be a uniform solid sphere,


(Since the boy reaches the centre, the final I = 2 MR2
angular momentum of boy is zero). 5

162

Chapter 01: Rotational Dynamics

Then equation (i) becomes, 2 MR2 =


1 1
Mv2 + Mv2
5 2 4
2
3 3
  = 2 M  R  1   = 1 = Mv2 =  8 = 6 J ….[From (i)]
5 2 1 4 4 4

T1 = 1  2  244. In this case, 1 mv2  1  K  = mgh


2
 ….      2 
T 4  T 2  R 
 T1 = T = 24 = 6 hours  1 mv2  K 2  = mg 3v
2

4 4  1  
2  R2  4g
238. The angular frequency of the composite system 2 2

can be obtained by using the principle of  1 + K = 3  K2 = R


R2 2 2
conservation of angular momentum.

ns
MR 2
Total initial angular momentum of the two discs  MK2 =  The body is a disc.
I11 + I22 2
Since the two discs are brought into contact face 245. Acceleration of an object rolling down an
to face (one on top of the other) and their axes inclined plane,

io
of rotation coincide, the moment of inertia Ic of gsin 
the composite system will be equal to the sum a=
 K2 
of their individual moments of inertia, 1  2 
R 

at
i.e. Ic = I1 + I2
If c is the angular frequency of the composite K2
For a ring, 1
system, the final angular momentum of the R2
system is gsin 

Icc = (I1 + I2)c
lic aring =
11
 0.5gsin 

Since no external torque acts on the system, K2 1


For a solid cylinder, 2 
Final angular momentum = Initial angular R 2
momentum g sin 
ub
or (I1 + I2)c = I11 + I22  acyl. =  0.67 g sin 
 1
 1  
I11  I22  2
or c =
I1  I2 K2 2
For a solid sphere, 2 
R 5
P

1 2 1 2
241. I  mv gsin 
2 2  asph =  0.71gsin 
 2
I 3  22
2
 1  
 v2 =   1  v  1m/s  5
et

m 12
As acceleration of the solid sphere is maximum,
1 1 hence the sphere will reach the ground with
242. K.E.trans. = mv2   0.4  22  0.8 J
2 2 maximum velocity.
rg

2
1 11  v 246. The disc rolls about the point of contact with the
K.E.rot = I2   MR 2   2
2 2 2  R horizontal surface, therefore speed of centre of
1 1 mass is v = r  and that of topmost point is
= Mv 2   0.4  22  0.4 J
Ta

4 4 2 r  = 2 v.
 K.E.tot = 0.8 + 0.4 = 1.2 J 247. In the case of rolling, as K.E.,
1 2 1 1  I 
243. Total K.E. of the loop = I + Mv2 E= Mv2 1  2 
….(i)
2 2 2  MR 
1 1 For ring, I = MR2
= MR22 + Mv2
2 2  2
Ering = M ring v ring
2
= Mv = 8 J
E ring
….(i) [ R2 2 = v2]  vring = .…(ii)
0.3
1 1
 Total K.E. of the disc = Mv2 + I2  For cylinder, I = 1 MR2
2 2 2
2
1 1 1 v 3
= Mv2 +  MR2  2  2
Ecylinder = M cylinder v cylinder .…[from (i)]
2 2 2 R 4

163

MHT-CET Triumph Physics (Hints)

4E cylinder 1
 vcylinder = 252. ET = mv2 and
3  0.4 2
1 1 v2 1 K2
E cylinder ER = I2 = (MK2) = Mv2 2
= .…(iii) 2 2 R 2
2 R
0.3
1
According to problem, Mv 2
ET 2 R2 5
Ering = Ecylinder  R= = = =
ER 1 K2 K 2
2
 vring = vcylinder .…[From (ii) and (iii)] Mv 2 2
2 R
As the motion is uniform, both will reach the
wall simultaneously. 253. For slipping or sliding without rolling,
1 a = g sin  and v = 2gh
248. ET =  1  K  Mv2
2

 2  2 For rolling without slipping,


 R 

ns
 a = gsin 
 K2  1
ER =  2  Mv2 (1  K 2 / R 2 )
R  2
 v = 2gh
 The fraction of total energy associated with
(1  K 2 / R 2 )

io
ER K 2 /R 2
rotation is = As a < a and v < v, slipping cylinder reaches
ET 1+ K 2 /R 2
the bottom first with greater speed.
For solid sphere, K2/R2 = 2/5

at
ER 2 gsin 
 = 254. a = = gsin 30
E total 7  K2   2
1  2  1  
 R   5
2
MR2
249. For solid sphere, I =
5
lic  a = 5g   1  = 5g
7 2 14
1
ET = Mv2
2 256. The acceleration is given by,
1 1 2 1 1
ER = I2 =  MR 2  2 = MR2 2 = Mv2 a = gsin 
ub

2 2 5  5 5  K2 
1 1 7 1  2 
 2
E = Mv + Mv = 2
Mv2  R 
2 5 10
 a = gsin  ….( I = MK2)
 I 
P

1 1  2 
250. E1 = Mv2,  MR 
2
1 1
258. ET = 1  K 2  Mv2
E2 = Mv 2  I2
2
1
et

2 2
 R  2
1 1
= Mv2 + (MR2)2 ER = K 1 Mv2
2
2 2
1 1 R2 2
= Mv + Mv2 = Mv2
2
rg

2 2  The fraction of total energy associated with


1 ER K2 / R2
Mv 2 rotation is, =
E1 1 E Total 1  K 2 / R 2
 = 2 2 =
E2 2
Ta

Mv K2
 For a ring, =1
251. Total energy = K.E. of translation + K.E. of R2
rotation ER 1 1
 = =
1 1 1 1 2 ET 11 2
= Mv2 + I2 = Mv2 +  MR22
2 2 2 2 5
1 1 7 259. As the body rolls the inclined plane, it loses
2 2 2
= Mv + Mv = Mv potential energy. However, in rolling, it acquires
2 5 10
both linear and angular speeds and hence gains
1 2 1 the kinetic energy of translation and that of
  I
2
K.E.of rotation   Mv 2
 =  2  =  5  = rotation. So, by conservation of mechanical
Total energy 7 2 7 2 7 energy,
  Mv   Mv
 10   10 
2 Mgh = 1 Mv2 + 1 I2
 Percentage of (K.E.)R =  100% = 28.57% 2 2
7 But for rolling, v = R
164

Chapter 01: Rotational Dynamics


265. Using,
 Mgh = 1 Mv2 1  I 
2 MR 2  smg  mr2

I sg = r2 (For minimum angular speed)
Let 1 + =
sg 25
MR 2 2 = = 0.25×9.8 =  9.8
r 5×10-2
5
 Mgh = 1 Mv2
2 = 9.8  5 = 49.0
2gh   = 7 rad/s
Hence v =
 mv 2
266. Tension at mean position, mg +  3mg
r
, where h = l sin 
260. v = 2g h
1  K2 / R 2
v= 2gl .…(i)
and if the body displaces by angle  with the

ns
For solid sphere, v = 10 gh
7 vertical then v = 2gl (1  cos ) .…(ii)
10  10  3.5  sin 30 = Comparing (i) and (ii), cos  = 0
 v = 10  g  l sin  = 25
7   = 90
7

io
 v = 5 m/s 267. Let the bead starts slipping
after time t
261. Centripetal force on electrons is provided by For critical condition,

at
electrostatic force of attraction.
frictional force provides the
 F  1 and r  n2 where n is principal quantum centripetal force
r2 L
m2L = R = m  a1 = Lm
number.  m(t)2L = mL
 F1 n 42  3 
 4   
81
4
lic 
F2 n1  2  16 t= ... [  = t]

262.  = 3  2 268. Using,


ub
2 mv 2
= mg
 Work done W =  = Fr r
3   v2 = gr
= 200  3    2 
2 
P

v = gr = 10×12.1 = 121 = 11 m/s


= 5652 J
269. Moment of inertia of complete disc about O is I
263.
= 1 MR2.
et

m
O T 2
l M
Mass of the cut - out part is m =   .
 4
T
rg

m The moment of inertia of the cut-out portion


m about its own centre,
2
1 1  M  R  1
Tension T in the string will provide centripetal I0 = mr2 =    = MR2
Ta

2 2  4  2  32
mv 2
force  T ….(i) because r = R/2.
l From the parallel axes theorem, the moment of
Also, tension T is provided by the hanging ball inertia of the cut out portion about O is
of mass m, 2
1  M  R  3
 T = mg ….(ii) Ic = I0 + mr2 = MR2 +    = MR2
32 4
   2 32
mv 2 v2
mg = g=  Moment of inertia of the shaded portion about O
l l
is
264. Using, Is = I  Ic
v = rg = 0.4  50  9.8 = 196 1 3
= MR2  MR2
v = 14 m/s 2 32
v 14 13
= = = 0.28 rad/s = MR2
r 50 32

165

MHT-CET Triumph Physics (Hints)


270. From the law of conservation of energy, we have Friction will act in upward direction.
Potential energy = Translational kinetic energy Since velocity is a constant,
+ Rotational kinetic energy  mv 2 
1 2 1 2 N =  mgsin   
or mgH = mv  I  R 
2 2
 mv 2 
1 11  3 f =   mgsin     mg cos  [at = 0]
or mgH = mv22   mr 2  2 = mr 22  R 
2 2 2  4
4gH As  increases, cos  decreases  friction
or 2 = decreases.
3r 2 mv 2
1 2 f R N
Now the rotational kinetic energy = I
2
mg sin  mg cos
 Substituting for 2 and I, we have, mg

ns
1  1 2  4gh
Rotational kinetic energy =  mr  2 
2 2  3r
mgH Again, at = 0
=
3

io
 mv 2 
 Friction =   mgsin    = mg cos 
mv 2 k  R 
271. = 2
r r  As  decreases, cos  increases  friction

at
k increases.
 mv2 =
r
275. m2r cos  = mg sin 
1 k
 K.E.= mv2 = g tan  N
2 2r  2 =
k
P.E. =  Fdr =  2 dr = 
k
lic r
 m2r
v2
r r  tan  = h
k k k rg 
 Total energy = K.E + P.E =  = 2 l
  1000 m  
ub
2r r 2r
 72   
1 h   3600 s   mg
272. Ic = MR 2  
2 l (400m)(10m/s)
 MR2 = 6  2 = 12 h

1
P

Using theorem of C 1m 10
parallel axes,  h = 10 cm
IR = Ic + MR2
= 6 + 12 = 18 kgm2 276. At the highest point,
et

Ic IR g
= = 2n
R
273. Let particle A be situated B 1 g g
 n= 
rg

on the inner part and B on R2 2 R 42 R


the outer part of the ring.
As the ring is moving with g 900g
A  r.p.m. = 60n = 60 
uniform angular speed, 42 n 2R
R1
Ta

both the particles will 277. For option (A),


experience a centrifugal Net force = Mv2/r = Mass  acceleration
force For option (B),
F1 FA m2 R1 F R  
    1 1 at and  are perpendicular hence cross product
F2 FB m R 2
2
F2 R 2
is not 0.
274. mv 2 For option (C),
N R Angular velocity and angular accleration have
f the same direction or opposite direction
according to the type of motion.
mg For option (D),
The correct statement is:
The resultant force acts always towards the
 centre.

166

Chapter 01: Rotational Dynamics


10. Tangential acceleration: at = r …(i)
Competitive Thinking v2
Radial acceleration: ar = …(ii)
2π r
1. hour = Dividing equation (i) by equation (ii),
Thour
r 2
2π 180 180°  at
= r = 2
= × ....{ 1c = } ar  v2  v
12 × 60 × 60 π π  
1  r 
hour = degree / s
120 11. Centripetal acceleration,
4 2 r 4 2
2. Angular speed of second hand, ac = 2r =   5  102 = 5 ms2
2 T2 (0.2) 2
1 = (T = 60 seconds) As particle is moving with constant speed, its
60

ns
Angular speed of hour hand, tangential acceleration, aT = 0.
2
The acceleration of the particle,
2 = (T = 12 hr) a = a c2  a T2 = 52  02 = 5 m/s2
12  60  60
1 720

io
= 12  60 = 12. In given figure,
2 1   
Total acceleration a  a t  a r
2  ar = a. cos  a.sin
3. Angular speed of minute hand, m =

at
60  60 v 2
a.cos
also, ar = 
2 r a
Angular speed of second hand, s =
60 v2 O
 a. cos  =
 s  m =
2

2
=
60 3600 1800
59 
rad / s
lic r
v2
 15cos (30) =
4. Degree moved by hour hand, 2.5
for 1 revolution = 360  v2 = 32.5
ub
360 v = 5.7 m/s
for 1 hour = = 30
12 14. v
30
for 1 min = = 0.5 T m
60
P

l
 for 20 mins = 20  0.5 = 10
Hence, at 12.20 pm
Here, tension provides required centripetal force.
Angular seperation = 120 – 10 = 110
et

mv 2
  
i.e., =T
6. l
ar   v
2
mv 2 m  p  p2
7. 15. Radial force =    =
rg

ax  P(R,) r r m mr
a
….[ p = mv]

2
R ay 16. F = m r
Ta

2
Substituting for r = 2l,  =
T
2

v2 v2  2 
a =  cos  ˆi  sin  ˆj kl = m(2l)   ….(i)
R R T
….( F = kx and x = l here)
8. They have same angular speed .
Centripetal acceleration = 2r Upon speeding, F1 = m12 r1
a1 2 r r 2
= 21 = 1 Substituting for r1 = 3l, 1 =
a2  r2 r2 T1
2
2
 2 
9. Since, n = 2,  = 2  2 = 4 rad/s k(2l) = m(3l)   ….(ii)
 T1 
25
So acceleration = 2r = (4)2  m/s2 = 42 ….( x = 2l here)
100
167

MHT-CET Triumph Physics (Hints)


Dividing equation (i) by equation (ii), For equilibrium of the block,
kl m(2l )(2 / T) 2 F  mg
=
k(2l ) m(3l )(2 / T1 ) 2  µN  mg
2  µ(mrɷ2)  mg (∵ N = mrɷ2)
 T1  3
   = g
T
  4  ɷ
µr
3
 T1 = T
2 g 10
 ɷmin = = = 10 rad/s
µr 0.1  1
mv12  2m  v 2
2
17. 
r r 25.
2 Tcos

 v12  4v22  T

ns

 v1 = 2v2 h L Tsin m

18. r1 = 4 cm, 2 = 21


r2 = constant mg

io
 r1 12 = r2 22  r1 12 = r1 (21 )2 = r1 = 4 r2 O m
r
r 4
 r2 = 1 = = 1 cm
4 4

at
The centripetal force required for circular
19. Time period of rotation of both the particles (A motion is given by
and B) is same mv 2
 TA = T B = T sin ….(i)
r

2π 2π 
lic
2π  Also we have,
=
ωA ωR  t  ω  mg = T cos  ….(ii)
 
Dividing eq(i) by eq(ii) we get,
ωA
 =1:1 mv 2 1
= T sin 
ub
ωB 
r mg T cos 
21. As the car moves on a plain horizontal circular  v2 = rg tan
track, the only force that can provide centripetal  v = rg tan  ….(iii)
acceleration so that the car does not skid is
P

From figure,
frictional force.
r
mv 2 v2 tan =
  mg = h
r rg
r
et

5  tan = ….(iv){L2 = r2 + h2}


v = 60 km/hr = 60  m/s, r = 60 m, g = 10 m/s2 L  r2
2

18 Substituting equation (iv) in equation (iii) we


2

 =  60  
5 get,
60  10
rg

 18  r
v= rg
  = 25/54 L  r2
2

v2 g
For banking, tan  =  v= r
Ta

23.
Rg L  r2
2

v2 28. rg
tan 45 = =1 vH =
9010
v = 30 m/s
3rg
24. vM =
O M
f
B
N mrω2 vL = 5rg
mg Centripetal acceleration at midway point (M)
D
v2 3rg
= M  3g
r r

168

Chapter 01: Rotational Dynamics


5 33. Y
29. (K.E)L = mgr …(i)
2
D C
1
(K.E)H = mgr …(ii)
2
 Divide equation (ii) by equation (i) X
1  A B
(K.E) H  mgr 
 = 
2  = 1 = 0.2 IAB = 0 ….(i)
(K.E) L 5  5 ml 2
 mgr  IAD = IBC = ….(ii)
2 
3
IDC = ml2 ….(iii)
30. Tension at any given point,
From equations (i), (ii) and (iii),

ns
mv2  Total moment of inertia
T= + mg cos 
r ml 2 ml 2 5
I=0+  + ml2 = ml 2
At the lowest point, 3 3 3

 = 0 34. A distance of masses

io
2 and 3 from axis of m
 cos  = 1 rotation is zero, they 1 2
mv 2 don’t contribute to

at
 T= + mg moment of inertia. l
r
I1 = I4 = mR2
i.e., Tension is maximum at the lowest point So  l 
2
ml 2
=m  3
the chances of breaking are maximum.  = 2 4
lic 
 2
ITotal = I1 + I4 = ml2
1
31. M.I. of disc, I = MR d2 ...(i)
2 35. According to the given condition,
2 MR 2  R 2 L2 
ub
M.I. of sphere, Isphere = MR S2 ...(ii)  M  
5 2  4 12 
 volume of disc = volume of sphere R 2 R 2 L2
  
R  4 2 4 12
 πR d2  d  = π R S3 R 2 L2
P

 6  3  
4 12
 3
R = 8R
d
3
S 12 R 2
 L2 =
RS = R d 4
et

 ...(iii)  L2 = 3 R 2
2
Substitute equation (iii) in equation (ii)  L = 3R
2
R  2
rg

2 2 1
 Isphere = M  d  = × MR d2 36. Isphere = Is = mR 2
5  2  5 4 5
Let s be angular speed of sphere,
1 1 2 I
=  MR d  = ...from (i)  Esphere =
1
Is s 2
Ta

5 2  5
2
1 2 
ML2 =  mR 2  s 2 ....(i)
32. M.I. of thin Rod about one end, I = 2 5 
3
Similarly,
L 1
Now, L = 2R  R = Icylinder = Ic = mR 2
2 2
M.I. of ring about diameter, Let c be the angular speed of cylinder,
 L  2 Then it is given
M 2  c = 2s
MR 2  4   ML
2
I1 =  1
2 2 8 2  Ecylinder = Icc 2
2
I ML2 82 82 11
    
=  mR 2   2s 
2
I1 3 ML2 3 ...(ii)
2 2 

169

MHT-CET Triumph Physics (Hints)

1 2 2
R

E sphere  mR  s
2 5
2 41. M=  0
0 r  (2rdr)
 =  ...[From (i) and (ii)]
E cylinder 11 2 20 R 3
 mR   4s   M=
2

2 2  3
1 R 2
=
5
IC =  0
0 r  (2rdr)  r

20 R 5
37. K.E. possessed by rotating body,  IC =
5
1
(K.E.)rot = I2 By parallel axis theorem,
2
I = IC + MR2
= 1 (MK 2 )  v 2 
2

2 0 2R 5 0  2R 3
R   I=   R2

ns
5 3
= 1 Mv 2  K 2 
2

 1 1  160 R
5

2 R  = 02R5    
5 3 15
For M, R and  same, v becomes constant.
8 2 3 2 8 2
 I=  0 R  R = MR

io
2
Hence, as K2 increases, K.E. i.e., work done in 5 3  5
R
bringing body to rest increases. 8
 a= (∵ I = aMR2)
5

at
K 2
2 K  1 K  2 2

 2   ,  2   and  2  = 1
 R A 5  R B 2  R C 42. I0 = Ic + Mh2
 WC > WB > WA Ml 2 l Ml 2 Ml 2
2

= + m  = +
38. Initial K.E., (K.E.)i =
1 1
I 12 + I22
lic 12 4 12 16
2 2 7Ml 2
 I0 =
1 48
Final K.E., (K.E.)f =  (2I2)
2 I 7 Ml 2
ub
2 But K = =
 1  2  M 48M
=I  
 2 
7
 Loss in K.E. = (K.E.)i  (K.E.)f  K= l
2
48
1 2 1 2  1  2 
P

= I1  I2  I  
2 2  2  Ml 2 MR 2
44. I1 =  and l = 2R
I 12 4
= (2 12 + 2 22  12  212  22 )
4 Ml 2 MR 2
et

I2 =  and l = 2R
I 3 4
= (1  2)2
4 4MR 2 MR 2 MR 2 MR 2
I2  I1 =   
3 4 3 4
39. M.I. of rod about an axis passing through centre,
rg

4MR 2 MR 2 MR 2
IC =
ML2
= MK12 ...(i) =  = (4  1)
3 3 3
12
M.I. of rod about an axis passing through one  I2  I1 = MR2
Ta

end, ML2
ML2 45. Icm = (about middle point)
IE = = MK 22 ...(ii) 12
3
L
Divide equation (i) by equation (ii)
2 2 I Icm
MK 1 ML 3
2
= ×
MK 2 12 ML2
K12 1
 =
K 22 4
L/3 L/6 L/2
K1 1
 =
K2 2  Applying theorem of parallel axes,
2
40. According to perpendicular axis theorem, ML2 L ML2
I = Icm + Mx2 =  M  
Iz = Ix + Iy = 20 +25 = 45 kg m2 12 6 9

170

Chapter 01: Rotational Dynamics


46. 48. Y

m1 m2 P
O
x1 x2
O X
l

Let O be the centre of mass of the system


m 2l
 x1 = ...(considering m1 as origin)
m1  m 2

ns
Using parallel axes theorem,
m1l
x2 = ...(considering m2 as origin) M.I. about origin O,
m1  m 2
MR 2  MR 2 2
 M.I. of the system is given by, IO = +6   M  2R  
2  2 

io
I = m1x12 + m2x22
2 2 MR 2 54MR 2
 m 2l   m1l   IO = +
= m1   + m2   2 2
m
 1  m 2  m1  m 2 

at
55 2
m1m 22l 2  m 2 m12l 2 m1m2 (m2  m1 )l 2 IO = MR
= = 2
 m1  m 2  (m1  m2 )2
2
Similarly, using parallel axes theorem,
M.I. about the point P will be,
=
m1m2l
(m1  m 2 )
2
lic IP = IO + 7M(3R)2
55
47. IP = MR2 + 63MR2
I 2
ub
181
IP = MR2
2
R R/2
49. Y 9M,R
P

R
2R
Moment of inertia of disc is given by r= 3
3
Idisc = Ir + Ihole ….{Ir = M.I. of remaining part} X
O
et

 Ir = Idisc  Ihole ….(i)


MR 2
Idisc = ….(ii)
2
rg

By parallel axes theorem we get,


 M  R 2 
   2 Mass of portion removed will be,
M  
R
Ihole =       
4 2 2
M0
 (r)2 = 2    = M
9M R
Ta

 2 42  m=




 R 0 
2
R 3
M.I. of the remaining part of the disc,
 M 
 M hole  disc
….  4

  R
2

 the surface density is same  2 M   2

I=
9MR 
  3   M  2R  
  
 MR 2 MR 2 
2 2  3  
 Ihole =   ….(iii)  
  
 32 16 
Substituting eq (iii) and eq (ii) in eq (i) we get, 9MR 2  MR 2 4MR 2 
 I=   
MR 2 MR 2 MR 2 2  18 9 
Ir =  
2 32 16 9MR 2  9MR 2  9MR 2 MR 2
 I=   
 1 1 1  13 2  18  2 2
= MR2    = MR2
 2 32 16  32  I = 4MR2
171

MHT-CET Triumph Physics (Hints)


50. I I Differentiating w.r.t. l on both sides,
dI m  V 2l 
   
dl 4  l 2 3 

O But for moment of inertia to be minimum,


I dI
0
dl
V 2l
 
l 2 3
2l 3
From the figure,  V=
3
2 2
MR MR
Ic = and I =  MR2 = 4I ….(i)  R2l =
2l 3
2 4

ns
3
Using theorem of perpendicular axes, 2
l 3
Ic = 2Id = 2I ….(ii)  
Now, using theorem of parallel axes, R2 2
It = Ic + MR2 = 2I + 4I = 6I 
l

3

io
….[from (i) and (ii)] R 2
51. Moment of inertia of a rod about 53. Angular momentum acts always along the axis
an axis passing through centre

at
perpendicular to the plane of rotation.
and perpendicular to its length is
ml 2 l 54. Angular momentum
= = I1 = linear momentum  Perpendicular distance of
12 2 3
Where l = length of the rod.
Using parallel axes theorem;
lic line of action of linear momentum from the axis
of rotation
M.I about centroid = (M.I)cm + Mh2 = mv  l
l 55. We know,
Here h =
ub
2 3 1
K.E. = I2
ml 2 ml 2 2
 M.I about centroid = 
12 12 Here,
2ml 2 (K.E.)A = (K.E.)B ...(Given)
 M.I of each rod about centroid =
P

12 1 1
2 2
 I A A 2  I B  B 2
2ml ml 2 2
 M.I of system = 3  = = I2
12 2 As IB > IA,
et

Given I2 = nI1 B < A


ml 2  ml 2  Also, K.E. =
1
...( L = I)
 = n  L
2  12  2
rg

1 1
 n=6  L A A  L B  B
2 2
52.  as B < A
LB > LA
Ta

R 56. We know that, L = I


 L1 = I11 and L2 = I22
l L1 I11
 
L2 I22
L 2 / 5M1R 21
 
mR 2 ml 2 L 2 / 3M 2 R 22
I= 
4 12 ( L1 = L2 = L and R1 = R2 = R is given)
m  l2  3 M1 1
I =  R2   1=
4 3 5 M2 2
m  V l2  M1 10
=    ....( V = R2l)  
4  l 3  M2 3

172

Chapter 01: Rotational Dynamics


57.  = mg  l sin . (Direction 65. Torque at angle 
parallel to plane of rotation l
 = Mg sin  ….(i)
of particle) 2

as  is perpendicular to L , Z
l l
direction of L changes but
magnitude remains same. , 
     m
59. Power =  . = ( r  F).  
 Mg
60. As  = 0 + , X
  0 0  4.6 4.6 Also,
 =   rad s2
t t t  = I ….(ii)
Negative sign is for retarding Torque l

ns
 I = Mg sin  ….[from (i) and (ii)]
Using  = I, 2
4.6 M.I. of rod here is,
6.9  102 = 3  102 
t Ml 2
….(Considering magnitude only) I=
3

io
3  10 2  4.6 Ml 2 l
 t=  2s   = Mg sin 
6.9  102 3 2
l gsin 

at
1  =
61. R = 20 cm = m 3 2
5
Moment of inertia of
R 3gsin 
 =
flywheel about its axis, 2l
I=
1
MR2
lic 66. As no external torque acts on the body, its
2 angular momentum will be conserved.
2
1 1 2
=  20    = 0.4 kg m 67.
2 5
ub
Using  = I, F = 25 N
1
25 
 FR 5  5 Nm = 12.5 s2
=  
I I 0.4 0.4 kgm 2
P

62. 30 N Here, the law of conservation of angular


momentum is applied about vertical axis passing
through centre. When insect is moving from
et

circumference to centre, its moment of inertia


will first decrease and then increase. Hence
 = I angular velocity will first increase and then
rg

 RF F 30 decrease.
 = = = = = 25 rad/s2
I mR 2
mR 3  0.4
68. I11 = I22
63. Torque zero   is zero I = 2 I2 ….( I2 = 2I)
Ta

 = 2t3  6t2 
d  2 =
 = 6t2  12t 2
dt 1 2
d 2  K.E.1 = I
  0  12t  12 = 0 2
dt 2 1
 t = 1 second K.E.2 = I222
2
64. Using, 2
….  I 2  2I, 2   
1
=  2I 
Tr = I , 2 4  2
I mr 2  mr T 2
T=    r I
r 2 r 2 =
50  0.5  2  2
4
= N
K.E.1 – K.E.2 = 1 I2 1  1   1 I2  1  I
2
2 
= 157 N 2  2 2 2 4

173

MHT-CET Triumph Physics (Hints)

69. Initial angular momentum of ring, L = I = Mr2


= 1 Mv2 1  2 
Final angular momentum of the system 2  5
consisting of ring and four particles,
 
L = (Mr2 + 4mr2)  K2  2 
….   
As there is no torque on the system, hence  R 2 solid 5 
angular momentum remains constant.  sphere 
M 1
 Mr2 = (Mr2 + 4mr2)   = Mv 2
M  4m  Kt = 2 = 1 5
 t
K  K r 1  2 7/5 7
Mv 2  1  
70. I = (I + I) 2  5
 
 I   I  8 76. aslipping = g sin 
 =   =   = 
 I  I  I
 I   9 gsin  gsin  5
arolling = = = gsin 

ns
 8  K2   2  7
 1  2  1  
71. By conservation of angular momentum,  R   5
 I1  a rolling 5
I1 1 = (I1 + I2)2  2 =   1  

io
 I1  I 2  a slipping 7
 Loss in kinetic energy = (K.E.)i  (K.E.)f
77. Krolling = Kf + Ur
1 1 1 I I 
= I112   I1  I2   22  =  1 2  12 Ktrans + Krot = 0 + Mgh

at
2 2 2  I1  I2 
1 1 3v2
 Mv2 + I2 = Mg 
72. According to law of conservation of angular 2 2 4g
momentum, v2 3
I11 = I22
lic  Mv2 + I
R 2
= M. v2
2
 25I1 
 I11 =  I1   2 I 3 MR 2
 100   M+  .M  I =
R2 2 2
 I11 = 0.75I12
ub
I1ω1 78. Velocity of the small object is given as,
 2 =
0.75I1 2gh
v =
1.5π k2
= ….(Given: 1 = 1.5 ) 1+ 2
r
P

0.75
= 2 2g3v2
2π 2π  v2 =
But f = = = 1 rps æ k2 ö
ω 2π 4g çç1+ 2 ÷÷÷
çè r ÷ø
et

In rpm; f = 60 rpm
2

73. Hollow cylinder will take more time to reach the  1 + k2 = 3  k2 = 1 r2


r 2 2
bottom because it possesses larger moment of
rg

inertia. I
But k =
M
74. Acceleration of a rolling body on an inclined
 I = 1 2  I = 1 Mr2  disc
plane is given by r
Ta

M 2 2
g sin 
a=
K2 1  K2 
1 2 79. K.E.initial = mv2  1  2 
r
2  R 
 K2  2  K2  1
 2  = ;  2  = 1  1 3
 r sphere 5  r disc 2  For disc, K.E.initial = mv2  1    mv2,
2  2 4
 asphere > adisc K.E.final = 0
 sphere will reach the bottom of the plane first.
Work done = change in kinetic energy
75. For solid sphere: 3
=0– mv2
1 4
Kt = Mv2
2 3
=– (100) (0.2)2 = – 3J
and (Kt + Kr) = 1 Mv 2 1  K 2 
2
4
2  R   |Work done| = 3 J

174

Chapter 01: Rotational Dynamics


80. 86.
 T
x
h
mg mg cos
mg sin
30
From the figure,
T = mg cos + mg sin
2gh
v=  T = mg cos + mv2/L
K2
1 2
R 87. Q2
2  10  h 4 0 R 2
 42 =

ns
1
1 vH
2 Mg
 K2 1  Q
....  For solid cylinders 2   

io
 R 2
3 vL
 2  10  h = 16  Q
2

at
24 At highest point, T = 0
 h= = 1.2 m Q2 mv 2H
20  Mg – =
40 R 2 R
h
Now, sin  =
x
lic But Mg =
Q2
.... (Given)
40 R 2
1.2
 sin 30 =  vH = 0
x
According to work- energy theorem
ub
1.2  W =  KE
 x= = 2.4 m
1/ 2 1 2 1 2
mg (2R) = mvL  mv H
2 2
81. If ring, solid disc and solid sphere are released
1 2
from same height on an inclined plane, then = mv L ....( vH = 0)
P

after rolling they will reach the bottom in an 2


order:  vL = 2 gR
Solid sphere, solid disc and ring ;as smaller the 88.
et

K2
value of 2 , early it will fall.
R Fx
 vS  vD  vR M
rg

Fy
1 2E
83. E= mv2  v2 =
2 m
At midway point (M),
v2 2E
Ta

a= = Fy = mg
r mr
Fx =
mv 2M
r
= 3mg 
.... v M  3rg 
84. Change in momentum
 mg    3mg 
2 2
= Mv – (–Mv) = 2 Mv Fnet = Fy2  Fx2 =

85. Centripetal acceleration = 10 mg


v2 89. v2 = u2 + 2atS
= K2 t2 r
r  v2 = 2atS ....{ u = 0}
2
 v=Ktr at =
v
dv d  2S
acceleration, a = = (K t r) = Kr
dt dt At the end of second revolution, the particle
F=ma travels a distance equal to twice the
circumference of circle.
and P = F  v = mKr  Ktr = mK2 t r2
175

MHT-CET Triumph Physics (Hints)

 S = 2(2r) = 4r  v2 = K 
R
= K  1 n 1
v2 mR n mR
 at =
2  4πr 
 
 v = K  1 ....  K  K 
v2  n 1   m
 at = 
8πr R 2

The time period of rotation is,


90. m = 10 g = 0.01 kg n 1
r = 6.4 cm = 6.4  10–2 m, n 1
T = 2R  2R  R 2
2

1  R 2
K.E. of particle = mv2 v K K
2 n 1

1  TR 2
 mv2 = 8  10–4 J
2
16  104 95. Potential energy is given to be,

ns
 v2 = = 16  10–2 k
0.01 U=  ….(i)
2 2
v = u + 2ats 2r 2
 v2 = 2ats ….{ u = 0} The force acting on the particle will be,

io
s = 2(2r) F = dU  d  k2    k  32 
 v2 = 2at4r dr dr  2r  2 r 
v2 16  102 k
 at = = = 0.1 m/s2  F= 

at
8r 8  3.14  6.4  102 r3
As the particle is moving in circular path, the
91. Speed of the body after just reaching at the
force acting on it will be centripetal force.
bottom is v = 2gh ….(i)
It just completes a vertical circle using this
lic
 F =  mv   k3
r r
2
 mv2 =
k
r2
velocity.
1 k
To complete vertical circle, speed required is v Now, K.E. = mv2 = 2 ....(ii)
2 2r
v = 5g D
ub
….(ii)  Total Energy E = K + U = 0
2
From equation (i) and (ii), ....[from (i) and (ii)]
D
 2gh = 5g 96.  
2
P

aT at
5 
 h= D
4 R

92. Given ac
et

Angular acceleration
 = 2 rad s2
 Angular speed  = t =
rg

(2) (2) = 4 rad/s Velocity of object is given as


ac = r2 = 0.5  16 = 8 m/s2 V= K S ….(i)
at = r = 1 m/s2 Centripetal acceleration of the object is,
Resultant acceleration is given by, V2
Ta

2 ac = ….(ii)
a= a c2  a 2t = 8  1  8 m/s
2 2
R
Tangential acceleration is given by,
93. Work done = increase in kinetic energy
dV dV dS
1 2 1 2 1 2 at = =
W= I 2  I 1 =  2  12  dt dS dt
2 2 2
dV
= 2 I  v 2  v1 
2 2 2 =V
dS
W
 I =
2  v22  v12 
2
= K S
d
dS
K S   ….from (i)

1
94. The centripetal force acting on the particle is = K2 S
provided by the central force, 2 S
2
mv 2
1 K
 =K n at = ….(iii)
R R 2

176

Chapter 01: Rotational Dynamics


from figure, 99. For the rod PQ,
V  2 ML2
ac =T L
2
tan  = =  2 ….From (ii) and (iii)
at  R K 3 2
L/2
2 K 2S P Q
 tan  = ….from (i)
R K2
2S Mg
 tan =
R Now, T = Mg
ML2
97. At an instant, speed of P = v, going in clockwise   = Mg  L
direction 3 2
Speed of Q = v, going in anticlockwise direction  = 3g
2L
Relative angular velocity of P w.r.t.

ns
Q =  () = 2  100.
0 
Relative angular separation of P and Q in time t,
vCM
 = 2 t.

io
Relative speed between the points P and Q at
time t 1 2

| vr | = v  v  2vvcos(2r)
2 2 L1 = L2;

at
L1 = LCM + mvCMr = LCM + mr20 = mr20
= 2v 2 (1  cos 2r)
[ LCM = 0 initially]
= 2v 2  2sin 2 r lic L2 = LCM + mvCMr = mr2 + mr2 = 2mr2
= 2v sin r 2mr2 = mr20

Since, | v r | will not have any negative value so 0
 =
the lower part of the sine wave will come upper 2
side.  vCM = r = r0
ub
2
98. B
MR 2 
101. Torque:  = I = 
r 2 t
MR 2
P

r   =
2t
A C D
2
But  = R × F
 F =  = MR 
et

Let A be initial position of point of contact and R 2t


B be its position after the wheel completes half
102. 0 = 0,  = 24 rad/s, t = 8 s
revolution.
  0 24
= 3 rad/s2
rg

Distance travelled by the wheel in half  = =


t 8
C
revolution = = AD From kinematical equations for rotational motion,
2
1 2
 from figure ;  = 0t + t
Ta

2
Displacement of initial point of contact after 1
half revolution = AB =0+  3  (8)2
2
 AB2 = AD2 +DB2 = 96 rad.
2
C 103. I = 2 kg m2
AB2 =   +  2r 
2
2 0 = 60 rad/s
C We know,
But r =
2   0
=
2 2 t
C C
 AB2 =   +   After time t = 5 min = 300 s,
2 π
=0
C2 C2 1 1 0  60 1
 AB =  = C   = =  rad/s2
4 2  2 4 300 5

177

MHT-CET Triumph Physics (Hints)


3 min before stopping i.e., 2 min from starting, 1
108. K.E. in first case = I2 = E
 = 0 + t 2

 1 In second case,
= 60 +     120
5
  I = 3I
= 36 rad/s According to conservation of angular
Now, L = I = 2  36 = 72 kg m2/s momentum,
I = I
L2
104. K.E. = I I 
2I   = = =
I 3I 3
From conservation of angular momentum about 1
centre, L has to remain constant Now, K. E. in second case = I 2
2

ns
L2
K.E. = 1 2
2(mr 2 ) =  3I 
2 9
L2 L2 1 1
 K.E. =  4 
 r2 2(mr 2 ) =  I2 

io
2  m.  3 2 
 4  1
= E
 K.E. = 4 K.E. 3

at
 K.E. is increased by a factor of 4.  1 
E  E
K  3  2
105. Using principle of conservation of angular  = =
K E 3
momentum
R0
lic 109. Using principle of energy conservation,
mv0R0 = mv  v = 2v0
2 K.E. of rotation + K.E of translation of falling
1 2 mass = loss in P.E.
K.E. = mv  2mv02
ub
2 1 2 1 2
mgh = I  mv
2 2
106. According to law of conservation of energy,
P.E = K.E 1 2 1
 mgh = I  m2 r 2 [ v = r]
2 2
P

1 1
 mgh = mv2 + I2 2mgh
2 2  2 =
(I  mR 2 )
1 1  mr 2  2
 mgh = m (r)2 +  
et

1
2 2  2   2mgh  2
 =  2
 mr 2   I  mr 
.…  v  r, I  
 2 
rg

110. a = R
3
 mgh = m2r2 mg – T = ma ….(i)
4 α
Also, T  R = mR2
3 22 R
Ta

 gh = r or T = ma ….(ii)
4
 Solving eq. (i) and (ii), T
4gh 1 4gh mg = 2 ma
 = 2
= T
3r r 3
g
 a= m a
2 2 2
107. Isphere = MR2 =  100  102 = 4000 mg
5 5 111. lP > lQ
1 1
Idisc = MR2 =  100  202 = 20000 gsin   gsin  
2 2 aP = and a Q  
lP  mR 2
 lQ  mR 2 
Changein I 20000  4000
 = 1
time 60  60  aP < aQ  v = u + at  t   tP > tQ
40 a
= kgm2 s–1
9  v2 = u2 + 2as  v  a  vP < vQ

178

Chapter 01: Rotational Dynamics


1 2  0 
2
 Translational K.E. = mv     0  2 (2n)
2
2  4 
 (Translational K.E).P < (Translational K.E.)Q 02
 2n(2) = 02 
v = R    v  P < Q 16
Hence cylinder Q reaches the ground with larger 15  02 
angular speed.  2n =   ….(i)
16  2 
1 2 when the fan stops rotating, 0 = 02  2  (2n)
112. Using S = at g sin 
2
02
1  2n = ….(ii)
S = g sin .(4)2 ….(i) 2
2 16
Comparing equations (i) and (ii), n = n

ns
S 1  15
  gsin .(t) 2 ….(ii)
4 2
Dividing equation (ii) by (i), 116. Work done; W = 
1 t2  W = I ….(  = I)
or t2 = 4  t = 2 s

io

4 16 W
 I=

1
113. Rotational K.E.of sphere = I 2  f2  i2  2 

at
2 W
 I= 2 2 2 ….  
2  n 2  n1   22  n 22  n12  
75% of K.E. = Heat energy  
1 2 75
 I  = MS 2π π
2 100
lic 117. ω1 = 3 rpm = 3  =
60 10
rad/s, ω2 = 0
1 2 2 2 75  2 
 MR   = MS  Isp  MR 2   = 2π rev = 4π2 rad
2 3 100  3 
π2
0
ub
R 
2 2
ω12  ω22 100  1
  = 
4S 2θ  
2 4π 2 800

km MR 2
114. v = 54 = 15 m/s  = I = 
h
P

2
v 15 rad
 
2
ω0 = = ,=0 2  4  102 1
r 0.45 s =  = 2  10–6 Nm
2 800
ω = ω0 + αt
et

15 118. When the truck moves towards right with


0= + (15)
0.45 acceleration ‘a’, then due to pseudo force, the
15 1 rad pendulum will tilt in backward direction (to the
rg

=– 
0.45  15 0.45 s 2 left) making an angle  with the vertical
 The magnitude of average torque a
3 300
Ta

 = I =  
0.45 45 
T
= –6.66 kgm /s2 2 ma

115. The fan initially rotates with angular velocity 0. 

 After switching off in time t, 2 = 02  2 From figure, mg


here,  = t T sin  = ma
and  = 2N T cos  = mg
as n revolutions are made in time t, a
 tan  =
n n g
N=   = 2    t = 2n
t t a
  = tan1  
  =   2(2n)
2 2 g
0

179

MHT-CET Triumph Physics (Hints)

Hints to Evaluation Test

1. For an hour hand, T = 12 hr = 12  3600 s 7. Angle moved =  in time t


2 2  l
= = = rad/s t= ….(v = velocity of bullet)
T 12  3600 21600 v
2. Using, v = r Also,  = t
l l 
= r  (2n) = 0.4  2  5   =    v =
= 0.4  2  3.14  5 v  B
A l
= 12.56  12.6 m/s

ns
3. Since this is not a case of a normal string, the
velocity at the topmost point can be zero. M M
 (T.E.)initial = (T.E.)final 9. = 3
=
4 3 4 R 7 4 3
1 R      R 
 mgh + mv2 = mg (2R) 3 3 2 8 3 

io
2
Mentire sphere = V
 v = 2g (2R  h)
M 4 
In case of a string, v at the topmost point =   R 3 
 7  4 3   3 

at
should be equal to Rg to complete the  8  3 R  
  
vertical circle as T = 0 and ball will fall
8
vertically down if v = 0. = M
7
4. Using,
lic = M1
1
 = 0t + t2 M
2 Mrem sphere = = M2
7
1
= 4  10 +  2  (10)2 = 140 rad
ub
2 2 R 
2

2  Isystem = M1 R2 –
 M 2 R  M 2   
2

140 5 5 2 
θ
n= =  22
2π 23.142 2 13
= M1 R2 – M 2 R2
5 20
P

  
5. v AB  v A  v B 2  8  2 13  M  2
=  M R –  R
Now, 5 7  20  7 

v AB = v 2  v 2  2v 2 cos(180  ) 16 13MR 2
et

= MR2 –
35 140
 
 [smaller angle between v A and  v B
=
 64  13 MR2
= 180  ] 
140
rg

vB
= 2v 2 (1  cos ) 51
B = MR2
140
= 2v 2  2sin 2 ( / 2) 
Ta

r2 r1
= 2 v sin (/2)  
= 2 R sin (/2) vA M CM M
A
2
10. In one complete revolution, total displacement
mv is zero. So average velocity is zero.
6. N cos  = 0
and N sin  = mg
r
g N d d
 tan  = r mv02 11.    k    kt  c1 
 v 02  dt dt
  r
 r    =  (kt  c1 ) dt
2 mg
v0 kt 2
 r= tan  =  c1t  c 2
g 2

= quadratic equation which has a graph of
parabola
180

Chapter 01: Rotational Dynamics


900 19.
12. n = 900 r.p.m. = r.p.s = 15 r.p.s, Initially Finally
60
1.2
d = 1.2 m  r = = 0.6 m
2
1.2
a = 2r = (2 n)2  = 540 2 m/s2
2
L L
 d  L
13.  =    So  is negative, if
 d 
d d
 > 0,  0 or  < 0, 0 ML2 ML2 2
d d Ii = + = ML2
6 2 3

ns
14. net = I   ML 2 2
 L  ML
2
 3L  
2

 If =   M    M  
 MR 2   12 2 12  2  
(Mg) R =   3mR 2  
 2  =
8
ML2
3

io
1  MR 2

Also, (Mg)R =   3mR 2  2  Ii i = If f
2  2  2 8
2 4mgR  ML2  = ML2 f
  = 3 3

at
 M  6m  R 2 
 f =
4mg 4
 =
R  M  6m  20. The catch here is that the incline is smooth/
lic frictionless. Hence, the rotational KE. of the
T sphere will not be affected.
15. Using, T = mr2  2 =
mr  Conserving Energy,
1
ub
6.4 mv2 = mgh
 =  3 rad/s 2
0.1  6
 v = 2gh
16. Using,
v= rg
P

= 0.4  30  9.8
= 10.84 m/s
et

17. The area under the t graph gives change in


angular velocity.
(2) 2 4
rg

Area =  = 2
2 2
 2  1 = 2
 2 = 2 + 2 = 4 rad/s
Ta

18. P
v


O v = R

vR = v 2  v 2  2v 2 cos 
= 2v 2 1  cos  


= 2 v sin  
2 

181
02 Mechanical Properties of Fluids

Hints

77. Refer Notes 2


Classical Thinking

ns
4T
F 40  9.8
88. Excess pressure inside soap bubble, P =
3. Pavg = = = 1.96  105 Pa r
A 2  10  10 4 Smaller bubble has more excess pressure.
Force 90. Since the soap bubble has two surfaces, excess

io
5. Pressure =
Area pressure is
mg 2  2T 4T
 hg = P= =
A r r

at
 Weight = mg = Ahg = V    g 1
91. P
P 105 r
7. h= = 3 = 10 m
g 10  10 P1 R 1

10.
lic
When air is blown through a hole on a closed

P2
= 2 =  P1 : P2 = 1 : 2
R1 2

pipe containing liquid, then the pressure will 1 r P 1


92. P   1  2 
increase in all directions. r r2 P1 3
ub
16. 70  13.6  g = h  3.4  g V1  r1  1
3

70  13.6    
 h= = 280 cm V2  r2  27
3.4
4T 4  0.04
93. P= =
P

47. Refer Shortcut 2


r 5  103
49. Refer Shortcut 9 4  40  103
= = 32 Pa
51. Weight = 2rT 5  103
et

Hence, radius remaining constant, W  T 2T 2  7.2  10 2


94. P= =
W1 T 30 1 r 10 3
 = 1 = =
W2 T2 60 2 = 14.4  101
rg

= 144 N/m2
52. Using T = F/l we get,
1N 1  105 dyne 4T 4  30
T= = = 103 dyne / cm 95. P= = = 400 dyne/cm2
m 102 cm r 3  101
Ta

53. Surface Tension = 70 dyne/cm 2T cos 


99. h= hT
70  105 rg
= = 7  10–2 N/m
102 1 1
102. h  
59. Since for such liquid (Non-wetting), angle of r D
contact is obtuse.
h 3 3
108. l =   = 6 cm
60. Cohesive force decreases; so angle of contact sin  sin 30  1 
decreases.  
2
61. Angle of contact is acute. vCD 8  1  1
118. Rn = = = 800
67. Waterproofing agents are used so that the  10 2
material does not get wet. This means angle of Since 800 < 1000
contact is obtuse.  The flow is streamline.
182

Chapter 02: Mechanical Properties of Fluids


121. Force of adhesion is more between the liquid 140.  (2R)2  v1 = (R)2 v2
layer and bottom of vessel. Hence velocity of  R 2 v1
liquid layer of bottom is least and velocity  v2 = = 4v1
R2
increases towards the surface.
141. A1v1 = A2v2
dv 12
124. = = 15/s  (1)2  5 = (0.5)2  v2 ( A = r2)
dx 0.8
15
dv  v2 = = 20 cm/s
125. Velocity gradient = 0.5 0.5
dx
1
dv 156. v = 2gh = 2  10  20 = 20 m s
 5=
2.5
 dv = 12.5 cm/s 157. v = 2gh = 2  9.8  0.1 = 1.4 m/s

ns
dv 0.05 158. P1  P2 = g (h2  h1)
128. F = A = 2  0.04  =8N
dx 0.0005 = 1040  9.8 (0.5)
P1  P2 = 5096 N m2
129. F = A  
dv

io
 dx  159. From the Bernoulli’s Principle
1
 =
F
=
2000 P1  P2 =   v 22  v12 
1 2
 dv  10 

at
A  1
 dx  0.1 =  1.3  [(120)2  (90)2]
2000  0.1 2
= = 20 poise = 4095 N/m2 or pascal
10

131. Since F = 6 r v


lic Critical Thinking
 Frv
1. At a point, pressure acts in all directions and a
F1 6r1v r r 1 definite direction is not associated with it, so
132. = = 1 = =
ub
F2 6r2 v r2 2r 2 pressure is a scalar quantity.
2. When two holes are made in the tin, air keeps
133. F = 6 rv
entering through the other hole. Due to this the
= 6  3.142  1.8  104  0.05  200 pressure inside the tin does not become less than
P

= 0.034 dyne atmospheric pressure which happens when only


2 2 one hole is made.
r g   
136. v = 9 4. pressure (Pg) = 200 kPa,
et

 P0 = atmospheric pressure = 1.01  105 Pa


2  0.1  10   9.8   8000  1330 
2
2 = 101 kPa
= Absolute pressure (P) = P0 + Pg
9 8.33  101
= 101 + 200 = 301 kPa
rg

 0.01743 m/s
= 17.43  103 m/s 5. Total pressure = Pa + gh
= 1.01  105 + 103  10  10
137. Neglecting buoyancy due to air,
....[ water = 103 kg/m3]
Ta

2r 2g
v= = 2.01  105 Pa  2 atm
9
6. Pressure difference between lungs and
2   2  105   1.2  103  9.8
2

= atmosphere = 760 mm – 750 mm


9  1.8  105 = 10 mm = 1 cm of Hg
= 5.81  102 m/s Also, Pressure difference = 1  13.6  g
v  5.8 cm/s i.e., one can draw from a depth of 13.6 cm of
water.
2r 2g
138.  = (Neglecting density of air) 7. External pressure
9v
= atmospheric pressure + gh
2  10 
5 2
 1000  9.8
= where  is density of water = 1000 kg/m3
9  1.21  102  External pressure = 105 + 1000  10  20
 = 1.8  105 N s/m2 = 105 + 2  105 = 3  105 N/m2
183

MHT-CET Triumph Physics (Hints)

8. Pressure at bottom of the lake = P0 + hg 20. From Notes 2,


FC
Pressure at half the depth of a lake = P0 + h g FA < or FC > 2 FA
2 2
According to given condition, Clearly, the cohesive force dominates.
2
P0 + 1 hg = (P0 + hg) 22. Surface tension of oil is less than that of water.
2 3 So oil spreads on water.
1 1
P0 = hg 23. A membrane has two free surfaces, therefore
3 6
total force acting on each side = T  2L
2P 2×105 T  2L
h = 0 = 3 = 20 m Force per unit length of the frame = = 2T
ρg 10 ×10 L
9. P = P0 + hg 24. In case of a square frame,

ns
= 1.01  105 + 0.20  1000  10 Total length L = 2  4L = 8L
= 1.01  105 + 0.02  105 = 1.03  105 Pa  Force due to surface tension,
 F = PA F = T  L = T  8L = 8TL
 F = 1.03  105  1 = 1.03  105 N F 720

io
25. T= = = 72 dyne/cm
2l 25
10.
Glycerine

Oil h 26. Force required to separate the plates,

at
10 cm 2TA 2  70  103  102
10–h F   28 N
t 0.05  103
A B
2AT 2  8  75
27. F= = = 105 dyne
Mercury
lic 28.
t 0.12  101
The force on disc = T  circumference
At the condition of equilibrium
Pressure at point A = Pressure at point B = 7  10–2  2   r
PA = PB 22
= 7  10–2  2   (20  10–2)
ub
 10  1.3  g = h  0.8  g + (10  h)  13.6  g 7
 h = 9.6 cm = 8.8  102 N

11. Pressure depends on depth alone. 29. F = T  (2R)


F 75  104
 = 12.5  102 m
P

13. Hydraulic brakes work as per Pascal’s law. (2R) = =


T 6  102
Hence change in liquid pressure is transmitted
equally to wheels. 30. F = T  l = 2  2r  T = 0.0616  105 dyne
6160  7
et

5 10  T= dyne cm1
15. r1 = m , r2 = m, 4  22  7
100 100
= 70 dyne cm1
F2 = 1350 kg f = 1350  9.8 N;
31. Force due to S.T. = 2 (2r) T
rg

F1 F2
As,   Force required to lift the ring,
a1 a 2
F = 2 (2r) T
a1 r 2
 F1  F2  12 F2 =22
22 3
  102  0.07
Ta

a2 r1 7 4
 F1 
r12
F 
(5 / 100) 2
 1350  9.8 = 22  3  102  0.01
r12 2
(10 / 100) 2 = 66  104 N
= 1470N 32. F = T  (2 r1 + 2r2)
F F = T  2  (1.75 + 2.25)  102
Pressure, P  1 = 12
a1 r1 = 0.074  2  3.14  4  102
=
1470 = 1.86  10–2 N
(22 / 7)(5 / 100) 2
33. Net force on stick = F1 – F2 = (T1  T2)l
= 1.87  105 Pa = (0.07 – 0.06)  2 = 0.01  2 = 0.02 N
P1  P2 v 2 4.5  105  4  105 v 2 34. Pull due to surface tension = T  2  (l + t)
16.   
g 2g 103  g 2g = 0.07  2(9.8 + 0.2)  102
 v = 10 m/s = 14  103 N
184

Chapter 02: Mechanical Properties of Fluids


35. Refer Shortcut 2 47. W = 8   r22  r12  T
r  r12  r22  9  16  5 cm = 8  3.14  [(6  102)2]  (4  102)2]
36. Refer Shortcut 3  0.035
r1r2 5 4 = 17.58  104  1.8  103 J
r   20 cm
r1  r2 5  4 48. Work done = S.T.  increase in surface area
37. Using Shortcut 2, = 25  103  2  4  [(9  102)2
r2 + r2 = R2 – (6  102)2]
3 4
= 200  10    [45  10 ]
 R = 2 r = 1.4 r
= 9000   107 = 90  105 J
38. Here, Assertion is false but Reason is true. As
work done is, 49. dW = T  8 (R 22  R12 )

ns
W = S.T.  increase in area = T  8(25R2  9R2)
W = T  8 (16 R2) = 128 R2T
or S. T. =
increasein area
50. Work done in blowing a soap bubble of radius R
2  104 is given by, W = 8R2T
= 5  10–2 N/m.

io
=
(10  8  10  4)104 2
 6 102  2
= 8  3.14     2.1  10
39. W = T  Surface area of bubble  2 

at
Since the soap bubble has two surfaces, = 47.4  105 J
W = T  2  4R2 = 8R2T
40. W = 2  4R2   ; R is increased by a factor of 51. Since conditions are isothermal, therefore,
2, so W is increased by a factor of 4. energy will be conserved.

41. Increase in surface area = n  4r2  4R2


lic  2[2  4r2T] = 2  4R2T
R2 = 2r2
Required energy is equal to the product of  R = 21/2 r
surface tension and increase in surface area.
= (4nr2  4R2)  T 52. Work done in increasing the radius of soap
ub
bubble is W = 8T[r22 – r12] = 8T (4r2 – r2)
Work done = 24r2T
42. T=
Change in area
53. Work done = surface tension  change in
3  104
 = 3  10–2 N/m
P

T= surface area
2  (10  11  10  6)  104
= T  (2A  A)
43. Effective area = 2  0.02 m2 = 0.04 m2 =TA
Surface energy, TA = 5 N m1  0.04 m2 = 3  103  1.3  104
et

= 2  101 J = 3.9  107 J


44. Due to volume conservation, 54. Area of film = 2 (2  103) = 4  103 m2
rg

4 4 W = TA
2 R3 = R3  r = 21/3 R
3 3 = 40  103  (4  103)
Surface energy  Area = 16  105 J
Surface energy  before 2A1 2R 2
Ta

   2 55. Work done = T  A


 Surface energy  after A2 r
= 0.072  [(20  0.2  104)
=
2R 2
= 21/3 – (20  0.1  104)]
22 / 3  R 2 = 0.072  0.1  20  104
45. As volume remains constant, = 0.072  2  104
R3 = 8000r3  R = 20r = 1.44  105 J
Surface energy of one big drop 4R 2T 56. Initial surface area = 2  length  separation
 
Surface energy of 8000 small drop 8000 4r 2T = 2  10  0.5
R2  20r   1
2 = 10 cm2 = 10  10–4 m2
  Final surface area
8000r 2 8000r 2 20
= 2  10  (0.5 + 0.1)  104 = 12  10–4 m2
46. W = 8r2T = 8  3.14  (5  102)2  30  102 Work done = W = T  A
= 1.88  102 J = 0.070  [12  10–4 – 10  10–4] = 14  10–6 J
185

MHT-CET Triumph Physics (Hints)

57. Area of film = 2 (10  102  5  102) u = 1000  T  4r2


= (50  104 m2)  2 = 1000  T  4
R2
W = TA 100
= 0.035  (50  104)  2 = 10  4R2T = 10 u
= 0.035  100  104 4
= 0.035  102 = 3.5  104 J 65. Volume of small droplet = r3
3
58. Let r = radius of each small drop and 4
Volume of big drop = R3
R = radius of a big single drop. 3
4 4 Due to volume conservation,
Then n   r3 =  R3
3 3 4 4 
R3 = 64   r 3 
 R=n r 1/3 3  3 
Initial surface energy = E1 = n  4  r2  T = n E  R3 = (4)3r3  R = 4 r

ns
Final surface energy R 1
 r= = = 0.25 mm
= E2 = 4  R2  T = 4  r2n2/3  T = n2/3 E 4 4
Energy released = E1 – E2 = E (n – n2/3) Work done = T  A = T [n4r2 – 4R2]

io
= 4T [nr2  R2]
59. W  r2 = 4  72  103 [64  (0.25  103)2  (103)2]
 W1  r12 and W2  r22 = 288  103 [4  106  106]
= 2.7  106 J

at
2 2
W1  r1   4 
       = 16 : 9
W2  r2   3  66. As volume is conserved,
60. Wr 2 4 4
R3 = n r3
 W1  R2 and W2  (3R)2
lic 3
R3
3
 0.5  102 
3
W R2  n= = = (5)3 = 125
 1  2 =1:9 3 
W2 9R r 3
 1  10 
 R3 = 125r3  R = 5r
ub
4
61. V = r3  V  r3  r  V1/3 W = n4r2T  4R2T
3
= n4r2T  4(25r2)T
Now, = 4r2T (125  25)
W = 4  r2 T  W  r2  V2/3 22
= 400   10–6  7  10–2
P

2 2 /3
W  r   2V  2/3 1/3 7
 =  =   = (2) =4
W r  V  = 88  10–6
 W = 41/3 W  W = 8.8  105 J
et

4 3 4 67. Let r be the radius of each droplet and R be the


62. 2 r = R3 or R = 21/3r radius of the big drop.
3 3
Since the total volume is the same, we have
Final surface area = 4R2 = 422/3 r2
rg

4r 3 4R 3
Initial surface area = 2  4r2 106  =
3 3
 Energy released = [8r2 – 4  22/3 r2]T
 R3 = 106 r3  R = 100 r
63. Surface energy = surface tension  The surface energy of one million drops,
Ta

 surface area E1 = 4r2T  106


E = T  2A The surface energy of one big drop,
 New surface energy, E1 = T  2(A/2) = T  A E2 = 4R2T
2 2
E  E1 E2 R 1  100r  1 1
 % decrease in surface energy =  100  =   6 =   6 = 2
E E1 r 10  r  10 10
2TA  TA
=  100 = 50% 4 4
2TA 68.  R3 = 8  r3
3 3
64. u = T  4  R2  R3 = 8r3  R = 2r
When drop is sprayed into 1000 droplets each of Work done = T (n  4 r2 – 4R2)
radius r, then R2
= T (8  4  – 4R2)
4 4 R 4
R3 = 1000  r3  r =
3 3 10 = T 4 (2R2 – R2) = 4R2T

186

Chapter 02: Mechanical Properties of Fluids


4 4 3
 r1  8 r1 2
69. R3 = n r3  R3 = nr3 
3 3     
r
 2 1 r2 1
 R = n1/3 r  1.4 = 5r
1
1.4 But P 
 r= = 0.28 mm r
5
P1 r2 1
Change in energy = T  A  = =
P2 r1 2
= 75  [n4r2 – 4R2]
= 75  4   [125 (0.28  10–1)2 – (1.4  10–1)2] 1 1
= 300  3.14 [5(1.4  10–1)2 – (1.4  10–1)2] 78. P = T   
= 300  3.14  4  1.96  10–2  r1 r2 

= 9.42  7.84  74 erg As r1 = r and r2 = ,


T
70. T1 + T cos ( – ) = T2 P = But r = d/2

ns
r
T2  T1 2T
 cos ( – ) =  P =
T T d
T2  T1 2T 2  75  10
 – cos  =  F = P.A = A=
T  

io
T1 d 0.01
T1  T2 T2 = 150  103 dyne
 cos  =
T = 150 gm-wt

at
1 P r r 1 79. The length of the water column will be equal to
73. P   1  2  
r P2 r1 4r 4 full length of capillary tube.
4T 1 80. In an artificial satellite, there is a state of
 P 
74. P 
r r
lic weightlessness. So, water will rise up to full length
of tube and will form a new surface of higher
Further, as radius of soap bubble increases with
1 radius of curvature but will not come out.
time, P 
t 2T cos 
ub
81. From h = , the rise in capillary depends
75. P1 = 4 P2 rg
4T 4T upon the surface tension of the liquid and
 =4  r2 = 4r1 surface tension of soap water solution is less
r1 r2
than water. Hence, height will be less in second
P

4
 V = r3  V  r3 case. Also, as the soap solution wets the surface
3 of capillary in contact, the shape of meniscus
3 3
V1 r  1 1 will be concave.
 =  1 =   =
et

V2  r2  4 64
2Tcos 
82. h=
4T 4T rg
76. P1 = , P2 =  P1 = 2P2
r1 r2 2T cos 
 hg =
rg

1 2 r 1 r
 =  1 =
r1 r2 r2 2 2T cos  1
83. h= h 
4 4 rg r
Now, V1 =  r13, V2 =  r23
Ta

3 3
84.
 V1 = nV2
4 4 R
 r13 = n r23  r13 = n r23 
3 3 r
3 3
r  1 1 
 n =  1  =   = = 0.125
 r2  2 8

V1 8
77. 
V2 1
4 3
 r1 
 3  = 8
4 3 1 r
 r2  From figure, R =
3  cos

187

MHT-CET Triumph Physics (Hints)


1 2T
85. h 95. h=
r rg
r1h1 r1  1.2
 r1h1  r2h 2  h 2    2.4 mm hm g  ge 
 r1   = e =6 ....  g m 
r2
  he gm  6 
2
 hm = 6 he = 6 h
2T cos  1
86. h= h 2Tcos 1 2Tcos0
rg r 96. h= = =4
rg rg
hA r r 1
 = B = B = 2T
hB rA 2rB 2  =4
rg
2Tcos  2Tcos 2
87. h=  =2
rg rg

ns
1 1
Here, h   h1r1 = h2r2  4  cos 2 = 2  cos 2 =
r 2
hr 4 2  2 = 60
 r2 = 1 1 = = 1 cm

io
h2 8
2Tcos  hrg
97. h= T=
88. l cos 60 = 2 or l = 2  2 cm = 4 cm rg 2cos 

at
h Tl  h
89. l =  = l  l
sin (90  ) Tw w hw
h 6 12 850
= = = = 4 3 cm =  3.0 = 2.55
1000
sin 60 3/2 3
lic  Tl = 7.0  102  2.55 = 0.18 N/m
r1 h 2 6.6 3
90.   
r2 h1 2.2 1 2T cos   1 1 
98. h2 – h1 =   
g  r2 r1 
ub
2T
91. h= 4T cos   1 1 
rg =   
g  D 2 D1 
2T
 r = (where r = radius of curvature) 4  7  10 2  cos 00
hg 1 1 
= 3  6 
P

2  547 103  10  10 3  
=
1.356  13.59  980 28  10 2  1 1 
= 0.06 cm
=
10 3  6 m
 
et

2T cos  = 4.66  10–3 m


92. Rise in capillary = h = = 4.66 mm
rg
As angle of contact  = 0  cos  = 1 and 
rg

102. vC = Rn
 = 1 g/cc D
2T 2  70 For laminar flow, Reynold’s number Rn = 200
 h= =
rg (1/ 42)  1 980 200  103
Ta

 vC =
140  42 103  2  102
 h=  h = 6 cm
980 = 0.01 m s1
2T cos  2  0.072  cos0 vρD
93. h= = 103. Reynold’s number Rn = ….(i)
rg 0.024  102  1000  10 η
= 6 cm ….[ cos 0 = 1] where v is the speed of flow.
Rate of flow of water Q = Area of cross section
2T cos  hrg  speed of flow
94. h= T=
rg 2cos  πD 2 4Q
Tw h cos m w Q=  v or v =
 = w   4 πD2
Tm hm cos w m Substituting the value of v in equation (i),
10 cos 135 1 1 4QρD 4Qρ
=   = Rn = =
3.42 cos 0 13.6 6.5 πD 2 η πDη

188

Chapter 02: Mechanical Properties of Fluids


Substituting the values, 121. Terminal speed v  r2
5
4  5  10  10 3
v1 r2 r2
Rn = = 5100  = 2 = 1/3 2
 22  v2 R (2 r)
   1.25  10 2
 103

 7  (21/3 r)2
For NR > 2000, the flow is turbulent.  v2 = v1 = 5  22/3 = 5  41/3 cm s1
r2
Hence, the flow of water is turbulent with
Reynold’s number 5100. 122. In the first 100 m, body starts from rest and its
velocity goes on increasing and after 100 m it
104. From kinetic theory point of view viscosity acquires maximum velocity (terminal velocity).
represents transport of momentum. Further, air friction i.e., viscous force which is
111. With rise in temperature, viscosity of liquid proportional to velocity is low in the beginning
decreases while viscosity of gases increases. and maximum at v = vT.
Hence, work done against air friction in the first

ns
112. F  r3  V 100 m is less than the work done in next 100 m.
As volume becomes doubled, F changes to 2F.
123. According to the equation of continuity,
115. Terminal velocity is caused due to viscosity, Av = constant
which is absent in vacuum. The speed of still water is very small and hence

io
area will be large. This makes the still water run
117. Volume of big drop = 2 (Volume of small drop) deep.
4 4
 r23 = 2   r13

at
3 3 124. In steady flow of incompressible liquid rate of
flow remains constant i.e., V = av = constant.
 r2 = 2 r1 1/3
This is equation of continuity.
Also v1  r12 , v2  r22 When pipe is placed vertically upward velocity of

v2 r2
= 22
lic flow decreases with height so area of cross
section increases and when pipe is placed
v1 r1
2
vertically downward, velocity of flow increases
r2 23 r 2 in downward direction so area of cross section
 v2 = 22  v1 = 21  0.15
ub
r1 r1 decreases i.e., it becomes narrower.
v2 = 0.15  22/3 cm/s 125. The equation of continuity is derived on the
basis of the principle of conservation of mass
4 4
118. R3 = 8  r3 and it is true in every case, whether tube is kept
P

3 3 horizontal or vertical.
 R = 2r
126. If velocities of water at entry and exit points are v1
Terminal velocity v  r2
and v2, then according to equation of continuity,
v1 r2
et

  2 v A r  2 4
2 2
v2 R A1v1  A2v2  1  2   2     
v 2 A1  r1   3  9
10 r2
 
v 2  2r 2 127. If the liquid is incompressible then mass of
rg

 v2 = 40 cm/s liquid entering through left end, should be equal


to mass of liquid coming out from the right end.
2r 2  M = m1 + m2
119. Using v = (  ), v  (  )
Ta

9η  Av = A1v1 + A2v2
vgold 19.5  1.5 18  24  10 = 12  6 + 8  v2
= = =2 240  72
vsilver 10.5  1.5 9  v2 = = 21 m/s
8
v gold 0.2
 vsilver = = = 0.1 m s1 129. As air under the pan is blown, pressure below
2 2
the pan decreases. This as per Bernoulli’s
120. v  r2  (neglecting density of liquid) theorem causes downward motion.
where  = density of material of sphere. 130. According to Bernoulli’s theorem, when wind
4 ρ r3 velocity over the wings is larger than the wind
Now,  r 13 1 = 4 r 32 2  1 = 23
3 3 ρ2 r1 velocity under the wings, pressure of wind over
3 the wings becomes less than the pressure of
v1 r2 ρ r2 r r
 = 12  1 = 12  23 = 2 wind under the wings. This provides the
v2 r2 p2 r2 r1 r1 necessary lift to the aeroplane.
189

MHT-CET Triumph Physics (Hints)


131. According to Bernoulli’s theorem, when 4.5105 4 105 1v 2
velocity of liquid flow increases, pressure +0= +
g g 2g
decreases and vice-versa. When two boats move
105 105
parallel to each other, close to one another, the v22 = = 3
stream of water between the boats is set into  10
vigorous motion. As a result, the pressure v2 = 10 m/s
exerted by the water in between the boats 140. Let A = cross-section of tank
becomes less than the pressure of water outside a = cross-section hole
the boats. Due to this pressure difference, the V = velocity with which level decreases
boats are pulled towards each other. v = velocity of efflux i.e., velocity with which the
133. P + 1gh1 + 2gh2 liquid flows out of orifice (i.e., a narrow hole)
h = h1 + h2 = height of free surface above hole A
While at hole, horizontal velocity will be zero

ns
1
P + 1 gh1 + 2 gh2 = P + 1v2 3m
2 B v
52.5cm
 h  h    
 v= 2g  1 1 2 2  = 2g  h1  h 2 2 

io
 1   1 
From equation of continuity av = AV
1 1 av
134. P1 +  v12 = P2 +  v 22 V=
2 2 A

at
2  P1  P2  2 By using Bernoulli's theorem for energy per unit
 =v
 volume
Energy per unit volume at point A = Energy per
2  P1  P2  2  3.5  3  105
 unit volume at point B
v=

=
103
lic
= 10 m/s
1
P + gh + V2 = P + 0 + v2
1
136. According to Bernoulli's theorem, 2 2
2 2gh 2  10  (3  0.525)
v2 (2.45)2 v =  = 50 m2/s2
h= h= = 0.300 = 30.0 cm 2

ub
2
a 1 (0.1)
2g 2  10 1  
A
 Height of jet coming from orifice
= 30.0  10.6 = 19.4 cm 141. Horizontal range will be maximum when
H 90
137. The height of water in the tank becomes h= = …(Using Shortcut 15)
P

2 2
maximum when the volume of water flowing
= 45 cm i.e., hole 3.
into the tank per second becomes equal to the
volume flowing out per second. 142. The pressures are
et

Volume of water flowing out per second 2T 2T


Patm  , Patm + , Patm respectively.
= Av = A 2gh .…(i) r R
Volume of water flowing in per second 143. Below the surface of the earth, pressure
= 70 cm3/s
rg

.…(ii) increases with increase in depth. Hence pressure


From (i) and (ii) in the mine is higher than atmospheric pressure.
A 2gh = 70 The acceleration due to gravity below the
 1  2gh = 70  1  2  980  h = 70 surface of the earth decreases uniformly with
Ta

the distance from the centre, as shown in the


4900
 h= = 2.5 cm. figure below.
1960
g
1 2 1
138. P + v = P +   4v2
2 2 g
ρ 2
P = P + v (1  4)
2 d
3 o R r
P = P  v2
2
144. P1V1 = P2V2
139. Using Bernoulli’s theorem, 4 4
P1 v12 P v2
 (H  h)g  r 3  Hg  (2r)3
+ = 2 + 2 3 3
g 2g g 2g  H + h = 8H  h = 7H

190

Chapter 02: Mechanical Properties of Fluids


2T 4S
145. = hg  4T/ 2R = hg 150. Excess pressure inside the soap bubble =
R r
4T Hence the pressure inside the soap bubble
 = hg
D 4S
= Patm +
4T 4  0.07 1 r
 D= =   10–3 m
hg 0.40  10  9.8 14
3
From ideal gas equation, PV = nRT
1  4S  4
 D= mm 8   (rA )
3

14 PA VA n A  rA  3 n
   A
PBVB n B  4S  4 nB
146. W1 = 8R2T1 8   (rB )
3

W2 = 8(2R)2T2 = 32R2T2  rB  3
W1 T Substituting S = 0.04 N/m, rA = 2 cm,
  1

ns
W2 4T2 nA 1
rB = 4 cm we get, 
When T1 = T2, W2 = 4W1 nB 6
When temperature increases, surface tension nB
decreases.  = 6.
nA

io
 W2 < 4W1
151. Outside pressure = 1 atm
147. T T Pressure inside first bubble = 1.01 atm
  Pressure inside second bubble = 1.02 atm

at
 Excess pressures will be
P1  1.01  1  0.01 atm and
P2  1.02  1  0.02 atm
lic
Here, Weight of metal disc = total upward force 1 1
Now, P   r 
= upthrust force + force due to surface tension r P
= weight of displaced water + T cos  (2 r) r1 P2 0.02 2
   
ub
= W + 2 rT cos  r2 P1 0.01 1
2 4
148. h = Now, V  r 3
dgr 3
where, h = rise of liquid in capillary tube  V  r3
P

Work done by surface tension 3


V1  r1   2  8
3

 2  4 2      
= Fh = (2)   = . V2  r2   1  1
 dgr  dg
et

Hence option (A) is correct. 4T


152. Pressure inside tube = P = P0 +
h    r
P.E. = mg   = (dr2hg)   Let hemispherical radius be r1 and
2  dgr 
sub-hemispherical radius be r2
rg

 2     2 2
= r2dg       = Hence pressure on side 1 will be greater than
 dgr   dgr  dg side 2. So, air from end 1 flows towards end 2.
Hence option (C) is correct.
153. Velocity of ball when it strikes the water surface
Ta

2 2
Remaining energy is liberated as heat. v = 2gh .…(i)
dg
Hence option (D) is correct. Terminal velocity of ball inside the water
2 2    1
2Tcos  v= rg .…(ii)
149. Using, h = , 9 
rdg
Equating (i) and (ii)
Mass of the water in the first tube,
2 r 2g
2  2T cos  
2 r2Tcos  2gh = (  1)
m = r hd = r   d = 9 
 rdg  g 2
2 4   1 
mr  h= r g 
m r 2r 81   
   2
m r r 154. A part of pressure energy is dissipated in doing
 m = 2m = 2  5g = 10 g work against friction.
191

MHT-CET Triumph Physics (Hints)


156. Area of each wing = 20 m2 ave
V
1 5 A
Speed, v1 = 216 km h = 216  = 60 m s1
18 Now applying Bernoulli’s theorem,
5 1  av e 
2
1
Speed, v2 = 180 km h1 = 180 = 50 m s1 0 + Hg +  0  v e2
18 2  A   2
Let P1 and P2 be the pressures of air at the upper
 1  av   1
2
and lower wings of plane respectively, then    Hg   e    v e2
P1 1 2 P2 1 2  2  A   2
 v1   v2
 2  2
2
1  av e  1 2
 Hg +  ve
1 1 2  A  2
 P1  P2 =   v12  v 22  = 1 (602  502 )
2 2  a 
2

= 550 Pa 2Hg = ve2 1   


  A  

ns
(Air density,  = 1 kg m3)
2Hg 2   4  0.6  10
pressure 
Force ve2 =  = 71 m2/s2.
1   0.2 
2 2
area a
1  
 Force = pressure  area A

io
ma = 550  20 (m = mass of a wing)  ve = 71 = 8.4 m/s
11000
m= = 1100 kg ( a = g) 163. v1 = 10 m/s, a1 = 8 cm2 = 8  104 m2,
10

at
Assuming mass of the plane is mostly due to its a2 = 16 cm2 = 16  104 m2, P1 = 1  105 N/m2,
wings, P2 =   105 Nm/s2
Mass of plane = 2m = 1100  2 = 2200 kg. a1v1 = a2v2
a1v1 8  104  10
lic
158. A streamlined body offers less resistance to air.  v2 =
a2

16  104
= 5 m/s
159. Weight of the ball 1 2 1
= Buoyant force + Viscous force P1 + v1  P2  v22
2 2
V1g = V2g + kv2
ub
1
 kv2 = V(1  2)g  P2 = P1 +  v12  v22 
2
Vg(1 2 ) 1
v=  P2 = 105 +  103 (102  52)
k 2
 P2 = 1.37  105 N/m2
P

160. Pressure at the bottom of tank


P = hg = 3  105
Pressure due to liquid column Competitive Thinking
Pl = 3  105 – 1  105 = 2  105
et

and velocity of water v = 2gh 1. P = gh


Hence, pressure is independent of area of liquid
2Pl 2  2  105 surface.
 v= = = 400 m/s
rg

ρ 103
P hg  Pa (10  103  10)  1  105
3.   =2
161. Time taken by water to reach the bottom, Pa Pa 1  105
2(H  D)
t= and
Ta

4. The system is in equilibrium and pressure on


g
both sides is equal.
velocity of water coming out of hole, This means,
v = 2gD hww g = hoo g
 Horizontal distance covered, h ww 130  10 3  103
 o = = = 928.6 kg/m3
2(H  D) ho 140  10 3
x =vt= 2gD  = 2 D(H  D)
g
5. The pressure of lowest point in water
162. Let ‘A’ be the area of cross-section of the tank, = The pressure at lowest point in oil.
‘a’ be the area of hole, ‘ve’ be the velocity of Pa + hwater g = Pa + hoil g
efflux. ‘V’ be the speed with which level  Pa + 0.15  103  g = Pa + 0.20  oil  g
decreases. 0.15  103
So according to equation of continuity  oil =
0.20
ave = AV [i.e., area (a)  velocity (v) = constant] = 0.75  103 = 750 kg/m3
192

Chapter 02: Mechanical Properties of Fluids


6. Pressure difference between sea level and on the  R = 8r
hill measured by mercury barometer will be, 512  4πR 2 
(h1  h2)  Hg  g = h  air  g  A2 =
64
 (76  h2)  Hg = 3  105  air  A2 = 8 (4R2)
 (76  h2)  104 = 3  105 Surface energy  Area
E 2 A 2 8  4πR 
2
 76  h2 = 30
 = 
 h2 = 46 cm on Hg E1 A1 4πR 2
F mg  E2 = 8E1 = 8E ... { E1 = E}
7. T= 
2l 2l
2Tl 15. Let R be the radius of bigger drop and r be the
 m= radius of single small water drop.
g

ns
Volume of big drop = n(Volume of small drop)
Fflat T  2r 2 4 4
8.    R3 = n  r3
Fcurved T  r  3 3
 R3 = nr3
9. 2Tl = mg

io
1
mg 1.5  102 1.5 R = n3 r
 T= = 2
= = 0.025 N/m
2l 2  30  10 600 Surface energy of n drops (En) = n  4r2  T
Surface energy of big drop (E) = 4R2T

at
10.
1
E n nr 2 nr 2 nr 2
  2  2
 2
 n 3
= 3
n:1
E R  13  2
n r n 3
r
lic  

16. n = 1000, R = 1 cm,


For wire to float into water, its weight should be By applying conservation of volume
balanced by the surface tension of the water. initial volume = final volume
ub
 mg = Tl ....(where, l = length of the wire) (1)3 = nr3
 Vg = Tl 1 1
r= =
 r2lg = Tl n1/3
(1000)1/3
T 1
P

 r2 = r= cm
g 10
T r = 0.1 cm
 r=
g r = 0.001 meter
et

Gain in surface energy


12. W = 4r2T (n  n2/3) = TS
 2  10 
3 2
= 0.075 {4 [1000  (0.001)2  (0.01)2]}
 
 0.072 1000  103 
2/3
W = 4 
rg

4   Gain in surface energy = 8.5  104 J


W = 8.146  10–4 J 17. E = 2(T  A)
13. The rectangular film of liquid has two surfaces. = 2T  4r22  4r12 
Ta

Hence, the increase in surface area is,


22
A = [(5  4) cm2  (4  2) cm2]  2 = 2  0.035  4 
7
= (20  8)  2 cm2 = 24  104m2
  6 102    4 102  
2 2
Also,  
W = T.A
22
W 3  104 = 2  0.035  4   (36 – 16)  10–4
 T= = = 0.125 Nm1 7
A 24  104
= 1.76  10–3 J
2
14. Surface area of drop, A1 = 4R
18. W = TA
Surface area of 512 droplets, A2 = 512 (4r2)
= 0.03 [ 2  4  (52 – 32)  104]
 volume of drop = n  (volume of droplet)
= 24 (16)  106
4 3 4 = 0.384   103 J
 πR = 512 × πr 3
3 3  0.4 mJ
193

MHT-CET Triumph Physics (Hints)


20. The pressure inside the bubble is equal to the cos 1 > cos 2 > cos 3
pressure at a point Z0 below the free surface of  1 < 2 < 3
water As the liquids rise in capillary tube,
4T
 = gZ0 
R <
2
4T 4  2.5  10 2
 Z0 =  3 = 1 cm 
R g 10  103  10  0  1 < 2 < 3 <
2
21. P1 = pressure difference between smaller
25. Rise of water in capillary tube is given by
bubble and larger bubble
2Tcosθ
P2 = pressure difference between inside and h=
outside the larger bubble Rρg
4T 4T For water, cos = 1
Now, P1 = , P2 =

ns
R1 R2 Also, the radius of capillary tube becomes
4T (R – r) after inserting wire of radius r.
As required pressure difference P = 2T
R  h=
P = P1 + P2 R  r  ρg

io
4T 4T 4T
   26. In a freely falling platform a body experiences
R R1 R 2
weightlessness, So,the liquid will rise upto to a
R1R 2

at
R= length of the capillary i.e., 20 cm.
R1  R 2
27. The angle of contact is given by,
2  1  104
= = 6.67  10–3 m ghr
 2  1  102 cos  =

22. Height of water column > length of tube.


lic 2T
 = density of water
So liquid will rise to the top of capillary tube but h = height of water in capillary
will not overflow. r = radius of capillary
ub
2T cos  T = surface tension of water
23. h= 1000  10  5  102  0.2  103
rg  cos  =
1 2  7  102
 h 5 5
r  cos  =   = cos–1  
P

 h1r1 = h2r2 7 7 
h1 r2
  ….(i) 28. h1r1 = h2r2
h 2 r1
et

h1r1 1.8  r
Also, area A = r2  h2 = = = 2 cm
r2 0.9r
 r A
r2 A2 R n  3000  10 3
rg

  ….(ii) 29. vc = = = 0.15 m/s


r1 A1 D 103  0.02
From equations (i) and (ii), R n 2  103  6  103  101
h1 A2 A/9 1 30. vc = = = 0.33 m/s
Ta

   D 720  5  103
h2 A1 A 3
Flow becomes turbulent, if the velocity is above
 h2 = 3h1 = 3h 0.33 m/s.
24. Rise in capillary tube, 31. Flow of water (Q) = 100 liters per minutes
2T.cos  100  103
h= =
r g 60
Given that, h, T, r and g are constant.
= 5 × 10–3 m3/s
cos  3
 = constant
 5  103
Velocity of water, v = Q =
cos 1 cos2 cos 3 3    5  102 
2
i.e. = = A
1 2 3
as 1 > 2 > 3  v = 2 = 0.2 m/s
3

194

Chapter 02: Mechanical Properties of Fluids


Reynold’s number (Rn) 2 r 2 (  )g
Terminal velocity, v =
=
vD
=
0.2  1000  10  10  2
 ≈ 2 × 104 9 
 1  103  v  r2
order of (Rn) = 104
2
 vT1 r 
 =  1
dv vT2  r2 
34. F=A 2
dx v r  1 1
2

F dv =  1  or    [Using (i)]
 shearing stress =  nv r
 2 n  2
A dx
n=4
5
9
2 2 r2
 shearing stress = 10  18 = 0.25  103 N/m2 41. Terminal velocity, vT = (  )g
10 9 

ns
2  102 2  12 2
35. F = A
dv
= 0.9  500  10–4  = 1.8 N  vT1 = (1  0.01 2)g = (82  0.1 2)g
9 9
dx 0.5  103
2
36. Fv  vT2 = (7.9)g ….(i)
9

io
2  22
vT2 = (2  0.12)g
9
8

at
 vT2 = (0.9)g ….(ii)
9
Fv = mg vT1 7.9
4
mg  = = 79
 6rv = r3g vT2 4  0.9  36
3
4r 2g 4  12  1.75  980
lic 42. A1v1 = A2v2
= = = 1089 poise 2
3 6 v 3  6  0.35 v1 A d   10 
2

 = 2 =  2 =   =4:1
v2 A1  d1   5 
37.
ub
6πηrv 43.
R n holes

Using equation of continuity,


P

av = constant
VR 2
R2V = nr2v  v =
nr 2
et

mg 44. v2 = v12  2gh = (0.4) 2  2 10  0.2 = 2 m/s


4 3 A1v1 = A2v2
6πηrv = πr g 2
3  810 3  d2
rg

2 2    0.4 =   2
 v= r g  2  4
9
 d  3.6  103 m
6
2  0.9  10  10  9.8
2 3
 v= = 98 ms–1
Ta

45. Volume of water flowing per second = Av


9  1.8  10 5
= A 2gh
38. When the net force is zero, body moves with
terminal velocity. = (2 × 10–6) 2  10  2

39. For a given material, terminal velocity is = 2 × 2 × 10 × 10–6


independent of mass of the body but depends on = 12.6  10–6 m3/s
density of the material. 46. According to Bernoulli’s principle,
4
40. Mass = Volume  Density  M = r 3   P=
F 1 2
 v
3 A 2
As the density remains constant 1 1
 M  r3  F = v2A =  1.2  (40)2  250
2 2
1/ 3 1/3
r1  M1   M  1 = 2.4  105 N
   =  = ….(i)
r2  M 2   8M  2 Also, net force acting on the roof is upward.

195

MHT-CET Triumph Physics (Hints)

47. v= 2gh  2  10  2  40 cm/s 2Twater


 Mg =  r2g
2H 28 rg
t=  s
g 10 = 2r Twater
= 2  3.142  0.1  10–3  0.07
28
 distance travelled = 40  = 8 cm = 4.4  10–5 N
10
= 44 N
48. Using equation of continuity,
52. F = 105 dyne = 105  10–5 N,
av = AV ….(i)
T = 7  10–2 N/m
where, V is velocity with which liquid level
Now the force due to surface tension on the
decreases and v is velocity of efflux.
circular cross-section of capillary with inner
radius r will be,
R F = 2rT

ns
1.25
Q F 105  105
 2r = =
0.75 T 7  102
= 15  10–3 m = 1.5 cm
According to Bernoulli’s theorem,

io
Energy per unit Energy per unit 53. Energy released = (Af – Ai)T
=
volume at point R volume at point Q 3 R 3 3V
 Af = 4R2 = 4  and

at
1 1 3 R R
P + gh + V2 = P + 0 + v2
2 2 V 3V
 Ai = n  4r2 = 4r 2 =
av 4 3 r
But, V = ….[from equaion (i)] r
A 3

 gh +
1  av  1
2

   = v2
lic  Energy released = T (Ai  Af)
2 A 2 1 1
= 3VT   
2gh 2  9.8  1.25 r R
 v2 = = = 30.625
 
ub
1  (a / A) 2
1  0.2
2
54. Excess pressure inside soap bubble is given as
4T
 v = 5.53 m/s  5.5 m/s Pi  Po = ;
r
49. For maximum range, height of the hole Pi = Pressure inside soap bubble
P

h Po = Pressure outside soap bubble


Total height h
= = 2 = 3h Let excess pressure inside for 1st bubble and 2nd
2 2 4 bubble be P1 and P2 respectively.
…(Using Shortcut 15) 4T 4T
et

From PQ level, hole number 2 is at height of  P1 = , P2 =


r1 r2
3h
. P1 r2
4  
P2 r1
rg

50. Water cannot penetrate till hydrostatic pressure r1 P2 1


> excess pressure due to surface tension.   
r2 P1 3
At limiting condition when water is just about to
penetrate, the two pressures are equal. As volume  radius3
Ta

3
hg = 2T/r V1  1 
  
2T 2  0.07 V2  3 
 h= =
gr 103 10 104 V1 1
 
 h = 0.14 m = 14 cm V2 27

rhg 4 3 4
51. Twater = 55. As, b  N  a 3
2 3 3
(Assuming water is pure and angle of contact  b3 = Na3
zero) Energy released,
2Twater U = T  4a2  N  T  4b2
 h= ....(i)
rg b3
2
= T  4  T  4b2
Weight of water = Mg = r hg a
Substituting for h ....[From (i)] This energy is converted into K.E.

196

Chapter 02: Mechanical Properties of Fluids


1 2 1 1  1 2 1
 mv  T  4b3    60. P1 + v1 + gh1 = P2 + v22 + gh2
2 a b 2 2
1 1
Patm +   0  + gh1 = Patm + v22 + gh2
2
1 4 3 2 31 1
  b  v = T  4b    2 2
2 3 a b
1/ 2
[ speed of water of top = 0]
 6T  1 1  
v =     1 2
   a b   g(h1  h2) = v2
2
56. In equilibrium,  v2 = 2g  h1  h 2  = 2gD
For air inside capillary,
= 2   9.8   0.2 = 1.98 m/s
P0(lA) = P(l – x)A
Where, P is pressure in capillary after being Q = A2v2 = (6  104)(1.98)
= 1.19  103  1.2  103 m3/s

ns
submerged into water.
P0l
 P = 61. Velocity of efflux for A: v1 = 2gh
l x
Now since level of water inside capillary Velocity of efflux for B: v2 = 2g  3h = 6gh

io
coincides with outside, the excess pressure, Water flowing out from A = Water flowing out
2 from B. ….(Given)
P = P – P0 =
r  v1A1 = v2A2

at
P0l 2 Since, Area of square (A1) = L2
  P0 
lx r
Area of circle (A2) = r2
Solving above equation, we get, 2 2
 2gh × L = 6gh × r
x=

l
P0 r 
lic 6gh
1    L2 = × r2 = 3r 2
 2  2gh
1 1 1 1
[vc] = [xyrz] L = 3  2 r = r    2  3 4
ub
57. 4

[M0 L1T–1] = [M1L1T–1]x [M1L3]y[L1]z


[M0L1T–1] = [Mx + y] [L–x 3y + z][T–x] 62. Velocity of efflux when the hole is at depth h,
Comparing both sides, v = 2gh
Rate of flow of water from square hole
P

x + y = 0, –x – 3y + z = 1, x = 1
 x = 1, y = 1, z = 1 Q1  a1 v1 = L2 2gy
Rate of flow of water from circular hole
58. Three vessels have same base area and equal
Q2 = a2v2 = R2 2g(4y)
et

volumes of liquid are added in them.


Considering the geometry of vessles, liquid in According to problem Q1 = Q2
vessle ‘C’ will rise to maximum height amongst  L2 2gy  R 2 2g(4y)
the three.
rg

L
R=
Force on base, F  Pressure exerted on base, P 2
 height of liquid (h)
Hence, the force on the base will be maximum 63. Power of heart = F  velocity
Ta

at vessel C. F
=  Area  velocity
Area
59. From Bernoulli’s theorem,  area  displacement 
v12 v2 = Pressure   
P1 + + gh1 = P2 + 2 + gh2  time 
2 2
volume
Here h1  h2 = Pressure 
time
 2
 P1 – P2 =
2
 v2  v12  = 0.6 (702 – 602) = 780 Pa = P.
dV
= hg 
dV
dt dt
This pressure difference causes uplift of plane 5  10 3
 Net force = upward force  downward force = (0.15)  (13.6  103) (10) 
60
= (P1  P2)  area  weight 13.6  5  0.15
=
= 780  14  1000  10 ( weight = mg) 6
= 920 N (in upward direction) = 1.70 watt

197

MHT-CET Triumph Physics (Hints)

Hints to Evaluation Test

4 3 1 4 3 10. This is same as saying the there is no gravity in


1. r g = 2rT +  r g
3 2 3 space as the weight will cancel the pseudo force
4 4 3  1 of the lift. Thus the force of surface tension will
 2rT = r3g   r g   take it to the maximum possible height.
3 3  2
4 2   11. Using Shortcut 2,
2T = r g   
3  2 R = r12  r2 2
2T
 r2 = h1r1
4   12. h1r1 = h2r2 or h2 = ….(i)

ns
g    r2
3  2
A1 r12 A
2 3T 3T Here  where A1 = A and A2 =
 r = r= A 2 r2 2 16
g(2  ) g(2  )
r12 16

io
r
2. Air flows from high pressure to low pressure  2
=  1 =4 .…(ii)
r2 1 r2
region. Thus the smaller bubble will be
 From (i) and (ii), h2 = 5  4 = 20 cm
engulfed.

at
4 4 4
3. Balancing forces on the edge, 13. R3 = 64   r3 = (4r)3
3 3 3
(Tcos) 2r = mg
 R = 4r
0.157  10  103
 r=
2  3.14  0.075  1
m
lic S1 = 64  4r2  T and S2 = 4R2T
2
= 3.3 mm S1 64  4r 2  T  r  64
   64    =4
S2 4R  T
2
 R  16
4. F1 and F2 are balanced.
ub
14. Using Shortcut 3,
a F3 1 1 1
 
R r1 r2
F2 F1 r1r2
R= = 4 mm
P

r1  r1

15. W = 8r2T
b
F4 = 8  3.14  (1  102)2  3  102
et

= 7.54  10–5 J
Resultant force = F3  F4
= 1l  2l 16. Then P = P0 +
4S
= (1  2)l r
rg

4 3
5. If an bubble is formed, its radius is equal to that Now P  r = nRgT
3
capillary
 4S  4
Ta

2s   P0   r 3  2RgT
 Required pressure = P0 + gh +  r 3
r
For 2 bubbles,
6. P = P0 + hg = 1.01  105 + (3  103  1030  9.8)  4S  3
 3  107 Pa  P0   A
 rA  n
 A
8. The surface area is given by (S.T.)  Area  4S  3 n B
 P0   B
Work Done = Final surface energy  Initial  rB 
surface energy. 4  0.004 
 2 3
2
= 4 (2r)  4r 2
8   (2  10 )
 2  102  n
= 12r2   A
 4  0.004  2 3 nB
8  2 
(4  10 )
1  4  10 
9. rh = constant  r 
h nB
 8
Hence, if h is halved, then r is doubled. nA

198

Chapter 02: Mechanical Properties of Fluids


17. Force of surface tension balances the weight of Also, pressure at A = h2g,
liquid raised pressure at B = h1g
(d 2 2  d12 ) Net force = Net pressure  area
  (d2 + d1)S =  hg
4 = (h2g  h1g)  A ….(ii)
4s Equating (ii) and (i)
 h=
(d 2  d1 )g  (h2  h1) gA = (yA) x
4  0.075 xy
= 3  h2  h1 =
10  (2  1.5)  103  10 g
= 0.06 m = 6 cm
h
18. Tension in the thread is uniform. We can find 23. v1  2g    gh .…(i)
2
the tension in any portion of thread as follows:

ns
From Bernoulli’s theorem,
Force = Surface Tension  length
h 1
i.e. Tension in the wire = (S)  r 2gh + 4g     4  v 22
2 2
 1 
=S 

io
  v 2  2gh .…(ii)
 8  2 
S v2
=   2
8  2 v1

at
19. Fluids move from higher pressure to lower 24. Gauge pressure at point A = hg
pressure. In a fluid, pressure increases with Total pressure at point A
lic
depth, so pressure at the top Pa (the atmospheric
pressure) is lesser than at the bottom [Pa + dg].
= atmospheric pressure + gauge pressure
= Pa + hg
Hence the air bubble will move from bottom to
top. (It cannot move side ways as the pressure at 25. Using Bernoulli’s equation,
ub
the same level in a fluid is same). In coming 1 2 1
P1 + 1v1  P2  2 v 22 ….(i)
from bottom to top, pressure decreases, so in 2 2
accordance with Boyle’s law i.e., PV = constant, Also, P1  P2 = g  6 .…(ii)
volume V will increase. Thus, the air bubble From (i) and (ii),
P

will grow in size and its radius will increase. 2  P1  P2  2g  6


v 22  v12 = 
 
20. P = hg
= (2g)  6
et

h and  being constant pressure in all four


= 2  980  6
containers is same.
v 22  v12 = 12  980 cm2/s .…(iv)
21.
rg

Gasoline (0.6) 4h From equation of continuity,


Water A1v1 = A2v2
h  1.5
Liquid v1 A 2   0.52
  = = 0.25
  12
Ta

(1.6) v 2 A1
v12  0.252  v 22

Pleft side = Pright side Substituting in (iv),


v 22 1   0.25    12  980
2
w  g  2.5 = gas  g  (4  h) + liqg(h  1.5)  
1000  g  2.5 = 600g (4  h) + 1600g(h  1.5) 12  980
2500 = 2400  600 h + 1600 h  2400 v2 
0.9375
2500 Quantity of water flowing
 h= = 2.5 m
1000 = A1v1 = A2v2
22. Mass of liquid in AB = yA 12  980
=   0.52 
Net force = mass  acceleration 0.9375
= (yA)  x .…(i)  88 c.c per s

199

MHT-CET Triumph Physics (Hints)


26. The pressure of water at the base of aquarium Now, pressure at A, P1 = Pa + hArg
P = hg Pressure at B, P2 = Pa + hBrg
Pressure being linear function of height, average P1  P2 = (hA  hB)rg = hrg
pressure is half of the maximum pressure. Hence option (A) is correct.
Hence force on the lateral wall, Bernoulli’s principle is applicable for non-
F = Pav  A viscous, streamlined flow of liquid. Hence
= Pav  (h  l) option (C) is also correct.
hg
= hl
2
0.4  103  10
=  0.4  0.5
2

ns
= 400 N
27. According to equation of continuity,
Av = constant.

io
By attaching a jet, area of cross-section is
reduced. This results into increasing the velocity
of water flowing out of the pipe.

at
28. For a freely falling body, g = 0 Hence v = 0.
29. When the snow accumulates on the wings of an
lic
aeroplane, the upper surface of the wing
becomes flat. It means the curvature of the
surface decreases. Pressure difference which
causes the lift off of the aeroplane depends on
ub
the curvature of the wing. Thus, due to the
decrease in curvature, the lift-off of the
aeroplane also decreases.
P

30. Velocity of efflux, v  2gd


2H
Time taken for the range r 
g
et

2H
r  2gd 
g
rg

2H
 r2 = 2dg   4dH
g
r2
 d
Ta

4H

31. According to equation of continuity,


Av = constant
At A, area is larger than B hence v is smaller at
A than at B.
Also, from Bernoulli’s principle,
1 2
P+ v = constant
2
This means where v is small, P is more.
At A, pressure is higher. Hence liquid at
point A will raise to greater height than at point
B. Hence option (B) is incorrect.

200
03 Kinetic Theory of Gases and Radiation

Hints

3RT
34. vrms =
Classical Thinking M0

ns
1
10. Ideal gas equation is, PV = nRT Now, K.E.(gram molecule) =  M0 v2rms
2
n P
 = = constant 1 3RT
V RT =  M0 
2 M0

io
Hence, at constant pressure and temperature,
both balloons will contain equal number of gas 3
= RT
molecules per unit volumes. This result is 2
known as Avogadro’s law.

at
3
35. K.E. = kBT
12. Ideal gas law is most accurate for monatomic 2
gases at low pressure and high temperature. 3
=  1.38  1023  300
20. P=
1 mn 2
vrms
lic 
2
K.E. = 6.21  1021 J
3 V
3PV 3  105  100  106 36. Energy = 300 J/litre = 300  103 J/m3
 n= =  5.4  1020
2
mvrms 4.556  1025  3502 2 2  300  103
ub
Using, P = E= = 2  105 N/m2
3 3
3P P
26. vrms = ,v=
  3
37. kBT = 1 eV
2
v rms 3
P

3
 = =  1.46 1.6  1019  2
v  1.41  T=  7730 K
3  1.38  1023

3P 3  1.013  105 5
et

27. vrms = =  1838 m/s 58. For ideal monatomic gas, Cp = R


 0.09 2
2
 R= Cp = 0.4Cp  n = 0.4
v O2  M 0 2 2 1 5
rg

28.   
v H2  M0 1 32 4
Cp 7 2 7
59. = R = = 1.4
v 2rms (500) 2  6  102 Cv 2 5R 5
29. P= =
Ta

3 3 65. Highly polished mirror-like surfaces are good


= 25 × 104 × 2 × 102 = 50 × 102 reflectors but not good radiators.
= 5 × 103 N/m2
67. Open window behaves like a perfectly
30. vrms  T blackbody.
300 27  273 72. Perfectly blackbody is black in colour because it
 =
v rms 927  273 does not reflect or transmit the radiation.
300 1 1 75. When light incident on pin hole enters into the
= = =
1200 4 2 box and suffers successive reflections at the
 vrms = 2  300  vrms = 600 m/s inner wall, at each reflection some energy is
absorbed. Hence the ray once enters the box can
K.E. 3 never come out and pin hole acts like a perfect
33. = P. Here P is constant.
vol 2 blackbody.

201

MHT-CET Triumph Physics (Hints)


Q
83. R=
At Critical Thinking

 [R] =
Q =  M L T 
1 2 2

= [M1L0T3] 8.3  103


 A  t   L2   T1  1. Gas constant = = 2.96  102 J/kg K
28
89. According to Kirchhoff’s law of radiation, 3. P1V1 = n1RT1
R n1RT1 1 R(300)
a=e=  V1 = = × = 75 R
Rb P1 2 2

92. Black cloth is a good absorber of heat. P2V2 = n2RT2


Therefore, ice covered by black cloth melts RT2 R(350)
 V2 = n2 = 1.5× = 105 R
more as compared to that covered by white P2 5
cloth.  P (V1 + V2) = (n1 + n2) RT

ns
93. Blackbody has maximum radiated energy at  P (75 R + 105 R) = (0.5 + 1.5) R (273 + 69)
same temperature.  P  180 R = 2  R  342
342
96. A blackbody has a continuous emission  P= = 3.8 atm

io
90
spectrum
4. The equation of state is, PV = nRT
b 2.93  103
102. T = = = 107 K nRT
m 2.93  1010 P= (ideal gas condition)

at
V
103. According to Wien’s law, Let for M mass there is n moles, then for mass
T1  m24800 48 4 3Mn
= = = = 3M, there are = 3n moles
T2  m1 3600 36 3
lic M
V
Let n = 3n, T = T/3 and V =
1 3
104. As m 
T T
n 'RT ' 3nR
3 = 3nRT = 3P
ub
T Then P = =
 Temperature of other star must be
2 V' V/3 V

105. According to Wien’s law, 6. According to the gas equation, PV = NkBT


mT = mT For the gas A, we have,
P

0 PV = N1kBT ....(i)
 0T =   2T   =
2 V
For the gas B, we have, (2P)   = N2kB(2T)
8
106. According to Wien’s law,  m T  constant
et

 PV = 8N2 kBT ....(ii)


r > y > b
 From equations (i) and (ii),
 Tr < Ty < Tb or TA < TC < TB
N1
N1 = 8 N2  =8
J N2
rg

111. In M.K.S. system, unit of  is 2


m  s  K4
J 7
10 erg 7. Since PV = nRT,
 1 2 = 4 2 For 1 mole of gas, 50  100 = 1  R  T
m sK 4
10 cm  s  K 4
For 2 mole of gas, 100  V = 2  R  T
Ta

erg
= 103 2 50  100 1
cm  s  K 4  =
V  100 2
112. Average K.E. of molecules per mole of ideal gas  V = 100 mL
3
= RT M
2 8. PV = nRT = RT
Where, R = universal gas constant M0
T = same for all gases M density M0
Average K.E. of molecules for one mole of all   
VP P RT
ideal gases at same temperature is same.
 density  M0
    x ….(i)
115. As for a blackbody, rate of absorption of heat is  P At 0C R(273)
more, thermometer A shows faster rise in
 density  M0
temperature but finally both will acquire the    ….(ii)
 P  At 100C R(373)
atmospheric temperature.
202
 Chapter 03: Kinetic Theory of Gases
and Radiation
 From equations (i) and (ii) we get, 5  (5)
16. vavg = = 0 cm/s
 density  273x 2
   =
 P  At 100C 373 52   5 
2

vrms = = 5 cm/s
2
9. Using ideal gas equation,
M 23 4
PV = nRT = RT 17. vmean = = 3 m/s
M0 3
MRT 2.88300 (27  273) 22  32  42 4  9  16 29
 V = = vrms = = =
M0P 28 0.8211.013105 3 3 3
2.99 105 = 3.109 m/s
=  3 litre
105 v mean 3
 = <1
v rms 3.109

ns
10. i. The dotted line in the diagram shows that
PV v1  v 2  v 3  v 4  v 5
there is no change in the value of for 18. v =
nT 5
different temperatures T1 and T2 for 10  20  30  40  50 150

io
increasing pressure. Hence this gas = = = 30 m/s
5 5
behaves ideally. Hence, dotted line
corresponds to ‘ideal’ gas behaviour. 102  202  302  402  502
vrms =

at
ii. At high temperatures, the deviation of the 5
gas is less and at low temperature the 100  400  900  1600  2500
deviation of gas is more. In the graph, =
5
deviation for T2 is greater than for T1.
 T1 > T2
lic =
5500
5
= 1100 = 33.16 m/s
iii. The two curves intersect at dotted line.
PV v rms 33.16
Hence, the value of at that point on  = = 1.105
nT v 30
ub
the y-axis is same for all gases.  vrms : v = 1.105 : 1
3  7 1 2  4  3 20 TA T
11. Mean free path = = 19. 4 B
6 6 MA MB
P

32  42  52 TA T
12. Mean square velocity =  2 B
3 MA MB
50
=  16.7 m/s 3RTA 3RT
et

3  2
MA MB
3 45   c rms A = 2  c rms B
13. Average velocity = = 4 m/s
3
 crms  A
rg

 =2
14. Mean square velocity =
(5)  (6)  (7)
2 2 2
 crms B
3
25  36  49 20. Using, vrms  T,
Ta

=
3  v rms 1 T1
110 =
=  v rms  2 T2
3
Given that,
= 36.7 m2/s2
 vrms  2  v rms 1 1
15. The R.M.S. velocity of the molecule of a gas is T2 = 273 k,  v rms 1 = or =
2  v rms  2 2
3kT
given by, vrms = , where k is the 1 T1 273
m    T1 = = 68.25 K
Boltzmann’s constant and m is the mass of a 2 273 4
molecule.
1
 v rms 1 2 9
 vrms   v  m1/2 21. = =
m  v rms  2 1 8

 (vrms)1 : (vrms)2 : 9 : 8
203

MHT-CET Triumph Physics (Hints)


T 3RT T
22. v rms  27. vrms =  vrms 
M0 M0 M0
(vrms )He THe M H  v rms 2 M1 T2 1 1
=   =  = 
(vrms )H TH M He  v rms 1 M 2 T1 2 2

(vrms)2 =  rms 1  300 = 150 m/s


1 THe 2 v
 =  
2 273 4 2 2
1 T 1 1 1 M 2
 = He  28. P=  v2rms = vrms
4 273 2 3 3 V
273 1 (M / 2) 1 M 1
 THe = K = 136.5 K  P = (2vrms )2 = (4v2rms )
2 3 V 3 V 2

ns
1 M 2 
3RT =2  v rms  = 2P
23. vrms = 3 V 
M0
P 1
 v rms 2  =
T2 (273  90) P 2
 = =  1.1

io
 v rms 1 T1 (273  30)
1
v   29. P=  v2rms ….(i)
3
 % increase =  rms 2  1  100
  v rms 1  3RT

at
Let v2rms =
= 0.1100 M0
= 10% From equation (i) we get,
1  3RT  RT
24. vH =
3RT
, vO =
3RT
lic P= 
3  M0 
 =
M0
MH MO
PM0
1   =
As T = constant, vrms  RT
ub
M0 P P
  and  
MH < MO  vH > vO T 
….( M0 and R are constant)
3RTN 3RTO
25. (vrms)N = and (vrms)O = Given that, P = 2P
P

MN MO
P T 1
Given that, (vrms)H = (vrms)O  = 2 and =
P T 3
3RTN 3RTO ρ P T
 = Then, =  =23=6
et

MN MO ρ P T'
T  273 127  273   = 6 
 =
28 32
30. Mean kinetic energy of molecule depends upon
rg

400
 T + 273 =  28 = 350 K temperature only. For O2, it is same as that of H2
32
at the same temperature of 73 C.
 T = 350 – 273 = 77 C
31. In the mixture, gases will acquire thermal
Ta

26. Using, vrms  T, equilibrium (same temperature). Hence, their


 v rms 1 T1 kinetic energies will also be same.
= .
 v rms 2 T2
32. K.E. =
1
2
mN v2rms  
1
=
1
2

m (2N) v2rms  2
v  v 
Given that,  v rms 2 = rms 1 or rms 1 = 2, Where, m = mass of each molecule.
2  v rms 2

v 2

rms 1
=
2
T1 = 327 + 273 = 600 K
600
v 2

rms 2
1
 2=
T2
or

v 2

rms 1
=
T1
….(  v  T )
600 v 2

rms 2
T2
T2 = = 150 K
4 T
 T1 =
= 150  273 =  123 C 2

204
 Chapter 03: Kinetic Theory of Gases
and Radiation
3 3R 3R
33. K.E.av = kBT R+ =
2 2 2
 K.E.av  T 5
 =
K.E.2 T 600 3
 = 2 = =2
K.E.1 T1 300
45. Let molar heat capacity at constant pressure
 K.E.2 = 2K.E.1 = 2K.E. = CP
K.E. 3 3 and molar heat capacity at constant volume
34. =  P =  105 = 1.5  105 J = CV
V 2 2
 CP – CV = R
35. K.E.  T C
Now, principal specific heat, S =
K.E.1 T 27  273 M
 = 1 =
K.E.2 T2 T  273 R R

ns
 SP – SV =  For H2, a =
But K.E.2 = 2K.E.1 M 2
K.E.1 300 R a
 = For N2, b =  = 14
2K.E.1 T  273 28 b
 T + 273 = 600  T = 327 C  a = 14b

io
3 3 46. Substances having higher specific heat take
36. E= RT =  8.31  273 = 3.4  103 J more time to get heated to a higher temperature
2 2

at
and longer time to get cooled.
37. Using, K.E.  T,
K.E1 T T
= 1. P
K.E 2 T2
Given that, K.E.1 = 2K.E2, T2 = 273 K
lic Q
T1 R
 2=  T1 = 546 K
273
ub
38. From Shortcut 3,
Internal energy of a gas with f degrees of t
freedom, tP tQ tR
f
U= nRT If line is drawn parallel to the time axis, it cuts the
2
P

given graphs at three distinct points.


5 3 Corresponding points on the time axis shows that
Now, f H = , fHe =
2 2
2 t R  t Q  t P  CR  CQ  CP
5 3
 Utotal = (3) RT + (6) RT = 16.5RT.
et

2 2 Qa Q
48. a=  0.75 = a
Q 200
Q 294
39. Cp = = = 29.4 J/mole K  Qa = 0.75  200 = 150 cal
nT 25
rg

40. Cp  Cv = 300 49. Q = Qa + Qr + Qt


Cp  10 = 2 + 7 + Qt  Qt = 1 J
CP
= 1.4  Cv = Qt 1
Cv 1.4  Coefficient of transmission, tr = = = 0.1
Ta

Q 10
Cp
 Cp  = 300 50. For athermanous body, Qt = 0
1.4
 1  Qa Q
 Cp 1   = 300  If = 20 % then r = 80 %
Q Q
 1.4 
 0.4 Cp = 300  1.4  Coefficient of reflection,
300 1.4 Qr 80
 Cp = = 1050 J/kg K r= = 80 % = = 0.8
0.4 Q 100

41. Cp – Cv = R 51. Q = p, Qr + Qt = q
 Cp = R + Cv ….(i) Let, Q = Qa + Qr + Qt
also, Cp = Cv ….(ii)  p = Qa + q  Qa = p – q
3R Qa pq
 substituting Cv = in equations (i) and (ii)  Coefficient of absorption, a = =
2 Q p

205

MHT-CET Triumph Physics (Hints)


52. Initially, the blackbody at room temperature is 65. By Stefan’s law,
darkest and when placed in furnace, it absorbs Q1  T1 
heat till its temperature becomes that of furnace. Q  T4   
Q2  T2 
After this, it emits the radiation of all 4
wavelengths and appears bright. 7  273  227  1
   =
Q 2  273  727  16
54. R  A  r2
2 cal
R2 r   Q2 = 112 2
 =  2 cm  s
R1  r1 
5
2
1 66. For the blackbody,
 R2 =    (10) =  10 = 2.5 J/m2 s Using, Qb = T4,
10
  4
 81 = (300)4 ....(i)
Q (Q / t) For ordinary body, Using,

ns
55. R= =
At A Q = eT4,
 A=
(Q / t)
=
60
= 6  102  Q = 0.8    (500)4
R 1000 81
But area of cube, A = 6l2 = 0.8   (500)4 ....From (i)

io
(300)4
 6l2 = 6  102  l2 = 102 64.8 625
 l = 101 = 0.1 m = 10 cm  Q= = 500 J/m2s
81

at
Q 0.3
56. R=  = 0.50 kcal/m2 s 67. From Shortcut 6,
At 15  103  40 2 4
Q2 r  T 
57. In vacuum, heat flows by the radiation mode =  2    2  = (2)2  (2)4 = 64
Q1  r1   T1 
only.
lic
T1 2000 2 2 68. By Stefan’s law,
58. m =  m =  m = m = m 4 4
2 T2 1 3000 1 3 1 3 Q1 T   27  273 
Q  T4  =  1 =  
Q2 T  927  273 
ub
b  2
59. m = 4
T  300  1
=  =
b 2.93  103  1200  256
 T= = = 7325 K = 7.325  103 K
 m 4000  1010
69. Q1  T14 and Q2  T24
P

60. According to Wien’s law,  m1 T1   m2 T2 Q2 T4 T


 = 24 But T2 = 1
 m T1 700 Q1 T1 2
  m2 = 1
= 4.08  = 2.04 m
et

T2 1400  T1 
4

Q2   1 Q
2
 m2 T1  =  4 =  Q2 = 1
61. By Wien’s law, = Q1 T1 16 16
 m1 T2
rg

T 1500 T 
4
  m2  1   m1   5000 = 3000 Å
70. Q  T4 
Q1
=  1
T2 2500
Q2  T2 
1 4
Ta

62. By Wien’s law, T   


m Q1  T  16
 =  =
TS ( N )max 350 Q2 T T 81
    0.69  2
TN (S )max 510
81
1
 Q2 = Q1
63. By Wien’s law,  m  and from the figure, 16
T Q2  Q1
( m )1  ( m )3  ( m ) 2  % increase in energy =  100  400%
Q1
 T1 > T3 > T2. 4 4
Q1 T   727  273 
dQ 71. =  1 =  
64. By Stefan’s law, = A  T4 Q2  T2   127  273 
dt
dQ 1 J 1 (1000) 4 104 625
  =  =    2 4 = W/m2K4 = 4
= 4 =
dt AT 4  s  m K (400) 4 16

206
 Chapter 03: Kinetic Theory of Gases
and Radiation
72. Radiated power by blackbody, dQ
78.  AT4  r2T4  m2/3 T4
Q dt
P=
t  dQ1 
  2/3 4
From Shortcut 6,
  dt    m1    T1 
   
P1 r  T 
2 4
 dQ 2   m 2   T2 
 =  1   1   
P2  dt 
 r2   T2  2/3 4
2 4 8  2000 
440  20   500  =    
 =     1  1000 
P2  10   1000 
= 4  16 = 64 : 1
1
 P2 = 440   16
4 Q
79. For a blackbody, = P = AT4
 P2 = 1760 W t

ns
4

dQ P2 T 
73. =  T4 Ae  =  2
dt P1  T1 
4
300 P2  273  727 
 = 5.67  10–8  (727 + 273)4  50  10–4  e  = 

io
60 20  273  227 
300 P2
 = 5.67  10–8  1012  50  10–4  e  = (2)4  P2 = 320 W
60 20

at
300
 e= = 0.0176  0.018 EA
283.50  60 80. For A, eA =  EA = eA(Eb)A
(E b )A
74. Energy radiated from a body, EB
Q = AeT4t
4
lic For B, eB =
(E b )B
 EB = eB(Eb)B
Q2 T 
 =  2  eA(Eb)A = eB(Eb)B ....[ EA = EB]
Q1  T1  (E b )A e 0.6
 = B = =2
ub
1/ 4
T2 Q  (E b )B eA 0.3
 =  2
T1  Q1  Now, Eb  T4
 4.32  106 
1/ 4 (E b )A T4 T
=  = A4 = 2  A = (2)1/4
3  (E b )B TB TB
 2.7  10 
P

1/ 4  TA = (2)1/4 TB
16  27
=  
 108 
 27  81. According to Wien’s law,
mT = constant
et

2
= 2  10
 T2 = 200  T1 = 200  400 = 80000 K   m1 T1   m2 T2
 m1 0 4
75. Rate of heat loss  (T4  T04 )  T2 = T1 =  T1 = T1
 m2  3 0  3
rg

Q1 (T 4  T 4 )  
 = 14 04  4 
Q2 (T2  T0 ) Now, P  T4
Q1 (600)  (300)4
1215 4
P2 T 
4
Ta

 = =  =  2
Q2 (900)  (300)
4 4
6480 P1  T1 
16
 Q2 = Q P2  4 / 3 T1  256
4

3  =  =
P1  T1  81
76. For same temperature, refer Shortcut 6
82. Since, Q = mcd
77. Heat radiated per second per unit area  T4 dQ d
Here, T1 = 127 C = 400 K  = mc = A(T4 – T04)
dt dt
T2 = 527 C = 800 K d 4r 2 (T 4  T0 4 )
Since T2 = 2T1 and E  T4,  =
dt 4 3 
4
Q 2  T2   2T1 
4
 r c 
=  = 4 3 
 = (2) =16
Q1  T1   T1  d 1
 
 Q2 = 16 Q1 = 16  6 = 96 J dt rc

207

MHT-CET Triumph Physics (Hints)

3RT  Ae(T 4  T04 )


83. vrms = 87. Rate of cooling (R) = 
M0 t mC
Let momentum of A, pA = MAcA A Area r2 1
 R   3
m volume r r
3RT
= MA 1 1
MA  Rate (R)   1/3
r m
p 2A M A p2 
 3 RT = = A ....(i) 4 3 1/3 
MA 2
MA  m    3 r  r  m 
 
Let momentum of B = pB = MBcP 1/3 1/ 3
R1  m 2  1
3RT     
= MB R 2  m1  3
MB

ns
p 2B M B p2B  dT 
 3 RT = = ....(ii) 88. Rate of cooling     emissivity (e)
M 2B MB  dt 
From equations (i) and (ii) we get,  dT   dT 
From graph,         ex > ey
p 2
p 2  dt  x  dt  y

io
A
= B

MA MB Further as good absorbers are good emitters,


emissivity (e)  Absorptive power (a)
 MA  2
 p 2A =   pB  ax > ay

at
 MB 
1/ 2
 MA  Competitive Thinking
 pA =   pB
 MB 

1
lic 1. Number of moles in 4 g of hydrogen,
M 4
84. P =  v2rms n= = =2
3 M0 2
Since mass and volume is same, the density is  PV = nRT = 2RT
ub
constant.
1 1 2. Ideal gas equation gives,
 P  v2rms But v2rms   P PV = nRT
M0 M0
 For n = 1
PO M H 2 1
P

  = = V=
RT
....(i)
PH MO 32 16
P
1 molar mass
 PO =  4 = 0.25 atm  density =
16
et

volume
m(N A )P
f = ....[From (i)]
85. For 1 kg gas, energy, E = rT RT
2
R
rg

From Shortcut 2, But, =k


NA
P
rT = k = Boltzmann constant

mP
 density =
Ta

5 8 10 4
 E=  ….( f = 5 for diatomic gas) kT
2 4
 E = 5  104 J 4. By ideal gas equation,
V nR
 PV = nRT  =
 Ae(T 4  T04 ) T P
86. Rate of cooling, R = 
t mC V
= constant ….(at constant P)
A Area T
R 
m Volume Hence, graph (A) is correct.
1
 For the same surface area, R  5. From ideal gas equation
Volume
PV = nRT
 Volume of cube < Volume of sphere PV = n1RT
 RCube > RSphere i.e. cube cools down at a faster After leakage,
rate. PV = n2RT

208
 Chapter 03: Kinetic Theory of Gases
and Radiation
No. of moles of gas leaked is given by n1  n2 RT
12. vrms =
PV PV M0
i.e. n1  n2 = 
RT RT 1
V vrms 
 n1  n2 =  P  P M0
RT
 v1 rms  M 0 1 32
6. Using ideal gas equation,  
before heating, at T1 = 17 + 273 = 290 K,
 v2 rms  M 0 2 2

PV = n1R  290 ....(i) v2 = 2 km/s


After heating, at T2 = 27 + 273 = 300 K, 1
13. vrms 
PV = n2R  300 ....(ii) M0
where, n1 and n2 are number of moles at T1 and Let v1 be the rms velocity of uranium of mass
T2 respectively.

ns
M1 = 235 units and v2 be the rms velocity of
From equations (i) and (ii), uranium of mass M2 = 238 units
PV PV
n2 – n1 =  v1  v 2 M 2  M1 238  235
R 300 R  290  = = = 0.0064
v2 M1 235
But, nf – ni = (n2 – n1)NA

io
v1  v 2
i.e., nf – ni = 
PV  10   % ratio = × 100 = 0.64
   6.02310
23
v2
R  290  300 
Given: P = 105 Pa and V = 30 m3

at
3RT
14. vrms =
105  30 10  6.0231023 M0
 nf – ni =  = –2.5  1025
8.3 290  300 lic  vrms  T
1 1 v2 T
7. = =   2
d n 2 4r n 2
2 2
v1 T1
1
 2  v2 = v1
T2
r
T1
ub
8. Mean free path of gas molecules is,
127  273
1 k BT  v2 = 200 
 = 27  273
2 d P
2

1 k BT 400
 v2 = 200 
P

 L= 300
2 4 r P
2

400
According to given conditions,  v2 = m/s
1 k B T 3
et

L = v 1 T 1 6 1
2 4   r   P
2
15. vrms  T     
v 2 T 2 300 100
L T r 2P 2T r 2P 1  The rms velocity will increase nearly by 1%
=  =  =
L  r 2 P T  2r    2P 
2
T 4
rg

16. Its known from kinetic theory of gases-


L
 L = v1 T
 1 but v2 = 2v1 (given)
4 v2 T2
Ta

10. The rms velocity is related to Temperature as


v1 27  273 1 300
vrms  T  =  
2v1 T 4 T
 v1 rms T1  v1 rms 0  273
 T = 1200 K = 927 C
   =
 v2 rms T2 1  T2
 v1  1 2
 2 rms 17. Using, P = v rms ,
3
273
 T2 = = 68.25 K   v rms 
2
4 P1
= 1 2 1
 t2 = T2 – 273 = 204.75 C P2 2  v rms 
2
2
3RT   v rms 1   P    3 2
11. vrms =    =  1    2  =  = 1
M0  v
 rms 2   P2   1  2 3

v'rms =
3R(2T)
= 2 vrms  vrms 1
=1
M0 / 2  vrms 2
209

MHT-CET Triumph Physics (Hints)


1  3  5  7 16 27. Diatomic gases have 5 degrees of freedom,
18. Average speed = = = 4 km/s
4 4 neglecting vibrational mode.
12  32  52  7 2 84 2 2 7
R.M.S. speed = =  For hydrogen, 1 = 1 + = 1 + =
4 4 f 5 5
= 4.583 km/s Monatomic gases have 3 degrees of freedom,
2 2 5
 R.M.S. speed  average speed = 0.583 km/s  For helium, 2 = 1 + =1+ =
f 3 3
3
20. Average kinetic energy = RT Considering two vibrational modes, diatomic
2 gases have 7 degrees of freedom,
i.e. K.E.  T 2 2 9
As T is constant, K.E. remains same.  For gas X, 3 = 1 + =1+ =
f 7 7
21. Pressure exerted by the gas on wall of container  7 5 9 
is given by,  (1, 2, 3) =  , , 

ns
5 3 7
1 2
P= v .…(v  r.m.s. speed) 28. Using Shortcut 3,
3
Amount of energy required is given as,
1 M
 P =   v2 f f
n RT  NK  T2  T1 

io
3 V E
2 2
2 1 M 2 f
P=
3
 v
2 V   E =  n  N A   k B  T2  T1 
2

at
2  K.E  1 where N = nNA and kB = Boltzmann constant
 P=   ....( K.E. = Mv2)
3  V  2 3
 E= n N A k B (T 2 T1 )  ….( f = 3 for He)
2
3
22. K.E.avg = kBT
2
lic Now, n 
M 1
M0 4

2 0.69  1.6  1019
 T=  = 5333.33 K = 5060 C 3 1 3
3 1.38  1023  E=  N k (T  T1 ) = N A k B (T2  T1 )
2 4 A B 2 8
ub
23. The average kinetic energy of monatomic gas
3 29. Molar specific heat at constant pressure
molecule (K.E.) = k BT 7
2 Cp = R
2
3
K.E. =  (1.38  1023 J K1)  (300 K) Using, Cp  Cv = R
P

2
7 5
3 (1.381023 JK 1 )  (300K) Cv = Cp  R = R R= R
= 2 2
2  (1.6 1019 J / eV)

Cp  7 / 2 R 7
= 3.9  102 eV = 0.039 eV = =
et

Cv (5 / 2)R 5
24. Average kinetic energy per molecule for any
kind of molecule of an ideal gas is 30. Given,
3 R
= 0.4
rg

K.Eavg = kT Cv
2
3 Cp  Cv
 (K.Eavg)hydrogen = kT1 and  = 0.4
2 Cv
Ta

3 Cp
(K.Eavg)oxygen = kT2  = 0.4 + 1
2 Cv
But T1 = T2   = 1.4
 (K.Eavg)O = (K.Eavg)H  the molecules of the gas are rigid diatomic.
26. Using Shortcut 3, CP
Internal energy of a gas with f degrees of 31. Given: =
CV
freedom,
f C P  CV  1
U= nRT  =
2 CV 1
5 3 R
Now, fO2 = , fAr =  =–1 .…( CP – CV = R)
2 2 CV
5 3 R
 Utotal = (2) RT + (4) RT = 11RT.  CV =
2 2  1

210
 Chapter 03: Kinetic Theory of Gases
and Radiation
32. For rigid diatomic molecule, 2.88  106nmK
CP 7 5  max =  500 nm
=   CV = CP 5760 K
CV 5 7  wavelength of maximum energy = 500 nm
Also for molar specific heats, i.e., U2 is maximum energy.
5
CP – CV = R  CP  CP = R 42. From Wien’s displacement law-
7
max T = constant
2 2
CP = R  n= = 0.2857 If T is also same, max = constant
7 7
Hence, max  max
33. dU = n × Cv × d
R  R  44. Using Shortcut 6,
=n× × d ...   C v   l b
 1   1  4
 3 × 3   327 + 273  4
E 2  A 2   T2 
=    =    

ns
8.314
= 2000 × × (–10) = –4.2 × 105 J E1  A1   T1   l × b   27 + 273 
0.4  
4
f   1   600 
34. CP =   1  R =   
2   9   300 

io
E2 1
 f=2 
 33.24 
 1 = 2  3 = 6    16 
 8.31  E1 9
16

at
35. r + a + tr = 1  E2 = E ....( E1 = E)
9
 tr = 1 – r – a = 1 – 0.8 – 0.1 = 1 – 0.9 = 0.1
Q = 1000 J/min 45. Q  T4
 Heat energy transmitted per minute
Qt = Q  t = 1000  0.1 = 100 J
lic 
Q1
Q2
T   T 
=  1 =  1  =
4
1
16
4

 T2   2T1 
 Heat energy transmitted in 5 minutes
 Q2 = 16Q1
= 100  5 = 500 J
For blackbody, P = AT4.
ub
36. From Wien’s displacement law, 46.
1

1 For same power, A 
T4
T 2 4
T  r1   T2 
   =  
P

This means more the temperature higher will be r


 2  T1 
the corresponding frequency. 2 2
r1 T  r T 
Given T2 > T1, hence frequency corresponding  =  2  i.e., 2 =  1 
to maximum energy is more at T2. r2  T1  r1  T2 
et

37. As Red > Green > Violet, 47. For perfectly blackbody,
λQ > λR > λP. Q = AT4t
According to Wien’s law, TQ < TR < TP = 5.7 × 10–8 × 1 × (727 + 273)4 × 60
rg

= 3.42 × 106 = 34.2 × 105 J


38. From Wien’s displacement law,
1 48. Using Shortcut 6,
m  2 4
T P2 r  T 
Ta

 m T = constant  =  2  2
P1  r1   T1 
40. From Wien’s displacement law r
Now, r2 = 1 and T2 = 2T1
b 2
T=
 max P2 1
 =  16
b = Wien’s constant P1 4
2892  10 6  P2 = 4  450 = 1800 W
 T= = 200 K
14.46  106
49.   4R2 (T4  T04 ) = 912  R2
41. According to Wien’s displacement law, 912 912
1  T4  T04 = = = 40  108
max  4  45.7 108
T
 T4 = 40  108 + (300)4 = (40 + 81)  108
 maxT = b
 T  330 K
also T = 5760 K
211

MHT-CET Triumph Physics (Hints)


50. The rate of radioactive energy emission from a hot 56. Escape velocity at the surface of the earth
surface is given by Stefan-Boltzmann Law- = 11.2 km/s = 11.2  103 m/s
dE
R=  A  Thot
4
 Tambient
4
 Oxygen will escape when rms speed of its
dt molecules,
R   400  200  3 vrms = 11.2  103 m/s
4 4

Hence,  
R  6004  2004  16 3k BT
 = 11.2  103
M0
51. Rate of loss of heat by radiation is given as
11.2 
2
dQ 106  2.76 1026
– = A  Thot 4
 Tcold  = R  T=
dt 31.381023


RA  Thot4  Tcold
=
4
 A = 8.363  104 K
RB T 4
hot  Tcold
4
 1 U  1

ns
B
4
 (327  273) 4  (27  273) 4 
57. P=    kT
RA 3 V  3
 = 
RB  (227  273) 4  (27  273) 4  u
(  T4 and k is constant of proportionality)
V
(6004  3004 ) 9
= =

io
PV = nRT
(5004  3004 ) 4
nRT 1 4
52. Using Shortcut 6,  kT
V 3
2 4
 V T–3
2 4

at
R1 r  T   8   127  273 
 =  1    1  =    
R2 r
 2 T
 2  2   527  273  Volume of spherical shell of radius R =
4
R3
4 3
 400  16
= 16    = =1 i.e., V  R3

53. According to Avogadro’s law,


 800 
lic
16
R
1
T
1 mole = 22.4 L of any gas
 67.2 L = 3 mole  n=3 58. Speed of sound in a gas is,
ub
3 P
Cv = R for monatomic gas v=
2 
8.31  3 In standard conditions, P = 1 atm = 105 Pa
 Q = n Cv T = 3   20 = 748 J
2
  105  1089
P

54. From Wien’s displacement law,  330 =


1400
max T = b
  105  1089
T    500 5  108900 =
Hence, A  max B   1400
et

TB   max A 300 3
  = 1.4
Now, from Shortcut 6, 2
Q r2 T4 But,  = 1 +
power Ratio, A  A2 A4 f
rg

Q B rB TB 2
 f= =5
where, A = 4r2    1
2
Q A 32 54  5 
i.e.     59. Q = msdT
Ta

Q B 52 34  3 
dQ dT
55. According to Wien’s displacement law, = ms
dt dt
mT = constant Pdt = msdT
T1  m 2 3 / 4  0 3 P
    dT = dt
T2  m 0 4 ms
1

Power radiated for a blackbody, P = AT4 Rise in temperature (dT) 


1
P1  T1 
4
s
   From graph we can observe that rise in
P2  T2 
4
temperature in graph A is more than B and C.
P 3 81  dT is maximum for A and minimum for C and
   
nP  4  256 specific heat value is maximum for C and
 n=
256 minimum for A.
81

212
 Chapter 03: Kinetic Theory of Gases
and Radiation

Hints to Evaluation Test


1. Black is a good absorber and also a good 7. 3PV = nHRT ….(i)
emitter as per Kirchhoff’s radiation law. P(2V) = nOR(3T) ….(ii)
2. Since power radiated is same for body A and Dividing equation (i) by (ii),
body B, 3 nH 1 n 9
=  H 
TA 4
0.49  1  2 nO 3 nO 2
  ….   T4 
Using Avogadro’s principle,
TB 4 0.01  emissivity 
1 H

 2n H N A  / V  n H  1  9
TA  0.49  4
O  32n O N A  / 2V n O 8 16
or   = 2.6
TB  0.01 
Qr
T 5200 8. Using r = ,

ns
or TB = A = = 2000 K Q
2.6 2.6 15
Using Wien’s displacement law r= = 0.1
150
i.e., m T = constant
Using a + r + tr = 1,
we get, ATA = BTB

io
tr = 1  (a + r) = 1  (0.6 + 0.1) = 0.3
 TB   B Qt
or A = B  = Now using, t = we get,
 TA  2.6 Q

at
But B  A = 1 m (given) Qt = Qt = 150  0.3 = 45 J
 1.6
 B  B = 1 m or  B = 1 m  vrms 1 T1
2.6 2.6 9. Using, vrms  T, 
2.6  vrms 2 T2
or B =
1.6
 B = 1.6 m
lic Given that, T2 = 273 K,
 vrms 1
 M  RT 10  R  293 (vrms)1 = 4 (vrms)2 or 4
4. P0 =    ….(i)  vrms 2
 M0  V M0V
ub
T1
Gas is heated to 50 C and x gram of gas  4=
273
escapes, pressure is still P0
(10  x)g (273  50)  T1 = 273  16 = 4368 K
 P0 = R ….(ii) = 4368  273 = 4095 C
M0 V
P

 10(293) = (10  x) (323)  x  0.92 g 11. Using, a + r + tr = 1,


….[From (i) and (ii)] a + 0.74 + 0.22 = 1  a = 0.04
By Kirchhoff’s law, a = e  e = 0.04
5. Isothermal compression  T = constant
et

12. By Stefan’s law,


3RT Rate of loss of heat  Area
vrms =
M0 For sphere, A = 4r2
 A  r2
rg

8RT
 Mean momentum = mv = m
M 0  R1  r12 and R2  r22
2
3 r  1
2

=  1  =   =
Mean kinetic energy = RT R1 1

Ta

2 R2 r  2  4
 2
All the above equations are functions of
temperature, which is a constant. 13. According to Wien’s displacement law,
1
6. According to Kirchhoff’s law, good absorbers m   m2   m1 …. ( T1  T2 )
T
are good emitters and bad reflectors. While at
Therefore I– graph for T2 has lesser
lower temperature, a blackbody absorbs all the
wavelength (m) and so curve for T2 will shift
incident radiations. It does not reflect any
towards left side.
radiation incident upon it when it is thrown into
the furnace. Initially, it is the darkest body. 14. From PV = nRT as per given data,
At later times, the blackbody attains the PHe n M /  M He 0  M H 0
Pn = He = =
temperature of the hot furnace and so it PH nH M /  M H 0  M He 0
radiates maximum energy. It becomes the
brightest of all.  M H 0 2
 PHe = PH × = 4 × = 2 atm
Option (A) represents the answer.  He 0
M 4

213
04 Thermodynamics

Hints

31. For an isothermal process,


Classical Thinking U = 0 and work done = W = P (V2  V1)

ns
As volume decreases, the work done by the gas
7. Internal energy depends only on the temperature is negative.
of the gas.
32. Refer Mindbender 3(i)
10. W = PV; here V is negative. Hence W

io
will be negative 35. When heat is supplied at constant pressure, a
part of it goes in the expansion of gas and
17. Heat always refers to energy in transit from one remaining part is used to increase the
body to another because of temperature temperature of the gas which in turn increases

at
difference. the internal energy.
19. Refer Mindbender 3(ii). lic 37. Work done = PV = P(V2 – V1)
20. Q = U + W where W = PV 41. The process is very fast; so the gas fails to gain or
21. st
From 1 law of thermodynamics, lose heat. Hence, this process is adiabatic.
U = Q  W 45. Air inside the tube of wheel when suddenly
 U = Q (< 0) ….( W = 0) comes out, undergoes adiabatic expansion.
ub
During adiabatic expansion the temperature of
 U < 0 the system decreases and thus, the air becomes
So, temperature will decrease. slightly cooler.
22. Q = U + W 48. In thermodynamic processes,
P

U = Q  W Work done = Area enclosed by PV-diagram and


V-axis.
23. Q = U + W
From graph, it is clear that (Area)iso > (Area)adi
 W = Q – U = 110 – 40 = 70 J
et

or Wiso > Wadi


25. As the system is not in equilibrium during rapid
expansion, it cannot be described by equation of P
state.
rg

Isothermal
26. As the volume is continuously increasing and
the work of expansion is always positive, so the Adiabatic
work done by the system continuously V
Ta

increases.
50. By the first law of thermodynamics,
27. Work done = Area of triangle Q = U + W
1 In adiabatic process, Q = 0  U = – W
=  AC  AB
2
51. For an adiabatic process, Q = 0
1 PV
= VP=  Work is done on the gas, W = – 90 J
2 2
 From Q = U + W,
28. A quasi-static process like a slow isothermal 0 = U  90
expansion or compression of an ideal gas is
 U = + 90 J
reversible process while the other given
processes are irreversible in nature. 53. In a closed cyclic process, the change in internal
energy is always zero  E = 0
29. In isothermal process, heat is released by the gas
to maintain the constant temperature. 54. In isochoric process, volume remains constant.
214

Chapter 04: Thermodynamics


55. Process CD is isochoric as volume is constant;
W = P  V  
10V
  V
process DA is isothermal as temperature  100 
constant and process AB is isobaric as pressure 10PV
is constant. = = 0.1  105  4 = 4  104 J
100
56. As work done is area under PV curve, 4. Work done = P(V2 – V1)
W1 > W2. Now, as density of water = 1 g/cm3
61. When the freezer is placed on top, the cold air  V1 = 1 cm3 = 106 m3
produced from it is denser than the warmer air  W = 1  105  (1.091 – 1)  10–6 = 0.0091 J
in the bottom. Cold air being dense flows down 5. Refer Mindbender 3(i)
and the warm air is forced to rise up. So, when
the warm air rises up, it becomes colder in the 6. Q = W + U
freezer. As a result, uniform cooling is  35 =  15 + U  U = 50 J

ns
maintained in the refrigerator. 7. By 1st law of thermodynamics,
62. In a refrigerator, the heat dissipated in the Q = U + W
atmosphere is more than that taken from the 2  103  4.2 = U + 500
 U = 7900 J

io
cooling chamber, therefore the room is heated if
the door of a refrigerator is kept open.
8. By 1st law of thermodynamics,
QC T Q = U + P(V)

at
64. = C  U = Q – P(V)
QH TH
300
= 1500 – (2.1  105) (2.5  10–3) = 975 J
 QH = 500   577 calorie
260 9. For the first process, using Q = U + W.

72.
lic
For path (2), V decreases for constant P which


8  105 = U + 6.5  105
U = 1.5  105 J
results in decrease in temperature of gas. Since final and initial states are same in both
73. The cyclic process 1 is clockwise whereas processes, U will be same in both processes
ub
process 2 is anticlockwise. Clockwise area For second process, using Q = U + W,
represents positive work and anticlockwise area  105 = 1.5  105 + W
represents negative work. Since negative area  W = – 0.5  105 J
(2) > positive area (1), hence net work done is Negative sign indicates work is done on the gas.
negative.
P

10. Q = U + W
74. As internal energy is a state function, therefore  U = Q  W = 2240  168 = 2072 J
change in internal energy does not depend upon 11. Q = U + W = (Uf  Ui) + W
the path followed i.e., UI = UII.
et

 30 = (Uf  30)  10


75. Given that, Q = 15 J  Uf = 10 J
Work done (W) = Force  displacement 12. Using Shortcut 2(ii)
rg

=52 W 1 3 2
 = 1 – = 1 – = = 0.4
= 10 J Q  5 5
 From first law of thermodynamics,
CP 5
U = |Q| – |W| = 15 – 10 = 5 J 13. For monatomic gas,  = 
Ta

CV 3
Using Shortcut 2(i),
Critical Thinking
U C V 3
 = = 0.6
1. In first case, there is no change in volume. Q C P 5
So work done is zero. Similarly, from Shortcut 2(ii),
In the second case, W 1 2
 1  = = 0.4
W = P(nV – V) = (n−1)PV Q  5
 Percentage of heat utilized in increasing internal
2. W = P(V2  V1) = 105  (2  1) = 105 J
energy,
3. External work done = PdV = P (V2 – V1) U
 100 = 0.6  100 = 60%
V1 = V = 4 m3 and Q
10V W
V2 = V + 10% V = V +  100 = 0.4  100 = 40%
100 Q

215

MHT-CET Triumph Physics (Hints)


14. From Mindbender 3(ii), 22. As PV = constant it is isothermal process. As a
U = mL – P(V2 – V1) result, the temperature remains constant.
P(V2  V1 ) Also from PV graph we know that, pressure is
= 540  decreasing hence P2 < P1 which is represented
J
1.013  105   (1671  1)  106 
by graph (iii).
= 540 
4.19 23. Differentiating the equation,
= 540  40.37  500 calorie PV = constant
P V
15. State of a thermodynamic system cannot be PV + VP = 0  =–
P V
determined by a single variable (P or V or T).
V2
16. Work done by the system = Area of shaded 24. Using, W = nRT ln
portion on P-V diagram V1

ns
= (300 – 100)10–6  (200 – 100)  103 = 20 J m V 
=   RT ln  2 
As anticlockwise area of cyclic process denotes M  V1 
negative work, net work done by the system in
m V 
this case, W = −20 J ≈ 2.3  RT log10  2 
M  V1 

io
17. Work done = Area of PV graph (here trapezium)
96  140 
1 = 2.3  R (273 + 27) log10  
= (1  105  5  105 )  (5  1) 32  70 
2

at
= 12  105 J = 2.3  900 R log102

18. WAB is negative (volume is decreasing)  V2 


25. W = nRT ln  
WBC is positive (volume is increasing)  V1 
Since |WBC| > |WAB|,
lic = 0.2  8.3  (27 + 273)  ln 2
 net work done is positive. = 0.2  8.3  300  0.693
Area between the semicircles is,
 345 J
 1
2
 r2 
ub
= = atm-litre  V2 
2 2 2 26. W = 2.303  nRT log  
 V1 
19. As initial and final states in the two processes
are same,  V1 
= − 2.303  nRT log  
Therefore, u1 = u2.
P

 V2 
Also, area under curve a > area under curve b,
= –2.303  1  8.31  (273 + 0) log 
22.4 
 W1 > W2 
 11.2 
As Q = u + W
et

= 8.31  2.303  273  log 2


 Q1 > Q2
= 1572.5 J ….( log 2 = 0.3010)
20. Being the same gas, n = constant
From equation of states, 27. Using, P1V1 = P2V2 we get,
rg

P1V1 PV 80V P 10
= 2 2 PV = P   =
T1 T2 100 P 8
P1V1 5P V P  P  10 
 = 1 1   100 =   1  100
Ta

T1 3T2 P 8 
5T1 2 
 T2 = =   100 
3 8 
 T = T2  T1 = 0.667  67% 1
=  100
21. In step-1 piston is not fixed hence, pressure is 4
constant and volume changes. So work is done = 25 %
while maintaining temperature constant. Hence, V1
the process is isothermal and c = ∞. 28. V2 =
4
In step-2 piston is fixed hence, pressure
The process being slow, is isothermal.
increases and volume is constant so work is not
done which increases the internal energy and  T2 = 75 C and P1 = 50 cm ….(Given)
temperature of gas. Hence, process is not P1V1 50
 P2 = = = 200 cm of Hg
isothermal and c ≠ ∞. V2 1/ 4

216

Chapter 04: Thermodynamics


29. Due to compression the temperature of the system 3
Since for monatomic gas such as helium,  =
increases to a very high value. This causes the 2
flow of heat from system to the surroundings, thus 2/3
1
decreasing the temperature. The decrease in  T1 = T2    4T1 = T2
temperature results in decrease in pressure.
8 
1
30. For adiabatic process, PV = K nR  T2  T1  R 3T1  9
 Work done = =4 = RT1
Differentiating above equation,  1 2 8
 PV – 1 dV + dP V = 0  
3
 
dP
 =– K
P 37. P shows it’s an adiabatic process
dP  dV  V
  100 = –   100  = –1.4  5 = −7 % For adiabatic process, C = 0.
P  V 

ns
Negative sign indicates decrease in pressure. 3
38. Here,  =
31. As the change is sudden, the process is adiabatic 2
 P1V1 = P2V2 For adiabatic process, TV–1 = constant
 1

io
 T2  V1  3
=  22 = (2)1/2
3/ 2 1
P2 V  4 8   
 =  1 =   = T1  V2 
P1 V
 2 1  1
 T2 = 2 T

at

P V  P
32. Using, 2 =  1   = (8)5/2 39. For adiabatic process, TV–1 = constant
P1  V2  P 1/    1 1/ (  1)
V2 T 
=  
 P = P  (2)15/2 1
 =  1
33. As the change is sudden, the process is adiabatic
lic V1
V
 T2 
1
2  

 PV = constant  P1V1 = P2V2 or 2 < ....(  > 1)


 1.4 V1 2
V   300 
 P2 = P1  1  = 106    = 106 (2)1.4
ub
V
 2  150  40. Change in internal energy of the gas,
= 2.64  10 dyne / cm 6 2 R  T2  T1 
U =  W= ....( n = 1)
 1
34. P1V1  P1V2
8.3
= (308 – 300) = 166 J
P


 P2   V1  (1.4  1)
  =  
 P1   V2 
1
41. For adiabatic change,
P   PV = constant.
et

 V2 = V1  1  = V1C1/ …(C > 1)


 P2   dP +  dV = 0
1 dP 2 2 4
P    dV =  =   %=  %
 V2 = V1  1  = V1C
1/  3 3 9
rg

P
 2 Negative sign indicates volume is decreasing.
   Monatomic 
  >     V2  V2 42. For a cyclic process, U = 0
   Polyatomic 
 By 1st law of thermodynamics,
Ta

This means, Assertion is false, Reason is true.


Q = U + W = 0 + W
nR  Ti  Tf  = Area of closed curve
35. Using, W =
 1  20 
2

 Q = r2 =    kPa  litre


R  T  Tf   2 
 6R = …. (Given: n = 1)
5  = 100  103  103 J
  1
3  = 100 J
 Tf = (T – 4) K
43. From the given VT diagram,
36. As 22.4 litre of gas contains 1 mole of gas, 5.6 For process AB, V  T  Pressure is constant
1 For process BC, V = Constant and for process
litre of gas contains number of moles of He =
4 CA, T = constant
Using, T1V1 – 1 = T2 V2 – 1, These processes are correctly represented on PV
 T1(5.6) – 1 = T2(0.7) – 1 diagram by graph (C).
217

MHT-CET Triumph Physics (Hints)


44. AB is isobaric process; BC is isothermal Dividing equation (i) by (ii),
process; CD is isometric process and DA is 500 5
=  TC = 400 K
isothermal process. TC 4
These processes are correctly represented by
graph (A). TC 300 1
52. max = 1 – =1– = = 25%
45. For isothermal process (gas A) TH 400 4
P1V1 = P2V2  24% efficiency is possible.
 P0(2V0) = P2(V0) TC W
 P2 = 2P0 53. =1– =
TH Q
For adiabatic process (gas B)
PV = constant 
 TC    273  27    Q
W = 1   Q = 1  
P2  TH    273  627  
 = (2)

ns
P1  300  6
 W = 1    3  10 cal
 P2 = (2) P0  900 
(P2 ) B (2)  P0 = 2  106  4.2 J
Hence,  = (2)  1
(P2 ) A 2P0 = 8.4  106 J

io
W Q  QC 200 TH  TC
46. = = H = 54. In first case,  =
QH QH 746 TH

at
 % = 26.8 %  27 % 2TH  2TC T T
In second case,  = = H C 
2TH TH
1g 2kcal
47. Input energy  = 2 kcal/s
s g TH  TC 50 T

Output energy = 10 kW = 10 kJ/s 


lic
10
kcal / s
55. Efficiency,  =
TH
=
100
=1 C
TH
4.2 TC 1 273
 =  TC =  137 K
o/p energy 10 TH 2 2
    1, which is impossible.
i/p energy 4.2  2
ub
….( TH = 0 C = 273 K)
48. QC = 2000 cal
56. TC = 0 C = 273 K,
QC
As Coefficient of performance = , TH = 17 C = 17 + 273 = 290 K
W
QC TC
P

 4 = 2000/W Coefficient of performance = =


W TH  TC
 W = 500 cals = 500  4.2 J = 2100 J
mL
=
49. QH = 104 cal = 42 kJ = 42  103 J W
et

TC 300 801000 4.2 273 273


 QC =  QH =  42 103 = 0.6  42  103  = =
TH 500 W 290  273 17
 W = QH – QC = 0.4  42  103 80 1000  4.2 17
rg

 W= joule
= 16.8  103 = 1.68  104 J 273
TC 33.6 17 10 4
50. =1  W= kWh = 0.058 kWh.
TH 273 3.6 105
Ta

30 350 57. Here, PV = constant


 =1
100 TH  PdV = VdP
350 30 7 dP P
 =1 = i.e. = 
TH 100 10 dV V
 TH = 500 K = 227 C dP dP
Bulk Modulus, K =  V
dV / V dV
TC  P 
51. =1– =  V  = P
TH
 V 
1 500 500 1
 =1–  = ….(i) 58. From first law of thermodynamics,
2 TH TH 2
Q = U + W
60 T T 2 Given: W = 0
=1– C  C = ….(ii)
100 TH TH 5  Q = U = CV (T)

218

Chapter 04: Thermodynamics


3 5 
For monatomic gas, CV = R 65. dU = CvdT =  R  dT
2 2 
3 2(dU)
 Q = nCV (T) = 3  R  100 = 450 R  dT =
2 5R
59. From first law of thermodynamics, From first law of thermodynamics,
Q = U + W Q 3Q
dU = dQ  dW = Q  =
Work done at constant pressure, 4 4
(W)p = (Q)p  U dQ Q
 (W)p = (Q)p  (Q)v ….[(Q)v = U]  Molar heat capacity, C = =
dT  2(dU) 
 
Also, (Q)p = mCp T and (Q)v = mCvT  5R 
 (W)p = m(Cp  Cv)T 5RQ 10
= = R
 (W)p = 1  (3.4  103  2.4  103)  10

ns
 3Q  3
2 
= 104 cal  4 
60. We know that, slopes of isothermal and
adiabatic curves are always negative and slope Competitive Thinking

io
of adiabatic curve is always greater than that of
isothermal curve. AT  BT 2
1. V=
Hence, in the given graph, curve A and B P

at
represent adiabatic and isothermal changes W = P  V = P[V2  V1]
respectively.  AT2  BT22  AT  BT12  
= P  1 
61. For both the paths, U remains same.  P  P 
For path iaf : U = Q  W = 50  20 = 30 J. =  A(T2  T1 )  B  T22  T12 
For path fi : U = 30J and W =  13J
lic
 Q =  30  13 = 43 J. 2. Using first law of thermodynamics,
62. In adiabatic process, Q = 0 Q = U + W
ub
 work is done on the system  internal energy  U = Q – W
of the system increases Given that, Q = 35 J,
 U = W  n  Cv  T = W Here, W is negative because work is done on
 R  3 the system.
 n    7 = 146  10
P

   1   W = –15 J
8.3  U = 35 J – (–15 J) = 50 J
 103   7 = 146  103
   1
3. Q = U + W
et

 On solving we get,  = 1.4


mL = U + PV
 The gas is diatomic.
 U = mL – PV
63. From the graph, WAB = 0 and = (1  540  4.2) – (105  1650  10–6)
rg

WBC = 8  104 (5  2)  103 = 240 J  U = 2103 J


 WAC = WAB + WBC = 0 + 240 = 240 J
 QAC = QAB + QBC = 600 + 200 = 800 J 4. Using Mindbender 3(ii)
By 1st law of thermodynamics,
Ta

For boiling process,


QAC = UAC + WAC U = mL – P(Vsteam  Vwater)
 800 = UAC + 240 = 1  2256 – [105  (1671  1)  106]
 UAC = 560 J = 2256 – 167
64. For the given cyclic process, = 2089 J
total work done = WAB + WBC + WCA
WAB = PV = 10(2 – 1) = 10 J and 5. Given: P = 1.013  105 N/m2,
WBC = 0 ….( V = constant) Q = 54 cal = 54  4.18 J
m = 0.1 g = 10–4 kg,
 By first law of thermodynamics,
Vsteam = 167.1 cc = 167.1  10–6 m3
Q = U + W
U = 0 (Process ABCA is cyclic) Initial volume of water at 100 C,
 Q = WAB + WBC + WCA mass 10 4
Vwater =  = 10–7 m3
 5 = 10 + 0 + WCA  WCA = – 5 J density 103

219

MHT-CET Triumph Physics (Hints)


From Mindbender 3(ii) 12. In ideal gas, by definition, there is no interaction
Change in internal energy, between particles, no intermolecular forces. So
U = mL – P(Vsteam – Vwater) pressure change at constant temperature does
= Q – P (Vsteam – Vwater) not change internal energy i.e., U = 0.
= (54  4.18) – 1.013  105 But real gases have interactions i.e., attraction
 (167.1  10–6 – 0.1  10–6) between molecules at low pressure and
= 225.72 – 16.92 repulsion at high pressure. Thus, even if
= 208.8 J temperature is constant, change in pressure
changes their internal energy.
CP 5  U  0
6. For a monatomic gas,  = =
CV 3
13. Q = U + W
Using Shortcut 2(i)  Q = 0 – 150 J
U CV 3

ns
= = Thus, heat has been given by the system.
Q CP 5
14. In isobaric process,
 Fraction of heat energy to increase the internal Q = nCPT and W = n (CP  CV) T
energy be 3/5. W C
 =1 v

io
5 Q Cp
7. For monoatomic gas,  =
3 Cp
Using Shortcut 2(ii) but  = = 1.4

at
Cv
W 1 3 2
1 1  300 1
Q  5 5  1
Q 1.4
8. Considering anticlockwise sense of area, 300  1.4
PV
WBCOB = – Area of triangle BCO   0 0
lic  Q=
0.4
2 = 1050 J
Considering clockwise sense of area,
15. In adiabatic process, exchange of heat Q = 0
P0V0
ub
WAODA = Area of triangle AOD =
2 16. From Shortcut 3
 Total work = 0 For an adiabatic process,
P  T/1
9. QABC = QAC + WABCA Given that, P  T3
P

In this case, 
1  = 3   = 3 3
WABCA = Area of PV graph =  AB BC  1
2
3
  2 =  3   =
et

1
 500 = QAC +  (4  10  2  103)
4
2
2
 QAC = 500  40 = 460 J 17. T1 = 27 + 273 = 300 K
T2 = 627 + 273 = 900 K,  = 1.5
rg

10. For cyclic process, heat absorbed For an adiabatic change,


1
= work done = Area = (P1  P2 )(V1  V2 ) T
2 = constant ....(Refer Shortcut 3)
P  1
Ta

11. At S.T.P., 1/ 2
 P2   T2 
3/ 2
 P2 
1/ 2
 900 
3/ 2

T1 = 273 K and P1 = 1 atm    =   5 


= 
 P1   T1   10   300 
V1
Given that, V2 = and P2 = 50 atm  P2 = 27  105 N / m2
16
Being the same gas compressed, n = constant 18. As the work is done on the system,
Equation of state W = – 32.4 J
P1V1 PV In an adiabatic process, Q = 0
 = 2 2
T1 T2  U = – W = 32.4 J
P2 V2 Now, the internal energy is a state function.
 T2 =  T1 Thus, it does not depend on the path.
P1V1
For second case,
50  V1
=  273 Q = U + W
16  1  V1  13.5  4.2 = 32.4 +W
T2 = 853 K  W = 56.7 – 32.4 = 24.3 J
220

Chapter 04: Thermodynamics


19. In case of adiabatic process, Q = 0 23.
 W = – U
Thus, work done by the air will be,
P2 V2  P1V1 adiabatic
 W = –
 1
For an ideal gas at S.T.P., isothermal
P1 = 105 Pa
For adiabatic process, PV = constant

 V1  V/2 V
 P2 = P1   
 V2  Work done = area under curve
1
1.4
105 While compressing the gas adiabatically, the
= 105    = Pa area under the curve is more than that for

ns
 3 4.6555
isothermal compression.
V1 = 1 L = 1  10–3 m3 and
V2 = 3 L = 3  10–3 m3 24. V1  V1/2
 105 
 3  10 3   105  1  10 3 

io
 adiabatic
4.6555
 W = –  
1.4  1
isothermal
 0.6444  1  102

at
=–
0.4
= 88.9 J
The nearest answer in the option is 90.5 J. Thus, isobaric
(C) is the correct option.
lic
20. For isobaric process, work done,
w1 = P(V2 – V1) V1/2 V1
= P(2V – V) ...( Volume is doubled)
ub
Work done = area under curve
For isothermal process, Wadiabatic > Wisothermal > Wisobaric
 V2  For isochoric process, W = 0
w2 = nRT ln  
 V1  25. Figure shows the PV diagram of the given
P

 2V  process.
= PV ln  
 V 
adiabatic
w2 PVln  2  P3

et

= isothermal
w1 PV P1
 w2 = w1 ln 2
21. To raise the temperature of a gas, the amount of V2
rg

V1
heat that must be supplied From figure it can be said that,
At constant volume area under the curve for isothermal expansion is
Qv = mCvT lesser than area under the curve for adiabatic
Ta

At constant pressure compression.


Qp = mCpT  The work done is negative.
Qv Cp Also, pressure P3 > P1
 
Qp Cv 26. For step-1: Isothermal Expansion
For diatomic gas, P
PV = P2(2V) or P2 =
Cp 7 2
= 1.4 or For step-2: Adiabatic Expansion
Cv 5
Qv 1 5 P2 V2 = P3V3
 = = P 5 5
Qp 1.4 7  (2V) 3 = P3 (16 V) 3
2
22. 1st process is isothermal expansion and 2nd 5 5
P  1 3 P
P3 = 
P 2V  3
process is isobaric compression, both of these   =    =
are shown correctly in option (D). 2  16V  2 8 64

221

MHT-CET Triumph Physics (Hints)


27. P 80 180 
34. Given,  =  + =
Pf 100 100
9
 = 
Pi 5
Adiabatic
also, TH = 100 K (say)
it is increased by 25 %
Isothermal  TH = 125 K
TC = TC
 T T TH
Vi V0 V  = H C
 TH 
TH  TC
28. In process – I, volume is constant.
5 100  TC 125 100  TC 125
 The process is isochoric.   = 

ns
9 100 125 TC 125  TC 100
In process – II, P and V are related non-linearly
and also temperature is changing.  625  5TC = (900  9TC)  1.25
 The process is adiabatic. = 1125  11.25 TC
In process – III, temperature is constant.  6.25 TC = 500

io
 The process is isothermal.  TC = 80 K
In process – IV, pressure is constant. TH  TC 125  80
 The process is isobaric.   =  100 = 100
TH 125

at
TC   = 36 %
29. Efficiency,  = 1 –
TH
TC 1
T  T 100 35. =1– =
= H C
TH 373
lic
= 0.268 = 26.8%
1 
TH 6
TC
 =1– = ….(i)
 T  TH 6 6
η = 1  C   100 = 1 
300 
30.   100 = 40%
 TH   500  When T2 is reduced by 62 C,
ub
2 1
TC 1 T ' = 2 ×  = =
31. =1– i.e., =1– C 6 3
TH 10 TH

1  T  62 
=1– C
TC 1 9 T 10 3 TH
 =1– =  H =
P

TH 10 10 TC 9
TC  62 2
T   =
 W = QC  H  1 , TH 3
 TC  5  TC  62  2
et

 = ….[From (i)]
 10  6  TC 3
10 = QC   1
 9  6  310
 TC = 310 K and TH = = 372 K
1 5
rg

10 = QC    QC = 90 J
9
36. Efficiency of Carnot engine is,
T 390 20 TC TH  TC
32.  = 1 – C = 1–  =1 
Ta

TH 590 59 TH TC
20 If both TH = TH + x
Heat used in work, W = Q =  500
59 And TC = TC + x
= 169.49 kcal
TH  TC (TH  x)  (TC  x) TH  TC
Heat delivered to sink = Q W = 330.51 kcal   =  =
TH TH  x TH  x
33. TH = 400 K, TC = 300 K As denominator of  is greater that of 
For heat engine   < 
W TH  TC
 
Q TH QH T
37. = H
400  300 QC TC
 W=Q
400 300 500
 =
W  400 150 TC
 Q= = 800  4 = 3200 J
100  TC = 250 K
222

Chapter 04: Thermodynamics


100  80  Now, change in internal energy,
38. A =   × 100 = 20%
 100  f
U = nRT
40  20  2
B =   × 100 = 50% Where,
 40 
f = degrees of freedom of a monatomic gas = 3
80  60 
C =   × 100 = 25%
As the gas expands adiabatically, the internal
 80  energy decreases.
60  40 
D =   × 100 = 33.3% 
3
U =   2  8.3  111
 60  2
 Option (A) has least efficiency.  U = 2.76 kJ
39. Given: K = 5; 43. As the room works as a source here, the heat
TC = temperature of source (freezer) delivered will be more. Hence, the amount of

ns
= – 20 ºC = – 20 + 273 = 253K heat delivered to the room by refrigerator is
TC given by,
K= QH TH
TH  TC 
253 W TH  TC

io
5= Where, TH = room temperature = t1C
TH  253
TC = temperature inside the refrigerator
 5TH – 1265 = 253
= t2 C

at
1518
TH = = 303.6 K = 30.6 ºC ≈ 31 ºC QH t1  273
5  
W (t1  273)  (t 2  273)
QC TC Q H t1  273
40.   
W TH  TC
here, TC = 4 C = 277 K, TH = 303 K
lic W t1  t 2

QC = 600 cal 44. According to second law of thermodynamics,


600 277 efficiency cannot be > 1. Thus, only option (B)
  shows the correct correlation.
ub
W 303  277
600 45. Given: t = 60 – 30 = 30 C
 W= = 56.31 cal
10.65 As the pressure remains constant,
W 56.31 For isobaric process, work done is
 P= =  4.2 J W = P(V)
P

t 1s
Due to thermal expansion,
 P = 236.5 W
M
V = V0 (t ) = (t )
41. Change in internal energy, 
et

P2V2  P1V1 2  6  5 4 1.5


U = = =  5  105  30
 1 7
1 9 103
5
= 250  109 m3
rg

….    for idealdiatomicgas 


7
 W = 105  250  109 = 25  103 J
 5 
=  20 kJ 46. Given TH = 27 C = 300 K
TC = 0 C = 273 K
Ta

42. Given: T1 = 27 C = 273 + 27 = 300 K, QC TC


V2 coefficient of performance K = 
=2 W TH  TC
V1 here, QC = mL
For adiabatic process, mL TC
TV1 = constant  
W TH  TC
 T1 V1 1 = T2 V2 1 2  333  103  (300  273)
5  W=
 = for monatomic gas. 273
3 2  27  333 3
 1 5
1
2 1  W=  103 = 65.87  10 J
T2 V   1 3  1  3  1 3 273
 =  1 =  =   =   = 0.63
T1  V2   2  2  4 47. During adiabatic expansion, temperature of the
 T2 = T1  0.63 gas decreases.
 T2 = 300  0.63 = 189 K  TH V1 1 = TC V2 1

223

MHT-CET Triumph Physics (Hints)

As the gas is diatomic,  = 1.4 As heat is extracted from source, it is supplied


TC  V1 
 1 1.4 1 to the system (here the gas) and hence it will be
 1  1
   =   = positive. The heat Q is positive for the path
TH  V2   32  4
DAB.
T 1 3  QDAB = UDAB + WDAB
  = 1 – C = 1 – = = 0.75.
TH 4 4
= n Cv T + Area under curve DAB
3R  4 P0 V0 P0 V0  9 3 
48. U = nCv T= n     P0 V0 = n  R  2p0 v0
2  nR nR  2  2  TB  TD 
 2 P0  P0  3P0 V0 3
W =  2 V0  V0  = ....( Cv = R for monatomic gas)
2 2 2
Q = W + U = 6 P0V0 3
=  (nRTB  nRTC) + 2p0v0

ns
49. 2
P
3
=  (4p0v0  p0v0) + 2p0v0
2
A B
2p0
…( PV = nRT)

io
=   2  p0v0
9
p0 C 2 
D

at
13
= p0v0
V 2
v0 2v0 lic Hints to Evaluation Test

1. W = 105  (3  1) = 2  105 joule i.e., TC = 335 K = 62 C


ub
3
2. Change in internal energy, and TH =  335  502 K
2
U = nCvT = nCv (T2 – T1) …(i)
 TH = 502 – 273 = 229 C
 Option (A) is correct.
Using Q = U + W
P

7. As V2 = V1/8
 U =  W ....( Q = 0 in adiabatic process) V1
 =8
V2
Option (B) is correct.
5
et

In equation (i) if (T2  T1) = 0, then U = 0 For monatomic gas,  =


 Option (C) is also correct. 3
 (D) is correct. For adiabatic process, TV – 1 = constant
2
T2 5 1
 = (8)–1 =  (2) 3  3 = (2)3  3 = 4
rg

3. P3 = k V5
5
T1
 P = k V3  T2 = 4T1
But, for adiabatic process, PV = constant As T1 = 17 C = 290 K
Ta

 CP / CV = 5/3  T2 = 4  290 = 1160 K


= 1160 – 273
TH  TC
5. Here,  = = 887 C
TH
1 T T 8. For the given line AB, V and T both increase.
 = H C  Using PV = nRT, we get
3 TH
P (kT) = nRT ( V = kT here)
 3TH  3TC = TH
3 nR
 TH = TC or P = = constant
2 k
3 T  (TC  335) Therefore, in P-V diagram the corresponding
and = H line will be a straight line parallel to X-axis (V-
3 TH
axis) such that V is increasing.
3 / 2TC  TC  335
 1= For the given line BC, volume is constant but
3 / 2TC temperature is decreasing.
224

Chapter 04: Thermodynamics


nRT QBC = CPT for 1 mole of the gas
 P=
constant  T
= CP  T  
or P  T (decreasing)  n 
In P-V diagram, the corresponding line will be a  n 1
straight line parallel to Y axis (P axis) with QBC = CPT  
 n 
decreasing P.
 Qnet = QAB + QBC
For the given line CA, temperature is constant
with volume decreasing 1 n   n 1
= CVT   + CPT  
nRT  n   n 
 P= i.e., PV = constant T
V = (CV  nCV + nCP  CP)
 In P-V diagram, corresponding line is a n
hyperbola with P increasing. T
= {(n(CP  CV)  (CP  CV)}

ns
n
10. As a and d are two points on the same adiabatic T T
path, = (nR – R) = (n – 1)R
n n
 T1 (Va)1 = T2 (Vd)1 = RT (1  n1)
T1 (Vd )  1

io
i.e., = 14. TH = 400 K, TC = 200 K
T2 (Va )  1
For heat engine
Similarly, T1 (Vb)1 = T2 (Vc)1 W TH  TC
 

at
T1 (V )  1 Q TH
i.e., = c  1
T2 (Vb ) 400  200
 W=Q
(Vd )  1 (Vc )  1 400
 =
(Va )  1 (Vb )  1
lic  Q=
W  400
200
= 1200  2 = 2400 J
Vd Vc V V
i.e.  or a  b
Va Vb Vd Vc 15. Given that, Q = 10 J
Work done (W) = Force  displacement
ub
12. Coefficient of performance =32
TC =6J
K=
TH  TC  From first law of thermodynamics,
(273  13) 260 U = |Q| – |W| = 10 – 6 = 4 J

P

5 
TH  (273  13) TH  260 16. Coefficient of refrigerator,
 5TH  1300 = 260 TC 300
K= = =3
 5TH = 1560 TH  TC 400  300
et

 TH = 312 K = 39 C
13. P
rg

P A T
Ta

P T
n
B C
V
AB is an isochoric process
PA PB P P 1 T
  or    TB =  
TA TB T  n  TB n
For 1 mole of the gas,
T  1 
QAB = CVT = CV   T  = CVT   1
n  n   
1 n 
= CVT  
 n 

225
05 Oscillations

Hints

20. vmax = A where  = 2n = 2  100


Classical Thinking  vmax = 0.5  2 (100) = 100  m/s

ns
6. F = – kx ma = –kx 21. v2= 9(16  x2)
x
=    = constant
m
  v  3 16  x 2
a  k
kg m [M 1 ] Comparing with v =  A2  x2 , we get

io
7. Unit of k = N/m = 2 = kg/s2 = 2
s m [T ]  = 3, A = 4
= [M1L0T2]  vmax = A = 4  3 = 12 unit

at
8. The standard differential equation is satisfied by 22. Here, Assertion is false because, the direction
only the function sint  cost. Hence it of velocity in S.H.M. can be towards or away
represents S.H.M. lic from mean position whereas the displacement
is always away from mean position.
k 4.84
10. = = = 2.22 rad/s
m 0.98 24. Velocity is same. So by using v = A,
A11 = A22 = A33
k  m1
11. =  2 =
ub
m 1 m2 25. Maximum acceleration of S.H.M.,
 = 2A
m1 m
 2= m2 = 1 Maximum velocity of S.H.M.,
m2 4
 = A
P

13. In S.H.M., at mean position, velocity is maximum 2A  A 2 A 2 2


 = = =
So v = A (maximum) A A A
14. a = 2x, at mean position x = 0  Amplitude of oscillation is,
et

So acceleration is minimum (zero) 2


A=

15. Acceleration = 2A is maximum at extreme
position 26. A = 10 cm, T = 4 sec
rg

2 2 
a =  
16. a = 2x  = 2 T 4 2
x
x = 5 cm when t = 0
Ta

17. 2
A is the acceleration of the particle when it  5 cm = 10 cm sin (t + )
is at one extreme point. 1
 sin =
2
18. a = 2x

a a 2  =
 2 = = = 100 6
x x 0.02
 Equation of displacement is
  = 10 rad/s t 
x = 10 cm sin   
19. Since the particle start from x = 0 and have the  3 2
same amplitude but different time periods, they
will meet again at x = 0 where their velocities m 200  103
27. T = 2 = 2
are maximum equal to A1 and A2, i.e. k 80
v1  2 T2 6 = 2 25  104 = 2 5  102
= 1 =  = =2
v2 2 T1 2 3 = 10  102 = 0.31 s
226

Chapter 05: Oscillations

m 49. Comparing given equations with standard form,


28. T = 2 A1 = 10 and A2 = 25
k
A1 10 2

T2
=
m2
=
4m
=2  = =
T1 m1 m A2 25 5

 T2 = 2  2 = 4 s 50. Phase difference between two S.H.M.s,


29. As mg produces extension x, hence  2     
 t  t = t = (1) =
mg  3 2  6 6 6
k=
x 51. Two equations are,
(M  m) y1 = A1 sin (t + 2) and
 T = 2
k y2 = A2 sin (t + 4)
(M  m)x The phase difference,  = 4 2 = 2

ns
= 2
mg Resultant amplitude,
R= A12  A 22  2A1A 2 cos 2 =  (A1 + A2)
30. With respect to the block, the springs are
connected in parallel combination 1

io
 Combined stiffness k = k1+ k2 54. E= m2A2E  A2
2
1 k1  k 2
 n=
2 m 1
55. Total energy = m2A 2 = constant

at
2
31. Force of friction = µmg= m2 A = m (2πn)2 A
1 μg 61. F = kx
 n=
2π A  dW = Fdx = kxdx
38.
lic
For S.H.M., displacement x = a sin  t and
 
W

0
dW  
x

0
 kx dx
acceleration A = 2x sin t are maximum at 1

 W = U =  kx2
t = . 2
ub
2
1
39. Phase change = 2  2 = 4 radian 62. K.Emax = m2A2
2
40. y = Asin(2nt + ). 1
=  1  (100)2 (6  102)2 = 18 J
Its phase at time t = 2nt +  2
P

 63. K.E. = 3  P.E.


41.  = t  =  4
2 1 1
K.E. = m2 (A2 x2) = 3  m2x2
et

 2 2 2
 ==  T = 16 s
8 T  A2 = 4x2 A = 2x
8
 x= = 4 mm
 = tan–1   = tan–1   = tan–1  
b 6 3
42. 2
rg

a 8 4


64. Kinetic energy at mean position,
47. x = Acost 1
K.Emax = mv 2max
a 2
Ta

2 K.E max
 vmax =
O t m
2  16
 vmax = = 100 = 10 m/s
0.32
dx
v  Aωsin ωt
dt 3 A2 16
65. x= A 2  ….(i)
d2x 4 x 9
a = 2  Aω2cosωt 1
dt m 2 A 2
T.E. 2 A2 16
This is correctly depicted by graph in (C).  = = 2 = ….[From (i)]
P.E. 1 x 9
m 2 x 2
48. Displacement and force (ma) are out of phase 2
(= ) in S.H.M. Therefore, the correct graph 80 16
will be (D)  =  P.E. = 45 J
P.E. 9

227

MHT-CET Triumph Physics (Hints)

66. A = 10  102m = 101m


1 Critical Thinking
K.Emax = m2 A2
2
2. F = kx
1
= kA2 3. Acceleration  – displacement and acceleration is
2
1 always directed towards the equilibrium position.
 5=  k  (10–1)2
2 4. Acceleration in S.H.M. is directly proportional
10 to displacement and is always directed to its
 = k k = 1000 N/m mean position.
102
5. For S.H.M., F = kx
l
67. T = 2  Force = Mass  Acceleration  – x
g

ns
F = –Akx; where A and k are positive constants
l g
  = constant 6. As F = – kx | F |  x
T 2 42

69. In the given case, effective acceleration geff. = 0 7. x = 6 cos  3t  

io
 3
 T=
dx  
 = –6 sin  3t   3 and
70. When the pendulum is falling freely with dt  3

at
acceleration g,
d2x 
l  = 6(92) cos  3t  
T = 2 = dt 2  3
gg
d2x
 = – 92x
71. For seconds pendulum, T = 2 s
lic dt 2
l d2x
 2 = 2 8. Comparing given equation with  2 x  0
g dt 2
ub
g 4.9 we get,
 l= = 2  50 cm
2   2 = = 

k p  2n =  n=
76. 2= ,r= 2
m 2m
P

Angular frequency, A  4 2 1 4  (3.14)2


9. amax = A2 = =
 =  2
 r2  T2 0.2  0.2
0.1 4  (3.14)2
et

Fmax = m amax =
k p2 0.2  0.2
= 
m 4m2  Fmax= 98.596 N
80. When two bodies have the same frequency, then 10. x = 0.25 sin (200 t)
rg

one is excited and other vibrates with its natural Comparing with x = A sint,
frequency due to resonance. A = 0.25 m,  = 200 rad/s
 vmax = A = 0.25  200 = 50 m/s
84. When the springs are stretched by the same
Ta

force F, the extensions in springs A and B are x1 11. a = 2x


and x2 respectively which are given by, 8
F = k1x1 = k2x2  = a/x = = 2 rad/s
2
x1 k  vmax = A = 6  2 = 12 cm/s
= 2 ….(i)
x2 k1
12. vmax = A and amax = A2
1 1
Work done, W1  k1x12 and W2  k 2x 22 a max A2 0.64
2 2     = 4 rad/s
v max A 0.16
W1 k x 2
 = 1 . 12 ….(ii)  0.16 = A  4  A = 0.04 m = 4  102 m
W2 k2 x2
Using equation (i) in equation (ii) we get, 13. amax = A2
W1 k k2 k a max 7.5
= 1 . 22 = 2  A= = = 0.61 m
W2 k 2 k1 k1 2 (3.5)2

228

Chapter 05: Oscillations


14. vmax = A and amax = A2 24. Maximum acceleration, amax = 2A
a max 4 Amplitude remaining constant, amax2
 = = = 2 rad/s
 a max 1  1 
2
v max 2 2
 100   1 
2

  =    
15. From the given equation, A = 5 and  = 4,  a max . 2  2   1000   10 
x=3 1
 Ratio of max. accelerations =
 v =  a  x = 4 (5)  (3) = 16
2 2 2 2 10 2
25. Comparing given equation with standard equation,
16. vmax = A

v max 10 y = Asin(t + ), we get, A = 2 cm,  =
 = = 2
A 4 2
 2
Now, v =  A2  x 2  amax = 2A =    2 = cm/s2
2 2
 v2 = 2(A2 x2)

ns
v2 dx 
 x2 = A2 26. v= = 4  cos  t  
2 dt  3
52  
v2 = 4 cos  4   = 4 cos  

io
 x= A2  = 42  = 2 3 cm
2 (10 / 4)2  3 3
1
17. Velocity of a particle executing S.H.M. is given = 4 = 2 cm/s
2

at
by
dv 
v   A2  x2 27. a= = – 42 sin  4  
dt  3
2 A 2 2 3A 2 A 3
 A2   
T 4 T 4 T
lic = – 42 sin

3
= – 42
2
3
= – 2 3 2 cm/s2
18. vmax = A
 100 = 10  = 10 rad/s 2 2
28. = = = 100  rad/s,
 v = 2 (A2 x2) T 0.02
ub
 (50)2 = (10)2 (102 x2) A = 2.5 m at t = 0
 25 = 102 x2 Equation of particle performing S.H.M. is given
by,
 x2 = 100  25 = 75  x = 5 3 cm
x = A sin (t + )
P

19. v A 2
 x 2   2 60 2  20 2  113mm / s  2.5 = 5 sin (100 0 + )
2.5 
 = sin  = 30 or
20. A = 50 mm = 50  103m 5 6
et

2 Hence, the correct equation is,


 vmax = A = A 
T 
x = 5 sin  100 t  
2  6
= (50  103)   0.16 m/s
T
rg

29. When particle starts from extreme position,


21. Velocity, v =  A 2  x 2 and x = A cos t …(i)
acceleration = 2x 60
n = 60 r.p.m. = = 1 r.p.s.
Ta

Now given that, 2x =  A 2  x 2 60


 2.1 =  2 2  12  = 3  = 2n = 2 1 = 2
2 2  x = 0.1 cos (2 2)…[From (i)]
 T=  = 0.1 cos 4 = 0.1 m ….[ cos 4  = 1]
 3

22. Acceleration, a = 2x 30. Velocity, v =  a 2  x 2


2
aT  xT 2
 2  4 2
At x = s, let v = v0
   2T    T 
x x  T  T  v0 =  a 2  s 2
It is a constant term for S.H.M. i.e., it does not  v 0 2 = 2(a2 – s2) ….(i)
change with time. Due to blow, the new velocity at x = s,
23. Maximum acceleration, v0
v=
2A = A  4π2n2 2
= 0.01  4  (π)2 (60)2 = 144π2m/s2  v2 = 2(a2 – s2)
229

MHT-CET Triumph Physics (Hints)

 v0 
2
2
 2 2 2 35. T= s , 2A = 4 cm  A = 2 cm
  =  (a – s ) 3
 2
v=A …(Given)
v02
 = 2(a2 – s2) ….(ii)   A 2  x 2  2 x …(Numerically)
4
Dividing equation (ii) by equation (i) 2 A2
 A 2  x 2   2 x 2 x =
1 a2  s2 2  1
= 2 2
 2
2
4 a s A2 4
 x2 = =  1
 a2 – s2 = 4a2 – 4s2  4   4  3  4
2 2

 2  1   1
a2 + 3s2 = 4a2   4
2
 T 
a 2  3s 2  x = 1 cm
 a =
2 2
36. y = a sin t

ns
31. We have, T
v2= 2(A2 – x2) and  = 2x 
a
 a sin
2t
 v2 = 2A2 – 2x2 and 2 = 4 x2 = 2 (2 x2) 2 3
2 1 2t

io
 v2 = 2A2 –  sin
2 2 3
2 2t  2t  1
 v2 + = 2A2  sin  sin   t  s
2 3 6 3 6 4

at
v2 2 4
 + 4 2 =1 38. 2A = 4 cm  A = = 2 cm
A
2 2
A 2
 v 
2
  
2 4π2
amax = A2 = A  2
 
 A
 +  2  =1
  A
lic T
which is an equation of an ellipse. A 2 1
 T = 2π  2 π  = 2π  = 2 s
a max 2π 2 π
32. In simple harmonic motion,
ub
y = A sint and v = A cost. From these 2 2  rad
39. = = =
y2
v 2
T 12 6 s
equations, we obtain  = 1, which is an
A2 A22 
 2 = 4  sin t1  ….(For x = 2 cm)
equation of ellipse.  6 
P

2   
33. For S.H.M., v =  A 2  x 2  = sin t1   t1
4 6 6 6
2A1
v1 = v0 = 1 A12  0  1A1   t1 = 1 s
T1
et

Similarly, for x = 4 cm, it can be shown that


2A 2 t2 = 3 s
v2 = 2 A 22  0  2 A 2 
T2 So time taken by particle in going from 2 cm to
1 extreme position is t2 t1 = 2 s. Hence required
rg

Given that, A2 = 2A1 and T2 = T1


3 1
ratio will be .
v 2 2A 2 T T A 2
   1  1 2
v1 T2 2A1 T2 A1 40. In S.H.M., velocity of particle also oscillates
Ta

v2 simple harmonically. Speed is more when the


 = 3  2 = 6  v2 = 6v0 particle is near the mean position than when it is
v0
near the extreme position. Therefore, the time
v max A
34. v= .…(Given) taken for the particle to go from 0 to will be
2 2
x = a sin t A
 v = a cost and vmax = a less than the time taken to go from to A.
2
a Hence, T1< T2.
 a cos t =
2 41. y = 5sin( t + 4).
1  Comparing it with standard equation
 cost = t =
2 3 y = A sin (t + ) we get,
 3a 2t
 x = a sin = A = 5 m and = t T = 2 s
3 2 T

230

Chapter 05: Oscillations


42. x = A sin t 1
47. k  . Since one fourth length is cut away, the
 6.5 = 13 sin t l
1 3
th
4
sint = remaining length is   . Hence k becomes
2  
4 3
t = sin 1  
1
 times i.e., k =
4
k.
2 3

 t = 48. k1x1 = k2x2 = F
6
2
2t  1 1 F F2
   W1 = k1x12 = k1   =
T 6 2 2  k1  2 k1
T 12 2
1
 t=  =1s F
Similarly, W2 = 2  W 
12 12 2k 2 k

ns
 time required for travelling from x = 6.5 to
 W1> W2 k1<k2Reason is true.
x = 0 is t = 1 s
1 1
 time required for x = 6.5 to x =  6.5 is 2 t = 2 s  Assertion, W1 = k1x2 and W2 = k2x2
2 2

io
43. Comparing the equation with x = A sin t, we  W2> W1
get,  Assertion is false.
 = 20  2n = 20  n = 10 Hz
49. T  l,

at
44. Comparing with x = A sin (t + ) we get,
 The effective lengths have the relation,
1
 =  2n =  n = lsitting>lstanding (T)Sitting> (T)Standing
2
50. From the graph, T = 0.04 s

1
n per min =  60 = 30 per min
2
lic 1 1
 f= = = 25 Hz
T 0.04
45. As it starts from rest, we have,
x =Acost. At t= 0, x = A 51. v1max = v2max
ub
When t= , x= A–a and A1 ω2 k2 m
 A11 = A22    
Whent = 2, x = A –3a A 2 ω1 m k1
A – a = Acos ….(i) 1

Aa A1  k 2  2
 cos =   
P

A A 2  k1 
 A – 3a= Acos2  ….(ii)
1 2
A  3a 52. W1 = kx
 cos 2 =  2
et

A
1
W2 = (2k)x2 = 2.  kx 2 
 As, cos2= 2cos2– 1, 1
2 2 2 
A  3a Aa 
  2  1   W2 = 2W1
rg

A  A 
A  3a 2A 2  2a 2  4Aa  A 2 5T T 5
   53. T   
A A2 4 T 4
 A2 3aA = A2 + 2a2 4Aa Here, the hanging mass performs S.H.M.
Ta

 a2 = 2aA  A = 2a M
With T = 2 and
Now, A a = Acos….[From (i)] k
1 Mm
cos= T = 2 
2 k
2π π
 τ  T6

T Mm k
T 3  
T k M
46. F = kx 5 Mm
 
 mg = kx m kx 4 M
m1 k x M  m 25
 = 1 1  
m2 k2 x2 M 16
4 k 1 m 9
 =  x2 = 3 cm  9 M = 16 m  
6 k / 2 x2 M 16

231

MHT-CET Triumph Physics (Hints)


54. F = k x (in magnitude) 61. r = 10 cm for the particle performing U.C.M.
f 0.1  10 Now, projection of U.C.M. along any diameter
k=  = 10 N/m
x 0.1 of the circle is an S.H.M.
Now, period of oscillations of the system, Hence, in the given example,
m 0.1 1 A = r = 10 cm
T = 2  2  3.14   6.28 
k 10 10 
 T = 0.628 s 62.  = 10t 
2
55. For the given figure  41
substituting t = 2 = 20  = 
1 k eq 1 2k 2 2
f=  ….(i)
2 m 2  m 63. Equation of linear S.H.M.,
If one spring is removed, then keq = k x = 8 cos (12t)

ns
1 k 
 f= ….(ii)  x = 8 sin(12t + )
2 m 2
 From equations (i) and (ii),
f
 2  f 
f 
 Initial phase angle = rad
f 2 2

io
1 64. x = A sin (t + )
56. T  T1 : T2 : T3
k  +5 = 10 sin (2 0 +) = 10 sin 
1 1 1 1 

at
 = sin1 
= : :  1: 2 : 5  1  1 
  = sin   = 6
k k/2 2k 2  10  2
57. System is equivalent to parallel combination of 65. y = 10 sin (20 t + 0.5)
springs
 keq = k1 + k2 = 400
lic Comparing with equation y = A sin (t + )
we get,
m 0.25 
 T = 2 = 2 = s initial phase  = 0.5 rad
k eq 400 20
66. y = 5 sin (t + 4)
ub
m x Comparing with standard equation,
58. mg = kx 
k g y = A sin (t + )
x 2 2  A = 5,  = 4
= 2  9.8  10 =
m
 T = 2 = 2 s
k g 9.8 10
P

67. x = A sin t
59. Extensions in springs are x1 and x2 then 2 t
 2.5 = 5 sin
k1x1 = k2x2 and x1 + x2 = A 6
2 t  1
et

k 1x1
 x2   = or t = s
k2 6 6 2
k 1x 1 Phase difference corresponding to 6 s is 2.
 x1  A
k2 1
rg

So, phase difference corresponding to s


k 2A 2
 x1  2 
k1  k 2 is i.e.
12 6
Ta

60. In series combination


2 2 
2k1 68. = = =
k1 2k1 T 12 6
k1
Using v = A cos (t +) we get,
 2
6.28 = 24 cos    
k2 6  6 
k2
1 
 = cos    
m
2 3 
m

+  = cos–1  
1 1 1 1
  
k s 2k1 k 2 3 2
1  
 1 1  +=
ks =    3 3
2k
 1 k 2
 =0
232

Chapter 05: Oscillations


2 2   R2(cos2 + sin2)
69. = = =
T 6 3 = A12 + A22 cos2 + 2 A1 A2 cos  +A22 sin2
1 
c
 R2 (1) = A12 + A22 + 2 A1 A2 cos 
t= s,  =  
2 6  R = A12  A 22  2A1 A 2 cos 
Equation of S.H.M. is,
76. x1 = A1 sin (t + 1) and x2 = A2 sin(t + 2)
x = A sin (t + )
 x = x1 + x2
 1  2
= 10 sin     = 10 sin   = A1 sin (t + 1) + A2 sin (t + 2)
3 2 6  6 
= A1[sin t cos 1 + cos t sin 1]
= 10 sin 60 + A2 [sin t cos 2 + cos t sin 2]
3 = sin t (A1cos 1 + A2 cos 2) +
= 10 
2 cost (A1 sin 1 + A2 sin 2)

ns
= 5 3 cm Put A1 cos 1 + A2 cos 2 = A cos
 A1 sin1 + A2 sin2 = A sin
70. = rad  x = A cos sint + A sin cost
2
= A sin(t + )
y = Asin(t + )

io
Hence resultant is S.H.M. with same period T.
2
 y = Asin  
t  
 T  77. R= A12  A 22  2A1A 2 cos 
2 
y = 0.5 sin 

at
 t  =  
 0.4 2 4 2  32  2  4  3cos   
3 6

 y = 0.5 sin  5 t   = 0.5 cos 5t 25  12 3
 2 =

71. Resultant amplitude = 32  42 = 5


lic 78. Initial phase of resultant motion is given by,
 a1 sin 1  a 2 sin 2 
 = tan–1  
72. If x1 = A1 sin  t and x2 = A2sin(t + 0)  a1 cos 1  a 2 cos 2 
= A2 sin  t
ub
 1 4 3 
 3  
2  = tan–1  3  4 3 
But A1 = A2
 x2 = x1 = tan–1  2  43 3 
 3 1  
This represents a straight line.  3  4 
 2 2
P

73. The given relation can be written as,


79. x = 8 sint + 6 cost
x = 4 cos  t + 4 sin  t

Resultant amplitude 42  42 = 4 2 = 8 sin t + 6 sin  t  
 2
et

dy1    R = 82  6 2 = 10 cm
74. v1 =  0.1  100 cos  100t  
dt  3
80. In S.H.M., a = 2x

v2 = 2  0.1 sin t  0.1 cos  t  
dy
Acceleration is always opposite to displacement.
rg

dt  2
Phase difference of velocity of first particle with 81. f = F = kx and
respect to the velocity of 2nd particle at t = 0 is 1
P.E. = V = m2x2
   2
Ta

  1  2   
3 2 6 V m2A 2
For option A :  x  x  0
F 2kx
75. x1 = A1 sin t and
Hence option (A) is incorrect.
x2 = A 2 sin (t + )
F kx
 x = x1 + x 2 For option B : x x0
V m2 x 2
= A 1 sin t + A 2 (sint cos  + cos t sin )
Hence option (B) is incorrect.
= A1 sint + (A2sin t cos +A2 cos t sin ) 1
= sin t (A1 + A2cos ) + cos t (A2 sin ) 2V 2  m2 x 2
For option C : x 2 x
Let R cos  = A1 + A2 cos  F  kx
R sin  = A2 sin  m2 x 2
R = amplitude of resultant = x
 m2 x
 R2 cos2 + R2 sin2
=x+x=0
= (A1 + A2cos )2 + (A2 sin )2
Hence option (C) is correct.
233

MHT-CET Triumph Physics (Hints)


F  Kx 2P.E.1 2P.E.2 2P.E.
For option D : x x0   
2V 1 k k k
2  m2 x 2
2
P.E.1  P.E.2  P.E.
Hence option (D) is incorrect
82. K.E. = P.E. 1 1 2
87. K.E. = P.E.  mv2 = kx
1 1 2 2
 m2 (A2 x2) = m2x2
2 2 1 1
 m2(A2 x2) = m2x2
A 2 2
 A2 x2 = x22x2 = A2  x =
2  A2 x2 = x2
 x = 0.71A A2 x 1
 x2 =  =
2 A 2
1
83. K.E. = m2 (A2 x2)

ns
2 88. y = 0.05 sin 4(5t + 0.4)
1
P.E. = m2x2  y = 0.05 sin (20t + 1.6)
2 Comparing this with standard equation,
At extreme position, x = A y = A sin (t + ) we get,

io
 K.E. = 0 and P.E. =
1
m2A2 A = 0.05,  = 20
2 1 1
T.E. = m2A2 =  0.1  (20)2 (0.05)2
At mean position, x = 0 2 2

at
1 1
K.E. = m2A2 and P.E. = 0 =  101  4  102  2  25  104
2 2
K.E. increases and P.E. decreases. lic = 0.05 2 J
1 89. Comparing the given equations with the
84. P.E. = m2 x2 = 2.5 J
2 standard form we get,
1 A
2
A A1 = 4, A2 = 5, 1 = 10
 m2   = 2.5 .…[ x = ]
2 2 2 1
ub
E= mA22 E  (A)2
1 A2 2
 m2 = 2.5  (A11)2 = (A22)2 A11 = A22
2 4
1  4  10 = 5  = 8 unit
 m2 A2 = 10
P

2  2 A2 
1 A  2 
1
Total energy of system = m2A2 = 10 J K.E. 2 m  2
 A 2
 x 2
 n 

2 90. = = = n2  1
P.E.  1 2 2 A 
2

 m x   2
et

2E 2  n 
85. K.E. =
3
1
m2 (A2 x2) =  m2 A 2 
1 1
1 91.
K.E. 2 m (A  x )
2 2 2
A2  x 2 x2 2 4 2 
rg

= = = 1 
T.E. 1 A2 A2 A2
m2 A 2  A2 x2 =
2 4
 2E  3A 2 3A
  x2 =
Ta

 3  =1 x
2
 x=
 4 2
E A2
x2 2 1 A 1
 =1 = x= 92. K.E. = m2A2 cos2 t,
A 2
3 3 3 2
1
1 2 2P.E1 P.E. = m2A2 sin2 t
86. P.E.1 = kx  x = 2
2 k 1
1 2P.E 2
K.E.  P.E. = m2A2 [cos2 t  sin2 t]
P.E.2 = ky2 y = 2
2 k 1
1 = m2A2 .cos2t
and P.E. = k(x + y)2 2
2  Angular frequency = 2
 x+y=
2P.E 2    T
 T =   =2s
k 2  2

234

Chapter 05: Oscillations


93. Force increases linearly. i.e. F  x  If frequency of particle is 10 then the kinetic
F x energy of the particle will vary with frequency
 
F x 2  10 = 20
F A
=     = – 2
4 96. Potential energy of particle performing S.H.M. is

F 2  A 1
given by, P.E. = m2 x 2 , i.e., it varies
x 2
 F = – 2F  = 2
x parabolically such that at mean position, it
Potential energy, P.E.  x2 becomes zero and maximum at extreme positions.
2
P.E.  x  97. The relation for kinetic energy of S.H.M. is
 =   = (2)2 = 4
P.E. x given by
 P.E. = 4P.E. 1
= m2  A 2  x 2  ….(i)

ns
Speed of particle is given by 2
v =  A2  x 2  v  A2  x 2 Potential energy is given by
A 1
At x = , = m2 x 2 .…(ii)
4 2

io
2 Now, for the condition of question and from
v  A 2    =
A 15
A equations (i) and (ii),
4 16
1 1 1
m2  A 2  x 2    m2 x 2

at
A
 At x = , 2 3 2
2 4 1 3
2 or m2 x 2  m2A2 or x 2  A2
A 3 6 2 4
v  A2    = A

v
2  4
lic so, x 
A
2
3 = 0.866 a = 87% of amplitude.
3 16 4
 =  =
v 4 15 5 1 2
98. T.E. = m2 A2 = 1 m  2   A 2
4 2
ub
2  T 
 Velocity at x = A/2 may be  v
5 1 42A2 22mA2
Kinetic energy will be = m 
2 T2 T2
2
K.E.  v  4
=   = = 0.8 A
P

K.E.  
v 5 99. x=
2
 K.E. = 0.8 K.E.
1
W = m2A2
94. Total energy of a particle executing simple 2
et

harmonic motion is constant.



1
K.E. = m2 A  x
2
2 2
 
1
95. K.E. = k(A2 – x2) 1  A2  3
2 = mω 2  A 2    mω A
2 2
rg

As x = A sin (t + ) 2  4  8
1 31 2 2
 K.E.= k[A2 – A2 sin2 (t + )] =  mω A 
2 4 2 
Ta

1 3W
= kA2 [(1 – sin2 (t + )] =
2 4
1 1
= kA2 cos2 (t + ) ….(i) P.E. = mω2 x 2
2 2
1  cos 2 1 A2 1 1
As cos2= ,  mω2   mω2A 2  W
2 2 4 8 4
cos2 (t + ) = 1  cos 2( t   )
2 1 1
100. K.E. = mv 2 = mA 22 cos 2  t
 Eq. (i) becomes 2 2
1 1  cos 2(t  )   1  cos 2  t 
K.E. = kA2   =
1
m2 A 2  
2  2  2  2 
 Kinetic energy of particle varies with two times hence kinetic energy varies periodically with
of frequency of particle. double the frequency of S.H.M. i.e. 2f.
235

MHT-CET Triumph Physics (Hints)


101. K.E. at mean position 108. T cos  = mg
3
1 1 mg 50 10 10
= m2(A2 0) = m2A2  T = m2l = = =1N
2 2 cosθ 0.5
2
A 1 A 1
P.E. at x =  m2    m2A 2 109. Restoring force = |– mg sin |
2 2 2 8 = 200  10–3 10  sin 30
 The required ratio 200  102
=
1 2 2 2
 m A 
=  2  = 4:1 =1N
1 2 2
 m A  110. Period of simple pendulum,
8 
l
T = 2
102. T.E. in S.H.M. = K.E.max = P.E.max. Here, the g

ns
maximum kinetic energy of the oscillator.
l
1 Now, 2T = 2
K.E.max is kA2 g
2
T l
1  = l = 4l

io
=  2  106  (0.01)2 = 100 J 2T l
2
But T.E.  100J. l
111. T = 2

at
P.E. at equilibrium position = 160–100 = 60 J. g
 P.E.max = 100 + 60 = 160 J Te gm ge / 6 1
   =
1 Tm ge ge 6
m 2 x 2
103.
P.E.
P.E.max
= 2
1
lic  Tm = 6Te  clock becomes slower.
m 2 A 2
2 112. h = 10 cm = 10  102 m = 0.1 m
1 x2 A According to the principle of conservation of
 = 2x=
ub
4 A 2 1
energy, mv2 = mgh
2
104. x = 0 at mean position,
or v = 2gh = 2  9.8  0.1 = 1.4 m/s
1
T.E. of S.H.M. = m2A2
P

2 l 98 2
1 113. T = 2 = 2 =
 25 =  0.5  A2 2 g 980 10
2
2 2
 2A2 = 100 A = 10 = vmax  = = = 10
et

T 2 / 10
 The particle in S.H.M. has maximum velocity
 vmax = A = 10  2 10 = 20 cm/s
when it passes through mean position.
 v = 10 m/s 114. Linear momentum will be maximum, if velocity
rg

of bob is maximum.
3 In S.H.M, vmax = A ….(i)
105. K.E. =  T.E.
4 1
T.E. = m2A2 = E
Ta

1 3 1 2
 m2 (a2 x2) =  m2a 2
2 4 2 2E
2 2 2 = 2A2 = v 2max [From equation (i)]
 4 (a  x ) = 3a which on solving gives m
a 2E
a =  2x or x =   vmax =
2 m
Linear momentum,
106. In vacuum, the bob will not experience any 2E
frictional force. Hence, there shall be no Pmax = mvmax = m = 2mE
m
dissipation. Therefore, it will oscillate with a
constant amplitude. l
115. T = 2 
g cos
107. Time period of simple pendulum
(T = 2 l / g ) is independent of the amplitude 1 1
= 2 = 2
of vibration, when amplitude is small. 9.8  cos 60 9.8 1 / 2

236

Chapter 05: Oscillations

2 1 10 l
= = = 120. T = 2
9.8 4.9 49 g
1
=  3.16 = 0.45 s l l
7  T = 2 = 2
g 4g
g
116. Period of a second’s pendulum is 2 s. 5 5
It will perform 100 oscillations in 200 s 5 l 5
 T = 2 = T
117. Function of wrist watch depends upon spring 4 g 2
action so it is not affected by gravity but
121. T  l . Time period depends only on effective
l
pendulum clock has time period, T = 2 . length. Density has no effect on time period. If
g
length is made 4 times, then time period

ns
During free fall, effective acceleration becomes becomes 2 times.
zero. Hence time period comes out to be infinity
122. n1: n2 = 7:8
i.e. the clock stops. Suppose at t = 0, pendulums begins to swing
118. Let T1 and T2 be the time period of vibrations of simultaneously.

io
pendulum A and B respectively. If n1T1 = n2T2,
n1 T l2
Then, T1 = 2
l1
and T2 = 2
l2  = 2 =
n2 T1 l1

at
g g
2
T1 l1 1.69 13 l1 n  8
2
64
 = = =  =  2 =   =
T2 l2 1.44 12
lic l2  n1  7 49
If the two pendulums go out of phase in time t,
123. le = 1 m, gm = g/6
then in time t, if pendulum A completes n Time period of second’s pendulum is 2 s
vibrations, the pendulum B will complete Te = Tm
(n + ½) vibrations.
ub
le l
 t = n T1 = (n + ½) T2  2 = 2 m
ge gm
T1  n  1 / 2  13
   le 1 g 1
T2 n 12  lm =  gm =  = m
ge g 6 6

P

12n + 6 = 13n or n = 6
1 1 l
 n+ = 6 + = 6.5 124. T = 2
2 2 g
et

4  2l 4 2
119. T1 = T  T2 = where = constant
g g
T1 l1 1 1
= = = ….(i)  T2  l
T2 l2 16 4
rg

dl
2  
dT
x1 = A sin 1t and x2 = B sin 2t   100  =  100
 T  l
They are in phase after time t and phase dT 1  dl 1

difference is 2   100 =  100  =  (2) = 1 %
T 2  l  2
Ta

 1t – 2t = 2
 There is change of 1% per second
 2 2   In a day, there are 24  60  60 = 24  3600 s
    t = 2
 T1 T2  24  3600 1
 = 24  36 = 864 s
1 1 100
   t = 1
 T1 T2   There will be change of 864 s per day.
t  T1  l
 1   = 1 125. T = 2 T l ….(i)
T1  T2  g
t  1 169 l1
 1   = 1 .…[From (i)] l2 = l1 + 69% l1 = ….[Given]
T  4 100
t 3 4 l2 169
  =1t= T  =
T 4 3 l1 100

237

MHT-CET Triumph Physics (Hints)

T2 l2 169 132. When a little mercury is drained off, the


 = = ….[From (i)] position of c.g. of ball falls (w.r.t. fixed end) so
T1 l1 100
that effective length of pendulum increases
T2 13
  hence T increases.
T1 10
l
T2  T1 3 133. T = 2
 100 =  100 = 30 % g
T1 10
T g g 4 2
126. When they are in phase again, the phase  = = = =
T g g
g 5 5
difference is 2. 4
1 1 
 2    t  2 l
 4 4.25  134. T = 2 π ….(i)
g
0.25

ns
 t 1 When the lift moves upwards with acceleration a,
4  4.25
l
17.00 T = 2
 t= = 68 s ga
0.25
T l

io
  2 ….(ii)
l 2 ga
127. T = 2  T  g1/2
g  Dividing equation (ii) by equation (i) we get,
a = 3g

at
1 –1/2
 dT – g
2
l
dT 1 dg 1 135. T = 2
 =– = –  (2%) = 1% g
T 2 g 2

lic
As acceleration due to gravity decreases, the  T
1
g
time period increases.
T g g 2
  
128. l2= l1 + 300 % of l1 = 4l1 ….[Given] T g'  
g 1
ub
l1 1  
  2
l2 4
 T = 2T
Now, T  l GM
136. On earth’s surface, g =
P

T1 l 1 R2
  1 =
T2 l2 4 GM GM 1 GM
 At a height R, gR = = = .
  R +R 
2
T2 = 2 T1 4R 2 4 R 2
et

T2  T1 1
Hence % increase =  100 = 100 %  gR = g
T1 4
1 1 1
L Now, T   T1 and T2 
129. T = 2
rg

g g gR
g g
g 1 gR 1
Let a = (g2 + a2)½  = = = 0.5
T2 g 4
L

Ta

T = 2 a
(g 2  a 2 )1/ 2 137. l2 = 44% of l1l2 = 1.44l
T  l  T1 l1 and T2 l2
130. For a simple pendulum,
T2 l T
T  l or T2l   2  2  1.44  1.2
T1 l1 T1
1 1
 E 2 2
E T2  T1 1.2  1  100
T l  % change in T =  100 
Hence energy will become two times if length is T1 1
halved. = 20%

131. Inside the mine, g decreases. 138. At B, the velocity is maximum. Using
conservation of mechanical energy,
l
Hence from T = 2 , we conclude that T P.E. = K.E.
g 1
increases.  mgH = mv2 v = 2gH
2

238

Chapter 05: Oscillations


139. Time period is independent of mass of bob of 2 2
144.  = = = 2rads1
pendulum. T 1
MR 2 0.2 0.12
140. If t is the time taken by pendulums to come in I= = = 103 kg m2
same phase again first time after t = 0. 2 2
and NS = Number of oscillations made by c c
= =
shorter length pendulum with time period TS. I 10 3
NL = Number of oscillations made by longer  c = 42 103 Nm
length pendulum with time period TL. max = cm
Then t = NSTS = NLTL 
= 42 103
5 20 l 3
 NS  2 = N L  2 (  T = 2 )
g g g 4  3
= 103

ns
 NS = 2NL i.e. if NL = 1, then NS = 2 3
4  31
1 =
141. T = 2 l / g = 2 =2s 3 1000
2 = 0.04133 Nm

io
 l 2 + b2  B
142. I = m   145.  =
 12  I

at
1600  100  3  1.6  105
I = 0.12    10
6
= = 4 rad/s
 12  3  10 6
5 
= 1.7  10 Am2 n= =
4 1
s
T = 2
I
lic 2 2
4
μB =  60 min1
2
I = 38.19 oscillations per minute
  = 2
ub
μB
146. O
2 2
I 
  = 4  
 μB  
4  1.7  105 R
P

 B= = 2  105T
3.4
mgsin
143. With like poles together, the effective magnetic 
Mean
et

moment is (μ1 + μ2).


position
From formula (i),
I The block is performing a circular motion about
T1 = 2 ….(1) O.
rg

 μ1 + μ 2  B
Restoring force at angle  = mg sin  mg
With unlike poles together, the effective …(Assuming  is very small)
magnetic moment is (μ1 μ2).
Torque applied to the block, ext = I
Ta

From formula (i),


= (mR2) 
I ….(i)
T2 = 2 ….(2)
 1 μ2  B
μ  Restoring torque =  (mgR)
Dividing equation (1) by (2), At equilibrium,
ext = restoring
T1  μ1  μ 2  100  50
  =  mR2 = mgR
T2  μ1  μ2  100  50
g
5
 = 
50 1 R
 = =
T2 150 3 g
 = ….( = 2)
25 1 R
 =
T22 3 R
 T = 2
 T2 = 25  3 = 5 3 = 8.66 s g

239

MHT-CET Triumph Physics (Hints)


147. Amplitude of damped oscillator, 2  2   4  A    A 1
 π=A  cos  =  cos    
A  A 0 e   t ;   constant, t = time 24  24  12  3  12 2
A0 24
For t =1 min., = A 0e t  e = 2  A=  24m
2 
A A
For t = 3 min., A = A 0 e 3 =  0 3 = 30 =
1  Path length = 2A = 48 m
(e ) 2 x
 x = 23 155. The stone executes S.H.M. about centre of
R
A earth with time period T = 2 ; where
148. In the first case, A1 = 0 and t1 = 100 T g
3
R = Radius of earth.
A0 100bT
 = a0e
3 156. If first equation is x1 = A1 sin t,

ns
1 x1
 e100bT = = sint ….(i)
3 A1
In the second case, then second equation will be
A2 = A 0e  bt 2 = A0e200bt = A0(e100bt)2 
x2 = A2 sin  t  

io
2
 2
1 A
 A2 = A0   = 0
 3 9 = A2 sin  t cos   cos  t sin  
 2 2 
 The amplitude will be reduced to 1/9th of its 

at
initial value. = A2 cost
x2
149. The initial mechanical energy of a harmonic cost = ….(ii)
A2
1
oscillator at time t = 0 is E1 = kA2
2
lic By squaring and adding equation (i) and (ii)
x12 x 22
But because of damping, its energy at time t sin2t + cos2t = +
 bt A12 A 22
1
becomes E2 = KA 2e m where b is the damping x12 x 22
2 + = 1; This is the equation of ellipse.
ub
E A12 A 22
constant. It is given that at time t, E2 = 1
2 157. Wavelength = velocity of wave  Time period
bt

E1 1
=  bt  E1 = 2 = em λ = 300  0.05  λ = 15 metre
E2  m   E1  According to problem, path difference between
P

e    two points = 15  10 = 5m
   2 
2
bt  Phase difference =  Path difference
 = loge2 
et

m
2 2
mloge 2 0.25 loge 2 = 5 
 t= = 15 3
b 0.05
 t = 5 loge 2 159. U = k|x|3
rg

d(P.E.)
150. For a damped oscillator, the amplitude after time  F= =  3k|x|2 ….(i)
dx
t is, A = A0 et , where  is the damping Also, for S.H.M., x = A sin t and
constant.
Ta

d2x
A0 A  2 x = 0
 = A0e6 ….[ A = 0 ] dt 2
27 27
d2x
1 Acceleration, a =  2 x  F = ma
 e6 = ….(i) dt 2
27
d2x
Let A be the amplitude after 2 minutes = m 2  m2 x ....(ii)
dt
Then A = A0e2 = A0[e6]1/3
3kx
A From equation (i) and (ii) we get,  =
1/ 3
 A = A0  1  = 0 m
 27  3
2 m m
154. As the body starts from mean position,  T= = 2 = 2
 3kx 3k(A sin  t)
v = A cost
1
2  2t   T
 v=A  cos   A
T  T 

240

Chapter 05: Oscillations


160. The rotation of earth about its axis is periodic 
165.  = 30 =
but not to and fro about a fixed point, hence 6
not a simple harmonic motion. Using F = kx, we get
161.  = 1, T = 2 T | Fmax| = kA = m2 A
The effective acceleration of a bob in water 1
 E= m2 A2
  2
= g = g  1   where and  are the density of 1
  = | Fmax|  A
water and the bob respectively. Since the period 2
of oscillation of the bob in air and water are 2E
 A=
given as, | Fmax |
l l 2  3  105
T = 2 and T = 2 respectively, =
g 1.5  103

ns
g
T g g(1   / ) = 4  10–2
 = = = 0.04 m
T g g
2 2
 1  = = =  rad/s

io
= 1 = 1 T 2
 
 The equation of motion is,
T 1 
Substituting, =

, we obtain, x = A sin (t + ) = 0.04 sin  t  

at
T 2 6

1 1
= 1   = 2
2  166. With mass m2 alone, the extension of the spring
l is given by,
162. For body to remain in contact amax = g
 2A = g  42n2A = g
lic m2g = kl ….(i)
With mass (m1 + m2), the extension l is given
g 10 by,
 n2 = = = 25
4 A
2
4  (3.14) 2  0.01 (m1 + m2)g = kl = k(l + l) ….(ii)
ub
 n = 5 Hz The increase in extension is l which is the
amplitude of vibration. Subtracting equation (i)
163. For the graph given, amplitude (A) = 1 cm
from equation (ii), we get,
Time period (T) = 8 s
m1g
2  m1g = kll =
P

 =  Hz k
8 4
Acceleration, a = 2A sin t 167. The coin will leave contact when it is at the
4 2  4 highest point and for that condition
At t = s, a =   1  sin   
et

3 16  4 3 Maximum acceleration = Acceleration due to


2
gravity
  3 2
 a=  sin    A   cm / s 2 g
16  3  32  2A = g  A =
rg

2
164. Total energy of particle performing
1 dT 1 dl
S.H.M. = m2A 2 . Kinetic energy of particle 168. =
2 T 2 l
Ta

dl
1  2   = (t2 t1) = (40 – 20) = (20)
performing S.H.M. = m2 A 2 cos 2  t l
2  T 
1 dl
According to problem, kinetic energy = 75% of  dT = T   
total energy 2 l 
2  1
 m 2 A 2 cos 2 
1 31 2
 t   m A 
2
= T   20
2  T  4  2  2
 2  3  2  3 1
 cos 2   t   cos   t 
= 86400   12  10–6 20
 T  4  T  2 2
2   …[ 1 day = 86400 s]
 
T
t   t  s 5
 T  6 12 = 86400  10  12
1 = 0.864  12
 t= s
6  10.4 seconds
241

MHT-CET Triumph Physics (Hints)


Dividing equation (i) by equation (ii), we get
Competitive Thinking   2
= 
  T
a
1. x = a sin2t = 1  cos 2t  
2 T = 2

2. On comparing with standard equation
d2y 11. Displacement of the particle, x = A sin t
 2 y  0 we get, Velocity of the particle,
dt 2
2 dx
2 = K  = = K T
2 v= = A cos t ….(i)
T dt
K
Given that,
d2y v =  m/s, T = 16 s,
3. For S.H.M., 2   y

ns
dt 2 
 = = rad/s
T 8
4. The distance covered by a particle undergoing
Substituting in equation (i), we get,
S.H.M. in one time period is 4A.
 
=A  cos   2 

io
5. In one complete vibration, displacement is zero. 8 8 
 Average velocity, vavg = 0 A  A 1
 1= cos   = 
6. Acceleration, a =  x 2 8 4 8 2

at
=
a 20
….( a = 20 m/s2, x = 5 m)  A= 8 2m

x 5
12. Refer Shortcut 1(i)
 = 2 rad/s
Period, T =
2
=s
lic y = A sin t
A Asin 2
   t
2 T
7. Acceleration, a = 2x 2 t 1 
  sin 1   
ub
 16  10–2 = 2(4  10–2) T 2 6
 = 2 rad/s T
2 2  t=
T=     3.142 s 12
 2
P

13. The given equation can be written as,


8. Given, (amax= 1.0 m/s2vmax= 0.5 ms1) 1
amax= 2 A =  (A) = vmax v2 = (25  x 2 )
4
a max 1 Comparing with general equation,
et

 = =
v max 0.5 v2 = 2 (A2 x2)
  = 2 rad/s 1 2
 = T= = 4
9. Particle velocities are 2 
rg

v12   2 (A 2  x 12 )
14. When velocity is u and acceleration is , let the
v 22   2 (A 2  x 22 ) position of particle be x1.
On subtracting the relations When velocity is v and acceleration is , let the
Ta

v12  v 22 = 2 (x 22  x12 ) position of particle be x2.


v12  v 22
If  is the angular frequency then,
=  = 2x1
x 22  x12
and = 2x2
2
As  = we get,   +  = 2(x1 + x2) ….(i)
T
Also, velocity of particle at particular instant
x 22  x12 can be given as,
T = 2
v12  v 22 u2 = 2A2 – 2 x12
10. Maximum acceleration is given as, and v2 = 2A2 – 2 x 22
 = Aω2
Maximum velocity is given as,
....(i)

i.e., v2 – u2 = 2 x12  x 22 
β = Aω ....(ii) v2 u2 = 2(x1 – x2)(x1 + x2) ….(ii)

242

Chapter 05: Oscillations


from equation (i) we get 2 A2  x2
v2 – u2 = (x1 – x2)( + )  =
 x
v2  u 2
 x1 x2 = A2  x2 32  2 2
  = =
x 2
u 2  v2
or x2 x1 = 5
 = rad/s
2
15. Given, 2 2 4
 T= = = s
A=2m;x=1m   5  5
 2
amaxvmax = 4  
 2A A = 4
18. Velocity of particle performing SHM is given
 (2)A = 4
 (2) 2 = 4 by, v =  A2  x 2

ns
 2 2 = 0 2A
When the particle is at a distance from
 2 2 +  – 2 = 0 3
( 2) + 1 ( – 2) = 0 equilibrium position it’s speed is,
 ( + 1) ( – 2) = 0

io
2

v =  A 2  
2A 
  = 2 rad/s 
 3 
{ –1,  Angular velocity cannot be negative}
4A 2 5A2
=  A2  =

at
2
Time period, T =  9 9
 5A
2 22  v=
 T= 2 == 7 s 3
lic
velocity of particle at x = 1 is given by Now, v  3v = 3 
 5
A= 5A
3
v =  A2  x 2 = 2  2   1 = 2 3 m/s
2 2
2

But v  =   A    
2 2A
ub
16. Using v =  A2  x 2  3 
 v2 = 2 (A2 – x2) Where A is new amplitude of motion,
2
v  4A 2 
 = A2 – x2   5 A =   A   
2

 2
 9 
P

v2
 + x2 = A2 4 A2
2  5A2 = ( A )2
9
13 2
 Case 1: 2 + 32 = A2 .…(i) 4 A2
et

  ( A )2 = 5A2 +
9
12 2
Case 2: 2 + 52 = A2 ….(ii) 49 A 2
 ( A )2 =
From equation (i) and (ii) 9
rg

7
13 2
+ 32
=
12 2
+ 52  A = A
2 2 3
1
 (132 – 122) = 52 –32 m
Ta

2 19. T = 2
K
1 25  9
 = KT 2
2 169  144  m=
42
1 16
 =  weight = mg =
KT 2
g =
KT 2g
2 25 4 2
42
5
 = rad/s
4 m
20. T = 2
 5 1 5 k
 But f = =  =
2 4 2 8  T m
17. Given: A = 3 cm T1 m1
i.e., 
when x = 2 cm, v = a T2 m2
i.e.,  A2  x 2 = 2x m1 = m, m2 = m + 1

243

MHT-CET Triumph Physics (Hints)

3 m x and y both in series


 
5 m 1 1 1 1 1
   
m 9 k x y k
 
m  1 25 m m
 Time period T = 2π  2
 25m = 9m + 9 k k
9
m= 1
16 25. As k ,
l
m T m2 4m l l l
21. T = 2π  2  2 length of spring segments = , ,
k T1 m1 m 6 3 2
 T2 = 2  2 = 4 s  k1 = 6k
k2 = 3k
1

ns
22. n = 5 Hz, T = s k3 = 2k
5 when connected in series combination,
m 1 1 1 1
T = 2 = + +
k k 6k 3k 2k

io
The restoring force is equal to the weight of the  k = k ….(i)
spring. when connected in parallel combination,
 kx = mg k = 6k + 3k + 2k

at
m x  k = 11k ….(ii)
 
k g Dividing equation (i) by equation (ii),
x k k 1
 T = 2 = =
k 11k 11
g
A
lic 1 k
 T = 2 ….( At highest position, x = A) 26. n
g 2 m
1 A n k m k 2m

ub
 2     1
5 g n m k m 2k

1 A
 n  n
 4 2 
25 g x y
27.  sin t and  cos t
P

g 10 1 a a
 A= 2
 2

100 100 102 y2 x 2
   1 y 2  x 2  a 2
1 1 a2 a2
 vmax = A = 2 5   m/s
 a circle
et

10 2

m 28. The projection of the particle on Y-axis


23. T = 2 .
K represents simple harmonic motion.
rg

1 t = 0, y displacement is maximum,
Also, spring constant (K) 
Length(l ) so equation will be cosine function.
When the spring is half in length, then K y = a cos ɷ t
becomes twice.
Ta

2π 2π π
m T 1 T
ɷ=   rad/s
 T = 2     T  T 4 2
2K T 2 2 a=3 m
24. ∴ y = 3cos 
πt 

Q  2
P

31. From the graph of velocity (v) v/s distance (x),


we see that the particle executes S.H.M. whose
R S
m
time is recorded from the extreme position.
32. Amplitude of resultant S.H.M.
Springs P and Q, R and S are in parallel R= A12  A22  2A1A2 cos90
Then, x = k + k = 2k ….[for P, Q]
and y = k + k = 2k ….[for R, S] R= A12  A22 = a 2  b2

244

Chapter 05: Oscillations


33. Standard equation of S.H.M., is of the type 1
39. K.Emax = m2 a2
y = a sin  t, y = a cos t or combination of the 2
two. Comparing with standard equation
But the equation, y = a tan  t does not belong a = 8 cm,  = 100 rad/s2
to any of these types. 1
 K.Emax =  4  104 64  10–4 = 128 J
34. 2
B
ඥAଶ + B ଶ
1
40. W1 = kx2 and
2
1
W2 = k (x + y)2
2
A 1 1
W2 – W1 = k (x2 + 2xy + y2) – kx2
y = A0 + A sin t + B sin t 2 2

ns
 y  A0 = A sin t + B cos t ky
= (2x + y)
Resultant amplitude, 2
R = A2  B2  2ABcos900 1
kx2

io
41. W1 = ….(i)
 R= A B 2 2 2
1
W2 = k(2x)2 ….(ii)
1 2
35. K.E. = m2A2 cos2t

at
2 Dividing equation (i) by (ii),
K.E. is maximum at mean position and W1 1
minimum at extreme position and extreme =
W2 4
T
position is reached at every
4
lic
. This is best  W2 = 4W1
W = W2 – W1 = 4W1 – W1
depicted by graph (B). = 4  10 – 10 = 30 J
K.E. 1 2
ub
42. P1 = kx1
2
2P
 x12  1
k
t 1 2
P

T T P2 = kx 2
2
4 2
2P
 x 22  2
36. K.E. is maximum at mean position and P.E. is k
et

minimum at mean position. 1


P = k  x1  x 2 
2

1 2
37. K.E. = m2 (A2 x2) 1
2 = k  x12  x 22  2x1x 2 
rg

1 2
P.E. = m2 x2
2 1  2P 2P 2P1 2P2 
= k  1  2  2  
K.E. A 2  x 2 2  k k k k 
 =
Ta

P.E. x2
P=
1 2

k  P1  P2  2 P1P2
2 k

1
38. K.E. = m2 (A2 x2),
2 P = P1 + P2 + 2 P1P2
1
P.E. = m2x2 43.
2
K.E. A2  x 2 
=
P.E. x2
A l
Here x = l
2
A2
K.E. A  4
2
3A 2 4 3
 = 2
=  2= h
P.E. A 4 A 1
4

245

MHT-CET Triumph Physics (Hints)


From figure, 9 g
 =
l  h 10 l
cos =
l 2 9 g
 h = ll cos = l (1  cos)  = =
T 10 l
P.E = mgh
l 10
 P.E = mg l (1 cos)  T = 2
g 9
K.E. is maximum at mean position, which is
equal to maximum P.E. at extreme position. 10
 T = T
 (K.E.)max = mgl(1  cos) 9

44. Potential energy of particle at extreme position l


1 46. T = 2π
is, P.E. = M2A2 g
2

ns
Time period of pendulum of bob with material
1 g  g 
= M   A2 ….    density ‘’ oscillating in liquid of density ‘’ is
2 L  l  l
T1 = 2π
 ρ

io
45. As pendulum is immersed in liquid, its apparent 1   g
weight is mg mg.  σ
It is evident from the figure that restoring force T1 1
 =
on bob is- 1

at
T  ρ 2
1  
 σ

lic Given  = 1 g/cc = 103 kg/m3
9
= × 103 kg/m3
T 8
T1 1
l = 1
T   2
ub
 10 3 
(mm)g sin  1  9 
(mm)g cos   × 10 
3
x  8 
(mm)g
 T1 = 3T
P

F = (mg mg) sin 


47. Time period of simple pendulum,
for small , sin 
l
Hence, T = 2
g
et

F = (mg  mg)
x 1
But  = = 2
l  g  2
rg

x 1
 F = (mg  mg) = 2
l 12
Now, mg = vg and mg = vg 
=
 = density of brass bob,  = density of liquid 3
Ta

1 20
But  =  48. TA = = 2 sec
10 10
1 x 9 x 10
 F =  (vg  vg) = vg TB = = 1.25 sec
10 l 10 l 8
9 x But T  l
 F= mg
10 l
TA lA
9 x  =
 ma = mg TB lB
10 l
9 x lA T2 22
a= g but a = 2 x  = A2 =
10 l lB TB 1.25 2
9 g lA 64
 2 = =
10 l lB 25

246

Chapter 05: Oscillations

l  New time period,


49. For simple pendulum, T = 2
g I
T = 2
μ B
 T l
I
T1 = 2 .…[From (1) and (2)
Now, T2= , l2 = l1 0.6 ….(given) μB
2
=T
T2 l l T2
  2  2 = 22   
T1 l1 l1 T1 53. A particle oscillating under a force F   k x  b v

l  0.6 T2
is a damped oscillator. The first term kx
 1  12 represents the restoring force and second term
l1 4 T1

b v represents the damping force.

ns
 4l1 2.4 = l1
 3l1 = 2.4 54. Elastic support
 l1 = 0.8 m

io
 l1 = 800 mm
B
C A
Given l2 = (l1 + 0.36) m ; T2 =  T1 
25 
50. T1  D

at
 100 
The disturbance produced in A is transferred to
Time period of simple pendulum is given by,
all the pendulums through the elastic support. A
l and C will be in resonance, since they are of
T = 2
g
lic same length.
 TA= TC = T = 2 l
 T l g
ub
 l T2 1
2  n=
 l1   T1  T
   
 l2   T2   frequency of A and C will also remain same.

P

   
2 amplitude of A and C will be maximum.
l1 T1
   
 l1  0.36   T1  0.25 T1  55. At mean position, velocity is maximum.
2
 vmax = A
et

 l1   T1 
   
l
 1  0.36   1.25 T1   v1 = A
 (1.25)2l1 = l1 + 0.36 v2 = A11
rg

From conservation of linear momentum,


1.56 l1 = l1 + 0.36
m1v1 = m v2
 0.56 l1 = 0.36
 m1A  = (m1 + m2) A11
Ta

 l1 = 0.36 A1  m1  ω
0.56  = 
A  m1 + m2  ω1
l1= 0.64 m
k k
 l1 = 64 cm But = ; 1 =
m1 m1 + m2
51. When magnet is cut along its axis, A1  m1  k  m1 + m 2 
μ  = 
A  m1 + m 2  m1 k
 µ = ….(1)
2 1/ 2
 m 1   m1 + m 2 
New moment of inertia, =   
 m1 + m 2   m1 
m 2
 l I A1 m1
I =  2  = ….(2) 
A
=
m1 + m2
12 2

247

MHT-CET Triumph Physics (Hints)

56. Total distance covered in one oscillation = 4a l


60. T = 2 (when stationary)
1 g
Total time for one oscillation =
n l
T = 2
4a g2
Average speed = = 4 an
1
  (When lift is accelerating upwards)
n 2
 y=t
57. x1 = A sin (t + 1), x2 = A sin (t + 2)
dy
vy = = 2t
 x1 x2 = A  2 sin   t  1   2  sin  1   2   dt
  2   2 
dv y
  gy = = 2 m/s2
 A = 2A sin  1 2  dt

ns
 2 
l l
1   2  1 T = 2 , T = 2 
 sin   10 12
 2  2
T 12 5
1  2      T = T

io
  1  2  T 10 6
2 6 3
61. The total energy of particle performing SHM is
A

at
58. OP = A = 25 cm and OQ = = 12.5 cm 1 2 1
2 E= ka  E = m2 a2
2 2
OPQ = 30
2E 2 2E
Similarly MNO = 30  =  

 PON = 60 =
 Q
licP
ma 2 T

ma 2
ma 2

A/2 A
3 30  T = 2
O L  2E

 t = A/2 30 0.2  (2  102 ) 2
ub
3 30 = 2
M N 2  4  105
2 
t
T 3 0.2  4  104
 T = 2 = 2 seconds
T 2  4  105
 t=
P

6
62. Relation between ‘v’ and ‘x’ in SHM is
3
= (Given: Period = 3s) = 0.5 s v2 x2
6 + 2 =1  Ellipse
A
2 2
et

A
59. Given: l = 1 m, Major axis = 2A Y
Path length (2A) = 16 cm Minor axis = 2A
16 2A A
rg

 Amplitude (A) = = 8 cm Given: = 20


2 2A X
Time period of simple pendulum,   = 20 A
 2f = 20
Ta

l
T = 2 f = 10 Hz
g
2 2 63. T sin  = mL sin2
But = =
T l
2
g

g
=
l T

2
= =
1
For maximum velocity;
vmax = A = 8 cm/s
mg

248

Chapter 05: Oscillations


324 = 0.5  0.5 2 67. At maximum compression,
324 Gain in P.E. of spring = loss in K.E. of sphere
 2 = 1 2 1 1
0.5  0.5  kx = mv2 + I2
2 2 2
324
 = 1
mv2 +  mr 2  2
0.5  0.5 1 2
=
2 2 5 
18
 =  36 rad/s 1 1
0.5 = mv2 + mv2 ….( v = r)
2 5
64. 4.9 7
= mv2
10
120 14 mv 2
 x2 =

ns
A 10 k
240
B=A 14  2  (6) 2
=
10  36

io
A = 2.8
4 i.e., x = 2.8 m
 B = A,  = 240 =
3 68. When the spring gets compressed by length L,

at
K.E. lost by mass m = P.E. stored in the
65. Frequency of oscillation is,
compressed spring
1 1 k
f= = 1 2 1
mvmax  kx2
T 2 m
lic 2 2
 k = m(2f)2 k
 vmax = x
Mole weight (i.e., atomic mass) of silver is m
given 108.
ub
Maximum momentum of the block,
 Mass of 1 atom, Pmax = mvmax = mk x
108
m =
6.02 10 23 69. On the surface of the earth,
P

= 18  10–23 g 2h
time taken for falling, t =
g
= 18  1026 kg
Time period of simple pendulum,
 k = 18  10–26 (2 1012)2
et

l
= 42 18  102 T = 2
g
 k = 7.1 N/m
On the surface of other planet,
rg

66. At maximum compression, the solid cylinder Time taken for falling,
will stop. 2h 2h g
t = = = 2t ( g= )
So loss in K.E. of cylinder = Gain in P.E. of g g/2 2
Ta

spring Time period of simple pendulum,


1 2 1 2 1 2 l
 mv  I  kx T = 2
2 2 2 g
2
1 2 1 mR 2  v  1 2 l
 mv     kx = 2
2 2 2 R 2 g/2

3 2 1 2 = 2 T ( g = )
 mv  kx ଶ
4 2
2t
3 1  t/T = =2
  3 (4) 2   200 x 2 2t
4 2
….( given: t = 2T  t/T = 2)
36
 = x2  x = 0.6 m
100  t = 2T

249

MHT-CET Triumph Physics (Hints)

Hints to Evaluation Test

t  1 1
1. x = cos(t), y = cos    (100) x2= (9)  
1
 2 2 2  27 
1  cos  t  1 1 10
y= i.e. 2y2 – 1 = cos(t)  x= = m = cm
2 300 10 3 3
 2y2 = x + 1 represents a parabola.  5.8 cm
2. Since the amplitudes of the SHM is small, 6. U = 5x(x – 4)
1 = 0sin(1t), (taking first one as reference) = 5(x2 – 4x)
2 = 0sin(2t ) = 5[(x – 2)2 – 4]

ns
For the two to be in same phase,  The particle executes SHM about x = 2.
1t = 2t  dU
F= = 5[x + (x – 4)]
2 2 dx
Substituting,  = = we get,
T 3 ma = 5(2x – 4)

io
2 2 21  a = 100x – 200 = 100(x – 2)
 t= t+t= s  2 = 100  = 10 rad/s
3 7 8
2 

at
3. The concept is that projection of a circle on its Time period = = s
 5
diameter where the circular motion is uniform,
is an SHM.
8.
 Amplitude of motion = 0.5 m ma
 = 60 rev/min = 2 rad/s
lic
2
 T= =1s

mg
ub
4. k n1 = 2k(x2) = 3k(x3)
as tension in the spring remains the same.
Also, x1 + x2 + x3 = A
x1 x1
 x1 + + =A The block will lose contact when N = 0
2 3
P

i.e. mg = ma

 6  3  2  x1 = A g = A2
6
g g
6A  A= =

et

x1 = 2  4  2 
11  2 
x 3A  T 
 x2 = 1 = 10
2 11  A=
rg

 A 2
x1 6  2
 Ratio of amplitudes = =  11  = F
 
x1  x 2  A  3
9  9. B= A
 11 
Ta

 Ax 
 
5. At the mean position,  V0 
M1v = (M1 + M2)v  BA 2 
3v = 9v  F = – x
 V0 
 v = 3v
1 1 BA 2
Also, Kx2 = Mv2  Time period = 2
2 2 MV0
K 10 1 10. At mean position,
 v= x= (0.1) = m/s
M 3 3 1 2
1 P.E. = kx = 0
 v = m/s 2
3 3 i.e., P.E. is minimum.
1 1 Also, velocity is maximum at mean position.
 Kx2= (M1 + M2) v2
2 2 K.E.is maximum.

250

Chapter 05: Oscillations

11. vmax = A = 0.20  100 = 20 cm/s 18. amax = A2


4π 2 3  4   3.14 
2


12. x = a sin  t   = A
T2

 2  3.14 
2
 6
dx   12
v=  a cos   t   .…(i) =  3 cm / s 2
dt  6 4
We know that vmax. = a l
a 19. T = 2
 By substituting v = in equation (i) we get g
2
 T  l , hence if l is made 9 times then T
time (t)
becomes 3 times.
a  
 a cos  t  
2  6

ns
   2 T
  t    .t  t 
3 6 6 T 12

1
m2A2,

io
14. T.E. =
2
(where A = amplitude) Potential energy
1
m2(A2 x2)

at
K.E. =
2
1  A 
2

= m2  A 2    
2   2  

=
1
m2
3A 2
lic
2 4
1
= m2A2  
3
2
ub
4
3
 K.E. = T.E.
4
15. For a particle performing S.H.M.,
P

x = A sin t and
v = A cos t
a = – A2 sin t = A2 cos (90 + t)
et


 a = A2 cos (t + )
2

 The acceleration shows a phase lead of
rg

F 8
16. From graph, slope K = = =4
x 2
Ta

m
T = 2
K
0.01
 T = 2 = 0.3 s
4

2
17. x = A sin t
T
A 2
….  x =
A 
 = Asin  t m
2 T  2 
2 1 
 sin t = = sin
T 2 4
2  T
 t = t=
T 4 8

251
06 Superposition of Waves

Hints

74. Intensity  (amplitude)2


Classical Thinking Let the constant amplitude of each wave be ‘a’.
At the minimum loudness, amplitude is zero

ns
7. y = A sin (t – kx) (ideally)
  Ratio = 0 : a = 0
Wave speed, v =
k
Maximum particle speed, vp = A 75. The loudness is higher if area of the vibrating

io
body is more. The hollow boxes are set into
According to given condition, vp < v
forced vibrations along with the strings. Thus
 1
 A <  A< providing higher area of vibrating body and
k k

at
increase in the loudness of sound.
  2 
 A< ….  k 
2   
lic Critical Thinking

17. When pulse is reflected from a rigid support, the 1 1 10


1. n= = = = 5 Hz
pulse is inverted both lengthwise and sidewise. T 0.2 2

22. Phase difference between the two waves is 2. Comparing the given equation with standard
equation y = A sin (t – kx) we get,
ub
 = (t  2)  (t  1) = (1  2)
Resultant amplitude, 2
= 0.01    = 200 m

A = A12  A22  2A1A2 cos(1 2 )
2
Phase difference =  (Path difference)

P

47. In open organ pipe, both even and odd


harmonics are produced. 2 
=  25 =
200 4
48. In an open organ pipe, all harmonics are present.
et

For pth overtone, we have (p + 1)th harmonic 3. Comparing the given equation with standard
form,
50. In closed pipes, only odd harmonics are present. 2x
y = A sin (t  + ) we get,

rg

51. For closed pipe, in general,


2 1
v 1 = 20  T =
n= (2N  1)  n  T 10
4l l
1

Ta

 If length of air column decreases, then n= = 10 Hz and


T
frequency increases.
2 2
= 5   = = 0.4 m
v v  5
52. nclosed = , nopen =
4L 2L Using, v = n = 10  0.4 = 4 m/s
 nopen = 2nclosed = 2n
4. A = 0.5 m,  = 1 m, n = 2 Hz
57. Frequency of pth overtone is General equation of wave travelling in negative
np = pn1 x-direction,
where p = no. of segments or loops  2 
y = A sin  t  x
n1 = Fundamental frequency   
(given) p = 1 2
 y = 0.5 sin (22t + x) …[ = 2n]
 np = n1 1
i.e., fundamental mode or 1st harmonic  y = 0.5 sin (4t + 2x)
252

Chapter 06: Superposition of Waves

5. y = 4 sin  t  x  Compare with standard wave equation,


 16  y = A sin  2t  2x  we get,
Comparing with standard form,  T  
y = A sin (2nt + 2 x) we get, 2 
= 1.5   =
1
= 0.66
  2 1.5
A = 4 cm, 2n =   n = 0.5 Hz and  x = 0.66 m
 = 32 cm ….  2    12. y = 0.5 (314 t – 12.56 x)
  16 
Compare this equation with standard wave
Using, equation,
v = n = 16 cm/s ….(negative x-direction)
y = A sin  2t  2x  we get,
2t  T 
6.  = = 2 nt = 2    0.5  0.4 = 0.4 

ns
T 2 2  3.14
= 12.56   = = 0.5 m
1 1  12.56
7. n= = = 25 Hz, v = 25 m/s,
T 0.04 13. n = 400  T = 1/ 400
v 25 1 = t1 – kx

io
Using,  = = =1m
n 25 2 = t2 – kx (at same point)
Equation of the wave is,  = 2 – 1 = (t2 – t1) = 2n  (t2 – t1)
y = A sin 2   
t x = 2  400  10–3 = 0.8 

at
T   
 0.8  = 180  0.8 = 144 ….[  = 180]
= 0.02 sin 2 (25t  x)
14.
8. y1 = A1 sin   t  2x  and
  
lic Compressional
maximum
Rarefactional
maximum

y2 = A2 sin   t  2x     
  2
ub
LONGITUDINAL
So phase difference,  =  +  and WAVE
2
Using, x =  . we get,
1
2 T = 0.2 s  n = = 5 Hz
T
P

x =       Time interval between two consecutive


2  2
1 1
compressional maxima, T = = s
9. The given equation is y = 10 sin (0.01 x  2t) n 500
et

Hence  = coefficient of t = 2 Time interval between compressional maxima


Maximum speed of the particle vmax = a T 1 1
and rarefactional maxima, = = s
= 10  2 = 10  2  3.14 = 62.8  63 cm/s 2 2n 1000
rg

5
10. x = 5 sin  t  x  cm 15. Here, A  0.05m,  0.25    0.1m
 0.04 4 2
Now using standard equation of wave,
x = 5 sin 2  t x 
Ta

   2
 2  0.04 2  4  y  A sin (vt  x) we get,

Comparing with standard form,
y  0.05sin 2(3300t  10x)
x = a sin 2  t  x  we get,
T  16. y = 0.5 sin [ (0.01x – 3t)]
T = 2  0.04,  = 2  4 = 0.5 sin [0.01 x – 3t]
 4 Comparing with standard wave equation,
 v= = = 100 cm/s = 1 m/s
T 0.04 y = A sin  2t  2x  we get,
 
 T
2
11. Phase difference =  Path difference 2 2
 =3 T=
2  T 3
 = x  =x 1 3
 2  n= = Hz
From equation, y = 0.04 sin (500t + 1.5x) T 2

253

MHT-CET Triumph Physics (Hints)

y1 = 10sin  3t   
2
= 0.01   = 200 m 24. ....(i)
  3
3 and y2 = 5 sin 3t  3 cos3t 
 Velocity = n =  200 = 300 m/s
2
 
= 5  2  1  sin 3t  3  cos3t 
17. Comparing with standard equation we get
 2 2 
2
= 10   = 0.2 m
 = 10  cos  sin 3t  sin  cos t 
 3 3 
 = 2
2 2 y2 = 10 sin  3t     ....(ii)
 n=  = 1 Hz   t 
 2
and the wave is travelling along the positive ( sin (A + B) = sinA cosB + cosA sinB)

ns
direction. Comparing equation (i) and (ii), we get ratio of
amplitudes as 1 : 1.
ct
18. Here, is dimensionless and unit of ct is same 25. According to given information,

5 = 4   = 0.8 m

io
as that of x. Also unit of  is same as that of A,
which is also the unit of x. Hence frequency,
v 128
n= = = 160 Hz
19. Given equation is,  0.8

at
 t x   t x  and Angular frequency
y = 3 sin     = 3 sin 2   
 0.02 20   0.04 40   = 2n = 2  3.14  160 = 1005 rad/s
Comparing with the standard form, Also, propagation constant,
y = A sin 2p we get,
lic k=
2 2

 0.8
= 7.85 m1
1 1 100
T = 0.04 s  n =   = 25 Hz On substituting these values in standard
T 0.04 4
equation we get,
y = (0.02) m sin (7.85x  1005 t)
ub
400
20. n=  = = 200 Hz ….[  = 400]
2 2 26. Comparing the given equation with standard
equation,
21. Given equation is 2
k= =   102   = 200 m and
 t x 
P

y = 5 sin 2   . 
 0.04 40   = 2n = 2  106  v = 106 Hz
Comparing with the standard form, 27. Points B and F are in same phase as they are 
et

 t x distance apart.
y = A sin 2    we get,
T   28. Given equation is,
 = 40 cm  t x 
y = 0.03 sin 8    
rg

22. Comparing the given equation with  0.016 1.6 


y = A cos(t  kx) we get,  t x 
= 0.03 sin 2   
 0.004 0.4 
k = 2 =    = 2 cm
Ta

  Comparing with the standard form,


t x
23. Comparing the given equation with standard y = A sin 2    we get,
T 
equation,
1 1 1000
 x T = 0.004 s = n = = = = 250 Hz,
y = A sin 2  nt   we get, T 0.004 4
  
 = 0.4 m
 = 2n = 200   n = 100 Hz  Using, v = n = 250  0.4 = 100 m/s
20
Also, k= 
17 29. Phase difference of 90 or rad
2
2 2 
 =  = 1.7 m corresponds to a path difference of
k 20 / 17 4
 200    = 4  0.8 m = 3.2 m
and v =  = 170 m/s
k 20 / 17 Using, v = n = 120  3.2 = 12  32 = 384 m/s
254

Chapter 06: Superposition of Waves

 Superposing, (i) + (ii) is the same as


30. Given that, phase difference of rad y = sin C  sin D
6
Corresponds to a path difference of x m. CD CD
 y = 2A cos sin
2 2
 A phase difference of 2 rad corresponds to
 y = 2A cos t sin kx
path difference of , we get,  The stationary wave is given as
v 100 2x
Now,  =  =2m y = 0.06 sin cos (120 t)
n 50 3
2 1 2 2
 x=  m Here, k =  and  = 120
12 6  3
31. By comparing given equation of progressive 120 
  = 3 m, n = = 60 Hz
wave with standard equation 2

ns
y = a cos (kx  t) we get,
38. I1 A12  I1 = 25 = 1
2 2 
k=  = 25 I2 A 22 I2 100 4
 0.08
2 2 39. Resultant amplitude

io
and  =  
T 2 
A= A 2  A 2  2AA cos  = 4A 2 cos 2  
32. Amplitude of reflected wave = 0.9 A  2

at
On reflection at free end (rarer medium), no As I  A2 , in this case, I  4A2
phase change is introduced.
 Equation of reflected wave is 40. When the waves are in same phase,
I1 = (A+ A)2 = 4A2
y = 0.9 A sin (2nt)
33. Frequency remains constant in both media
lic When the waves are 90 out of phase,
I2 = A2 + A2 + 2A2 cos 90 = 2A2
n = 100 kHz = 105 Hz I1 4A 2 = 2 : 1
 
vair = 340 m/s, vw = 1450 m/s I2 2A 2
ub
Reflected wave travels in air and its wavelength
is 41. In case of interference of two waves, resultant
v air 340 intensity
air = = 5
n 10 I = I1 + I2 + 2 I1I2 cos
P

= 3.4  10 3 m = 3.4 mm If  varies randomly with time,


Transmitted wave travels in water and its (cos)av = 0
wavelength is  I = I1 + I2
v w 1450
et

w = = For n identical waves,


n 105 I = I0+ I0 + …… = nI0
= 1.45  102 m = 1.45 cm  I = 10 I0
rg

  42. a2 = a12 + a 22 + 2a1a2 cos  ….(i)


36. y1 = A sin  t   ,
 6  Here,  = 1 – 2,
  a1 = a2 = a
y2 = A cos t = A sin  t  
Ta

2  Substituting these values in equation (i) we get,


1
  cos  = –   = 2/3
Using, AR = A 2  A 2  2A 2 cos    2
2 6

= 1
= 43. y = 1 sin t  1 sin  t   
A 2  A 2  2A 2  3A a b  2
2
Here, phase difference = 
37. Waves travelling to the right can be given by 2
y1 = A sin (t  kx) ….(i) The resultant amplitude
When getting reflected from the fixed end of the 2 2
 1   1 
string, there is an additional phase difference of A=    
. The reflected wave is  a  b
y2 = A sin (t + kx + ) 1 1 ab
=  
 y2 = A sin (t + kx) ….(ii) a b ab

255

MHT-CET Triumph Physics (Hints)

44. x1 = A sin (t  0.1x) and v 350


48. = = = 1 m = 100 cm
x2 = A sin (t  0.1x  /2 ) n 350
 x1 + x2 = A sin (t0.1x) + A sin(t0.1x/2) Also, path difference (x) between the waves at
= A sin(t  0.1x)  sin  t  0.1x     the point of observation is AP  BP = 25 cm
  2  2 2  25  
  = (x) =   =
 t  0.1x  t  0.1x  ( / 2)   1  100  2
= A  2 sin  
 2   A = (A1 )2  (A 2 )2 = (0.3) 2  (0.4) 2 = 0.5 mm
 t  0.1x  t  0.1x  ( / 2) 
cos  
 2  49. Amax = A2  A2 = A 2 , frequency will remain
   same i.e. .
= 2A sin t  0.1x   cos  
 4  4
50. Imax = I1 + I2 + 2 I1I 2 and

ns
  
= 2A cos   sin  t  0.1x   Imin = I1 + I2  2 I1I 2
 4  4
 Required amplitude = 2A cos   Sum of maximum and minimum intensities
4 = 2 (I1 + I2)

io
45. Given that y1 = 3 sin 2(50)t and 51. Resultant amplitude AR = 2A cos   
2
y2 = 4 sin 2(75)t
= 2  (2A)cos    = 4A cos   

at
 Comparing given equations with standard form,
y = A sin 2nt we get, 2 2
n1 = 50 and A1 = 3 and n2 = 75 and A2 = 4
53. Progressive waves propagate energy while
Now, I  A2 n2

I1 2

=  A1    n1 
2
lic stationary waves do not propagate energy.

I2 54. Waves z1 = A sin(kx  t) is travelling towards


 A2   n2 
2 2 positive x-direction.
=  3    50  =
9 4 1
 = Wave z2 = A sin(kx + t) is travelling towards
ub
4  75  16 9 4 negative x-direction.
A1 = 5  A = 5 A Wave z3 = A sin(ky  t) is travelling towards
46. 1 2 positive y direction.
A2 3 3
Since waves z1 and z2 are travelling along the
P

2
5  same line, so they will produce stationary wave.
 Imax = (A1  A 2 )2 =  A 2  A 2 
3 
Imin (A1  A 2 )2 2 59. Comparing given equation with the standard form,
5  
 A2  A2   2x 
et

3  y = A sin   .cos (2nt) we get,


 8A 2 
2
  
2
=  3  =  4  =
16 8
1 2tn = 8t  n = = 4 cycles / s
 2A 2  1 2
rg

 
 3 
 Imax : Imin : : 16 : 1 60. In closed organ pipe, if
yincident = A sin(t  kx), then
2
 A1  yreflected = A sin(t + kx + )
Ta

  1
47. Imax = (A1  A 2 ) 2
=  A2  =  A sin(t + kx)
Imin (A1  A 2 ) 2  A1 
2
Superimposition of these two waves gives the
  1  required stationary wave.
 A2 
 A1  (   ) (  )
1 61. cos  + cos  = 2 cos cos
 A max =  A2  2 2
A min  A1   y = y1 + y2 = 2  0.05  cos (x) cos (4t)
  1 For node, cos (x) = 0
 A2 
 3 5
2
9  x = , , ,…
Given that, I1 = A12 = 2 2 2
I2 A2 1
1 3 5
A1 = 3  A max = 3  1 = 4  x = , , ,….
 2 2 2
A2 1 A min 3 1 2  x = 0.5 m
256

Chapter 06: Superposition of Waves


2 v 332
62. Using, = coefficient of x in the argument of 74. For closed pipe, n   n  2Hz
 4l 4  42
2
the sine function = k   = 75. n1 – n2 = 10 .…(i)
k v
Distance between adjacent nodes = /2. Using n1 = and n2 = v we get,
4L1 4L 2

 The distance between adjacent nodes = n1 L2 26
k = = ….(ii)
n2 L1 25
x On solving these equations,
63. y = 6 sin cos 8t
6 n1 = 260 Hz, n2 = 250 Hz
Comparing with the standard wave equation v 1
 2x   2t  76. n= n
y = A sin   cos   we get, 4L L
  

ns
 T  L1 100 n2
2x x  = =
=   = 12 L2 101 n1
 6 As L2 > L1 , hence n2 < n1
 The distance between two consecutive nodes,  n1 – n2 = 5

io
 12 100 n2
= =6  =
2 2 101 n2  5
64. Energy is not carried by stationary waves.
 101 n2 – 100 n2 = 5  100

at
65. The given equation can be written as,  n2 = 500 Hz
 x   n1 = n2 + 5 = 500 + 5 = 505 Hz
y = 4sin  4t  
 16  77. According to problem,
 v=
Co-efficient of t()
lic 1 T
=
v
....(i)
Co-efficient of x(k) 2l m 4L
4 1 T 8 3v
 v= = 64 cm/s along + x direction. and = .…(ii)
 / 16
ub
2l m 4L
66. y = A sin (100t) cos (0.01x) Dividing equation (i) by equation (ii),
Comparing with standard wave equation, T 1
=  9T = T + 8  T = 1 N
 2t   2x  T 8 3
y = 2A sin   cos   we get,
P

 T    
v
2t 78. For closed pipe, n1 =
= 100 t 4L
T v
2  250 =
et

1 100
 T= n= = 4  0.2
100 T 2  v = 200 m/s
2πx 2
Also, = 0.01 x   = 79. Fundamental frequency of a closed pipe is given
 0.01
rg

Velocity of wave, v
by n0 =
100 2 4L
v = n =  = 104 mm/s Length l of air column first decreases and then
2 0.01
becomes constant (when rate of inflow = rate of
Ta

3
= 10  10 mm/s = 10 m/s
outflow). Therefore, f0 will first increase and
67. When two bodies have the same frequency, then then become constant.
one is excited and other vibrates with its natural
80.
frequency due to resonance.
/4
71. L2 = 3L1 = 3  24.7 = 74.1 cm
72. For closed organ pipe, only odd harmonics are
present. Hence note of frequency 100 Hz will 
not be emitted as 100 = 2  50. The first resonance will occur at length L =
4
73. For a closed pipe, For closed pipe, only odd frequencies are present.
2nd overtone = 5th harmonic So next resonance will be obtained at length
 5th harmonic = 5  fundamental frequency 3 5
, , …...
= 5  50 = 250 Hz 4 4

257

MHT-CET Triumph Physics (Hints)


81. Fundamental frequency of open pipe, 88. Let L1 and L2 be the lengths of open and closed
v 350 pipes respectively. (Neglecting end correction)
n1 = = = 350 Hz
2L 2  0.5  1 = 2L1, 2 = 4L2
Given that, 1 = 2
82. For open organ pipe,
v 320  2L1 = 4L2
n0 = = = 400 Hz L1 1
2L 2  40  102  =
L2 2
n = 1200 Hz = 3  400 Hz
 The mode of vibration is 3rd harmonic 4v 2v
 2nd overtone 89. For open pipe, no = 4nf = =
2L o Lo
83. L1 = 50 cm, L2 = 50.5 cm 7v
For closed pipe, nc = 7nf =
As L2 > L1, so n2 < n1 4L c

ns
For open pipe, no = nc ....[Given]
v
n= 2v 7v L 8
2L  =  o =
Lo 4L c Lc 7
n1 – n2 = 3 beats/s

io
v 1 1  90. Fundamental frequency of closed pipe,
    =3
2  L1 L 2  v v
n = L=
v  1 1  4L 4n '
    =6

at
102  50 50.5  Fundamental frequency of open pipe,
v v
6  50  50.5  102 n= L=
 v= = 303 m/s 2L 2n
0.5
v v n
84. 1 = 2L, 2 = 2L + 2L
lic 
4n '
=
2n
 n =
2
v v
n1 = and n2 = 91. When one end is closed, n1 = 100/2 = 50 Hz
2L 2L  2L
n2 = 3 n1 = 150 Hz,
v 1 1 
ub
 No. of beats = n1  n2 =    n3 = 5 n1 = 250 Hz and so on
2  L L  L 
vL 92. Let ‘L’ be the length of the pipe,
= v
2L2 n= ….(i)
2L
P

85. Given frequencies are 425, 595, 765 Hz


2
v = 340 m/s When the pipe having a length of L is inside
Option A : For a closed pipe having L = 1 m, 5
water, then length of the air column,
et

v 340
nc = = = 85 Hz 2L 3L
4L 4 L1 = L – =
Option B: For L = 2 m, nc = 42.50 Hz 5 5
Option C: For open pipe having, L = 1 m, v

rg

n =
v 340 4L1
n0 = = = 170 Hz
2L 2 v 5v
Option D: For L = 2 m, n0 = 85 Hz = =
3L 12L
4
Ta

Open pipe has all the harmonics, which is not 5


possible. 5  v 
Closed pipe has only odd harmonics. Hence =   ….[From (i)]
6  2L 
L = 2 m is not possible.
 Correct option is (A). 5
 n = n
6
86. First overtone frequency for closed pipe
3v 93. For open pipe, fundamental frequency,
=
4L v
n=
v 2L
Fundamental frequency for open pipe =
2L v 1  v  1
For closed pipe, n = =    n
First overtone frequency for open pipe 4L 2  2L  2
 v  v 4 3v n
= 2  =   = 512 Hz  n = 2  512 = 1024 Hz
 2L  L 3 4L 2

258

Chapter 06: Superposition of Waves


v v 104. Let e be the end correction then according to the
94. For open pipe, n1 =  L1 =
2L1 2n1 information given,
v v v 3v
For closed pipe, n2 =  L2 =   0.35 + e = 3(0.1 + e)
4L 2 4n 2 4(l1  e) 4(l2  e)
After joining, L = L1 + L2 e = 2.5 cm = 0.025 m.
Since it is a closed pipe, l2  3l1 48  3(15)
v v 105. e = = = 1.5 cm
n= = 2 2
4L 4(L1  L 2 )
106. e = 0.3 d
v
= l2  3l1
 v v  d=
4   2
 2n1 4n 2  l2  3l1 0.62  3  0.2
8n1 n 2  d= =

ns
= 0.6 0.6
4  4n 2  2n1  6.2  6
=
n1n 2 6
=
2n 2 + n1 = 0.033 m = 3.33 cm

io
500  450
= v
 2  450   500 107. Fundamental frequency n =
2L
= 160.7 Hz  161 Hz 350 1

at
 350 =  L = m = 50 cm
96. For a closed organ pipe, 2L 2
n1 : n2 : n3…. = 1 : 3 : 5 .…
108. For a pipe open at both ends,
97. n = 100 Hz and n = 500 Hz = 5  100
 n = 5 n  Pipe is closed at one end.
lic n=
v

333
2l 2  33.3  102
= 500 Hz

98. For a closed pipe, frequency of second note  Frequency of 5th overtone,
3v 3  330 n = 6n = 6  500 = 3000 Hz
=   165 Hz
ub
4l 4  1.5 3v 4v
nd
109. nc = and n0 =
99. Frequency of 2 overtone 4L1 2l2
n3 = 5n1 = 5  50 = 250 Hz.  nc = n0 gives,
l1 3
100. For a pipe closed at one end, 3l2 = 8l1  
P

v 340 l2 8
n= =
4l 4  34  102 110. For resonance,
 Frequency of 5th overtone  nc = n0
et

340  102 v v L 1
n = 11 n = 11  = 2750 Hz    1 
4  34 4L1 2L 2 L2 2
101. For a closed pipe, 111. Frequency of 5th overtone of closed organ pipe =
rg

Frequency of 1st overtone, Frequency of fifth overtone of open organ pipe.


n 480
n = 3n  n = = = 160 Hz  11 n = 6 n
3 3 v v
 11  = 6
Ta

v 330 4L 2L
102. For a closed pipe, n = =
4l 4  1 L 11
 
 Frequency of second note = 3 L 12
3  330
n= Hz 112. Difference between two successive resonance
4 1 frequencies
103. Number of beats per second, n = 595  425 = 170 Hz
16 4 v 1 1  Similarly n = 425  255 = 170 Hz
n   n  n1  n 2     If pipe is open at both ends, air column will
20 5 4  l1 l2 
vibrate with all harmonics i.e. n1, 2n1, 3n1, ….
4 v  1 1  0.01v v
     =  n = n1 = 170 Hz
5 4  1 1.01  4  1.01 4  101 But in that case, successive resonance
16  101 –1 frequencies will be multiples of 170 Hz which
 v  323.2 ms
5 contradicts the given data.

259

MHT-CET Triumph Physics (Hints)


If pipe is closed, air column will vibrate with 3
 L=  64 = 48 cm
only odd harmonics i.e., n1, 3n1, 5n1,…. 4
 n = 2n1  Length of the tube = L – e = 48 – 1 = 47 cm
170
 n1 = = 85 Hz 
2 120. Here, L2  L1 = or  = 2(L2  L1)
In this case, 3n1, 5n1, 7n1 corresponds to 2
frequencies 255, 425 and 595 Hz. Using, v = n,
v v
2v n= =
113. Open pipe resonance frequency, f1   2(L 2  L1 )
2L
nv 330
Closed pipe resonance frequency, f 2  = = 500 Hz
4L 2(49  16)  102
n
 v v 2v 3v

ns
f2 = f1 where, n is odd 121. np = , nq = , nr = , ns =
4 4L 2L 2L 4L
As f2 > f1  n = 5 np : nq : nr : ns
114. Frequency of first overtone of closed pipe = v 2v 4v 3v
: : :

io
Frequency of first overtone of open pipe 4L 4L 4L 4L
3v1 v 2 3 P 1 P  P   np : nq : nr : ns : : 1 : 2 : 4 : 3
     v  
4L1 L 2 4L1 1 L2 2    2L

at
122. For a vibrating string,  =
4L1 1 4L 1 p
 L2  
3 2 3 2 where p = Number of loops = Order of vibration
or mode
115. Fundamental frequency of a pipe closed at one
lic
end = Frequency of 2nd overtone of pipe open  For fourth mode p = 4,  =
2(2)
4
=1m
1
at both ends   v = n = 500  1 = 500 m/s
2
123. y = 0.021sin(x + 30t)
ub
v 1 3v 1 3
    
4nL1 2 2nL 2 L1 L 2 Comparing this equation with the standard form
 L2 = 3L1 = 30 cm we get,
 = 30 rad/s and k = 1
116. t1 – t2 = 1  30
P

L L  v= = = 30 m/s
  1 k 1
340 3740
T
 3400 L = 340  3740 Using, v = we get,
m
et

34  374
 L= = 374 m
34 T
30 =  T = 0.117 N
1.3  104
117. Critical hearing frequency for a person is
rg

20,000 Hz. 124. Here,  = 2  8 = 16 m


For a closed pipe vibrating in Nth mode, 1 T 1 T
frequency of vibration n= =
2L m 2 L2m
(2N  1)v
Ta

n1 = = (2N  1)n 1 T 1 T
4l = =
 20,000 = (2N – 1)  1500 2 2M 2 ML
L 
 N = 7.1  7 L
Also, in closed pipe, 1 96
Number of overtones = = 5 Hz
2 0.120  8
= (Number of mode of vibration) – 1
= 7 – 1 = 6. 125. Stretched wire produces integral number of
harmonics
l2  3 l1 49  3  16
118. e = = = 0.5 cm Let 420 = 6  70 Hz
2 2
490 = 7  70 Hz
119.  = (15 + 1)  4 = 64 cm  Fundamental frequency of wire is 70 Hz
3 1 T
For second resonance, L = n=
4 2L m

260

Chapter 06: Superposition of Waves

1 T 131. n  T
 L=
2n m  n1  T1 and n2  T2
1 450 But T2 > T1  n2 > n1
 L=
2  70 5  103  n  n1 = 5
1  n  k T1 = 5
=  3  100
2  70
 n  k 100 = 5
30
= = 2.1 m  n  10 k = 5 ….(i)
14
 n2  n = 5
126. L1 + L2 + L3 = 110 cm and  k T2  n = 5
n1L1 = n2L2 = n3L3
n1 : n2 : n3 :: 1 : 2 : 3  k 121  n = 5

ns
n1 1 L L  11 k  n = 5 ….(ii)
 = = 2  L2 = 1 and Adding equeations (i) and (ii),
n2 2 L1 2
k = 10
n1 1 L L
= = 3  L3 = 1 Substituting in equation (i),

io
n3 3 L1 3 n  100 = 5  n = 105 Hz
L1 L
 L1 + + 1 = 110 0.01
2 3 132. m = = 2  102 kg/m
0.5

at
 L1 = 60; L2 = 30 cm, L3 = 20 cm
1 T
T
n=
T T 2l m
127. v = = v
m r 2 r
2  102
vA TA rB 1 1 1
lic  n=
1
2  0.5
800
2  102
=
1
= 200 Hz
 = . = . =
vB TB rA 2 2 2 2
133. nA = 324 Hz, nb = 6 Hz
1 The frequency of string B is
ub
128. n  nB = nA  nb = 324  6 = 330 or 318 Hz
L
n L
Now, the frequency of a string is proportional to the
 =  square root of tension. Hence, if the tension in A is
n L
slightly decreased, its frequency will be slightly
If length is decreased by 2%, then frequency
P

reduced, i.e., it will become less than


n 2  n1 2 324 Hz. If the frequency of string B is 330 Hz, the
increases by 2% i.e., 
n1 100 beat frequency would increase to a value greater
2 2 than 6 Hz if the tension in A is reduced. But the
et

 n2  n1 =  n1 =  392 = 7.8  8
100 100 beat frequency is found to reduce to 3 Hz.
Hence, the frequency of B cannot be 330 Hz. It
129. n  T is therefore 318 Hz. When the tension in A is
rg

T2 n2 reduced, its frequency becomes 324 – 3 = 321


 = 22 Hz which will produce beats of frequency 3 Hz
T1 n1
2
with string B of frequency 318 Hz.
n 22  320 
 T2 =  T1 =    16 = 25 kg-wt
Ta

n12  256  n1 T1
134. n  T= = ….(i)
 T = T2  T1 = 25  16 = 9 kg-wt n2 T2
50 150n1
1 T  n2 = n1 +  n1 =
130. n = 100 100
2l m n1 100 2
 = = ….[From (i)]
1 n2 150 3
Let T = 2T , A = A
2 T1 4
 =
Now, m = AL T2 9
1 m T2  T1 T 
 m = AL = AL =  % increase = 100 =  2  1 100
2 2 T1 T
 1 
1 T 1 2T 1 T 9  500
 n = = = 2   = 2n =   1  100 = = 125%
2l m 2l (m / 2)  2l m  4  4

261

MHT-CET Triumph Physics (Hints)


135. When a musical instrument is played, it produces a 146. The sonometer wire vibrates in second overtone
fundamental note which is accompanied by a as shown in the figure
number of overtones called harmonics. The number
of harmonics is not the same for all instruments. It N N A
A A
N N
is the number of harmonics which distinguishes the
note produced by a sitar from that produced by a
violin.  4 Nodes and 3 Antinodes
T 1 T
136. n 147. Using, n  n T
l 2l m
 l  T ( n = constant) As T1  T2  n1  n 2
l2 T2  n1  n 2  6
 =

ns
l1 T1 The beat frequency will remain fixed at 6 if
i. n1 remains same but n2 is increased to a
 l2 = l1 169 new value  n 2  n 2 12  by increasing
100
 l2 = 1.3l1 = l1 + 0.30 l1 = 30% of l1 tension T2.

io
ii. n2 remains same but n1 is decreased to a
137. According to law of tension, new value  n1  n1 12  by decreasing
N T tension T1.

at
Therefore, when the tension is doubled, the
frequency becomes 2 times. 1 L n
148. n   2 = 1
L L1 n2
138. Distance between six successive nodes,

5
 85 cm
lic  L2 = L 1 
 n1 
n
 2
 = 50 
270
1000
= 13.5 cm
2
2  85 T 60.5
   34 cm = 0.34 m 149. v = = = 110 m/s
5
ub
m  0.035 
 Speed of sound in gas,  
 7 
= n = 1000  0.34 = 340 m/s
150. String vibrates in five segments
140. 5 2l
A A  l  
P

N N 2 5
N
v v 20
1.21 Å  n   5  5  5 Hz
 2l 2  10
et

141. In fundamental mode of vibration, wavelength 1 T


151. n = n T
is maximum 2L m
 For octave, n = 2n
 = 40 cm   = 80 cm
rg

L=
2 n T
 = =2
142. At fixed end, node is formed and distance n T
between two consecutive nodes,  T = 4T = 16 kg-wt
Ta


= 10 cm   = 20 cm n1 T1
2 152. n  T  
n2 T2
 v = n = 100  20  10–2 = 20 m/s
n 10
 =  T2 = 40 N
143. n  T 2n T2
144. Here,  = 2l 153. n  T
 v = n = 480  2  0.3 = 288 m/s n 1 ΔT
 =
v 36 1 n 2 T
145. Here,  =  = 0.5 m = m
n 72 2  1 ΔT 
 Beat frequency, n =  n
 Distance between wall and first antinode 2 T 
 1 1 2
=  m =   400 = 4
4 8 2 100

262

Chapter 06: Superposition of Waves

1 T 1 T1 T1
154. Using, n =  = =
2 m 3 T2 T1  8
1 T 1 1 T1 1
Number of beats =     =
2 m  l2 l1  T1  8 9
1 20  1 1   9 T1 = T1 + 8  T1 = 1 kg-wt
= 3 
 7
2 1  10  49.1  10 2
51.6  102 
1 T
161. n =
1 T 2L r 2d
155. Fundamental frequency n 
2l r 2 1 9T
n =
1 n r l r 2L 1 2L r2d
n  1  2  2   
lr n 2 r1 l1 2r L 1 n 3L r
   ….(i)

ns
156. At resonance, frequency of A.C. will be equal to n L r
natural frequency of wire,  mass remains the same
1 T 1 10  9.8 100
n    50 Hz r L
2l m 2  1 9.8  103 2 

io
r L
157. Here, nl = constant Substituting in eq. (i)
 n1l1 = n2l2  110 (l1) = (l1 – 5) n2 n L
3

at
110  60
 = n2  n2 = 120 Hz n L
55
 Number of beats = 120  110 = 10  L > L
 n < 3 n
158. Mass per unit length of the string
1.0  103
lic
m= = 5  103 kg m1 162. L1 = 40 cm, L2 = 30 cm
20  102 1 T T
speed of waves in string n=  = constant
2L m L
ub
I 0.5
v=  = 10 ms1 T1 T2 L T1
m 5  103  =  1 =
L1 L2 L2 T2
Now, v = n
2 2
v 10 T2 L   30  9
 = = = 0.1 cm = 10 cm  =  2 =   =
P

n 100 T1  L1   40  16

 separation between successive nodes = Let T1 = Vdg and density of fluid in which
2 weight will be immersed is 
et

= 5 cm
 T1 – T2 = Vg
159. If a rod clamped in the middle, then it vibrates T1  T2 
similar to an open organ pipe as shown in the  =
T1 d
figure.
rg

A
T2  9 7
N
 1– = =1– =
T1 d 16 16
A A d 16
l N  =
Ta

 7
l
163. On earth:
A
1 Mg 1 g
 Fundamental frequency of vibrating rod is, n= = , Since M = 1 kg
2L m 2L m
v v
given by n1   2.53  On moon:
2l 2 1
 v = 5.06 km/s. n =
1 Mg / 6
=
1 Mg
2L m 2L 6m
n1 T1
160. n  T  = For resonance: n = n
n2 T2
n2 = 3n1 ….[Given] 1 g 1 Mg
=
n1 1 2L m 2L 6m
 =
n2 3 which gives M = 6 kg
263

MHT-CET Triumph Physics (Hints)


164. Probable frequencies of tuning fork be n + 4 or 173. Beat frequency = 258 – 256 = 2 Hz
n4  Time interval between two maxima
1 1 1
Now, n  = = = 0.5 s
l beat frequency 2
n  4 100
  or 95(n + 4) = 100(n  4) 174. Time interval between a maxima and
n  4 95
consecutive minima is
 95n + 380 = 100n  400 1
1 1
 5n = 780  n = 156 Hz t = = = s
2(n1  n 2 ) 2  4 8
1
165. n   nl = constant 175. n56 = n1 + (56  1)4
l
Also, n56 = 2n1
 n1 l1 = n2 l2
 2n1 = n1 + 55  4

ns
n1 l1 = (n1 – 2)l2 ....[n2 < n1 as length increases]
 n1 = 220 Hz
l2 n1 250 125
   
l1 n1  2 248 124 176. Forks arranged in a series of increasing
frequency from n1 to n32
166. When the length of sonometer wire increases by  n32 = n1 + 31(6) = n1 + 186 ….(i)

io
4%, the new length, Given condition is n32 = 2n1 ….(ii)
l2 = 1.04 l1 From (i) and (ii),
Now, nl = constant 2n1 = n1 + 186  n1 = 186 Hz

at
 n1 l1 = n2 (1.04 l1)  n1 = 1.04 n2
177. Here, 1 = 2n1 = 500 , n1 = 250 Hz
 n2 = n1  8 …( n2 < n1)
 2 = 2n2 = 506 , n2 = 253 Hz
 n2 = 1.04 n2 – 8 No. of beats / s = n2  n1 = 253  250 = 3 Hz
 0.04 n2 = 8  n2 = 200 Hz
lic No. of beats / minute = 3  60 = 180
167. Let the frequency of tuning fork be N. v v
178. n1 = , n2 =
As the frequency of vibrating string 1 2
ub
1  1 = 2 m, 2 = 2.02 m

length of string Since 1 < 2 ,
For sonometer wire of length 20 cm, frequency  n1 > n2
must be (N + 5) and that for the sonometer wire  n1  n2 = 2
of length 21cm, the frequency must be (N – 5)
P

= 2  v   2  1  = 2
v v
as in each case, the tuning fork produces 5  
1 2 
beats/s with sonometer wire  1 2 
 n1l1 = n2l2  (N + 5)  20 = (N – 5)  21 2    2  2  2.02
 v= 1 2 = = 404 m/s
et

 N = 205 Hz 2  1 2.02  2

1 T T 179. Frequency of fork A = fA = 200 Hz


168. n  2
2l r 
2
r No. of beats per second = 4
rg

Hence, frequency of fork B is either


2 2
n1  T  r      1  2   1  200 + 4 = 204 Hz or 200  4 = 196 Hz.
   1  2   2         1
n2  T2  r1   1   2  1   2  When B is loaded with wax, the beats stop. On
loading, the number of beats per second has
Ta

 n1  n 2 decreased. Hence, the answer should be


204 Hz. This is because after loading with wax,
1 T
169. n =  n  l1 the frequency will decrease to 200 Hz (i.e. to
2l M frequency of fork A) and beats disappear.
n l
 % =   100 10
n l 180. Beats per second =
3
= l
1 = 100 cm, 2 = 101 cm ….[Given]
= 1% = 1% (In magnitude)
Suppose the velocity is v
171. For producing beats, there must be small  Frequency of first wave = n1 = v =
v
and
difference in frequency. 1 100
172. n  4 = 250, or n + 4 = 250 frequency of second wave = n2 =
v
=
v
 n = 254 or n = 246 2 101

264

Chapter 06: Superposition of Waves


10
n1  n2 = 186. Using v = n,
3 v v v v
n1 =  and n2 = 
v v 10 101 100  10 1 0.50  2 0.51
  =  v=
100 101 3 3  1 1 
 n = n1  n2 = v    = 12
 v = 33667 cm/s = 336.67 m/s  0.5 0.51
12  0.51  0.50
181. Let the frequencies of the 28 forks be  v= = 306 m/s
0.01
n1 …… ni …….n28
Such that ni 1  ni = 4 Hz 187. Frequency of string = 440  5
When frequency of tuning fork is decreased,
 n1  n28 = 108 Hz
beat frequency is increased.
n1
= 2  n1 = 2n28  Frequency of string = 445 Hz
n 28

ns
188. Comparing given equation with standard form,
 2n28  n28 = 108 Hz
y = A sin 2nt we get,
n28 = 108 Hz and n1 = 216 Hz
n1 = 316 and n2 = 310
182. n11 = n22 2 2

io
110 110 Number of beats heard per second,
 < ….[ n1 > n2]
177 175 316 310 3
n1 – n2 =  =
 n1 = n2 + 6 2 2 

at
 (n2 + 6)1 = n22 189. nA = Known frequency = 288 c.p.s
110 110 x = 4 b.p.s.,
 (n2 + 6) = n2 
177 175 After loading of wax on tuning fork B, nB
 175 (n2 + 6) = 177 n2
lic decreases. If we consider nA > nB then,
after loading, nA  nB will increase. But it
 n2 = 3  175 = 525 Hz
 n1 = n2 + 6 = 525 + 6 = 531 Hz contradicts the given data that x decreases to
2 b.p.s.
ub
183. Let n be a frequency of given fork.  nB = nA + x = 288 + 4 = 292 c.p.s.
We have following possibilities for n:
190. nA = 512 Hz
Case I: When 2 beats/s are produced, oscillator
reads 514 Hz. Given that, nA  nB = 8
When B is loaded with wax, the number of beats
 n – 2 = 514 or n + 2 = 514
P

reduces to 4 per second.


 n = 516 Hz or n = 512 Hz ….(i)
 nB – nA = 8 is the correct equation.
Case II: When 6 beat/s are produced, oscillator
 nB = nA + 8 = 512 + 8 = 520 Hz
reads 510 Hz
et

 n – 6 = 510 or n + 6 = 510 191. Here, n11 = n1 + (11 – 1)  8 = n1 + 80


 n = 516 Hz or n = 504 Hz ….(ii) and n11 = 2 n1
 From equations (i) and (ii),  2 n1 = n1 + 80  n1 = 80 Hz
rg

 n = 516 Hz  n10 = 80 + (10 – 1)  8 = 152 Hz

v 1 1 1
184. Using, v = n or n = we get, 192. T = = = = 0.2 s
 n 2  n1 325  320 5
Ta

v 330 v
n1 =  = 66 Hz 193. Using, v = n we get, n =
1 5 
v 330 Given that, n2 – n1 = 5
and n2 =  = 60 Hz
 2 5.5  1 1
 v  = 5
Number of beats per second, 
 2 1 

n1  n2 = 66 – 60 = 6  1 1  5  52  52.5
 v  = 5  v =
185. From the given equations of progressive waves,  52 52.5  0.5
1 = 500 and 2 = 506 = 10  52  52.5 = 273 m/s
 n1 = 250 Hz and n2 = 253 Hz 273
 n1 = = 520 Hz and
Hence, beat frequency = n2  n1 52.5  102
= 253 – 250 = 3 beats per second 273
n2 = = 525 Hz
 Number of beats per minute = 180. 52  102

265

MHT-CET Triumph Physics (Hints)


194. nA = 305 Hz
 vmax = y0  2v 
Given that, nA  nB = 5   
When B is filed, the number of beats reduce to 3 Let, vmax = 2v
beats/s.
 The correct equation is, y0  2v  = 2v   = y0
  
nB – nA = 5  nB = nA + 5 = 305 + 5 = 310 Hz
195. nB = 384 Hz  
204. Speed = n = n(4ab) = 4n  ab …  ab  
Given that nA  nB = 4  4 
When A is filed, the number of beats reduce 3
 Path difference between b and e is
to 3 per second  The correct equation is, 4
nB – nA = 4  nA = nB  4 = 384 – 4 = 380 Hz 2
Now, Phase difference =  Path difference

ns
196. Given equations are,
2 3 3
y1 = a sin (2000  t) = a sin 2 (1000 t) and = . 
 4 2
y2 = a sin (2008  t) = a sin 2 (1004 t)
 Comparing with the standard form, 205. Given that, vmax = 4 vp

io
y = A sin 2 nt we get,  A = 4  n
n1 = 1000 Hz and n2 = 1004 Hz 2 1
 A =4 
 Number of beats = 1004 – 1000 = 4 beats/s T T

at
A
197. Loudness depends upon intensity while pitch  A   = 2  or  =
depends upon frequency. 2
199. The loudness of sound is measured on decibel 206. For the second resonance, x = 3L1 = 54 but
scale which is logarithmic. during summer, temperature increases and
lic  I
hence velocity of sound increases.
Loudness or sound level = 10 log   . Each  x > 3L1 i.e., x > 54 cm
 I0 
increase in intensity by a power of 10 increases I 12 2
207. dB = 10 log10   ; where I0 = 10 Wm
ub
decibel reading of 10 units.  I0 
Hence, to increase the decibel reading by 20,
 I1  I1
there needs to be an increase in intensity by Since, 40 = 10 log10   = 104
10  10 = 100. I
 0 I0
P

 I2  I2
 Also, 20 = 10 log10   = 102
200. At t = 0 and x = , the displacement I0
2k  I0 
 I2 d2
y = A0 sin  (0)  k    =  A0 sin =  A0  = 102 = 12  d 22 = 100d12
et

 2k  2 I1 d2
Point of maximum displacement (A0) in  d2 = 10 m ….[ d1 = 1 m]
negative direction is Q.
rg

201. Maximum pressure at closed end will be p T


208. The frequency of vibration of a string n 
atmospheric pressure added to acoustic wave 2l m
pressure. Also number of loops = Number of antinodes.
 pmax = pA + p0 and pmin = pA  p0  With 5 antinodes and hanging mass of 9 kg, we
Ta

p max p p 5 9g
 = A 0 have p = 5 and T = 9g  n1 
p min p A  p0 2l m
With 3 antinodes and hanging mass M, we have
I
202. Energy density (E)   2  2 n 2 A 2 3 Mg
v p = 3 and T = Mg  n 2 
2l m
vmax  A  2nA  E  (v max ) 2

5 9g 3 Mg
i.e., graph between E and vmax will be a  n1 = n2  
2l m 2l m
parabola symmetrical about E axis.
 25  9 g = 9  Mg  M = 25 kg.
203. Wave velocity = v
209. In a stretched string, all multiples of
Particle velocity,
fundamental frequencies can be obtained.
= y0  2v  cos  2 (vt  x) 
dy i.e., if fundamental frequency is ‘n’, then higher
vmax =
dt      frequencies will be 2n, 3n, 4n, 5n …
266

Chapter 06: Superposition of Waves


For N = 1, 2, 3, ... we get l = 25 cm, 75 cm, 125
cm ... etc.
As the tube is only 120 cm long, length of air
column after water is poured in it may be
75 cm 25 cm or 75 cm only. Hence, the corresponding
 Any two successive frequencies will differ by ‘n’ length of water column in the tube will be
Given that, n = 420 – 315 = 105 Hz. (120 – 25) cm = 95 cm or (120 – 75) cm = 45 cm.
 The lowest resonant frequency of the string is Thus minimum length of water column is 45 cm.
105 Hz. 215. A2 = A2 + A2 + 2A2 cos
210. Using  = 2(l2 – l1)  v = 2n(l2 – l1)  cos = 
1
 2  512 (63.2 – 30.7) = 33280 cm/s 2
Actual speed of sound,  1  2
  = cos1    

ns
v0 = 332 m/s = 33200 cm/s  2 3
 Error = 33280 – 33200 = 80 cm/s
216. For both the positions in Melde’s experiment,
M AL Tp2 = constant.
211. m    A
L L  T1p12  T2 p 22

io
T/A YlA
Y   T .  (m0 + m1)g p12 = (m0 + m2)g p 22
l/L L
Hence lowest frequency of vibration,  m 0 p12  m1p12  m 0 p 22  m 2 p 22

at
l  m 0  p12  p 22   m 2 p 22  m1p12
Y A
1 T 1  L   1 Yl m 2 p 22  m1p12
n   m0 =
2l m 2l A 2l L p12  p 22

 n
1 9  1010  4.9  104
 35Hz.
lic 217. Minimum time interval between two instants
2 1 1  9  103 T
when the string is flat = = 0.5 s
212. As string and tube are in resonance, n1 = n2 2
ub
|n1 – n| = 4 Hz  T=1s
When T increases, n1 also increases. It is given Hence  = v  T = 10  1 = 10 m
that beat frequency decreases to 2 Hz.
218. From the figure,
 n  n1 = 4  n = 4 + n1
Given that, v1 T1
P

n1  ; T2 = 2 T1
n1 = n2 v2 T2
 n = 4 + n2 where, T1 = tension in string AB
3v 3  340 N frequency = n and T2 = tension in string CD

et

n2 = 
4l 4  (3 / 4) v1 T1 1
  =
= 340 Hz A v2 2T1 2
 n = 344 Hz 219.
rg

N reflected transmitted
213. The waves 1 and 3 reach out of phase. Hence
resultant phase difference between them is . 4/

Ta

 Resultant amplitude of 1 and 3 = 10 7 = 3m



This wave has phase difference of with 4 m incident rigid boundary
2
 Resultant amplitude = 32  42  5 m Reflected wave will have a phase inversion of 
while the transmitted wave will not.
214. Because the tuning fork is in resonance with air Hence, yt = (4 mm) sin (5 t + 40 x)
column in the pipe closed at one end, the
10
(2N  1)v 220. y =
frequency is n  where N = 1, 2, 3 .... 10x  t
4l

corresponds to different modes of vibration A = 10 cm,  = cm
Substituting n = 340 Hz, v = 340 m/s, the length 5
of air column in the pipe can be 1
 f = Hz
(2N  1)340 (2N  1) (2N  1)  100 2
l  m  cm
4  340 4 4  Assertion is false but Reason is true.

267

MHT-CET Triumph Physics (Hints)

 2 
221.  = sin t  (x cos   ycos ) 
   Competitive Thinking
represents a wave travelling along a line in x-y
 600
plane through origin making an angle  with x- 1. v=  = 300 m/s
k 2
axis and  with y-axis.
2
 = (x 2  x1 )cos   (y2  y1 )cos  2.
2
y = a sin  2nt  x 
  5 
Comparing with the given equation, we get
For particle velocity vp,
 = 30,  = 60,  = 1 m,  = 30/s
dy  2 
Let (x1, y1)  (2 3 m,2m) and  a  2n cos  2nt  x
dt  5 
(x2, y2)  (3 3 m,3m)
(vp)max = 2na

ns
On substituting the values and simplifying
Comparing with standard equation progressive
we get,
constant,
  = 4 = n  n = 4
2 2
k=  =5
222. y1 = 4 sin (400 t), y2 = 3 sin (404 t)  5

io
Comparing with standard form, y = A sin 2nt Wave velocity v = n = 5 n
we get, (v p ) max 2na 2a
A1 = 4, A2 = 3, n1 = 200, n2 = 202   =
v 5n

at
5
 Beat frequency = n2  n1
= 202  200 = 2 beats/second 3. Given equation of the wave can also be written as,
(A  A 2 ) 2 (7) 2 49
 Intensity ratio = 1 = =   t x  
(A1  A 2 ) Y = 3 sin  2    
2
(1) 2
lic
1
2
  6 10  4 
 A1  4   t x  
I  A 1  1 49 Comparing with y = A sin  2     
223. Using, max = 2  = 3
 =   T   4
1
Imin  A1  1   4  1 
ub
A   3 
(where, x and y are in metre)
 2 
we get,
224. In the given case, the wave must be bounded. 1
A = 3 m, F = = 0.17 Hz,  = 10 m and
225. T
P

v = F = 1.7 m/s
Hence, option (D) is correct.

4. From, y = 60 cos (1800 t  6x)


et

A = 60,  = 1800, k = 6
Velocity of wave propagation is
 1800
rg

 vw = n ; n =  ,
Fundamental frequency (2n + 1) = L 2 2
4
2  2
(2n  1) = 
 f= v k 6
Ta

4L
1800 2
3  vw =  = 300 m/s
For 1st case, l = m 2 6
8
f 680 3 Velocity of particle is
 (2n + 1) =  4l = 4 =3 dy
v 340 8 vp = = 1800  60 cos (1800 t  6x)
 n=1 dt
 Next overtone is for n = 2  v pmax = 1800  60 m/s
Thus,
 v pmax = 1800  60  106 m/s
5 5 1 5
L = =  = m
4 4 2 8 v pmax 1800  60  106
 =
5 3 1 vw 300
 X =  = m = 25 cm
8 8 4 = 360  106
226. String crosses mean position simultaneously. = 3.6 × 104

268

Chapter 06: Superposition of Waves


5. Given, y = 12 sin (5t – 4x) cm 11. The velocity v of a transverse wave is given by,
 5t 4x  T T 1 T
 y = 12 sin 2    v=  
 2 2  m r 2 r 
Comparing above eq. with, where m = mass per unit length of wire.
x t 1
y = A sin 2     v
  T  r
2  For both the wires,
We get,  = cm
4 vA rB 2r
  =2
Relation between phase difference and path vB rA r
difference is
 vA = 2vB
2
 = x Now, v = n.λ

ns
 A 2vB n
 2  
 = x B n vB
2  2 
   λA = 2λB
 4 

The no: of loops (n) in the wire are given as,

io
 x = cm 2l
8 n=

6. Given equation is, n1 2l 

at
y = A sin (100 t + 3x)    B
n 2 2 B 2l
  n1 1
 t x   
= A sin 2    n2 2
  1   2  
  50   3  
lic Hence no: of loops in wire are l and 2
respectively
 Comparing with the standard form, A
t x 2 A
y = A sin 2    we get,  =
ub
T  3 N N N N
 A
A phase difference of rad corresponds to a
3  ratio of no: of antinodes p:q is 1:2
x 1 2 
path difference of m =  = m
P

6 6 3 9 13. Given: l = 83  10–2 cm, v = 332 m/s


v 332
 t x  n0 =   100
7. Comparing equation y = 2 sin2    4L 4  83  102
 0.01 50 
et

n0 : n1 : n2 : n3 : n4 = 1 : 3 : 5 : 7 : 9
 t x = 100 : 300 : 500 : 700 : 900
with y = A sin 2   
T   Number of possible natural frequency = 5.
Here, A = 2 cm
rg

14. In closed organ pipe, for pth mode


1 1
n=   100Hz corresponding frequency is
T 0.01
(np1)c = (2p  1) nc
 = 50 cm
Ta

v
v = n = 100  50 = 5000 cm/s where, nc =
4L
Hence, option (C) is incorrect.
In open organ pipe, for pth mode corresponding
9. For stationary wave, the resultant particle frequency is
velocity at all points is zero. (np  1)0 = p n0
v
 x  where, n0 =
10. y = 2 sin   (cos 48t) 2L
 15 
2  ….(Given length and medium is same for both
= the pipes)
 15
 v 
 = 30 units
Distance between a node and its next antinode, 
 n p 1  0

p
 2L 


2p
 30 n p 1 c
 v  2p  1
 2p  1  
 = 7.5 units
4 4  4L 

269

MHT-CET Triumph Physics (Hints)


15. In a closed pipe, odd harmonics are observed so 22. The air column in a pipe open at both ends can
lengths for resonance are also in sequence of l1, vibrate in a number of different modes subjected
3l1, 5l1,…, where, l1 is the minimum length of the to the boundary condition that there must be an
column for which resonance occurs. antinode at the open end.
 Next length = 3l1 = 3  50 = 150 cm Hence option (A) is correct.
16. For a pipe closed at one end, The ratio of frequencies when pipe is open at
Fundamental frequency, both the ends is given as,
v L/t 1 1 n:2n:3n:4n:5n
n=   = = 25 Hz v
4L 4L 4t 4  0.01 where n =
2L
17. As tube is closed at one end and open at other
 Both odd as well even i.e., All harmonics are
end.
present.
 2n  1 v = 260 Hz

ns
….(i) Hence, option (B) and (C) are correct
4l
Pressure variation is minimum at antinode
 2n  1 v = 220 Hz ….(ii)  Option (D) is incorrect.
4l
Subtracting equation (ii) from equation (i),

io
23. For an open pipe,
2v e = 0.6 d
= 40
4l e
v  d =

at
 Fundamental frequency = = 20 Hz 0.6
2l e
 2r =
18. For 1st resonance, 0.6
v 340
l0 =
4n
=
4  340
= 0.25 = 25 cm
lic  r =
0.8
1.2
2
= cm
3
Next resonance will occur at a distance of
3l0 = 75 cm and further at 5l0 = 125 cm (which 24. Fundamental frequency of closed organ pipe
is not possible). v
ub
=
Hence, h = 120 – 3l0 = 120 – 75 = 45 cm 4L
= 0.45 m v 3v
 
v v 4L 2l0
19. nl+e=
4(l  e) 12  20
P

4n l0 =
v 2
 l= e
= 120 cm
4n
Here, e = (0.6)r = (0.6)(2) = 1.2 cm
et

25. First overtone frequency of a closed pipe


336  10 2
 l=  1.2 = 15.2 cm = second harmonic frequency of an open pipe
4  512
3v 2v
=
rg

v 4l1 2l2
20. Fundamental frequency of open tube, n =
2L l1 3
where v is the velocity of sound in air and L is =
l2 4
the length of the tube
Ta

330 26. Difference between successive resonance


 n= = 660 Hz
2  0.25 frequencies n = 170 Hz
The emitted frequencies are n, 2n, 3n, 4n, … If pipe is open, air column will vibrate with all
i.e., 660 Hz, 1320 Hz, 1980 Hz, 2640 Hz, … harmonics i.e, n1, 2n1, 3n1, …
21. For a closed pipe, fundamental frequency n = n1 = 170 Hz
v But in that case, successive resonance
n1 = = 100 Hz
4L frequencies will be multiples of 170 Hz which
For an open pipe, fundamental frequency contradicts the data given in question.
v If pipe is closed, air column will vibrate with
n1 = = 2n1 = 200 Hz only odd harmonics , i.e., n1, 3n1, 5n1
2L
In an open pipe all multiples of the fundamental  n = 2n1
are produced. Hence, frequencies produced can 170
be 200 Hz, 400 Hz and so on.  n1 = = 85 Hz.
2

270

Chapter 06: Superposition of Waves


In this case 5n1, 7n1 and 9n1 resonance  2(L  d)  2L 
frequencies will correspond to 425, 595 and 765  fb = V  
 4L(L  d) 
Hz respectively as given in the question. Hence,
given pipe is closed pipe and 2d
= V
v 340 4L(L  d)
length of pipe lc =  = 1 m.
4n1 4  85 Vd
 fb =
2L(L + d)
27. Before dipping in water,
v 31. Second overtone of open pipe is third harmonic,
Fundamental frequency, f =
2l 3v
 n3 =
After dipping in water, pipe will get filled with 2l
water partially and will act as closed organ pipe First overtone of closed pipe is third harmonic,
l 3v

ns
of length . n2 =
2 4l
 After dipping in water, here, L be length of open pipe,
v v
Fundamental frequency f  = = =f 
3v
=
3v
l 2l

io
4  2L 4L
2 4L
 L = = 2L
28. For a closed pipe, 2

at
7v
n3 = ....(i) 32. Fundamental frequency of open organ pipe,
4l
v
For an open pipe n1 =
2lo
3v
n2 =
2l
....(ii)
lic Frequency of third harmonic for closed organ
pipe,
According to given condition, we have
3v
7v 3v n2 =
= + 150 ….[from (i) and (ii)] 4lc
4l 2l
ub
7v 3v Given: n1 = n2
  = 150 v 3v
4l 2l  
7v  6v 2lo 4lc
= 150
4l 2lc 2  20
P

 lo =  = 13.33 cm
v 3 3
 = 150
4l
33. For a pipe closed at one end,
Fundamental frequency of pipe open at both
et

v
ends is n1 = and for a pipe open at both ends,
4L1
v
= 2 150  = 300 Hz v v v
2l n2 =  L1 = and L2 =
rg

2L 2 4n1 2n 2
v
29. no = v v
2Lopen For the new pipe, L = L1 + L2 = +
4L1 2 L2
v
nc = 3 
Ta

v v n1n 2
4Lclosed n= = =
4L  v v  2n1  n 2
v  v  4  
nc = 3   3  4n
 1 2n 1 
Lopen 
4  2Lopen 
2 v
34. n1 =
= 3  100 = 300 Hz 2(l1  2e)
30. For a pipe open at both ends,  v = 2n1 (l1 +2e) ….(i)
V v
f= n2 =
2L 2(l2  2e)
V V  v = 2n2 (l2 + 2e) ….(ii)
 f1 = , f2 =
2L 2(L  d) From equation (i) and (ii), we get
V V n 2l2  n1l1
 beat frequency fb = f1  f2 =  e=
2L 2(L  d) 2(n1  n 2 )

271

MHT-CET Triumph Physics (Hints)


35. For a closed organ pipe, the frequency of  Beats produced = n  n
v 3v v
fundamental mode is nc = = 
4L c 2L 12L
For an open organ pipe, the frequency of 17 v 17
=  =  12 ….[From (i)]
v 12 L 12
fundamental mode is n0 =
2L o = 17
Lc = Lo ….[Given] 38. For an organ pipe open at both ends.
 n0 = 2nc …(i)
n0 – nc = 2 [Given] …(ii)
 Solving equations (i) and (ii), we get,
A
n0 = 4 Hz, nc = 2 Hz
When the length of the open pipe is halved, its N

ns
frequency of fundamental mode is A
v
n0  N
 L0 
2  A
 2 

io
= 2n0 = 2  4 Hz = 8 Hz N
When the length of the closed pipe is doubled, A

at
its frequency of fundamental mode is N
v 1 1 A
nc   n c =  2Hz = 1 Hz
4  2Lc  2 2
Hence, number of beats produced per second
= n0  nc = 8 – 1 = 7.
lic For third overtone, the pipe has 4 nodes and 5
antinodes
(2n  1)v
36. Here n =  1250 l2  3l1 74.1 3 24.1
4L 39. e= =
ub
2 2
(2n  1)  340
  1250 e = 0.9
0.85  4 But e = 0.3d
 2n – 1  12.5  n  6.75 e 0.9
  d=  = 3 cm
P

Number of possible oscillations is 6. 0.3 0.3

37. For open pipe first overtone, n1 =


v 40. v = 2f(l2  l1)
L l1 = 9.75 cm = 0.0975 m,
et

3v l2 = 31.25 cm = 0.3125m,
For closed pipe first overtone, n1 =
4L f = 800 Hz
 Number of beats produced are,  v = 2  800(0.3125  0.0975)
rg

v 3v  v = 2  800(0.215)
n1  n1 =  =3  v = 344 m/s
L 4L
v 1 T 1 1
 =3 41. n=  n  and v  n  v 
Ta

4L 2lr  r r
v
 = 12 ….(i)
L 1
42. Velocity of transverse wave on string, V 
L r
When length of open pipe is made , the 1
3 VA 
 ….(i)
fundamental frequency becomes, rA
v 3v 1
n= = VB  .…(ii)
 L  2L rB
2 
3 Divide equation (i) by equation (ii)
When length of closed pipe is made 3 times, the VA r r
 = B = B .…{ rA = 2rB}
fundamental frequency becomes, VB rA 2rB
v v VA 1
n = =  =
4(3L) 12L VB 2

272

Chapter 06: Superposition of Waves


1 T
43. n 48. We have, v =
lr m
 n1l1r1 = n2r2l2 T = v2 m ….(i)
n1l1r1 = n2 2r1  2l1  m = 2  102
 n1 = 4n2

n1  K=
 n2 = v
4

44. In fifth overtone, number of loops = 6 v=
K
 Length of 6 loops = 2.4 m
120
2.4 v=
 Length of each loop = = 0.4 m 2 / 3
6
v = 180 m/lE

ns
 Distance between a node and antinode is half of
0.4 From equation (i)
length of loop = = 0.2 m T = (180)2  2  102
2
T = 648 N
45. n T

io
1 1 1
n T 49. l1 : l2 : l3 = : : =6:3:2
 n1 n 2 n 3
n T

at
6
n 2T l1 =  99 = 54
 11
n T
3
n = 2 n l2 =  99 = 27
11
46.
lic 2
l3 =  99 = 18
11
m1
50. Fundamental frequency of the first wire is
ub
1 T 1 T 1 T
n= = =
2l1 m 2l1 r1 
2
2l1r1 
m2
1 T
The first overtone n1 = 2n =
P

Let velocity of pulse at lower end be v1 and at l1r1 


top be v2
Similarly, the second overtone of the second
 2 v2 v wire will be,
  (  = and n = constant)
et

1 v1 n
3 T
velocity of transverse wave on string n2 =
2l2 r2 
T
v= Given that n1 = n2
rg

m
where, m is linear density. 1 T 3 T
 =
l1r1  2l2 r2 
In this case, v  T
 m 2  m1   3l1r1 = 2l2r2
Ta

 2 v2 T2
    l1 2r
1 v1 T1 m2 = 2
l2 3r1
Where, T2 is tension at upper end of rope
and T1 is tension at lower end of rope. 2r2
= ….( r1 = 2r2)
3( 2r2 )
n T 1
47. = =
2L m 3
….(where ‘m’ is mass per unit length)
M 1 T 1
But, m = 51. = 
L 2l m l
n T n T l = l1 + l2 + l3 ….(Given)
 = =
2   2
M 2 ML 1 1 1 1
 L    
L  1  2  3

273

MHT-CET Triumph Physics (Hints)


52. Let the length of original string is l Now, np  T
l = l1 + l2 + l3 ….(i) Also, tension of piano wire is increasing so np
V increases.
 n=
2l Hence, if np > nf then beat frequency increases
V with increase in tension, which contradicts the
n1 =
2l1 given data.
V  nf > np
n2 =
2l2  nP = nf – x = 256 – 5 Hz.
V 57. Let n be frequency of tuning fork.
n3 =
2l3 Let n1, n2 be frequency of wire at tension T1, T2
From equation (i), respectively.
V V V V n  T …(i)
  

ns
2n 2n1 2n 2 2n 3 n1 = n – 6 …(ii)
1 1 1 1 n2 = n + 6 …(iii)
  
n n1 n 2 n 3 n1 T1 225 15
 = = =

io
53. String vibrating in second overtone forms four n2 T2 256 16
nodes and three antinodes as shown, n  6 15
L L 5L
 = …from (i), (ii), (iii)
L 2L n+6 16

at
6 2 6  16n – 96 = 15n + 90
3 3
 n = 186 Hz

L 58. If m frequencies are arranged in increasing

54. n=
1 T
lic order, then,
nm = n1 + (m  1)X
2L r 2d where X = no. of beats produced.
1 4T  here,
n =
ub
2L r2d n3 = n1 + (2)X
n 2L r  n + 1 = n  1 + 2X
   ….(i)
n L r 2X = 2
 mass remains the same  X=1
P

r L 59. Let the frequency of first fork be ‘n’ then



r  L frequency of 56th fork will be
Substituting in eq. (i) n = n + 4  55
et

n L this is because each successive tuning fork is


2 separated by 4 Hz in frequency from the
n L
previous one.
 L > L Also, n = 3n ….(given)
rg

 n < 2 n  3n = n + 4  55
55. nx = 300 Hz  n = 110 Hz
x = beat frequency = 4 Hz, which is decreasing
a = 250  4 = 254 Hz or 246 Hz
Ta

60.
after increasing the tension of the string Y.
b = 513  5 = 518 Hz or 508 Hz
Also,  n  T , tension of wire Y increases so
Now, b = 2a
ny increases Which is 508 = 2(254)
Hence, if ny > nx
  = 254 Hz
beat frequency increases, which contradicts the
data. 61. Let n be the frequency of fork C
 ny < nx 3n 103n 2n 98n
 nA  n   and n B  n  
 nx  ny = x 100 100 100 100
n y  n x  x  300  4  296 Hz 5n
But n A  n B  5   5  n  100Hz
56. Suppose np = frequency of piano 100
nf = Frequency of tuning fork = 256 Hz (103)(100)
 nA 
x = Beat frequency = 5 b.p.s., which is decreasing 100
after changing the tension of piano wire. = 103 Hz
274

Chapter 06: Superposition of Waves

1 T For string of length L, integrating over,


62. n= L t
2l M dx
n 1 T

0 xg 0
 dt
 
n 2 T t
1
L

T n 9 2
  dt =
g 0
x 1/ 2 dx
  2 =2 = = 0.02 0
T n 900 100 20
1  x1/ 2 
 t=   ( L = 20 m)
64. v = 4nl ….(i) g 1 / 2  0
P 2
v= ….(ii) =  20 = 2 2 s
 10
P
 = 4nl ....[From equation (i) and (ii)]   t x 
 68. y = 0.02 sin  2   
0.04 0.50 

ns
  
84  4 
2
 1.2  1 
 =
1.0  105 T  T  

0.04 
Using, v =  
= 1.354  1.4 m k 0.04  1 

io
 
 0.50 
65. na = 250  4 = 254 Hz or 246 Hz 2
nb = 513  5  518 Hz or 508 Hz  0.50  2
 T=    0.04 = (12.5)  0.04
Now, nb = 2na  0.04 

at
Which is 508 = 2(254) . = 6.25 N.
 n = 254 Hz 69. As string is clamped resulting wave is a
n1  n2 = 6 standing wave of equation y = 2A sin kx cost
66.
1 T 1 T
lic Comparing with given equation,
  6 2
2l m 2l m  = 60 and k =
3
1 T  60
  600 = 6
ub
2l m Now velocity v = = = 90 m/s
k 2
1 T 3
 = 606 ….(i)
2l m Also, velocity of transverse wave,
1 T T T
P

also, = 600 ….(ii) v= =


2l m m M/L
Dividing Equation (i) by Equation (ii), we get M 902  3  102
 T = v2  = = 162 N
 1 T  L 1.5
et

  606
 2l m   70. For a resonance tube experiment, difference
 1 T  600 between lengths of column for two successive
  resonances is given by,
 2l m 
rg

T T  v
 = (1.01)  = (1.02) Ln+1 – Ln = 
T T 2 2n
 v = 2n(Ln+1 – Ln) = 2  320  (0.73 – 0.20)
 T = T(1.02)
Ta

= 339.2 m/s
Increase in tension,
T = T  1.02  T = (0.02T) 71. A A
T N
 Fractional increase in the tension, = 0.02
T

T
67. Velocity of transverse string v = , where, m 1
m 
is linear density. L = 2 60 cm
Tension T = Mg = mxg v
Fundamental frequency, 0 =
mxg 1
 v=
m here, 1 = 2L
dx Y
 xg Also, v =
dt 

275

MHT-CET Triumph Physics (Hints)

v 1 Y M M 6  103 kg
 0 =  74. = = = 6 2
1 2L  V AL 10 m  0.6 m
kg
 0 =
1

9.27  1010  = 104
m3
2  60  102 2.7  103
= 4.88  103 Hz  5 kHz T
As v = ….(i)

1 T where T: Tension in wire
72. f =
2l m : linear mass density
1 stress  A l
=  T = YA ….(ii)
2l M/L L
M V AL
=
1 stress
=
1 stress And  = = =
L L L

ns
2l M/V 2l density
=  A ….(iii)
1  strain 1 2.2 1011  0.01 from (i), (ii) and (iii),
= =
2l density 2 (1.5) 7.7 103 L
YA
 178.2 Hz L = Yl

io
v=
A L
73. 2 f1 = 600  f1 = 300 and
squaring and rearranging, we get,
2 f2 = 608  f2 = 304
v2    L 90 2 104  0.6

at
 |f1  f2| = 4 beats L = = = 3  105 m
Y 16  1011
I max (A1  A 2 ) 2 (5  4) 2 81
     L = 0.03 mm
I min (A1  A 2 ) 2 (5  4) 2 1 lic
Hints to Evaluation Test

1. Direction reverses after reflection and phase 4. A node will be formed in the middle with two
ub
difference introduced after each reflection antinodes at the ends of the pipe. Pressure
depending upon nature of support. antinodes are displacement nodes.
2. Given equation is, 3
5. k= and  = 300
y = y0 sin 2  f t  
x 2
P

  4
dy  x   = m and f = 150 Hz
 = [y0 cos2  f t   2f
3
dt   2 2
…[  = and f = ]
et

 dy  K 
   = y02f
 dt max x = 0 is pressure maximum, hence a node.
Given, maximum particle velocity = 4 times  It is closed at x = 0

rg

wave velocity (f) For a pipe closed at one end, L = (2n + 1)


 y0 2f = 4f     = (y0)/2 4
3. Let f = 250 Hz, then f – 2 = 248 Hz, n
For a pipe closed at both ends, L =
f + 2 = 252 Hz 2
Ta

At x = 0, Let us check for x = 2m,


y = y1 + y2 + y3 = A sin 2 (f + 2) t n
=2
+ A sin 2 (f  2) t + A sin 2ft 2
 y = 2A sin 2 ft cos 4 t + A sin 2 ft  n = 3 which is valid.
 y = A (2 cos 4 t + 1) sin 2ft  The pipe is closed at x = 2m
Intensity, I  R2, I = KA2 (2 cos 4t + 1)2 6. Options (C) and (D) will not form a standing
For maximum and minimum intensity, wave.
dI (A): At x = 0, it has amplitude = 0
 0  2KA 2 (1 + 2 cos 4t) (  sin 4t)
dt  Sum of the two amplitudes will be ‘a’ which is
1 1 3 1 not the condition of the problem.
 t = 0, , , ,1  t 
4 2 4 4 (B): At x = 0, it has amplitude =  a
1 which will cancel out to give zero.
Beat frequency = = 4 Hz
t Hence, option (B) is correct.

276

Chapter 06: Superposition of Waves


7. 17. L = 45 = 5  9
l/2
L = 99 = 11  9
Hence other lengths between these values are,
l/4
L1 = 7  9 = 63 cm
l L2 = 9  9 = 81 cm
String vibrates with two loops. (Second So fundamental length is 9 cm
Harmonic) 
 9=   = 9  4 = 36 cm
The point where we touch the string becomes a 4
node and where we pluck it becomes an 18. For stationary waves, the distance between
antinode. 
successive nodes and antinodes is always .
5 4
8. v = f, l =
2

ns
1 T
 2l  2  82.5  19. Using, n =
 2L m
v =   f =    1000
5
  5  100  T
 330 m/s  For given m, n 
L

io
9. Comparing the given equation with n1 L T1 1 1 1
 = 2 = =
y = A sin(t  kx) we get,  = 3000  n2 L1 T2 4 4 8
n =  = 1500 Hz  n2 = 8n1 = 8  200 = 1600 Hz

at

2 22. By comparing the given equation with standard
and k = 2 = 12   = 1 m form, we get
 6 A = 0.05 m,  = 40  rad/s
Using v = n,
lic  vmax x  0.375 = A = 0.05  40 = 2 m/s
v = 1500  1 = 250 m/s 23. In this case, n(2) = (n + 1) (1.6)
6
n 1 2 5
  = =
ub
11. Since  = , n 1.6 4
2  5n = 4n + 4
 A= A12  A 22 = (4) 2  (3) 2 = 5  n=4
 L = 8.0 cm
13. Velocity, v = n ,
P

v 1200 24. If x is at an angle .


= = =4m The  between x and 1 = 2,
n 300
the  between x and 2 = 2 and
 The distance between a node and the
et

the  between x and 3 = 2



neighbouring antinode is = 1 m.  points x and 3 are in phase.
4
 
15. It is given that 25. L = (n + 1) and = d
rg

2 4
First overtone of closed pipe = First overtone of
 L = 2(n + 1)d
open pipe
 v   v  26. The frequency of the wire remains the same.
 3  = 2 ;
Ta

 4L1   2L 2  p T
n=
where L1 and L2 are the lengths of closed and 2L 
open organ pipes. p1 p2
 =
L1 3 l  4l 4
 =
L2 4 p1 1
 =
p2 8
16. Since they are turned to same pitch,
fundamental frequencies are same, no = nc 2l 2(4l )
 = = =l
v v p 8
 =
2L o 4L c
Lo 4
 = =2:1
Lc 2
 Lo : Lc : : 2 : 1

277
7 Wave Optics

Hints

c a

ns
Now, ang =
Classical Thinking vg  g
7. Light is electromagnetic in nature. It does not a 7500
 g = = = 5000 Å
require any material medium for its propagation. a ng 1.5

io
12. Direction of wave is perpendicular to the  a  g = 7500 – 5000 = 2500 Å
wavefront. = 2500  1010 m = 2.5  107 m

at
20. From Huygens’ principle, if the incident va  a
30. an m = 
wavefront be parallel to the interface of the two vm m
media (i = 0), then the refracted wavefront will
a 3
also be parallel to the interface (r = 0). In other  = 1.5 =
m
words, if light rays fall normally on the
interface, then on passing to the second
lic m 2
2
   a 2  3 1
   m  
medium, they will not deviate from their a 3 a 3 3
original path. 1
 Percentage change =  100
ub
24. Speed of light in glass depends upon the colour 3
of the light. Violet colour travels faster than the = 33.33% (in magnitude)
red light in a glass prism.
32. Ultrasonic waves are longitudinal waves.
This is because refractive index of glass for
P

violet colour is less than that for red. 35. The magnitude of electric field vector varies
periodically with time because it is the form of
sin30 1
25. sin r =  = 0.3125 electromagnetic wave.
1.6 3.2
et

 r = 18 43. Using Malus’ law,


I I
λ 5460 I = cos 2  
26. g= a  = 3640 Å 2 6
rg

n 1.5 1
 cos  =
27. vg = 2  108 m/s, vw = 2.25  108 m/s 3
vg 2 10
8   = 55
 gnw =  = 0.89
Ta

vw 2.25  108 46. According to Brewster’s law, when a beam of


ordinary light (i.e. unpolarised) is reflected from
c sin i
28. n= = a transparent medium (like glass), the reflected
v sin r light is completely plane polarised at angle of
sin r sin 30 polarisation.
 v=c = 3  108 
sin i sin 45
47. At polarizing angle, the reflected and refracted
28 rays are mutually perpendicular.
= 3  10  = 2.12  108 m/s
2
48. Let i, r and r be the angle of incidence,
29. Using c = ν , reflection and refraction respectively.
c 3  108 Let r + r = 90
a = = = 0.75  106 m
v 4  1014  r = 90  30 = 60
 a = 7500 Å  i = r = 60
278

Chapter 07: Wave optics

49. n= 3 = tan θB 
76. Path difference = 5 = 10 
 θB = tan 1
 3  = 60  Point is bright.
2

 Using, yn = nW we get,
50. tan θB = n = 1.55
0.5 = 5W  W = 0.1 mm
 θB = 5710
r = 90 θB = 90  5717 = 3249 D
77. W=  W   for the same set-up.
d
52. For interference, phase difference must be W1 1
constant.  
W2  2
54. For destructive interference, path difference is 1.0 5000

 
odd multiple of . W2 6000
2

ns
6000
 W2 = = 1.2 mm
5000
56. I  (amplitude)2
78. W
D
62. Fringe width (W) = W2  

io
d  = 2  W2 = W1  2
W1 1 1
 W 
4800
As red > yellow, hence fringe width will = 0.32 
6400

at
increase.
= 0.24 mm
63. For interference, wavelength of two interfering  Change in W = 0.32  0.24
waves must be same. = 0.08 mm = 8  105 m
64.
lic
If slit width increases, the contrast between the 79. W=
D
d=
D
fringes decreases. For very large width, uniform d W
illumination occurs. 6000  10  (40  102 )
10
 d= = 0.2 cm
0.012  102
ub
71. Let a1 and a2 be amplitudes of two waves.
a1 4 D D'
 80. W= and W =
a2 3 d d'
a1  a 2 43 7 d
 = = But d = and D = 2 D
P

a1  a 2 43 1 2
 (2D) D
 a1  a 2 
2
49  W = =4 = 4W
   = (d / 2) d
 a1  a 2  1
et

 Fringe width will become four-times.


 81. Distance of nth dark fringe from central fringe,
72. Path difference = 12.5  = 25   (2n  1) D
2 yn =
rg

 2d
 odd multiple of (2  2  1) D 3 D
2  y2 = =
2d 2d
 destructive interference
3   1
Ta

3
 1  10 =   = 6  105 cm
73. Path difference = 29 2  0.9  103
 82. Distance of 6th bright fringe,
= 58
2 nD 6D
 y6 = =
= even multiple of d d
2 Distance of 4th dark fringe,
 point is bright (2n  1)D 7 D
y4 = =
2d 2 d
  
74. x = 260 = 130 = even multiple of D  7  5 D
4 2 2  y6  y4 = 6   =
d  2 2 d
 point is bright.
5 4 107 1
= 
75. For maxima, path difference, l = n 2 1103
 For n = 1, l =  = 6320 Å = 103 m = 1 mm
279

MHT-CET Triumph Physics (Hints)


83. Using Shortcut 4, 104. The angular half width of the central maxima is
n11 = n22 given by,
 62  5893 = n2  4358 
sin  = 
 n2  84 a
6328  1010
84. Using Shortcut 4,  = rad
0.2  103
n2 
n11 = n22  = 1 6328  1010  180
n1 2 = degree = 0.18
0.2  103  
n2 5898
 =  n2 = 99  Total width of central maxima = 2 = 0.36
92 5461
105. Distance between the first dark fringes on either
 side of central maxima = width of central
85.  = ;  can be increased by increasing , so 
d 2D 2  600  109  2

ns
has to be increased by 10% maxima = =
d 1  103
10 = 2.4 mm
 Increase in  =  5890 = 589 Å
100
1.22 y
110. d = =

io
86. From Shortcut 6, a d
N 1.22d 1.22  5  107  103
t=  y= =
n 1 a 10  102

at
Here N = 3 = 6.1 10–3 m
5
3  5.4510 = 6.1 mm  6 mm = 0.6 cm
 t= = 32.7  105 cm
1.5  1
111. Resolving power of telescope,
88.
lic
Colours of thin film are due to interference of
R.P. = 
 d 

1.22
light.  1.22   1.22  5000  1010
(n  1)t  D  R.P. = 2  106
91. Shift in the fringe pattern 
ub
d
5 2
(1.5  1)  2.5  10  100  10 Critical Thinking
= = 2.5 cm
0.5  103
2. When the point source or linear source of light
92. Fringe shift, is placed at very large distance, a small portion
P

W of spherical or cylindrical wavefront appears to


W0 = (n  1) t
 be plane and is called a plane wavefront.
W
= (1.5  1)  2  106
et

(5000  1010 )
= 2W
i.e., The central bright maximum will shift
rg

2 fringes upwards.
2
93. 2 = (where d = slit width)
d
Ta

 As d decreases,  increases. Amongst the given options none of the sources


generates plane wavefront, it can be artificially
D produced by reflection from a mirror or by
98. For a diffraction pattern, W =
a refraction through a lens.
1
 W , where a is slit width. 4. The plane wavefront with the ray at the
a periphery has to travel least distance through the
  lens whereas the ray along the principal axis has
103. For first minima,  = or a = to travel thickness of the lens hence this is
a 
delayed than the peripheral ray. This results in a
6500  108  6 
 a= ….( 30 = radian) spherical converging wavefront.
 6
= 1.24  104 cm 5. i = 60 , r = (60  15) = 45
= 1.24  106 m  n=
sin i
=
3

2
= 1.22
= 1.24 micron sin r 2 1

280

Chapter 07: Wave optics


6. 1 sin  = 2 sin  = 3 sin  = 4 sin  11. From the figure,
B
As AB and CD are parallel,  =  A
r
 1 = 4  BOC = 90 i
90  r
P O Q
AB CD  90  r
7. = cos i, = cos r  n g  a  1.5 r
AD AD g
AB cos i C
 =
CD cos r sin i sin 45o
12. a ng   1.5 =
sin r sin r
8. Let g (in cm) and w (in cm) be the 1
wavelengths in glass and water. By definition,  sin r =  r = 287
1.5 2
in a distance  there is one wave. Therefore,
cosi cos 45
Number of waves in 8 cm of glass = 8/g, Ratio of widths = 

ns
cos r cos 287
Number of waves in 10 cm of water = 10/w
....(Refer Notes 1)
8 10 w 5
Thus,    1
g  w g 4  Ratio of widths = 0.801 =
1.2475

io
c c
Now, ng = and nw =
vg vw sin i
13. n=
sin r
ng v w n w  w

at
 = = = sin i sin 35 0.5736
nw vg n  g  g sin r = = =
 1.5 1.5
w 5 4 5
 ng =  nw =  =  sin r = 0.3824
g 4 3 3
lic  r = 22.48 = 22 29
5 vd 5 3 v 3 W2 cos 22.48
9. vd = c = ; vw = c w =  Required ratio =   1.13
12 c 12 4 c 4 W1 cos35
ub
nd c / vd 12 / 5 12 3 9 ....(Refer Notes 1)
wnd = = = =  =
nw c / vw 4/3 5 4 5
2 3
sin i 14. vd = c vw = c
Using, wnd = 5 4
sin r
c 5 c 4
P

9 9 1 9   = nd   nw
 sin i = wnd  sin r =  sin 30 =  = vd 2 vw 3
5 5 2 10
 d 5 / 2 15
 wnd =  
i = sin–1  
9
 w 4 / 3 8
et

10  
sin i sin i
 wnd = 
10. sin r sin 30
i
O
15 sin i
rg

 
8 sin 30
(i – r)
r t 15 1 15
M  sin i =  
8 2 16
Ta

x  15 
NA  i = sin1  
 16 
x
In OAM, sin (i – r) = 15. In the propagation of e.m. waves, plane of
OA
polarisation contains the direction of
 x = OA sin (i – r) ….(i)
propagation.
ON
OAN, cos r =
OA 17. When unpolarised light is made incident at
t polarising angle, the reflected light is plane
 OA = ….(ii)
cos r polarised in a direction perpendicular to the
From (i) and (ii), plane of incidence.

t sin (i  r) Therefore, E in reflected light will vibrate in
x =
cos r vertical plane with respect to plane of incidence.

281

MHT-CET Triumph Physics (Hints)


18. Using law of Malus, If I0 is the intensity of the incident unpolarised
Intensity of light transmitted from 1st polaroid, light, then the intensity after passing the first
I1 = E2 /2 polariser,
 Intensity of emergent light, I1 = I0 (cos2 ) =
I0
I2 = I1 cos2 50 = (E2 cos2 50)/ 2 = 0.21 E2 2
I2 = I1 cos2  and
19. If the intensity of the unpolarised light in the
I3 = I2 cos2  = I2 cos2 (90  ) = I2 sin2 
incident beam = I0, then the intensity of the
 I3 = (I1 cos2 ) sin2 
unpolarised component transmitted is same for
I1 I 2
all orientation of the polarising sheet  I3 = sin 2 2  0 sin 2
4 8
 I0 
 I0    Now, I3 = 2 Wm2 and I0 = 32 Wm2
2
32
Using Malus’ law, transmitted intensity of the  2= sin2 2

ns
8
polarised light component
1 1
Ip  I p cos 2   sin2 2 = or sin 2 =
2 2
 (Ip ) max = Ip for  = 0 and  2 = 45 or  = 22.5

io
 22. From Brewster’s law, n = tan θB
(Ip ) min = 0 for  =
2 c
Now the maximum transmitted intensity n= = tan 60 = 3
v

at
I0
= Ip + c 3  108 8
2  v=   3  10 m/s
3 3
I0
and the minimum transmitted intensity =
It is given that,
lic 2 23. From the figure,
i + r = 90  r = 90  i
I0 I  sin i sin i
Ip +  3 0   = = = tan i
2 2 sin r sin (90  i)
ub
Ip 1
Ip = I0  =1:1  sin ic = = cot i
I0 

20. Let I0 be the intensity of unpolarised light. The 24. i + i' = 90
 i = 45 ….( i = i')
P

I0
intensity transmitted by the first sheet is .
2 sin i 3
n= 
 I0  I 0 sin r 2
 Transmitted intensity =  I0  = .
et

 2 2 2 2 1 2
 sin r =  sin i =  =
This will be the intensity of incident light on the 3 3 2 3
second polaroid. The intensity transmitted by the
25. =ir
I 
rg

second polaroid will be  0  cos2  but i = θB


2  θB  r =  = 24 ....(i)
where  is the angle between their axes θB + r = 90 ....(ii)
9 Solving equations (i) and (ii),
Ta

sin  =
15 θB = 57
12
 cos  = ….[ 1  sin2  = cos2 ] 26. Here θB + 90 + r = 180
15
 I0   I0   12 
2
8 i.e., θB = 90 – r
 2 θB
  cos  =     = I0
As θB – r = 34
2
  2
    15 25
Ratio of intensity of emergent light to that of  90 – r – r = 34
8 r
unpolarised light = i.e., 2 r = 56  r = 28
25

21. Let  be the angle between the first two 27. The refractive index of air is slightly more than
polarisers and  be the angle between the next 1. When chamber is evacuated, refractive index
two. Here, decreases and hence the wavelength increases
 +  = 90 and fringe width also increases.
282

Chapter 07: Wave optics


28. Interference occurs in longitudinal as well as But y = b/2
transverse waves. The choices (A), (B) and (D) are b D
 = (2n  1)
conditions for sustained or permanent interference. 2 2b
2
29. In interference between waves of equal b
 =
amplitudes ‘a’, the minimum intensity is zero (2n  1)D
and the maximum intensity is proportional to b2 b2
4a2. For waves of unequal amplitudes ‘a’ and A  For n = 1,2,…;  = , ...
D 3D
(A > a), the minimum intensity is non-zero and
the maximum intensity is proportional to 36. P is the position of 11th bright fringe from Q.
(a + A)2, which is greater than 4a2. From central position O, P will be the position
of 10th bright fringe.
30. Contrast between the bright and dark fringes Path difference between the waves reaching
will be reduced. P = S1B =10  = 10  6000  10–10 = 6  10–6 m.

ns
31. Two independent light sources cannot be 37. For dark fringe at P,
coherent because they cannot generate waves
S1P  S2P = l = (2n  1)/2
having a constant phase difference.
Here, n = 3 and  = 6000

io
32. y1 = a sin t and 5 6000
 l = =5 = 15000 Å = 1.5 micron
  2 2
y2 = b cos t = b sin   t  
 2  38. Distance between nth bright fringe and mth dark

at
So phase difference,  = /2 fringe (n > m)
2  1
33. For maxima, 2 n  (XO)  2l x =  n  m   W
  2 


2
(XO)  2(n  l ) or (XO) = (n + l)
lic 
= n  m  
1  D
  2 d
34. The interference patterns due to different  1 6.5  107 1
= 5  3     1.63 mm
component colours of white light overlap. The  2  1 103
ub
central bright fringes for different colours are at

the same position. Hence, the central fringe is 39. = ,
white. For a point P for which S2P  S1P = b/2 3
where b(= 4000 Å) represents wavelength of Let a1 and a2 be amplitudes of two waves.
P

blue light, the blue component will be absent  a1 = 4, a2 = 3


and the fringe will appear red in colour. Slightly Resultant amplitude, a = a12  a 22  2a1a 2 cos 
r
farther away where S2Q  S1Q = b = where  a= 37  6
et

2
r (= 8000 Å) is the wavelength for the red 40. Resultant intensity,
colour, the fringe will be predominantly blue. IR = I1 + I2 + 2 I1I 2 cos 
Thus, the fringe closest on either side of the For maximum IR,  = 0
rg

central white fringe is red and the farthest will


 
2
appear blue.  IR = I1 + I2 + 2 I1I 2 = I1  I2

35. In the given situation, 41.


Ta

D S1 P
y = (2n  1)
2d
D d = 2
= (2n  1) 6 d = 12 
2b S2
x
….( ‘missing wavelength’  minima
Let x = S2P – S1P
and here, d = b)
S1  (S2P)2 = (S1P)2 + (S1S2)2
y = (12)2 + (2)2
b = 1442 + 42
= 1482
S2
 S2P = 12.17

 x = 12.17 – 12 = 0.17 
D 6

283

MHT-CET Triumph Physics (Hints)


2 x 2  46. Fringe width,
  = = =
 6 3 D W 
W=  =
 60  d D d
 I = Imax cos2 = I0 cos2  
2  2  For sharp fringes, S < W
3 S W 
 I = I0 cos2 30 = I0  < =
4 D D d
S 
 
42. Applying IR = I1 + I2 + 2 I1I2 cos , at central D d
fringe (where  = 0) we get, 47. Using,
IR = I1 + I1 + 2I1 = 4I1 D 50001010 1.2
Phase difference at a distance x when path W= =
d 0.5103
xd
difference becomes , is given by = 12  104 m = 1.2 mm

ns
D
3
2 xd  Number of fringes = = 2.5
 = × 1.2
 D
 Phase difference,
2xd 
 IR = I1 + I1 + 2I1 cos   = 2n = 2  2.5  = 5 radian

io

 d 
D
I I I  2xd  48. W= W D
=   2 cos   d
4 4 4  D 

at
W1 D1 W  W2 D1  D 2
I 2xd  2  xd     1 
or IR = 1 cos  = I cos   W2 D 2 W2 D2
2 D   D 
W W2  2
  
43.

lic
Let the amplitudes of the two waves be a1 and a2
a12  4I and a 22  I
D D 2 d 2
3  105
 2   103  6  107 m  6000 Å
Let amplitude of the new wave = a 5  102
 a2  3I 49. The fringe width between first and seventh
ub
Let K be the constant of proportionality bright fringes is
 a12 = K (4I), a 22 = K(I) D
W = (7 – 1)
a2 = K (3I) d
a2 = a12 + a 22 + 2 a1a2 cos 
10
 = 6  6000  103  1.0
P

(where  is the phase angle) 1.2  10


36  107
K (3I) = K (4I) + KI + 2 K(4I) KI cos  = = 3  103 = 0.003 m
12  104
et

 3 = 4 + 1 + 4 cos 
1 50. For nth maxima in Young’s double slit
 cos  = experiment,
2
nD yd (103 m)(2 103 m)
  = 120 y= or  = =
rg

d nD n(2m)
44. Resultant intensity,
100001010 m 10000
I = I1  I2  2 I1I 2 cos   = = Å
n n
Ta

At central position with coherent source, But 3500 Å <  < 7000 Å
Icoh  4I0 ( I1 = I2 = I0) ….(i) For n = 1, 2, 3
In case of incoherent at a given point,  varies  = 10000 Å, 5000 Å, 3333.3 Å
randomly with time  (cos )av = 0 For n = 2,  = 5000 Å lies between 3500 Å to
7000 Å. The other wavelengths cannot fulfil this
 I In coh  I1  I 2  2I 0 ….(ii)
condition.
Icoh 2
  ….[from (i) and (ii)] 51. For dark fringes,
I Incoh 1
D
yn = (2n – 1)
45. Fringe width of maximum just opposite to slit, 2d
nD d 3 D
Wn = = For n = 2, 3 = …(i)
d 2 2 d
d2 D
n= For bright fringe, yn = n
2 D d

284

Chapter 07: Wave optics


D D
 y4 = 4 …(ii) 59. Fringe width, W =
d d
From equations (i) and (ii), Half-angular width of central bright portion,
y4 4  D 2d 
= =
3 d 3 D a
y4 8 Overlapping length,
 =  y4 = 8 mm 2D
3 3 y = (2) D  d = d
a
3
52. d = d1d 2 = 4.5  2  106 = 3  10 m Number of bright fringes
D 10
6000  10  1  2 D 
d
 W = = y  a  = (2D  da)d
d 3  103 = 
W D / d aD
= 2  104 m = 0.2 mm

ns
60. In case of an excessively thin film, the path
53. D = 1 m, d = 1 mm, v = 40 cm, u = 60 cm

v 40 difference is . As the path difference between
 d1 = d=  1 mm = 0.67 mm 2
u 60

io
two rays is , the film appears dark.
u 0 2
54. d = d1 = 1.2  = 0.3 cm = 3 mm
v 80 61. In the presence of thin glass plate, the fringe

at
55. Using Shortcut 4, pattern shifts but no change in fringe width
n1 1 = n2 2 occurs.
 n1(2500) = n2(3500) 62. Path difference at P,
n 3500 7 S1
 1 =
n2 2500
=
5
lic = (S1P + (n  1)t)  S2P

So we can say, 7th order of 1st source coincides = (S1P  S2P) + (n  1)t S2
with 5th order of 2nd source. P
ub
56. Using Shortcut 4,
(n + 1) g = n r tD tD 5  D
63. (n1 – 1)  (n2 – 1) =
 (n + 1)  5200 = n  6500 d d d
 52n + 52 = 65n tD 5 D
 [(1.7 – 1)  (1.4 – 1)] =
P

n=4 d d
0.3tD 5  D
57. Using relation, d sin = n we get,  =
d d
n 5  4800  1010
sin  = 5
et

d  t= =
0.3 0.3
 For n = 3, = 8  106 m
3
sin  = = 8  103 mm
rg

d
3  589  109 64.  = 600 nm = 600  10–9 m
= = 3  106 t = 18 m = 18  10–6 m
0.589
tD
  = sin1(3  106) Shift, S = (n – 1) ….(i)
Ta

d
58. In Young’s double slit experiment, Fringe width,
sin  =  = (y/D), so  = (y/D) D
W= ….(ii)
Hence, angular fringe width 0 =  d
(with y = W) will be (n  1)t  W
 From equations (i) and (ii), S =
W D 1  
0 = =  =
D d D d S (n  1)t
 No. of fringes = =
Here 0 = 1 = (/180) rad and W 
 = 6000 Å = 6  10–7 m (1.6  1)  18  106
 180 =
 d= =  6  10–7 600  109
0  0.6  18  103
=
= 3.44  10–5 m 600
= 0.03 mm = 18
285

MHT-CET Triumph Physics (Hints)


 D  
65. For 5th dark fringe, x1 = (2n – 1)  sin  = or  =
2 d a a
9 D ….( sin    for small angles)
=
2d Let distance of first dark band from axis be y
th D 7 D x
For 7 bright fringe, x2 = n  = then angle of diffraction  is given by
d d f
D x  
 x2 – x1 = ( – 1) t  = or x = f
d f a a
7 D 9 D D
  = ( – 1)t n D
d d d 71. Position of nth minima, xn =
d
2.5 
 t= For n = 1,
(  1)

ns
1  5000  1010  1
5  103 =
66. For Young’s double slit experiment, the position d
of minima is;  d = 104 m = 0.1 mm
 1  D 2
y = n  

io
 2 d 72. Angular width,  = 
a
Adjacent minimum is the 1st minima or n = 0 1 1 1 6000
   
 1  D D 2  2  70  2

at
 y1 =  0   =  1 
 2 d 2d  100 
   2  4200Å
When immersed in liquid,  =
m


 D 
y1 = 
lic 73. In a single slit diffraction experiment, position
of minima is given by, d sin  = n

 2 md   
Now fringe shift due to introduction of sheet on So, for first minima of red, sin  = 1  R 
 d 
ub
the path of one of the beams is W. and as first maxima is midway between first and
D
W= (  1)t second minima, for wavelength , its position
d will be
The requirement is, minimum must appear on   2 3
the axis. d sin  =  sin  
P

2 2d
D  p  D  According to given condition sin  = sin 
 W = y1 or  1 t =
d  m  2 m d 2
  = R

et

3
 t=
2( p  m ) 2
    589 = 392.6 nm = 3926 Å
3
67. Band width  
rg

74. For the first minimum,


 yellow < red, hence for red light, the diffraction
ad1
bands become broader and further apart. a sin 1 =   a1 = ....(i)
D
68. Diffraction is obtained when the slit width is of For the sixth minimum,
Ta

the order of wavelength of EM waves (or light). ad 6


a sin 6 = 6  a6 = ....(ii)
Wavelength of X-rays (1-100 Å) is very less D
than slit width (0.6 mm). Therefore, no  By subtracting equation (i) from equation (ii),
diffraction pattern will be observed. a
(6  ) = (d6  d1)
69. Position of first minima = Position of third D
maxima 5D 5  0.5  5000  1010
 a= =

1  1D
=
 2  3  1  2 D (d 6  d1 ) 0.5  103
d 2 d  a = 25  10–4 m = 2.5 mm
 1 = 3.5 2 75. For minima, d sin  = n
70. Here for minima, Here n = 1,
a sin  = n  y1d 
d  = 1(5400 Å)
For first dark band, n = 1  D 

286

Chapter 07: Wave optics


D 82. i = 60, reflecting angle, r = 60
y1d = (5400 Å)
d Let r = angle of refraction M
Now, first maximum is approximately between   BOC = 90 B
the first minima and second minima.  r + r = 90 A
 y1d  y 2d   1  2  D  r = 90  60 = 30 r
y1b =  =  
 2   2  d From Snell’s law,
O
sin i
D 3D n= r
As y1d = y1b  (5400 Å) =    sin r
d  2 d
sin 60 C
  =
2 5400Å
= 3600 Å  n=
sin 30 N
3
 3
  
77. As limit of resolution, R = 1.22  2 

ns
d  n= n= 3
1
The angle subtended by the object at the human  
2
y
eye is  = 85. For 10th order fringes (for 1),
D

io
where, y is the separation between the marks 101D
= 2.37  1.25 = 1.12 mm
and D is the distance of the marks from the eye. d
Now for clarity of vision,  > R 
1.12mm
=
10 D
….(i)

at
y 1.22 yd 1 d
 > D<
D d 1.22 For  2, 10th order fringes,
yd (1103 m)(1.8103 m) 10 2D  1.12mm 
 Dgreatest = = = 2  ….[From (i)]
1.22
lic
1.22  5550 1010 m d  1 
= 2.66  2.7 m 7000
=  1.12  10–3 = 1.30 mm
78. Limit of resolution (d)   6000
  Reading for 10th order would be
ub
d1
 = 1 1.25 mm + 1.30 mm = 2.55 mm.
d2 2
 The zero order reading would be same for both
0.1 6000 wavelengths.
   d2 = 0.08 mm
d 2 4800
86. I = I0 cos2 
P

1.22  d IA = (I0 A) cos2 , where A is the area of the


79. R.P. = = polariser.
D x
Dd 103  0.1 P = P0 cos2 , where P represents power.
et

 x= = = 163.9 m P0
1.22 1.22  5  107  PAverage = P0 (cos2 )Average =
2
80. d = angle of the cone of light from the objects 1
….[ average of cos2  over a cycle is ]
rg

diameter of the telescope 2


d =
distance of themoon 2 2  
T= = =2s
5  
=
4105 103  Energy passing through per revolution
Ta

1.22 1.22  1.22 500010 10 = Paverage  2 s


 d = D= = 1 1
D d 5 / (4108 ) = P0    2 = (102 W)     2 = 102 J
 D = 48.8 m  50 m 2 2
energy E
sin i 87. Intensity = =
81. Using, n = time  area t  2 dl
sin r
1
sin 50  Intensity 
sin r = 50 d
 40
40 But intensity  Amplitude2
0.76 i
= = 0.57 1
1.33  Amplitude2 
d
 r = sin1 (0.57) r 1 1
or Amplitude =  1/ 2
 r = 35 d d

287

MHT-CET Triumph Physics (Hints)


1
88. I  I = Kr–2
r2 Competitive Thinking
–3
 dI = K(–2)r dr
2. ν = 9 GHz = 9  109 Hz
dI (2)dr
 = Velocity of radiation in air,
I r
c = 3  108 m/s
dI
 =–21%=–2% c 3  108 101
I  =   m
 9  109 3
 Intensity must decrease by 2%
Wave number for the wavelength,
89. For spherical wavefront, radius = r 1

2 1 
Also, I  a but I 
r2  Here, number of waves in 1 m,

ns
1 length 1
 a   1 = 30
r  10
90. In polar regions, magnetic compass becomes 3
inoperative hence sunlight which is easily 3. On the wavefront, all the points are in same phase.

io
available and scattered by earth’s atmosphere
gives plane polarised light when scattered sin i c
6. n= and n =
through 90. This is used for navigation sin r v

at
purpose.  For same i, as r increases, value of n decreases.
92. For diffraction at circular aperture, 1
But n  , hence as value of n decreases v
1.22  v
=
d
1.22 (6107 m)
lic increases.
This means as sin r increases v increases.
= = 3.66  104 rad Therefore, speed of light is minimum where
(2 103 m)
angle of refraction is minimum.
If r is the radius of the image formed by the lens
ub
r sin i i
at its focus, then  =   7. n= = ( i << , sin i  i)
f  sin r r
 r = f  = (6  102 m) (3.66  104 rad) n=
c
=
c
=
i
= 21.96  106 m v 0.75c r
P

A = r2 = (3.14) (21.96  106 m)2 3


 r = 0.75 i = i
= 15.14  1010 m2 4
P 3 i
I = =ir=i
et

i=
S 4 4
8103 W
= ca c
15.141010 m 2 8. n= = a
cg 0.8ca
rg

kW
 5.2 sin i 1
m2  =
sin r 0.8
93. When a beam of light is used to determine the for small angle i, sin i  i and sin r  r
Ta

position of an object, the maximum accuracy is


i 1
achieved if the light is of shorter wavelength,  =
because r 0.8
1  r = 0.8 i
Accuracy  Angle of deviation,
Wavelength
 = i – r = i – 0.8 i = 0.2 i
94. Referring to diagram, while reaching to point P, i
wavefront AB has crossed distance equivalent to =
5
three wavefronts.
As, the gap between successive wavefront is , sin i
9. µ=
distance covered by wavefront AB in reaching sin r
point P is 3. Given: i = 2r
(3) i
Hence the required time, t =  r=
c 2

288

Chapter 07: Wave optics


sin i I 0  I0 
µ= 16. Using law of Malus,    cos2   = 60
i 8 2
sin  
2
17. Using law of Malus,
i i
2sin   cos   Intensity of light transmitted from 1st polaroid,
µ=  
2 2
I0
 
i
sin   I1 = I0 cos2  =
2
2
 Intensity of emergent light, I2 = I1 cos2 45
i
 µ = 2 cos I1 I
2 = = 0
2 4

 i = 2 cos–1  
2 18. Using law of Malus, I2 = I1 cos2 
 I2 = I1 cos2 60

ns
10. S 2
1
 I2 = I1   = I1 = 0.25 I1
d 2 4

5 cm Hence, intensity becomes 25% of initial i.e., it


decreases by 75%.

io
thickness of slab (t) = 5 cm 19. By law of Malus, I = I0 cos2
n = 1.6 Now, IA = IAcos230

at
Now, ng =
c d/T d
  and IB = IBcos260
v t/T t As IA = IB
 d = n t = 1.6  5 = 8 cm  IA  3 = IB  1
4 4
11.
c
= 
c
v t/T

cT
t
lic IA 1
=
IB 3
t
 T=
c 20. Intensity of unpolarised light after passing
ub
through polariser A,
12. Speed of light in medium of ref. index (4/3) is I
IA =
c 3  108 9  108 2
v=  
n 4/3 4 Given that the intensity of light after passing
P

d 4.5  4 through polariser B is,


t=  = 2  10–8 = 20 ns
v 9  108 I
IB =
2
13. Given: Ng = Nw  Polarisers A and B are parallel to each other.
et

d Let polariser C makes angle  with the polariser A.


But number of waves N = ;
 Hence, it will make same angle with polariser B.
where d = thickness of the medium I
 Using law of Malus, IC = cos2 
rg

dg d 2
 = w
g w Now, when the light passes through C and then B.
 air  I  I
But g = and w = air IB =  cos2  cos2  =
2  8
Ta

g w
1
 ngdg = nwdw  (cos2)2 =
4
n gdg 1.5  6 9
 nw = = = = 1.286  cos2 =
1
dw 7 7 2
1
14. When polaroid is placed making 45 to y-axis,  cos  =
intensity of emergent light would lie between 2
2   = 45
Imax i.e., I0 and Imin. i.e., I0. Amongst given
3 21. Using Malus’ law,
options, only option (C) satisfies this condition. I = I0 cos2,
15. Using law of Malus, where  is angle between polarizer and analyser.
I0
I = I0 cos2  = I0 cos2 60 = I 0 = I0 cos2
4 10

289

MHT-CET Triumph Physics (Hints)


1 33.
 cos  = = 0.3162
10
   71.6
For intensity to be zero, polarizer and analyser
should be at angle 90. S1 S2
O
 Angle through which analyser should be rotated d
further,  = 90   = 90  71.6 = 18.4
23. Given: Reflected ray and refracted ray are
perpendicular to each other. This implies that
the angle of incidence equals polarising angle Screen
(θB). For i = θB, reflected light is completely
plane polarised i.e., its electric vector is Amongst the options only for a circle with

ns
perpendicular to the plane of incidence. centre as O, path difference will be constant,
giving steady interference.
24. Here i = 57
Given: 34. A2 = a12  a 22  2a1a 2 cos  , where A is amplitude of

io
tan 57 = 1.54 = refractive index of glass slab resultant wave and given that, a1 = a2 = a, where,
This means according to Brewster’s law, light is a is amplitude of individual wave.
incident at polarizing angle.   
Therefore, reflected light will be plane  A2 = 2a2(1 + cos ) = 2a2 1  2cos 2  1

at
 2 
polarized. Hence, upon analysing, its intensity
will gradually reduce to zero and then increase 
 A2  cos2
as the polaroid is rotated. 2
Now, I  A2
25. θB + r = 90
lic  
r = 90 – 57 = 33  I  A2  cos2  I  cos2
2 2
27. For polarising angle,
I 
= cos2   ;
ub
tan  =  =
c 35.
I0 2
v
v 2
 cot  = phase difference =  path difference
c 
2 2  
P

v = x =  =
  = cot1     6 3
c
   3I
29. Given that,  I = I0 cos2   = 0
6 4
et

reflected ray is plane polarised.


Using Brewster’s law, 2
36. Phase difference,   (x)
ng 
  tan i
For path difference , phase difference 1 = 2
rg

nw
 n g = tan(51)  n w = 1.235  1.4 = 1.73 for path difference /4, phase difference
2 = /2.
30. According to Brewster’s law, 
Using, I = 4I0 cos 2
Ta

n = tan θB 2
sin i 1
But n =  (In this case). I1 cos 2 (1 / 2)
sin r slope  
I2 cos 2 (2 / 2)
1 1
 tan θB =   3 K cos 2 (2 / 2) 1 K
slope tan 30o     I2  .
I2   / 2  1 / 2 2
 θB = tan1  3  = 60 cos 2 
 2

31. Path difference for all wavelengths at the central 


37. Path difference, x = 
point will be zero. Hence, central band will be 2
white a few coloured bands are observed on 
Given: x =
either side of the central band. 3
32. The two sources of light emitting different 2
 phase difference,  =
wavelengths will not form interference fringes. 3

290

Chapter 07: Wave optics


2 Substituting equation (i) above, we get
Assuming second wave differs in phase by
 
2
3 Imax n 1
2 =
from first wave,  = .
 n  1
2
Imin
3
Intensity, I = I0 cos2 
2 Intensityof bright band 16
 2 
2  1  K 42. 
= K cos   = K   = Intensityof dark band 1
 3 2
  4
But I  a2
38. Let A1 = A0. Then A2 = 2A0  amplitude of bright band ab = 4 and
Intensity I  A2 amplitude of dark band ad = 1
Hence I1 = I0, I2 = 4I0  Intensity of individual sources,
We have I = I0 + 4I0 + 2 I0  4I0 cos  Imax = (ab + ad)2 = (4 + 1)2 = 25

ns
For I = Imax , cos  = 1 Imin = (ab  ad)2 = (4  1)2 = 9
Im
 Im = 9I0 or I0 = I max  a 2  a1 
2
9 43.  
For a phase difference of , I min  a 2  a1 

io
I = I0 + 4I0 + 2 4I02 cos  a 1
Given, 1 
a2 3
= I0 + 4I0(1 + cos )
  a2 3
   

at
= I0 1 8cos 2  ....  1  cos  2cos 2  a1 1
 2
  2
a 2  a1 3  1 a a 3 1
Im   = 4 and 2 1  =2
=
9
1  8cos  / 2 
2 lic a1 1 a1 1
2
39. Resultant intensity is given by, I max  4 
  
I min  2 
IR = I1 + I2 + 2 I1I 2 cos
I max
π  =4
At point P,  = I min
ub
2
 (IR)P = I + 9I + 0 ...{ cos90° = 0}
   
2 2
44. Imax  I1  I2 4I  9I  25I
(IR)P = 10I
   9I 
2 2
At point Q,  =  I min I1  I2 4I  I
P

 (IR)Q = I + 9I  2 I × 9I ...{ cos180° = 1}


= 10I – 6I 45. I max  ( I1  I 2 ) 2  ( I  4I) 2  9I
(IR)Q = 4I I min  ( I1  I 2 ) 2  ( I  4I) 2  I
et

 Difference between resultant intensities at point


P and Q is = 10I – 4I = 6I 46. a1 = I1 , a2 = I2
40. Since, superimposing waves have Intensity I0
 
2
Maximum intensity: Imax = (a1 + a2)2 = I1  I 2
rg

 I1 = I2 = I0
Minimum intensity: Imin = (a1  a2)2 =  
2
So Imax = 4 I0 I1  I 2
and Imin = 0
Imax + Imin =  I  I  +  I  I 
2 2

Ta

I I 1 2 1 2
Hence, Iaverage = max min
2 = I1 + I2 + 2 I1 I2 + I1 + I2 – 2 I1 I2
 Iaverage = 2I0
= 2(I1 + I2)
I1
41. =n  I1 
2
I2 I1
1  1
We know, I  a2 Imax I2 I
47. = =  2 

a1 Imin I1 I1
  n ….(i) 1   1 
a2 I2  I2 
2 2
 a1   1 
  1  1 2
I max (a1  a 2 ) 2  a 2   5 1 36 9
Now,   =  25  =   = =
I min (a1  a 2 ) 2  a 
2
 1  1 5  16 4

1
 1   1 
 a2   25 

291

MHT-CET Triumph Physics (Hints)


2
51. screen
 I   
2

 1 1  9 1 S1 P d
I max I
48.  2   1  4 yn =
2
I min  I   9  1 d = 5 O
 1  1   1  S2 x E
I
 2   1 
D = 10 d
49. I0 3
I0
4 8 Path difference between two interfering waves
I0 arriving at point P is,
d
d
yd  2  d
x= = =
D (10d) 20
3 3
I0 I0 5 
8  x= 

ns
4 20 4

 phase difference,  = = 90
Given that, 25% of total intensity of incident 2
light is reflected from upper surface. This 
I = I0 cos2 = I0 cos2 45

io
implies, if intensity of incident light is I0, the 2
intensity of light reaching the lower surface of I0
3  I =
plate will be I0. 2

at
4
As 50% of this intensity is reflected, the final 
52. From formula, I = Imax cos2  
intensity of light emerging from glass plate will 2
3  I 1
be
8
I0.
lic  cos2   =
2 I max
=
2
I 2
 cos  = 0,  cos  = 0
 I1 = 0
4  3 5
3  = , , ,…
ub
I2 = I0 2 2 2
8 Corresponding path difference,
 
2
Imax I1  I 2  3 4
Now,  Δx= , ,
4 4 4
 
2
I min I1  I 2 
P

 Δ x = (2n + 1)
1 3
2
4
  
D
= 2 8
53. =
1 3
et

d
  
 2 8   increases from violet to red
50. Given,  R > G > B  R > G > B
I1 54. Fringe width is independent of the order of fringe.
rg

=n
I2
Path difference 1.8  105 1.23  105
 I1 = n I2 55. =
 6000  1010
   
2 2
 Imax = I1  I2 = nI2  I2
Ta

(1.80  1.23) 105


=
Say I2 = I 60001010
 
2
 Imax = n 1 I 57
= = 9.5
6
 
2
Similarly, Imin = n 1 I  Path difference = 9.5 

 n  1   
2 2
n 1 As path difference is odd multiple of , point
I max  I min 2
 =
 n  1   n  1
2 2
I max  I min is dark.

n 1 2 n  n 1 2 n
56. Second minimum is exactly in front of one slit
= d
n 1 2 n  n 1 2 n indicates, y2 =
2
4 n 2 n (2n  1) D
= = But y2 =
2n  2 n 1 2d

292

Chapter 07: Wave optics


For n = 2 n D
62. Fringe width, W =
d (2  2  1)D d
 = For fourth bright fringe,
2 2d
d2 4D
 = (y4)1 =
3D d
4D
57. Distance between 1st order dark fringes and (y4)2 =
d
= width of principal maximum 4D
2D 2  600 10 9  2  (y4)1  (y4)2 = (  )
 Wc = = d
d 10 3
4  1.2   6500  5200   1010 
= 2400  106 =
2  103
= 2.4  103 m –4
= 3.12  10 m = 0.312 mm

ns
= 2.4 mm
D
58. In Young’s double slit experiment, for nth 63. W=
d
minimum,

 2n  1  for n = 1, 2, 3,...  W  and  =

io
Δyn = '
2 W 
 2  5   1  9  
 Δy5 =   W'  '

at
2 2  W 3
 W'  W  = 0.2  = 0.15
'  4
59. The dark band formed at point A is of the order
n = 5. D 6000 107  D
Path difference of nth dark band is given by, 64. Using, W = , W1 = = 2 mm
λ
lic d d
yn =  2n  1 
D
=
2 1
=  104
2 d 6000 10 7
3
 2  5  1 6 × 107 When the apparatus is dipped in water,
 y5 = 
ub
2 wavelength and hence fringe width decreases by
= 2.7  106 m a factor of  (R.I.).
–4
= 2.7 × 10 cm W1 2
 W2 = = = 1.5 mm
 1.33
W
P

60. yn = (2n − 1)  Change in fringe width = 2  1.5 = 0.5 mm


2
For 5th dark fringe, n = 5 65. For 5th dark fringe in air
9 9  2  5  1 D  9 D
y5 = W =  2  103 = 9  103 cm  y5 a 
et


2 2 2d 2 d
For 8th bright fringe in medium,
61. For nth bright band, 8D
D  y8 m  , where  is refractive index of
rg

yn = n d
d medium
For nth dark band,
D y   y 
'
5 a 8 m
yn = (2n –1)
Ta

2d 9 D 8D
 
D D 2 d d
 yn+1 = [2(n+1) –1) = (2n + 2–1)
2d 2d 8 2
 = ≈ 1.78
D 9
= (2n + 1)
2d 66. Using Shortcut 4,
Since, dark fringe is on the other side of bright n11 = n22
fringe. n11 4  5000
D nD  2 =  = 4000 Å
 yn+1 + yn = (2n + 1) + 2 5
2d d
D 67. Using Shortcut 4,
=  2n 1 2n  n11 = n22
2d
D n11 3  700
= (4n + 1)  2 =  = 420 nm
2d 2 5

293

MHT-CET Triumph Physics (Hints)


68. Using shortcut 4, 0.20 d
 
 n1 1 = n22 0.21 2
9  5896 Å = 11  2 2  0.2
 d = = 1.9 mm
9  5896 0.21
 2 =
11
75. Path difference for maxima, n = dsin
 2 = 4824 Å
Given: d = 2
69. Using Shortcut 4,  n = 2 sin 
m1 = n2 Maximum value of sin  being 1,
m  2 600 3 n≤2
   
 Values taken by n = 0, ±1, ±2 i.e., 5.
n 1 400 2
 Maximum no. of interference maxima = 5
70. Using Shortcut 4,

ns
76. For any point in interference pattern,
n11 = n22

 nRed = (n + 1) Green I = Imax cos2
2
n  1  Re d 6000 6
    I max 
n  Green 5000 5   Imax cos 2

io
4 2
 6 n = 5n + 5  1
 n=5  cos2 =
2 4

at
71. Given: Wn = Wn+1  1
 cos =
Using Shortcut 4, 2 2
n1D (n  1) 2 D  
   = 60 =
2 3

d d
n  780  10–9 = (n + 1)  520  10–9
lic  =
2
 780n  520n = 520 3
 260n = 520 We know that,
 2 
ub
520  =   x , where ∆x is path difference.
 n= =2
260   
and x = dsin 
72. Using Shortcut 4,
2 2 
n11 = n22    dsin 

P

3
n1  520 4
 = 2 = = 
n2 1 650 5   sin 
3d
 n11 = n22 = 520  5 = 650  4 = 2600 nm
 
et

n11D 2600 109 1.5   = sin 1  


 y1 = =  3d 
d 0.5103
= 7.8  103 m = 7.8 mm. 77. Using Shortcut 6,
rg

N 7 7  600  109
73. Using shortcut 4, t= = = = 7 m
n  1 (n  1) (1.6  1)
n1  10000 Å 5
= 2 = = 78. λ  d, size of the obstacle.
n2 1 12000 Å 6
Ta

n11 = n22 = 5  12000 = 6  10000 = 60000 Å 79. Angular spread of central maxima is  = 2/a
n  D 60000  1010  2
Therefore, x = 1 1 = 80. blue < yellow
d 2  103 Hence diffraction bands become narrower.
= 6  103 m
= 6 mm 84. In diffraction of light by single slit, the width of
central maximum is given as -
 2D
74. Angular width of fringe:  = width of central maxima, Wc =
d d
For  = constant, Given: Wc = d
1 2D
  d=
d d
 d d2
 = D=
 d 2

294

Chapter 07: Wave optics


85. Given :  = 600 nm = 600  109 m  Dividing equations (i) by equation (ii),
2 2  600 109 1/ 2 2 3
Total angular width, 2 =  =  n = sin1  
a 0.2 10 3 sin n 3 4
= 6  103 rad 91. Distance of nth minima from the centre of the
86. Angular width of central maxima is, n D
screen is, ynd =
 a
 here, n = 1
a
Given that, D 5 105  60
 y1d = = = 0.15 cm
 = (1  0.3) = 0.7  a 0.02
  92. Distance of 1st minima from central maxima
 
  D
y1d =

ns
 6000  0.7 a
  =  = 4200 Å
  Distance between two minima on either side of
87. In single slit diffraction, for small angle, the central maxima is
 2D 2  5000  1010  2
2y1d = = = 102 m

io
d = 2n is the condition for minimum. a 0.2  103
2
n 1  698  109 1.22f
 d= = 93. d =
   
c

at
D
 2  
 180  1.22  5000  1010  5
= = 1.22  10–3 m
 d = 2  105 m 2.5  103
 d = 0.02 mm
88.
lic
For first minima in diffraction pattern,
94.
a sin  = 1  Red 0.25 cm 
For first maxima in diffraction pattern,
ub
3
a sin  =  Pupil
2
3 1.22
As both coincide, Red =  1.22  (500  109 m)
2 R.P. = =
2nsin   1 
2  1 
P

2 2 
  = Red  = 6600  = 4400 Å  100 
3 3 = 3.05  10–5 m  30 m
89. For nth minima in single slit diffraction, 
95. N. A =
et

a sin  = n 2a
For n = 2 and  = 60 For  = constant
a sin 60 = 2 1
 N.A 
a
rg

 3/2 3
  
a 2 4 96. Angular magnification  focal length of
For 1st minima, n = 1 and  = 1 objective lens.
 a sin 1 =  Angular resolution  aperture (diameter) of
Ta

 3 objective lens.
 sin 1 =   0.43
a 4 97. Resolving power of a microscope is,
 1 = sin1(0.43)  25 1
R.P. 
90. th
For n secondary minimum, 
R.P.1  6000 3
path difference = a sin n = n and  = 2 = =
for nth secondary maximum, R.P.2 1 4000 2

path difference = a sin n = (2n + 1) D
2 98. R. P. of telescope =
st 1.22
 For 1 minimum, a sin 30 =  ….(i)
 1
For 2nd maximum, a sin n = (2 + 1)  R. P. 
2 
….(ii) As  decreases, R. P. increases.

295

MHT-CET Triumph Physics (Hints)


1.22  106. Let geometrical spread be a and spread due to
99. Limit of resolution  =
D diffraction be c such that size of spot b = a + c
1.22  6000  1010
=
0.1
 L sin 
= 7.32  10–6 rad a
D 2
100. R.P. = =
1.22 1.22  0.5  106
4 6
= 106 = 3.28  10
1.22 From the figure,
101. Resolving power of telescope, c = L sin 
D 1 For  < c , sin   
R= R

ns
1.22   L
 c = L =
R1  2 5500 5 a
    L
R 2 1 4400 4  b=a+ ….(i)
a

io
102. Consider a plane wavefront travelling horizontally. For minimum value of b,
When it moves, its different parts move with
a 2  L
1  0=
different speeds (as n  ). Ray 1 will travel a

at
v
faster than Ray 2. So, its shape will change as  a2 = L ….[considering magnitude]
shown and beam will bend upward.  a = L
Substituting value of a in equation (i)
Higher R.I.
lic bmin = L 
L
= 2 L  4L
Ray 2 L

D
107. W =
ub
d
Ray 1  % change in fringe width,
W D d
Small R.I.  100 =  100   100
W D d
P

103. n = tan θB = tan 54.74 = 2 = 0.5  ( 0.3)


sin 45 d
 2 = ….[Here, is negative
sin r d
et

1 as d is decreased]
 sin r =  r = 30
2 = 0.8 %
10D 2D
Positive value indicates increase in fringe width.
104. 
rg

d a
 Fringe width increases by 0.8 %.
2d 108. Angular width of central maximum,
a= = 0.2d = 0.2  1 mm = 0.2 mm
10 2
=
Ta

105. Given: 2 = 60 d


Considering condition for minima in diffraction, Change in wavelength
Path difference (∆x) = a sin = n c c 3  108 3  108
 = i  f =  = 
As a = 1 m,  = 30 and n = 1, i  f 4  1014 5  1014
a sin  1 = (0.75  0.6)  106 = 1.5  107 m
 = = 1  10–6 
n 2 Change in wavelength, causes change in angular
  = 0.5 m 2
If same setup is used for YDSE, width as,  =
d
D
Fringe width W = 2 2  1.5  10 7
d  d=  = 5  107 m
As, W = 1 cm and D = 50 cm,  0.6
D 0.5106  0.5
 d= = = 25 m
W 0.01

296

Chapter 07: Wave optics

Hints to Evaluation Test

v a  a  a x(Blue) 4360
1. an m =    =
v m  m  m x(Green) 5460
Also, anm = tan ip  x (Green) > x (Blue)

 tan ip = a 7. Distance of 5th bright fringe from central fringe,
m
5D
 1  W5B = ….(i)
 m = a   d
 tan i p  Distance of 3rd dark fringe from central fringe,
 a = m tan ip (23 1)D 5 D

ns
W3D = = ….(ii)
2d 2 d
2. Angle made with surface = 60
From equations (i) and (ii), required distance,
 i = 90  60 = 30
 5  D 5 5107 1
1.5 =
sin i W5B  W3D =  5   = 
2 1103

io
 2 d
sin r
= 1.25 mm.
sin i sin 30o
 sin r =   0.3333
1.5 1.5 8. Let I0 be the intensity of unpolarised light. The

at
 r = 19 28 I0
intensity transmitted by the first sheet is .
Ratio of the width 2
cos r cos19 28 0.9428  I0  I 0
=   = 1.088  1 : 1  Transmitted intensity =  I0 
cosi cos30 0.8661
lic 
= .
2 2
ng vm 4
This will be the intensity of incident light on the
3. mn g = = = second polaroid. The intensity transmitted by the
nm vg 3
I 
ub
v m  vg 43 1 second polaroid will be  0  cos2 
 = = ….(i) 2
vg 3 3
where  is the angle between their axes
Given that, vm  vg = 6.25  107 4
Substituting in equation (i), sin  =
P

5
 vg = 3  6.25  107 m/s 3
 vm = 6.25  107 + 3  6.25  107  cos  = ….( 1  sin2  = cos2 )
5
 = 4  6.25  107
et

2
 I0  2  I0   3  9
= 25  107 = 2.5  108 m/s    cos  =     = I0
2
  2
    5 50
I1 a2 100 Ratio of intensity of emergent light to that of
4. = 12 = , where a1 and a2 be amplitudes
rg

I2 a2 1 9
unpolarised light =
of two waves. 50
a1 10 a a 11 
 =  1 2= 9. Using, l = (2n  1)
a2 1 a1  a 2 9
Ta

2
2 2
I max a a   11  121 5000 108
 =  1 2 =   =  0.05 = (2n  1) 
I min a  a
 1 2  9 81 2
0.1
D 6000107 mm  2510mm  = (2n  1)
5. Using, W = = 5105
d 1mm
10000
= 15  102 mm = 0.15 mm  2n  1 =  n  1000
5
nD 10. Angular width of central maxima
6. Distance of nth bright fringe, xn =
d 2 2  589.3  109
 xn   = = rad
d 0.1  103
x n1 1 180
 = = 0.0117  = 0.68
x n2 2 

297

MHT-CET Triumph Physics (Hints)


D 3 2
11. Fringe width, W =  sin 2 =
d 2a
D But 1 = 2 or sin 1 = sin 2
 W ….(i)
d 1 3 2 2
 = 2  2 = 1 =  600
D a 2a 3 3
W  ….(ii)
d  2 = 400 nm
Dividing equation (ii) by equation (i), (2n  1)D
W D d 17. yn = where n = 1, 2, 3, ….
 2d
W d D 5 D 19 D
d y3 =  =
and y10
But D = 1.25 D; d = 2 d 2 d
2 Since the bands are on opposite sides of the
W (1.25D)(d) central bright band, the distance between these

ns
 
W (d / 2)D bands is y3 + y10

 W = 2.5 W 5 D 19 D
 y3 + y10
 = +
2 d 2 d
12. Using Shortcut 4,

io
n  900 6 12  5896  1010  0.60
= = = =
n  750 5 0.4  103
Hence the first position where overlapping  1.1  10–2 m

at
occurs is,  1.1 cm
nD 18. Interference effects are commonly observed in
y = y6 =
d thin films when their thickness is comparable to
=
6(1.5m)(750 109 m)
(2 103 m)
 3.4 mm
lic wavelength of incident light.
For excessively thin film, as compared to
wavelength of light, it appears dark and for a
diameter of the telescope film which is too thick, it results into uniform
14. d = illumination of the film. In thin film,
ub
distance of themoon
interference takes place between the waves
5 5
 d = = reflected from its two surfaces and waves
4  10  10
5 3
4  108
refracted through it.
1.22
 d = D
P

d 19. W=
1.22 1.22  6  107 d
 d= = 6000  1010  1
d  5  = = 104 m
 8  6  103
et

 4  10 
 Fringe width = 10–4 m
 d = 58.5 m  59 m
103
 No. of fringes formed per mm = = 10
15. For the first minimum on either side of the 104
rg

maximum,

20. Phase difference between any two points on a
a sin  =  or sin  = wavefront is zero. Also two points on a given
a
wavefront are equidistant from source hence
3
Ta

 sin  = = 0.6 independent of distance.


5
  = 36 52 23. Bending of light results in encroachment in
geometrical shadow by diffraction of waves.
Since central maximum spreads on both sides
Angular spread =  36 52
16. Position of first minima in diffraction pattern of
1 is given by, a sin  = n
 a sin 1 = 11
1
 sin 1 =
a
For the first maxima of wavelength 2,
3
a sin 2 = 2
2

298
8
displ
Electrostatics

Hints

270

Classical Thinking 33. Work done, W = q  pE sin  d


90

ns
4. As no charge is enclosed within the cylinder, =  pE cos 90
270
=0
 Total flux. = q = 0
C
50. C1 = (for series); C2 = 4C (for parallel)
5. Electric field is zero at any interior point as 4

io
there is no line of force. C1 1
 
C2 16
13. The potential at the centre is same as on surface

at
i.e., 80 volt. 1 1 1 1 1
51. = + + =  Cs = 2 F
19 Cs 3 9 18 2
= 9  109  1.6  10 10
q
17. V= 1 
4 0 r 0.53  10 Cp = 3 + 9 + 18 = 30 F
 V = 27.2 volt
lic 
Cs
Cp
=
2
30
=
1
15
V = W = 4 10  1.6 10 = 256 V
20 19
18.
q 0.25 52. We will arrange the capacitors such that three of
ub
them are in parallel and the fourth one is in
19. V= W series with the combination,
q 1 1 1 1 1 4 1
6 W  = + = + = =
8  10 = Ceff . (4  4  4) 4 12 4 12 3
40
P

 Ceff. = 3 F
W = 40  8  106 = 320  106 = 3.2  108 J
53. Let C be capacitance of each capacitor
pcos 
23. V connected in parallel.
et

r2
 Ceff. = 3C
If  = 0° then V = maximum
Now, 3C and C are in series.
If  = 180° then V = minimum
1 1 1 4
 = + =
rg

p Ceff . 3C C 3C
24. V = 9  109
r2 3C
 Ceff. = = 3.75
= 9  109  1.6  1019   1.28  1010 4
Ta

3.75 4
12  10 
10 2
 C= = 1.25  4 = 5 F
3
V = 0.128 V = 0.13 V
54. Potential of both spheres will be same.
29. U = pEcos 
It has minimum value when  = 0° Q 40 40
55. C= = = =2F
 U = pE  cos 0° = pE V 10  (10) 20
31. The work done is zero along any one of the path
56. Two capacitors of 1.5 F each are in parallel.
AB, AC, AD and AE.
 Ceff = 1.5 + 1.5 = 3 F

32. In the given condition angle between p Now, 3 F, 3 F and 3 F are in series,
 1 1 1 1 3
and E is zero. Hence potential energy  =   = =1
Ceff 3 3 3 3
U = pE cos = pE i.e., minimum. Also in
uniform electric field Fnet = 0  Ceff = 1 F

299

MHT-CET Triumph Physics (Hints)


Q 4. T.N.E.I. does not depend upon shape or the size
58. C=
V of Gaussian surface but depends only upon
W charge enclosed within the surface.
But, V = ….(W = work done)
Q 5. Electric intensity at a distance r from the centre
Q 2 (It) 2 of a charged spherical conductor of radius R,
 C=  q
W W E= .…(i)
[A T ] 2 2 4k0r 2
 [C] = = [M–1L–2T4A2] Since the charge is uniformly distributed on A,
[M1L2T 2 ]
the surface density of charge on A will be
A q
59. C= C =  q = 4R2
d 4 R 2
1 Substituting in eq. (i), we get

ns
 C  A and C 
d 4 R 2 R 2
E= =
4  k 0 r 2 k 0 r 2
V V 20 1
60. E=  d= = = m = 5 cm
d E 400 20 1 1 1
6. E  E1  and E2 

io
k k1 k2
Ak 0
61. C= E1 6
d  E2 = = = 2 N/C
k2 3

at
(5  104 )  5  8.85  1012
=
2  103  1
7. E= i.e. E 
= 1.10  1011 = 11  1012 F = 11 pF lic 2K0r 2
67. The type of current which flows from the E r 20 1
 = = =
positive plate of the capacitor to the negative E r 40 2
plate of the capacitor without passing through E 0.4
 E = = = 0.2 N/C
the battery is conduction current. 2 2
ub
1 q2 R 2
71. q = CV and U = CV2 = 8. E=
2 2C k 0 r 2
1 R2
72. U= CV2 Just outside the conductor, R  r  1
P

2 r2
1  4
=  10  106  (1000)2  E= =
2 k0 40 k
= 0.5  10  106  106 = 5 J 12  10 12  4  3.14  9  109
et

=
3.14
1 1 = 43.2  10–2 = 0.43 V/m
73. U= CV2 =  8  106  (100)2
2 2
rg

= 4  102 = 0.04 J Q
9. E=
40R 2
1 1
74. U=  QV =  6  106  500 Qmax
2 2  Emax =
40R 2
Ta

= 15  104 J
 Qmax = 40R2  Emax
1
Critical Thinking =  (10  10–2)2  2  106
9  109

q 2
1. Flux,   =  105 C
0 9
For, q = 1 C, numerically,  = 0–1 10. E1 +  (E2) = 0
   E1 = E 2
3. Flux =   E  d s Let x be the distance of the point from centre of
A where electric field is zero.
=   Eds cos  1 q1 1 q2
 =
 Flux is maximum when, cos  = 1 40 x 2 40 (80  x) 2
  = 0
300

Chapter 08: Electrostatics


(80  x) 2 15 q 10103
 = =3 16. = =
x 2
5 2rl 2 3.14 1103 103

80  x
= 3 = 1.59  103 C/m2
x
R 1 R  4
....[Retaining positive square root] 17. E= = 
k0r 40 kr
 80 – x = 3 x
9  109  2  10 6  5  10 3  4  3.14
 80 = 3 x + x  80 = (1 + 3 )x = = 90 V/m
6.28  2
80
 x=  29 cm
1 3 
18. E=   = 20rE
2k0 r
q
11. Charge density  = 1
 = 40   E =     4.5  104
r 2
A 
2 9  109
2

ns
 q = .A =  (4 R2)
1
 Distance of point from centre =  105 = 5 C/m
2
r = R + 0.2 = 0.1 + 0.2 = 0.3 m
1 q q

io
E= 19.   0  [ charge on dipole is zero.]
40 r 2 0
2
1 (4 R 2 )  R 20. Total flux = (14  78.85  56)nC / 0
= =
40 r2 0  r 

at
4
1.8  106  (0.1)2 2  107  8.85  109 C 
= = 40
(0.3)2 0 0
= 8.85  109  9  109  4
12. As 1 = 2,
lic = 1000 Nm2C1
Q1 Q2
 = 21. By Gauss’ law,
4r1 2
4r2 2
1
Q1 Q2 = (Qenclosed)
ub
 = 0
40 r12 4 0 r2 2
 Qenclosed = 0
E 1 = ( 8  103 + 4  103)0
 E1 = E2  1 = or E1 : E2 = 1 : 1
E2 1 =  4  103 0 C
P

1 ne  1
13. E= 22. E=  E
40 r 2 20 r r
Er 2 23. Relation for electric field is given by,

et

n= 40
e 
0.036  0.1  0.1 E=
 n= 20 r
9  109  1.6  1019 2  2 0 rE
  = 20rE =
rg

360 2
=  105
144 1  2  102  7.182  108
= 2.5  105 =
2  9  109
Ta

14. Given that, = 7.98  104 C/m


s = c 24. The cube has six surfaces and as the charge is at
 R2  R its centre. Hence, it will produce equal number
Now, Es = s 2 and Ec = c
r r of lines of forces through each surface.
R R R Q
 Es =  = Ec The charge of Q will produce in all lines of
r r r 0
force.
Q
15. Flux linked with the given sphere   ;  Q 
0  Each surface will allow  .
where Q = Charge enclosed by the sphere.  60 
Hence Q = 0 = (E  Area)0 25. Electric field near the surface of the conductor is
A 
 Q =   (4 02) 0 = 40A0. given by, and it is perpendicular to surface.
 0  0

301

MHT-CET Triumph Physics (Hints)

R R4 36. If charge acquired by the smaller sphere is Q,


26. E= =
k0 r 40  k  r kQ
then it’s potential, V =
6 3
0.25 10  4 10  4  3.14  9 10 9 r
= kQ
6.28  2  120   kQ = 240 ..... (i)
= 9 V/m 2
Whole charge resides on the outer sphere,
  Potential of the outer sphere,
27. E=
20 r kQ
V 
40rE 6
  = 2 0r E =
2 240
1
 V = ….[From (i)]
=  4  10–2  9  104 = 2  10–7 C m–1 6
2  9  109  V = 40 V

ns
28. Total flux  E
37. Electric potential inside a conductor is constant
 
= E . A = E A cos  and it is equal to that on the surface of
(where  is an angle L conductor.

io
between E and A) R E kq 1
For top and bottom 38. V i.e. V 
R R
faces of the cylinder,
 Potential on smaller sphere will be more.
 = 90

at
  = EA cos 90 ....( cos 90 = 0) 39. Potential energy U = 1 q1q 2
 =0 4  0 r
where, q1 = 50 e = 50  1.6  1019 C,
29.
lic
The electric field is due to all charges present
whether inside or outside the given surface. q2 = 1.6  1019 C, r = 1012 m
9 109  50 1.6 1019 1.6 1019
q U=
30. Total = A + B + C = ; 1012
0
ub
= 11.5  1015 J
 B =  and A = C =  [assumed]
q 1 q  40. Change in potential energy (U) = Uf – Ui
 2 +  =   =     . q3
0 2  0 
P

31. According to Gauss’s law, total flux coming out


of a closed surface enclosing charge q is given
  q 40 cm
by,  =  E.d S   50 cm
et

0 q2
From this expression, it is clear that total flux q1
D
linked with a closed surface only depends on the 30 cm
enclosed charge and independent of the shape 10 cm
rg

40 cm
and size of the surface.
  q 1  q1q 3 q 2q 3   q1q 3 q 2q 3  
=  E.d S   0
= 20 Vm ….[Given]  U  
40  0.4
 
0.1   0.4
 
0.5  
Ta

q 1 q
Thus, is constant as long as the enclosed  U  [8q 2q3 ]  3 (8q 2 )
0 40 40
charge is constant  k  8q 2
 The flux over a concentric sphere of radius
20 cm = 20 Vm. 41. A free positive charge moves from higher
33. Inside the hollow sphere, at any point the (positive) potential to lower (negative)
potential is constant. potential. Hence, it must cross S at some time.
A
34. Electric potential is a characteristic of electric 42. Since VA = VB = VC = VD
field. It does not matter, whether a charge is  WAB = q (VB  VA) = 0 r
Q r
placed in the electric field or not. B r D
Similarly WAC = WAD = WAA = 0 r
35. At any point inside the sphere, the potential is
same and is equal to that at the surface. C

302

Chapter 08: Electrostatics


43. Potential energy of the system 50. The electric potential V(x, y,z)  4x 2 volt
= (e)(e) =
2
e   V ˆ V ˆ V 
40 r 40 r Now E    ˆi j k 
 x y z 
As r decreases, potential energy increases.
Now V  8x, V  0 and V  0
dV x y z
44. E= = (10x + 10)
dx 
At x = 1 Hence E   8 xiˆ ,
E = (10  1 + 10) = 20 V/m so at point (1 m, 0, 2 m)

45. Spheres have same potential, E   8iˆ volt / metre or
K Q1 = K Q2 8 along negative X-axis.
R1 R2

ns
1 1
Q1 = R1 51. For a charge V  and for a dipole V 
r r2
Q2 R2
52. 12  10–9 C
1 q 12  10–9 C
46. V=

io
40 r 6 cm A 4 cm
q1 q2
V = 1 q'
40 r ' 1  q1 q2 
V = V1 + V2 = 

at
 2 
1 nq 40  6  10 4  102 
= ( q = nq, r = n1/3 r )
40 n1/3r
= 9  109  12  102  12  102 
9 9

2/3
V = Vn  6  10 4  10 

47. E=
V
= (V2  V1 ) =
(VQ  VP )
lic 9 7
= 9  10  10 (2  3) =  900 V.
x (r2  r1 ) (rB  rA ) 53. Potential at A = Potential due to (+ q) charge
= 
5  10  + Potential due to (– q) charge
= 5 V/m
ub
65 1 q 1 (q)
  0
48. 40 a  b
2 2 40 a 2  b 2
A B
5
54. Electric potential due to each charge at the
Potential in volts

4
P

centre of the square is 1 2q .


3 40 l
2 1 2q = 1 2q
 total potential = 4 
et

1 40 l 0 l
C
O 55. Diagonal of the cube,
1 2 3 4 5 6
 
2 2
a12 = 2a + a
rg

Distance R in metre
From the graph, it is observed that slope of BC = 2a2 + a2 = 3a2
is same, indicating uniform electric field a12 = 3a2  a1 = 3a
between B and C.
Ta

 Potential at the centre of the cube is


Hence, electric field at R = 5 m is,
E = slope of line BC V=8 1 Q = 8 2Q
40 3 40 3 a
dV  05 a
= =   = 2.5 V/m 2
dr 64
 V= 4Q
dV dV 30a
49. Ex =  = (5) = 5, Ey =  = 3
dx dy
56. Potential will be zero at two points
dV
and Ez =  =  15
dz q1 = 2 C M q2 = –1 C N
Enet = E 2x  E 2y  E 2z O
x=0 x=4 x=6 x = 12
= (5)  (3)  (  15)
2 2 2
l l'
6
= 7 units.
303

MHT-CET Triumph Physics (Hints)


At internal point (M), 65. 1 q1q 2
U ;
1  2  106 (1  106 )  40 r
  0
4 0  (6  l ) l  2
 net potential energy U net  3  1 q
 l=2m 40 l
So distance of M from origin; x = 6 – 2 = 4 m
At exterior point (N), 66. Electrostatic potential energy between two
1  2  106 (1  106 )  protons at a distance r in 92U235 nucleus is given
  0 by
4 0  (6  l' ) l' 
 l = 6 m U= 1 q1q 2
So distance of N from origin, x = 6 + 6 = 12 m 40 r
Substituting the values,
57. From charge configuration, at the centre electric 19 19
U = 9  10  1.6  10 15 1.6  10

ns
9
field is non-zero. Potential at the centre due to 
9  10
2q charge V2q  2q 14
r  U = 2.56  10 J
2q
67. Potential energy of the system when charge Q is

io
r at O is
r r qQ qQ 2qQ
U0 =  
E– q E2q E– q a a a

at
–q –q
When charge Q is shifted to position O, the
and potential due to – q charge potential energy will be
q
V q  
r
(r = distance of centre point)
lic q Q
q
 total potential V  V2q  V q  V q  0 O O
x

Potential at the centre is 5   1 q 


ub
58. (a + x) (a  x)
 40 l 
The electric field due to the oppositely placed qQ qQ qQ(2a)
U=  
charges cancel and net electric field is (a  x) (a  x) (a 2  x 2 )
 1 q
P

 2 
. 2qQ  x 2 
1

 40 l  =   1 
a  a 
2

60. Conducting surface behaves as equipotential


2qQ  x 2 
et

surface.    1 2  ( x < < a)


a  a 
61. Potential of points lying perpendicular to
electric field is same.  U = U  U0 =
2qQ  x 2   2qQ
 1 2 
rg

a  a  a
62. For the isolated charged conductor the potential
is same at every point. 2qQ 2
= (x )
Y a3
63.
Ta

68. +10 V 4 V
Equipotential  P Q
surfaces E
X Work done in moving 100e from P to Q, (Work
done in moving 100 negative charges from the
Z positive to the negative potential).
64. Potential inside the sphere will be same as that W = (100e)  (VQ  VP)
q = (100  1.6  1019)(14 V)
on its surface i.e., V  Vsurface  stat volt,
10 W = 2.24  1016 J
q
Vout  stat volt 69. V = W  4  10  1.6  10
20 19

15
q0 0.25
Vout 2 2
   Vout  V
V 3 3 = 256 V ( 1eV = 1.6  1019 J)

304

Chapter 08: Electrostatics


70. P.E of charge Q1 Usystem =
= 9  10  Q1Q 2 + 9  109  Q1Q3
9 1  (10  106 ) 2 (10  106 ) 2 (10  106 )2 
   
r12 r13 40  0.1 0.1 0.1 
12
 6 6 6 6

= 9  109  8  10 ×(4)  10  8  10 ×5  10  = 3  9  109  100  10 = 27 J
 4 5  0.1
=0J 74. In case of metallic sphere either solid or hollow,
the charge will reside on the surface of the
71. Work done = final P.E.  initial P.E.
sphere. Since both spheres have same surface
W = Uf  Ui area, they can hold equal amount of maximum
Ui = 1 [(q) (2q) + q(2q) + (2q) (2q)] charge.
4  0 r
75. For providing path to charge induced on the

ns
=0 surface of the carriers which take inflammable
Uf = 1 [(q)(2q) +q(2q) +(2q)(2q)] material.
4  0 (2r)
76. The conductor may be at positive, zero or
=0 negative potential, it is according to the way one

io
 W = 0. defines the zero potential.
72. The total potential energy of the arrangement of 80. E without dielectric 2  105
k  2

at
charges is the sum of the energies of each pair E with dielectric 1  105
of charges. The potential energy of the system
comprising the three charges q1, q2 and q3 is 81. Q = VC  V = Q / C
U = W1 + W2 + W3 As V is constant,

= 1  q1q 2 q1q 3 q 2 q 3 
   
lic 
Qg
=
Qo
Co
Q
Qo
C
 g = g where Cg is the new
Co
4  0  r12 r13 r23  Cg
capacitance and Qg is new charge.
q3 =  q  Cg > Co  Qg > Qo
ub
Electron
82. Q = CV = 6  106  18 = 108 C
r13 r23 83. Potential difference in the circuit = 24 – 12 = 12
Proton volt. This potential difference is divided among
P

Proton
two capacitors C1 and C2 in the inverse ratio of
their capacities (as they are joined in series)
q1 = + q r12 q2 = + q
 V1 = C2 V = 4  12 = 8 volt
et

C1  C 2 24
Let q1 , q3 be charges on two protons and q2 be
charge on electron. As plate of capacitor C1 towards point B will be
at positive potential, hence
 1.6  1019  1.6  1019
rg

 U = 9  109   VB – VA = 8 volt
 1010  VA – VB = –8 V
1.6  1019  1.6  1019 1.6  1019  1.6  1019  84. On connecting O at A, 4 F capacitor is charged
+ 
1.5  1010 1010  to a constant potential (E).
Ta

9  109  As connection of O is switched over to B, the


3  1.6  1019  + 1.6  1019  
2 2
= 10  total charge on 4 F capacitor that will be
1.5  10  
shared between 4 F and 2 F capacitors is
2  9  109  1.6  1019 
2
4 2
= J = of original charge.
1.5  1010 42 3
18  109  1.6  1019 
2

= eV 85. The effective capacitance is C1 when three


1.5  1010  1.6  1019 capacitors are connected in series
= 19.2 eV. 
1 1
= +
1 1
+ =
37
C1 4 5 6 60
73. For pair of charge
 C1 = 60 / 37 F ....(i)
U= 1  q1q 2 q1q 3 q 2q 3  When three capacitors are connected in parallel
   
40  r12 r13 r23  mode, the effective capacitance is C2

305

MHT-CET Triumph Physics (Hints)

 C2 = 4 + 5 + 6 = 15 F ....(ii) 91. C1 = 2 F, V = 100 V


From (i) and (ii),  Q1 = C1V = 2  100 = 200 C
C2 15 37 If we connect this condenser to uncharged
= = condenser, then total charge, Q = Q1
C1 60 / 37 4
Q = 200 C (due to parallel combination)
86. The capacitors of capacitance 2 F and 6 F are Total capacitance C = C1 + C2
connected in series. Hence, their effective = 2 F + 3 F = 5 F
6 2 Totalcharge
capacitance, Cs = = 1.5 F.  Common potential =
6 2 Totalcapacitance
These two branches are connected in parallel. 200106
= = 40 V
 Equivalent capacitance (C) = 1.5 + 1.5 = 3 F 5106
Now 4 F, 4 F and C are connected in series.
92.

ns
 Relation for the capacitance between P and Q,
A F
1 1 1 1 5 6
= + + = or C = F X E B Y
C 4 4 3 6 5 C C C
87. Given six capacitors are in parallel

io
Figure (a)
 Ceq = 6C = 6  2 F = 12 F Join B and E together. Similarly, join A and F.
Then the given circuit becomes as shown in
88. Capacitance of first capacitor (C1)
figure (b)

at
= 30 F = 30  106 F and its voltage (V1) C
= 500 V X Y
Capacitance of the second capacitor (C2) C
= 15 F = 15  106 F and its voltage (V2)
= 300 V
lic C
C1V1  C2V2 Figure (b)
 Common potential (V) =
C1  C2  Ceq = C + C + C = 3C = 3  2 = 6 F
ub
6 6
(3010  500)  (1510  300) 93. Given circuit can be drawn as,
=
(30106 )  (15106 ) 8 F
 433 V
8 F
P

1 1 1
89. = + A B
Ceff C1 C 2  C3
8 F
= 1 + 1
et

2 2 1 8 F
= + = 5
1 1
2 3 6  Equivalent capacitance between A and B
= 6 F = Cp = 4  8 = 32 F
rg

 Ceff
5
Q
 Total charge, Q = 6  10–6 120 = 14410–6 C 94. V= But Q = Ceff V ....(i)
C
5
Cp = 3 + 6 + 3 = 12 F
Ta

 Potential difference across C1, 12  2 24 12


6
V1 = Q = 144  10 = 72 V  Cs = Ceff = = = F
12  2 14 7
C1 2 106
12
 Q=  70 = 120 C ....[From (i)]
90. Charge on capacitor, 7
Q = CV = 8  106  12 = 96 C  V=
120
= 60 V
Q 2
 V=
C 95. When two capacitors are connected in series
Q combination,
 C=
V 1 1 1 C C
= + = 1 2
96 CS C1 C2 C1C2
Total capacity, C = C1 + C2 = = 32 F
3 C1C2 15
 CS = = F
 C2 = 32 F  C1 = 32  8 = 24 F C1  C2 4

306

Chapter 08: Electrostatics


15 (C1 + C2) = 4 C1 C2 ….(i) 1 1 1 1 3 C
101.      Ceff =
When two capacitors are connected in parallel Ceff C C C C 3
combination,
 V = V1 + V2 + V3 = V + V + V = 3V
C1 + C2 = 16 ….(ii)
Substituting eq. (ii) in eq. (i), 102. Since charge remains same in series combination,
15  16 = 4 C1 C2  C1V1 = C2V2
 15  4 = C1 C2 V1 C 4
60 = C1 C2  = 2 = =4
V2 C1 1
 C1 (16 – C1) = 60 ….[From (ii)]
C12 – 16C1 + 60 = 0 C C 5C
103. CPR =  
 C12 –10C1 – 6C1 + 60 = 0 2 3 6
 (C1 – 10) (C1 – 6) = 0 C 5C C 2
CPQ = C +   1
 C1 = 10 F or C1 = 6 F

ns
4 4 C2 3
Hence, values of capacitors are 6 F and 10 F.
104.
96. P P
q1 q2 q3 2C 2C

io
2C 2C
2C 2C C
C1 C2 C3 
C
C C 2C Q
Q

at
V1 V2 V3
C + C = 2C 2C/ 2 = C
V
2C P 2C P
In series grouping of condensers, the charge on
each plate is same,  q1 = q2 = q3 = q
lic 
2C
  CPQ=3C
2C
q C
Let q = CV  V = C 2C
C Q Q
ub
q1 q2 q C+C=2C
 V1 : V2 : V3 = : : 3
C1 C 2 C 3
1 1 1 1 1 1 1
= : : 105.   
C1 C2 C3 Ceq 3 10 15
P

 Ceq = 2 F
23 6
97. Equivalent capacitance   F  Charge on each capacitor,
23 5
Q = Ceq  V = 2  100 = 200 C
6
et

 Total charge by Q  CV   1000  1200 C


5 106. Given circuit can be reduced as follows:
 Potential (V) across 2 F is 3F 6F
Q 1200 +1200 VB=0
V   600volt
rg

C 2 A VP B
 Potential on internal plates = 1000  600 = 400 V
1 1 1 1 In series combination, charge on each capacitor
98. = + +
Ta

Ceff . C1 C2 C3 remains same.


1 1 1 3 So using Q = CV,
=   = C1V1  C2 V2  3(1200  Vp )  6(VP  VB )
6 6 6 6
 Ceff. = 2 pF = 2  1012 F  1200  Vp  2Vp ….( VB  0)

99.
1
=
1
+
1
+
1  3Vp = 1200  Vp = 400 volt
CR C1 C2 C3
1 1 1 1 3 2 4 9
CR =  C11  C 21  C31  107. =   = =
1
 Ceff . 20 30 15 60 60

100. The given arrangement is effectively an 60 60


 Ceff. = F  Q =  90 = 600 e.s.u.
arrangement of (n  1) capacitors connected in 9 9
parallel. Q 600
 V2 = = = 20 e.s.u.
 CR = (n  1) C C2 30

307

MHT-CET Triumph Physics (Hints)

108. 12 F and 6 F are in series and again are in 117. The given arrangement is equivalent to the parallel
parallel with 4 F. combination of three identical capacitors. Hence
 Effective capacitance resultant of these three 0 A
equivalent capacitance  3C  3
capacitor will be d
12  6
  4  4  4  8 F 118. Total charge on capacitors connected in parallel
12  6 is,
This system is in series with 1 F capacitor. C0
8 1 8 Q0 = ….(i)
 Its equivalent capacitance   F ....(i) V0
8 1 9
Where C0 = effective capacitance of parallel
Now, equivalent of 8F, 2F and 2F combination.
4  8 32 8 = C + C = 2C
   F .....(ii)
4  8 12 3 ….( C1 = C2 = C)(ii)

ns
Combinations (i) and (ii) are in parallel and are
in series with C C1
32
8 8 32 C
   and Ceq  1  9

io
9 3 9  32 
  C C2
 9 
32
 C F V0

at
23
Let C2 be kept in a dielectric medium,
109. The given figure is equivalent to a balanced then, C2 = kC
Wheatstone’s bridge.  C0 = C + kC = (1+k)C
 Ceq = 6 F
lic Hence, total charge on the capacitors,
110. C p  4Cs C0 1  k  C
Q0   ….(iii)
CC V0 V0
 (C1  C 2 )  4 1 2 Dividing equation (iii) by equation (i)
ub
(C1  C 2 )
 (C1  C 2 )  0
2
 C1  C 2 Q0 1  k  C V0 1  k 
   ….from (ii)
Q0 V0 C0 2
C1V1  C2 V2 1  k  Q0
111. Common potential =  Q0 
Ceff .
P

2
20106  500 10106  200
= = 400 V Q
20  106 10 106 119. C = ….(i)
V
et

112. Since d decreases, so C increases. Q


V= ….(ii)
 battery is disconnected  Q is constant . 40r
1 From equation (i) and (ii)
 V 0A
rg

C 40r =
Since V decreases, so C will increase. d
A (20  103 ) 2
Ak0  d= = = 0.1 mm
113. C = 4r 4  1
Ta

d
0A  kA
114. By inserting the dielectric slab, capacitance (i.e. 120. C = = 8 pF and C = 0
d d
ability to hold the charge) increases. In the
But A = A , d = d/2
presence of battery more charge is supplied
0 k  A 2 50A
from battery.  C = =
d/2 d
115. Refer Shortcut 10  C = 10  8 pF = 80 pF
For a spherical capacitor,
0A
 ab  121. Without dielectric, C0 =
C  40 k    Ck d
 ba 
k A
With dielectric, C1 = 1 0 = 2k1 C0
116. The two capacitors thus formed are in parallel. d/2
0 A k 20 A
 C (k1  k 2 ) and C2 = = 2k2 C0
t2 d/2

308

Chapter 08: Electrostatics


As C1, C2 are in series, 126. While drawing the dielectric plate outside, the
CC capacitance decreases till the entire plate comes
Cs = 1 2
C1  C2 out and then becomes constant. So, V increases
2k1C0  2k 2C0 2k k C and then becomes constant.
 = 1 2 0
2C0 (k1  k 2 ) k1  k 2 12  9  104 
127. C = 40k 
ab  1
   6  
Cs 2k1k 2  b  a  9  10
9
 3  10
2

 =
C0 k1  k 2 = 24  1011 = 240 pF
122. Capacity of capacitor = C 128. Initial charge on the capacitor Q = 10  12
0 AV = 120 C
Q = CV = .…(i)
d Final charge on the capacitor Q
After inserting a slab, capacitance becomes C1
= (5  10)  12 = 600 C

ns
and charge remains same, Q = C1V1
 Charge supplied by the battery later = Q  Q
By increasing the distance, we get same
potential difference as in first case. = 480 C
Q = C 2V ….(ii) 0 A
A 129. Using, C 

io
C= 0 d
dt
Cd 3  5  103
1 d  3  2.4 3 d  0.6 3  A   1.695109 m 2
 = + = +  0 8.85  1012

at
C2 0A k0A 0A k0A
From equations (i) and (ii), Q Qd
130. V =  Vd
C = C2 C 0 kA
1 1

C
=
C2
lic 131. C
1 C d
 1  2
15 2

d d  0.6 3 d C 2 d1 C2 6
 = +  C2 = 45 F
0 A 0A k0A
ub
3 A A A
 d = d – 0.6 + 132. C =  C1 = 1 and C2 = 2
k d d1 d2
3
 k= =5 C2 A d A d 1 1 1
0.6  = 2  1 = 2 1 =  =
C1 d2 A1 A1 d 2 2 2 4
P

123. Capacity of plate in medium, C1 1


k A  C2 = = 12 = 3 F
Cm = 0 ….(i) 4 4
d
et

If medium is removed, 133. Cmedium = k Cair


A C medium 110
C= 0 ….(ii)  k= = = 2.2
d Cair 50
From equations (i) and (ii),
rg

Cm = kC 134. Aluminium being a metal, the field inside it will


C 16 F be zero. Hence it would not affect the field in
 C= m = = 2 F between the two plates. Hence capacity
k 8
q q
Ta

124. Potential difference across the condenser,   remains unchanged.


V Ed
 
V = V1 + V2 = E1t1 + E2t2 = t1  t2 0A  A  (5A)
k10 k 20 135. C = and C = 0  0
d 2d 2d
  t1 t 2  Q  t1 t 2 
V=    =    6 A 3 A
 0  k1 k 2  A 0  k1 k 2  = 0  0
2d d
125. If length of the foil is l, then 3 A  A 2 A
 C = C  C = 0  0 = 0
k0 (l  b) d d d
C ….[ A = l  b]
d Percentage change in capacitance,
2.5  8.85  10 12 (l  400  10 3 )  20 A 
 2  10 6 
0.15  10 3 C  d 
  100 %  200%
2  10 6  0.15  10 3 C  0A 
 l = = 33.9 m  
2.5  8.85  10 12  400  10 3  d 

309

MHT-CET Triumph Physics (Hints)

1 Q2 Ak0 A A A


136. U = and C = 0   k0   0  
C1    ,C 2 
2 C d 4   ,C   4 
2
141. 3
1 1 d d d
 U 
C k A/4 A/2 A/4
U1 k
 = 2 k
U2 k1 d
If k1 = 1 and k2 = 2 then, 1 2 3
U U
U2 = 1 = 1
k2 2
 C eq  C1  C 2  C3
137. Presence of proton will not affect field between the  k 1  A  4 1
plates (since proton charge is quite small = 
0
   10  25 F

ns
 2  d  2 
compared to the charges on the plate)
V 200 20000 144. Displacement current does not depend on the
 E= = 2
= = 10000 V/m moving charges but it changes with time which
d 2  10 2
causes variation in the field.

io
138. The two condensers filled with k and with air 145. Change in the field results in the formation of
are in parallel. magnetic fields. Hence displacement currents
0  3A  3 0 A lead to magnetic field between the plates of a

at
With air: C1 =   capacitor.
d 4  4d
0 K  A  0 Ak 1 1
146. U1 = CV12 , U2 = CV12
With medium: C2 =   2 2
d 4 4d
 Ceq = C1 + C2
lic 
U1 V2
= 12
U2 V2
30 A 0 Ak A 3 k
= 0     (k  3)
C
 Ceq =  V22 900
4d 4d 4d  4 4  4  U2 = U1 =  U1 = 9 U1
ub
2
V1 100
C
 Ceq = (k  3) 1 1
4 147. Increase in energy = C1V12 – C0V02
2 2
0 A 1
= C (V12 – V02) ....( C1 = C0 = C)
P

139. C = 2
d
1
  0 A    0 r A  =  10  10–6 (121 – 100)
C1C2   .  2

d/ 2  d/ 2 

et

C= 1
C1  C2  0 A 0 r A 

=  10  21  10–6 = 105  10–6 J = 105 J
  2
d/2 d/2 
20 Ar 2Cr 1 1
=  148. U1 + U2 = C1 V12 + C2 V22
rg

d(1   r ) (1  r ) 2 2
1
= [4  106  50  50 + 2  106  100  100]
140. In air, the potential difference between the plates, 2
 1 3
= [102 + 2  102] =  102 J
Ta

Vair = .d
0 2 2
In the presence of partially filled medium, 149. U = 1 CV2 = 1 Q
2

potential difference between the plates, 2 2 C


 t Here, Q in both cases is same
Vm = (d  t  )
0 k Q2
 U1 = and
Potential difference between the plates with 2C1
dielectric medium and increased distance is, 2
U2 = Q = Q
2

 t 2C2 2kC1
Vm = (d  d ')  t  
0  k Now, C  k  C2 = kC1
According to question, Vair = Vm which gives  Decrease in energy = U1  U2
= Q  Q = Q  1  1 
2 2 2
t 2
k= k= =5 2C1 2kC1 k 2C1  
t  d' 2  1.6

310

Chapter 08: Electrostatics


U1  U 2 Q2 (40 106 ) 2 16 1010
 Fractional decrease in energy = 157. U = = = = 8 105 J
U1 2C 2 10 106 2 105
Q2  1  = 8  105  107 = 800 erg
 1
= 2C1  k  = 1  1 1 1
 Q2  k 158. U = CV2 = 121012  (50)2 = 1.5  108 J
  2 2
 2Cl 
159. Total capacitance of given system,
1 1 1 1 1 1
150. U = 3 2
CV 2 =  4 106  (10 ) = 2 J   
2 2 Ceff 4  4  4  4
1 1 1 1
152. If C is the capacitance of each capacitor then,    
Ceq 4 8 4
1 1 1 1 3
=   =

ns
Ceff C C C C 8
 Ceq = F
5
C
 Ceff = = 2F  C = 6 F 1 1 8
3  U= CeqV2 =   106  225
2 2 5
Now, for parallel combination,
= 180  106 J

io
Ceff = 3C = 3  6 = 18 F
= 180  106  107erg = 1800 erg
1 1
 U= Ceff V2 = 18106  (200)2
160. Energy stored in fully charged capacitor,
2 2

at
6 4
= 9  10  4  10 = 36  10 2 1
U= CV2
= 0.36 J 2
But work done by battery W = QV or
153. In parallel combination, U = W = CV.V = CV2
Ceff = C1 + C2 + C3 + C4 + C5 + C6 = 6  1
lic Energy required to charge the capacitor,
= 6 F and V = 2 V 0 A 2 0 Ad 2
 U = CV2 = .V = 2  V
 Q = CV = 6  2 = 12 C d d
ub
Q 12  V
 Q1 = = = 2 C = 0 E 2 Ad ….  E 
6 6  d 
Q 2 C
In series, V1 = 1 = =2V 1 1
C1 1 F 161. U  CV 2   6  106 (100) 2  0.03J
2 2
P

 VT = 6V1 = 12 V
C 1 162. Work done in placing the charge = Energy
Ceff (in series) = = F stored in the condenser
6 6
Q2 (8  1018 ) 2
et

1  W   32  1032 J
Using E = CV2, 2C 2  100  106
2
1 1 1
 E=  106  12  12 = 12  106 J = 12 J 163. U  QV  Area of triangle OAB
2 6
rg

2
C 3C 164. Heat produced = Energy stored in capacitor
154. Ceff = C + =
2 2 1 1
= CV 2 =  4106  (400)2 = 2  106  16  104
Ta

1  3C  3CV 2 2 2
 Work done =   V 2  = 0.32 J
2 2  4
1 1
1  165. U = CV 2 =  700 1012  (50)2
 qV  1 2 2
155. The required ratio, 
2  = 350  1012  2500 = 8.75  107 J
qV 2
166. C1 = 4  106 F, V1 = 50 volt, C2 = 2  106 F,
2
1Q V2 = 100 volt
156. Energy, U = .
2 C  Total energy before connection
For a charged capacitor, charge Q is constant 1 1
= C1V12 + C2V22
and with the increase in separation, C will 2 2
1 1
decrease (C  ). = (4  10  50  50 + 2  106  100  100)
6
d 2
Hence overall U will increase. = 1.5  102 J
311

MHT-CET Triumph Physics (Hints)


Equivalent capacity in parallel combination, 172. As A and B are at the same potential.
CP = C1 + C2 = 4  106 + 2  106 = 6  106 F WAB = qo (VB  VA) = 0
Common potential in parallel combination of The amount of net work done will be zero for
equipotential surface.
C1V1  C2 V2
capacitors, V = 173. W = qV
C1  C2
Change in K.E = work
6 6
= 4  10  506  2  10 6  100 
1
mv2 = qV
4  10  2  10 2
4 104 2 2qV
= =  102 volt v=
6 106 3 m
 Total energy after connection 174. Since the electric field is directed from south to

ns
1 north hence rate of change of potential will be
= CpV2
2 along this direction, but it is zero along east and
west.
1 2 2
=  6  106    (102)2 Q1 Q2
2 3 3 175. When  ; current will flow in

io
R1 R2
4
=  102 connecting wire so that energy decreases in the
3 form of heat through the connecting wire.
= 1.33  102 J

at
176. Work done, W = Uf  Ui
1 1 (C) 1
.(3V0)2 = 3  CV02
2
1Q Ui = CV02 and Uf =
167. Using, U = , 2 2 3 2
2C

1.21U =
1 (Q  2) 2
lic 
1 2
Uf  Ui = CV 0 (3  1) = CV 0
2
2

2 C
 AV 2   A
1.21 (Q  2) 2 1.21 Q  2  W= 0 0 ….  C  0 
    d  d 
Q2
ub
1 1 Q
 1.1Q = Q + 2  Q = 20 C 177.

k2 = 6
k1 = 3
169. Suppose charge on inner sphere is + Q as A air A
shown.
P

Potential on inner sphere d


d/3 2d/3
Q Q –Q
V  0A
4 6 Cair = = 9.
4 cm d
et

 1 1
3  Q   +Q 6 cm d1 d2
4 6 1 1 1 
=  = k10A k 20 A
 Q = 36 e.s.u. Cmed C1 C 2
rg

1 1 0A 2 k1k 2 0 A
170. U = CV2 = V  Cmed =
2 2 d k1d 2  k 2 d1
At any instant, let the separation between plates 3  6  0A 180 A
= 
Ta

be x 2d d 4d
3  6
3 3
1 0A 2
 U= V C med 18 0 A d 18
2 x    
Cair 4d 0A 4
1  0 AV
2
dU 1 1 dx
 = 0 AV2(–1) 2 =– 2
(v)  Cmed =
18
9
dt 2 x dt 2 x
4
i.e., potential energy decreases as (1/x2). = 40.5 pF
171. As separation between plates is reduced, C 178. Initial energy of the system,
1 1 1
increases but charge on it remains same. Hence, Ui = CV12  CV22 = C  V12  V22 
1 q 2
2 2 2
from the relation U = , U decreases. Also,
0

2 C When the capacitors are joined, common


work done in charging the capacitor is stored as CV1  CV2 V V
potential, V = = 1 2
potential energy. 2C 2

312

Chapter 08: Electrostatics


 Final energy of the system, 1
2
186. Work done = qV
1 1  V V  2
Uf = (2C)V2 = 2C  1 2 
2 2  2  1
=  4  4  106
1 2
= C(V1 + V2)2 = 8  106 J
4
 Decrease in energy = Ui  Uf work 8  106
 Power = = = 80 MW
1 time 0.1
= C(V1  V2)2
4 187. Here, V is constant.
179. Using Shortcut 11 1
U= CV2
20 L 2
Capacitance of a cylindrical capacitor =
b C  kC  energy stored will become k times
l n 
q = CV  q will become k times

ns
a
Energy stored in the capacitor, kq
 Surface charge density,  = = k0
b A
Q2l n  
1 Q2 1 2
 a   Q k
U=  188. A charged cloud induces opposite charge on

io
2 C 2 20 L L
pointed conductors. At sharp points of the
where k is a constant.
conductor, surface density of charge is very high
If the charge and length are doubled,
and charge begins to leak from the pointed ends

at
Q2 4  Q2 
k    = 2 times the energy. by setting up oppositely charged electic wind.
L 2 L 
When this wind comes in contact with the
180. Heat produced = Energy of charged capacitor charged cloud, it neutralizes some of the charge
1
 CV 2
lic on it. Hence, the potential difference between
2 the cloud and the building is reduced. This in
1 turn reduces the chance of lightening striking
  (2  106 )  (100)2
2 the building (if the lightning strikes the building,
ub
 0.01 J then the charge is conducted to the earth and the
1 building remains safe).
CV 2 40  106  (3000) 2
181. Power = 2  = 90 kW 189. Volume = 1 litre = 1  103 m3
t 2  2  103
P

1 1
u= 0E2 =  8.85  1012  (103)2
182. The ±q charges appearing on the inner surfaces 2 2
of A, are bound charges. B is uncharged initially = 4.425  106
and as it is isolated, the charges on A will not be
et

affected on closing the switch S. No charge will  Energy stored in 103 m3 of air
flow into B. = 4.425  106  103 = 4.425  109 J
 qin
rg

183. E inside  r ….(r < R) 191. E = = 0  qin = 0


30 0
R 3 Now,
E outside  ….(r  R)
30 r 2 qIN for S1 = 3q  q + q =  3q
Ta

i.e. inside the uniformly charged sphere, field qIN for S2 = + q  q = 0


varies linearly (E  r) with distance and that qIN for S3 = 3q + q =  2q
1 qIN for S4 = 3q
outside it varies according to E 
r2 192. Assertion is true, Reason is true and Reason is a
184. Work done = Energy stored in condenser correct explanation for Assertion.
1 1  Q1  Q 2 
 mgh = CV 2 VA =   R2
2 40  R 2 
CV 2 10106  (6 103 ) 2
 h= = = 1800 m 1  Q1 Q 2  Q1 R1
2mg 210103 10 VB =    B
4 0  R1 R 2  Q2
185. Work done in compression A
1 1 1 
= Energy stored in condenser  VB  VA = Q1   
 Ratio of energies = 1 4 0  R 1 R 2 

313

MHT-CET Triumph Physics (Hints)


193. The given circuit can be redrawn and reduced to 194. Let the plates be numbered as shown below.
the following: Plates 2, 3, 4 and 5 may be treated as a collection
E of two plates as shown in the diagram.
We get five capacitors with top and bottom
C capacitors having a capacitance C/2 and the rest
C with capacitance C.
C C 2C 1
C 2L
2 A
L
C C L 3
C
L 4
B
E
Now, the potential across 2L 5

ns
the two capacitors in C 6
parallel in E. Hence the Hence the circuit gets reduced as shown in the
charge stored in each is (19/11)C figure below. C/2
C C/2
q1, q2 = CE .…(i)

io
The other two capacitors
are in series. Hence the
charge in each of them is C A B

at
q3 = (19/30) CE .…(ii)
C
Therefore the potential across the (19/11) C
capacitor is
V1 = q3 /[(19/11) C] = (11/30)E
lic
Now working backwards we get the circuit,
C
The equivalent capacitance of the above
C

E arrangement (Cnet) = 3/7 C.


C/2 C/2
ub
(11/30)E

(8/3)C
2C A B

C
P

C
C
C C
2/7 V 3/7 V 2/7 V
et

Since the potential is (11/30)E, the charge on


If the potential applied across AB is V, the
the parallel capacitor,
charge on the capacitors (q)
q4 = (11/30)CE ….(iii) q = CV
For the two series capacitors, net C = (8/11)C
rg

Hence the charges on plate X = q = (1/7) CV


Hence, charge in the capacitors
195. Metal plate acts as an equipotential surface,
q5 = (8/30)CE
therefore the field lines should enter normally to
The potential across the (8/3)C capacitor,
Ta

the surface of the metal plate.


V2 = (3/30)E
We now consider the following circuit:
E Competitive Thinking
2. Charge enclosed by cylindrical surface is,
(3/30) E

(2/3)C
Q enc  100 Q. By applying Gauss' law,
1 1
2C  (Qenc. )  (100Q)
2C 0 0

q enclosed
C C 3. Electric flux, ϕ =
C 0

The charge on the X = 2 C capacitor is The flux through a closed surface containing an
Q6 = (3/15)CE = (1/5)CE electric dipole will be zero.

314

Chapter 08: Electrostatics


Totalchargeenclosed 9. Electric field intensity at a point outside
4. Flux =
0 uniformly charged thin plane sheet is given by,
i.e. for first surface, 
E
q 20
1 =
0  It is independent of ‘d’.
For second surface, 1 q
q 10. E=
2 = 4 0 r2
0
9  109   4  1010  1.6  10 19 
=
5. E 
Qenclosed
; Qenclosed remains unchanged.  20  10 
2 2

0
= 1440 N/C

ns
6. Charge 8q is placed at one corner of the cube, 11. There will be zero charge inside closed surface
we can imagine it to be placed at the centre of a
large cube which can be formed using an 1 q
12. E= 
arrangement of 8 similar cubes. 40 r 2
Charge 8q is at centre of the 8 cubes arranged to

io
9  109  3  109
form a closed box.  E= = 3  104 V/m
(3  102 ) 2
 By using Gauss’s law,
8q kq

at
total flux through the bigger cube = 13. E=
0 r2
Er 2 2   0.3
2
1 8q q 2  9  102  109
 Flux through one small cube =   . q=  =
8 0 0 k 9  109 9

7.
lic  q = 2  10–11 C
14.
+
a
ub
q 60
30
+
Let charge enclosed in the sphere of radius a be      
E cos60o   xˆ     sin 60o   yˆ 
P

q. According to Gauss’ theorem, 2E 0  2E 0 2E 0 


  q
 E.ds =    3 xˆ 
0 E  1   yˆ  
2E 0  2  2 
et

q
E.4r2 = A B
0 15. Electric field at point P
q + + -
4Ar3 = ….( E = Ar) due to line charge A as + -
0
rg

shown in figure is, + -


 q = 4ε0Aa3 ….( r = a) +
P -
 q -
8. The field due to a uniformly charged spherical EA  + R
2 0 r -
Ta

+
shell is zero at all points inside shell.  -
= î +
i.e., E = 0 for r < R 20 R
According to Gauss’ theorem, Similarly, electric field due to line charge B,
q   ˆ
E= EB  i
4 0 r 2 20 R
 For r > R, Total electric field at point P is,
  
E 1 E  EA  EB
r2
 ˆ  ˆ
Hence, for r > R, value of electric field = i i
2 0 R 2 0 R
decreases with increase in r.
As, the net charge on dipole is zero, the net flux 
= N/C
0 R
across the sphere is also zero.
315

MHT-CET Triumph Physics (Hints)


16. For a charged conductor of any shape (assuming 24.

dV = E  dx
air medium), VA 2
  dV    30x dx
2
E1 = ….(i)
0 VO 0

2
For a infinite thin plane sheet (assuming air VA – VO =  10x 3  = 80 J
0
medium),
 25. Potential at surface of a sphere is,
E2 = ….(ii)
20 q
V=K = 10 V
Comparing (i) and (i) r
E1 = 2E2. where, q and r are the charge and radius of the
small drop respectively.
Q
17. Total flux = using Gauss’ law As the volume of 27 small drops equals volume
0

ns
of the large drop,
Q 4  4
 flux through one face = 27  r 3  = R 3 
60 3  3
18. where, R is radius of large drop.

io
 R = 3r
a/2 a/2 The total charge on large drop is Q = 27q
O Q
a Q a P Hence, potential at surface of this drop is

at
KQ (27q) Kq
V = = K =9 = 9  10
R (3r) r
Flux due to charge at O,  V = 90 V
Q
1 = 5 
60
lic 26.

E
dV
dr
Flux due to charge at P i.e., electric field is directed along decreasing
Q potential.
2 =
ub
60  VB > VC > VA
Q  Potential is maximum at B.
  = 1 + 2 =
0
 dV
19. Eight identical cubes are required to arrange so 27. E= 
dr
P

that this charge is at centre of the cube formed.    

q r = xi + y j + zk
 
8 0  V  V  V 
E =  i j z
et

x y z
  
q = (6 y) i  (6 x 1 + 2z) j  (2 y) k
a 2a At point (1, 1, 0),
rg

      
a E =  6 i  5 j  2 k = (6 i +5 j + 2 k )

21. V1 + V2 = 0
2a
28. V=
1 q
= 9  109 
65 109  
Ta

40 r 25  102
kq kq r 
  = 0  q =   1  q  V = 2340 volt
r1 r2  r2 
q q 29. Electrical potential, V = Q
22. V=  40 R
40 r 40 R
Now, q = .4r2 and q = .4R2 Electric field, E = 0
.4r .4R
2
(R  r)
2
30. Potential at the surface of bubble,
 V=  V=
40 r 40 R 0 kq
V=
Kq  Kq  r
23. U =  Vdq   dq V  
R  R  Since bubble collapses into droplet of radius R.
1 KQ 2 4
= So (4r2)t = R3
2 R 3

316

Chapter 08: Electrostatics


 R = (3r2t)1/3 when r increases, U decreases
Now the potential of the droplet In case of unlike charges, (q1 q2) is negative
kq Vr  r 
1/3
when r increases, |U| decreases
V1 =   V 
R  3r 2 t 1/3  3t  but because of negative sign, U increases.
35. Potential at A,
31. In region A, V = constant  EA = 0
1  Q1 Q 
42 V1 =   2 
In region B, EB = = 1 Vm1 40  R 2R 
2
In region C, V = constant EC = 0 Potential at B,
40 1  Q2 Q 
= 4 Vm1 V2 =  1 
In region D, ED =
1 4 0  R 2R 

ns
Here EA = EC and EB < ED Work done,
W = q(V1  V2)
32. q q =1C
q  Q1 Q  Q Q 
r W=   2    2  1 
4 0  R 2R   R 2R  

io
2 2 m O
W=
q  Q1  Q2   2 1 
2  40 R 

at
q q
The potential at point O is the addition of the 37. As V will is same in all condition, work will
potentials created by all four charges. be same.
 V = 4 Vq W = q (Vf  Vi)
q 1
Now, Vq = K where, r =  diagonal
lic 38. +q 2q +q
r 2
but, diagonal of a square = 2(sideof square) a a
ub
 Diagonal = 2  2 2 = 4 m 1  q1q 2 q 2q 3 q 3q1 
U=    
1 40  r12 r23 r31 
 r = 4 = 2 m
2 1  2q 2 2q 2 q 2 
1 106 9 =    
P

 Vq = 9  109  = 103 V 4 0  a a 2a 


2 2
q  1 7q 2
 V = 4 Vq = 18  103 volt = 4  =
4 0a  2  80a
et

33. Potential at the centre O,


1 Q 1 1 1 1
V=4 40. =  
40 a / 2 Cs C1 C 2 C3
1 1 1 1
rg

+q +q =  
A B Cs 1 2 4
4
a
O Cs = pF
7
Ta

2
Q Q
41. For series combination, V1 = and V2 =
D C1 C2
C
+q +q V2 C1
  =4:1
10 V1 C 2
 10 9
 9
V = 4  9  10  3 = 1500 2 volt
8  10 2 42. The given circuit can be redrawn as shown in figure
2 where, C = (3 + 2) F = 5 F
1 1 1 1 12 F
 = + +
34. Potential energy of a system of charges is CPQ 5 20 12
 1  q1q 2 P
=
20
=
1
Q 5 F C
U=  
 4 0  r 60 3
In case of like charges, (q1 q2) is positive  CPQ = 3 F
20 F

317

MHT-CET Triumph Physics (Hints)


43. 46. The Given circuit is
4 F 4 F 2 F
12 F 12 F
A B

4 F 4 F
 A B 2 F 2 F B 
B
4 F

A 2 F A 2 F
4 F 4 F 2 F


3 F
8 F
A B 
B

ns
 CAB = 8 F
44. C 2 F
A
C C

io
A B 47. 3 F
C
Q Q2
+80 C

at
3.75 F
4 F
CP = C + C + C = 3C
Q1
3C C 2 F
A B
lic As C =
Q
V
3.75 F  CQ
ub
Q1 C1
3C  C  
Ceq = Q 2 C2
3C  C
3C 2 Q1 2
 3.75 = i.e., 
4C Q2 3
P

3C Q = Q1 + Q2
 3.75 =
4 i.e., Q1 = Q – Q2
3.75  4
 C= = 5 F i.e., Q1 = 80 – Q2
et

3
80  Q 2 2
 
45. The circuit resembles Wheatstone’s balanced Q2 3
network 3(80 – Q2) = 2Q2
rg

240 – 3Q2 – 2Q2 = 0


4 F 4 F
240 – 5Q2 = 0
240
Ta

Q2 =
5
A B 
Q2 = 48 C

48. Effective capacity when connected in parallel


4 F 4 F
= C + C = 2C
2 F
C
Effective capacity when connected in series =
4 F 2
C
A B  A B  2C  =6
2
3C
=6
2
2 F
C = 4 F

318

Chapter 08: Electrostatics


49. Initial charge on 6 F condensor is – C1V1  C2 V2
Common potential V =
Q = CV = 6  106  100 = 6  104 C. C1  C2
When 6 F and 14 F are joined, the total charge 3  106  600  6  106  300
in circuit must remain same. Also, the potential on =
3  106  6  106
both condensor will be finally same as they are
1800  106  1800  106 3600
connected at ends with each other. = 6
= = 400 V
9  10 9
Q
 V =
C parallel 52. C1 C2
4
6  10
 V = = 30 V
(6  14)  106 V1 V2
6 F 6 F
+ 

ns
+  V
Q Given,
C1 = C2 = C (say)
We have,
14 F V = V1 + V2

io
100 V
When capacitor C1 is completely filled with
Q1 C1V1 6 F  V dielectric material of constant K,
Now,  
Q 2 C 2 V2 14 F  V V2 q

at
V1 = ...{ initially V1 = V2 ; V2 = }
Q1 6 K C2
 
Q 2 14 V2
 V= + V2
K
50.
1 F 2 F 5 F
lic  KV = V2 + KV2
 KV = V2(1 + K)
+ – KV
 V2 =
10 V 1+K
ub
Equivalent capacitance of capacitor is given by, 53. The given circuit can be redrawn as follows.
1 1 1 1 The P.D. across 4.5 F capacitor,
   4.5 F 9 F
Cs 1 2 5 9
V  12
P

10 9 
Cs = F   9
17 2 
Now, charge is given by,
= 8V
et

10 100
Q = C sV =  10  C 12 V
17 17 54.
 Potential difference across 2F capacitor
100/17 50 6 F
rg

=  V
2 17
51. 3 F 6 F 6 F 6 F
6 F
Ta

C1 C2 240
Here, 6F, 6F and 6F are in series.
+ –
900 V 1 1 1 1
 = + +
Cs 6 6 6
Q = CV, Here Q is a constant
 Cs = 2 F
1
 C The circuit can be drawn as,
V
C1 V2 3 V 2 F
    2  V1 = 2V2
C 2 V1 6 V1
6 F
Also V1 + V2 = 900 V
 2V2 + V2 = 900 V
V2 = 300 V and V1 = 600 V 240
319

MHT-CET Triumph Physics (Hints)

Here, 2 F and 6 F are in parallel, 59. Balanced Wheatstone’s Bridge


 Cp = 2 + 6 = 8 F 20 µF 20 µF
55. Here 100pF and 100pF are in series, hence their
equivalent is 50pF which is in parallel with 20 µF 20 µF
50pF, whose equivalent is 100pF.
100  50 100
CAB =  pF
100  50 3 150 V
 Ceff = 20 µF
56.  Q = Ceff V = 20 × 10–6 × 150 = 3 × 10–3 C
Q S T V 60. The two capacitors attain common potential

ns
A P C R C B
C C C U C (VC) given by the relation,
Total charge Q0 Q
VC =  
Total capacitance C  2C 3C
C C
The final charges on two capacitors are

io
C Q.S.U C
 A.P.R T.V.B
CQ Q 2Q
Q1 = CVC =  and Q2 = 2CVC =
C C 3C 3 3

at
61. The charge flowing through C4 is
3C 3C q4 = C4  V = 4 CV ….(i)
 A B
lic For the series combination of C1, C2 and C3,
1 1 1 1
  
The equivalent capacitance between A and B is Ceq C 2C 3C
1 3C  3C
  Ceq = 1.5C 1 6  3  2 11 6C
Ceq (3C)(3C)     Ceq =
Ceq 6C 6C 11
ub
57. C1 and C2 are in parallel, Now, Ceq and C4 form parallel combination
 Ceq1 = C1 + C2 = 18 pF giving,
Ceq2 and C3 are in series, 6C 50C
C = Ceq + C4 =  4C 
11 11
P

Ceq1  C3
 Ceq 2 = = 6 pF 50
Ceq1  C3  Net charge, q = CV = CV
11
Ceq2 and C4 are in parallel, Total charge flowing through C1, C2, C3 will be
et

 Ceq2 + C4 = 6 + 9 = 15 pF 50 6CV
q = q  q4 = CV  4CV = ….(ii)
58. To hold 1 kV P.D., minimum four capacitors, 11 11
which can withstand P.D. upto 300 V, As C1, C2, C3 are in series combination, the
rg

connected in series are required. charge flowing through them will be same.
1  From equations (i) and (ii),
 Cs = F 6CV/11 3
4 the required ratio = 
4CV 22
Ta

Now, to get capacitance of 2 F, 8 such series


combinations should be connected in parallel. 
8 62. Electric field in vacuum Ev = and
i.e. Ceq = = 2 F. 0
4

1 2 3 4 in medium, E =
1 0 k
If k > 1, then E < E0.
2
Electric field between plates, E = q


63.
8 0 A
1 kV 2
Electrostatic force, F = qE = q
0A
Hence, the minimum number of capacitors
required are 8  4 = 32. Thus, F is independent of distance between the
plates.
320

Chapter 08: Electrostatics


0A 72. Force between plates of capacitor
64. C=
d  q 
F = qE = q  
Hence as d increases, C decreases.  2A 0 
Q is constant  V increases. q2
F=
66. After separation: 2A 0
i. charge = constant  q = CV
A C2 V 2
ii. capacity C = 0 F=
d 2A 0
Capacity decreases with increase in distance.
Q  A 0  2
iii. V=   CV
C F=  d 
Potential increases as capacitance decreases. 2A 0

ns
CV 2
V 30   10  F=
67. E=  = 2000 Vm1 2d
d 2  102
73. +q –q
68. Capacitance of a parallel plate capacitor with a

io
dielectric of dielectric constant k,
C = k 0 A
d

at
….  d 
V
 C = k 0 AE 
V  E
12 4 As separation between the plates are decreasing
= k  8.86  10  10  10
6
–12
 15 × 10
500
lic as they approach each other
and V = E.d
 k = 75 = 8.5 Electric field remains constant between the
8.86
plates, so V  d
ub
69. Volume of 8 drops will be same as volume of 1 q2
large drop formed by combining smaller drops. Now, force on each plate = But, F = ma
2A0
4  4
 8  r 3   R 3 acceleration (a) =
F
3  3 m
d
P

 R = 2r q2
the capacitance of bigger drop is i.e., acceleration (a) =
2A 0  m 
C = 40R = 40 2r = 2C t
a = constant
et

70. The equivalent circuit is shown in the figure. So V-t curve V


P
1 2
R
rg

4 3
t
X Y
74. When a dielectric is introduced between the
3 2 plates, as battery remains connected, E or V
Ta

Q remains unchanged.
The condensers P and Q are in parallel. Hence Charge on plates before introduction of
their equivalent capacitance is 2C. This dielectric medium is, q0 = C0V
combination is in series with capacitor R. Hence After inserting the medium, q = kC0V
the equivalent capacitance between X and Y is Induced charge, q = q  q0
given by = C0V (k  1)
C  2C 5 
CPQ = = 90  1012  20   1 = 1.2 nC
C  2C 3 
2 2 0 A
= C= . 75. Electric field inside parallel plate capacitor having
3 3 d Q
 charge Q at place where dielectric is absent =
71. E=   = k0E A 0
k 0
Q
  = 2.2  8.85  1012  3  104  6  107 C/m2 and where dielectric is present =
kA0

321

MHT-CET Triumph Physics (Hints)


76. 1

Ceq =     1 2
1 1 CC
+
–  C1 C 2  C1  C 2
+
– 5  10 10 10
A +
k =  F =  10–6 F
+ –
15 3 3

+ 1 1 10
+ – U = CeqV2 =   10–6  3002 = 0.15 J
2 2 3
t
1 Q2
Potential difference between the plates, 82. U=
2 C
 
V = Vair + Vmedium =  (d  t) + t  Increase in energy =
1
Q 22  Q12 
0 k 0 2C 
 t Q t 1
 V= (d  t + ) = (d  t + ) = [0.52  0.12]

ns
0 k A 0 k 2  4810 6
Q 0 A 0 A 106
 C= = = = [24  102]
V  t  1 96
dt  d  t 1 
 k  k = 0.25  104 = 2500 J

io
 0 A  0 A  0 A 2 0 A
77. C=    Q
q
2d d 2d d 83. W= C dq

at
q
78. Ceq= C1+C2+C3 10C
A A A
q = 10C
q q 2  75C
= 0 + 0 + 0
3d 6d 9d lic  W= 
q = 5C C
dq =
2C
5C
=
2
11 A Let W be the work done in increasing the
= 0
18d voltage across capacitor from 10V to 15V.
15C
dE q 2 
q = 15C
q 125C
79. Displacement current = Ak  0
dt  W =  dq =  10C =
C 2C 2
ub
6 12 q = 10C
2.21  10 = 1  k  8.85  10  25
W 125
6
2.21  10  10 3  =
 K= 12
= 9.98  103  10 W 75
8.85  25  10
 W = 1.67W
P

80. 2F 4F 84. When connected in series,


 Ceq 1  NC1 ; V1  3V
1
et

When connected in parallel,


+ –
6V  Ceq 2  N2 C2 ; V2  V
cc 2 4 8 1
Ceff = 1 2 = = F
rg

c1  c 2 24 6 U= CV2
2
8 1 1
   
–6
Q = Ceff V =  106  6 = 8  10 C  Ceq V12  Ceq V22
6 2 1 2 2
Ta

 Q = 8C 1 C1 1
Now , 9V 2  N 2C 2 V 2
2 N1 2
1
U= CV2 C2 N1N 2
2 C1 =
9
1 8 
=    106   62 85. When another capacitor is connected in parallel,
2 6  then capacitance increases by a factor 2 and
= 24J potential difference becomes half.
1
81. C1 = 5F C2 = 10F  Final energy (U) = CV2
2
2 2
1  V  CV
=  2C    =
2 2 4
 Total electrostatic energy of resulting system
300 V decreases by a factor 2.
322

Chapter 08: Electrostatics


1 1 q1q 2
86. U= C  V22  V12  91. F =

40 r  t  t k

2
2
1
 15  106   252 152 
2
=  r r   3r 
2
9 2
2 r     4  
F   2   2  
r
 =  2
= 42
15  10 6   625  225  F r2 r 2
r
=
2 F 9

6
15  10  400 F 4
= 4
2 F = F
9
= 15  10–6  200
92. When two air capacitors are connected in series,
= 3  10–3 J
their effective capacity is,

ns
C  C C2 C
87. Given that net FE and FG is zero. C1 =  
C  C 2C 2
i.e., FE = FG When one of them is filled with dielectric
1 (e) 2 Gm 2 material, effective capacity becomes,

io
  2  2 ….(i)
4 0 d d 1 1 1
  ….(where K is dielectric constant)
In case of hydrogen atoms, net charge on one H- C 2 C KC
atom will be e 1 1 1

at
  1
G C2 C  K 
 e = m ….[from (i)] C CK
 1   C2 = =
   1 (K  1)
 40 
lic 1  K 
6.67  1011 Change in effective capacities,
= 1.67  10–27
9  109 CK C
C2 – C1 = 
= 1.438  10 –37
C K 1 2
ub
 K 1
= C  
88. Force on charged particle in electric field,  K 1 2 
F = eE  2K  (K  1) 
= C 
eE  2(K  1) 
P

 Acceleration experienced by it, a =


m  K 1 
eE eE = C 
For electron, ae = and for proton ap =  2(K  1) 
me mp
et

C  K  1
=
As, mp > me, ae > ap 2  K  1 
As electron is pulled with greater acceleration, it  
will take lesser time to cover height h. 93. Electric flux, E = E.d S =   EdScos 
rg

=  EdScos90 0
 
 

89. AB =  B  A  = a 4ˆj  3kˆ
  The lines are parallel to the surface.
Ta

 
 Work done = F AB 95. Refer Shortcut 7
  ˆ Let r be radius of each small drop and R be
= q
 20 

 k  a 4jˆ  3kˆ  radius of bigger drop.
The volume remains constant
3qa 4 4
  R3 = n  r3
20 3 3
90. As it is evident from symmetry of figure, plates  R = n1/3r
2 and 4 have charges + Q/2 each. For the small drop,
q  Q / 2 Capacitance, C0 = 40r and
We know that, C = C= charge q0 = C0V = 40rV
V V
For the bigger drop,
0 A  0 A  Capacitance, C = 40R and
 Q = 2CV = 2 V ….  C  
d  d  charge Q = nq0
323

MHT-CET Triumph Physics (Hints)


Q nq 0 This shows as C increases Q increases but not
 Potential of bigger drop = 
C 4 0 R linearly. Also the given relation does not
correspond to exponential graph. Hence correct
n(4 0 rV) r  1 
= = nV    n  1/3  V = n2/3V choice is (B).
4 0 R R n  Charge

96. Let the charge of each drop is g Q2


g
 C=  g = CV
V
C
 charge of final drop  Q = ng 1F 3F
Let ratio of each small drop is r and big drop is
99. The capacitance of a parallel plate capacitor in
R
the absence of the dielectric is
V = nv
0A
C0 = ….(i)

ns
4 3 4
R = n. r 3 d
3 3
The capacitance of a parallel plate capacitor in
R3 = nr3
the presence of dielectric slab of thickness t and
R = (n)1/3 r
dielectric constant k, is
Potential on big drop

io
0A 0 A
kQ C = =
V = t  3   3d 
R (d  t)     d  d    
k  4   4k 
kng

at
V = 1/3 0A 4k0 A
n r C =  ….(ii)
 d 3d  d(k  3)
Ratio of energy start in big drop to small drop   
1 kg 2/3  4 4k 

U 2 QV ng. n
U

1

QV
gV
 r
lic
kg
Dividing equation (ii) by equation (i) we get,
C 4k 0 A d 4k
gV g.   
2 r C 0 d(k  3)  0 A k  3
n 5/3 100. In steady state, current through capacitor is zero
U =
ub
1 E r
1
97. Using, U = CV 2 , I r1
2
1 A0  V P Q
P

U=   (Ed)2 ….  E  C
2 d  d  r2
R S
1
= 0E 2Ad  VPQ = VRS
2
et

E
Also, I =
98. Q1 r  r2
Q  E 
1F  VPQ =   r2  = VRS
rg

 r  r2 
C r2
Q2  Charge on capacitor is, QC = CVPQ = CE
2F (r  r2 )
Ta

101. Electric field E is given by,


E V V  potential difference 
E= ….  
d d  plate separation 
2 2Q
Q2 = Q= ….(i) V
2 1 3  E= ….{ d = h}
Q = CRV h
….(i)
CR = (1 F||2 F) series with C
Q
3C But V =
 CR = C
C3
it
C3  V= ….(ii)
 Q = E   C
 C3
Substituting equation (ii) in equation (i) we get,
 Q2 = 
2 3CE  2CE it
 …. using (i) E=
3 C3 C3 Ch

324

Chapter 08: Electrostatics


102. Cnet = 5 F 1 2F  8 F
=   V2
Qnet = 5  8 = 40 C 2 10 F
We know, 1
Q2F = 2  8 = 16C =  1.6 F  V 2
2
 Q4F = Q12F = Qnet  Q2F U
 % loss of energy = 100
...{9F|| 3F = 12F} U
= 40  16 = 24C =
1.6
 100
Voltage across 4F and 12F can be given as, 2
V4F + V12F = V = 80 %
Q12F 24
 V4F = V  =8 = 6V kdq
C12F 12 106. Potential at O, V =  Lr
 V12F = 2V

ns
Q
 dq = dL
i.e. V9F = 2V L
 Q9 F = 9  2 = 18 C kQ
L
dL
 Q = Q4 F + Q9 F = 42 C  V=
L  Lr

io
0
kQ 9  109  42  106
 E=  = 420 N/C kQ kQ
ln  L  r 0 =
L
r2 30  30 = ln2
L L
103. q1 = CV1 Q1, C,V Q

at

= ln 2
+ 40 L

 lic 107.
+ 
q2 = CV2 Q2, C,V
Q a
Initially from charge conservation
q1 + q2 = Q1 + Q2
ub
r dr
CV1 + CV2 = CV + CV
C(V1 + V2) = 2CV
V1  V2
V=
2
P

Charge in potential energy of system Electric field due to charge Q at r = a is,


U = Ui  Uf kQ
Ea = ….(i)
1 1  1 1  a2
=  CV12  CV22    CV 2  CV 2  Consider a shell of thickness dr in the region
et

2 2  2 2 
a  r  b.
1
= CV12  V22  2V 2  A
2 charge on shell, dq = Area   = 4  r2d r
r
rg

1   V  V  
2
b
= C V12  V22  2  1 2  
2   2    total charge in the region a  r  b is, q =  dq
a
1  1 1 
 C V12  V22  V12  V22  V1V2 
b
b
 r2 
Ta

2  2 2  = 4A  r dr = 4A  
a  2 a
1  V12 V22 
 C   V1V2   2
q = 2A [b  a ] 2
2  2 2  Electric field at r = b is,
1 1
CV12  V22  2V1V2   C  V1  V2  k  2  A  b 2  a 2   Q 
2

4 4 Eb = ….(ii)
b2
1
U = C  V1  V2 
2
For electric field to be constant in the region
4
a  r  b we must have, Ea = Eb
1 from equation (i) and (ii)
104. Initial energy stored, U =  2F  V 2  2 A  b 2  a 2   Q 
2 kQ    
 Energy dissipated on connection across 8 F,  k
a2 b2
1  C1 C 2  2 Q b2
U=  V   Q  2 A  b 2  a 2 
2  C1  C 2  a2

325

MHT-CET Triumph Physics (Hints)

Q b2  Q a 2 Kq
 2 A  b 2  a 2  Electric field, E =
a2 r2
Q  b2  a 2  6
 2 A  b 2  a 2   E= V/m
a 2 r2
Q
 A= 112. y
2a 2

108. To make potential zero net charge on two p


capacitors must be made zero. Hence, capacitors  x
must be connected such that
Q = Q1  Q2 = 0 z

 C1V1  C2V2 = 0 p = p cos î + p sin ĵ
 C1V1 = C2V2

ns

 120 C1 = 200 C2 E1 = E î ….(Given)


  
 3 C1 = 5 C2 T1  p  E1
Q = (p cos î + p sin ĵ )  E ( î )
109. C =

io
 
V
kˆ  pEsin  kˆ ….(i)
1 Q
but V = for a spherical body.
4 0 R 
E 2  3 E1 ˆj ….(Given)

at
 C = 40R
 

 C = 4  8.85  1012  6400  103 T2  p cos  ˆi  p sin  ˆj  3 E1ˆj
 C = 7.1  104 F
kˆ  3 pE1 cos  kˆ ….(ii)
110.
lic
r1 = 9 cm
from (i) and (ii), we get,
r2 = R pE sin  = 3 pE cos 
9 cm sin  3 pE
O 
ub
R cos  pE
tan  = 3
 = tan–1 3
q
Capacity of isolated sphere, C =  = 60
P

V
C = 40r1 ….(i) 113. Potential at O, V = Kdq
Capacity of earthed concentric hollow sphere  Lr
et

 r1 r2  Q
is; CH = 40    dq = dL
 r2  r1  L
 r1 R  KQ L
KQ
 10C = 40   ….(ii)  V= dL = ln  L  r 0L
 Lr
rg

 (R  r1 )  L L
0
Dividing equation (ii) by equation (i),
10C  r1 R  1 KQ Q
= ln2 = l n2
=  L 40L
Ta

C  R  r1  r1
9R 1 KQ
 10 = 114. V =
(R  9) 9 R
r
 10R  90 = R VR A
 Q=
 9R = 90 K
 R = 10 cm KQ
As, E =
q r2
111. Electric potential, V = K
r Substituting for Q,
Considering the 60 V equipotential surface, K  VR 
Kq
 E=  
60 = r2  K 
10 102
VR
 Kq = 6  E=
r2

326

Chapter 08: Electrostatics


q q For electric field to be constant in the region
115. V 
40r 40 R a  r  b, we must have
Ea = Eb
q = 4r 2 and q  4R 2
From equations (i) and (ii),
4r 2 4R 2 KQ  2 A  b 2  a 2   Q 
V   K  
40r 40R a2 b2
Q b2
 r  R    Q  2 A  b 2  a 2 
 V a2
0
Q b2  Q a 2
 2 A  b 2  a 2 
a2
1 q1
116. For 1st sphere: V1 = and Q  b2  a 2 

ns
40 r1
 2 A  b 2  a 2 
1 q2 a2
For 2nd sphere: V2 =
4 0 r2 Q
 A=
But V1 = V2, 2a 2

io
q1 q 118. Initially, charges on the two spheres will be,
 = 2
r1 r2 q1 =  (4R2) and

at
q q2 =  [4 (2R)2] =  (16R2)
Let,  = be the surface charge density of the
A  q1 + q2 = 20 R2
sphere. lic After redistribution of charges, final charges on
1 A1 2 A 2 the spheres will be,
then, 
r1 r2 Q1 R

 A r 4r 2 r r 5 Q 2 2R
 1  2 1  22 1  2 
2 A1r2 4r1 r2 r1 4  Q2 = 2Q1
ub
117. As charges are conserved, total number of
charges on both the spheres will remain
constant.
i.e., q1 + q2 = Q1 + Q2
P

a  Q1 + 2Q1 = 20 R2
b Q 20 5
dr  Q1 = R2 = (4R2)
et

r 3 3
40 5
and Q2 = R2 = (16R2)
3 6
5 5
rg

Electric field due to charge Q at r = a is,  1 =  and 2 = 


3 6
KQ
Ea = ….(i) 1
a2 119. Initial energy stored U = CV2
Ta

Consider a shell of thickness dr in the region 2


a  r  b. When terminals b and c are connected, C1 and
b b b
A
C2 are connected in series with each other.
q =  dq =   dV =  r  4  r dr 
2
 Energy dissipated on connecting switch across
a a a
C2,
….(Volume of spherical shell = 4r2 dr)
1  C1C 2  2 1  C  2
b
 r2 
b
U=   V =   V
= 4A  r dr = 4A   2  C1  C 2  2 2
a  2 a
U
 q = 2A [b2  a2] % loss of energy =  100
U
Electric field at r = b is,  C / 2   100
=
K  2  A  b 2  a 2   Q  C
Eb = ….(ii)
b2 = 50 %

327

MHT-CET Triumph Physics (Hints)


120. 1  4Q 
 V0 =  
40  r2 
 1 Q
 Vsphere =   + V0 and
T T cos  4 0  r1 
1 Q
 Vshell =   + V0
40  r2 
qE
T sin  Q 1 1
  Vsphere – Vshell =   
40  r1 r2 
mg
At equilibrium, =V ....[from (i)]
T cos θ = mg ….(i) This shows that because of the presence of
T sin θ = qE ....(ii) charge on the outer shell, potential everywhere

ns
From equations (i) and (ii), inside and on the surface of the shell will
qE change by the same amount and hence the
tan θ =
mg potential difference between sphere and shell
q = 5 × 10–6 C, E = 2000 V/m, m = 2 × 10–3 kg

io
will remain unchanged.
5  106  2000
 tan θ = K0 A
2  103  10 122. Capacitance of element =
dx

at
1
 tan θ =
2
 θ = tan–1 (0.5) lic
121. Shell  
x dx
K 1  x  0 A
Sphere Capacitance of element, C =
dx
+Q –Q +Q
ub
d
r1 1 dx
 C   K A 1  x 
0 0
r2
1 1
 ln 1  d 
C K0A
P

x2 x3
Let r1 and r2 be the radii of the sphere and Now, log(1+x)=x− + ........
2 3
hollow shell respectively.
Given, d << 1
et

1 Q 1 Q
 Vsphere =   and Vshell =   1 1   2d 2 
40  r1  40  r2    d  
C K0A  2 
 potential difference between the two spheres
rg

Vdiff = Vsphere – Vshell 1 d  d 


 1  
Q 1 1 C K0 A  2 
=    =V ....(i)
40  r1 r2  K0 A  d 
C= 1  
Ta

When the shell is given (– 4Q), the potential at d  2 


its surface and inside will change.

Hints to Evaluation Test


   
rp
  VP  VQ =  E  ( r p  r Q )
1. VP  VQ =  E.d l

rq 
=  (2iˆ  ˆj). (1 2)iˆ  (2 1) ˆj  (0  1)kˆ 

If E is constant, then

=  (2iˆ  ˆj). 1iˆ 1jˆ  1kˆ 
rp
 
VP  VQ =  E   dl = (2 + 1)

rQ =1V

328

Chapter 08: Electrostatics


2. The initial potential of the outer shell, We are traversing the loop from negative to
KQ K(2Q) positive. Therefore, potential drop due to battery
V2 = +
R2 R2 can be taken as positive.
K(Q  2Q) Writing the equation for net potential drop along
= the loop,
R2
 q1 q1 
After connecting the shells, by a wire, the E   0  q1 = 3CE
potentials of the shells,  6C 6C 
Kq K(3Q  q)  Charge flow along the 6C capacitor = 3CE
V1 = + and
R1 R2  Energy stored in the capacitor
Kq K(3Q  q) 1
V2 = + = QE
R2 R2 2
1
where ‘q’ is the remaining charge on inner shell. = q1(E/2)

ns
As inner and outer shell are connected, 2
V1 = V2 = 3/4 CE2
In the second case, when the switch 2 is ‘ON’,
Kq Kq
 =  q = 0 or R1 = R2 the circuit diagram would be as given in figure
R1 R2

io
(ii)
The later is not possible  q = 0 B
K(3Q) 4C 4C
Thus, V2 =  V2 = V2

at
R2
So the potential of the outer shell does not 3C
change after connecting with wire.
6C 6C
 (A) is correct.
3. q +q q +q
lic A q

C 2C
5V E
ub
Figure (ii)
The circuit is symmetric about AB. Therefore,
As both the earthed points are at 0 V, we can we can say that charge entering the 4C capacitor
redraw the circuit as, to B would be the same as charge leaving B
2CC 2 through the other 4C capacitor. Therefore, there
P

C and 2C in series = = C
2C  C 3 would be no charge flow along 3C capacitor.
2  Hence, Energy in the 3C capacitor = 0.
q = CeqV =  C  (5 V)
3  5. Total number of surfaces = 6
et

2  Total charge, Q = 24 C
=   6 F  (5 V) = 20 C Q
3  Total flux,  =
0
rg

4. The circuit with the switch 1 in ‘ON’ position is So, flux through each surface
shown in figure (i). We apply the Kirchhoff’s
 24 4
2nd law. Consider the closed loop through the = = = V-m
6 60 0
6C capacitors.
Ta

1
4C 4C 6. Potential V 
r
r
V = V
r
6C 6C q 16
= V =8V
2
7. An electric dipole produces uniform electric
E field. For uniform electric field, equipotential
Figure (i) surface is planar.
Potential drop across the capacitors 8. By using W = QV
q q  2
=  1  1  V = = 0.1 volt
 6C 6C  20

329

MHT-CET Triumph Physics (Hints)


9. Net charge inside the sphere 16. If two charges are of the same sign, they repel
=   dV each other. Thus if two protons are brought
closer, some work is done against the force of
Due to spherical symmetry, we get,
R
repulsion. Therefore the electric potential
Q = 0
4r 2rdr energy of the system increases.
R
= 4A 0 r 2 (R  r)dr 17. q q
+Q
R R 
4 4
x 2x
= 4A   
 3 4 
Total potential energy of the system is
3Q 1  (q)(Q) (Q)(q) ( q)(q) 
 A= U=  
R 4 40  x 2x 3x 
10. Electric flux through: U = 0 (given)

ns
i. X-Y plane 3  100 = 300 1  qQ qQ q 2 
 0=    
ii. Y-Z plane 8  100 = 800 40  x 2x 3x 
iii. X-Z plane 4  100 = 400
3 qQ q 2
Hence, the required ratio = 3 : 8 : 4 0= 

io
2 x 3x
11. Using Shortcut 7 Q 1 2 2
Potential of the bigger drop    
q 3 3 9

at
= n 2/3  potential on each droplet
6
V = 1  = 9  109  25  10
= 642/3  10 q
18.
= 42  10 40 r 10
= 16  10 V = 22500 V
= 160 V
lic
19. V = V1 + V2
 9 9

12. Redistribution of charges takes place and during = 9  109   4  10  2  10 
flow of charges some energy is lost as heat.  3 2 
ub
20 =3V
20
10V 2 
13. 5  (10V  4)dV = 5   4V  20. There are two loops, each having two capacitors
0  2 0
of 20 F each in parallel.
= 5[5  400 + 80]
 Ceff. = 20 + 20 = 40 F for each loop.
P

= 5[2000 + 80]
Now, these two capacitors of 40 F each are in
= 5  2080
series.
= 10400 J
40 40 1600
 Ceff. = = = 20 F
et

Q k 40  40 80
14. C= ,C= 0
V d
1
1 1 21. u= CV2
C C 2
rg

V d
2u 250
 As ‘d’ increases, C decreases  V= = = 10 6
Hence ‘V’ increases. C 100106
 V = 103 V = 1000 V
The electric potential V (x, y, z) = 3y2  2z
Ta

15.
dV
E = 
dr
  ˆ V ˆ V ˆ V 
E =   i x  j y  k z 
 

=  6y ˆj  2 kˆ 
=  6y ˆj  2kˆ
At point (1, 2, 4)
 ˆ ˆ
E =  6(2) j  2k
= 12ˆj  2kˆ

 |E|= 12 2  2 2  148  2 37 units

330
9 Current Electricity

Hints

 I  Ig = nIg
Classical Thinking I
 Ig =

ns
n 1
2. At a junction,
Ig 1
Current entering = Current leaving  
 I+4+2=5+3I=2A I n 1
38. The resistance of an ideal voltmeter is

io
8. In Wheatstone bridge experiment, some
smaller part of the experimental wire may be found considered as infinite so that it does not change
under metallic strip at the ends. This additional the current in the circuit.

at
length has some resistance known as end 40. The voltmeter is a high resistance galvanometer.
resistance. Hence, to measure the accurate length,
first the balancing length is obtained by keeping
the known and unknown resistances left and right Critical Thinking

by interchanging the known and unknown


lic
gap and then again the balancing length is obtained 1. According to Kirchhoff's first law,
At junction A, IA = 2 + 2 = 4 A
resistances to nullify the effect of unaccounted
At junction B, IB = IBC + 1 = 4 A  IBC = 3A
resistance offered by terminals.
2A
ub
1A 1.3A
X 20 1
16. = X=  = 0.25 
1 80 4 A B
C
27. Potentiometer is said to be more sensitive if it 2A I
gives large change in the balancing length for a
P

At junction C, I = IBC  1.3 = 3  1.3 = 1.7 A


dE
small change in p. d. (i.e., should be small) 2. According to Kirchhoff’s law,
dl
V ICD = I2 + I3
l
et

E=
L 3. By Kirchhoff’s current law,
dE V V
 =  be small. 15 A
dl L L I2 B 3A
rg

A
28. When null point is obtained on potentiometer wire,
the cell whose potential difference is to be 8A
I3
measured does not supply current to potentiometer
D
Ta

wire since galvanometer deflection is zero. C


Therefore, current through the potentiometer wire I
5A I1
is due to auxiliary battery.
29. Potential difference per unit length, Let the currents I1, I2 and I3 be as shown in the
V 2 figure.
= = 0.5 V/m Applying Kirchhoff’s junction law to D,
L 4
I1 + 5 = 8  I1 = 3 A
G Applying it to A,
35. Given: S =
n I2 = 8 + 15 = 23 A
 Ig  Applying it to B,
But S =   G I2 + 3 = I3  I3 = 26 A
 I  Ig 
Applying it to C,
G I
 = g G I1 + I = I3
n I  Ig  I = I3  I1 = 26  3 = 23 A
331

MHT-CET Triumph Physics (Hints)


Alternate Method: 9. Since E1(10 V) > E2 (4 V), hence current in the
circuit will be clockwise.
Total current entering the loop ABCDA is
E1
15A + 3A + 5A = 23 A. Hence, total current a 1 e E2 2  b
leaving the loop ABCDA will be same as 23 A.
10 V 4 V
4. According to Kirchhoff’s junction law, I
3
(+ 5) + (+ 4) + (3) + ( 5) + I = 0 4A d c
 I=1A x 3A
[ve sign shows that Applying Kirchhoff’s voltage law,
current is flowing away 5A I (1  I) + 10  4  (2  I)  3I = 0
from x.]  I = 1 A (a to b via e)
5A
5. 20 V 2  V 4 5V 10.

ns
I1 2V 2 I1
A I1 I2 B A B
2
I3 D C
I2 2V 2  I2

io
(I1 + I2) (I1 + I2)
0V
Let V be the potential of the junction as shown
in figure. Applying junction law, we have E F

at
5
20  V 5V V0
+ =
2 4 2 Applying Kirchhoff’s second law for closed
 40 – 2V + 5 – V = 2V loop AEFBA we get,
 5V = 45  V = 9 V
lic  (I1 + I2)  5  (I1  2) + 2 = 0
V  7I1 + 5I2 = 2 ….(i)
 I3 = = 4.5 A Again, applying Kirchhoff’s second law for a
2
closed loop DEFCD we get,
ub
6. According to Kirchhoff’s voltage law in loop (I1 + I2)  5  (I2  2) + 2 = 0
ABQPA,  5I1 + 7I2 = 2 ….(ii)
  (i1 + i2)R  i1r1 + E1 = 0 Multiplying (i) by 5 and (ii) by 7 we get,
 E1  (i1 + i2)R  i1r1 = 0 35I1 + 25I2 = 10 ….(iii)
P

7. Applying Kirchhoff’s voltage law to the 35I1 + 49I2 = 14 ….(iv)


given loop QPQ, Subtracting (iv) from (iii) we get,
1
 24I2 =  4  I2 = A
et

1  4 V 8V 2  6
P Q
I Substituting the value of I2 in equation (i)
we get,
I
9 1 7 1
rg

7I1 = 2  5   7I1 =  I1 = A
6 6 6
1 The current through the 5 ,
2I + 8 – 4  (1  I)  9I = 0  I = A 1 1
3
Ta

= I1 + I2 = +
1 6 6
 Potential difference across PQ = 9=3V 1
3 = A
3
8. Applying Kirchhoff law,
(2 + 2) = (0.1 + 0.3 + 0.2)I 110
11. I= ….(i)
 I=
20
A 20  103  R X
3 Now, V = IR
 Potential difference across A  110  3
20 4  5=    20  10 ….[From (i)]
= 2  0.1  = V (less than 2 V)  20  10  R X 
3

3 3
Potential difference across B  105 + 5RX = 22  105
5
20 10
= 2  0.3  =0  RX = 21  = 420 k
3 5

332

Chapter 09: Current Electricity


12. Suppose current through different paths of the 15. 2V
circuit is as follows: I1
A B
28 54 I1
I 1 + I2
I2
6V 4V 4
i1 1 2
i3 i2 2V

D I2 C
8V 12V 2
Applying Kirchhoff’s voltage law to loop (1) Applying Kirchhoff’s voltage law to ABCA,
and loop (2) we get, 2  4I1  4(I1 + I2) = 0 ….(i)
1 Applying Kirchhoff’s voltage law to ADCA,
28i1  6  8  i1   A and

ns
2 2  2I2  4(I1 + I2) = 0 ….(ii)
1 Subtracting equation (ii) from equation (i),
54i 2  6  12  i 2   A
3  4I1 + 2I2 = 0
5  2I2 = 4I1  I2 / I1 = 2
 i3  i1  i 2   A

io
6 16. I – I1
A R D R E
13. I 1 I – I1
I1

at
I1 V
6V R
A 2 R=2 I R

B V
lic
Applying Kirchhoff’s law to loop A, B C F
6 – I  2I1 = 0 ….(i) For loop ADCBA,
Applying it to loop B, IR + I1R + V – V = 0
ub
– 2(I  I1) + 2I1 = 0 ….(ii)  (I + I1)R = 0  I1 = – I
 – 2I = – 4I1  I = 2I1 Now, In loop AEFBA,
Substituting in equation (i), IR + (I – I1) R + (I – I1)R – V = 0
6 – 2I1  2I1 = 0  IR + IR – I1R + IR – I1R = V
P

6  3IR – 2I1R = V
 I1 = = 1.5 A
4  3IR – 2(– I)R = V
14. The circuit can be simplified as follows: V
 5IR = V  I =
et

B C 5R
I1 30 
I3 I3 17. 60 
A D
i
rg

40  40 V 15 1 5
I2 A B
F E i1
40  80 V 1A 2 1A
i2
Ta

Applying Kirchhoff’s current law to junction A,


I3 = I1 + I2 ….(i) 10 
Applying Kirchhoff’s voltage law for the loop
ABCDA, Applying Kirchhoff's law
At junction A:
 30I1 + 40  40I3 = 0
i  i1  i 2  1 ....(i)
  30I1  40(I1 + I2) + 40 = 0 ….[From (i)]
 7I1 + 4I2 = 4 ….(ii) For Loop (1)
Applying Kirchhoff’s voltage law for the loop  60i  (15  5)i1  0
AFEDA,  i1  3i ....(ii)
 40I2 + 80 + 40  40I3 = 0 For loop (2)
  40I2  40(I1 + I2) =  120 ….[From (i)] – (15 + 5) i1 + 10 i2 = 0
 I1 + 2I2 = 3 ….(iii)  i2 = 2i1 = 2(3 i) = 6i ....(iii)
On solving equations (ii) and (iii), On solving equations (i), (ii) and (iii) we get
I1 =  0.4 A i = 0.1 A
333

MHT-CET Triumph Physics (Hints)


Alternate Method: 20. Four resistances forming a Wheatstone’s
Branch current = network are 8 , 12 , 6  and 27 . After
 Resistance of opposite branch  shunting the 27  resistance with say, S, the
Main current   
 Total resistance  balance condition will be,
 20  8 6 1 3  27  S 
 3 
=  =
12  27S  3 27S
 i  1   = 0.1 A  
20  27  S 
  60 
 3   27 S = 243 + 9 S  S = 13.5 
(Branch 60
current) 21. Given circuit is a balanced Wheatstone bridge
i circuit. Hence it can be redrawn as follows:
12 

ns
1A 15 5 4 8 12 
10
a b a
20/3  b
18.

io
A 2 4 6
R1 = 3  R2 = 6  6
12  6

at
 RAB = =4
X 10  Y (12  6)

22.
R4 = 5  R3 = 10  R R

B
lic R
R R R 2R/3 2R/3
R1 R 2
 =
R4 R3 A R B A R B
ub
Wheatstone’s bridge network is balanced.
Hence there is no current flowing through AB 2R/3 2R/3
R R R R
(through R5).
 The given circuit is equivalent to R R
P

3 A 6
As the bridge is balanced,
X Y 2R
 Req =
et

3
5  B 10 
Alternate Method:
Rxy = (3 + 6) | | (5 + 10) Refer Shortcut 1.
9  15
rg

 Rxy = 23. Let S be shunted with resistance X.


15  9
9  15 At balanced condition,
= P P 2 2 3X
24     2
Ta

45 Q SX 2 3 X 3 X
=  S X 3 X
8
3X = 6 + 2X  X = 6 
R R
19. As the bridge is balanced, AB = AD
R BC R DC 24. The resistances in four arms of a Wheatstone’s
15  6 15  (6 || 6) bridge are, 10 , 10 , 10  and 20 .
 =
(X || 8)  3 4  (4 || 4) Let S be the resistance to be connected across
21 18 20 .
 =
 8X  42  Balance condition is,
 3
8X 10

10
 20 S = 10 (20 + S)
 168 + 21 X = 33X + 72 10  20S 
 
96  20  S 
 12X = 96  X = =8
12  10 S = 200  S = 20 

334

Chapter 09: Current Electricity


25. B 31. For the balance condition,
4 4 P R
= where X is the resistance with which
Q S || X
A 16  C
S is shunted,
3 4
4 4  =
3  6 X 
D  
 6 X 
According to the principle of Wheatstone’s
 6X = 24 + 4X  X = 12 
bridge, the effective resistance between the
given points is 32. As current through G is zero, it balances
= (4 + 4)  || (4 + 4)  Wheatstone’s bridge.
= 8 || 8 = 4 6 9
   R = 18 

ns
12 R
Alternate Method:
Refer Shortcut 1. 33. Since it’s a balanced Wheatstone’s bridge, the
circuit can be redrawn as
26. This is a balanced Wheatstone’s bridge circuit.

io
Hence potentials at B and D will be same and no I 10  2
current flows through the resistance 4R.
27. The bridge is balanced. 1.4 A

at
The balance condition after replacing 10 
resistor by 20  resistor will remain the same.
4  28 4  28 7 (1.4  I) 25  5
 Req. = 4 || 28  = = = 
V 12  2
4  28 32
lic
2
 12 I = 30 (1.4 – I)
 I= = = 3.4 A  12 I = 42  30 I  I = 1 A
R eq. 7
34. The equivalent circuits are as shown below
ub
P Q
28. Wheatstone’s network is balanced as  C C C
R S 2 2
 No current flows through galvanometer.
25  50 A B A B
 Reff. = A
P

25  50
2 2
25  50 50
= =  D B
75 3 D D
V 6
et

 I=   0.36A The circuit is a balanced Wheatstone’s bridge.


R 50 / 3 Hence effective resistance between A and B
29. The given circuit is a balanced Wheatstone’s = 4  || 4  = 2 
bridge circuit. Hence potential difference
rg

35. The given network is a balanced Wheatstone


between A and B is zero. bridge. Its equivalent resistance will be
30. As the bridge is balanced, 18
R= 
R AB = R BC 5
Ta

R AD R CD V V 5V
 i=  
R 18 / 5 18
44
 = 10 || 5
 4  5 || 5 36. As I is independent of R6, no current flows
  X
3  through R6. This implies that the junction of R1
8 50 / 15 and R2 is at the same potential as the junction of
 = R3 and R4. This must satisfy the condition
4  25 / 10
  X R1 R 3
3   , as in the Wheatstone’s bridge.
R2 R4
8 50 10 4
 =  
4  15 25 3 R1 2
  X 37. 1st case: = .…(i)
 3  X 3
4 4 14 R2 3
 +X=6X=6– =  2nd case: = .…(ii)
3 3 3 X 2
335

MHT-CET Triumph Physics (Hints)


Adding equations (i) and (ii), 1 5
 =
R1 3
R2 2 3 10  R Q
+ = +
X X 3 2
 10 + RQ = 15
R1  R 2 13
=  RQ = 5  and RP = 10/3 
X 6
Let l be the distance of null point from left. 40. For meterbridge,
l 13  lx 
= X = R 
100  l 6  lR 
 6 l = 1300 – 13 l  lX  since null point is obtained 
1300 =10   ….  
 19 l = 1300  l = = 68.4 cm from left.  lR   at the middle point of wire,lX  lR 
19
= 10 

ns
38. Let X be the smaller resistance in the metre
100  l 
bridge lX = 20 cm 41. X =  R
 l 
 lR = 100  20 = 80 cm
100  l 
As the bridge is balanced, Initially, 30 =    10
 l 

io
lX X
=  l = 25 cm
lR R
100  l 
20 Finally, 10 =    30
 = X  l 

at
80 R
 l = 75 cm
X 1
 = So, shift 75 cm  25 cm = 50 cm
R 4
 42. For first case, the balancing condition is
R = 4X
From second condition,
....(i)
lic 10  R1 50

X  15 40 R2 50

R 100  40  R2 = 10 + R1.
ub
X  15 40 For second case, the balancing condition is
 
R 60 R1 40

X  15 2 R 2 60
 
R 3 R1 2
   R1 = 20 
P

2R = 3X + 45 10  R1 3
3X  45
 R= ....(ii)
2 R1 l1 l1
43. In balancing condition,  
Equating (i) and (ii) we get, R 2 l2 100  l1
et

3X  45 X 20 1
= 4X    ....(i)
2 Y 80 4
 8X = 3X + 45 4X l
rg

and  ....(ii)
 5X = 45  X = 9  Y 100  l
4 l
RP 2  
39. 1st case:  4 100  l
RQ 3
Ta

 l = 50 cm
2
RP = RQ ....(i) 44. Balancing length is independent of the area of
3
nd
2 case: Resistance, instead of RQ is cross-section of the wire.
10R Q lP P 1
RQ||10 = = R 45.    3P = Q
10  R Q lQ Q 3
Now, R P R =1  RP = R  3P  Q = 0 ....(i)
P  40 3
 RP =
10R Q
....(ii) 
Q  40 5
10  R Q
 5P + 200 = 3Q + 120
From equations (i) and (ii),
 5P  3Q = 80 ....(ii)
2 10R Q
RQ = Solving equations (i) and (ii) we have,
3 10  R Q P = 20 , Q = 60 

336

Chapter 09: Current Electricity


46. G is a sensitive galvanometer and to protect it V E  R AB 
from damage of heavy currents, some resistance 55. =  
L L  R  R AB 
R is introduced.
VE  R AB 
 E1 =  l=  .l
V L  R  R AB 
L
47. = 10 V/m
L E1 20 1
 = 5=
 V = 10  L = 10  25  10–2 = 2.5 V E (20  20).10 4

I  E : E1 : : 4 : 1
48. Potential gradient =
A e
2 3 2
56. I=
10  10  10  10 9
(R  R h  r)
=
102  104 V 2 5
 = 
102

ns
L (15  5  0) 1
= 6
10  K = 0.5 V/m = 0.005 V/cm
= 108 V/m
E 4 1
57. I= = = A
I 0.1  107 Rr 30  30 15

io
49. Potential gradient  = = 10–2 V/m
A 106 1
 V=IR=  30 = 2 V
15
E 2
50. I=   0.2A 2 1

at
R r 82 K= = V/m
10 5
 V = IR = 0.2  8 = 1.6 V
V 1.6 e R
 Potential gradient =  = 0.4 V/m 58. K=
L 4
lic 10 3
(R  R h  r) L
2 3
E 2 2  = 
51. I=   A 102 (3  R h  0) 1
R  r 990  10 1000
 Rh = 57 
ub
2
 V = IR =  10
1000 e R
59. K= .
V 2 1 (R  R h  r) L
 Potential gradient =   = 0.01 V/m 3
L 100 2 0.2  10 2 R
 = 
P

2
10 (R  490  0) 1
E 5 5
52. I=    0.1A  R = 4.9 
R  r2 40  10 50
e
et

53. Resistance per unit length is 1 /m 60. V = I×R = R


(R  R h  r)
Balancing length = 2.9 m
2
Resistance across balancing length = 2.9   103 =  10
(10  R  1)
rg

e.m.f. = 1.45 V
 R = 19,989 
1.45
Current, I = = 0.5 A 61. P.D. across potentiometer wire = 2 V
2.9
V 2
Ta

Potential gradient = = V/cm


2 L 100
54. I=
R  10
Now, E =   l
V
2 20 L
 V = I RAB =  10 =
R  10 R  10 2
 E=  75
V 20 20 100
 = =
L (R  10)1 R  10 3
=2 = 1.5 V
4
E1 = l  
V

L 62. El
10  10 = 0.4 
–3 20  E 75
   =
 R  10  1.02 50
8 3
 R + 10 = 2 = 800  R = 790   E=  1.02 = 3  0.51 = 1.53 V
10 2

337

MHT-CET Triumph Physics (Hints)


63. Let EA, EB and EC be the e.m.f. of three cells A,  4I 
S I  
B and C respectively. S(I  Ig )
 
100 
For the given potentiometer,  G =
Ig  4I 
EA + EB + EC = kl1 = k  740 ….(i)  
 100 
EA + EB = kl2 = k  440 ….(ii) 96IS
EB + EC = kl3 = k  540 ….(iii) = = 24 S = 24  5 = 120 
4I
From eq. (i) and (ii), we get
EC = 300k 72. Potential drop across galvanometer = Potential
From equations (i) and (iii) we get, drop across the shunt
EA = 200k i.e., IgG = (I  Ig) S
Substituting value of EA into equation (ii) Ig
S = G
we get, I  Ig
EB = 240k

ns
I
 EA : EB : EC = 200k : 240k : 300k For Ig =
10
= 10 : 12 : 15 = 1 : 1.2 : 1.5 I / 10 G
 EA = 1 V, EB = 1.2 V, EC = 1.5 V S= G
 I  I / 10  9

io
64. Using,
Ig 1 S
E1  E 2 l1 73. = =
 I 34 S  3663
E1  E 2 l2

at
3663
1.5  1.1 260  S= = 111 
  33
1.5  1.1 l2
2.6 260 260 Ig G 15  103  5
   l2 =  0.4 = 40 cm 74. S= = = 0.0505 
0.4 l2 2.6
lic I  Ig 1.5  15  10 3

E1 l1  l2 58  29 87 3 IG S 2.5 2.5 25 1
65. = = = = 75. = = = = =
E2 l1  l2 58  29 29 1 I SG 2.5  25 27.5 275 11
ub
E1 l l (6  2) 2 76. Ig = 5.4  106 A,
66. = 1 2 = = Ig S
E2 l1  l2 (6  2) 1 =
I SG
l   120  1 SG  6  1  30 
  = 5.4  10 
P

67. r = R   1 = 5   1 = 5  = 2.5  I = Ig   
 l1   80  2  S   1 
= 5.4  106  31 = 1.67  104 A
l 
r = R   1  = 10  
75
68.  1
et

 60   S 
 l1  77. Ig =  I
SG 
= 10 
15 
 = 2.5  10  S 
 60  I =  I
rg

100 SG 
 l1  l2  1  10 
69. r=    R  = 
 l2  10  10  G 
55  50  
r = 
Ta

 10 + G = 100
  10 = 1 
 50   G = 90 
60 78. Shunt resistances,
70. n= =2
30 Ig G 10  99
S= = = 11 
G G (I  Ig ) (100  10)
Now, S = =
n 1 2 1
G
79. Ig = 10  106 A
 S =  S = G = 12  Ig
1 Using, S = G
I  Ig
4
71. Ig = I 10  106  1000
100 =  102 
1  10  106
Ig G
Using, S = we get, = 0.01 
I  Ig
 S = 0.01  is parallel
338

Chapter 09: Current Electricity


80. (Reff = 30 || 30 = 15  = G) 33
 R + 50 =  103
IgG Ig (15) 2
 S= = = 15  ....( I = 2Ig)
I  Ig 2Ig  Ig  R + 50 = 4.5  103
 R = 4500 – 50 = 4450 
81. Fraction of current passing through the
Ig 88. A voltmeter always has high resistance as R is in
S
galvanometer is = series.
I SG
To increase the range of ammeter i.e., to
10 10 1 increase I, its resistance must decrease.
= = =
10  90 100 10  High range  low resistance.
Fraction of current passing through shunt is
Ig Ig 1 9 I
=1– =1– = 89. Ig = 10% of I =
I I 10 10 10

ns
G 90
82. G = 6000  3 = 18000  = 18 k  S= = = 10  in parallel
(n  1) (10  1)
V 3 1
Using, Ig = = =  103 A, SG S  500
G 18  103 6 90. Reff. =  25 =
V SG S  500

io
 Value of series resistance, R =  G 500
Ig  500 S = 25 S + 12500  S = 
19
12
=  18  103

at
1 3  GS
  10  91. Resistance of shunted ammeter =
6  G S
= 72  103  18  103 Also,
I
=1+
G
= 54  103 = 5.4  104  Ig S

83. For the actual measurement of potential


lic GS I .G 0.05 120
 = g = = 0.6 
difference, it is necessary that the current G S I 10
between two points of the conductor should
92. 10 div = 1 mA and 2 div = 1 mV
ub
remain the same after putting the measuring
 150 div = 15 mA and 150 div = 75 mV
device across two points. This is the case when
V 75
resistance of device is very high (i.e., infinite).  Ro = G = = =5
I 15
V 10 S Ig
84. X= –G= – 40 = 39960  
P

Ig 0.25  103
=
SG I
S 15  103 5  103
85. Ig = 5  103 A  = =
S5 6 2
V
et

Using, X =  G  2S = 5  10–3 S + 25  10–3


Ig
 S ≈ 0.0125 
20
 3960 = G 93. With increase in temperature, the value of
5  103
rg

unknown resistance will increase.


 G = 4000  3960 = 40  For balanced Wheatstone bridge condition,
V 3 R l1
86. Ig = = = 15 mA 
X l2
Ta

R 200
In (A), 10 mA < 15 mA  I < Ig l1
To take null point at same point or to remain
 I  10 mA l2
87. The current through the galvanometer R
unchanged, should also remain unchanged.
3 X
= Therefore, if X is increasing R should also
2950  50
–3 increase.
= 10 A
 To reduce the deflection from 30 divisions to 20 94. V1 = 80 volt,
divisions, the current required R1 = 200  80 = 16000  = 16 k,
20 2 V1 80
=  10–3 =  10–3 A I1 = = = 5  103 A
30 3 R1 16000
3 2 Current in series connection of voltmeter
 The required resistance, R = =  10–3
R  50 3 remains constant.
339

MHT-CET Triumph Physics (Hints)

 I2 = 5  103 A, R2 = 32  103 , 7R
Req =
V2 = I2R2 = 5  103  32  103 = 160 V 5
 Line voltage = V1 + V2 = 80 + 160 = 240 V 3b 5b
Entering current, (a + b) = b I
95. Since in a given loop the current coming out 2 2
from the positive terminal is equal to the current b I
Current in common side, (a – b) =  .
entering the negative terminal, the current in the 2 5
respective loop will remain confined to the loop. 99. iAD = iDB + iDC
 current through 2  resistor is zero. Let potential at D be V
96. Voltmeter is connected in series in the circuit (7  V) (V  0) (V  1)
 
hence there is no Potential difference across 10 20 30
voltmeter. On solving the above equation, we get VD  4 V
Hence option (A) is correct.

ns
97. The circuit for the dashed lines can be drawn as,
Currents through the sections DB and DC are,
1 1 1 1 1 74
A B = 0.3 A,
10
 Req = 5  1 = 5  4

io
= 0.2 A,
The circuit obtained by adding dashed lines can 20
be drawn as, 4 1
= 0.1 A
1 30

at
Hence option (B) is correct.
1 1 1 1 1 Power = I2R
A B
 Total power drawn = (0.3)2  10 + (0.2)2  20

1
lic + (0.1)2  30
= 0.9 + 0.8 + 0.3 = 2.00 W
Considering Wheatstone’s network, R eq for this Hence option (D) is correct.
combination after simplifying the circuit,
ub
R eq = 3  Competitive Thinking
 Difference in the final and initial values of Req is 2. Given circuit can also be drawn as,
2 . 2
A
P

98. Let the currents through various branches be as 2A


shown
R D (a+b)
C b
3V V A  VB
et

R R
(a  b) a
B F
R 1
B
rg

R R
a
A By Kirchhoff’s voltage law,
(a+b) b R
VA  (2  2)  (3)  (2  1)  VB = 0
 VA  4  3  2  VB = 0
Ta

E  VA  VB = + 9V
Applying Kirchhoff’s voltage law in loop 3. B
ABCDEA and loop ABFA we get, 4V
E  aR  2Rb = 0 ….(i)
 aR  (a – b)R + 2Rb = 0 ….(ii)
I2 4
2aR = 3Rb  2a = 3b
A I1 O
3b
E=R  2Rb = (a + b) Req 8V
2 I3
2
7Rb  3b  2
   b  R eq
2  2 
7Rb 5b 2V
  R eq C
2 2 Let potential at O be V0.

340

Chapter 09: Current Electricity


According to Kirchhoff’s current law, Now, p.d. between points A and C is,
I1 + I2+ I3 = 0 2
VA –  20 = VC
8  V0 4  V0 2  V0 50
  =0
2 4 2 2
i.e., VA = VC +  20
 2 (8 – V0) + 4  V0 + 2(2 – V0) = 0 50
 16  2V0 + 4  V0 + 4 – 2V0 = 0 p.d. between points A and D is,
 5 V0 = 24 2
VA –  30 = VD
24 50
 V0 = = 4.8 V
5 2
3 i.e., VA = VD +  30
4. + – 50
A B
2 V, 1  2 2
 VC +  20 = VD +  30
0.1 A 0.1 A 50 50

ns
2
+ –  VC – VD = (30  20) = 0.4 V
D C 50
E, 1  5
7. 1 1
Applying Kirchhoff’s voltage law in loop

io
ABCDA,
1
–2 – 0.1(1) – 0.1 (3) – 0.1 (5) – 0.1 (1) + E = 0 1.2 V 3 1
 –2 – 1 + E = 0 1

at
 E=3V 2 1
5. 100  B
A C Req = (1 + 2)  = 3
I2 .....( Considering shortest path)
I1 I3
50 
lic
25  I=
V
=
1.2
= 0.4 A
10 V R eq 3
5V
F D 8. Voltage across 10  = voltage across 40 
ub
E  I1 (10) = I2 (40)
Considering direction of I1 and I2, 2.5  10
I1 + I3 = I2  I2 = = 0.625 A
40
Applying Kirchhoff’s voltage law in loop 50 50  25
BCDEB  R= = =8
P

I1  I 2 2.5  0.625
25I3 + 5 + 50I2 = 0
25I3 + 5 + 50(I1 + I3) = 0 9. 4V 2V
6V
 50I1 + 75I3 = 5 ….(i) H F D B
et

Applying Kirchhoff’s voltage law in loop 0V


ABCDEFA, 2V 2V 2V
100I1 + 25I3 =  10 ….(ii) R1 = 1  R2 = 1  R3 = 1 
From equation (i) and (ii),
rg

175I3 = 20 2V 2V 2V
4 0V
I3 = A G E C A
35 6V 4V 2V
Ta

6. From figure, resistance in branch ACB is, Consider potential at points A and B be zero.
RACB = 20 + 30 = 50  Hence, potential at points C and D will be 2 V.
2 Similarly, potential at E and F is 4 V.
 Current in ACB, IACB = A This implies, potential drop across each resistor
50
R1, R2 and R3 is zero.
2
Similarly, current in ADB, IADB = A  current through each resistor is zero.
50
20  C 30  10. Current from D to C = 1 A
A  VD  VC = 2  1 = 2 V
B
VA = 0  VC = 1 V,
V
 VD  VC = 2
 VD  1 = 2  VD = 3 V
30  D 20   VD  VB = 2
 3  VB = 2  VB = 1 V
2V
341

MHT-CET Triumph Physics (Hints)


11. 12 V 1 16. 10 
P S

13 V 2 10  10  10 
10  AB
Q
10 
10 
Eeq= E1r2  E 2 r1  12  2   13  1  37 V
r1  r2 1 2 3 This network can be redrawn in the bridge form
r r 1  2 2 as, S
Also, req = 1 2 =  
r1  r2 1 2 3
Current in the circuit will be, 10 
10 

ns
37 10 
I= E eq 3 37
  A
R  req 2 32 P
10  A B
3

io
The voltage across the load, 10 
10 
37 10 
V = IR =  10 = 11.56 V
32

at
12. For parallel combination of batteries, Q
E1 E E3 AS AQ
 2  In this case,  Hence, bridge is balanced
2R1 R 2 2R1 SB QB
Eeq =
1 1 1

2R1 R 2

2R1
lic and no current will flow through SPQ branch
and thus, is neglected.
2 4 4 This modifies circuit into,
 
= 2 2 2 S
1 1 1
ub
 
2 2 2 10 
10 
5 2
=
3
A B
10
 Eeq = = 3.3 V
P

3 10  10 
13. 1.5 R
R O
et

X Y Q

20  in series
3R 10  S 10 
rg

1.5R  3R
ROY = =R
(1.5  3)R A B
RXO = ROY = R 10  10 
Ta

 VXO = VOY Q 20  in series


 VA = VB = VC 1
1 1
14. When I = 0 A, V = E = 3 V RAB = [20  || 20 ] =    = 10 
 20 20 
E 3
When V = 0, r =  = 0.5 
I 6 17. No current flows through the 6  resistor as the
Wheatstone network is balanced.
15. For balancing the bridge B
P R
 15 
Q S 3
I1
S1S2 A C
 S= ….( S1, S2 are in parallel)
S1  S2 I = 2.1 A
I2
P R  S1  S2  4
 = 20 
Q S1S2
D
342

Chapter 09: Current Electricity


In parallel combination voltage remains same. X lx
20. 
15  3  R lR
20 lx
I1 
30 lR
I = 2.1 A I2 I
lx 40
 
20  4 lR 60
 I1 × (15 + 3) = I2 × (20 + 4) as for metrebridge, lx + lR = 100 cm
I1 × 18 = I2 × 24  lx = 40 cm
 3I1 = 4I2 After reducing resistance,
X lx
 I2 = 3 I1 
4 R 100  lx

ns
According to Kirchhoff’s current law, 10 lx
I1 + I2 = 2.1  
30 100  lx
I1 + 3 I1 = 2.1  lx = 25 cm
4
The distance through which balance point is

io
7
I1 = 2.1 shifted lx  lx = 40 – 25 = 15 cm to the left
4
I1 = 2.1  4 = 1.2 A 21. Let lX be balancing length obtained in front of

at
7 smaller resistance.
3  lX = 40 cm, lR = 60 cm
18.
r r When the bridge is balanced,
1 X lX 40 2
1
lic
r 2   
R lR 60 3
….(i)

 r r when 30  is connected in series with X,


3 effective resistance becomes (X + 30) 
4
Also, length shifts by 20 cm
ub
2
4 lX + 30 = 40 + 20 = 60 cm
r
The centre resistor will be neglected X  30 60 3
3   
R 40 2
2(X  30)
P

r r
R= ….(ii)
r r 3
1 2
From equations (i) and (ii),
r r X 2
r r
et

 (X  30)
=
3
4 2
r 3
3X 2
 
rg

r
2(X  30) 3
1 1 1 1
 = + +  5X = 120
Rp 2r 2r r
 X = 24 
r
Ta

 Rp = 5 R
2 22. Initially, = ….(i)
l1 100  l1
P R
19.  5 R/2
Q S Finally, = ….(ii)
P
1.6 l1   1.6 l1 
100
Since P and Q is constant will be constant R R
Q  =
R 1.6 100  l1  2 100  1.6 l1 
 = constant
S  160  1.6 l1 = 200  3.2 l1
R 5 7  1.6 l1 = 40
 
S S S3  l1 = 25
 5(S + 3) = 7S From equation (i),
 15 = 2S 5 R
  R = 15 
 S = 7.5  25 75

343

MHT-CET Triumph Physics (Hints)


23. l1 = 52 + 1 = 53 cm, l2 = 48 + 2 = 50 cm 26. Initially, when the bridge is balanced against
As the bridge is balanced, lengths l1 and l2, the circuit can be redrawn as
follows,
l1 X 53 X
   P
l2 R 50 10 Q

 X = 10.6  G
24. Initially, l1 l2
R 1 l1 60 3
   ….(i)
R 2 l2 40 2
When, wire is stretched by 20 % i.e., becomes
E
1.2 L
When galvanometer and cells are interchanged,

ns
Using shortcut 3, the balance condition remains unchanged.
Resistance will increase to 1.44R2
Hence, after stretching wire, P Q

io
R 1 l
 E G
R 2 100  l
But R1 = R and R 2 = 1.44 R2 l1 l2

at
R1 l
 = l1 2
1.44R 2 100  l 27. Unknown resistance, X = R =6
l2 3
From (i),
3 l
lic  X=4
Resistance of bridge wire RW = 0.1 /cm = 10 

1.44  2 100  l
X =4 R =6
 300  3l = 2.88 l
ub
300
 l=  51 cm G
5.88
2 3
25. Let balancing length be l , RW = 10 
P

R1 = l ()
 ....(i)
R2 100 l E = 5V K
Equivalent resistance,
If R1 and R2 are interchanged balancing length
et

Req = (X + R) || RW = (10 ) || (10 )


becomes, ( l  10)
Req = 5 
 R2 = l 10 = l 10 ....(ii) Current drawn from the battery is,
100  (l 10) 110  l
rg

R1 E 5
I= =
Req 5
From equations (i) and (ii),
 I=1A
l = 110  l
Ta

100 l l 10 29. l P


2 2 A B
 l  10 l = (110  100) + ( l  210 l )
L
 200 l = 110  100
 l = 55 cm
Substituting in equation (i), we get, + 
()
R1 55 11 ....(iii) K
  E
R 2 45 9 For a potentiometer wire AB of length L,
When R1 and R2 are connected in series, V 
VAP =  AB  l
R1 + R2 = 1000  . ....(iv)  L 
On solving equations (iii) and (iv), we get, VAP l
=
 VAB L
R1 = 550  and R2 = 450 
344

Chapter 09: Current Electricity

 VAP  34. Current drawn when resistors are in series,


The ratio   would remain constant if the E E
 VAB  Is = I =  ....(i)
length of the wire is increased. nR  R (n  1)R
 Ll Current drawn when resistors are in parallel,
Hence balancing length ‘l’ will increase if Ip = 10 I = E ....(ii)
length of potentiometer wire is increased. R
R
30. EMF of cell, n
E = kl Substituting for I using equation (i) in equation
E (ii),
E =  l
l 10E E

E=
E
 2.5 ….(i)
 n  1 R 1  1  R
 
10  n

ns
After increasing the length by 1 m,
E  n + 1 = 10 1 1 
E = x n  
11
10
Substituting for E from equation (i)  n– =9

io
E E n
 2.5 =  x  n2 – 9n – 10 = 0
10 11
2.5  11  n2 – 10n + n – 10 = 0

at
 x= = 2.75 m
10  n (n – 10) + 1 (n – 10) = 0
l  (n + 1) (n – 10) = 0
31. R= Neglecting negative value of n,
A


R  40  108
  = 5  10–2 /m
lic n = 10
l A 8  106
35. E1  L1 and E1  L2
Potential gradient is given by,
E1 L1 1.25 30
V IR    =
ub
 = 0.2  5  10–2 = 10–2 V/m E 2 L2 E2 40
l l
5
32. P.D. across the wire  E2 =  1.67 V
= Potential gradient  length 3
V0 = 1mV/cm × 400 cm = 0.4 V
P

36. While assisting net E.M.F = E1 + E2


0.4
Current in the wire, I = = 0.05 A opposing net E.M.F =  E1  E2
8
for potentiometer E  l
V  V0 2  0.4
= 32 
et

R= = 1   2 50 5
I 0.05   
E1  E 2 10 1
33. (E, r1) (E, r2) E 1 5 1 6 3
+ – + –    
rg

E 2 5 1 4 2
I V1 37. E0 r1
Ta

R A l
P r
2E B
Current in the circuit: I =
R  r1  r2
Terminal p.d across 1st cell is V1 = E – Ir1 E
Given: V1 = 0 E0
 E – Ir1 = 0 Current in wire AB =
r1 + r
 2E 
E–   r1  0 i r  E0  r
 R  r1  r2  Potential gradient (K) = =  
L  r1 + r  L
2Er1
E= E = Kl
R  r1  r2
R + r1 + r2 = 2r1  E0  r
 E=   l
 R = r1 – r2  r1 + r  L

345

MHT-CET Triumph Physics (Hints)

l  49. With ideal ammeter, V = IR


38. r =  1  1 R 6
 l2  R= =2
3
 3  3A R
=  1 9.5
 2.85  A
0.15
=  9.5 = 0.5 
2.85

39. Internal resistance of a cell, V


 l1  l2   240  120  6V
r = R   2  =2 but the ammeter has resistance of its own hence,
 2 l  120 
external resistance has to be less than 2 .

ns
40. Internal resistance, 50. I
l  G
r =  1 1 R S
l
 2 
 52  12  5

io
=  1  5 = = 1.5 
 40  40 S
 GS 
42. Reason is correct as, to increase range additional Now, G =    S
 G S

at
shunt is connected across it. But, assertion is
GS
incorrect as shunt is added in parallel.  G  S
G S
l G2
43. R=
A
lic  S =
G S
 Shunt wire must have low resistance. IG S
For low resistance, wire should be short (l ≪) 51. 
I SG
and thick (A≫, i.e., radius of wire ≫) .
ub
1 3

 G  4 3G
44. Is =  I  3 + G = 12
 S  G
 G=9
P

 8  8 If additional shunt of 2  is connected then total


=  1= = 0.8 A
 2  8  10 shunt resistance becomes,
1 1 1
I 100  
et

45. n=   500 S 2 3
Ig 0.2 23 6
 S =   = 1.2 
G G 23 5
 R= 
n 500 I S 1.2 1.2 1
rg

Now, G    
I S  G 1.2  9 10.2 8.5
Ig G 5  103  102 0.5
46. S=  = 250mV
I  Ig 1  5  10 3
1  5  103 52. For full scale deflection, Ig = ampere
G
Ta

5 5 Value of shunt required for converting it into


= = 
10  0.05 9.95 ammeter of range 250 milliampere is,
G
Ig G5  103  99 S=
47. S=  =1  I 
I  Ig (0.5  5  103 )   1
I
 g 
G IgG 250 mV
48. To convert an ammeter to range nI, S   S=  1
n 1 I  Ig 250 mA
Here, I = 1 mA = 103 A
V
nI = 10 A 55. Using, Rs =  G we get,
Ig
 n = 104
V
100  for 1st case, 100 = R ….(i) and
 S = 102 = 0.01  Ig
104
346

Chapter 09: Current Electricity


2V From equation (i) and (ii),
for 2nd case, 1000 = R ….(ii)
Ig R2
= 500
R1
By subtracting equation (i) from (ii) we get,
V 58. V = Ig ( G + R)
= 900
Ig V 3
i.e., Ig = =
 R = 900   G  R  50  2950
3 1
56. Using, Rs =
V
 G we get, = = = 10–3 A
Ig 3000 1000
Now, 30 divisions represent 10–3 A
st V Let 20 divisions represent I A
for 1 case, 50 = G ….(i) and
Ig 2
 I=  10–3 A
2V

ns
3
for 2nd case, 500 = G ….(ii)
Ig V 3
Also, I = =
By subtracting equation (i) from (ii) we get,  eq 
R  r 3000 r
V 2 3
= 450    10–3 =

io
Ig 3 3000  r
Substituting this value in equation (i),  r = 1500 
 G = 450 – 50 = 400 
59.

at
57. Case I:
Ig
G 50  G R
I
lic 10 mA
I1 R1
V
V = Ig (G + R)
ub
V  100 = 10  10–3  (50 + R)
I = Ig + I1
I  50 + R = 10000
I= + I1  R = 9950 
51
50I
P

 I1 = V
51 60. R= G
Ig
But P.D across R1 = P.D. across G
 I1R1 = IgG 3 3  104
R=  50   50
et

50 I 5  104 5
 IR 1  G = 6000  50 = 5950 
51 51
 G = 50 R1 ….(i) 61. Given: Ig = 5  103 A and G = 15 
rg

Case II: Let series resistance be R.


 V = Ig (R + G)
I R2  10 = 5  103 (R + 15)
G
 R = 2000  15 = 1985 =1.985  103 .
Ta

62. As the galvanometer is to be converted into


voltmeter, the resistance should be connected in
series.
V V 20
V = VG + V2 R  G =  50 = 3950 
V Ig 5 103
V= + V2
11
63. As the voltmeter has full scale deflection of 6 V
10
 V2 = V and is graded as 3000 /V, hence total
11
resistance of voltmeter is G = 6  3000 
VG V2
But I =   G = 18000 
G R2 The full scale deflection current of voltmeter is
R2 6 1
 G= ….(ii)  Ig = = A
10 18000 3000

347

MHT-CET Triumph Physics (Hints)


The resistance in series that must be connected From figure (b), null point is obtained when,
for 12 V full scale deflection is R1 R 2

V 12 R3 R 4
RS =  G =  18000
Ig 1
R1 R 3
3000 i.e.,  ….(ii)
R2 R4
 RS = 36000  18000 = 18000 
 From equations (i) and (ii), we can say that null
64. Assume voltmeter and ammeters to be ideal. As, point is not disturbed when galvanometer and
ideal voltmeter provides infinite resistance, the cell are interchanged.
current through the branch containing voltmeter
will be zero. Hence, connecting a 10 Ω resistor 68. A
in series with voltmeter as shown in circuit 2
will not change the readings of the voltmeter as

ns
well as ammeter.
230 V (100V, 500 W)
R left gap l
65. =
R right gap 100 l

io
Resistance increases with the increase in B
R I
temperature. Hence, the new balancing length
Power P = IV

at
will be greater than l.
P 500
 I= = = 5A
66. S1 is open and S2 is closed V 100
12 12 By Kirchhoff’s law,
So, I = =
(6  4) 10
lic 
230 V = IR + VAB
230 V = IR + 100 V
 I = 1.2 A
 IR = 130 V
67. B 130
 R=  = 26 
ub
R1 R3 5

A C 69. F = qE
G
F F V
 E=  
R2
P

R4 q q L
D FL 2.4  1019  6
 V= = V=9V
q 1.6  1019
+ –
et

E  e.m.f. of cell = V = 9 V
Figure (a) 70. Equivalent circuit is given by
In the Wheatstone bridge shown in figure (a), 100  400 
rg

null point is obtained when,


R1 R 3
 ….(i)
R2 R4 200  300 
Ta

When the positions of galvanometer and cell (E)


are interchanged, we get circuit shown in figure
(b). 6V
B
R1 R3
B Capacitors behave as infinite resistance in
A C R1 R3 steady state
Voltage
A Isteady =
R2 R4  G C Resistance
D Equivalent 6
R2 R4 =
circuit (200  400)
G D 6 1
Figure (b) = = A = 10 mA
600 100

348

Chapter 09: Current Electricity


3 2
71. I=  0.5103 A  0.5  103 =
6  103  + r
R 
VAD = IR
2
= 0.5  10–3  3  103  R + r =
0.5 × 103
= 1.5 V
Here, capacitors 1 F and 2 F are in series.  R = (4  103) – 30
2  R = 3970 
 Ceq= F
3 74.
Q = CeqVAD RS = 20 
2  1.5 40.8 
=
3 RA = 480 
= 1 C I

ns
Applying Kirchhoff’s voltage law from B to C
via D, 30 V
3 1  10 6 3
VB  0.5  10  2  10 + = VC 480  20
2  10 6 Req = 40.8 +

io
1 500
VB – VC = 1  = 0.5 V
2 = 40.8 + 19.2
72. = 60 

at
50 
G 30
I= = 0.5 A
60
Ig
So reading of ammeter is 0.5 A
lic
75. Applying Kirchhoff’s voltage law to loop
8V 3950  containing VCC, RL and Transistor -
the current in the circuit for which galvanometer + VCC  ICRL  VCE = 0
ub
shows full scale deflection of 30 divisions is  VCC = VCE + ICRL
V 8
Ig = = = 2 mA Applying Kirchhoff’s voltage law to loop
R 3950  50
containing VBB, RB and Transistor-
For deflection to become 15 divisions, the
+ VBB  IBRB  VBE = 0  VBB = VBE + IBRB
P

current through galvanometer must be halved.


Ig
 Ig = = 1 mA 76. We know current through the capacitor will be
2
50  zero at steady state and ammeter is ideal.
et

V
but, Ig = G
R 5 F
8 Ig 2
=
R s  50 2
rg

= 1 mA 2
8 8V Rs 2 
 = 103 3V
R s  50
Ta

3V A
 Rs + 50 = 8  103 A

 Rs = 7950 
73. 1
I I
I I
3V
A
I I I
1970  2V
V 3
V 2 2 I= 
I= = = = 1 mA R 1
 R + r  1970 + 30  2000
I=3A
 for 10 divisions of deflection, I = 0.5 mA

349

MHT-CET Triumph Physics (Hints)

Hints to Evaluation Test

E e R P R
1. = 5. For balanced Wheatstone bridge, 
l (R  R h  r) L Q S
5 5 12 x6
 0.4 =  l  
(5  45  0) 10 1 / 2  1 / 2 
 l=8m x=6
X 2
2. The given circuit is a balanced Wheatstone’s 6. =
network as shown in figure (ii). Hence, points 100  X 3
Q and S are at the same potential  3X = 200 – 2X

ns
 VQ – VS = 0 V  5X = 200  X = 40 cm
Q 7. Metrebridge is balanced,
4 12  R AC 20
 = =

io
R 80 BC 80
P, T G  R = 20 
8 24  8. RAB = 2  10 = 20 

at
S 3 3 1
 I= = =
10  20 30 10
+ 
1
 V = I RAB =  20 = 2 V
6V
Figure (ii)
lic V 2
10

 = = 0.2 V/m
L 10
3. Applying Kirchhoff’s junction rule to point A,
(see figure)  l1  l2 
ub
 I1 – I2 – I3 = 0 9. r = R
 l2 
 I1 + I2 + I3 = 0 .…(i)
 25 
If VA is the potential at A, by applying Ohm’s  R=   2 = 0.5 
law to R1, R2 and R3 then we get,  100 
P

VA – V1 = I1 R1, G 50
VA – V2 = I2 R2 and X 10. S = 12  = ,n= =5
n 1 10
VA – V3 = I3 R3 I1 G G G
VA  V1  S= = =
et

 I1 = , R1 n 1 5 1 4
R1  G = 4 S = 4  12 = 48 
V  V2 A
I2 = A , R3 11. I = 40 mA
R2 R2
rg

I2 I3 = 40  103 A
VA  V3 E
I3 = Z Using, I =
R3 Y
R net  r
Substituting for I1, I2 and I3 in equation (i) we
Ta

3
get, 40  103 =
100 R V
1 22
1 1   V1 V2 V3  100  R V
VA         0
 R1 R 2 R 3   R1 R 2 R 3  100 R V 3
4+   75
 V1 V2 V3  1 1 1 
1
100  R V 0.04
 VA =        
 R1 R 2 R 3   R1 R 2 R 3   RV  245 
12. i2 4
4. This is a balanced Wheatstone bridge. Hence
no current will flow from the diagonal i1
resistance 10 .
10  10   10  10  4 6
 Equivalent resistance =
10  10   10  10  i i1
= 10  4V 2V

350

Chapter 09: Current Electricity


Potential difference across upper 4  18. 4.3 V 10 
resistance is zero
 current is zero  i2 = 0
Other two resistors are in series combination. 50 2
Hence current is same. A
42
= = 0.2 A s
46 V
 i = i1 = 0.2 A, i2 = 0 200 
S 52  200
13. Ig = .I Req = 10 +
SG 252
S 2520  10400
 2= 5 =
S  12 252

ns
 S = 8  in parallel = 51.269 
Ig S 4 4.3
14. = =  I= = 0.08 A
I SG 40 51.269

io
Ig 19. 1.5 V = k.l1 = k(76.3) ….(i)
  100 = 10 %
I E – ir = i(9.5 ) = kl2
V E 1.5
15. X= –G  i= 

at
Ig 9.5  r 9.5  r
(1.5)
V (9.5)  kl2 ….(ii)
 0= – 100 9.5  r
3  103
Dividing (ii) by (i), we get,
 V = 100  3  10–3 = 0.3 V
lic 9.5 l
 2
64.8

9.5  r 76.3

16. E r1 E r2 9.5  r l1 76.3 9.5 64.8
r  76.3 
   1
ub
I I 9.5  64.8 
 76.3 
X  r=   1 (9.5)
 64.8 
From the figure, = 1.7 
P

EE 2E
I=  ….(i)
r1  r2  X r1  r2  X
P.D. across first cell, V1 = E  Ir1
et

2E
=E  r1
r1  r2  X
Given that, V1 = 0
2 E r1
rg

 E=
r1  r2  X
 X + r1 + r2 = 2r1 or X = r1  r2
Ta

17. R
x y
x y

V
Figure (i) V
Figure (ii)
Equivalent resistance decreases. Hence current
will increase. As a result, ammeter reading
will increase.
 Vx + Vy = V
Due to the change, Vx increases
 voltmeter reading will decrease.
351
10 Magnetic Fields due to Electric Current

Hints

qB
20. Cyclotron frequency, f =

ns
Classical Thinking 2m
where, q = charge of proton
7.

  1.6  1019  1.4 49
F  q v B
   f= =  107 Hz
 22 27  22
 2   1.6  10 

io
Electron is a negatively charged particle,  7 

therefore force F will be acting in negative
Z-direction. mv p 2mK 1 2mV
21. R=   

at
qB qB qB B q
8. F = qvB sin 
where, K = K.E. of the charged particle.
= 200  106  2  105  5  105  sin 30
m
F = 103 N R
lic q
9. F = qvB sin  2 2
m1  R1   q1   R   q 
For  = 90 and v = 103c,  =   =    
m 2  R 2   q 2   R / 2   4q 
F  10 2  3
= 1.732
ub
q= m1
vB 10  3  108  2  10 5
3
 =1
m2
3 3
=  102
3 2
26. F=BIl
= 5  103 C
P

F mg 7.5  105  10
B= = =
10. F = qvB sin  Il Il 4  10 1
F B = 1.875  103 T
B=
et

qvsin 
27. F = BIl sin 
F
Bmin = ….(when  = 90) 7.5 = 2  5  1.5 sin 
qv
 = 30
rg

10 10
 Bmin = 12
= 103 tesla in ẑ - direction 28.
10  105 A A
4 4 2
 10 A
11. 1 G (gauss) = 10 T (tesla) = 10 Wb/m 10 A B
Ta


B
14. For motion of a charged particle in a magnetic 3 cm  
mv F
field, we have R = i.e., R v
qB
B 4 cm C B C
18. Particles are entering perpendicularly. Hence,
they will describe circular path. Since their Force on the conductor ABC = Force on the
masses are different, they will describe paths of conductor AC
different radii. F = Il B sin 
mv =IlB ….(  = 90)
19. R=
eB  F = 10  (5  10–2)  5 = 2.5 N
Now, v  2v
34.   NBIA  100  0.5  1  400  10 4  2 N-m
 R  2 R = 2  2 cm = 4 cm
352
 Chapter 10: Magnetic Fields due to Electric
Current
35. Field is radial (plane of coil parallel to magnetic 0I
53. B=
field) 2 R
  = NIAB 4  107  10
= = 5  105 N/A m
= 100  100  10–6  (5  2  104)  0.1 2  4  102
= 106 N m
0 I
54. B= 
36.  = NIAB sin 2 r
= 20  12  (101)2  0.8  sin 30 0 
 5  105 = 
= 0.96 N m 2 r
0  
37.   NBIA  100  0.2  2   0.008   0.32N m  r=
5  105  2
Direction is given by Fleming's left hand rule.  r = 1040 metre

ns
 5 0 I
38. B= = =2T 55. B=
NIA 100  5  50 10 4 2R
1
K 5  107  45  B
39. I= = R

io
NAB 200  0.02  0.08  0.2
B1 R
 I = 3.5  104 A = 2
B2 R1
40. B = 80 gauss = 80  104 tesla 10 8 12

at
For equilibrium of coil,  =
B2 4
NBIA = K  B2 = 3.33  109 T
NBIA
 K =

4 3
lic4
56. In the figure, magnetic field at mid-point M is
given by,
40  80  10  0.2  10  5  10 P Q
= Bnet  BQ  BP
20
0 2
= 1.6  109 Nm/degree   (IQ  IP ) 2.5 A 5A
ub
4 r M
41. m = NIA, thus independent of magnetic field of  2
 0 (5  2.5) 2.5 m
induction. 4 2.5

42. m = NIA = NI(r2)  m  r2 = 0 5m
P

2
43. m = NIA = 5  1  (4  102)2
58. Since currents are flowing in opposite direction.
m = 8  103 A m2
Hence force of attraction does not exist.
et

44. d = 9 cm = 9  102 m  0 2I1I 2  0 I 2


 A = r2 =  (4.5  102)2 59. Force per unit length    
4 d 2 b
 m = NIA = 30  1    (4.5  102)2
 m = 19.08  102 Am2 60. The force per unit length is,
rg

F  0 2I 2
47. W = mB cos1  mB cos2  
L 4 R
= mB (cos1  cos2) If R is increased to 2R and I is reduced to I/2,
Ta

Here 1 = 0, 2= 90 the force per unit length becomes,
 W = mB(1  cos 2) = 40  0.25 (1  cos 90) F 0 2(I/2) 2
= 10 J  
L 4 2R
0 Idl sin   0 2I 2 1 F
51. dB = × =   
4 r2 4  R 8 8L
F
0 I  F =
52. B= 8
2r
r  0 I1I2 L
New distance = 61. F=
2 2d
 0 2I F 4  107  2  4
 New magnetic field = = 2B =
4  r  L 2  101
 
 2 = 1.6  105 N/m

353

MHT-CET Triumph Physics (Hints)

F  0 2I1I2 74. B = 0.4  104 T = 4  105 T


62. =
L 4 d 0nI
Using B = we get,
F1  0 2I 2 2r
= (Attraction)
L 4 x 2Br 2  4  105  200  103
n= =
F2  0 2I  2I  2I 2 0I 4  3.14  107  0.25
= = 0 (Repulsion)
L 4 2x 4 x  n = 50.9  51
Thus F1 = F2
75. r1: r2 = 1: 2 and B1 : B2 = 1 : 3
63. Force on wire Q due to wire P is 0 2nI
B=   I  Br
2  30  10 4 r
FP = 107   0.1
0.1 I1 Br 11 1
 = 11 = =
= 6  105 N (Towards left) I2 B2 r2 3 2 6

ns
Force on wire Q due to wire R is
2  20  10 76. Magnetic field on the axis of circular current
FR = 107   0.1 0 2nIa 2
0.02 B= × 2 2 3/ 2
= 20  105 N (Towards right) 4 (a  x )

io
Fnet = FR – FP = 14  105 N = 1.4  104 N B
na 2
(a 2  x 2 )3/ 2
F  2I I  2I 2
64. = 0  12 = 0 ( I1 = I2 = I)
l 4 d 4 d

at
0 nIR 2
2 78. B =
2I 2(z 2  R 2 )3/ 2
 2  107 = 107 
1 4  107  50  1  (5  102 ) 2
I= 1A =
2[(0.05) 2  (0.2) 2 ]3/ 2
68. Element ‘dl’ and radius are perpendicular
lic   105  25  104
=
0I 4  107  107 [(25  400)  104 ]3/ 2
69. B= =  0.5 =  B = 9  106 Wb/m2
2r 2r r
ub
But 22  102 = 2r 0 IR 2 1
2 79. Bz   Bz  3
1110 2r 3 r
 r=
π
80. Magnetic moment, m = IA
  10 7  
P

 B=  9  106 Wb/m2 and magnetic field at the centre of a loop


11  10 2 0 I X(2r)
carrying current = = X or I =
0nI 7
4  10  100  0.1 2r 0
70. B= =
et

2r 2  5  102 X.2r
5 So, m =  r2
= 4  10 tesla 0

0nI 0 2nI 2nI 2 Xr 3


71. B= =  = 107   m=
rg

2r 4 r r 0
107  2  250  (20  103 )
= 0 I n R 2
 40 103  81. Since z >> R, B 
2z 3
Ta

= 7.85  105 T  7.9  105 T 4  107  5  250  (0.1) 2


=
 0 n1I1 n I 2  (5)3
72. B= + 0 22
2r1 2r2 = 2  108 T
0  5  0.20 5  0.30   
=
2  0.20

0.30 
= 50 84. From Ampere’s circuital law,  B. dl
c
= 0I

where I is the current in the closed path.


0 2I
73. BA = 
4 R 0 2I
85. As B  , for the same distance, field will
 2 (2I)  2I 4 r
 BB = 0  = 0
4 2R 4 R remain the same
BA 1 86. Magnetic field due to solenoid B = 0ni
 =
BB 1 Thus, it is independent of diameter.

354
 Chapter 10: Magnetic Fields due to Electric
Current
88. B = 0ni = 4  107  10  5 = 2  105 T
Critical Thinking
90. B = 0 n I  B does not depend upon radius.
2. Fm = qvB sin ,
1014
91. Mean radius = = 12 cm Since v = 0
2
 Fm = 0
The magnetic field B,
 N    
F  q (v B)  10 11 (108 ˆj 0.5 ˆi)
B = 0ni ....  n 
 3.
 2R 
 5  10 4 (ˆj  ˆi)
1500  100
= 4  107   10
2  12 = 5  104 N along k̂
= 25  103 T 4. Force on the charge in motion in magnetic field,

ns
92. F = qvB sin  = qvB sin 90

  
F  q v B , implying F is perpendicular
F = evB  

0I
B= to v .
2 R

io
 

ev 0 I 1.6  1019  107  4  107  50 Work done W = F v


 F= = = F v cos 
2R 2  5  102
F = 3.2  1016 N   

at
= F v cos 90  F  v 
 
0I
93. B= =0
2 R lic
4  107  I 5. As per the figure,
1.33 104 =
2  0.1 F = q(E + v  B) = qE + q(v  B)
 I = 66.5 Now, Fe = qE
n=
It = 16  1018  104 ( k̂ )
ub
e = 16  1014 k̂
66.5  1
n=  4.16  1020 And Fm = 16  1018 (10 î  B ĵ )
1.6  10 19
= 16  1017 B( k̂ )
94. Magnetic induction at O due to coil,
P

 F = Fe + Fm = 16  1014 k̂  16  1017 B k̂
0I
B1 = Since, the particle will continue to move along
2r
+ X-axis, so resultant force is zero. Therefore,
Considering perpendicular distance between Fe + Fm = 0
et

wire and O as nearly r,


Magnetic induction at O due to wire.  16  1014 k̂ = 16  1017 B k̂
0 I  B = 103 Wb/m2
B2 =
rg

2r 6. The charge will not experience any force if


As both fields are directed inside the plane of  

paper magnitude of net magnetic field, | Fe |  | Fm | . This condition is satisfied in option


B = B1 + B2 (B) only.
Ta

I I
= 0 + 0 9. t = 2.3  108 s
2r 2r
 T = 2t = 4.6  108
I  1
= 0 1   
1

1
= 2.17  107 Hz
2r    f=
T 4.6  108
95. Magnetic field at distance of 10 cm due to wire,
0I 5 0 2mK
B=  10. r r K
2r 2  0.1 qB
Magnetic force on electron, R K
   R2  R 2
Fm = qvB R2 2K
5  4  107
= 1.6  1019  3  106  2 qB  2m 
2  0.1      v0
11.  T  
T m  qB 
= 4.8  1018 N
355

MHT-CET Triumph Physics (Hints)


12. Kinetic energy, 23.

m = NI A

2 2 2
1 2 qBR     
mv   = m  B = NI A  B = NIAB sin 
2 2m
For an -particle, the charge is two times that of but  = 90  

the proton but mass is 4 times that of the proton.   = NIAB sin(90  ) = NIAB cos 
Hence compared to kinetic energy of a proton,
Plane of the
for the same conditions in the cyclotron, energy
coil
of alpha particle is E.
13. Kinetic energy in magnetic field remains  
constant and it is K.E. = qV B0
 K.E  q 
(V = constant)

ns
 K.Ep : K.Ed : K.Ea = qp : qd : qa = 1 : 1 : 2
normal
14. Maximum force will act on proton so it will
move on a circular path. Force on electron will 24. Suppose length of each wire is l.
be zero because it is moving parallel to the field. 1

io
r
B
15. From figure, vector length of wire l = 3 cm 2
F = BIl = 2  2  3  102 = 0.12 N  l  l2
A circle  r 2     
 2  4

at
Now, F = ma l/4
0.12
 a= = 12 m/s2 (along Y-axis)  Magnetic moment m  IA
10  103
msquare Asquare r
 
16. Let Fm be the force arising due to magnetic
lic
field, then the given situation can be drawn as
m circle A circle
follows l 2 / 16 
Fm cos 60°  
l 2 / 4 4
60°
ub
Fm 25. As shown in the following figure, the given
B situation is similar to a bar magnet placed in a
mg sin 60° 60°
mg cos 60° uniform magnetic field perpendicularly. Hence
60° mg torque on it is given by,
P

B
Fm = BIl  mg sin 60  BIl cos 60
o o
S
0.01 10  3
B 1T
0.1 1.73
et

17.  = N I B A sin  N
 does not depend upon shape of the loop.
  Area. The area of circle is largest.
rg

18.  = mB sin 90 = (Ir2)B


19.  = NBIA sin , so the graph between  and  is 26. m = nIA = 100  5  2 102 = 10
a sinusoidal graph.
1 = mB sin  ; 2 = mB cos 
Ta

20. For  = 90°,  12 = m2B2 sin2 ,  22 = m2B2 cos2 


3 2  2 2
Area of equilateral triangle = l 12 + 22 = m B
4
 (0.09 + 0.16) = 102B2
 3 2 0.25
  = NIAB = 1  I   l B B2 = = 2.5  103
 4  100
3 2 B = 0.05 T
 l BI
4 27. Work done here is equivalent to magnetic
21. Here, θ = 90 60 = 30 potential energy of dipole.
 = NIAB sin 30  W = mB cos  = mB (cos 2  cos 1)
1 = mB (cos 60  cos 0)
= 500  0.2  4  104  103  1  mB
2 W = mB   1 = ….(i)
5 2  2
  = 2  10 N-m
356
 Chapter 10: Magnetic Fields due to Electric
Current
Now, when  = 60, torque acting on dipole 33. Every point on line AB will be equidistant from
should be X and Y-axis. So magnetic field at every point
3 on line AB due to wire 1 along X-axis is equal
 = mB sin  = mB sin 60 = mB in magnitude but opposite in direction to the
2
magnetic field due to wire along Y-axis. Hence
Using (i)
Bnet on AB = 0
 = 3W
34. Net current I = I1 + I2 + I3 + I4 + I5 + I6
 2I = 10  13 + 10 + 7  12 + 18
28. B 0 
4 R = 20 A
Hence, if distance is same, field will be same. 0 I
Magnetic induction B =
2R
29. Magnetic field lies inside as well as outside the
Here R = 10 cm = 0.1 m

ns
solid current carrying conductor.
4  107  20
 B= = 40  106 T = 40 µT
0 2I 2  0.1
30. B=  (For infinitely long conductor)
4 r
0 I
1 35. By using, B  (sin 1  sin 2 )

io
B 4 r
r 0 I
Hence, graph (C) is correct.  B (2sin )
L/2 5
L
4 (L / 4)
4

at
31. L/2 2 
Also, sin    P
4 cm 5L / 4 5 L/4 
4 0 I L/2
 B
I P 3I
lic 5L
36.
x 4x
45 l
ub
At neutral point P, B1 = B2
2l i
0 I  3I
 = 0 O
2 x 2 (4  x)
 x = 1 cm
P

32.
Using Biot Savart’s law,
I 2I Magnetic field due to one side of the square at
et

r r centre O
A C 0i
O B1 =   sin 45  sin 45 
4l
rg

 2i
= 0
4 l
Let two wires be A and C carrying current I and Hence magnetic field at centre due to all sides,
2I respectively. The magnetic field produced by  0 ( 2)i
Ta

B = 4B1 =
two wires at mid-point ‘O’ will be in opposite l
direction. Hence net magnetic field at O is, Magnetic field due to n turns
I I 2 0 nI
B = B1  B2 = 0 1  0 2 Bnet = nB =
2r 2r l
  
= 0 (I1  I 2 ) = 0 (I  2I)  0 (I) 37. Bnet
2r 2r 2r B1
Here negative sign indicates the direction of B.
Hence neglecting it, |B| = |B1| ….(i) B2

When 2I wire is switched off, field produced at


2cm
point O will be only B1, which referring to
equation (i), equals B.
Thus, when 2I wire is switched off, field will be At the point, magnetic induction due to external
B. magnetic field be B1 = 4  104 T.
357

MHT-CET Triumph Physics (Hints)


Now, due to wire carrying current magnetic 41. The wires are in parallel and ratio of their
I resistances are 3 : 4 : 5. Hence currents in wires
induction produced at that point be B2 = 0
2a 1 1 1
are : : .
4  107  30 3 4 5
= Force between top and middle wire,
2  2  102
4  0 I1I 2 L
= 3  10 T F1 = (r1 = distance between these wires)
2r1
38. Magnetic field at P due to wire (1),
0 2(8)  0 (1 / 3)(1 / 4)L
B1   F1 =
4 d 2r1
P (0, 0, d) Y Force between bottom and middle wire,
B2  0 (1 / 4)(1 / 5)L
B1
F2 =
 2r2

ns
6A
As the forces are equal and opposite,
X
1 1 r 5
8A  F1 = F2  =  1=
3r1 5r2 r2 3

io
0 2(6)
and that due to wire (2), B2  42. Force between two long conductors carrying
4 d current,
  16    12 
2 2
0 I1I 2
F= ….(i)

at
 Bnet  B12  B22   0    0  l
 4 d   4 d  2 d
After carrying out changes,
0 2 5
   10  0  0 (2 I1 )(I 2 )
4 d d F = l
39. Magnetic field due to 1st wire,
lic 2 d
From (i) and (ii),
0I
B1 = …(R = 3 m) F 2 / d d  0.5  2
2 3   2    2  
F 1/ d  d   1.5  3
Magnetic field due to 2nd wire,
ub
2
0 I  F = F
B2 = …(R = 4 m) 3
2 4
Net magnetic field, 0 I1I 2l
43. F=
B= B B
2 2 2a
P

1 2
mg  0 I1I 2
0I 1 1 =
=  l 2a
2 9 16
20  103  9.8  2  2  102
et

I2 =
0I 25 4  107  200
=
2 144  m 
0 I 5 50 I  = Linear density 
=    l 
rg

2 12 24 I2 = 98 A
40. 44. Mass per unit length of conductor XY,
m = 5  102 kg/m
Ta

As magnetic repulsion is balancing the weight


x 30 O
of conductor XY
I
X Y
i 4 mm

0 i A I1 = 25A B
Here, B = 6  (sin 1 + sin 2)
4 r
F
3 mg =
For r  x l
2
0 I1 I 2   0  2I1 I 2
 0 i (2sin 30) 3  0i  mg = = 
 B = 6  2a  4  a
4  3  x 2  25  I2
 x  mg = 107 
 2  4  103

358
 Chapter 10: Magnetic Fields due to Electric
Current
25 52. Using Shortcut 3(iv),
5  102  9.8 =  104 I2
2 B = B12  B12 = 2 B1
2  5  102  9.8 B
 I2 = = 392 A  = 2
25  104 B1
0 I  I
46. B1 = , B2 = 0 53. Using shortcut 1,
4 a 4 b
Ratio of initial to final turns = 2 : 1
 0I  1 1  1
 Field due of ABCD = B1  B2 =     n=
4  a b  2
47. In the figure, magnetic fields at O due to B1
 B2 = n2B1=
sections 1, 2, 3 and 4 are considered as B1 ,B2 ,B3 4
B1 4
and B4 respectively.  =

ns
4 B2 1
R2 2 Alternate Method:
R1
L = n1(2r1) = n2(2r2)

io
r1 n
O 3
 = 2
1 r2 n1
B1  B3  0 2 2
B1  n I / 2r1 n r  n   4 4
= 0 1 = 1 2 =  1  =  =

at
0  i B2 0 n 2 I / 2r2 n 2 r1  n 2   2  1
B2  . (directed into the paper)
4 R 1
0  i 54. Using shortcut 1,
B4  . (directed out of the paper) Here n= 2,
4 R 2
As | B2 |  | B4 |
lic 2
B2  n 2 B1 = 2 B = 4B

 Bnet  B2  B4 Alternate Method:


 0i  1 1 
Magnetic field at the centre of current carrying
 Bnet 
ub
   (directed into the paper) 0 2Ni N
4  R1 R 2  coil is given by B    B
4 r r
48. Refer Shortcut 3(i) B1 N1 r2
   ….(i)
Alternate Method: B2 N 2 r1
P

  Given that,
2    I
0 (2  )I  0  2 N1 = 1, N2 = 2, r1 = r, r2 = r / 2,
B= =
4 r 4 r B1 = B
30I
et

 B 1 r/2 1
B=      B2  4B .…[From (i)]
8r B2 2 r 4
 0 nI 55. Using shortcut 1,
49. Magnetic field at the centre of coil B =
rg

2r Here n = 3,
L 2
n = 1 and 2r = L  r = B2  n 2 B1 = 3 B0 = 9B0
2 Alternate Method:
0 2I  0 I 0 NI 0I  1 0 I

Ta

B= = B0 =   for 1 turn.
2L L 2a 2a 2a
q For rewinding the coil in three turns, new radius
50. I= = 100  e a/3, number of turns (N) = 3.
t
 2I  2  100e 0I  3 9 I
Bcentre = 0 = 0  New magnetic field =  0  9 B0
4 r 4 r 2  (a / 3) 2a
 0  200  1.6  1019 56. The magnetic induction at centre of a coil is
= = 1017 0
4  0.8 0 Ni
B=
0I  q  qf 2r
51. B= = 0 = 0   10  0.2   10   0.3
2r 2r t 2r  B1 = 0 , B2 = 0
4  107  1.6  1019  6.8  1015 2  20  102 2  40  102
=  15  5
2  0.5  1010  B = B1 + B2 = 0  5   = 0
= 13.7 T  4 4

359

MHT-CET Triumph Physics (Hints)


57. Magnetic field at the centre of circular coil, and magnetic field due to circular portion,
(L = 2r1) 0I  2   2 
BPQ =  
 0 2i  0 4 i 2
2R  2 
Bcircular   and
4 r1 4 L 0 I
= (  )
Magnetic field at the centre of semi-circular 2R
coil, (L = r2 ) I
 B = BPQ + BPQ = 0 (   + tan )
  i  0 2 i 2R
Bsemi  circular  0 
4 r2 4 L
0Ia 2
Bcircular 61. B=
 =4 2(a 2  z 2 )3/ 2
Bsemi  circular
Where, a is the radius of the coil.
58. 0Ia 2

ns
=
1 2(a 2  a 2 )3/ 2
 0 Ia 2
=
r 2(2)3/ 2 a 3
2 O

io
4107 1 
= =  107 T
4 2 a 2a
3

at
Magnetic fields due to different portions 1, 2 0 Ia 2
62. B1 = ,
and 3 are respectively, 2(a  x 2 )3/ 2
2

B1 = 0, lic But x >>> a


0 I  0 Ia 2
B2 = . (directed outside the paper)  B1 =
4 r 2x 3
 I 0 Ia 2
B3 = 0 . (directed outside the paper) B2 =
4 r 2(a 2  x 2 )3/ 2
ub
I I But x = 0
 Bnet = B2 + B3 = 0 + 0
4r 4r 0 Ia 2
 B2 =
2a 3
59. The angle subtended by the circular part ABC at
B1 a3
the centre is 3/2.  = 3
P

B2 x
B
O 0 2I
63. Magnetic field at centre, B = ×
C 4
et

r
A D Magnetic field at a point on the axis,
0 2 I r 2
Field due to ABC, B = 
4  r 2  z 2 3/ 2
rg

0 I  3 
B1 =   B B
4 r  2  Given B =  =8
8 B
Field due to AD at O,
 0 2 I
Ta

0 I 0 I
B2 =    4 r =8
2r 2 4r 0 2 I r 2

…[ A is at the end of the wire] 4  r 2  z 2 3/ 2
0I  3  r  z2 
2 3/ 2
 Total induction =   1  =8
4r  2  r3
(r 2  z 2 )3
60. Here, magnetic field due to straight portion,  = 64
r6
0I
BPQ = (sin  + sin ) r 2  z2
4R cos   =4
r2
( OM = R cos )  r2 + z2 = 4r2
=
 0 I 2sin   0 I
 tan   3r2 = z2
4 R cos  2R  z= 3r

360
 Chapter 10: Magnetic Fields due to Electric
Current
0 I R 2 P
64. B=
2(R 2  z 2 )3/ 2
I
 0 I(6) 2  102 
2

108  106 = I
2  6 2  82   104 
3/ 2 3/ 2

0ni
0 I 108  10  10 
3/ 2
6 2
 10 6 108  105 72. B=
 = = l
2 62  10 4 36 4  107  n  10
….(i)  0.2 =
0.8
At the centre of the coil, z = 0
4  104
0 I 0 I  n=
 B= = 
2R 2  6  102 Since n turns are made from the winding wire,

ns
Using (i) so length of the wire (L)
108  105 = 2r  n [2r = length of each turns]
B= = 5  104 T = 500 T
36  6  102 4  104
 L = 2  3  102  = 2.4  103 m

io
 0 IR 2 I
65. Baxis = , Bcentre = 0
2(z  R )
2 2 3/ 2
2R 73. The magnetic field in the solenoid along its axis
R3
(i) at an internal point = 0ni
Baxis = Bcentre  = 4  107  5000  4 = 25.1  103 Wb/m2

at
(z  R 2 )3/ 2
2
(Here, n = 50 turns/cm = 5000 turns/m)
 Baxis   z 2  R 2 
3/ 2
(ii) at one end
Bcentre = 1  ni 25.1  10 3
R3 Bend = Bin = 0 =
Bcentre =
(54)(4  32 )3/ 2
2
=
54 125
= 250 T
lic 2 2 2
33 27 = 12.6  103 Wb/m2
74. I = 10 A;
0 2nIR 2 1
66. B  2  B 2 Total number of turns = 4000
ub
4 (R  z 2 )3/ 2 (R  z 2 )3/ 2 Mean radius of toroid,
2/3
8 (R 2  z 22 )3/ 2 8 R 2  0.04 25  26
  2    r= = 25.5 cm = 25.5  102 m
1 (R  z1 ) 2 3/ 2
1 R 2  0.0025 2
R 2  0.04 Total length (circumference) of the toroid
P

4
  2
1 R  0.0025
, = 2r = 2  25.5  102 = 51   102 m
Therefore, number of turns per unit length,
On solving, R = 0.1 m
4000
n=
et

67. Magnetic field lines about a current carrying 51102


wire get crowded when the wire is bent into a The field inside the core of the toroid,
circular loop. 4000
B = 0nI = 4  107   10
rg

0 2nI 51102
68. Using, B =  ,
4 r = 31.4  103 T
2nI 2    25  4
B = 107  = 107  75. The coil is made up of tiny current elements.
r 5  102
Ta

Force acting on each current element is directed


 B = 1.256  103 T outwards. As a result of this the coil expands.
69. Inside the pipe, I = 0 76. The normal to the plane of the coil (x-y plane)
0 I makes angle of 90 with the direction of the
 Binside  0
2r field.
 torque on the loop,  = BIA = BI (r2) .…(i)
70. Applying Ampere’s law,  B.dl   I ,
0 to any Also the torque required to just raise an edge of
closed path inside the pipe, we find no current is the loop is
enclosed. Hence, B = 0.  mg 
 = Fr =  r .…(ii)
 2 
71. The respective figure is shown below.
Equating (i) and (ii),
Magnetic fields at P due to inner and outer
mgr mg
conductors are equal and opposite. Hence, net BIr2 = I=
magnetic field at P will be zero. 2 2Br

361

MHT-CET Triumph Physics (Hints)


77. The net force on the particle is 80. Here magnetic force is zero, but the velocity
  
F  q (E  v  B)

….(i) increases due to electric force.
The solution of this problem can be obtained by 1 e2
81. Electrostatic force, Fe =  2
resolving the motion along the three coordinate 40 r
axes namely,
0  e2 v2 
F q  Magnetic force, Fm =  
a x  x  (E x  v y Bz  v z By )  4  r 2 
m m

Fy q  Fm v2
a y   (E y  v z Bx  v x Bz )  ….(ii)  = 00 v2 = 2 ( 00 = 1/c2)
m m Fe c

Fz q 
a z   (E z  v x By  v y Bx )  82.
m m  Straight wire
For the given problem,

ns
Circular loop
Ex = Ey = 0, vy = vz = 0 and Bx = Bz = 0
Substituting in equation (ii), Direction of magnetic
q field due to straight
ax = ay = 0 and az = [ E z  v x By ]
m current carrying wire

io
As electric field is along the negative z-axis,
hence Ez is negative. Magnetic field due to straight wire is either
Again az = 0, as the particle transverses through parallel or antiparallel to the current flow in

at
the region undeflected. loop depending on direction of current in wire.
 Ez = vx By Thus force F exerted by this magnetic field B is,
  
E z 5  104 F  Idl  B
 By =  = 2.5  103 Wb/m2
vx 20
lic = IdlB sin  = 0 (  = 0 or 180)
1
78. Ekinetic = mv2 83. Current carrying coil is a closed loop. Net force
2
acting on the coil due to uniform magnetic field
ub
1
2  1.6  1013 =  1.67  1027  v2 is always zero. But there will be a non-zero
2 torque acting on the coil, except when plane of
v2 = 3.83  1014 the coil is perpendicular to the field.
v = 0.196  108 m/s
84. B at the centre of a coil carrying a current, I is
F = qvB sin 
P

0I
= 1.6  1019  0.196  108  2.5  sin 90 Bcoil = (upward)
2r
F = 7.84  1012 N
0 I
1 B due to wire, Bwire = (downward)
et

79. The kinetic energy of the proton, mv2 = qV 2r


2
2Vq Magnetic field at centre C
 v2 = ….(i) Bc = Bcoil + Bwire
m
rg

0I I
If the proton is moving undeflected, then the = (upward) + 0 (downward)
deflection produced by the electric field must 2r 2r
nullify the deflection produced by magnetic I I  I  1
= 0  0 = 0 1   upward
field. 2r 2r 2r   
Ta

As, the deflection of the 4  107  8  1


proton caused by the = 1   upward
2  10  102  
magnetic field is upwards,
deflection produced by the B 4 107  8  2.14
= upward
electric field should be into 2  10  102  
the paper. Hence the direction = 3.424  105 N/A m upward
of the field is also into the
85. Number of revolutions completed by the
paper.
electron in one second,
qE = qvB  v = E/B ….(ii)
v
2Vq E 2 2Vq n=
 Equation (i) gives v2 =  2 2r
m B m Also current,
mE 2 v
V= I = nq = q
2qB2 2r

362
 Chapter 10: Magnetic Fields due to Electric
Current
Now, magnetic field,
0I O
B = r
2r
0 v 
  q
2r 2r N B
 vq 
 0 2
4r
4107  2.2 106 1.6 1019 t
=
4  510 
11 2

From Figure, t = r sin 


 B = 14.08 T t t Bq t Bq
or sin  =   1
86. For charged particles, if they are moving freely r mv

ns
 2Vq  2
m 
in space, electrostatic force is dominant over  m 
magnetic force between them. Hence due to
electric force they repel each other. Bt q q
=  Bt
m 2V 2Vm

io
87. Deflecting couple on magnet
= mB sin  = (2l × pole strength) B sin  92. Change in momentum = Impulse
= (10  8)  0.32  sin 45

at
t t

= 18.1  18 dyne cm i.e., mv =  Fdt   BIl dt


0 0

2e t
88. Current due to motion of  particle = = Bl  I dt = Blq
T
2e
lic 0

 Magnetic moment = I  A =  r2 Blq


T Or v = But v = 2gh
m
e  2r  r
=
ub
T Blq m 2gh
 2gh = or q =
= evr m Bl

89. For magnetic field to be zero at centre of loop, 93. Here magnetic force = BIa
 0 Ic  0 Ie Weight of a side is mag, where m is mass per
P


2R 2H unit length, and that of two sides i.e., 2 mag is
IR effective at the centre.
 H= e
Ic
et

a/2
90. If a wire of length l is bent in the form of a 
circle of radius r then 2r = l
rg

l 2  a/2
 r= = =
2 2 2
Magnetic field due to straight wire magnetic force (BIa)
0 2I  2 2 (2m  a)g
(m  a)g
Ta

B1 =  = 0 
4 r 4 1 102
Also, magnetic field due to circular loop, Then taking moments,
0 2I  2 2 a
B2 =  = 0 2 mag  sin  + mag  a sin  = BIa a cos 
4 r 4  / 2 2
B2 1 i.e. 2ma2g sin  = BIa2 cos 
 =
B1 50 BI
 tan  = But m = A
2 mg
1
91. Using qV = mv2, we get BI
2  tan  =
2Ag
v= 2Vq / m
2Ag
mv 2 mv  B= tan 
Again, Bqv = i.e., r = I
r Bq

363

MHT-CET Triumph Physics (Hints)


mv
9. Radius traced, R =
Competitive Thinking qB
Though proton and neutron have nearly same

 
1. F   q  v  B mass, neutron does not have charge. As a result,
  two particles will not trace circular path of same
As particle is projected towards east radius. Rather, neutron being charge less will
 
vi not experience magnetic force.
Hence, assertion is false but reason is true.
Force is acting in north direction


Fj

10. In cyclotron,
r

 
j  (i  B)

v
t
But we know,  vr

ns
  
i  (  k)  j Bq
While,  =
  m
 B = –k
i.e.,  is independent of r.
2. The particle is released from rest

io
 
11. Momentum p = qBR
and E  B
  
12. Time period of revolution is given by,
Fnet  FE  FB ….(i) 2m
T=

at
 
     qB
FE   q E and FB   q  v  B  Hence, time period is independent of velocity of
 
  the particle.
As v = 0, FB = 0
hence, Fnet = FE
 
lic13. Radius of circular path:
mv

r=
qB
As FE is acting along the direction of electric
1
ub
field, particle will always move in the direction  r
  B
of electric field. Also, v being parallel to B , B
particle will not deviate. When B is reduced to , r is doubled
2
3. When particle moves undeflected, magnetic  New radius of circular path is 2r.
P

force and electric force on particle exactly mv


balance each other, i.e., Fm = Fe 14. r=
qB
E
qvB = qE  v = mv
et

B i.e., B =
qr

 
 9.1  1031  106 9.1
4. FB =  q  v  B  B= 
–5
 105 = 2.84  10 T
  19
1.6  10  0.2 1.6  2
rg

  
  

FB =  q a i   b j  ck   mv v
   15. r= r
qB  
q
     B
=  q abk  ac( j)  m
Ta

 
109

  
 r= = 102 m
FB = qa  bk  c j  (1011 ) 104
 
Taking magnitude on both sides, 16. Initially FE = Fm
  qE = qvB
FB = qa b2  c2 E 2  10 4 2
 B= = = = 2  10–2 T
v 106 100
FB  qa(b2  c2 )1/ 2
Now when E is switched off,
mv mv v
mv 2Em r= = =
7. r=  qB eB e
qB qB B 
m
mv 106 1
8. r= rv   = 0.5 m
Bq 2  102  108 2

364
 Chapter 10: Magnetic Fields due to Electric
Current
17. From figure, we know that 2m  K.E.
r1 < r2 21. r=
Bq
also,
m 2m 4m
mv rp : rd : r : = : : = 1: 2 :1
r= q q 2q
qB
v and B being constant, 22. For cyclotron,
m mv m
r   B=  ….( v = R)
q eR e
m  2 2m
m1 m2 = 
  e e
q1 q2
1 2 1
K.E. = mv max  m  R
2
m m

ns
i.e.,      2 2
 q 1  q  2 1
= mR 2  42 2 
2
18. For charged particle moving perpendicular to
mv p
= 2 mR222

io
uniform magnetic field, r = =
qB qB 23. Radius in magnetic field
Given that, pH = pα = p(say) mv 2mE
R= 
p p qB qB

at
 rH = and rα =
qHB qa B q 2 B2 R 2
But, qα = 2qH E=
2m
rH q a B 2q H B 2 For proton
  
ra q H B q H B

1
lic E1 =
e 2  B2  R 2
2  mp
19. For a charged particle inside a magnetic field,
radius of path is, For -particle
ub
mv p (2e) 2  B2  R 2
r=  E2 =
qB qB 2  4m p

E=
1
mv2 =
p2  E1 = E2 = 1 MeV
2 2m
P

As K.E. for all the particles is given to be same, 2 mE


24. Radius of circular path: r =
qB
p m
Also, the magnetic field is same, q 2 B2 r 2
E= ....(i)
et

2m
p m
 r  or qB
q q Cyclotron frequency is f =
2m
For given particles,
rg

 q2B2 = 42m2f2 ....(ii)


qp = qe q = 2qp Using equation (ii) in equation (i),
mp = 1836 me m = 4mp 1
mp
E= (4 2m2f2)r2
2m
Ta

mp 4m p mp
 rp  , re  1836 , r    E = 2 2mf2r2 ....(in joule)
qp qp 2q p qp 2 mf r
2 2 2
 E= ....(in eV)
 re < rp = r e
2  10  1.67  1027   10  106    0.6 
2 2
q 2 B2 r 2 = eV
20. K.E. = 1.6  1019
2m
But here K.E. = qV = 7.5  106 eV
qV  2m = 7.5 MeV
 r2 =
q 2 B2 The closest value in the option is 7 MeV
 Option (C) is correct.
r m
2
m1  r1  2m T m q 2
  25. T=     p 
m 2  r2  Bq Tp m p q  1

365

MHT-CET Triumph Physics (Hints)


2m 34. As the coil is perpendicular to magnetic field
26. T= 
qB B2T ,
i.e., T is independent of v.
 = 90o
 Time period will remain the same. The loop formed is circular,
27. Here, f = 10 MHz = 107 Hz  l = 2r
r = 50 cm = 50  102 m As the force acting on the loop will be along
v = 2 rf = 2  50  102  107 radius,
= 3.14  107 ms1 r
l
2
28. Frequency of revolution is,
 Tension developed is
Be 3.57  102  1.76  1011
f= = T = F = B I r sin = B I r
2m 2  3.14
21.1 l

ns
 1  109 Hz = 1 GHz =
2
29. The oscillator frequency must be same as 1.1
proton’s cyclotron frequency. = = 0.35 N

qB

io
f= K
2 m 35. I=  I
27 NAB
2 mf 2  3.14  1.67  10  12  10 6
 B= =
q 1.6  1019 36. B

at
2
= 78.6  10 T  0.8 T
30
30. Component of velocity parallel to the field,
60 A
makes the particle move in direction of field and
due to perpendicular component of velocity,
lic
particle follows circular path making combined
path as helical.
ub
31. Parallel component drags the particle to side and
perpendicular component gives circular path. τ = NAIB sin θ
Hence the path is helical. = 50 × .012 × 2 × 0.2 × sin 60º
= 0.20 Nm
32. F = IlB sin 
37. A
P

 = 90
 sin 90 = 1 For square coil,
 F = IlB Area (A1) = length2 = a2
For circular coil,
 mg = IlB
et

2
2.5  50  102  0.5  a 
m =
IlB
= =
1
= 62.5 g Area (A2) =   (radius)2 =   
g 10 16  
a2
= = a2
rg

33. We know

FB = i  l eq  B  = i leff B ….( leff  B)
 
1 A a2
   = 1 = 2 =1
 
2 A2 a
Ta

For PQ l eq || B

38.  
(FB ) PQ = 0 B m
For PR

3
lPR = l (which is perpendicular to B )
2
 3 
(FB)PR = i leff B = i  l  B
 2  
B
3
(FB)PR = i lB
2 When  = 0 (parallel) it is in stable
Similarly for QR equilibrium.
3 When  = 180 (anti-parallel), it is in unstable
(FB)PR = i lB
2 equilibrium.

366
 Chapter 10: Magnetic Fields due to Electric
Current
39. For stable equilibrium 0 Idl sin 
 
45. By Biot-Savart’s law, dB = ×
m  magneticdipole moment  ||B 4 r2
1
z i.e., dB ∝
r2
46.
B I1
I B2
I y d
I B1
x I
I2

For unstable equilibrium m ||   B 


 

 

ns
z
 0 2 2 1/ 2
B= B12  B 22 =
2d
 I1  I2 
B
47. Magnetic field due to first wire is given by

io
I 0I
I y B1 =
I 2 r
x I Magnetic field due to second wire is given by

at
4
40. m = nAI = 2000  1.5  10  2 = 0.6 0 I I
B2 = = 0
2 1 2 (3r) 6 r
  = mB sin 30 = 0.6  5  10 
2 Net Magnetic field at P is, B = B1 + B2
 = 1.5  102 Nm
  
lic B=
0I I
+ 0
2 r 6 r
41.   m  B = mB sin θ 
m 30I + 0I
=
 6 r

ub
 = NIA × B sin 90
40I 2 0 I
2 = =
 τ = NI (πr ) B× 1 O 
B 6 r 3 r
= B πr2 IN
48.
P

42. m = I  Area of loop k̂


 a 2    P
= I  a 2   4  kˆ = I  a 2   1 kˆ
 4 2 2    90
30
et

d 3
43. K.E = mB(cos1  cos2)
2
Here, 1 = 0 (along East - West) 60
2 = 90(along North-South) 60
rg

K.E. = 2.5  3  105  (1 – 0)  


= 75  106 = 75 J  
 0 I
Bnet = 2    [1  sin 30]
44. P.E = U = nIABcos 4  d 3 
 
Ta

   
i.e., U = nI( A  B )   2  
also, magnetic dipole moment,
   0 2I 3  3 0I
m  nI A = 2   =
 4  d 3 2 2d
   
 given A  0.04 i  0.05 j  0.07k
49. 2
  
 m  15  12  A  180  A
   I 16 A
= 7.2 i  9 j  12.6k
   
    

 U =   7.2 i  9 j  12.6k  .  0.2 i  0.3 j  0.4k  
     r
x x
= [1.44  2.7  5.04] 2 O 1
= (6.3) 

U = 6.3 J Magnetic field at point r due to wire 1 = 0

367

MHT-CET Triumph Physics (Hints)



57. For distribution of current between any two
Magnetic field at point r due to wire 2,
points on the circumference of the loop, the
0 I
B= ….(As wire is semi-infinite) magnetic field at the centre of the loop is zero.
4r
4  107  16  0I
 B= ( | r | = 2 mm = 2  103 m) 58. B=
4  2  10 3 2r
= 8  104 T = 0.8  103 T = 0.8 mT q
I= =en
t
50.  e n
x B= 0
B1 1A = ii 2A = i2 2r
0.1m 0
B2 59. B= B12  B22 = I12  I 22
2r

ns
For net force to be zero, net magnetic field at 4107
= 32  42
that point should be zero, i.e., 2 2102
B1 = B2 = 5  105 Wb m2
 0 (1)  0 (2)

io
2x 2(0.1  x)  0 I
60. B1 
2x = 0.1 + x 4 R 1
c
x = 0.1m 
here,   60 

at
3
F  0 2I1I 2
52.  
l 4 r I
0 3  I
Here, r = b and I1 = I2 = I  B1   0


F  0 2I

2
lic 4 R1 12 R1
Similarly,
l 4 b

F 0 I1I 2 55 I
53. = = 2  107  0 3  I
B2   0
l 2d 1
ub
4 R 2 12 R 2
= 5  106, attractive Bnet = B1 – B2
 I I l 4107  5 5 5  102  I 1 1 
54. F= 012 = = 105 N =
0I I
 0 = 0  
2d 2 2.5102 
12R 1 12R 2 12  R1 R 2 
P

55. Net force on wire B, Fnet = FA2  FC2


61. Magnetic field at point O,
 0 I1I 2l
FA = FC =  0 I ˆ  0 I (ˆi) = 0 I [ˆi  2k]
ˆ
2 d B (2k)
4R 4R 4R
et

0i 2l
= ....( I1 = I2 = i) 62. Magnetic field at C due to semi-infinite wire,
2d
0i
  i 2l 
2
2 0i 2l  i 2l B1 =
 Fnet = 2 0  = = 0 4r
rg

 2d  2d 2d Magnetic field at C due to semi-circle,


 i2 0i i i
 Net force per unit length = 0 B2 = = 0 = 0
2d 4r 4r 4r
Ta

As directions of both fields is outward through


56. For sides AD and BC, force acting on them is plane of the paper, magnitude of net magnetic
equal and opposite. Hence the net force is zero. field, B = B1 + B2
 Fnet = FBA  FCD 0i i
 2IiL = + 0
here, FBA  0 4r 4r
4 L 0i
2 = (1 + )
4r
 2IiL
for FCD  0 63. When radius is doubled the resistance in the
4 3 L
2 circuit is also doubled. Therefore the current in
  the circuit becomes halved.
 0 IiL  1 1  0 IiL  4L  20 Ii Magnetic induction is given by,
 Fnet     = =
2  L 3 L  2  3L2  3 B=
0I
2 2 2r

368
 Chapter 10: Magnetic Fields due to Electric
Current
Now,  (R2 + z2)1/2 = 2R
B′ =
0 I I
where I′ = and r′ = 2r  R2 + z 2 = 4R2
2r 2  z 2 = 3R2
I B  z = 3R
∴ B′ = 0 =
8r 4
67. According to Ampere’s circuital law,
64. Refer shortcut 1 the magnetic induction on axial line of a straight
Alternate Method: current carrying conductor is zero.
Let the wire of length l be bent into circle of  The segments DE and AB do not produce a
radius R. magnetic field at O.
 0 nI For segments BC and EF,
 B=
2R  0 IC  I
BBC  , BEF  0 F
here, n = 1 4 rC 4 rF

ns
l Bnet = BBC + BEF
R=
2  4 9 
0I  Bnet  107    5
  5  10 T
 B=  0.02 0.03 
 l 

io
2 
 2  68. By Ampere’s circuital law,
 


 I
B= 0 ....(i)  B dl   I 0 enclosed   0 (2  1)   0
2l

at
When the same wire is bent into coil of n turns, 0 2I
69. B
let R be the radius of the coil, 4 r
 2nR = l  105  107 
2I

 R =
l
2n
lic  I=5A
(10  102 )

 nI 0nI  I
 B = 0 = = 0 n2 70. Net current through system I = I1 + I2 + I3
2R   l  2l
2  =35+8=6A
ub
 2n  Using Ampere’s law,
 B = n2B ….[From (i)] 0 I 4  107  6
65. Bcentre = 5 5 Baxis B=  = 12  106 T = 12 µT
2R 2  0.1
0n I 0n IR 2
=5 5 71.
P

2 R 2  z2 
3/ 2
2R

(R2 + z2)3/2 = 5 5 R3
(R2 + z2)3 = 125 R6
et

a
/2 a
R2 + z2 = 5R2 B
z2 = 4R2
2a
z = 2R
rg

B
z = 2  0.1 ….( R = 0.1) According to Ampere’s Circuital law,
z = 0.2 m For inside loop,
 0 nI  0 r I  I  A 
B ....  as I 
Ta

66. Magnetic field at the centre: Bc = 


2R 2r 2  A 
Magnetic field at the axial point: a
0I  
 0 nIR 2  B 2
Baxis =
2 R 2  z2  2a 2
3/ 2

I
Bc B o ....(i)
Given: Baxis = 4a
8 For outside loop,
 0 nIR 2  nI B  (2r) = µ0I
= 0
2 R 2  z2 
3/ 2
8  2R 0I I
 B   0 ....(ii)
2(2a) 4a
R2 1
 = From equations (i) and (ii),
2 R 2  z2 
3/ 2
16R
B
1
 (R2 + z2)3/2 = 8R3 B
369

MHT-CET Triumph Physics (Hints)


72. I 77. Rod will be stationary if component of magnetic
field balances component of weight of rod as
shown in the figure below.
B X
I BIl cos
R  F = BIl


Y
mg
As magnetic field inside conductor is zero,

For d < R, B = 0

ns
However, for d > R, To keep the rod stationary,
I BIl cos = mg sin
B= 0
2d mg tan 
 I =

io
1 Bl
i.e., B 
d g tan 
= ….( m/l = )
Hence, the variation is best depicted by graph B

at
(C). 0.5  9.8  tan 30 19.6
I=  = 11.32 A
0.25 3
73. The proton is moving parallel to the axis of
Force
lic
solenoid. The magnetic field inside the solenoid
is uniform hence it doesn’t affect the velocity of
78. Electric field =
Charge
ma 0
proton. = (in west direction)
e
B = 0ni Magnetic force = Fm
ub
74. ....(n = N/L)
= 3ma0  ma0
400
= 4  3.14  107  5 = 2ma0 (in west direction)
0.4  102  
( v  B is directed towards west)
= 0.628 T
P


Since, v is directed towards north for positive
75. Magnetic field, B = µ0ni 

Here, n = 50 turns/cm charge, B is directed vertically down.


  

= 50  102 turns/m Now, F  qv  B


et

 B = 4  107  50  102  2.5  2ma0 = ev0  B


3 2ma 0
= 5  10 T  B= (vertically down)
ev0
rg

76. Magnetic field inside a toroid is given by, work


 NI 79. Average Power =
B = 0nI = 0 time
2R 2
Ta

 For first toroid,


W= 0
Fdx ….(i)

 0 N1I
B1 = B
2R 1 l = 3m
For second toroid,
0 N 2I I = 10A z
B2 =
2R 2
x
B1 N1 R 2
 =  Magnetic force on conductor
B2 R 1 N 2
F = B I l sin 
=
200 0.2
 Here, B = 3.0  10-4 e0.2 x T, I = 10 A and
0.4 100 l = 1.5  (1.5) = 3 m
=1:1  F = 3.0  104 e0.2 x  10  3

370
 Chapter 10: Magnetic Fields due to Electric
Current
Substituting in equation (i), As the system is in equilibrium vertically,
2
T cos  = gL ….(i)

4 0.2x
W= 3.0  10 e  10  3dx
0
Along horizontal,
= 9  103
2
0
e 0.2x dx
T sin  =
0 I  I  L
….(ii)
9  10 3 2 (2Lsin )
=  e 0.2 2  1
0.2    0I1I 2l 
9  103  F  and here d  2Lsin  
=  1  e 0.4   2  d 
0.2 4 Lsin  Tsin 
= 45  103 [1  0.67]  I2 =
0 L
 14.84  103 J
14.84  103 T
P =  2.97 W I = 2 sin  ....(iii)
5  103 0

ns
80. For given two coils, magnetic induction at their Using equation (i),
centres is same. gL
T=
Let B1 = B2 cos 

io
0 I1 0 I 2 Substituting for T in equation (iii),

2r 2  2r  gL
I 1
I = 2 sin 
 1 0 cos 

at
I2 2
Using Ohm’s Law, 84. Dipole moment M = nIA = I  R2
1 If dipole moment is doubled keeping current
V
V1 
I
lic constant,
M = I  (R)2
 
V2 1  2M = I  (R)2
 2 (I  R2) = I  (R)2
81.
ub
 R = 2 R
Magnetic field at centre of loop is,
a 0I
B=
2R
P

1
 B
0 I  a 2 R
B= 
2 a  r 2 B1 R 2
et

  
0 I B2 R 1
B= a
2 r 2
85. Force on moving charge in magnetic field is
Ba
rg

given by,
82. m = nIA
For coil, magnetic induction at the centre, F = qvBsin θ
0nI but, θ = 90°
B= ∴ sin 90° = 1
Ta

2R
B  2R ∴ F = qvB
 I=
0n Kinetic Energy of proton is given by
For n = 1, Area A = R2 E=
1 2
mv
B  2R 2BR 3 2
m=  R2 =
0 0 2E
∴ velocity (v) =
m
83.
 2E
∴ F=q B
T m
2  2 106 1.6 1019
F = 1.6  1019   2.5
1.6 1027
(L)g F = 8  1012 N
371

MHT-CET Triumph Physics (Hints)


86. Magnetic field at the centre of a circular loop of  1 = 2 = 45o here
radius R carrying current I,
0 I  2  oI
0 2I 0I  BBC  =
B=  and M = IA = I(R2) 4 l   2  2 2l 
4R 2R
B 0I 1  l L
    0  x [Given] As l   
M 2R IR 2 2R 3 2 8
When both the current and radius are doubled, 4 0 I
 BBC 
the ratio becomes 2L
B 0 1   x  By all four sides
   0 3
M 2(2R) 8  2R  8
3 16 0 I
BB 
2L
87. K.E. of particle = qV
BA  0 I 2L 2  2 2

ns
2mqV    = 
 Radius of circular path: r = BB L 16 0 I 16 8 2
qB
89. The electron is revolving along a circular path
r  V where B is constant
 K.E. = qV ….(i)
i.e., V  r2

io
2
also, we know,
V2  r2  1
  K.E. = mv2
V1  r1  2

at
V2  2r 
2
but, v = r
   =4
1 2 2
V  r   K.E. = mr  ….(ii)
 V2 = 4V 2

88. Case I,
lic Equating (i) and (ii)
1 2 2
mr  = qV
Wire is bent to circle, 2
L = 2r mr 22 9.1  1031  (0.20) 2  (120) 2
 V= 
ub
L 2q 2  1.6  1019
r=
2
 V = 1.638 109 V
 magnetic induction at centre,
0I 0I 90.
 Bcircle  
q
P

2r L
2 
 2  Q
 I
 BA  0 ….(i)
et

Case II
KqQ
Wire is bent to square, Electrostatic force of attraction, F =
rg

r2
L = 4l mv 2
l But, centripetal force is given by, F =
L B l C r
 l= 2
mv KqQ
Ta

4
 = 2
1 2 r r
1
v
P r
Time taken by charge to complete a circular
2r
path is given by, T =
v
A D
r
 T
v
 1 
 T  r3/2 ….  v  
Magnetic induction at P due to side BC  r
0 I I
BBC  (sin1 + sin2) But, for circular loop, B = 0
4 l  2r

372
 Chapter 10: Magnetic Fields due to Electric
Current
I 93. Magnetic field at centre of coil
 B
r 0ni
B1 = ….(i)
Q 2r
As current I =
T Magnetic field at distance h,
1 1 0 nir 2
I  3 B2 = 
T
r2 2  h 2  r 2 3/ 2

r 3/ 2 0 nir 2
 B = 
r 2 3/ 2
 h2 
 Br 5/2 r 3 1  2 
 r 
1
i.e., B  5/ 2
Using equation (i)
r

ns
 3/ 2
 h2 
   B2 = B1  1  
91. F  qv  B  r2 
I/2  3h 2 
= B1 1   ….(Using Binomial equation)

io
 2r 2 

3h 2
I I  B2 is less than B1 by fraction

at
2r 2

94. Magnetic field due to semi-circular wire at O


I/2
0I
B =
Using shortcut 3(vi),

lic 4R

In this case, magnetic field at the centre, B = 0 4  107  8


=
 4  10  102
 F =0
ub
= 8  106 T
92. I L F
Force per unit length, =IB
l
d = 2 cm = 8  8  106
P

= 64  106 N/m
I L = 64 µN/m
et

Force of attraction between wires, 95. For a current carrying coil, a magnetic field line
0 I1I 2 forms a closed loop. Each of these loops cuts the
F= L
2 d plane twice, once outside the coil and once
rg

0 100  100 inside the coil as shown in the figure below.


=  4
2 2  102
4  107  104  4
=
Ta

4  102
= 0.4 N
The force would cause displacement in spring I
F
x= where, k is spring constant.
k
0.4
= m Since the flux in two regions are equal but
50
opposite in direction.
Work done = F  x
0.4 i =  0
= 0.4 
50
= 3.2  103 J
= 3.2 mJ

373

MHT-CET Triumph Physics (Hints)

Hints to Evaluation Test

q 2  1.6  1019 0 I
2. I= = 11. By using B  (sin 1  sin 2 )
t 2 4 r
= 1.6  1019 A 0 I L
 B (2sin )
L/2 17
0I   1.6  10 19
4 (L / 8) 8
 B= = 0
2r 2  0.8 L/8 1 
Also sin    P
= 0  1019 17 L / 8 17 L/8 
4 0 I L/2
q 2 B2 R 2  B
3. K.E = 17 L
2m

ns
=
1.6 10    0.5   4 10 
19 2 2 1 2
12. Bc =
0 I
2r
2  1.67  1027
0Ir 2 0I r 2 0I
1.6 
2
 1038  25  102  16  102 Ba = = =
2 r2  r2 

io
3/ 2
= 3 3/ 2
2r (2 ) 2r (2 2)
2  1.67  1027
1024  10 42
 Ba : Bc = 1: 2 2
= = 306.58  1015
3.34  1027

at
13. Y
= 3.06  1013 J B C
13
3.06  10
= eV
1.6  1019 10 A
= 1.9  106 eV
lic 15 cm
= 1.9 MeV
F  0 I1I 2 4  107  1  1 10 A
4. = =
ub
L 2d 2  1 A D
F 7 10 cm
 = 2  10 N/m 2 cm
L X
0 I 1 The effective force is only on AB and CD.
B
P

6. B=
2r r The force on AB is attractive and that on CD is
 When r is doubled, B is halved. repulsive.
Force between two current carrying conductors
et

K 1.5  109  10 0 I1 I2
7. I= = is F1 between XY and AB = l attractive
NAB 100  15  104  0.025 2a
= 4  10–6 A = 4 A  I I l
force and F2 between XY and CD = 0 1 2
2d
rg

Bq
8. Cyclotron frequency, f = repulsive force.
2m
1  1.6  1019  2 0  10  10  0.15 20 10  10  0.15
 F1 – F2 =   
 f =  4  0.02 4 0.12
2  3.14  9.1  1031
Ta

= 2.79  1010 Hz 0 100 100 


 Fresultant = 2   10  10  0.15   
= 27.9  109 Hz 4  2 12 
 28 GHz 0.15  500
= 2  107  100 
12
9. When a charged particle is moving in a region
= 1.25  104 N
with uniform electric and magnetic field parallel
to each other, it experiences force only due to 14. For -particle, q = 2 e
electric field, along the direction of field, due to   
5
F = q (v B) = q[(6  10 î )  (4 î  ĵ )]
which the path of a charged particle will be a
straight line. = q  (6  105) k̂
= 2e  (6  105) k̂
10. The perimeter in plane is two-dimensional.
Negative sign indicates particle is moving along
Amongst the given shapes, circle has maximum
negative Z-axis.
area. Hence, maximum torque will act on it.

374
 Chapter 10: Magnetic Fields due to Electric
Current



| F | = 2  1.6  10
19
 6  105  0 2 I
 4 r = 27
= 1.92  1013 N 0 2 I r 2

0 I 4 3

15. Using, B = , (r 2  x 2 ) 2
2r 3

2I 2 2 (r 2  x 2 ) 2
B = 107  = 107  = 8  108 T  = 27
r 5 r3
1
(r 2  x 2 ) 2
IAB 2 10 5
 =3
16. K= = = 2  106 Nm/degree r
 10
r2  x2
 =9
17. Here, net field, r2
 r2 + x2 = 9r2

ns
B = Field due to circular portion
 Field due to straight portion  8r2 = x2
 I  I  I 1  I(   1)  x= 2 2r
=  0  0   0 1    0
 2r 2r  2r    2r 21. Refer shortcut 3(vi)

io
(perpendicular to the plane of page and directed Alternate method:
into it) Here, the wire does not produce any magnetic
Field due to circular portion is directed into the field at O because the conductor lies on the line

at
plane of the paper and that due to straight through O. Also, the loop does not produce
portion is directed outward and perpendicular to magnetic field at O.
the plane of paper. Thus net field is directed into
the plane of the paper. 22. Using shortcut 1,
lic Here, n = 2
 
p 2 (mv) 2 (qBR) 2  | Bc | = 4 | B c | = 4  0.2 = 0.8 T
18. Energy, E =  
2m 2m 2m
 0 I1I 2
 mv 2  23. F= l
ub
  qvB  2d
 R   (2I1 )I2 2 0I1I2 2
(2e BR) 2 F = 0 l =  l =  F
Then, E = 2 3d 3 2d 3
2  4m p
P

where mp is mass of proton.


(2e BR) 2 E 2
and Ed =  d 
2  2 mp E 1
et

or Ed = 2E = 2  2 = 4 MeV
0 I
19. Magnetic induction, B =
rg

2r
For the coil,
2r = 4(2r)  r = r/4
40 I
Ta

 New magnetic induction, B =


2r
40 I
 B =  4 = 16B
2r

0 2I
20. Magnetic field at centre, B =
4 r
Let the distance be x.
Magnetic field at a point on the axis,
0 2 I r 2
B = 
4 (r 2  x 2 ) 2
3

B B
Given, B =  = 27
27 B'
375
11 Magnetic Materials

Hints

I
Classical Thinking 5. T = 2
M BH

ns
1 MBsin 1 sin 1 sin 90 1 Where I is moment of inertia
5. = = = = =
2 MBsin 2 sin 2 sin 0 0 m(L2  b2 )
I= ….For rectangular block
12
6.  = MB sin (m = Mass of magnet)

io
 M = MB sin 
T m,
 1 = B sin 90 [  = 90]
Hence, if mass is increased 4 times, period gets
 B = 1 Wb/m2 doubled.

at
Also, 1 Wb/m2 = 104 gauss
  e 
M 7. Orbital magnetic moment M    L
10. Gyromagnetic ratio = 0  2m e 
L0
lic Angle made by orbital angular momentum with
14. r < 1 and r > 1. direction of orbital magnetic moment is 180.
20. With rise in temperature, their magnetic e
susceptibility decreases, i.e., 8. I= = e and I  
ub
T
1
m   Minitial = IA
T
 Mfinal = 2IA = 2M
52. Soft iron is highly ferromagnetic.
9. As we know for circulating electron magnetic
P

moment
Critical Thinking 1
M  evr ....(i)
2
et

d
1.  = MBH sin   = MBH cos  and angular momentum J  mvr ....(ii)
d
eJ
This will be maximum when  = 0. From equations (i) and (ii) M 
2m
rg

2.  = MB sin     sin 
10. The magnetic moment of the revolving electron
1 sin 1  sin 90
    is
2 sin 2  / 2 sin 2
e 2r
M = IA =  r2 But T =
Ta

1
 sin 2 =  2 = 30 T v
2
ev evr
 angle of rotation = 90  30 = 60  M=  r2 =
2r 2
3. W = –MB(cos2  cos1) 1.6 10  2.5 106  0.5 1010
19
 M=
Here, 1 = 0 , 2 = 60 2

W = – 2.5  3  105  (
1
– 1) = 1023 Am2
2
= 37.5  106 = 37.5 J 11. r = 0.5 Å = 0.5  1010 m,
f = 1010 MHz = 1016 Hz
4 I The revolving electron is equivalent to a current
4. B=
mT 2 M = IA = (ef) r2
B=
4  10  8  106
= 1.6  10–3 T  M = 1.6  1019  1016  3.14  (0.5  1010)2
5  102  (2)2 = 1.256  1023 Am2
376

Chapter 11: Magnetic Materials


Distance travelled 
12. time(t) = 22. B = Mz Also, B =
Velocity A
2R  R B 
 t= = R (   2)  = =
v v Mz AMz
q qv 40  104
 I= =  = = 8.3  103 Wb/Am
t R (  2) 4  104  1200
qv R 2 Rqv 
 M=IA=  = 23. B = Mz Also, B =
R(  2) 2 2(  2) A
B 
14. Net magnetic induction B = B0 + Bm  = =
Mz AMz
= 0H + 0Mz
4  105
15.  = (r  1)  = = 2  104 Wb/Am

ns
0.4  104  5000
  = (600  1) = 599
25. Diamagnetic substances are repelled by
16. Relative permeability, magnetic field.
 0.1256
R = =

io
0 4107
26. Repelled due to induction of similar poles.
0.1256 30. On heating, different domains have net
= magnetization in them which are randomly
4  3.14  107

at
distributed. Thus, the net magnetisation of the
1256 104
= = 105 substance due to various domains decreases to
12.56 107 lic minimum.
M net M 1 32. As every atom of a diamagnetic material is not a
17. Mz = = =
V Al 5  10  6 102
4
complete magnet in itself, its susceptibility is
= 3.3  104 A/m not affected by the temperature.

18. % increase in magnetic field 33. As temperature of a ferromagnetic material is


ub
B  B0  H  100
raised, its susceptibility  remains constant first
=  100 = 0 and then decreases.
B0 0H
=  100 = 6.8  105  100 = 6.8  103 34. For paramagnetic substance, magnetisation M is
proportional to magnetising field H, and M is
P

19. Volume of the magnet, positive.


mass 75 103
V = = = 105 m3 36. Magnetism of a magnet falls with rise of
density 75 102
temperature and becomes practically zero at
et

M net 3 Curie temperature.


 Magnetization, Mz = = 5
V 10
37. The volume of the cubic domain is
 Mz = 3  105 A/m
V = (106 m)3 = 1018 m3
rg

20. The bar magnet has coercivity 4  103 Am 1 i.e., Net dipole moment
it requires a magnetic intensity H = 4  103 Am1 Mnet = 8  1010  9  1024 A m2 = 72  1014 A m2
to get demagnetised. Let i be the current carried M net
 Magnetization, MZ =
Ta

by solenoid having n number of turns per metre Domain volume


length, then by definition H = ni. 72  10 14 A m 2
=
Here, H = 4  103 A m1 10 18 m 3
N 60 –1 = 72  104 A m–1
n   500 turn metre
l 0.12 = 7.2  105 A m–1
H 4  103 1
i  8A 38. From Curie’s law,  
n 500 T
21. Magnetic intensity, 2 T  1
 = 1 but it is given that 2 =
H = nI = 500  1 = 500 Am–1 1 T2 1 2
r = 1 +    = (r – 1) and T1 = 273 + 127 = 400 K
 M = H = (r – 1)H = (500 – 1)  500 
1
=
400
= 2.495  105 Am–1 2 T2
 2.5  105 Am–1  T2 = 800 K = (800 – 273) = 527 C
377

MHT-CET Triumph Physics (Hints)


1 3. W = MB cos1  MB cos2
39. When  = 0.5, = 5  103/K
T = MB (cos1  cos2)
1 1000 Here 1 = 0, 2= 90
 T= = = 200 K
5 103 5  W = MB(1  cos 2) = 20  0.25 (1  cos 90)
C =5J
According to Curie’s law,  =
T 4. W = – mB (cos 60  cos 0)
 C = T = 0.5  200 = 100 K 1  1
=  mB   1  mB
42. The real distinguishing feature of ferromagnetic 9  2
material is that intensity of magnetisation I  mB = 2 W
(or B) is not directly proportional to Torque required  = mB sin  = 2W sin 60
magnetising field H.
 3
  = 2W    3 W

ns
43. In the given figure, OQ refers to retentivity while
 2 
OR refers to coercivity, for permanent magnet
both retentivity and coercivity should be high. I
5. T = 2
49. Iron is ferromagnetic in nature. Lines of force mB

io
due to external magnetic field prefer to pass When magnet is cut into two equal halves,
through iron. m I
m = , I =
50.  2 2

at
B1
 Time period
1 2
I
N S S N T = 2
mB
d

M1

M2
lic T = 
 1

2
T
 2 1 
Both the magnets are placed in the field of one
another, hence potential energy of dipole (2) is  T = T
ub
 2M I
U2 = –M2B1 cos 0 = –M2B1 = –M2  0 . 3 1 6. Period of oscillation T = 2
4 d MB
dU
By using F =  , 7.5 10 6
dr  T = 2 = 0.665 s
6.7 10 2  0.01
P

Force on magnet (2) is


dU  2M M
 Time taken for 10 oscillations = T  10 = 6.65 s
F2 =  2 =    0 . 13 2 
d
dr dr  4 d  7. Time period of oscillations is given by,
et

 0 MM I 12  106
= 6 14 2 T = 2 = 2
4 d MB (6  102  2  102 )
 0 6M1M 2
It can be proved F1  F2 = F =  1
4 d4  T = 2 
rg

10
1
 F Now, for 20 oscillations
d4
t = 20  T
B 1
Ta

51. B = 0rH  r  = slope of B- H curve  t = 20 2  = 12.57 s


H 10
According to the given graph, slope of the graph Closest answer in options is 12 s
is highest at point Q.
8. Period of oscillation for a magnet,
I
T = 2
Competitive Thinking MB
2. M = nIA M 1 L2
Where, I =
= 2000  2  1.5  104 12
= 0.6 J/T M = m(2L)
 = MB sin 30 M1 = mass of the magnet
1 As the length is doubled,
= 0.6  5  102  th

And pole strength is reduced to   ,


2 1
2
= 1.5  10 N-m 4  

378

Chapter 11: Magnetic Materials

I M net
T  2 17. Intensity of magnetization =
M B Volume
M net
M1  2L 
2 =
 m  2L  length×area of cross-section
T I M 12 2 12
  = =  = 8 3
T I M M1 L2  m  3 1 =
    4L  3  10 2  2  104
12  4 
= 5  105 A/m
 T = 2 2 T
19. Magnetic field inside a solenoid is given by,
I
9. T = 2 B=nI
MB
= 0 r n I = 0 (1 + ) n I.
1
 T 20.  m    r  1   m   5500  1  5499

ns
B
T B 21. H = nI
 
T B H 3  103
Also 30 osc/min  T = 2 s I  3A
n 1000

io
B 2
 T 2=  2s
B  / A  6  104
2B 2 22. = = = =
H H HA 2000  3  104
q e

at
nh
10. L= and I = =   = 10–3 Wb/Am
2 T 2
Now, M = IA 
23. B = Mz Also, B =
e eR 2 A
R2 =
 M=
2 2
lic  =
B
=

e 2 nh enh Mz AMz
 M= R  =
2 2mR 2 4m 5  105
M enh 2 e  =
ub
 =  = 0.5  104  5000
L 4m nh 2m
= 2  104 Wb/Am
M e 1
11. = =  specific charge of an electron 26. B = (1 + )H
L 2m 2
For paramagnetic materials,  is small and
P

12. The magnetic moment of the revolving electron positive.


neh For diamagnetic materials,  is small and
= n 
eh 
is given by, M = 
4m  4m  negative.
et

Thus, M  n (the principal quantum number). 30. Needle N1 is ferromagnetic. Ferromagnetic


M nIA q materials are strongly attracted by magnet.
13.   Needle N2 is paramagnetic. Paramagnetic
L mvr 2m
rg

materials are weakly attracted by magnet.


M e Needle N3 is diamagnetic. Diamagnetic materials
14. Gyromagnetic ratio, 
L 2m are weakly repelled by magnetic.
e
Ta

 m = 31. Diamagnetic will be feebly repelled.


2 M / L
Paramagnetic will be feebly attracted.
1.6  1019 1 Ferromagnetic will be strongly attracted.
= =  1029 kg
2  8.8  1010 11
M
15. From the relation, susceptibility of the material is 32. =
H
I  M=H
=  I
H = 3  104  4  104
Thus, greater the value of susceptibility of a = 12 A/m
material greater will be the value of intensity of
magnetisation i.e., more easily it can be 1
33. 
magnetised. T
1 T2
16. Magnetization is given by, MZ =
CBext    1T1 = 2T2
T  2 T1

379

MHT-CET Triumph Physics (Hints)

1  T 2 200
34.   1 2 
T 2 T1 0.0075 100
1.0  105 2 = 0.0150
 T2 =  (273 + 27)
1.5  105 39. Even in the case of a permanent magnet all the
= 200 domains are not perfectly aligned due to thermal
K = 73 C agitations.
1  T 40. Diamagnetic material is repelled by magnetic
35.   2 1
T 1 T2 field. This magnetic field energy of current
2 273  73 sources will be converted into potential energy
 of the rod which is set up by switching on the
0.0075 273  173
current source.

ns
Hints to Evaluation Test

1.  = MB sin  7. The magnetic field inside the toroid in the

io
= 200  0.25  sin 30 absence of tungsten, B0 = 0H
  = 25 N-m When filled with tungsten, B = 0(1 + )H
The increase in field = B – B0

at
2. W = –MB(cos2  cos1) = 0H
Here, 1 = 0 , 2 = 60 The percent increase in the magnetic field
1 B  B0
W = – 4.5  2  105  ( – 1) =  100
2 B0
= 45  106 = 45 J
lic 0H  100
=
0 H
I
3. Period of oscillation T = 2 =   100
MB
ub
= 4.6  10–5  100
 T = 2
5.5106
= 0.687 s = 4.6  103
4.6102  0.01
8. The relative permeability of the rod is given by,
 Time taken for 10 oscillations = T  10 = 6.87 s R = 1 + m = 1 + 599 = 600
P

4. Magnetic field lines avoid passing through  The permeability of iron =  = 0R
diamagnetic materials. Due to this reason, the   = 4  107  600
bar of diamagnetic material aligns perpendicular B = H = 4  107  600  800
et

to the magnetic field  B = 192  103


Magnetic field lines prefer passing though the  The magnetic flux produced in the coil,
paramagnetic materials. So, the bar of  = BA = 192  103  1  105
paramagnetic material aligns parallel to the   = 192  3.14  108
rg

magnetic field.  6  105 Wb


I 9. The bar magnet has coercivity 4  103 Am–1 i.e.,
5.  (susceptibility) =
H it requires a magnetic intensity H = 4  103 Am–1
Ta

For paramagnetic substances, to get demagnetised. Let i be the current carried


0 <  < E, where E is a small positive number. by solenoid having n number of turns per metre
Hence I vs H graph is a straight line with a length, then by definition H = nI.
small positive slope i.e., graph III. Here, H = 4  103 Ampere turn metre–1
N
6. Magnetic intensity H = nI = (500)(1) n=
l
= 5  102 Am1 50
Magnetization MZ = (B  0H)/0 =
0.10
= (r0H  0H)/0 = 500 turn metre–1
= (r  1)H H
 I=
= (350  1) (5  102) Am1 n
= 1.75  105 4  103
= = 8.0 A
 1.8  105 Am1 500

380

Chapter 11: Magnetic Materials

10. Net dipole moment is, Mnet = MZ  V. M net


15. Mz =
Volume of the cylinder V = r2l, where r is the V
radius and l is the length of the cylinder, then M
=
dipole moment, Al
Mnet = MZr2l 1
22
=
= (4.2  10 )   (0.6  10–2)2  (4  10–2)
3 6  10  4102
4

7 = 4.2  104 A/m


–2
 Mnet = 1.9  10 J/T
11. In paramagnetic substances, intrinsic magnetic
moment is not zero. Further, in the absence of
external magnetic field, spin exchange
interaction is present.

ns
12. Mean radius = r
68
= = 7 cm
2

io
= 7  102 m
 Number of turns/length,
N 1500
n=   3412.19

at
2r 2 7 102
As B = ni, where B = 2 T and i = 0.5 A
B 2
 = 


ni 3412.19  0.5
 = 11.7  104 Tm A1
lic
 11.7 104
r =  = 931.5
0 4107
ub
13. B = 0 (H + I) where, I be intensity of
magnetization.
B
 I = H
0
P

H
= H
0
et

= r H  H
= (r  1) H
For a solenoid of n turns per unit length carrying
current i; H = ni.
rg

 I = (r  1) ni
Here, n = 6 turns/cm = 600 turns/m
I = (900  1)  600  0.4
Ta

 I  2.16  105 Am1


As magnetic moment, M = I  V
 M = 2.16  105  104
= 21.6 Am2
14. From the relation susceptibility of the material
is
I
=
H
I
Thus, greater the value of susceptibility of a
material greater will be the value of intensity of
magnetisation i.e., more easily it can be
magnetised.

381
12 Electromagnetic Induction

Hints

21. Only a.c. dynamo has slip rings.


Classical Thinking

ns
 y
35.  = LI  L = = henry
1. The magnitude of induced e.m.f is directly I x
proportional to the rate of change of magnetic
flux. Induced charge doesn’t depend upon time. 36.  = LI = 5  103  2 = 0.01 weber

io
Since e  B, so by reducing magnetic field to  10  106
5. 37. L= = = 4  103 H = 4 mH
half, induced e.m.f. will also be reduced to half. I 2.5  103

38.  = LI = 2  5.8 = 11.6 Wb

at
d 240
6. |e|= = =2V
dt 2  60 dI
39. e=L =  (2)  (0.5) = + 1 V
3
d 3  10  2  10 3 dt
7. |e|  
dt 25
= 0.04  103 = 0.04 mV
lic 40. Inductance of coil,
e 8
L= = = 0.2 H
9. The energy of the field increases with the dI  8 4 
 
magnitude of the field. Lenz’s law infers that dt  0.1 
ub
there is an opposite field created due to increase dI
or decrease of magnetic flux around a conductor 41. e = L = 5  2 = 10 V
dt
so as to hold the law of conservation of energy.
e 5 5
dB 42. L= = =  103 H = 5 mH
P

11. | e | = nA = 100  102  103 = 103 volt  dI   (3  2)  1


dt    3 
 dt   10 
15.  = BAcos = 5  102  0.2  cos 60
= 5  103 Wb dI |e| 220
et

43. |e|  L L= = = 11 H


dt  dI   10  0 
16.  = nBA cos     
 dt   0.5 
 Plane of the loop is at right angles to the field.
rg

44. n = LI
  = 90
n 500  4  103
  = 1  4  103  0.4  cos 90 = 0 L=  = 1 henry
I 2
17. e.m.f. induced between ends of conductor,
Ta

dI
e = Blv = 5  10–3  1.5  5 = 37.5  10–3 V 45. | e | L
dt
18. e L  10   10  
 1=  L  25 mH
19. With the increasing speed,  increases. Thus 0.5
current reduces due to increase in the back
1 2 1
e.m.f. 46. Energy stored E = LI =  50  10–3  4
Also, according to Ohm’s law, 2 2
Ve = 0.1 J
I=
R
and e   47. uB =
B2
=
 2 10  = 159.2 J/m3
2 2

20 2   4  107 
Hence, as  increases, e increases reducing
current.
49. Induced current is produced in secondary coil,
20. Commutator converts a.c. into fluctuating dc. hence main current remains same.

382

Chapter 12: Electromagnetic Induction


es e.dt 1000  0.01 9. If the current increases with time in loop A, then
51. M= = = =5H
 dI p  dI 2 magnetic flux in B will increase. By Lenz's law,
  loop-B will be repelled by loop-A.
 dt 
10.  = BA = 103  102 = 10 weber
dI p
52. |es| = M
dt 2  1 B A BA
11. |e|= = 2 2 1 1
3 t t
 15  103 = M   M = 0.05 H
10 22
1.8  (100  104 )  1.0  (  49  104 )
= 7
dI 5 0.1
53. e = M =4 = 30 kV
dt  1  = 26 mV
 
 1500 
n(B2  B1 )Acos 

ns
12. e= 
Critical Thinking t
50  (0  2  102 )  100  104  cos0o
2. As  through coil is constant and there is no  t=
0.1
relative motion between magnet and coil,  t = 0.1 s

io
neither e.m.f. nor current is induced in coil.
13. d = nAB = 10  4  102  102
 n(2  1 )
3. e= = 4  103 Wb

at
t
d 4  103
50(1  106  31  106 )  |e|= = = 8  103 V = 8 mV
= dt 0.5
0.02
= 7.5  10–2 V dB
lic 14. | e | = nA
dt
 d 
  = 100  50 10–4 
 0.1  0.05  0.5 V
e 1 d
=   =
dt
4. I= (4t2 – 4t +1) 0.05
R R R dt
ub
8t  4 8  (1 / 2)  4 d dB
 I= = =0 15. e=– = – nA
R 10 dt dt
dB gauss tesla
d d Now, = 108 = 104
5. e=– = – (6t2  5t +1) dt s s
P

dt dt
 e = –10  10–3  104 = – 100 V
=  (12t  5)
 e = 100 volt (numerically)
As t = 0.25 s, e =  [12 (0.25)  5]
e 100
=  (3  5) = 2 V  I= = = 5 ampere
et

R 20
e 2
 I= = = 0.2 A
R 10
16. |e| =
d BdA
  B
 r 2  L2 
dt dt dt
rg

d
6. e=   2

dt  44 
0.2        222   104
= (10t  4)    
=
 e =  (10  0.1  4) = 3 volt 0.4
Ta

= 6.6  103 V
d d
7. |e|= = (5t2 + 3t + 16) = (10t + 3)
dt dt d d dB B
17. |e|   (BA)  A  =1 = 5B
When t = 3 s, e3 = (10  3 + 3) = 33 V dt dt dt 0.2
When t = 4 s, e4 = (10  4 + 3) = 43 V
d dA d 2 dr
Hence e.m.f. induced in fourth second 18. |e| = B  B (r ) = 2Br
dt dt dt dt
= e4  e3 = 43  33 = 10 V
 |e| = 2  0.04  2  102  2  103 = 3.2  V
8. The energy of the field increases with the
magnitude of the field. Lenz’s law infers that 20.  = 90  30 = 60, 1 T = 104 G
there is an opposite field created due to  = nAB cos 
increase or decrease of magnetic flux around a   = 100  (  104)  (106  104)  cos 60
conductor so as to hold the law of conservation 1
= 100    102  = 0.5 Wb
of energy. 2
383

MHT-CET Triumph Physics (Hints)


0 IA If in time t the rod turns by an angle , the area
21.  = BA cos t = cos t
2R generated by the rotation of rod will be
4  3.14  107  10  104 1 1
= cos t = 109 cos t  l  l   l 2
2  0.628 2 2
So the flux linked with the area generated by the
22.  = nBA = 103  102  104 = 103 weber
rotation of rod
23.  = nAB cos  = 1  0.5  4  cos 60 1  1 1
1   B  l 2  cos0  Bl 2  Bl 2 t
= 2  = 1 weber  2  2 2
2
d d  1 2  1 2
and so e    Bl t   Bl 
24. Since the magnetic field is uniform, the flux  dt dt  2  2
through the square loop at any time t is constant,
because 31. As the magnet moves towards the coil, the

ns
 = B  A = B  L2 = constant. magnetic flux increases (nonlinearly). Also
d there is a change in polarity of induced emf
 e = = zero when the magnet passes on to the other side of
dt
the coil.

io
25. e = Blv  IR = Blv
32. 2 = 377   = 60.03 Hz
IR
 v =
Bl 33. e = IR

at
Il  l  440 1
= ....  R    I= = A
BA.l  A (4000  400) 10
I 3  10 3  9  106  Voltage across the load, V = IR
= =
BA
27  10 9
2  1.8  107
3
lic =
1
10
 4000 = 400 V
= 8
=  101 = 0.075
36  10 4 34. For the A.C. generator,
 v = 7.5  102 m/s Flux  = NAB cos t and I = I0 sin t
ub
26. e = vt Bl = (v sin ) Bl = vBl sin 30 When I = 0, sin t = 0 or t = 0 and then
1  = NAB cos 0 = NAB
= 10  0.5  1  = 2.5 V
2 = 1000  2  0.2 = 400 Wb
Note: When I = 0, then  is maximum.
P

27. E = Blv sin


35. B = BA cos 
v sin 
v where  is the angle between normal to the
plane of the coil and magnetic field.
et

Induced e.m.f.,
 v cos 
 e = BA sin 
l
 = 0 ….[Given]
28. The magnitude of induced e.m.f. is given by
rg

 Magnetic flux is maximum and induced e.m.f. is


| e | = Blv zero.
v = 300 m/min = 5 m/s
|e| 2 36. e0 = nAB = nAB.2f = 2(nA)  B    f
 B=  = 0.8 tesla.
Ta

lv 0.5  5 22
= 2  2 7  105   100 = 88 mV
 7
29. Potential difference between B

1 37. e0 = nAB = 2fnAB
O and A is V0  VA  Bl 2
2 2000
=2  50  80  104  0.05
1 2 60
O and B is V0  VB  Bl 
2 4
A B = V
O 3
30. B

     38. e0 = 2 f nAB
       600  4 4
= 2    (5000)(50  10 )  8 10
A P  60 
   l  = 12560  104
     = 1.256 V

384

Chapter 12: Electromagnetic Induction


39. e0 = nBA = (2f)nBA dI dt
48. |e|=L L=e
= 2  3.14  100  5000  0.2  0.25 dt dI
= 157 kV dI = 2 – (–2) = 4 A
8  0.05
40.  L= = 0.1 H
4
dI
B V 49. |e|=L or L  dt
dt
L1 dt1 5
   = 100 : 1
L 2 dt 2 50  103
The induced emf between A & B = E = Blv
The induced current = i = E/R 0 N 2A
50. L=
i = Blv/R l

ns
B 2l 2 v 2 where N is the total number of turns.
Electrical power = P = i2R = As L  N2
R 2
Since v is doubled, the electrical power becomes L2 N 
 =  2  = (2)2
four times. Since heat dissipation per second is

io
L1  N1 
proportional to electrical power, it becomes 4  L2 = 4L1
times.
0 N 2A

at
41. The emf induced in the rod of length 0.5 m is 51. L= or L  N2
l
e = Bnvl = 0.50  4  0.5 = 1 volt 2

The free electrons of rod experience force along L1  N1 


  = 
L2  N 2 
BA therefore end A becomes negative and end
lic
B becomes positive. That is the direction of the 108  600 
2

  
induced emf is from B towards A. L 2  500 
The current in the circuit ABCD,  L2 = 75 mH
e 1
ub
i= = =5A 52. Let 1 = 2 = 
R 0.2
The force required to maintain the motion  
 L= I=
= ilB = 5  0.5  0.5 = 1.25 N I L
Mechanical work done by the force per second  

P

I1 = , I2 =
or mechanical power L1 L2
= Fv = 1.25  4  1 = 5 watts  
 
I1 L L 2  103 1
=  1 = 2 =
et

42. emf = Blv = Bl


l  =
2 I2    L1 8  103 4
 
L
 2
(emf ) 2
Work done in pulling out = t
rg

R 53. L = 0nI
l2
2 2 L2 
Bl 2   ….( n and I are same)
= 2  2 = 3.125  103 J L1  0
R
 L2 = rL1 = 900  0.18 = 162 mH
Ta

dI
44. e=L  L = volt-s/ampere 54. U=
1 2
LI
dt
2
2
45. N = LI U2 I 
2

=  2     
1 1
i.e.,
LI 8  10 3  5  103 U1  I1   2  4
 = =
N 400 1

 U2 = U1
= 107 = 0 Wb 4
4
di 2  0.5
55. emf = L = = = 50
nd LdI dA LdI dt 0.03
46.   nB 
dt dt dt dt 1 1
Estored = Li2 =  50  0.52
11 5  2 1  2 2
  L 3 
 L  10 H
103  2  10  = 25  0.25 = 6.25 J

385

MHT-CET Triumph Physics (Hints)


56. Magnitude of induced flux is higher at the Also, IPVP = ISVS
centre than at the edges. Therefore the flux VS
 IP = IS = 8  5 = 40 A
linkage per unit current (which is equal to VP
inductance) is higher at the centre than at the
edges. LM >Le N S VS
65. =
N P VP
B2
57. E= . Hence a graph between E and B will 1 V
2 0  = S
20 2400
be a parabola symmetric about E axis and
 Vs = 120 V
passing through origin.
For 100% efficiency, VSIS = VPIP
58. As I increases in outer loop,  increases  120  80 = 2400 IP
 Current in the inner loop is such that it opposes  IP = 4 A

ns
the increase in . Hence  decreases (By Right 66. For 100% efficient transformer, VSIS = VPIP
Hand Rule). The induced current will be counter
VS I P N S
clockwise.  = =
VP IS N P
dI I I IP 25
e = M = M  2 1

io
59.  =
dt t 4 100
05  IP = 1 A
= 4  3 = 2  104 V
10

at
68. PP = PS = ISES
60. Maximum induced e.m.f., PS 2000
 IS =  = 10 A
dI ES 200
e = M
dt
d
lic Now,
NS
NP
I
= P
IS
= 0.01  (5 sin 200 t)
dt N P I P 1000  0.1
 NS = = = 10
= 0.01  5  200  cos 200 IS 10
ub
= 0.05  200 ….( cos 200 = 1)
eS NS 1500
= 10 V 69. Using,   we get,
ep N P 50
61. VP = VI = 300 volt, eS = 30 eP
VS = Vo = 15 kV = 15  103 volt
P

d
NP V 300 2 1
Now, | ep | = = 4 volt
 = P = = = dt
NS VS 15  103 100 50  eS = 30  4 = 120 V
et

VS N N Pout VI
62. = S  VS = S  VP 70. n% =  100 = S S  100
VP NP NP Pin VP I P
500 2  440
=  220 = 1100 volt  0.8  100 =  100
rg

100 220  I P

NS V  IP = 5A
63. = S
NP VP Vs I p
71. Using, 
Ta

200 V Vp Is
 = S
100 120 11000  2
 VS = 240 V  Ip   100 A
220
VS I
= P d
VP IS 73. e = n
240 10 dt
 = e n d
120 IS  i= 
R  R  dt
 IS = 5 A
R 3R
Given, R +  ,
VP N P 500 1 2 2
64.   
VS N S 2500 5 d = 2  1, dt = t
200

2n  2  1   2n  1  2 
 VP = = 40 V i=
5 3R t 3Rt

386

Chapter 12: Electromagnetic Induction


74. Considering that the electron is moving from 83. When magnet falls through ring, there is change
left to right, the flux linked with the loop of flux associated with the ring. It produces
(directed into the page) will first increase and induced e.m.f. and hence induced current. By
then decrease as the electron passes by. Hence Lenz’s law, the current flows in such a direction
the induced current in the loop will be first so as to produce an induced e.m.f. which
anticlockwise and will change its direction as opposes the falling magnet. Acceleration of
the electron passes by. magnet is less than acceleration due to gravity.
75. Induced electric field is non-conservative. 84. When there is a cut in the ring, e.m.f. will be
Also we have, induced in it but there is no induced current in
  d   the ring. Hence there is no opposition to falling
 e  dl dt 
=  e  ds  0
magnet. Therefore, acceleration is equal to ‘g’.

76. e = Blv = 0.15  0.5  2 = 0.15 V 85.  = nBA cos  = 10 Ba2cos t

ns
e 0.15 d d 2 2
 I= = = 0.05  e=  (10Ba cost) = 10 Ba sint
R 3 dt dt
 F = BIl = 0.15  0.05  0.5 = 3.75  10–3 N 86. Comparing given equation with the standard
form,

io
77. Component of the length perpendicular to the
field l = l sin 60 e = e0 sin t we get,
e = 200 sin 100
 3
= 1.0    = 0.5 3 e0 = 200,  = 100 

at
 2  Now, e0 = nAB
 e = l Bv = 0.5 3  0.5  10  B=
e0
= 4.3 volt An

78.
lic
The e.m.f. induced is directly proportional to =
200
= 0.01 T
(0.25  0.25)  1000  100
rate at which flux is intercepted which in turn
varies directly as the speed of rotation of the 87. It is evident that when viewed from the magnet
generator. side, the induced current will be anticlockwise.
ub
e2 f 2
 ef  S
e1 f1
120 N
 f2 =  1500 r.p.m. = 1800 r.p.m.
100
P

79. F1 = mg
When bar is just ready to levitate,
IlB = mg
et

88. B = 1.25 mT = 1.25  103 T


mg
I= e = Bl
lB
 The mechanical power required,
1  10
 I= = 20 T
rg

P = eI = BlI
1  0.5
= 1.25  103  0.1  1  50
E = IR = (20)25 = 500 V = 6.25  103 W = 6.25 mW
d e = nBA sin t
Ta

80. dq = = I dt = Area under I – t graph 89.


R 0I
 d = R  (Area under I – t graph) Given that, n = 1, B = , A = a2
2b
1 0I
= 10 
2
 4  0.1 = 2 weber  e=
2b
 a 2  sin (t)
d 90. Magnetic field produced due
81. At B, flux is maximum, which means =0
dt  0 8 2I
d to large loop B 
As,  e   e  0 4 L i
dt Flux linked with smaller loop
l
82. If a horizontal straight conductor placed along  0 8 2Il 2
  B(l 2 ) 
N-S falls under gravity, then there is no induced 4 L L
e.m.f. along the length of the conductor as there   0 8 2l 2
l 2

is no change in flux.    MI  M   . M


I 4 L L
387

MHT-CET Triumph Physics (Hints)


d dB
8. |e|  A
Competitive Thinking dt dt
3 
1. When e is coming towards the loop, magnetic  B  3AB
flux of one type increases and when going  | e |  A 4  
away, the same magnetic flux decreases. So  2  8
 
induced current opposite will reverse its
direction as e– goes past the coil. 9. =n×A×B
d dB
d d  e= = nA
2. |e| =   3t 2  4t  9  = 6t + 4 dt dt
dt dt
(0.6  0.2)
at t = 2 sec, = 200 × 0.15 ×
0.4
|e| = 16 V
= 30 V

ns
d
3. e =  = (100t) d
dt 10. e=
dt
At t = 2 s,
 = B.A
e 100  2

io
I=  = 0.5 A Here A = r2 as magnetic field is restricted to
R 400
region of radius r.

4. 1 = 4  104 Wb  2
e =  r .
dB
in loop 1

at
2 = 0.1 1 = 0.4  104 Wb dt
 d = |2  1| = 3.6  104 Wb As the loop 2 is outside the region of magnetic
dt = t second
lic field, e = 0 for loop 2.
d 11. e = Bvl = 0.1 × 15 × 0.1 = 0.15 V
e=
dt (Considering B, l and v are mutually
3 3.6  104 perpendicular.)
 0.72  10 =
t 12. Induced emf, e = Blv
ub
4
3.6  10 = 5  10–4  0.1  5
 t=
0.72  103 = 2.5  10–4 V/s
 t = 0.5 second
13. e =Bvl
 n  2  1  d = 5.0  105  1.50  2
P

e
5. I= = …(e =  n )
R R t dt = 10.0  105  1.5
 n  2  1   n  2  1  = 15  10–5
= =
(R  4R) t 5Rt = 0.15 mV
et

d 14. e = Blv sin 


6. |e| = = B(2r)v sin 
dt
rg

d 5
iR = 15. v = 1080 km/hr = 1080  = 300 m/s
dt 18
 d = R  I dt Induced emf
This means e = B l v = 1.75  105  40  300 = 0.21 V
Ta

|d| = Resistance  area under current – time


graph 16. The e.m.f is induced when there is change of
flux. As in this case there is no change of flux,
1
= 100   10  0.5 = 250 Wb hence no e.m.f. will be induced in the wire.
2
17. For a wheel rotating about an axis passing
7.  = (5t2  4t + 1) Wb through its centre and perpendicular to the plane
d of wheel,
 = (10t  4) Wbs1
dt Br 2 0.1  10   0.5 
2

d e =  = 0.125 V
e= =  (10t  4) 2 2
dt
Br 2 0.2  10   2 
2
At, t = 0.2 s, e =  (10  0.2  4) = 2 V
18. e= 
e 2 2 2
 I=   0.2 A
R 10  e=4V
388

Chapter 12: Electromagnetic Induction


19. Given 1 2
30. Energy stored in inductor, E = LI
 = constant 2
 0 v  2E 2  25  10 3
 vavg =    L=  = 13.89 H
 60  103 
2
 2  I2
Emf induced between axle and rim of the wheel
is; 31. Given: I = I0et
e = Blvavg dQ  dQ 
 = I0 et ....   I  
Blv B l  r dt  dt 
= =
2 2 Q t 

  dQ  I  e
t
Bl 2  0 dt
 e= ….( r = l) 0 t =0
2
 1 1 
 Q = I0    0 

ns
20. e = Bleff v (where leff = Diameter)  e e 
= B(2r)v I0
= 2rBv  Q=

and R is at higher potential by Fleming’s right

io
hand rule. di
32. E= M
dt
The induced emf e = l .  v× B 
 
21. d(5sin10t)
  E = 2  10–2

at
dt
= l(vB sin ) = Bvl
= 2  10–2 5(cos 10t)  10
e
Induced current, i = Emax = 2  10–2  5  1  10 = 
R

wire, resistance R remains unchanged.


lic
If the straight wire is replaced by a semicircular
33. e=
MdI
dt
Hence, current remains same. d
e=M (Im sin t)
22. Induced emf e = NBA sin t dt
ub
For maximum value of voltage or emf, d
Now, (Im sin t) = Im cos t
sin t = 1 dt
 e0 = NBA dI
For maximum value of emf, is maximum
= 100 × 0.3 × 2.5 × 60 dt
P

= 4500  cos t = 1
= 4.5 × 103 volt 
dI
= Im
= 4.5 kV dt
et

 e = 0.005  10  100  = 5
23. Maximum emf
e0 = NBA 34. Q = M IP
= 100 × 0.01 × 2 × 102 × 2 × 3.14 × 50 But
rg

= 6.28 V dIQ
|ep| = M
dt
25. Time varying magnetic field gives rise to eddy
e  dt
currents in accordance with Lenz’s law. M= P
Ta

dIQ
27. Given: N = 1000; I = 4A;  = 4  10–3 Wb. e P  dt
 total magnetic flux linked with solenoid = N  Q =  IP
dIQ
N
Self inductance, L = ….(  = LI) 15  103
I P =  1.8
10
1000  4  103
 L= =1H P = 2.7  103 Wb = 2.7 mWb
4
35. As efficiency is always less than unity in
28. I practice, output power is less than the input
If solenoid is pulled out then flux decreases power.
resulting into decrease in the value of current.
Poutput 100 100 10
1 1 36. =  100     90%
Pinput 1 110 11
29. U = LI2 =  2  1 = 1 J 220 
2 2 2

389

MHT-CET Triumph Physics (Hints)


37. Given: Vp = 220 V, Vs = 3.3 × 103 V 45. Given that, coil is kept perpendicular to
Np = 600, P = 4.4 × 103 W magnetic field B = 5  105 T.
Power, P = VS Is Thus, angle between area vector A and magnetic
P 4.4  103 4 field B is,  = 0
 IS = = = A
Vs 3.3  103 3  e.m.f. induced,
d BAcos  800  5  105  0.05
P VI e =N =N =
38.  = out = s s = 0.8 dt dt 0.1
Pin Vp Ip = 2  102
(440)(2) = 0.02 V
 Ip = =5A
(0.8)(220)
46. The voltage equation in going from point
P A to B is
39. = o di

ns
Pi  IR + E  L – VAB = 0
dt
80
 Po = Pi =  4  103 W  VBA =  2  2 + 12  (5  103  102)
100
But Po = es Is  di 
 isdecreasing hence rate is negative 

io
 es Is = 0.8  4000  dt 
0.8  4000  VBA =  4 + 12 + 0.5 = 8.5 volt
 Is =
240 dI

at
 Is = 13.33 A 47. VAB – IR + E + L =0
dt
40. Transformer works on A.C. alone which  VAB = (2) (7) – 4 – (9  10–3) (103)
changes in magnitude as well as in direction. = 14 – 4 – 9

41.
N s Vs

lic  VAB = 1 V
N p Vp d
48. E= 
dt
50 V
  s d  B.A 
ub
1000 220 E= 
dt
 Vs = 11 V
Now, VsIs = VpIp dB d 0 I
E = A = A
 11  Is = 220  1 dt dt 2  vt 
0 d –1
P

 Is = 20 A  AI (t )
2v dt
VS N I
42. = S = P 
VP NP IS  AI 0 t–2
2v
et

N S IS 25 1
i.e., IP = = × 2 = 50 A E 2
NP 1 t
90 Area of square loop, A = 10 cm  10 cm
rg

43. Power output = 3  = 2.7 kW 49.


100 A = 100 cm2 = 100  104 m2 = 102 m2
Ip = 6 A Initial magnetic flux linked with loop,
 VS =
2.7  103
= 450 V and Ip =
3 103
= 15 A 1 = B1Acos  = 0.1  102  cos 45
Ta

6 200 0.1  102  1 103


=  Wb
2 2
44. P = VrmsIrms
Final magnetic flux linked with loop,
12 = 48  Irms
12 1 2 = 0 Wb ….[ B2 = 0]
Irms = = A
48 4  The induced e.m.f. in the loop,
I  103 
Irms = 0 0  
d (   ) 2 
 2 1 =  
2
e =
I0 = Irms ×2 dt t 0.7
3
1 10
I0 = × 2 =  10 3 V
4 0.7  2
1 e 103
I0 = A  I=  = 103 A = 1.0 mA
2 2 R 1

390

Chapter 12: Electromagnetic Induction


50. Force on the strip when it is at stretched position 3l 3l 3l
 l2  5Bl 2
x from mean position, 53. e =  Bvdl =  B(l )dl = B   =
F = – kx – il B 2l 2l  2 2l 2

    
54. Induced emf e =  Blv
kx
     
      x
B      iBl P + S
+
    
x a v
x v
Since, i is induced current, 2
e Blv
i= = 
R R Q  R

ns
Blv a a
 F = – kx – × lB 2 2
R
a
B 2l 2 x
 F = – kx – ×v 2
R

io
Above expression shows it is a case of damped εPQRS = εPQ + εRS
oscillations. 0I 0 I
= av  av
Comparing it with,  a   a
2  x   2  x  

at
F = – kx – bv,  2  2
B 2l 2 0 Iav  2 2 
b= = 
R 2  2x  a 2x  a 

For damped oscillations, amplitude is given by,
A = A0e–bt/2m
lic =
 0 Iav  2x  a  2x  a 

  (2x  a)(2x  a) 

A0
 For A = = A0e–bt/2m 2 0 Ia 2 v
e =
 bt (2x  a)(2x  a)
ub
= –1
2m 55. e
 B 2l 2 
 t
  R  =1 +
t
P

2m
2mR 2  50  103  10 –
 t= 2 2 
 0.1   0.1
2 2
Bl
e = nAB sint
et

 t = 104s
 e changes direction twice per revolution.
m 50  103 2π
Now, T = 2π = 2π  56. Current passing through the solenoid I(t)
k 0.5 10
= I0t(1  t)
rg

 Number of oscillations,
Magnetic field B at the centre of the solenoid is
t 10 4  10
N=   5000 B = µ0nI(t) = µ0nI0t(1  t)
T 2π
Area of the small ring placed co-axially at the
Ta

51. eo = io  XL centre of the solenoid is A = (2R)2 = 4R2


XL = L = 2fL = 2(50) = 100  Flux passing through the small ring is given by
2  = BA = µ0nI0t(1  t) 4R2
io = ampere
π = 4R2 µ0nI0 (t  t2)
2 Induced emf in the small ring is given by Lenz
 eo =  100 = 200 V
π law.
d d 2 2
52.  = BA |E| =  [4R µ0nI0 (t  t )]
dt dt
 = (B) (r2) = 4R2 µ0nI0 (t  2t)
d  dr  If E = 0, 1 2t = 0, t = 0.5 s
 e= = (B) (2r)  
dt  dt  dE dE
Also, = 8R2 µ0nI0t, < 0 for all t
= (0.025) (2) (2  102) (103) dt dt
=  V  the induced emf/current has reverse direction
391

MHT-CET Triumph Physics (Hints)

Hints to Evaluation Test

1. Magnetic field at the centre VP N P


9. =
3 3 1  VS N S
0 I
=   
L  3 2  220 
 NP =    2000
 2200 
d dB
Emf, |e| = = A = 200
dt dt
 d   I  3  3 1   10. Here, B is constant and radius r is linearly
= (r2)   0
     changing only during time interval 5 to 10
 dt  L  3
 2   
units.

ns
0  3  3 1  d d
 e = r2    (I0 et ) Using, e = (B r2)
L  3 2  dt dt
 dr 
0  r 2  3  3 1  = (B)  2r

io

=    I0  et  dt 
L  2 2
Hence during this period, the emf is as shown in
 I r 2  3  3 1  t (D).

at
= 00    e
L  3 2
11. Assertion and Reason both are correct and
d reason is correct explanation of assertion
2. |e|=
dt
lic because e = L 
 di 

 dt 
1  0.1
=
0.1 12. Rate of work done by external agent is:
0.9
ub
= =9V dW BIL(dx)
0.1  = BIL v and thermal power
dt dt
e 9 dissipated in resistor = eI = (BvL) I
 I= = = 0.09 A
R 100 Clearly both are equal. Hence (A) is correct.
P

If applied external force is doubled, the rod


dI
3. From e = L will experience a net force and hence
dt
acceleration. As a result, velocity increases,
e = (5  103)
 4  0  = 20 V
et

3
hence (B) is correct.
10
e
Both the statements are true and statement 2 is Since, I =
R
correct explanation of statement 1.
On doubling ‘R’, current and hence required
rg

 200 power becomes half. Hence (D) is correct.


4. M=  = 40 H
I 5 1
Since P = BIl v and I 
R
Ta

5. Statement 1 is false, mutual induction of two Hence option (C) is incorrect.


coils is in addition to their self induction.
Statement 2 is however true.  Bl 2 Bl 2 
  
2 2 
dI 13. i = 
6. e=L R
dt
B l 2
(5  3) =
= 1  10–3  R
103
=2V

8. Statement 1 is true, but the statement 2 is false. Bl 2 Bl 2


The two laws have different statements, through 2 2
the result of their application is the same.

392

Chapter 12: Electromagnetic Induction


14.  = MI
 0.8
M=  = 0.2 H
I 4

Es Ns
15. 
Ep Np

Ns
 Es =  Ep
Np

1000
=  20
100
= 200 V

ns
The frequency, in a transformer, remains
unaffected.

As  = MI

io
16.

 M=
I

at
6  102
=
0.03
=2H

17.
lic
Mutual inductance of the pair of coils depends
on relative position and orientation of the two
coils. This is in addition to other factors not
ub
given in the alternatives.

dI 2  2
18.  = 80 A/s
dt 0.05
P

dI
As e = L
dt
 16 = L (80) ; L = 0.2 H
et

19. As  = NAB cos  = NAB cos t


d
 e=
rg

dt
d
=  (NAB cos t)
dt
Ta

= NAB(sin t)
emax = NAB  sin 90
= NAB 

20. M= L1L 2

= 49
= 6 mH

21. A thin aluminium disc spinning freely is


brought to rest because of eddy currents induced
in the disc.

393
13
-
AC Circuits

Hints

27. e = 100 sin (100t) and I = 100 sin(100t)

ns
Classical Thinking Comparing these equations with the standard
forms,
9. Ipeak = I0 = Ir.m.s  2 = 10 2 A e = e0 sint and
I = I0 sint we get,

io
10. e = 100 sin (100 t + 0.6) e0 = 100 V and
Comparing with the standard form, I0 = 100  103 A
e = e0 sin (t + ) we get, e0 I 0
P = e rms Irms = 

at
Peak volt = e0 = 100 V 2 2
100 100  103 10
12. e0 = erms  2 = 220  2  311 volt =  = =5W
2 2 2

13. er.m.s. =
e0
=
423
 300 V
lic 40. At resonance, VL = VC
2 2  VT = VR = 100 V
e0 141.4 1
14. erms =  = 100 V 41. Using, fr =
ub
2 1.414 2 LC
1
18. The reactance of the circuit is zero either when =
the circuit contains only resistor or the circuit 2   100  106  4  108
contains an inductor and capacitor connected in 1
=
P

such a way that their individual reactance cancel 2  2  106


out. 106 25
= =  104 Hz
4 
21. XL = L
et

XL 1 42. Refer Notes 12


 =  = 1000
L 103
Critical Thinking
rg

22. Impedance of circuit, Z = XC


1 1 1. General equation for instantaneous e.m.f. is,
 Z= =  63.7 
2fC 2  50  50  106 e = e0 sin (t + ) = 200 sin (2 50t)
= 200 sin (100 t)
Ta

1 1
23. XC = = 2. In D.C. ammeter, a coil is free to rotate in the
C 2fC
magnetic field of a fixed magnet. If an
 XC =  ....[f = 0 for D.C]
alternating current is passed through such a coil,
1 1 the torque will reverse its direction each time
24. XC = C= the current changes direction and the average
2fC 2fX C
value of the torque will be zero.
1
 C= = 0.5  104 = 50 F 3. The instantaneous current in a circuit is,
400
2   25 i = sin (t + )

As i = i0 sin (t + )
XL  XC 300  200 100  i0 = 1 A
25. tan  = = =
R 100 100 i0 1
 irms = = A
 tan  = 1   = 45 2 2

394

Chapter 13: AC Circuits

erms e0C 100 2  100  0.5  106 1


4. irms = = = 14. XC 
XC 2 2 f
X C2 f1 1
= 5  10–3 A = 5 mA  = =
X C1 f2 2
erms 200
5. irms = = =5A 10
R 40  X C2 = =5
2
i0 = 2 irms = 1.414  5  7.1 A
 1 
6. Time taken by the current to reach the  L  C 
1
15.  = tan  
T 1 1 R
maximum value t =  = = 5  103 s  
4 4f 4  50  
and i0 = irms 2 = 10 2 = 14.14 A  1 
2fL 
1  2 fC 

ns
= tan  
e0 2fnAB  R 
7. erms = = = 2 fnAB
2 2  
 1000  4 4  2  
= 2    2  50  
  50  (30  10 )  5  10 1   2   50  106 

io
 60  = tan  
 10 
 5.55 mV  
8. e = 200 sin 50 t   90

at
Comparing this equation with the standard form, XL 2 f 2 L 2 2f1  2L1
e = e0 sin t we get, e0 = 200 V 16. XL = fL    =4
X L1 f1L1 f1L1
200
 erms = V
2
lic  X L2 = 4  1000 = 4000 
erms 200
Now, irms = = = 2 2 = 2.828 17. The impedance of combination,
R 2  50
 1 
Z =  2fL  
ub
9. Comparing the given equation with the standard  2fC 
form, i = i0 sin t we get, i0 = 4 A 1
= 2  50  1.2 
i0 4 2  50  105
irms = = = 2 2 ampere
2 2 = 376.8  318.5 = 58.3 
P

10. Comparing given equation with the standard 18. e= e2R  (eC  eL )2
form,
e = e0 sin t we get,  = 2f = (40) 2  (80  40) 2
et

 2f = 377  f = 60 Hz = 1600  1600


= 2(1600) = 40 2 V
11. Heat produced by A.C. = 3  Heat produced
by D.C 19. Figure below shows the graph for the given case
rg

2
 i rms Rt = 3  I Rt
2
X L e L L
2
tan 45 = = =
i 2
=32 X R eR R
rms
 L = R eL e
 irms = 2 3 = 3.46 A
Ta

R R
 L= =  = 45
12. Z= 2
R +X 2
L = 10 2   2f
100 eR
e0 = 2 e = 220 2 V =
2  3.14  1000
e0 220 2 100
 i0 = = = 22 A =  10–3
Z 10 2 6.28
 16  10–3 H = 16 mH
1 1 1
13. XC = = = e
C 2fC 2  3.14  50  10  106 20. i=
 318.5  Z
e 50
irms =
erms
=
200
= 0.6 A  Z= = = 25
XC 318.5 I 2
Z2 = R2 + (XC – XL)2
i0 = irms  2 = 0.6  2 A  252 = 202 + (XC – XL)2
395

MHT-CET Triumph Physics (Hints)

 (XC – XL)2 = 625 – 400 = 225  Voltage across capacitor


 XC – XL = 15 = i  Resistance across capacitor
 XC = XL + 15 = 10 + 15 = 25  = 5  10 = 50 V
100 25. Z= R 2  X 2L , XL = L and  = 2f
21. For D.C., R = = 100 
1
100
 Z = R 2  42f 2 L2
For A.C. Z = = 200 
0.5 
26. As the current i leads the voltage by , it is an
2
Now, Z = R + 2
X 2L 4
XC
 XL2 = (200)2  (100)2 RC circuit  tan  =
R
= 40000 10000 = 30000
 1
 XL = 30000 = 173.2   tan =
 CR

ns
4
XL = 2fL
XL   CR = 1
 L = Given that,  = 100 rad/s
2f
1 1
173.2  CR = s

io
= = 0.55 H 100
2  3.14  50
 From all the given options, only option (A) is
120 correct.
22. R =

at
0.5
e
= 240  27. i=
R  X L2
2
Effective impedance for A.C. source,
120 220 220
= 300  i= = = 3.33 A
Z=
0.40
lic (20)  (2    50  0.2)
2 2 66
Using, Z2 = R2 + X 2L
L 1
E 28. We have, XC  and X L  L  2 f
XL = Z2  R 2 C  2f
120 V
ub
= (300)2  (240)2
30. In series LCR circuit,
= 180  2
 1 
 2fL = 180 Z= R 2 +  L  
  c
180
 L =
P

2
2f  1 
 300   1000  0.9  
2
= 6 
180 1.5  1000  2  10 
= =  0.48 H
2(60)  2
 106 
et

= 9  104  900  
e e  2  103 
23. i= =
Z  1 
2

9  104  900  500


2
R 2   L  =
  C 
rg

As resistance is negligible, R  0 =  9  16   104


e = 500 
 i=
 1 
 L  31. For AC, XL = 2fL
Ta


 C 
For DC, f = zero
1 e 100 X L(AC)
Now, L – = =  = infinity
C i 5 X L(D C)
= 20  ….(i)
If the value of capacitor is decreased to half then, V 12
32. R= = =3
1 100 I 4
L – = = 10 
 C 10 e 12
    Z = rms = =5
 2 i rms 2.4
L 
2
= 10  ….(ii) Z2 = R2 + XL2
C  XL2 = 25  9 = 16
By equation (i) – equation (ii), we get  XL = L = 4
1 4
= 10   L= = 0.08 H = 8  102 H
C 50

396

Chapter 13: AC Circuits


33. Average power lost / cycle P = erms  Irms  cos 
e I 1 100 100  103 
= ermsIrms cos  = 0 0 cos  = e0I0 cos  =   cos
2 2 2 2 2 3
34. e = 5 sin(t + 90) and 104  103 1 10
=  = = 2.5 watt
I = 2 sint 2 2 4
 42. e = e0 sin (t + )
There is phase difference of between e and I
2 e0 200
 erms = =
P=0 2 2
35. Power dissipation in pure inductive and  Power, P = erms Irms cos 
capacitive circuit is zero. P 1000 2
 Irms = = = 10 2 A
e rms  cos  200  cos60
36. R = 40 + 40 = 80 

ns
 XL – XC = 100 – 40 = 60  43. Comparing given equations with the standard
Z = R 2  (X L  X C ) 2 = 802  602 = 100 forms,
R 80 e = e0 sin t and I = I0 sin(t + )
 Power factor, cos  = = = 0.8 

io
Z 100 e0 = 200 V, we get, I = 1 A,  = rad
3
37. IWL = Irms sin 200 1
erms = , Irms =
3

at
2 2
 3 = 2 sin   sin  =   = 60
2  P = ermsIrms cos 
1 200 1 
 Power factor = cos  = cos 60 = =  cos = 50 watt
2 2 2 3
1
lic 44. Z = (R) 2  (X L  X C ) 2
38. cos  =
2
L = (8) 2  (31  25) 2
 = 60, tan 60 =
ub
R = 64  36
3R 3  100 3 = 10 
 L= = = H
 2  50  R 8
 Power factor, cos  =  = 0.8
2
Z 10
 1 
P

39. Z= R2    46. Maximum energy stored in a capacitor,


 2fC 
Q2
1 E1 =
= (3000) 2  2C
et

2
 2.5  When energy is stored equally between the
 2  50   106 
   electric and magnetic fields, then energy in the
 Z= (3000) 2  (4000) 2 = 5  10 
3 1
capacitor is E2 = E1
rg

2
R 3000
 Power factor, cos = = = 0.6 If Q is the charge on the capacitor in this case,
Z 5  103
Q2
v 2 cos  then E2 = .
Power dissipated, P = erms Irms cos  = rms 2C
Ta

Z
Q2 1 Q 2
(200) 2  0.6  
 P= 2C 2 2C
5  103
Q
= 4.8 W Q = .
2
2
 Ip  I 2p R
40. Power = I2R =   R= 1 
 2 2 47. Magnetic field energy  LI2  changes from
2 
41. Comparing the given equations with the max. to zero, when current changes from I0 to
standard forms, zero i.e. in T/4 sec.
e = e0 sin t and I = I0 sin (t + ) 
T
= 5 ms, T = 20 ms = 20  103 s
we get, 4
 1 1 103
e0 = 100 V, I0 = 100 mA and  = rad f=  3
 = 50 Hz
3 T 20  10 20

397

MHT-CET Triumph Physics (Hints)


49. The current will lag behind the voltage when 1 1 1000
57. r =   = 83.3 rad/s
reactance of inductance is more than the LC 4.0  36  10 6 12
1
reactance of condenser. Thus, L > or From Notes 12,
C
1 1 r L 250 4
 > or n > or n > nr where Q factor of the circuit =   = 100
R 3 10 / 3
LC 2 LC
nr = resonant frequency. 59. e = e0 cos t = e0 cos(2ft)
50. Given that, XL = XC  2 50  1 
= 10cos  
1 1  600 
 = =
LC 4  10  10  106
3

1 1
= 10cos =5 3V
= = 6
2  104

ns
8
4  10
60. Phase difference relative to the current,
104
= = 5  103 rad/s   
2  =  314 t    314t =  rad
 6 6
1

io
51. f=
2 LC 61. Comparing given equation with the standard
1 form, I = I0 sin t we get,
f=  80 Hz
2 1

at
2 2  2  106 = 200  T = s
T 100
1 1 T
52. fr = = The current takes s to reach the peak value.
2 LC 2 9  103  10  106 4
=
1
=
10000
= 0.530 kHz
lic
 Time to reach the peak value =
1
s
2 3 10 4 6  3.14 400

1 1 2
53. Using, fr =  L  for fixed fr 62. e 2L = e  e 2R
ub
2 LC C = (20)2 – (12)2
L 2 C1 C L = 400 – 144
    L2 =
L1 C 2 2C 2 = 256
54. Impedance of LCR circuit will be minimum at  eL = 16 V
P

resonant frequency
63. XL = L = 2fL = 2   50  0.7  220 
1 1 105 1
 f0 = = = s Z= R 2  X 2L = 220 2  220 2 = 220 2 ohm
2 LC 2π 1×103 × 0.1×106 2π
et

ev 220 1
55. Given that, VL = VC  Iv = = = = 0.707 A
Z 220 2 2
1 1
fr = 
rg

2 LC 2 3  103  30  106 2
 1 
10 4 64. Z  R 2   2fL  
=  530 Hz  2fC 
2  3
From above equation at f = 0  z = 
Ta

1 1 1
56. fr =  fr  When f  (resonant frequency)
2 LC LC 2 LC
1/ 2
(f r ) 2 1  LC  ZR
  L1C1   1 1 
(f r )1 L 2C 2  L 2C 2  1
For f   Z starts increasing.
1/ 2 2 LC
 LC  1
=   1/ 2 i.e., for frequency 0 – fr, Z decreases and for fr
 2L  4C  (8)
to , Z increases. This is justified by graph C.
(f r ) 2 1
 
(f r )1 2 2 1
65. Brightness  Pconsumed  for Bulb
f1 R
(fr)2 =
2 2 As, resistance of the bulb remains same for both
f AC and DC supplies, the brightness will be
 (fr)2 = ….[ (fr)1 = f]
2 2 equal in both the cases.

398

Chapter 13: AC Circuits


V 80
12. Resistance, R = = =8
Competitive Thinking i 10
R L
2. Given,
I = 50 cos(100t + 45) A
10 A
Comparing the equation by I = I0 cos(t + )
I0 = 50 A 
I 50 220 V
 Irms = 0 = = 25 2 A
2 2 For a RL circuit;
3. e0 = 2 erms = 1.414  100 = 141.4 V e= e 2R  e 2L
2
5. In pure capacitive circuit, let an A.C. voltage be  e 2L = e – e 2R
supplied of the form i2(2fL)2 = e2  (iR)2

ns
e = e0 sin t ….(i) e 2  (iR) 2 (220) 2  (10  8) 2
q  L2 = = 2
we know that, C = i 4 f
2 2 2
10  4  (3.14) 2  (50) 2
e ~ L2 = 0.425  102
 q = Ce = Ce0 sin t

io
 L = 0.065 H
dq
 I= = Ce0  cos t
dt 13. Given, eL = 40 V; eC = 120 V; eR = 60 V
 I = I0 cos t ….(taking I0 = Ce0)

at
e2R   eL  eC 
2
 Source voltage, e =
 I = I0 sin (/2 + t) ….(ii)
Thus, on comparing (i) and (ii), we see that = (60) 2  (40  120) 2
current leads the voltage by a phase angle of
= (60) 2  (80) 2
/2.
lic  V = 100 volt
1
6. XC = 14.
C Vo = 200 V
 angular frequency () for D.C. source is Zero ~
ub
 Capacitive reactance becomes infinite.
R C L
 1 
XC  X L  2fC  2fL 
8. tan  =  tan 45 =  
P

R  R  R = 100 , Vo = 200 V, f = 50 Hz
  C-I: When capacitance is removed then circuit
1 is L  R circuit
 C=
et

2 f (2fL  R) X 
  = tan1  L 
XL 3R  R 
9. tan     3
R R X 
60 = tan1  L 
rg

   60o   / 3  100 
XL
XL tan 60 =
10. tan  = = 1   = 45 or /4 100
R
Ta

XL
 1  3=
 L  C 
100
XL  XC
11. tan = =  XL = 100 3
R R
  C-II : when inductor is removed then circuit is
 
 100 1  R  C circuit
3
 2  3.14  50    10  103   XC 
 2  3.14   50   = tan1  
= 2   R 
 10 
X 
  60 = tan1  C 
   100 
 
XC
i.e. tan = 1 tan 60 =
 = tan–1(1) 100
  = 45 XC = 100 3

399

MHT-CET Triumph Physics (Hints)

Now, Z = R 2  (X L  X C ) 2 XC 1 / C 1
 tan 30 = =  ….(ii)
R R CR
 
2
Z= R  100 3  100 3
2
From equations (i) and (ii),
tan 60o L
Z = R  Z = 100  = 2LC
tan 30o R  1
The current in L  C – R circuit is,
CR
Vo = io Z
3
io =
Vo = 2  60  103  0.5  106
1
Z
200 3
io =  3 = 2  3  108
100
io = 2 A  2 = 108
 = 104

ns
15. erms = 10 V,  = 200, R = 50 , 2f = 104
L = 400mH = 400 × 10-3 H, 104
C = 200F = 200 × 10-6 F  f= Hz
2
2
 1 
R 2   XL  XC  =
2
Z= R 2   L  

io
 22. For pure inductor  =
 c 
2
 1 
2

= 502   200  400  103  6  
Pav = VI cos  = VI cos
2

at
 200  200  10  
 Pav = 0
502   80  25
2
=
23. In LCR circuit power is always dissipated
Z = 74.3  through resistor.
irms =
erms
=
10
= 0.13459 A
lic 24. For LR series circuit,
Z 74.3
eL = irms XL = 0.1345 × 80 = 10.8 V Z = R 2  2 L2
V
R 2   XC  XL  =
 I=
ub
2
16. Z= (3) 2  (14  10) 2 R  2 L2
2

 Z=5 V 2R
 P = I2R =
e0 R  2 L2
2
17. i0 = ( Z = XL for pure inductive circuit)
XL
P

e2 rms
2 e rms 2  200
25. For purely resistive circuit Power (P) =
i0 = = R
XL 2fL When inductance is connected in series with
resistance
et

2  200
i0 = = 0.9 A P = erms irms cos 
2 50  1
e  R  e2
18. XL = L = 2fL = erms  rms   = rms R
 Z  Z  Z2
rg

XL  f
 PR  R
 The graph will be a linear graph. P =
Z2
 e 2
rms  PR 

19. i PR 2
P = 2
Ta

Z
26. P = VI
P 100 5
eC  eL I= = = A
V 220 11

20. Since, VL = VC it is a resistive circuit 27. P = erms Irms cos


V 220 R
 V = 220 V and I =  = 2.2 A But, cos  = and erms = Irms  Z
R 100 Z
e R 220  220  18
21. When L and R are connected in series,  = 60 P = erms  rms  = = 800 W
Z Z 33  33
XL L
 tan 60 = = ….(i) 28. Pavg = erms × Irms × cos
R R
Similarly, when C and R are connected in circuit, Pavg 63
cos = = = 0.1
 = 30 e rms  I rms 210  3

400

Chapter 13: AC Circuits


29. Average power dissipated = erms × Irms 35. Given: L = 20mH = 20  10–3 H
= Irms × R × Irms C = 100 F = 100  10–6 F, R = 50 
I0 I V = 10 sin 314t,
= ×R× 0
2 2 But, V= V0 sin  t
 2   10
2
I02 R On comparision we get,
= =
2 2  = 314 rad/s and V0 = 10 V
4 Inductive reactance,
= × 10 = 20 watt
2 XL = L = 314  20  10–3 = 6.28 
Capacitive reactance,
30. Pavg = VrmsIrms cos
1 1
 v0  I0    v0 I0 XC =  = 31.85 
=    cos 3  = 4 C 314  100  106
 2  2   
Impedance,

ns
31. P = erms Irms cos and Pmax = ermsIrms Z2 = R2 + (XL  Xc)2
Since P = 50 % Pmax = 0.5 Pmax Z2 = 502 + (6.28 – 31.85)2
 Z2 = 3153 
 cos = 0.5   =
3

io
Average power,
Using, P = VI cos  = I2 Z cos  we get, R V02 R 100  50
2
32. Vrms
Pav =   = 0.79 W
P 2 Z 2
2  Z2 2  3153
cos  = = = 0.5

at
I2 Z 4 1
R
33. Comparing given equations with the standard 36. Power factor = cos  =
Z
forms, as current remains same, we can write,
e = e0 sin t and i = i0 sin(t + ) we get,
e0 = 100 V, I0 = 100 mA
lic cos  =
R
=
VR
Z  VR    VL  VC 
2 2
e = 100 sin (100 t) V and
  80
I = 100 sin 100 t   mA =  0.8
ub
 3  80 
2
  60 
2

e0 I0
 Power =  cos 
2 2 37. For CR circuit, power factor is given by
100  100  R R
cos  = =
P

=  cos    103
2 3 R X 2 2
1
C R2 
100  100 1 (C) 2
=   103
2 2 R
 (cos )1 =
et

….(i)
= 2.5 W 1
R  2

2
Vrms V2  R  (1C) 2
34. P= cos = rms  
Z Z Z 1 R

rg

=
2
V R 2 1
 P= 0
….(i) R2 
2 Z2 (1C) 2
Given V0 = 10 V;  = 340 rad/s; L = 20 mH; 1 R2
Ta

C = 50 F; R = 40   =
2 R2  1
Z = R 2  (X L  X C ) 2 (1C) 2
(10) 2 1
 P=  (40)  R2 + = 2R2
2 (1C) 2
1 1
  R2 = ….(ii)
  3 1  
2
(1C) 2
(40)   340  20  10 
2
 
  340  50  106   Now,
2000 2000 R
= = (cos )2 =
1600  [6.8  58.8]2 1600  [2704] 1
R2 
2000 (2C) 2
=  0.46 W
4304 1
But, 2 =
Nearest answer is option (C). 2

401

MHT-CET Triumph Physics (Hints)


R 1 1
 (cos )2 = ….(iii) 43. L = =
4 C LC
R2 
(1C) 2  XL and XC will get interchanged.
Dividing equation (iii) by equation (i) 1
 200 L =
1 800C
R2  1
(cos ) 2 R (1C) 2  = 200  800 = 400 Hz
=  LC
(cos )1 4 R
R2 
(1C) 2 44. According to condition of parallel resonance for
1 LC circuit, at resonant frequency (fr) impedance
R2  of circuit is maximum and current is minimum.
(1C) 2
 (cos )2 = cos 1 
4 0 L
R2  45. Q= ....(Using Notes 12)

ns
(1C) 2 R
Using eq(ii), 1 1 100
0 = = = rad/s
1 R R 2 2 LC 9  100  10 6 3
(cos )2 =
2 R 2  4R 2 100 9

io
Q=  = 30
1 2R 2 3 10
= 2
2 5R 46. Irms = (I2 ) = (8  6sin t) 2

at
1
 (cos )2 = Irms = (64  96sin t  36sin 2 t)
5
Irms = (64)  96(sin t)  36(sin 2 t)
38. e = 100 sin 30 t
Since (sin2t) = 0.5 and (sint) = 0
 erms =
100
2
lic Irms = 64  0  36  0.5 = 9.05 A
  47. Alternating voltage: e = 200 2 sin(100 t) volt
I = 20 sin  30t  
 4  Comparing with e = e0 sin t
ub
20  = 100 rad/s, e0 = 200 2
 Irms =
2 Capacitive reactance,
 1 1
Also,  = XC = =  = 104 
4 C 100  106
P

 Average power consumed, e


I0 = 0
 XC
P = erms  Irms  cos
4 200 2
et

100 20 1 2000 1000 I0 =


=     W 10 4
2 2 2 2 2 2
I0 = 2 2 ×10–2 A

Wattless current, I = Irms sin I0 2 2  102
Irms = = = 2 × 10–2 A = 20 mA
rg

4
20 1 20 2 2
 I=   10 A
2 2 2 48. Ammeter measures the rms value of current
39. As the electric and magnetic fields share energy Vrms V0

Ta

Irms = = (C)
equally in an LC circuit, XC 2
1 2 1 50 2
Li = CV 2 =  100  10  10–6
2 2 2
= 5  10–2 A = 50 mA
1/ 2
 CV 2 
 I = 
 L  49. For AC circuit,
 16  10  20 
6 2 1/ 2
V V
= = 0.4 A Current, I =  = 0.2 A
3  Z R   XL  XC 
2 2
 40  10 
When connected to DC source, if the capacitor
1 1
42. f0 =  is present in the circuit, it will provide infinite
2 LC 2 5  103  2  106 resistance to current and the current in the
104 5  103 circuit will be zero. However, the current in the
=  Hz
2  circuit changes to 0.4 A.

402

Chapter 13: AC Circuits


This implies the circuit does not contain any Im
= Im sin (100 π t2)
capacitor. As current in the circuit increases 2
after connecting to DC source, the circuit must 
contain an inductor. Thus, the circuit must be a  100π t2 =
6
series LR circuit.
1
 t2 = s
50. Current in LR circuit is, 600
  Rt
 E  Rt
 1 1 2 1
I = I0 1  e L
   1  e L
 ….(i)  treq= t1 – t2 =    = 3.3 ms
  R  200 600 600 300

dI d  E E  LRt   52.
     e 
dt dt  R R   
E R  LRt  i0

ns
=  e 
irms
R L 
E  LRt  trms tpeak
= e  ….(ii)
L 

io
Rate of energy dissipated across resistance is, For i = i0,
PR = I2R i0 = i0 sin t
Rate at which magnetic energy is stored in coil 
 sin t = 1  t =

at
is, 2
 dI  2 
PL = EL I =  L  × I  t 
 dt  T 2
T
Let at time t,
PR = PL
lic  tpeak =
4
....(i)
Similarly, for i = irms,
 dI 
 I2R =  L  × I irms = i0 sint
 dt 
ub
i0
From equations(i) and (ii),  = i0 sin t
2
E  Rt
 E   Rt 
1  e L  × R = L ×  e L   sin t = 
 1 
R  L  
 2
 Rt  Rt
P

 1– e L
= e L  2 
i.e., t =  t 
 Rt
1 4 T 4
 e L
 T
2  trms =
et

Rt 8
 loge 2 =  Time for current to reach from rms value to
L
Substituting values for R and L,  i0 
peak value is i.e., irms  i0   i0 
rg

10  t  2 
ln 2 =
20 T T T
t = tpeak  trms =  
 t = 2 ln 2 4 8 8
Given that, f = 50 Hz
Ta

51. V (t) = 220 sin 100πt


T 1 1 1
220  t= = =  = 0.25  102
 I (t) = sin 100πt 8 f 8 50  8 400
50
= 2.5  103 s
 I = Im sin (100 πt)
 Option (D) is correct.
For I = Im
Im = Imsin(100 πt1) 53. V = 5 cos 1000t volt
 sin(100 πt1) = 1 V = V0 cos t
 V0 = 5 volt
 100 πt1 = ….[ sin (π/2) = 1]
2  = 1000 rad/s
 1 1 L = 3mH = 3 × 10–3 H, R = 4 
 t1 =   s
2 100 200 Maximum current,
I V0
For I = m 10 =
2 Z

403

MHT-CET Triumph Physics (Hints)


5 P 1000
10 = 60. I=  = 10 A
R  L
2 2 2 V 100
5 The voltage drop across heater must remain
= 3 2 same and the current it draws must be same.
4  (1000  3  10 )
2
Hence, voltage across coil is
5
= =1A VC = 200 2  100 = 182 V
5
VC
We know that I =
54. Comparing the given equation with standard form, L
e = e0 sin t we get, E0 = 200 2 v,  = 100 V
L= C =
182
Vrms V0C I 10  2 50
I rms  
XC 2  L = 0.057 henry
200 2  100  (1  106 )

ns
= 61. Quantity of heat liberated in the ammeter of
2 resistance R
= 2  102 A = 20 mA i. due to direct current of 3 ampere
= [(3)2 R/J]
55. eo = io  XL

io
ii. due to alternating current of 4 ampere
XL = L = 2fL = 2(50) = 100
= [(4)2 R/J]
2
io = ampere  Total heat produced per second
π

at
(3) 2 R (4) 2 R 25R
2 =  
 eo =  100 = 200 V J J J
π
Let the equivalent alternating current be I virtual
 1 
2
ampere; then
56. Z2 =  L 

 R
C 
2
lic I 2 R 25R
 or I = 5 A
J J

c 3  108
Z 62. f = 
ub
 300
= 106 Hz
 1 1
0 Now, fr =  LC 
2 LC 2f r
 As we gradually increase frequency, Z first
P

1
decreases and then increases  L=
42f r2C
57. Let 1 = 50  2  L = 20  1
 2 = 100  2  L = 40   L=
et

42 (106 ) 2  2.4  106


200 200 200
 I=  =  108 H
Z R 2  (L) 2 (30) 2  (40) 2
63.
 L C
rg

I=4A R
58. From V = 200 2 sin t, V0 = 200 2
V0 200 2 200 2
When L is removed,
 I0 = = 
Ta

R
Z R   X L  XC  20  15  15 
2 2
2 2
XC  /3
 tan
200 2 R 3
 I0 = = 10 2  XC
20  X C  R tan Z1
3
59. For inductor, When C is removed,
1 1 XL 
I   tan
XL f R 3
Hence, as frequency increases, current decreases. 
For capacitor, XL = R tan Z2
3 XL
1
I f  Z= R   X L  XC   R
2 2
XC
Hence, as frequency increases, current R /3
 cos  = =1 R
increases. Z

404

Chapter 13: AC Circuits


Vrms V V
64. Irms = I= 
1 Z  1 
2
R2  R2  
 C2
2 
 2fC 
 As  increases, Irms increases and hence the bulb 2fC
glows brighter. Or I = V
4f 2C 2 R 2  1
65. Impedance is given as, Voltage drop across capacitor
R +X 2 2 2fC 1
Z= L Vc = I  Xc = 
R + (L´2πf ) 2
2 4f 2C 2 R 2  1 2fC

 If frequency is decreased, impedance decreases. V


Vc =  2
If number of turns decreases, self inductance 4 f C 2 R 2  1
decreases and thus impedance decreases. When mica is introduced capacitance will

ns
At resonance, XC = XL and impedance increase, hence voltage across capacitor gets
decreases. decreased.
When iron rod is inserted, impedance increases.
1 1 1
Hence current decreases. Hence option (D) is 69. f=  f

io
correct. 2 LC C
1
 X L  XC  70. Frequency of oscillation, f =
66. Phase difference   = tan1   2 LC
 R 

at
1
For pure L, R circuit;  f
 L  C
 XL  1  
 = tan1   = tan  R  f air Cdielectric
 R   =
 2fL 
lic f dielectric Cair
 = tan1   C dielectric
 R  But =k ….(k = dielectric constant)
 25  C air
 2   2  f air
ub
 = tan1  100   = k
f dielectric
 
  125
= k
1 100
 = tan (1)
2
P

 = 45 5
 k=  
4
XL 1/ 3
67. tan  = =  k = 1.56
R 1
et

1 71. For a damped harmonic oscillator, equation of


tan = motion is given as,
3
d2x dx
–1 
1  m +b + kx = 0 ....(i)
 = tan   dt 2 dt
rg

 3 Consider series LCR circuit,


C L C R
 = 30 =
6
Ta

But,
 = t i i
 /6 1
 t= = = s
 2  50  600

68. R C In the above circuit,
q
 iR  L 
di 
  =0
 dt  C
~  L
 d 2q  dq 1
+ R   + (q) = 0 ....(ii)
V = V0 sin t 2 
 dt  dt
  C
1 Comparing equations (i) and (ii),
Xc 
2fC 1
L  m, C  ,Rb
current in ciruit k

405

MHT-CET Triumph Physics (Hints)

Hints to Evaluation Test

I0 6 i =  q 02  q 2
1. Irms = = = 3 2A
2 2
= 2.9  104  48  10    24 10 
6 2 6 2

1
2. f0 =
2 LC = 2.9  104  106  12 4  1
1 = 1.2 A
=
2  3.14 5  104  20  106 7. Through element X, current is in phase with the
4
10 E 0 100
 f0 =  1592 Hz voltage. Therefore, X = R =  = 40 A.
6.28 I0 2.5

ns
3. Comparing the given equation with standard form, Through element Y, current lags behind the
e = e0 sin t we get,  = 120, e0 = 240 V applied voltage by 90. Therefore, Y must be an
 120  7 inductor,

io
 f= =  19 Hz E 0 100
2 2  22 XL =  = 40 A.
I0 2.5
240
 erms = = 120 2  170 V When X and Y are connected in series,
2

at
Z  R 2  X 2L  40 2  40 2  40 2 ohm
1 1
4. XC =  XC  E E 100 5
2fC f I =  0   A.


X C
=
f
=
50
=
1
lic Z 2Z 2  40 2 4

XC f 200 4 8. Here, R = 100 , E = 100 volt,  = 300 rad/s


X 10 when capacitor is removed,
 XC = C = = 2.5 
ub
4 4 XL
tan  = = tan 60 = 3
R
1
5. f=
2 LC0 XL = 3 R
When inductor is removed.
= 10  103 Hz
P

XC
= 10000 Hz tan  = = tan 60 = 3
R
1
f = XC = 3R
et

2 LC
R 2   XL  XC 
2
= (10000  100)Hz Z=
= 9900 Hz
= 1002  0
rg

f C
= = K = 100 
f C0
E  200
2 2 I =  = 2A
 f   10000  Z 100
K =     = 1.02
Ta

 f    9900  R
P = EI cos  = EI  
Z
6. C = 2 µF = 2  106 F, V0 = 12 volt
100
q0 = CV0 = 2  106  24 = 48  106 C = (100)(2) 
100
When V = 6 volt,
q = CV = 2  106  12 = 200 W
= 24  106 C 9. On introducing an iron bar in the inductor,
=
1 inductive reactance XL = L increases.
LC Therefore, impedance of the circuit
1
R 2   XL  XC  ;
2
= Z=
3 6
0.6  10  2  10
Increases, current through the bulb decreases
= 2.9  104 rad/s and it becomes dim.
406

Chapter 13: AC Circuits


1 Z= R 2   XC  XL 
2
10. XC  XL =  L
C
502   31.85  6.28
2
When frequency is small,  is small. Reactance =

is positive. For certain value of freq.,


1
= L, = 56. 16 
C E 2 E 02
Average power, P = 
reactance is zero. For larger values of , Z 2Z
reactance becomes negative. Therefore, curve 55
= = 0.22 W
(ii) is correct. 2  56.16

11. In the circuit shown, L and C are in series.  Total energy dissipated in 10 min.
Therefore, it amounts to a series resonance = 0.22  10  60
circuit. The current through a.c. source is = 132 Joule

ns
maximum.

13. In a series LCR circuit, current will be


maximum, when  = natural frequency of LCR

io
system = 1/ LC

14. As VR = I0R

at
VR 100
 I0 =  = 0.1 A
R 1000
At resonance, XL = XC

L =
1
;
lic
C
1
L=
2C
ub
1
= = 50 H
100 
2
 2 10 
6

VL = I0XL = I0(L) = 0.1  100  50 = 500 V


P

E 0 200
15. R=  = 40 
I0 5
et

As current lags behind the applied voltage by


90, therefore, element Y must be a pure
inductor.
rg

E 0 200
XL =  = 40 
I0 5

Z = R 2  X 2L
Ta

= 402  402

= 40 2 

16. L = 20  103 H,
C = 100 µF = 100  106 F = 104 F
R = 50 , t = 20 min = 20  60 s, E = ?
From V = 5 sin 314 t, E0 = 5 V,
 = 314 rad/s
XL = L = 314  20  103 = 6.28 
1 1
XC =  = 31.85 
C 3.14  10 4

407
14 Dual Nature of Radiation and Matter

Hints

Classical Thinking Critical Thinking

ns
hc 1. For no emission of photoelectron,
13. =
0 energy of incident light < work function
hc  
  = =  h <   <

io
20 2 h

25. The maximum velocity or the kinetic energy of 2. If threshold frequency is 0, then light frequency
photoelectrons depends on frequency and not on becomes 1.50.

at
intensity. If we make it half it becomes 0.75 0, which is
smaller than threshold frequency, therefore
27. If the frequency of incident radiation is kept photoelectric current is zero.
constant at a value greater than 0 (threshold
lic
frequency), then the rate of emission of 3. 0 =
hc
=
6.6  1034  3  108
= 3  1019 J
photoelectrons from emitter is directly 0 6600  1010
proportional to intensity of incident radiation.
hc
28. Intensity  No. of photons 4. = 2 eV
ub
 max
 No. of photoelectrons
hc 6.63  1034  3  108
29. Intensity increases means that more photons of max = =
2eV 2  1.6  1019
same energy will emit more electrons of same 6.63  3
energy, hence only photoelectric current increases. =  10–7
P

3.2
30. Photoelectric effect is one photon, one electron = 6215 Å
phenomenon, i.e., one photon cannot eject more
than one photoelectron. 5. 0 = h0
et

0 3.3  1.6  1019


32. According to Einstein’s equation,  0 = =
h 6.63  1034
h = h0 + K.Emax 
5.28 1019
 K.Emax = h  h0. Comparing it with = 7.9  1014
rg

=
y = mx + c, we can say that, 6.63 1034
this is the equation of straight line having  8  1014 Hz
positive slope (h) and negative intercept (h0) 6. 0 = h0
Ta

on K.E. axis.
hc 6.63 1034  3 108
 0 = =
37. Velocity of photon c =  0 5000 1010
1 = 3.978  10–19 J
48.  3.978 1019
m = = 2.48 eV
me < mp < m 1.6 1019
 e > p >  7. Minimum kinetic energy is always zero.
34
h 6.63  10 8. The velocity of the photoelectron ejected from
49. = = = 6.63  1033 m
mv 103  100 near the surface is larger than those coming
from interior of metal because for the given
h 6.631034
50. = = energy of the incident photon, less energy is
mv 210 103 100 102
3
spent in ejecting the electron near the surface
= 3.32  1028 m than that from the interior of the surface.
408

Chapter 14: Dual Nature of Radiation and Matter

h0 6.63  10 34  1.6  1015 By Einstein’s photoelectric equation,


9. 0 = eV = = 6.63 eV
e 1.6  10 19 K1 = h  0 = K ….(i)
K.E = E – W0 = 8 – 6.63 = 1.37 eV and K2 = h(2)  0 ….(ii)
= 2h  0 = h + h  0
1 hc
10. mv2 =  0  K2 = h + K ….[From (i)]
2 
1 6.63  1034  3  108 18. Using Einstein photoelectric equation,
 mv2 =  2.5  1 eV
2  3.6 107  1.6 1019 E = 0 + K.Emax
1 2
2  1  1.6  1019 h1 = 0 + mv1 ….(i)
 v= = 0.6  106 m/s 2
9.1  1031 1
= 6  105 m/s h2 = 0 + mv 22 ….(ii)
2
Subtracting equation (ii) from equation (i) we

ns
11. If the voltage given is V, then the energy of
electron, get,
1 1
mv2 = eV h(1  2) = m(v12  v22 )
2 2

io
2h
2eV 2  1.6  1019  1000  (v12  v22 ) = (1  2)
 v= = m
m 9.1  1031
 
= 1.875  107  1.9  107 m/s But 2 = 1 

at
2 2
1 2 2h    h
12. mv max  eV  (v12  v22 ) =
2    
m 2 m
2eV 2  1.6  1019  9
 vmax =
m
=
9.1  1031
lic 19. eV0 = h  0
6 eV0 = (2.4 1.6) eV
= 1.8  10 m/s
 V0 = 0.8 V
13. K.Emax (eV) = E(eV)  0(eV)
(2 eV  0.6 eV)
ub
(E  0 )
= 6.2  4.2 20. V0 = = = 1.4 V
= 2 eV e e
 K.Emax(joule) = 2  1.6  1019 J hc
= 3.2  1019 J 21. eV =  0

P

14. E = 0 + Kmax 1 2
 mv max  eV
12375 2
 E  2.475eV
5000 v2  m v2 1
 V = max  max
et

 Kmax = E  0 = 2.475  1.9 = 0.58 eV 2e   2


e
2 
m
1
15. mv2max = h – h0 1.2  10  1.2  106
6
2 = =4V
2  1.8  1011 
rg

= 9.2 eV – 4.2 eV
= 5 eV
hc hc
= 5  1.6  1019 22. eV1 = – 0 , eV2 = – 0
1 2
= 8  1019 J
Ta

hc hc
hc hc  – eV1 = – eV2
16. E=  0 and 2E =  0 1 2
 
 1 1
 E  0  e (V2 – V1) = hc   
 
 2 1 
 2E  0
 1  0 / E   1  2 
  =     hc   = e (V2 – V1)
 2  0 / E   12 
(1  0 / E) 1 λ e 
Since > , so  >  h= (V2 – V1) 1 2
(2  0 / E) 2 2 c 1   2
h
17. Let K1 and K2 be the maximum kinetic energy 23. Retarding potential, V0 = (  0)
e
of photoelectrons for incident light of frequency
 and 2 respectively. 24. Refer Shortcut 3
409

MHT-CET Triumph Physics (Hints)

25. Slope of V0 –  curve for all metals are same K K


 12 = and I =
h (0.2)2 (0.4)2
  i.e. curves should be parallel.
e I (0.2) 2 1
   or I = 3 mA
26. Stopping potential is same for (a) and (b). Hence, 12 (0.4) 2 4
their frequencies are same. Also maximum  I = 3 mA  stopping potential = 0.5 V
current values are different for (a) and (b). Hence,
35. The work function has no effect on current as
they will have different intensities.
long as h > 0. The photoelectric current is
c 3  108 proportional to the intensity of light. Since,
27. 0  = = 6  10 7 m  6000 Å
0 5  1014 there is no change in the intensity of light,
therefore I1 = I2.
hc 1 2
28.  0  mv max h 6.625  1034
 2 = 6.625  1017 kg m s1

ns
36. p= =
hc  1017
Assuming 0 to be negligible in comparison to ,
 h
37 p
1 1 c
v 2max   v max 

io
  pc 3.3 1029  3 108
   = 1.5  1013 Hz
 On increasing wavelength from  to 4, vmax h 6.6 10 34

becomes half.
h h

at
29. Number of photons emitted per second 38. = v=
mv m
p 10  103
n = = 1.72  1031 =
6.6  1034
h 6.6  1034  880  103
(66  109 )  (9  1031 )
30. Intensity of light
lic = 0.011  106
I
Watt nhc
 = 1.1  104 ms1
Area A
4 / 1000
IA 39. mHe = kg
 Number of photon n 
ub
6.02  1023
hc
1 A 1 1  104  300  109
= 6.64  1027 kg
 n=  =  h
100 hc 100 6.6  1034  3  108  =
mv
= 1.5 1012 /s
P

6.63  1034
31. Energy received from the sun =
6.64  1027  2.4  102
= 2 cal cm2 (min)1 = 8.4 J cm2 (min)1 = 0.416  109 m = 0.416 nm
Energy of each photon received from sun,
et

hc 6.6  1034  3  108 h h


E=   3.6  1019 J 40. A = , B =
 5500  1010 mv 0.25m  0.75v
 Number of photons reaching the earth per cm2 B 1
= = 5.3
rg

per minute will be A 0.25  0.75


Energy received fromsun  B = 5.3 A = 5.3 Å
n=
Energyof onephoton
h 6.631034
41. =
Ta

8.4
= = 2.3  1019 2mE 2  9.11031  50  1.61019
3.6  1019
10
6.631034
32.  = 1 Å = 10 m = = 1.737 Å
34 1.456 1047
hc 6.6310  310 8
E = hmax = =
 110 10 h h
42. = =
= 19.8  1016 J mv 2mqV
33. The stopping potential gives maximum kinetic e m pq p V mp
energy of the electron. It depends on the  = =
p me  e V me
material as well as the frequency of incident
light whereas the current depends on the number [ V is the same and qp = e (in magnitude)]
of incident photons. Hence, it is 0.5 V. By  e   mp 
1/ 2

inverse square law, saturation current is    =  


inversely proportional to square of distance.   p   me 

410

Chapter 14: Dual Nature of Radiation and Matter

h 6.6  1034 2. For photo emission  ≥ 0 or λ ≤ 0


43. p  = 1.5  1027 kg.m / s
 4400  1010 3. For work function of 5 eV,
p 1.5  1027 4  10 15  3  108
and mass m   = 5  10 36 kg min = = 240 nm,
c 3  108 5
E For work function of 2 eV,
44. p=
c 4  10 15  3  108
16 10 6
max = = 600 nm
 E = p  c = 2  10  (3  10 ) = 6  10 erg 2
This means wavelength of 650 nm cannot be
45. In Davisson – Germer experiment,
used.
12.28
= Å ….(i)
V hc
4. Work function 0 =
We note that, 0

ns
V 1
34
hc 6.63 10  3 10
8
….(ii)  0 = = = 3.10  107 m

0 4 1.6 1019
i.e.,  will decrease with increase in V.
= 310 nm
If there is a maxima of the diffracted electrons

io
at an angle , then
5. o = hc
2dsin =  o
Hence,
hc

at
  sin  Case (i) 1 =
  0 1
i.e., with decrease in ,  will decrease. hc
Case (ii) 2 =
Thus when the voltage applied to A is increased,  0 2
lic
the value of  will be less than the earlier value.
Dividing equation (i) by (ii),
46. E = eV = 1.6 10–19 105 = 1.6  10–14 J
1  0 2 6000  1010 3
   
47. b sin  = n  2  0 1 4000  1010 2
ub
 12.28 
Here, b = 0.6 Å, n = 1 and  =  Å
 600   hc 
8. K.Emax =   joules  2.2 eV
12.28 
 0.6(sin ) = = 0.5013
600 hc
P

( K.Emax =  0 )
 sin  = 0.84   = sin 0.8 –1 
 hc 
1 K.Emax =  19 
eV  2.2 eV
48. Using Einstein equation, E = 0 + mv2    1.6  10 
et

2
K.Emax = 2 eV  2.2 eV = 0.2 eV
2(E  0 )
 =v As kinetic energy can never be negative, hence
m
photo-emission doesn’t occur.
A charged particle placed in uniform magnetic
rg

field experience a force 1


9. Using, K = mv2 = h  h0 for same metal,
mv 2 2
F= hc
r KA =  h0,
Ta

mv 2
mv A
 evB = r=
r eB 2hc
KB =  h0
2m(E  0 ) A
 r=
eB KB
 KA <
2
Competitive Thinking
10. E = 0 + K.Emax
1. Initially when the moving electron is very far 1 = 0.5 + (K.Emax)1  (K.Emax)1 = 0.5
away from stationary electron, it only has 2.5 = 0.5 + (K.Emax)2  (K.Emax)2 = 2
kinetic energy but as it approaches the (K.E max )1 0.5 1
 
stationary electron, its K.E. decreases due to (K.E max ) 2 2 4
repulsion and gets converted to P.E. according
 v1  (K.E max )1 1
to law of conservation of energy. Hence, K.E.    11. When, 1 = 20,
decreases and P.E. increases.  v2  (K.E max ) 2 2

411

MHT-CET Triumph Physics (Hints)

 (K.E.1)max = h(20) – h0 = h0 ….(i) 16.  = 3300 Å


When, 2 = 50 vmax = 0.4  106 m/s
(K.E.2)max = h(50) – h0 = 4h0 ….(ii) 1 hc
(K.E.)max = mv 2max = – 0
Dividing equation (i) by equation (ii), 2 
 K.E.1 max 1

1
 9.1  10–31  (0.4  106)2

 K.E.2 max 4 2
1 2 6.6  1034  3  108
As, (K.E.)max = mv max =  0
2 3300  1010
6.6  1034  3  108
v12 1   7.28  1020 = 0
  3300  1010
v22 4
 0 = 5.3  10–19 J
v1 1
  17. Using photoelectric equation, h = K.E. + 0

ns
v2 2
Initially,
12. Cut off frequency is given as  h = 0.4 + 0 ….(i)
Work function 0 = h After increasing incident frequency by 30%,
Now, E = K.E. + 0 h(1.3 ) = 0.9 + 0 ….(ii)

io
1 multiplying equation (i) by 1.3 and then
2h = mv2 + h subtracting from equation (ii),
2
0 = [0.9 – 1.3(0.4)] + [0 – 1.3 0]

at
1
 mv2 = 2h  h  0.3 0 = 0.9 – 0.52
2
1 0.38
 mv2 = h  0 = = 1.267 eV
2 0.3

 v=
2hν
lic 18.
1 2 hc
mv1   0
m 2 1
1 2 hc
13. Using, E = h  0 for the two cases we get, mv 2   0
ub
0.5 = h  0 ….(i) and 2 2
0.8 = 1.2 h  0 ….(ii)  hc 
 0 
 v1   1
2
By equation (i)  1.2  equation (ii) we get, 2
2
 v1 
        ….   2
0.2 0 = 0.2 or 0 = 1 eV  v 2   hc  0   1   v2 
 
P

 2 
1 2
14. Kmax = mv max = h  0 ….(i) 4hc hc
2   40   0
2 1
Now, when  is doubled, then
et

1 4hc hc
m (2 vmax)2 = 2h  0    30
2  2 1
1 4  1240 1240
 4  mv 2max = 2h  0   = 30
rg

2 310 248
 4(h  0) = 2h  0 ….[from equation (i)]  30 = 11  0 = 3.7 eV
 30 = 2 h 1 2
2h 19. mvmax = eV
Ta

 0 = 2
3
2eV
 vmax =
hc m
15. KE1 =  0

1 2 hc hc 1
hc 2hc 20. mv =  0 or = mv2 + 0 and
KE2 =  0 =  0 2   2
/2 
1 2 hc 4  1 
KE2 = 3KE1 mv1 =  0 =  mv 2  0   0
2  3  3 2 
2hc  hc   
  0 = 3  0   4 
  
4
hc  v12  v 2 + constant
 2 0 = 3
 1
hc  4 2
 0 = So, v1 > v  
2 3

412

Chapter 14: Dual Nature of Radiation and Matter

21. For ejected electron, 28. Using photoelectric equation,


1 1 1  hc
mv2 = hc    = 0 + 3V0 ....(i)

2   0 
hc
2hc  1 1  = 0 + V0 ....(ii)
 2
v=   
m   0  Subtracting equation (ii) from equation (i),
hc
2  4.14  1015  3  108  1.6  1019  1010 1010  = 2V0
=    2
9.1  1031  2536 3250 
hc
= 6.15  105 m/s  V0 =
4
 0.6  106 m/s–1 Substituting in equation (ii)
22. From Einstein’s photoelectric equation, hc hc hc
 0 =  V0 = 

ns
hc 1 2 2 4
= mv2max + 0 hc hc  hc 
λ 2 = ....  0 
hc 1 0 4   0 
 = eV0 + 0 ...( mvmax = eV0)
2

io
λ 2  0 = 4
1
 V0  29. From Einstein’s equation,

h = eV0 + h0

at
Thus, if incident wavelength is decreased, then
hc hc
stopping potential will increase.   = eV0
 0
23. According to Einstein’s photoelectric equation case (i)  =  ; V0 = V
E = 0 + Kmax
lic hc

hc
= eV ....(i)
hc  1 1   0
 V0    
e   0  V
case (ii)  = 3 ; V0 =
 As  decreases, V0 increases.
ub
6
hc hc eV
24. eV0 = h – h0  = ....(ii)
3 0 6
If  increases, V0 will increase.
dividing equation (i) by equation (ii)
P

25. eV0 = h  0  hc hc 
hc   
=  0   0 
=6
  hc hc 

et

 6.631034  3108   
=  1.07  eV  3 0 
9 19
 33210 1.610   1
1 1 1 
= 2.67 eV   = 6  
 0  3 0 
rg

Nearest answer is (D)


1 1 2 6
  = 
26. Energy of incident light  0  0
12375 1 6 2 1
E  6.18eV
Ta

2000  + = 
0 0  
Using, E = 0 + eV0
5 1
(E  0 ) (6.18eV  5.01eV)  =
V0  = 0 
e e
= 1.17V  1.2V  0 = 5
30. Using Einstein’s photoelectric equation
27. In photoelectric effect, energy is conserved.
Case I :
h
 V0 = (  0) hc hc 1 1 
e eV =  = hc    ....(i)
6.6  1034  (8.2  1014  3.3  1014 )
 0   0 
 V0 = Case II :
1.6  1019
6.6  4.9 V hc hc
=  101  2.0V e  
1.6 4 2  0

413

MHT-CET Triumph Physics (Hints)


4hc 4hc 40. E = 15 keV = 15 × 103 eV
 eV = 
2  0 12400
 E= eV Å
1 1 
= 4hc    ....(ii) 12400
 2  0  = eV Å

Equating (i) and (ii), 12400
1 1  1 1 = eV Å
hc    eV
 = 4 hc   
  0   2  0   = 0.826 Å (λ < 0.01 Å)
1 1 2 4 It belongs to X-rays.
  
 0  0
hc
 0 = 3  41. E=

31. The saturation photoelectric current is directly E1 

ns
 = 2
proportional to the intensity of incident radiation E2 1
but it is independent of its frequency. Hence, E11 3.2  1019  6000
saturation photoelectric current becomes double,  E2 = = = 4.8  10–19 J
2 4000
when both intensity and frequency of the

io
incident light are doubled. 42. The plate current reduces with increasing
wavelength. When wavelength exceeds certain
33. Photoelectric current  intensity of light
value, photo electric effect ceases, making

at
 I1 < I2 current value zero.
34. Above threshold frequency (0), the stopping 43. We know,
potential increases with the increase in (K.E.)max = h  0
frequency.
lic  2 eV = 5 eV  0
35. Energy radiated as visible light 0 = 3 eV
5 Hence, when h = 6 eV,
=  100 = 5 J/s
100 (K.E.)max = 6 eV  3 eV = 3 eV
ub
If n be the number of photons emitted per Also, (K.E.)max = eV0 = 3 eV
second, then, nhv = E = 5  V0 = 3 V
5 5  5.6  107 As, stopping potential is a retarding potential,
 n=  = 1.4  1019 potential of A relative to C =  3 V
hc (6.62  1034 )(3  108 )
P

h hc
 hc  25 45. p= , E=
36.    N = 200  ….[Given]  
 100
Thus, if  decreases, both p and E will increase.
et

200  25  200  25  0.6  106


 N=  = h 6.626  1034
100 hc 100  6.2  1034  3  108 46. =  = 6.626  10–34 m
= 1.5 1020 mv 0.1  10
rg

37. K.E. = E  0 h 1
47. = p
 0 = 10.20  3.57 p 
6.63  1.6  1019
 v0 = = 1.6  1015 Hz 48. E=
1
mv 2 
m 2 v2
Ta

6.67  1034 2 2m
1 2
38. From Einstein’s photoelectric equation, 
2m
 p  ….( momentum p = mv)
h1 = W0 + eV1
h2 = W0 + eV2 1 h2  h
=  ….  p  
1 W0  eV1 2m  2  
 
 2 W0  eV2 h2
=
 W01 + eV21 = W02 + eV12 2m 2
W0   2  1 
 e= 49. de-Broglie wavelength,
1V2  V1 2 h
=
p
hc 6.6  1034  3  108
39. = = = 3.5  10–11 1
E 35  103  1.6  1019 But p = 2mE  
= 35  10 –12
m E

414

Chapter 14: Dual Nature of Radiation and Matter

1 h h 1
i.e., E  55. = = 
2 mv 2mk k
2 2
 E 2   1   1 
      4 56. =
h

h
 E1    2   0.5  p 2mE
As, E2 = 4E1 h 1 h 
E = 3E1 Now,  = =  =  0.25
2m(16E) 4 2mE 4
h  % change =    =   0.25 
50. = ,
p = 0.75  75%
But, p =
2mE
h
h 57. =
 = , p
2mE

ns
1 P2 h2 (6.61034 ) 2
  K.E. = = =
E 2m 2m 2
2 9.11031  (5.5107 ) 2
1 E2  7.91  1025  8  1025 J
 
2

io
E1 58. Let p be initial momentum of electron,
0.4  10 10
E Given,
 = p  p  Pm
1.0  1010 1

at
 E = 0.16 keV as  
1
h p
51. Using  = , increase in p, decreases 
2mE

Eelectron = 2
h2
and Ephoton =
hc
lic    
0.5
=
 0.5 
 1 
   2m   100  100 
 p
E photon  hc  2  2m  2mc2 2  5  105 20  =
  . =    p
ub
E electron  h 2   hc  (50  103 ) 1
   0.5  p
   1 =
 100  p  P m
h
52. For electron,  e  and for photon, 0.995 p + 0.995 Pm = p
2mE e
P

p
hc 0.995Pm =
p  200
Ep  p  200 Pm
e h Ep
et

   59. For a charged particle, de-Broglie wavelength


p 2mE e hc
is,
1
e 1  E  2 h
 ….( Ep = Ee) 
 p c  2m 
rg

2meV
1
 
53. For any charged particle, de-Broglie wavelength is, V
h
= 1 V2
Ta

2mE  
2 V1
h
 2 =
2  2m  2E  h h
60. de Broglie wavelength,  = =
h  p 2meV
 2 = =
2  2mE 2 For an electron,
me = 9.1  10–31 kg, e = 1.6  10–19 C
54. For photon,
6.63  10 34
E=
hc  =
 2  9.1  1031  1.6  1019  10000
 For electron, 6.63  1034  1023
=
h h h 2  9.1 1.6
 = = =
2mE hc 2mc = 1.229  10–11m
2m
  12.2  10–12 m
415

MHT-CET Triumph Physics (Hints)


h λe m
61. =  p =
2meV 3 M
h2 λ m
 V=  p = ....( e = )
2me 2 3 M

=
 6.63  10 
34 2
h
68. =
2  9.1  1031  1.6  1019  1  1010 
2
2mqV
 150 volt We know, q = 2qP
m = 4 mp
12.27 12.27
62. =  = 1.227 Å P mq 4m P  2q P
V 100   =  82 2
 m Pq P mP  qP
12.27 1.227  109

ns
63. = Å= h
V 400 69. = ….(De-Broglie formula)
9 p
= 0.061  10 m = 0.06 nm
h h h
 = = = ….(i)
h p  m v 4mv

io
64. de Broglie wavelength is,  =
2mE [as mass of alpha is 4 times mass of
For proton, proton/neutron and velocity given is v]
h h h h
p =

at
d = = = ….(ii)
2m p E pd m d vd 2m 2v
For -particle, [as mass of deuteron is 2 times mass of
h h
proton/neutron and velocity given is 2v]
 =
2m  E
=
2  4m p  E
....( m = 4mp)
lic From (i) and (ii),
 1
=
d 1
p h 2 4m p E 4mp
 =  = =2:1 hc
 h mp 70. For photon: E =
ub
2m p E
p
65. K.E. = 120 eV hc
 p = ....(i)
 V = 120 V E
12.27 12.27 For electron: E = mc2 = pc
= 1.12 Å = 1.12  1010 m
P

= 
V 120 E
p=
12 c
= 112  10 m = 112 pm
h hc
 e =  ....(ii)
et

66. For electron, de-Broglie wavelength is, p E


1 1
=   Comparing equations (i) and (ii),
2meV V p  e
rg

1 V2
 
71. K.E. of electrons =
p2
2 V1
2m
V1  12 10000   2 10000 h
 V2 =   = 2500 V here, p = ....(De-Broglie hypothesis)
 2 
Ta

 22 2
4 
2
h
1  
67. db  
m V  K.E. =   ….(i)
2m
1 Also, if 0 is cutoff wavelength, maximum K.E.
 e 
me V hc
of X- ray photons = ....(ii)
1 0
p 
mp 9V Maximum K.E. of X- ray will be equal to that of
electrons.
λp me V
 = . hc h2
λe mp 9V  = 2 ....[from (i) and (ii)]
0 2 m
λp m 1 2 2 mc
 =   ...( me = m ; mp = M)  0 =
λe M  3 h

416

Chapter 14: Dual Nature of Radiation and Matter

72. The momentum of the final particle 1240


   78. EPh = eV = 2.48 eV
p f = p1 + p 2 500
K.Emax = EPh – 0 = 2.48 – 2.28 = 0.2 eV
 | pf| = p12  p 22 For electron,
h 12.28 12.28
From de-Broglie relation, we have, p = λmin = Å= Å = 27.459 Å
 K.E max (eV) 0.2
 For the two particles, = 27.459 × 10–10 m
h h λmin.= 2.7459 × 10–9 m
p1 = and p2 =
1 2 λ ≥ λmin
h h2 h2 79. Kinetic energy of neutron in thermal
 = 
 12  22 3
equilibrium is kT
1 1 2

ns
1
 = 2 2
2 1  2 h h h
= = 
mv 2m(K.E.) 3 
1 2m  kT 
73.  2 
V

io
h
To decrease wavelength potential difference =
between anode and filament is increased. 3mkT

at
75. Power of the incident photons is, 80. =
n hc mv
P=
t  1
 
But P = Fc v

 Fc =
n hc
lic Now, v  T
t  1
 
n F 6.62  105  5  107 T
  = 34
= 5  1022
t h 6.62  10  27
ub
T927
 
76. Power of the incident photons is,  927 T27

P=
E  27 = 2 927
t  27 
 927 = 
P

But, P = Fc 2 2
 E = Fct
81. de-Broglie’s formula is
= 2  107  3  108  10  109
h
= 600  109 J =
et

= 600 nJ 2m (K.E)
But kinetic energy of thermal neutrons is kBT
77. By conservation of linear momentum,
where, kB = Boltzmann constant
m
rg

mv = mv1 + v2 6.63  1034


2  =
where, v1 and v2 are velocities of particles A and 2  1.67  1027  1.38  1023  (27  273)
B after collision.   = 1.78  1010 m = 1.78 Å
Ta

 2v = 2v1 + v2 ….(i)
82. We know that de-Broglie wavelength,
As collision is head on and elastic,
h
v 2  v1 =
e= =1 mv
u1  u 2
Velocity of a body falling from height H is
 v = v2 – v1 ....(ii) given by
Solving equation (i) and (ii),
v = 2gH
3v2
v = 3v1 and v = h h
4  = =
1 m 2gH m 2g H
As,  
p h
Here, is a constant say ‘K’.
m m 2g
v
 A p2 2 2 (4 / 3)v
   = =2  
1
 B p1 mv1 (2 / 3)v H

417

MHT-CET Triumph Physics (Hints)

Hints to Evaluation Test

2. We know that, 1250


4. EPh = eV = 1.67 eV
1 hc 750
mv2 =  0
2  K.Emax = EPh – 0 = 1.67 – 1.35 = 0.32 eV
34
(6.6310 )(310 ) 8 For electron,
=  3.0  1.6  1019 12.28
120109 λmin = Å
K.E max (eV)
= 16.57  1019  4.8  1019
= 11.77  1019 J 12.28
= Å = 21.708 Å
 mv2 = 2  (11.77  1019) 0.32
= 21.708 × 10–10 m

ns
or mv2 = 23.54  1019 J
λmin.= 2.1708 × 10–9 m
23.541019 λ ≥ λmin
v= = 1.61  106
9.11031
5. Stopping potential does not depend upon the

io
mv 2 mv distance of source from photocell but saturation
Now, Bev = r=
r Be  1 
current   
9.11031  square of distanceof source 
r=  1.61  106

at
(4105 )(1.6 1019 ) 1 1
 I1  and I2 
or r = 0.228 m  0.23 m (0.2) 2 (0.4) 2

3. For the first wavelength: I2 (0.2) 2



eVs1 = h1  0
lic
….(i)
=
12 (0.4) 2
2
For the second surface:  0.2 
or I2 = 12   = 3 mA
eVs2 = h2  0 ….(ii)  0.4 
ub
Subtracting equation (i) from equation (ii), hc hc 12500 eV Å
6. 0 =  max = = = 3125Å
h  max 0 4 eV
Vs2  Vs1 = (2  1)
e
7. de-Broglie wavelength,
hc  1 1 
P

or Vs2 = Vs1 +    h
e   2 1  =
p
hc  1   2  But p = 2mE
= Vs1 +  
et

e   21  1
 
 450  120  E
= 0.2 + 1240  
 120  450  i.e., E 
1
rg

 hc  2
....   1240eV  nm  2 2
 e   E 2   1   200 
      4
= 7.78 V  E1    2   100 
Ta

From equation (i), As, E2 = 4E1


hc E = 3E1
0 =  eVs1 J
1
 2 ( 0 1 ) 2
0 hc
8. Here, =
=  Vs 1 eV 1 ( 0  2 ) 1
e e 1
or
5.4  0  3.4 107 
=
2
1240
=
450
 0.2 eV 3.4  0  5.4 10 
7
1

= 2.56 eV or 0 = 12.7  107 m


hc
0 2.56  1.6  1019 Now, 0 =
0 = = 0
h 6.6  1034
= 0.62  1015 =
 6.610  310 
34 8

= 0.98 eV
= 6.2  1014 Hz 12.710 1.610 
7 19

418

Chapter 14: Dual Nature of Radiation and Matter

9. Saturation current depends on intensity. Hence Subtracting equation (i) from 3  equation (ii)
B and C will have same intensity different from we get,
that of A. Stopping potential depends on  3  hc hc
frequency. So A and B will have the same   1 = 30  0 or 0 =
2   4
frequency different from that of C.
hc
Hence option (A) is correct. But 0 = , where 0 is the threshold
0
h h h wavelength, hence 0 = 4.
10. = = =
p 2mK 2mqV Hence, option (C) is correct.
h h
 P = ;  =
2m p (q p )V 2m  (q  )V
h h
Now, P =   =

ns
2m p q p V 2m  q  V
 m p q p V = m  q  V
m Pq P V (1)(1)V V
 V = = = volt
mq  (4)(2) 8

io
12. Gain in K.E. = Loss in P.E.
p2
 = qV  p = 2mqV

at
2m
pp 2m p (e)V
 =
p 2m  (2e)V

=
mP  e  1 1
lic
 = .
m  2e  4 2
1
=
ub
2 2
nhc
13. Using E = , we get

n(6.6 1034 )(3108 )
107 =
P

(30001010 )
 n = 1.5  1011
et

hc (6.6 1034 )(3108 )


14. 0 = = = 300 nm
0 4.1251.61019

1
15. Using E = mv2 = h0  W0 we get,
rg

2
E1 = (1  0.6) = 0.4 eV
and E2 = (2.5  0.6) = 1.9 eV
v2
Ta

E1 0.4
 = 12 =  0.21
E2 v2 1.9
v1
or = 0.458  0.5
v2

17. E = eV = 1.6 10–19 20  103 = 3.2  10–15 J


18. Using Einstein’s photoelectric equation,
h = 0 + Kmax
hc
 = 0 + e(3V0) ….(i)

( Kmax = eVs)
hc
Also, = 0 + eV0 ….(ii)
2

419
15
Doin
Structure of Atoms and Nuclei

Hints

51. E = 0 and
Classical Thinking 13.6

ns
E5 = =  0.544 eV
25
1 1
16. Wave number = =  E = E  E5
 6000  1010
6 1 = 0  (0.54) = 0.54 eV
= 1.66  10 m

io
1
17. Refer Shortcut 2 52. T.E. = (P.E.) =  (K.E.)
2
20. As n increases, energy difference between K.E. 1
=

at
adjacent energy levels decreases. P.E. 2
23. R  m. Thus, if mass is reduced to half, then 53. Minimum energy required to excite from ground
Rydberg constant also becomes half. lic state
25. Energy is absorbed when atom goes from lower 1 1 
 13.6  2  2   10.2 eV
state to higher state. 1 2 
30. As difference between the levels increases, energy 57. For 7N13, N = 13  7 = 6 and
emitted increases and hence wavelength decreases.
ub
for 6C12, N = 12  6 = 6
It means colour must change to violet.
As number of neutrons is same, they are
31. If the energy radiated in the transition be E, isotones.
then we have,
58. They have same mass number (A).
E R G  E QS  E R S  E QR  E PQ
P

For getting blue line, the energy radiated should 68. Using, R  A1/3
1/3 1/3
 1 R Li  Li 7   7  1
be maximum  E   .     =
et

  R Fe  Fe56  56
  2

34. Energy increases from lower state to higher state. 72. Since nuclear density is constant,
 mass  volume.
0 n 2 h 2
rg

38. Bohr radius, rn =


me 2 73. Actual mass of the nucleus is always less than
total mass of nucleons
39. r  n2  r  (3)2
 M  (NM n  ZM p )
Ta

2
40. rn
r1 n2 1 1
2 76. B.E. per nucleon is maximum for Fe56.
 = 12 =   =
r2 n2 2 4 77. Binding energy per nucleon increases with
atomic number. The greater the binding energy
1 v1 n 2 per nucleon, the stability of the nucleus will be
42. v  = 2 =
n v2 n1 1 more.
For 26Fe56, number of nucleons is 56.
43. A  r2, but r  n2
This is the most stable nucleus because
 A  n4
maximum energy is needed to pull a nucleon
1 away from it.
49. En 
n2 A 0 A 4 
1  2 He 
78. zx   z + 1Y  
E3 (5) 2 25
 = = A4 0 A4
E5 (3) 2
9 z  1K 
0
 z  1K

420

Chapter 15: Structure of Atoms and Nuclei


0.6931  1 0.6931  1 16 16
80. T= =  1620 years  = = = 9.72  10–8
 4.28  104 15R 15  1.097  107
81. Fraction of sample after n-half-lives is given by = 97.2  10–9 m
N 1 = 97.2 nm  97 nm

N 0 2n 1 1 1 
Where; n = t/T
7. For Lyman series,

= RH 12  n 2  ,
 
t/T 10/5
1 1 where n = 2, 3, 4,….
Nt = N0   = 50000   = 12500
2  2
  For shortest wavelength, n = 
1 RH
82. In fusion, two lighter nuclei combines which is  
not the radioactive decay.  1
1
 =

ns
RH
Critical Thinking 1
=
1. Using Shortcut 1 109678cm 1
n(n  1) 3(3  1) = 9.117  10–6 cm = 911.7 Å.
Number of lines, NE = = =3

io
2 2
1 1
2. For Lyman series, n1 = 1, n2 =   Br  2  2 11 / 9 44
= 
5 6 
8. = =
 1 1 

at
1 1 1  Pf  1 1 9/4 81
 =R  2  2 =R  2  =R  2 2
  n1 n 2  1   4 5 
1    1/
 =
R
lic 
 Br
 Pf

= Pf =
 Br
81
44
1 1 1  3R 4
3. = R 2  2 = , L =
L  1 2  4 3R
1 1
R = 1.0967  107 m1 = 1.0967  105 cm1 B  2  2 3/ 4 27
ub
= 
1 2 
4 9. = =
 L = cm L  1 1 5 / 36 5
3  109670  2  2
2 3 
1  1 1  15R 5 5
4.  R 2  2    L = B =  6563 = 1215.4 Å  1215 Å
P

  1 4  16 27 27
16 16 10. For Balmer series,
    105 cm
15R 15  1 1
= R 2  2
et

c 3  1010 B 2 n 
 n   2.81  1015 Hz
  16 5  And for Paschen series,
  10 
 15  1 1 1 
= R 2  2
rg

5. Least energetic wavelength in Paschen P 3 n 


coressponds to n= 3 and m = 4. Now, for series limit, n = 
1  1 1  7R  1 
  R 2  2  = 1
P  3 4  144    
Ta


 B  = 4
 Least energetic wave number, 
 1  1
7R    
 =  P  9
144
P 9
6. Given that, we get six wavelengths.  =
B 4
From Shortcut 1,
9
Maximum number of spectral lines,  p =  6400 = 91600 = 14400 Å
n(n  1) 4
= 6 which on solving gives n = 4
2 11. Frequency of radiation emitted
1  1 1 1  8  1 1
Using = R 1  2  we get,  = Rc  7
 2  = 10  3  10  2  2 
  4  2
n f n i  2 3 
1  1 15R 5 5
= R 1   = = 3  1015  = 1015  4  1014 Hz
  16  16 9  4 12

421

MHT-CET Triumph Physics (Hints)

1 1 1 1 1 1
12. =R  2 2 = 
  n1 n 2   AB 2000 6000
1  n2  n2  2 1
 = R  2 2 21  = =
 6000 3000
 n1 n 2 
 AB = 3000 Å
1  n12 n 22 
 =  
R  n 22  n12  1  1 1 3R
16. = R 2  2  =
36 1  n12 n 22   2 4  16
 =  
5R R  n 22  n12  16 16
 = =  105 cm
 n n = 36 and n  n = 5
2
1
2
2
2
2
2
1
3R 3
 On simplifying these two equations, we get c 3  1010
 n= =
n2 = 3, n1 = 2   16 5 

ns
  10 
 3 
13. For longest wavelength in Lyman series,
n1 = 1, n2 = 2 9
=  1015 Hz
1 1 1  4 16
 = R  2  2   L =

io
L 1 2  3R 17. For Lyman series,
For shortest wavelength n1 = 1, n2 =  1 1 1  3
1 1  1 = R 2  2 = R
 max 1 2  4

at
 = R  2  0   S =
S 1  R
1 1 1  R
L 4R 4 = R 2  2
=
 = =  min 1   1
S 3R 3

 L =
4
 912 = 1216 Å
lic 
 max
 min
=
4
3
3

1 1 1   4  1 3R 18. The wavelength of spectral line in Balmer series


14. = R  2  2 = R   = 4 1 1 1
ub
1 1 2   4  is given by

=R  22  n 2 
4  
 1 = = 121.6 nm ....(i) For first line of Balmer series, n = 3
3R
1 1 1 16  4  12R 1 1 1 5R
Let = R 2  2 = R   = R 2  2 = ;
=
P

2 2 4   64  64 1 2 3  36
64 For second line, n = 4
 2 = ....(ii)
12R 1 1 1 3R
 = R 2  2 =
et

From equations (i) and (ii), 2 2 4  16


2 2 64 3R 2 20
= =  =4  =
1 121.6 12R 4 1 27
rg

 2 = 4  121.6 = 486.4 nm 20
 1 =  6561 = 4860 Å
1 1 27
15. = RH  12  12  =
 AC  C A  2000 19. For Paschen series
Ta

1 1 1  1 1 1 
and = RH  2  2    R  2  2  ; n = 4, 5, 6....
 BC  C B   3 n 
1 For first member of Paschen series n = 4
=
6000 1 1 1 1 7R
R 2  2 
1 1 1  1  3 4  1 144
 = RH  2  2 
 AB B A  144 144
 R=  = 1.1  107
1 1 1 1  71 7  18800  1010
= RH  2  2  2  2 
 B C C A  For shortest wave length n = 
1 1  1 1  1 1 1  R
= R H  2  2   RH  2  2  So R 2  2 
C A  C B   3   9
1 1 9 9
=   =   8.182  107 m  8182Å
 AC  BC R 1.1  107

422

Chapter 15: Structure of Atoms and Nuclei


20. For Balmer series, 25. The absorption lines are obtained when the
1 1 1  electron jumps from ground state (n = 1) to the
= RZ2  2  2  higher energy states. Thus only 1, 2 and 3 lines
 2 n 
will be obtained.
where, n = 3, 4, 5
For second line n = 4, 1 1 1
26. = R 2  2
1  1 1 3   n1 n 2 
 = RZ2  2  2  = RZ2
 2 4  16
1 1 1 
Assuming atom to be hydrogen, Z = 1,  10
= 1.097  107  2  2 
970.6  10 1 n 2 
16
 =  n2  4
3R
From Shortcut 1,
1 1 1 5  Number of emission lines,

ns
21.  RZ2  2  2  = RZ2  
B 2 3   36  n(n  1) 4  3
N= = =6
2 2
1 1 1  9 
 RZ2  2  2  = RZ2 
 Br 4 5   400  27. Radius of 1st Bohr orbit is,

io
36 400 r1 = 0.053 nm = 0.053  109 m = 0.53  108 cm
 B =  Br =
5RZ2 9RZ2
1
B 36 9RZ2 28. L  n and p 

at
   = 0.162 n
 Br 5RZ2 400
 L  p  1  L  p  n0
22. For Brackett series,
nh
29. Angular momentum = mvr =
1
 max
= R
1 1
 4 2
 2 =
5  25
9
 16
lic
R and 2
 Angular momentum  n
1 1 1  R n1 1
= R 2  2 =  Ratio = =
 min 4   16 n2 2
ub
 max 25
 = 30. rn  n2
 min 9
2
rn n
 =   = (4)2 = 16
23. For Lyman series, r0 1
P

1 1 1   rn = 16  0.53 = 8.48 Å
= RZ2  2  2 
 1 n 
31. rn  n2
1  1 2

et

 RZ2 1   = RZ ….(i)  A  r2  n4
 min    4 4
For Paschen series, A2 n   3
 =  2  =   = 81
2 A1 n
 1 1
1  1 1  7RZ
rg

 RZ2  2  2   ….(ii)
 m ax  3 4  144  A2 = 81 A1 = 81 A
 By dividing equation (i) by equation (ii), e2 e2
 max 144 144 32. vn =  v1 =
 RZ2   20 hn 2 0 h
Ta

 min 7RZ 2 7
v1 e2
144  
 max =  912  18761 Å c 20 hc
7
1 v n 1
 1 1  33. vn   3 1
24. = R 2  2 n v1 n 3 3
n
 1 n 2 
2.1  106
X=R ….(Lyman series)  v3 = = 0.7  106 m/s
3
1
Z = R  ….(Balmer series)
4   34. Linear speed in 1st Bohr orbit is,
1.6 1019 
2
 1 3 e2
 Y = R 1   = R v= =
 4 4 2ε 0 h 2  8.85  1012  6.63  1034
From above, X = Y + Z  Z = X – Y  2.25  106 m/s
423

MHT-CET Triumph Physics (Hints)

 hc  Z
2
1 43. Change in angular momentum of electron,
35. E    2    2
  n Z h 6.64  1034
L5  L4 = [5  4] =
20.397 2 2  3.14 
    5.099cm
He 4 = 1.051034 J-s
36. P.E.  2  Total energy 13.6
44. Using, En = ,
= 2  (13.6)  27.2eV n2
13.6
2r n 2h 2 E3 = = 1.51 eV
37. T ; r = radius of nth orbit  32
v mZe 2
ze 2 13.6
v = speed of e in nth orbit  45. Energy of electron, En = eV
20 nh n2
13.6
  0.544 eV =

ns
4 2 n 3 h 3 n3 eV
 T 0 2 4  T 2 n2
mZ e Z
 n2 = 25  n = 5
13.6 Orbital velocity of electron in orbit n = 5,
38. En   Z2 . For first excited state, n = 2 and
n2 e2 e2 v

io
++ vn = = =
for Li , Z = 3 20hn 20h(5) 5
13.6
 E=  9  30.6 eV B
4 46. En = where B = 16  10–18 J

at
n2
39. n = 3 (– 1.51 eV)
16  1018 16  1018
E3  E4 = 2
= = 1  10–18 J
n = 2 (– 3.4 eV)
lic (4) 16
16  10 18 16  1018
n = 1 (– 13.6 eV)  E2 = = = 4  10–18 J
 2
2
4
E32 =  3.4  ( 1.51) =  1.89 eV hc
 |E32|  1.9 eV Let E2 – E4 = h =

ub
40. E2 hc h  3  108 h  3  108
 = = =
E2  E4 (4  1)  1018 3  1018
2.3 eV
= 1026 h
E1
P

47. Since for n = 3,


Using, E2  E1 = h we get,
E3 = 13.6 =  1.51 eV
E  E 2.3  1.6  1019 J 2
3
= 2 1
h 6.6  1034 Js 13.6
et

For n = 1, E1 = =  13.6 eV
= 0.56  1015 s1 = 5.6  1014 Hz 12
2r  The energy of the photon emitted in the
41. T= , r  n2 and transition from n = 3 to n = 1 is
v
rg

1 E3  E1 = ( 1.51)  ( 13.6) = 12.09 eV.


v   T  n3
n Mass of nucleus
48. Density of nucleus =
T1 n13 1 Volumeof nucleus
  =
Ta

3
T2 (2n1 ) 8 A  1.66  1027
=
4
 1.1  1015   A
3
42. Radius of hydrogen atom in the ground state,
r1 = 5.3  1011 m. (n1 = 1) 3
Radius of hydrogen atom in the excited state, = 2.97  1017 kg m3.
r2 = 13.25  1010 m. Since, density of nucleus is independent of mass
For a hydrogen atom, number, hence density of all nuclei is same.
r  n2 49. Using R = R0 A1/3,
2 1/3
r1 n  R1 A  R A
1/3
 =  1 =  1   
r2  n 2  R2  A2  R He  4 
5.3  1011 2 1/ 3
 10
= n1 2  (14)1/3 =  
A
13.25  10 n2 4
 n 22 = 25  n2 = 5  A = 56  Z = 56 – 30 = 26

424

Chapter 15: Structure of Atoms and Nuclei


50. R  (1)1/3 60. 4 Be9  2 He 4 6C12  0 n1 .
 R80  (80)1/3 and R10  (10)1/3
1/3 61. From Shortcut 4,
R 80  80 
 =   = (8)1/3 = 2 A  A 232  208
R10  10  n =  =6
4 4
 R80 = 2  R10 = 2  3  10–15 = 6  10–15 m
n = (2n  Z + Z) = (2  6  90 + 82) = 4
51. Matter is not uniformly distributed inside the 0
238 234  234 ( 1  ) 234
nucleus. 62. 92U

  90Th   91Pa  92U

52. E = mc2 = 3  (3  108)2 = 27  1016 J 238  222


63. Number of -particles emitted = =4
53. m = 1  0.993 = 0.007 g 4
 E = (m)c2 .... (Refer Shortcut 4)
= (0.007  103) (3  108)2 = 63  1010 J. This decreases atomic number to 90  4  2 = 82

ns
Since atomic number of 83 Y 222 is 83, this is
54. Let the percentage of B10 atoms be x.
 Percentage of B11 atom = (100  x) possible if one  particle is emitted.
Using Mindbenders 1 64. Average life

io
Average atomic weight Sum of all lives of all the atom 1
10x  11(100  x) T= =
  10.81  x  19 Total number of atoms 
100  T = 1

at
N 10 19
 B
 65. Fraction of sample after n-half-lives is given by
N 11 81
B
N 1

55. The equation is O  0 n  O
17 1 16 N 0 2n
 Energy required = B.E. of O17 – B.E. of O16
lic Where; n = t/T
15/5
= 17  7.75 – 16  7.97 N 1
  
1
= 4.23 MeV N0  2  8
ub
56. Binding energy per nucleon, 1 7
 Decayed fraction = 1  =
Eb 8 8
Ebn =
A 66. Fraction of sample after n-half-lives is given by
For deuteron, A = 2 N 1

P

E
 1.115 MeV = b  Eb = 2  1.115 MeV N 0 2n
A
Where; n = t/T
Now, Eb = mc2 t /T
Mass defect, 1
et

N = N0  
2  1.115 2  
m = u ….[ 1u = 931.5 MeV/c2] 30/ T
931.5 N0 1
 = N0  
= 0.0024 u 64 2
rg

57. Energy is released in a process when total 30


 T= =5s
binding energy (B.E.) of the nucleus is 6
increased or we can say when total B.E. of
67. Fraction that remains after n half lives,
Ta

products is more than the reactants. By n t/T


calculation, we can see that only in case of N 1 1
=  =  
option (C), this happens. N0 2 2
Given W  2Y T/2
1/ 2
1
=  
N 1 T 1
B.E. of reactants = 120  7.5 = 900 MeV and  =   =
B.E. of products = 2  (60  8.5) = 1020 MeV N0 2 2 2
i.e., B.E. of products > B.E. of reactants. dN
68. =  N
58. B  [ZM p  N M n  M(N, Z)]c 2
dt
 M(N,Z)  ZM p  N M n  B / c2 dN 0.693
 = N
dt T1/ 2
59. B.E. = mc2 0.693
= [2(1.0087  1.0073)– 4.0015]  931 =  4  1015
1.2  107
= 28.4 MeV = 2.3  108 atoms/s
425

MHT-CET Triumph Physics (Hints)

1
t/T
Dividing equation (i) by (ii) we get
69. Using N = N0   4 8 TY
2    
8 TX 8
 7 1
 N = 1   N0 = N0 1 TY T 2
 8 8   or X 
t/T
2 TX TY 1
1 1
 N0 = N0   60/ T
8 2 N0 1
75. = N0  
1
3
1
t /5
32 2
   =    t = 15 days
2 2 60
 5=  T = 12 days
T
dN
70. = N 76. From Mindbenders 2
dt

ns
dN T1T2 810  1620
 n =  N ….( = n) T  = 540 years
dt T1  T2 810  1620
n 1
 =  th of material remains after 1080 years.
N 4

io
0.693 0.693 0.693 N 77. Remaining amount
 Half-life = = = s
  n 32/ 2 16 12
1 1 1
= 16    = 16    =   < 1 mg

at
71. Using N = N0et, 2 2 2
N0
= N 0e  T1/ 2  2 = e T1/ 2 1
2 78. M  M 0e t ; Given t  2  

 By taking loge on both the sides,
loge 2 = T1/2  T1/2 = 0.693
lic  2
  
1
2

 M  10e 
 10    M = 1.35 g
e
72. A = A0et
975 = 9750 e5
2

ub
1 N 1
79. N = N0    
e5 = 10 2 N0 4
 5 = loge10 = 2.303 log10 10 = 2.303 N 1 3
Probability = 1  1 
   0.461 N0 4 4
P

N0 80. During fusion, binding energy of daughter


73. is the new N0
2 nucleus is always greater than the total energy
To reduce one fourth the time taken, of the parent nuclei so energy released
et

t  2  T1/ 2  = 2  40 = 80 years. = c – (a + b) = c – a – b
0.693 0.693 mv 2 1 e2
  =  0.0173years 81. 
T1/ 2 40 a0 40 a 02
rg

e
N 1
n
 v
74. As    ; where, Number of half lives, 40a 0 m
N0  2 
Ta

t 2r 2r
n= 82. v= T=
T T v
For sample X, e e ev
Now, I =  
1 1
8/ TX
1
4
1
8/ TX
T  2 r  2r
  or       
16  2  2 2  v 
8 1.6  1019  2.18  106
4= ….(i)  I=
TX 2  3.14  0.53  1010
For sample Y, = 1.04 10–3 A = 1.04 mA
8/ TY 8 8/ TY
 1  1 1 1 83. Since electron and positron annihilate,
   or     
 256   2  2 2 hc 6.6  1034  3  108
 
8 E Total (0.51  0.51)  106  1.6  1019
8= ….(ii)
TY  1.21 1012 m  0.012Å .

426

Chapter 15: Structure of Atoms and Nuclei


84. Ground state energy = (Ionisation potential) For n = 1,
= 13.6 eV r2 = h/2m
Ef = –13.6 + 12.1 = 1.5 eV 1 qh (1.6  1019 )(1.05  1034 )
 M= =
E i 13.6 4 m 2  9.1  1031
 Energy state, n2 =  =9
Ef 1.5 = 9.2  10–24 Am2
 n = 3 i.e., second excited state. 91. A photon is emitted when hydrogen atom comes
 Number of spectral lines from to first excited state i.e., n = 2
n(n  1) 3(2)  Energy transferred
n = 3 to n = 1 =  3
2 2 1 1
=  13.6  2  2 
85. For n = 1, maximum number of states = 2n2 2 1 
= 2 and for n = 2, 3, 4, maximum number of 3
states would be 8, 18, 32 respectively, Hence =  13.6 eV

ns
4
number of possible elements = 10.2 eV
= 2 + 8 + 18 + 32 = 60. By conservation of momentum,
86. The binding energy per nucleon of the nuclei of 1 1 1
mv2 = mv12 + mv22 + 10.2

io
high mass number is small as compared to that 2 2 2
of stable nuclei. Such nuclei undergo or v12  vv1 + 10.2 = 0 ….[eliminating v2]
radioactive decay so as to attain greater value of  v1 is real  v2  4  10.2
B. E. / A

at
4  10.2
mv 2 or vmin =
87. = qvB  mv = qBr m
r 1
= m  vmin 
2
 K.Emin
Now, mvr =
nh
2
lic 2
1 4  10.2
nh nh = m = 20.4 eV
 qBrn2 =  r 2n = 2 m
2 2qB
ub
88. Let the energy in A, B and C state be EA, EB and Competitive Thinking
EC, then from the figure
C 2. The hydrogen spectrum consists of different
1 series of spectral lines and each series can have
P

B infinite lines within itself. Hence, No. of


2 3 spectral line observed in hydrogen atom is .
A  1
1 1 
et

3. = R 2  2 
(EC  EA) = (EC  EB) + (EB  EA) λ  p n 
hc hc hc
  = a) For n = 5 to p = 4,
1  2 3 400
=
rg

 9R
 3 = 1 2
1   2 b) For n = 4 to p = 3
144
89. Mass of proton = mass of antiproton =
Ta

7R
 1.67  10 27 kg  1 amu
c) For n = 3 to p = 2
Energy equivalent to 1 amu  931 MeV 36
So energy equivalent to 2 amu  2  931 MeV =
5R
 1862  106  1.6  1019  3  1010 J . d) For n = 2 to p = 1
4
90. Using magnetic moment, =
3R
q
M = current  area = A   is minimum for n = 2 to p = 1 transition.
t
1  1 1 
 1 4.  RZ2  2  2 
 M=  q  r2 = qr2  n
 1 n 2 
2 2
But orbital angular momentum, For first number of Lyman series,
nh 1 1 1 4
L = mr2 =  RZ2  2  2   L =
2 L 1 2  3RZ2

427

MHT-CET Triumph Physics (Hints)


For first number of Paschen series, 4
b =
1 1 1 144 RZ2
 RZ2  2  2   P =
P 3 4  7RZ2 For last line of Lyman series: n1 = 1 and
L 4 / 3RZ 2
7 n2 = 
  
 P 144 / 7 RZ2 108 1 1 1 
 = RZ2  2  2 
l 1  
5. Given: R = 107 m1
1
For the last line of Balmer series, n1 = 2, l =
n2 =  RZ2
1 1 1  b (4 / RZ2 )
Wave number,  =  R  2  2   =4
 l (1 / RZ2 )
 n1 n 2 
107
= 0.25  107 m1  1 1 

ns
= 11.  = RZ2   2
4 2
 n1 n 2 
6. 0.58 eV
Series limit of Balmer: n1 = 2, n2 = 
0.85 eV
RZ2
 1 =

io
1.51 eV 4
3.4 eV n=2 Series limit of Paschen: n1 = 3, n2 = 
10.2 eV
13.6 eV n=1 RZ2

at
 2 =
9
It is clear that difference of 11.1 eV is not
possible to obtain. 1st line of Balmer series: n1 = 2, n2 = 3
2 2
 1 1  RZ RZ
 3 = RZ2   2=  = 1  2
7.  
 1 1 
 2
  n  1 n 
2
n 2  (n  1) 2

lic
= 2
2n 1 2
2 3  4 9
  n 2
(n 1) n (n 1) 2
12. Series limit for Lyman series is,
1
 For n >> 1,   3 1
n L =
ub
R
1 1 1 c
8.  RZ2  2  2   L = Rc ….(  = )
 n
 1 n  
1 Series limit for Pfund series is,
   2 for given n1 and n2
P

Z 25 Rc ν
p =  p = = L
 1 = 2 = 43 = 94 R 25 25

1  1 1  1 1 1
et

9. =R  2  2 13. = R 2  2
 n
 1 n 2   n
 1 n 2

Lyman series : n1 = 1, n2 = 2 1 1 1 5
 =   R
1 1 1   22 32  36
rg

=R  2 2 …(i)
 1 2  c 5
Balmer series:  f =  Rc
 36
n1 = 2, n2 = 3
Ta

1 1 1 14. Case I:
=R  2  2 …(ii)
 2 3  n = 3 to p = 2
Dividing equation (ii) by equation (i), 1 1 1 1 1
= R 2  2 = R  
1 1 5   p n  4 9
R  
 36 = 5  4  5
 
4 9
 1 5
  1 3 36 3 27 = R .…(i)
R 1    36
 4 4
Case II:
1 1 1 n = 4 to p = 3
10. = RZ2  2  2 
  n1 n 2  1  1 1  1 1 
= R 2  2 = R  
For last line of Balmer: n1 = 2 and n2 =    p n   9 16 
1 1 1  1 7
 = RZ2  2  2  = R ….(ii)
b  2    144

428

Chapter 15: Structure of Atoms and Nuclei


 Dividing equation (i) and (ii) 22. 4th excited state means n = 5 and 2nd excited
 5 144 20 state means n = 3
  
 36 7 7 E1 E
E5 = and E3 = 1
20 25 9
  = 
7 where, E1 = 13.6 eV
13.6 13.6
hc 6.63  1034  3  108  |E5  E3| =  = 0.967 eV
15. E=  eV 25 9
 975  1010  1.6  1019
 Energy of the destination orbit = 13.6 + 12.5 13.6 Z2
=  0.85 eV 23. Using En  we get,
n2
 The Hydrogen atom will be excited to n = 4
13.6(3) 2
Using Shortcut 1, E1 = 
(1) 2
4(4  1)

ns
Number of spectral lines = =6 13.6(3) 2
2 E3 = 
(3) 2
16. Let the energies in given three orbits be E1, E2
and E3 as shown in the figure.  E = E3  E1

io
 1
E3 n =3 = 13.6(3)2 1  
1  9
E2 n =2 13.6  9  8

at
=
2 3 9
E1 n =1  E = 108.8 eV

24. Hydrogen atom takes E amount of energy for


 (E3  E1) = (E3  E2) + (E2  EA)
hc hc hc
lic excitation from ground state (n = 1) to n = 3
 =  state.
3 1  2
 E = E3  E1
1 2
 3 =
ub
13.6
1   2 =   13.6 
 3
2

18. As electron transits from higher energy level to = 12.1 eV


lower, its n decreases, hence it K.E. increases.
This implies its velocity increases. This means 25. For an electron in a Bohr orbit in H-atom,
P

statements (A) and (B) are correct. K.E. = –T.E.


1 K.E. 1
Also, de Broglie wavelength   . Hence, as  =
v T.E. 1
et

velocity increases, associated de Broglie


i.e., 1 : 1
wavelength decreases. Hence, statement (D) is
correct. But, angular momentum L  n. This 26. T.E. = – K.E.
means, as energy level changes, associated
rg

Also, P.E. = – 2 K.E.


angular, momentum changes. Hence, statement  K.E. = + 3.4 eV
(C) is incorrect.
and P.E. = – 2  (+ 3.4) = – 6.8 eV
T2 23 8
Ta

19. T  n3    1
T1 13 1 27. K.E. =  T.E =  P.E.
2
 T2 = 8 T
Also, Total energy of 4th state of hydrogen atom
nh
20. Angular momentum, L = is
2
13.6
4h h 3h E4 = eV = 0.85 eV
 L = L4  L1 =   42
2 2 2 
 P.E = 1.7 eV, K.E = 0.85 eV
21. Ground state energy for hydrogen atom, 2
E1 =  13.6 eV  Z
28. K.E.    and
E1 n
Now, En =
n2 K.E. =  (T.E.), P.E. = 2(K.E.)
13.6 This implies as K.E. increases and as K.E.
 E2 = = 3.4 eV
22 increases, T.E., P.E. decreases.
429

MHT-CET Triumph Physics (Hints)

29. Ground state energy =  13.6 eV = E1 35. R = R0 (A)1/3


13.6 R 2  A2 
1/3 1/3
 64  4
Now, En =      
n2 R1  A1   27  3
13.6
 E2 = =  3.4 eV  R2 = 3.6 
4
= 4.8
2 3
 Energy released = 3.4 (13.6)2 = 10.2 eV
B.E. mc2
30. Radius of Bohr orbit is, 36. 
A eA
1 But 1 u = 931 MeV/c2
Rn 
m B.E. 0.03  931
 =
If radius of Bohr orbit for muon is R, A 4
Rμ me = 6.9825 MeV/nucleon

ns
Re mμ
37. Energy required to remove one neutron
R  m e 0.51  1013 ΔE = (17  7.75) – (16  7.97)
 R = e =
mμ 207 = 131.75 – 127.52

io
= 4.23 MeV
≈2.56  10–13 m
As, energy E  me, 38. Mass defect = m = 0.02866 u
Eμ mμ Total energy = E = mc2

at
 
Ee me = 0.02866  931 MeV
= 26.68 MeV
 Eµ = –13.6  207
E 26.68
= –2.8 keV Energy liberated per nucleon = =
31. To balance the atomic number and mass number
lic A
= 6.67 MeV
4

on both sides, 10 X  2 
40. X A   Y A  4   ZXA 4
 n
Z Z2
 X represents neutron 1
ub
0

42. 94 Pu 239  92 U 235  2 He 4


1
32. R = R 0A 3 Hence, the particle emitted when Pu decays into
1 1
U is, -particle.
R Te  A Te   125  3
3 5
P

=  =   = 44. An element is represented as AZ X


R Al  A Al   27  3
where, A is atomic mass No. Z is atomic
5 number.
 RTe = RAl
3 when a  particle is emitted,
et

1 A does not change, Z increases by 1


33. R = RoA 3 When an  particle is emitted:
1
A decreases by 4, Z decreases by 2.
rg

R Te  A Te  3
   
X 

 180    
73 X1   176
71 X 2  72 X 3   172
180 176
R Al  A Al  72 70 X 4

1 A A–12 A–12
45. ZX  Z–6(X) 0  Z–8(X)

 
15 125  3 4
3( 2  ) 2( 1  )
 RTe = 3.6  10
Ta


 27  Number of neutrons (A  12)  (Z  8)
 
5 Number of protons Z8
 RTe = 3.6  1015 
3 AZ4

 RTe = 6  1015m = 6 Fermi. Z8

34. R = R0A1/3 46. As emission of  doesn’t affect the atomic


R Ge  A Ge 
1/3 mass no. A, hence No. of  particle emitted to
   decrease A from 238 to 206 is
R Be  A Be 
238  206 32
2R Be = =8 .... (Refer Shortcut 4)
  (ABe)1/3 = (AGe)1/3 4 4
R Be (As single  decreases A by 4)
 23  9 = AGe Thus, 8 particles needs to be emitted to
 AGe = 72 decrease (A) from 238 to 206

430

Chapter 15: Structure of Atoms and Nuclei


But emitting 8 will bring down the atomic No. ln 2 0.7
Also, T1/2 = =
(Z) from 92 to 76.  
(As single  decrease Z by 2) 0.7
For A, A =
Thus 6  needs to be emitted to raise (Z) from 10
76 to 82.  0.7 
 t 
(As single  increases Z by 1)  MA = M0 e 10 
….(i)
0.7
47. A1 = N1 and A2 = N2 For B, B =
20
 A1  A 2 
 N1  N2 =   0.7 
 t 
    MB = M0 e  20 
….(ii)
 0.7 
 t 
48. Time taken to reduce from 2/3rd to 1/3rd should MA e  10  
 0.7 
t 

also be one half life i.e., 20 days.    0.7  = e 20 

MB  t 

ns
e 20 
N 1 1 1
49. = = = M 1
N0 (1+ 7) 8 (2)3 But, A =  
MB e
 1 1 
 n=3  n  3  
0.7
t = 1

io
 2 (2)  20
t 20 200
 n=  t = 3  20  t= = yrs
T 0.7 7

at
(Half-life of X = T = 20 years)
54. Half life T1/2 = 5 min
 t = 60 years Total time t = 20 min
X 1 t 20
50.   Number of half lives, n =  =4
Y 7
X 1 1
lic Now,
T1/ 2 5

   n 4
X  Y 8 23 N 1 1 
 
 3 half-lives N 0  2   2 
ub
 T = 3  1.4  109 years = 4.2  109 yrs. N 1
 
51. N = Noet N 0 16
No Disintegrated nuclei of given element will be,
  Noet
20  N0  N   N
P

  100  1   100
 ln 1  ln20 = t  N0   N0 
ln20
t=  6.93  1
ln2 = 1    100 = 93.75%
 16 
et

2.99  6.93
 t= = 29.9  30 days. 55. Nuclei remaining (N) = 600 – 450 = 150
0.693 n
N 1
52. Given: A = 8 , B = , (NB)0 = (NA)0 = N0 Comparing with = 
N0 2
rg

NA
For, NB = , 150  1 
n
1 1
n
e      
N e 8 t 600  2  4 2
N 0e t = 0 n=2
Ta

e
 t = 8t – 1 i.e., nuclei would disintegrate in two half-lives
 7t = 1 which in this case equals 20 minutes.
1 56. Using Mindbenders 2,
 t= 
7 T1T2 5103 105
T= = = 4762 yrs
Negative sign here, indicates process of T1  T2 5 103 105
disintegration,
0.693
1 57. T1/2 =
 t= 
7
1
Average life  =
53. We know 
N = N0et  T1/2 = 0.693 
Similarly for given masses, 10
M = M0et  = = 14.43 hours
0.693

431

MHT-CET Triumph Physics (Hints)

58. NA = N0et 20 5


 t = ln   ….(ii)
NB = N0  N0et 0.693  4 
N B N 0  N 0e t thus, t = t  t
  = 0.3
NA N0e t 20  5
 et  1 = 0.3
=  ln 5  ln 
0.693  4
 et = 1.3 20
 t = ln (1.3) = ln 4
0.693
ln (1.3)  ln (2)  = 40 min.
 t=T ….   
ln (2)  T  n
1
log(1.3) 62. Number of nuclei remaining N = N0  
t=T 2
log 2
For element A, TA = 20 min. Hence, 80 minutes,

ns
T N  correspond to 4 half lives.
59. t2  t1 = log e  1 
log e 2  N2   No. of nuclei decayed of A( NA )
20  50  = N0  N
= loge 

io
  1
log e 2  12.5  = N0 1
20  24 
= log e 4 = 40 minutes
Similarly for element B, TB = 40 min. Hence, 80
log e 2

at
minutes correspond to 2 half lives.
T   N   No. of nuclei decayed of B ( NB )
60. t= log e  0  
log e  2    N  lic = N0  N
T   N0   1
 t1 = log e  = N0 1
log e  2  

 N1    22 
 Taking ratio,
T   N0  
t2 = log e    15 
log e  2  
ub
 N 2   N0  
NA  16   5

T   N1   NB 3 4
 t2  t1 = log e   N0  
log e  2    N 2   4
For 40% decay, N1 = 60
P

t / T1/ 2
m 1
For 85% decay, N2 = 15 63.  
m0  2 
30   60  
 t2  t1 = log e    Given: T1/2 = 12.5 years, t = 50 years
log e  2    15  
et

50/12.5 4
m 1 1 1
30    =  
=  log e  4  m0  2   2  16
log e  2 
64
= 30  2 = 60 min  m= = 4 mg
rg

16
61. We know for radioactive decay, 64. Half life T = 10 days, t = 5 days
t N t 5 1
N = N0e (or) ln 0 = t  n=  
Ta

N T 10 2
For 20% decay N 1000X 1000X
1 N N = n0  1/ 2 = = 0.707  1000X
t= ln 0 2 2 2
 N
= 707X
20  100   0.693 
t=  ln  ….     65. Given:
0.693  20   T 
20
T1/2 = 2.2  109 s and rate of disintegration,
t= ln (5) ….(i) 0.693  0.693 
0.693 R = N = N ....    
For 80% decay T1/ 2  T1/ 2 
1 N 10 0.693
t = ln 0  10 = N
 N 2.2  109
20  100  2.2  109  1010
 t = ln    N = = 3.17  1019
0.693  80  0.693

432

Chapter 15: Structure of Atoms and Nuclei


66. Let rate of disintegration 10,000 dis/min be 71. N = N0et
taken as initial rate (N0) and let N0 1
N = 2500 dis/min. = N0e(5)   =
e 5
N
= et N0
N0  = N0e(t)  t = 5 loge 2
2
2500
 = e  4 ….(Given : t = 4 min)
10000 0.693 0.693
72.    0.03465
1 T1/ 2 20
 = e4  e4 = 4
4 2.303  N 0 
t log  
 4 = loge 4  4 = loge 22   N
2
 4 = 2 loge 2  = loge 2 2.303  100 
4  t1  log    11.6min
0.03465  67 

ns
  = 0.5 loge 2
2.303  100 
dN and t 2  log    32min
67. = –N 0.03465  33 
dt
Hence time difference between points of time
Where, negative sign indicates that nuclei

io
disintegrate = t1 – t2 = 32 – 11.6
dN = 20.4 min  20 min.
Given: = –55.3  1011
dt

at
ln2
 55.3  1011 = (7.9  10–10)  N 73. Half-life = 6 min. =

 N = 7  1021 ln2
l=
68. The number of nuclei decayed in 2 days is,
N2 = N0e–2/
lic 6
0.692
l=
Similarly, in 3 days, the number of nuclei 6
decayed will be,
 at t = 0, 1024 particles per minute
N3 = N0e–3/
ub
t1/ 2 3 After 42 minute, 7 half-life is complete
where  =  ....(i) 1024
ln2 ln2  no. of particles =
 Fractional Decay on third day 27
 N 0e2/   N 0e 3/   No. of particle = 8
P

N 2  N3
= = 
N0 N0 74. Rate disintegration, R = N0et
2 l n 2
0.693
= e 3
 el n 2 ....[using (i)] =
et

2
T
= 2 3  21 = 0.63 – 0.5 = 0.13 0.693
 R= N0 e0.693 t/T
T
1
69. By using N  N 0e t and average life time t  0.693 0.693
rg

 R1 = N0 e0.693  12/2 = N0 e6(0.693)


2 2
 N  N 0e 1/   N 0e 1
0.693 0.693
N 1 R2 = N0 e0.693  12/4 = N0 e3(0.693)
  e 1  4 4
Ta

N0 e
4
N 1 e 1  R1 : R2 =  e3(0.693) = 0.25 = 1 : 4
 Disintegrated fraction  1  1  2
N0 e e
75. Let X have atomic number Z and mass number
N 01 N 02 A
70. N1 = , N2 
(2) t / 20 (2) t /10  Z X  Z  2 Y + 2 He
A A 4 4

N1 = N2 ….[Given] 
 t  But A4
Z 2 Y A
Z Z + 2e i.e.,
40 160 2 
   2 t / 20  2 10  A4
Y A
(2) t / 20
(2) t /10
Z2 Z Z 1
0
e + 10 e


t t
 2 
t t
 2
 A  = A  4 and Z Z
20 10 20 10 Since X and Z has same atomic number and
t different mass numbers, they are isotopes of
 = 2  t = 40 s
20 each other.
433

MHT-CET Triumph Physics (Hints)

76. XY+Z 81. For transition n = 2 to n = 1,


Now, Py = Pz (P  linear momentum) Energy released ΔE1 = 13.6 –
13.6
 2
2
myvy = mzvz
my vz 2
   =2  1
mz vy 1 = 13.6 1  
4  
 Ay = 2Az 3
1/3 = × 13.6 = 10.2 eV
R (A )1/3  1  4
Now, z  Z 1/3   
R y (A y ) 2 Now, He+ ion absorbs this 10.2 eV energy in
transition from n = 2 to say n = n
 1 : 21/3
1 1 
77. Q = 2(B.E. of He)  (B.E. of Li)  ΔE2 = 13.6 × 4 ×   2
 4  n  

ns
= 2  (4  7.06)  (7  5.60)
= 56.48  39.2 13.6  4
 10.2 = 13.6 –
 n 
2
 17.3 MeV
 n2 = 16

io
hc
78. E =  n = 4

 possible transition is n = 2 → n = 4
From energy level diagram,
For He+ ions in state n = 1,

at
hc hc
1 = = 13.6  4
[ E  ( 2E)] E 10.2 = 13.6 × 4 –
 n 
2

hc hc
2 =   (n)2 = 1.23
  4E    E 
  E   3    3 
  
lic As n does not have integral value, there is no
possible transition from n = 1 of He+ ions.
1 1
 
2 3 82. E = E1 – E2
ub
13.6 13.6
1  1 1   E =  2
79. = R 2  2  1 2
  n p  3
E = 13.6  = 10.2 eV
In this case, n = 1 and p = 4 4
P

1 1 1  1 15 This is the energy associated with emitted photon


 = R  2  2  = R 1   = R
 1 4   16  16 i.e., h = 10.2 eV
Energy of photon is given by, but according to photoelectric equations,
et

c 15 h = 0 + eV0
E = h = h = hcR ….(i)
 16  10.2 eV = 4.2 eV + eV0
According to Einstein mass-energy relation,  eV0 = 6 eV
E = mc2
rg

….(ii)
83. For least energetic photon emitted in Lyman
From equations (i) and (ii),
series, E = E2  E1 = 10.2 eV
15
mc2 = hcR hc 6.63  1034  3  108
16 = 
Ta

E 10.2  1.6  1019


15hRc
 c2 = = 1.2187  107 m  122 nm
16m
15hR 84. Using principle of momentum conservation,
c=
16m m1v1 = m2v2
3
hc v1 m 2  R 2 
80. Ephoton = (in eV)    m  A  R3
 v 2 m1  R1 
4  1015  3  108 3
v1  2  8
=   
300  109 v2  1  1
= 4 eV
For an electron in the ground state of hydrogen E 6 1.6 1013
85. Momentum of photon = 
atom first excitation energy is 10.2 eV. Since c 3 108
Ephoton < 10.2 eV no excitation is possible. = 3.2  1021 kg m/s
434

Chapter 15: Structure of Atoms and Nuclei


As the momentum is conserved in nuclear Energy released per fission(E)
88. Power = = Ef
reactions, momentum of nucleus Timefor onefission(T)
= 3.2  1021 kg m/s where,
 3.2  1021 
2
p 2 f = frequency = No. of fissions per second.
 (K.E.)nucleus =  P 6.4 6.4
2m 2  20 1.6 1027  f = = =
E 200  106  1.6  1019 200  1.6  1013
= 1.6  1016 J = 1,000 eV = 1keV
 f = 2  1011 per second.
86. Although the beta spectrum is a continuous
spectrum, the energy states of daughter nucleus 89 (K.E.)initial = (P.E.)closest approach
are discrete. 1 2Ze 2 1
 mv2 =  r0 
Binding energy of Hydrogen nucleus is zero 2 4 0 r0 m
whereas for Helium it is 28.3 MeV.
90. According to Bohr’s second postulate,

ns
mv 1 nh
87. Using r = and mv2 = eV0 mvrn =
qB 2 2
2meV0 nh
 r = =
1 2m
V0  2rn =
eB B e mv

io
h
B2 r 2e But, de-Broglie wavelength  =
 V0 = = 0.8 eV mv
2m
 2rn = n
For transition between 3 to 2,

at
Circumferenceof
1 1 13.6  5 No. of waves theorbit
E = 13.6    = = 1.88 eV contained =
4 9
  36 wavelength
in the orbit
 Work function = 1.88 eV  0.8 eV
= 1.08 eV
lic =
2rn

=n=2
 1.1 eV
ub
Hints to Evaluation Test

1. rn  n 2  A n  n 4 where, An = area 72  1011


Or m = = 8  10–5 kg
P

A1  2  16
4
(3  108 ) 2
   
A0  1  1 m = 0.08 g
Since 0.90 milligram (= 0.90  103 g) mass is
2. K.E. of an electron revolving in nth orbit is,
et

lost in 1 g uranium, hence for a mass loss of


e2 1 0.08 g the uranium required
K.E. =  K.E. 
8 0 rn r 1 0.08
=
Hence, to double the K.E. of electron, its orbit 0.90  103
rg

radius should be halved. = 88.88  89 g


3. Power to be obtained from power house Thus, to run the power house, 89 g uranium is
= 200 megawatt required per hour.
Ta

 Energy obtained per hour 4. Lyman series belongs to the ultraviolet region.
= 200 megawatt  1 hour
= (200  106 watt)  (3600 s) 5. K.E. =
13.6
eV, P.E. =
2(13.6)
eV
= 72  1010 J n2 n2
Here only 10% of output is utilized. In order For Hydrogen, Z = 1
to obtain 72  1010 J of useful energy, the  K.E = K.Ef  K.Ei
output energy from the power house  1 1 
(72  1010 )  100 K.E. = 13.6  2  2 
=  (2) (1) 
10
= – 10.2 eV (decrease)
= 72  1011 J
Let this energy be obtained from a mass-loss  1 1 
P.E. = –2(13.6)  2  2 
of m kg. Then  (2) (1) 
(m)c2 = 72  1011 = 20.4 eV (increase)
435

MHT-CET Triumph Physics (Hints)


nh f = number of revolution in one second
Angular momentum, L =
2 N v
=  n
h h t 2rn
 L = (2 – 1) =
2 2  Number of revolutions,
–34
= 1.05  10 J-s (increase) 1.1  106
N=ft=  10–8
2  2.12  1010
6. Number of nuclei remained after time t can be
written as N = N0et = 8.2  106 revolutions
1
N1 = N0e5t ….(i)  Period = = 1.2  107
t 8.2 106
and N2 = N0e ….(ii)
2
Dividing equation (i) by equation (ii), we get, 9. 1H + 1H2  2He4 + Energy
N1 1
 e( 5   )t = e4 t  4 t Binding energy (B.E.) of 1H2 = 2  1.1

ns
N2 e = 2.2 MeV
2
N1  1  1
2  B.E. of two 1H = 2  2.2 = 4.4 MeV
   2 ….[Given] B.E. of 2He4 nucleus = 4  7.1 = 28.4 MeV
N2  e  e
 Energy released when two 1H2 fuse to form

io
1 1
  4
2He = 28.4 – 4.4 = 24 MeV
e 2 e 4 t
2 1 10. For Balmer series,
 2 = 4t t= 

at
4 2 1 1 1
 RZ2  2  2 
 2 n 
7. For Lyman series, nf = 1 and ni = 2 2
and Z = 2(He) 1  1 1  5RZ
  RZ2  2  2   ....(i)
 1
E = 13.6 Z2  
1 

lic 
1
2 3 
1
36
1  3RZ2
n2 n 2   RZ2  2  2   ....(ii)
 i f 
 2 4  16
 3 
= –13.6 (2)2    = 13.6  3  Dividing equation (ii) by equation (i),
ub
 4    3RZ2 36 27
 Total available energy = 3  13.6 Joule   
 16 5RZ2 20
Ionization energy of Hydrogen = 13.6 eV
Now energy available to an electron after the N 1
t/T
P

ionisation of hydrogen, 11. Using,  


N0  2 
1
E = 3  13.6 – 13.6 = 2  13.6 eV = mev2 N 67
2  For 33% decay, 
N 0 100
et

1
 mev2 = 2  13.6 eV  67   1 
1 t /10
2   
   ….(i)
2  2  13.6  1.6  1019  100   2 
 v2 = N 33
rg

9.1  1031 For 67% decay, 


 v = 3.1  10 m/s 6 N 0 100
t /10
33  1  2
8. Orbital frequency,    ….(ii)
100  2 
Ta

v
f= n Dividing equation (ii) by equation (i) we get,
2rn t 2  t1
1  t 2  t1  /10
33  1 
     
2.2  106 Z 10 1 1
vn = m/s  
n 67  2    2
2
2.2  106 (1) t 2  t1
=  = 1 or t2 – t1 = 10 min
2 10
= 1.1  106 ms–1 2r 2r
Now radius, 12. v= T=
T v
n2 1
rn = 0.53  10–10 = 4  0.53  10–10 m But r  n2 and v 
Z n
….[ n = 2] r n2
 T   n3
= 2.12  10–10 m v (1 / n)

436

Chapter 15: Structure of Atoms and Nuclei


13. From law of conservation of momentum, For three energy levels, the possible transition
u are as shown in the diagram.
mu = 2 mv or v =
2 It is given, 1 < 2 < 3  1 > 2 > 3.
Excitation energy, The largest gap will correspond to h1
1 1 u 1
2 hc hc hc
E = mu2 – 2  m   = mu2 h1 = h2 + h3 or = 
2 2 2 4 1  2 3
Minimum excitation energy 1 1 1
  
1 1 1  2  3
= 13.6  2
 2  eV
1 2  nh
3
18. Angular momentum of nth orbit = .
=  13.6 2
4 nh
= 10.2 eV Again, mvr =

ns
2
1 nh
 (10.2)(1.6  10–19 J) = (1.0078)(1.66  10–27)u2  v= ….(i)
4 2mr
4 1
 u = 6.25  10 ms The time taken for completing an orbit
2r 2r(2mr)

io
2r 2r T= 
14. v= T= v nh
T v
e e ev 42 mr 2
Now, I =   Or T = ….(ii)

at
T  2r  2r nh
  Now, r = r0n2 ….[ r  n2]
 v 
42 mr02 n 4 42 mr02 n 3
15. Sum of masses of deutron and lithium nuclei  T= 
before disintegration
= 2.0147 + 6.0169
lic nh h
106
Number of orbits completed in 106 s =
= 8.0316 amu T
Mass of  particles 106  h
=
ub
= 2  4.0039 42 mr02 n 3
= 8.0078 amu 106  (6.63  1034 )
Difference of mass =
4(3.14)2 (9.1  1031 )(5.3  1011 )2 (2)3
= 8.0316 – 8.0078
= 8.22  108
P

= 0.0238 amu
Mass converted into energy 19. To ionize the H atom in ground state minimum
= 0.0238  931.3 MeV K.E. of photoelectron needed = 13.6 eV.
Energy given to each  particle  0 = 1.9 eV
et

0.0238  931.3
=  Minimum energy (or maximum wavelength)
2 incident = 13.6 + 1.9  16 eV
= 11.08 MeV 6.6  1034  3  108
rg

 max =
0.693 0.693 16  1.6  1019
16. For C14,  = 
T1/ 2 5730 = 77.3 nm  77 nm
–4 –1
  = 1.21  10 yr since A = 0.144 Bq and 20. E = mc2 = 1.5  (3  108)2 = 13.5  1016 J
Ta

A0 = 0.28 Bq
1  A0 
Using, A = A0et or t = ln  
  A
1  0.28 
t= ln  
1.21  104  0.144 
 5500 years
17.
h3

h2 h1

437
16 Semiconductor Devices

Hints

P 240 103
Classical Thinking 6. I= = = 48 mA
V 5

ns
3. The output of a diode rectifier contains some VS  VZ 30  10
AC component and hence it is a pulsating wave. 7. IS = = = 13.33 mA
RS 1.5  103
16. In lightly doped diodes, the necessary voltage is VO 10
IL = = = 5 mA
higher and avalanche breakdown is then the RL 2  103

io
chief process involved. IZ = IS – IL = 13.33 – 5 = 8.33 mA
19. Zener diode regulates above zener breakdown 8. Voltage across RL = 5 V
voltage.

at
VZ 5V
41. Current through load resistance,  I= = 3 = 5 mA
RL 10 
V 5
IL = L = = 0.01 A E  Vz 3
RL 500
53.
lic
The arrow head in the transistor symbol always
9. I=
Rs
=
100
= 0.03 A

VZ 6
shows the direction of hole flow in the emitter IL = = = 0.006 A
region. RL 1000
ub
IZ = I  IL = 0.03  0.006 = 0.024 A
56. When n-p-n transistor is used as an amplifier,
majority charge carrier electrons of n-type PZ = VzIz = 6  0.024 = 0.144 W
emitter move from emitter to base and then base 10. I(mA)
to collector. a
P

57. IE = IB + IC  IC = IE – IB
Vz
71. The Boolean expression for ‘NOR’ gate is V(V)
d c b
Y = AB
et

e
If A = B = 0(Low), Y = 0  0  0 = 1 (High)
I(A)
72. If inputs are A and B, then output for NAND
When the reverse bias is greater than the Vz, it is
rg

gate is Y = A  B
breakdown condition. In breakdown region,
 If A = B = 1, Y = 1  1 = 1 = 0
(Vi > Vz) for a wide range of load; (RL), the
voltage across RL remains constant though the
Ta

Critical Thinking current may change. Hence portion ‘de’ of the


characteristics is relevant for the zener diode to
2. The output waveform pattern is shown as, operate as a voltage regulator.
+ Second Third
First 14. Knee voltage for GaAsP LED is 1.5 eV.
V 15. In p-n-p transistors, majority charge carriers are
O
T T 3T 2T 5T Time
holes while in case of n-p-n transistors, majority
2 2 2
 charge carriers are electrons which have greater
mobility.
4. During the positive half cycle of the input A.C.
signal, diode D1 is forward biased and D2 is 16. If forward bias is made large, the majority
reverse biased. Hence in the output voltage charge carriers would move from the emitter to
signal, A and C are due to D1. During negative the collector through the base with high
half cycle of input A.C. signal, D2 conducts. velocity. This would give rise to excessive heat
Hence output signals B and D are due to D2. causing damage to transistor.
438

Chapter 16: Semiconductor Devices


IC 
17. dc =  IC = 99  20 = 1980 A 30. =
IB 1
 IE = IC + IB = 1980 + 20 = 2000 A 1 1 1 1  1 1 
  =  = =1
80     
18. IC =  IE
100 31. Peak value of IC = 1.414 mA
80
 24 =  IE  IE = 30 mA 1.414
100  r.m.s. value of IC = = 1 mA
Using, IE = IB + IC, 2
IB = 30 – 24 = 6 mA IC 10 3
 = = = 100
IB 10 10 6
19.  is the ratio of collector current and emitter
current while  is the ratio of collector current 40. When either of them conducts, the gate

ns
and base current. conducts.
20. Vb = IbRb 41. (i) 1 0 1 1
9 1
 Rb = = 257 k 0
35×106

io
1
0 1
 VBE 0.04 (ii) 0
21. ri = = = 1000  1
I B 40  106
1 0

at
22. Emitter-base junction is always forward-biased (iii) 1
0
and collector-base junction is reverse biased. 1

23. IE = IB + IC The outputs of (i), (ii) and (iii) are respectively


 IC = IE  IB
 IC = 4  0.04 = 3.96 mA
lic 43.
1, 1, 0.

I C 3.96 A
 =  = 0.99 A
IE 4 Y  AB
ub
B
24. In CE amplifier current gain, B
IC
=
IB A B A B AB AB A+B
IC =   IB = 80  250 A 0 0 1 1 1 0 0
P

 0.96 0 1 1 0 0 1 1
25. = = = 24 1 0 0 1 0 1 1
1  1  0.96
1 1 0 0 0 1 1
0.8
et

26.  = 0.8   = =4
(1  0.8) From above truth table,
I A B = A + B = A + B
  = C  IC =   IB = 4  6 = 24 mA.
IB
rg

27. IC =   IE = 0.98  2 = 1.96 mA Competitive Thinking


 IB = IE  IC = 2  1.96 = 0.04 mA.
1. Rectifier converts AC signal into pulsating DC
28. Current gain,
Ta

signal. Filter circuit filters DC signal while


IC
= regulator makes the DC value stable.
IB
2.
1103
 IB = = 105 A = 0.01 mA
100
By using IE = IB + IC
 IE = 0.01 + 1 = 1.01 mA
 Half wave 
I rectifier
29.  = C Also, IC = IE  IB
IB
I
50 Hz
 IC = IE  C

 80IC = 648  IC 50 Hz
 IC = 8 mA
439

MHT-CET Triumph Physics (Hints)


4. Zener breakdown voltage = 6 V 15. In active region of CE amplifier, the
 Potential across 4 k = 6 V collector-base junction is reverse biased while
and potential across 6 k = (10 – 6) = 4 V emitter-base junction is forward biased.
4
Current through the 6 k = 16. The input characteristics of the CE mode
6000
transistor (common emitter mode) represents the
2 2 variation of the input current (base current IB)
= A  mA
3000 3 with input voltage (base emitter voltage VBE) at
5. Potential difference across 800 Ω resistor constant output voltage (collector emitter
V800Ω = 5.6 V voltage VCE).
 current across 800 Ω resistor, 17. During positive half cycle due to forward
5.6 biasing, emitter current and consequently
I800Ω = A = 7 mA
800

ns
collector current increases.
Potential difference across 200 Ω resistor, As, VCE = VCC  ICRL, increase in collector
V200Ω = 9 – 5.6 = 3.4 V current causes decrease in collector voltage. This,
 current across 200 Ω resistor, as collector is connected to positive terminal of
3.4

io
I200Ω = A = 17 mA VCC battery, makes collector less positive,
200 i.e., negative with respect to initial value.
 Current through zener diode, Thus, during positive half cycle, unlike input
Iz = 17 – 7 = 10 mA. signal voltage, output signal voltage at collector

at
varies through a negative half cycle.
6. Current through the load resistance,
VZ 10 Similarly, it can be seen that, during negative
IL = = = 5 mA half cycle, unlike input signal voltage, output
2  103
Now,
RL
lic signal voltage at collector varies through a
positive half cycle.
IZ = Is  IL = 5IL This shows, in a CE amplifier, input and output
 IS = 6IL = 30 mA voltages are in opposite (180) phase.
ub
VS  VZ 16  10
 RS = = = 200 
IS 30  103 Alternate method:
For a CE amplifier,
7. 5.6 V
100  input signal voltage Vi = IB  RB
P

where, IB = change in base current and


10 V RB = input resistance of emitter base circuit.
IC
AC current gain ac =
et

10  5.6 4.4 IB


I = =
100 100 where, IC = change in collector current.
= 0.04 = 44  10–3 A As, VCE = VCC  ICRL, considering change in
rg

= 44 mA VCE,
10. The energy gap values for different colours are VCE = 0 – ICRL
different. [Since change in base current IB changes
Ta

(Eg)red < (Eg)yellow < (Eg)green < (Eg)blue. collector current, but not VCC]
Hence, their knee voltages are different  VCE = (ac  IB) RL
accordingly. Output voltage Vo = VCE
11. Base is thinnest layer in a transistor and has the  Voltage gain of CE amplifier,
width of about 3-5 m. Thus, its thickness is of Vo  I R R 
the order of a micro meter. AV = = ac B L = ac  L 
Vin I B R B  RB 
12. npn
E Negative sign indicates that output voltage is out
C
of phase (180) with respect to input voltage.

 + 18. For a transistor,


Forward B Reverse
biased biased   dc 
+ dc =  (1 – dc) = dc ….(i)
1   dc dc

440

Chapter 16: Semiconductor Devices


Simplifying the ratio given in the question, IC
26. =  IC = IB = 2  10–3 A
   IB
dc 1  dc  1   dc
dc   dc
=  dc  = dc VCC  ICRL = VCE  10 – (2  10–3) RL = 4
 dc dc  dc dc  dc  RL = 3 k
1  1   dc 
= ….[Using equation (i)] 27. The collector current is given by,
 dc VC 0.6V
dc   dc 1  1   dc IC =  = 1  103 A = 1 mA
 = =1 R C 600 
 dc dc  dc
IC I 1mA
=  IB  C  = 0.05 mA
 0.98 IB  20
19. =   49
1   0.02
28. For CE amplifier,
 dc 69 / 70

ns
20. dc = = = 69 Current gain =  =
IC
= 25
1   dc 1  (69 / 70) IB
21. IE = IB + IC But, IE = IC + IB
IE IB IE I I I
  1  = C B = C + 1 = 25 + 1 = 26

io
IC IC IB IB IB
1 1 1 20
   1  1  29. Here,
  19 19

at
Collector current, IC = 25 mA
19
  = 0.95 Base current, IB = 1 mA
20 As IE = IB + IC = (1 + 25) mA = 26 mA
IC IC 25
22. =
IE
= 0.95 ….[Given]
lic As  = 
IE 26
 IC = 0.95 IE
30. Vi = IB  RB
Now, IE = IC + IB  0.05 IE = IB
20
I 0.2 mA  IB = = 40  10–6 A = 40 A
ub
 IE = B = = 4 mA 500  103
0.05 0.05
VC = IC  RC
23.  = 45 20
Ic  IC = = 5  10–3 A = 5 mA
= 45 4  103
P

Ib
IC 5  103
V = IcR =   125
IB 40  106
 5 = Ic  1  103
Ic = 5  10–3 A
et

 IC
31. = = 60  IC = 60 IB
I 5  103 IB
Ib = c  = 0.111  10–3 A = 111 A
45 45 But IE = IB + IC  IB = IE IC
rg

24. Given:  IB = 6.6  60IB  61IB = 6.6


 IC = 0.49 mA IB = 0.108 mA
 IE = 0.50 mA 32. The input resistance is
We know,
Ta

 VBE 0.04
I C ri = 
=  IB 20  106
I B
 ri = 2  103 = 2 k
 IE =  IC +  IB
the A.C current gain is
  IB =  IE   IC = 0.50  0.49 = 0.01 mA
 IC 2mA 2  10 3
0.49 = = = = 100
 = = 49 I B 20 A 20  106
0.01
IC 33. Given
25. Current gain for common emitter is,  =
IB 0.5 = IC R
95% of I E 0.5
 = IC =
5% of I E 800
95 IC = 0.625 mA
 = = 19
5  IC + IB = IE

441

MHT-CET Triumph Physics (Hints)

IC 37. Given: RL = 2 k = 2000 ,


 =
IE Ri = 150 , IB = 20 A = 20 × 10–6 A,
  = 0.96 (given) IC = 1.5 mA = 1.5 × 10–3A
Voltage gain is given by,
0.625mA V0
0.96 = Av =
IE V1
IE = 0.651 mA R L I C
=
IB = IE  IC R i I B
IB = 0.651  0.625 20001.5 103
IB = 0.026 mA = = 1000
150 20106
 VCE = VCC – IC RC
= 8 – 0.625  10–3  800 38. Given, RL= 800 , VL = 0.8 V

ns
 VCE = 7.5 V  IC =
VL
= 1 mA = 10–3 A
RL
VCE 2
34. IC =  = 0.5  10–3 A = 0.5 mA ri = 192 
R C 4  103
IC

io
I Current amplification =  = = 0.96
 = C IB
IB
103 1
3  IB = 
IC 0.5  10

at
 IB   = 10–5 A = 10 A 0.96 960
 50 V V 0.8
Also, AV = L  L   960 = 4
Vin I B ri 192
35. Given,
 
Rin = RB = 1 k
lic AP =
I C2 R L
2
=
10 3 2

2
800
= 3.84
Rout = RC = 2 k I B ri  1 
Vout = 4 V   192
 960 
 = 100
ub
41. Reversible operation is carried out by inverter
We know,
gate. E.g., NOT gate. For Option (B), for input 1
AV =   resistance gain output is 0 and for input 0 output is 1. Thus the
RC 2 k gate acts as NOT gate.
 AV =   = 100  = 200
RB 1k
P

42. Gate shown in option (B) is a NOR gate. Output of


Vout
Also, AV = NOR gate when both the inputs are 0, is 1.
Vin
Vout 44. Truth table for the given circuit is
et

 = 200
Vin
A B Y
4
 = 200 0 0 0
Vin
rg

0 1 0
4
 Vin = = 20 mV 1 0 0
200 1 1 1
Ta

R2 This belongs to AND gate.


36. Voltage gain = AV =  and
R1
47.
 0.98
Current gain  =  = 49
1   1  0.98 A Y = A⋅ B X= A ⋅ B = AB
 500  103  B
 AV = (49)  
 R1 
A B Y X
 Power gain = .AV
0 0 1 0
 500  103 
 6.0625  106 = 49     49 0 1 1 0
 R1  1 0 1 0
492  500  103 1 1 0 1
 R1 =
6.0625  106
 R1  198   X = A ⋅ B = AB

442

Chapter 16: Semiconductor Devices


49. A single terminal NAND works as NOT. For A = B = 1,
Y1 = 1, Y2 = 0
A AB  Y = Y1  Y2 = 0
AB  AB
B Similarly, for A = B = 0,
NAND NOT Y1 = 0, Y2 = 1
 Y = Y1  Y2 = 0
50.  
A  B  A  A  (B  1)
57. A
But for any value x, x+1 = 1 A
 A   B  1 = A  1  A A B

51. B
A A.B
B

ns
B Y =  A  B  C
From figure,
C
Output Y = A  B = A  B = A + B

io
58.
A B (AB) C Y=  A  B  C
A AB
0 0 0 0 1 A
1 1 1 1 0 B A  B  A B

at
B
52. A
A B C A+B Y = (A+B)C
1 0 1 1 1 B A B
0 1 0 1
lic
0
1 0 0 1 0 59. Y = AB A Y1  A
1 1 0 1 0
= AB
ub
53. Y  Y1  Y2
=AB Y2  B
B
Y2
Shorted NAND Shorted NOR 60. From time graph it is clear that output remains
P

Both shorted NAND and NOR gates act as a high when any of the input is high. This is
NOT gate. represented by OR gate.
54. 61. These gates are called digital building blocks
et

A
Y =  A  B because by using various combinations of these
B
 A  B  A  B = A + B
gates (either NAND or NOR) we can compile all
other gates (like OR, AND, NOT, XOR).
rg

63.  A  B  C   A  B  C 
Thus, given network is equivalent to NOR gate.
55. 64. 0
P(0)
A X(0)
Q(1)
Ta

A
A  B = AB Z(0)
Y R(0) 0 1
Y(1)
S(1)
B
B
56.
65. 
y = AB  CD   
A Y1 A A
 A  B
B Y B B

Y2 C C
  
y = AB  CD 
Y1 = AB, Y2 = A  B D
C  D
D
Y = Y1  Y2

443

MHT-CET Triumph Physics (Hints)


For Option (A) 69. For given circuit, LED will glow until the current
  
y= 00  10 =0+0=0  through A and B is zero.
Hence, for the condition in which A and B both
 Option (A) is incorrect. have high value (i.e., 1) the majority current
For Option (B) flows through A and B and LED does not glow.
  
y= 10  10 =0+0=0   We can write the truth table as,
 Option (B) is incorrect. A B Y
For Option (C)
0 0 1
  
y= 01  01 = 0+0=0  0 1 1
 Option (C) is incorrect. 1 0 1

ns
For Option (D) 1 1 0
  
y= 00  11 =0+1=1  This corresponds to NAND gate.

io
Hence answer is option (D).
70. For given circuit, LED will glow until the current
66. To get the output Y = 1 from the AND gate, through both A and B is zero.
both its inputs must be one. For this C = 1, and

at
Hence, for the condition in which either or both
for the OR gate, either A or B or both must A and B have high value (i.e., 1) the majority
be = 1. current flows through A and/or B and LED does
not glow.
67.
A D
lic  We can write the truth table as,

C Y A B Y
ub
0 0 1
B E 0 1 0
1 0 0
A B C D = AC E = CB Y 1 1 0
P

0 0 1 1 1 0
0 1 1 1 0 1 This corresponds to NOR gate.
et

1 0 1 0 1 1 V 3
1 1 0 1 1 0 72. E= 
d 300  1010
68. V
= 108
rg

A AB m
B  
Y = AB  A  B  V
= 106
cm
Ta

AB 73. Energy gap of LED is,


From Figure, hc
E = h =
   

Output Y  (A  B)  (A  B)  A  B  A  B
hc 6.63  10 34  3  108
 = =
 (A  B)  (A  B) E 1.9  1.6  10 19
6.63  3  10 7
A B AB A+B Y =
1.9  1.6
0 0 0 0 0
= 6.542  107 m
1 0 0 1 1
= 654 nm
0 1 0 1 1
1 1 1 1 1

444

Chapter 16: Semiconductor Devices

Hints to Evaluation Test

1. P.D. across series resistance, 9. Here, R i = 500 , R0 = 40  103 ,  = 75


= 9V – 4V = 5V  R0 
Voltage gain =   
 Current through series resistance,  Ri 
4 40  103 
i= = 0.04 A.  75   6000
100 500 
 Current through load resistance, Power gain = Voltage gain  Current gain
V 4 = 6000  75 = 450000  4.5  105
iL = L = = 0.01 A
R L 100
10. Given that,  = 0.96 I E = 8 mA,

ns
Changein collector current IC
2. = 
Changein emitter current IE
 0.94  IC =  I E = 0.96  8  7.7 mA.
 = = = 15.67

io
1 1  0.94 The base current,
I B  I E  IC = 8  7.7 = 0.3 mA
 IC = (IB) = 15.67  0.5 = 7.83 mA
 0.96

at
EV 83 1 11. A.C. current gain,     24.
3. I= = = A 1   1  0.96
R 60 12
Collector current,
V 3 1 Voltage drop across collector resistor
 IL = Z = = A IC 
RL 120 40
lic Load resistance
1 1 7 4V
 IZ = I  IL =  = A =  8  103 A
12 40 120 500 
IC
ub
4. The base in a transistor is made thin because Now,  
IB
most of the holes coming from the emitter are
able to diffuse through the base region to the IC 8  103 A
 Base current, IB   = 0.33 mA.
collector retion. Hence, the assertion is true  24
but reason is false.
P

hc
12. E
IC  R C  2  10  4  10  
3 3

5. Voltage gain = 
IB  R B 10 106   400  
hc
et

E
= 2000 6.63  1034  3  108
=
6. When A is V(0) or B is V(0) or both are 0, 0.74  1.6  1019
= 16.798  107
rg

accordingly D1 or D2 or both are forward


biased. Current flows via R, the potential at = 1679.8  109 m
Y is 0. But when both A and B are at V(1),  1680 nm
then D1 and D2 do not conduct current. So
Ta

potential at Y is V(1). Y is 1 only when A and


B are both 1.
Thus, this represents an AND gate.
 Option (B) is correct.

7. For 0 < t < t1, Input = 0  output = 1


For t1 < t < t3, Input = 1  output = 0
For t3 < t < t4, Input = 0  output = 1
Hence (B) is the correct option.

8. P  A and Q = B
Now Y = 1  both P and Q are 0
P = 0  A = 1 amd Q = 0  B = 1
445
MHT-CET 2020
14th October 2020

Hints

2 Given: MP = 3Me, RP = 3Re 10. Relation between kinetic energy (E) and angular
2GM e momentum (L):
(ve) =

ns
Re L2
E=
2mr 2
2GM P
vP =  L2 = 2mEr2
RP
 L = (2MEr2)1/2

io
vP MP × R e 3M e × R e
 = = =1 ML2
ve R P× Me 3R e × M e 11. I=
3
 vp = ve

at
MR 2
4 I1 =
an w = x, wng = y, gna = z 2
nw n n 2
 = x, g = y, a = z
=
M L 
…( L = 2R)
na nw ng  
nw ng na
lic 2  2π 
 xyz =   =1 ML2
na n w ng =
8π2
I ML2 8π 2
ub
5 
1  = ×
λ I1 3 ML2
 When light enters glass from vacuum, the 
I 8π 2
=
refractive index () increases, hence wavelength I1 3
() decreases.
P

m1v12
7. I1 = M1R12  λL1R12 = (2R)R2 12. F1 =
r1
I2 = M 2 R 22  λL 2 R 22 = (2nR)(nR)2 m 2 v 22
et

F2 =
I1 1 r22
 = 3
I2 n But F1 = F2 ....(Given)
1 1 mv 2
mv 2
 =   2
rg

1 1 2
8 n3 r1 r2
 n=2
m1  nv 2  3m  v1 
2 2

 =
ML2 r1 r1/3
8. I1 =  MK12
Ta

12 ….(Given: m2 = 3m = 3m1,
ML2 r1
I2 =  MK12 v1 = nv2, r2 = )
3 3

K12 3

 n2 = 3  3
K 22 12  n=3
K 12 1
  13. Initial volume = Final volume
K 22 4
4  4
K1 1 1000  πr 3   πR 3
  3  3
K2 2
10r = R ….(i)
2π 2×π Initial surface energy (E1) = 1000(4r2)T ….(ii)
9. ω= =  0.1 (in degrees per second)
T 60  60 Final surface energy (E2) = 4R2T ….(iii)
Dividing equation (ii) and (iii)
446

MHTCET - 14 Oct.2020

E2 4πR 2T R2 ΔW 1
 = = 17. 1
E1 1000  4πr 2T  1000 r 2 ΔQ γ
1  7
10r 
2
=1 ….  γ diatomic  
= ….[From (i)] 7  5

1000 r 2  
5
100 r 2 1
=  ΔW 2
1000 r 2 10  ….(i)
ΔQ 7

14. For Hole ‘A’, vA = 2gh But Q = U + W


ΔQ ΔU ΔW
Volume rate of fluid flow at hole A,  
ΔQ ΔQ ΔQ
VA = AAvA = L2 2gh
ΔU 2
 1= 
For hole ‘B, vB = 2g  4h   8gh

ns
ΔQ 7
Volume rate of fluid flowat hole B, ΔU 5
 ….(ii)
2 ΔQ 7
VB = ABvB = R 8gh
Since VA = VB From equations (i), (ii) and (iii)

io
 Q : U : W = 7 : 5 : 2
L2 2gh  πR 2 8gh
 L2 = 2R2 1
18. n

at
 L = 2π .R l
 n12l1  n 22l2
4T
15. P1 = l1 n 22  4  16
2

r1     

P2 =
4T
lic l2 n12  3  9

r2 19. Frequency of oscillation for S.H.M.


 P1 = 2P2 ….(Given) n=
1 g
2  4T
ub
4T 2π l
 =
r1 r2 mgL
But Y =
r1 1 Al
  ….(i) mgL
r2 2 l=
P

4 3 YA
πr1 3
V1 r  1 g  YA 
Now, = 3 =  1  n=
V2 4 3  r2  2π mgL
πr2
3
et

1 YA
1
3 =
=  ….[From (i)] 2π mL
 2 1
1  YA  2
= 1 =
rg

 
8 2π  mL 

3RT1 20. x = 0.25 sin(11t + 0.5)


16. C1 = Comparing with x = A sin (t + )
Ta

M
  = 11
3RT2
C2 =  Time period, T =

M
ω
C2 T2 2π 2 22 4
= = =  =
C1 T1 11 11 7 7
C2 327  273 m  v  0
 =
21. F=
C1 27  273 t

C2
=
600  Ft = mv ….(i)
C1 300 1
But K.E. = mv2
C2 2
 = 2
1  mv 
2
C1 1 F2 t 2 F2 t 2
= = × =
2 m 2 m 2m
447

MHT-CET Triumph Physics (Hints)

P T Dλ
22. Frequency of vibration of string, n = 26. Fringe width, air =
2l m d
Case I: 1.33  6300 1010
 air =
Number of loops (P1) = Number of antinodes 1 103
=5 = 8.379  104 m
5 T1 β air 8.379  104
 n1 = water =  = 6.3  104 m
2l m μ water 1.33
5 qg
 n1 = ….(i) 27. Imax = K(a1 + a2)2
2l m
But I1 = Ka12
Case II:
Number of loop (P2) = Number of antinodes = 3 I2 = Ka 22
2
3 T2  I1 I2 
 

ns
2
 n2 =  Imax = K  +  = I1  I 2
2l m  K K 
3 Mg
 n2 = ….(ii) 1
2l m 28. U= CV2
2

io
Dividing equation (i) and (ii),
1ε A
n1 5 9 =  0  (Ed)2
 2 d 
n2 3 M
1

at
Since n1 = n2 = 0 E2Ad
 9  M = 25  9 2
 M = 25 kg lic 29. C3 = 3C C2 = 2C C1 = C

23.  = 2nt = 2  50  0.01 =  rad


C4 = 4C
24. Fundamental frequency,
1 T 1 T
n= =
πr 2ρ
ub
2l m 2l
Given: n2 = n3 V
 2n1 = 3n1 ….( n2 = 2n1, n3 = 3n1) Here, C1, C2 and C3 are in series
1 1 1 1 11
 1 T   1 T  = + + 
P

 2   =3 Ceq 3C 2C C 6
πr12ρ   2l2
 2l1 πr22ρ 
 
6C
l1 2  r2 Ceq =
  11
l2 3  r1
et

Charge on capacitor (C1, C2, C3) in series


l1 2  r2 6
  ….(Given r1 = 2r2) = CV
l2 3  2r2 11
l1 1 Charge on capacitor (C4) = 4CV
rg

 
l2 3 6
Charge on C2 11 CV 6 1 3
25. Fundamental frequency of an open pipe,  = = × =
Charge on C4 4 CV 11 4 22
Ta

v
n= V
2L 30. When 200  is connected in series, i =
v 200 + G
L=
2n When 2000  is connected in series,
v v 3V
 L1 = , L2 = i=
2n1 2n 2 2000+ G
When joined in series, L = L1 + L2 V 3V
 =
v v v 200 + G 2000+ G
 = +
2n 2n1 2n 2  2000 + G = 600 + 3G

1 1
 
1  2G = 1400
n n1 n 2  G = 700 
n1n 2
 n=
n1  n 2

448

MHTCET - 14 Oct.2020

E1 l1  l2 64  32 96 3 2m  E  W0 
31.  =   = ….[From (i)]
E 2 l1  l2 64  32 32 1 eB
32. 10V h
39. p =
2m p E p
I 1 h
e =
2m e E e
200V 38 
λp me
 = ….(Ep = Ev)
Applying KVL in loop 1, λe mp
10  38I + 200 = 0 1
λp  m 2
 38 I =  190  = e
 λ e  m p 

ns
I = 5A
35. Magnetic moment, e2
M = IA 41. K.E. =
8πε 0 r
e  q e
A

io
= …  I = =  e 2
T  T T P.E. =
4πε 0 r
 1
=eA …  ν =  K.E. 1
 T   =

at
P.E. 2
 M
 New magnetic moment = 2M 42. Truth table of AND gate
Given: Mass = 2  103 kg,
36.
Magnetic moment = 8  107 A/m2
lic A
0
B
0
Y = AB
0
density = 4  103 kg/m3 0 1 0
Magnetic moment 1 0 0
Magnetization =
ub
Volume 1 1 1
Magnetic moment
=
mass/density FL
43. Y=
7
8  10  4  10 3 Al
=
P

2  103 FL
 l=
= 16  101 AY
= 1.6 FL  M M
 l= ….  ρ = = 
et

37. M  V AL 
i  Y
 ρL 
R
1
 l
rg

M
r
l1 M 2
 
l2 M1
l1 4
Ta

Mutual inductance,  
l2 3
πμ 0 N1N2r 2
M=
2R 44. Unit of ‘R’ = 
r 2
Unit of ‘L’ = s
M
R Unit of L  s
  =s
Unit of R 
38. Using Einstein’s photo electric equation,
L
K.Emax = E  W0 ….(i)  Dimension of = [L0M0T1]
K.E of electron is given by, R
e 2 B2 r 2
 ˆi  2jˆ +2kˆ    2iˆ  ˆj  kˆ   A  ˆj

K.Emax = 45.
2m

2m K.E max A  ˆj  ˆi + 2jˆ  2kˆ + 2iˆ  ˆj + kˆ
 r=
e2B2
449

MHT-CET Triumph Physics (Hints)



A  ˆi + 2jˆ  kˆ

 |A| = 12  22  12  6
ΔV
46. Strain =
V
stress
 Bulk modulus, K =
strain
PV
 K=
V
ΔV P
 =
V K
ΔV Δρ
But 

ns
V ρ + Δρ
Δρ P
 =
ρ + Δρ K
K  = P + P

io

  =
K  P

at
2
l
47. P  
λ
For linear antenna,
P  2
lic
48. According to dimensional analysis,
B
Dimension of = Dimension of 
λ2
ub
 Dimension of B
= Dimension of 2 = [L2M0T0]
….( Since  is dimensionless)
 Dimension of area = [L2M0T0]
P

 Dimensions of B are same as that of area.

49. Resultant = P 2 + Q2 + 2PQ cosθ


et

R
Here Resultant = , P = R, Q = R
2
R
 = R 2 + R 2 + 2R 2cosθ
2
rg

P2
 = 2R2 + 2R2 cos 
4
R2
 2R2 cos  =  2R2
Ta

4
7R 2
 2R2 cos  =
4
7
 cos  =
8
1  7 
  = cos  
 8 

450

You might also like